You are on page 1of 1269

MACHINE DESIGN - An Integrated Approach, 4th Ed.

1-1-1

PROBLEM 1-1
Statement: It is often said, "Build a better mousetrap and the world will beat a path to your door." Consider this problem and write a goal statement and a set of at least 12 task specifications that you would apply to its solution. Then suggest 3 possible concepts to achieve the goal. Make annotated, freehand sketches of the concepts.

Solution: Goal Statement: Create a mouse-free environment. Task Specifications: 1. Cost less than $1.00 per use or application. 2. Allow disposal without human contact with mouse. 3. Be safe for other animals such as house pets. 4. Provide no threat to children or adults in normal use. 5. Be a humane method for the mouse. 6. Be environmentally friendly. 7. Have a shelf-life of at least 3 months. 8. Leave no residue. 9. Create minimum audible noise in use. 10. Create no detectable odors within 1 day of use. 11. Be biodegradable. 12. Be simple to use with minimal written instructions necessary. Concepts and sketches are left to the student. There are an infinity of possibilities.

MACHINE DESIGN - An Integrated Approach, 4th Ed.

1-2-1

PROBLEM 1-2
Statement: A bowling machine is desired to allow quadriplegic youths, who can only move a joystick, to engage in the sport of bowling at a conventional bowling alley. Consider the factors involved, write a goal statement, and develop a set of at least 12 task specifications that constrain this problem. Then suggest 3 possible concepts to achieve the goal. Make annotated, freehand sketches of the concepts.

Solution: Goal Statement: Create a means to allow a quadriplegic to bowl. Task Specifications: 1. Cost no more than $2 000. 2. Portable by no more than two able-bodied adults. 3. Fit through a standard doorway. 4. Provide no threat of injury to user in normal use. 5. Operate from a 110 V, 60 Hz, 20 amp circuit. 6. Be visually unthreatening. 7. Be easily positioned at bowling alley. 8. Have ball-aiming ability, controllable by user. 9. Automatically reload returned balls. 10. Require no more than 1 able-bodied adult for assistance in use. 11. Ball release requires no more than a mouth stick-switch closure. 12. Be simple to use with minimal written instructions necessary. Concepts and sketches are left to the student. There are an infinity of possibilities.

MACHINE DESIGN - An Integrated Approach, 4th Ed.

1-3-1

PROBLEM 1-3
Statement: A quadriplegic needs an automated page turner to allow her to read books without assistance. Consider the factors involved, write a goal statement, and develop a set of at least 12 task specifications that constrain this problem. Then suggest 3 possible concepts to achieve the goal. Make annotated, freehand sketches of the concepts.

Solution: Goal Statement: Create a means to allow a quadriplegic to read standard books with minimum assistance. Task Specifications: 1. Cost no more than $1 000. 2. Useable in bed or from a seated position 3. Accept standard books from 8.5 x 11 in to 4 x 6 in in planform and up to 1.5 in thick. 4. Book may be placed, and device set up, by able-bodied person. 5. Operate from a 110 V, 60 Hz, 15 amp circuit or by battery power. 6. Be visually unthreatening and safe to use. 7. Require no more than 1 able-bodied adult for assistance in use. 8. Useable in absence of assistant once set up. 9. Not damage books. 10. Timing controlled by user. 11. Page turning requires no more than a mouth stick-switch closure. 12. Be simple to use with minimal written instructions necessary. Concepts and sketches are left to the student. There are an infinity of possibilities.

MACHINE DESIGN - An Integrated Approach, 4th Ed.

1-4-1

PROBLEM 1-4
Statement: Units: Convert a mass of 1 000 lbm to (a) lbf, (b) slugs, (c) blobs, (d) kg. blob := lbf sec in Given: Solution: Mass See Mathcad file P0104. M := 1000 lb
2

1. To determine the weight of the given mass, multiply the mass value by the acceleration due to gravity, g. W := M g W = 1000 lbf

2. Convert mass units by assigning different units to the units place-holder when displaying the mass value. Slugs Blobs Kilograms M = 31.081 slug M = 2.59 blob M = 453.592 kg

MACHINE DESIGN - An Integrated Approach, 4th Ed.

1-5-1

PROBLEM 1-5
Statement: Given: A 250-lbm mass is accelerated at 40 in/sec2. Find the force in lb needed for this acceleration. Mass M := 250 lb Acceleration in a := 40 sec Solution: 1. See Mathcad file P0105.
2

To determine the force required, multiply the mass value, in slugs, by the acceleration in feet per second squared. Convert mass to slugs: M = 7.770 slug a = 3.333s
-2

Convert acceleration to feet per second squared: F := M a F = 25.9 lbf

ft

MACHINE DESIGN - An Integrated Approach, 4th Ed.

1-6-1

PROBLEM 1-6
Statement: Units: Given: Express a 100-kg mass in units of slugs, blobs, and lbm. How much does this mass weigh? blob lbf sec in
2

M 100 kg

Assumptions: The mass is at sea-level and the gravitational acceleration is g 32.174 ft sec Solution: 1.
2

or

g 386.089

in sec
2

or

g 9.807

m sec
2

See Mathcad file P0106.

Convert mass units by assigning different units to the units place-holder when displaying the mass value. The mass, in slugs, is The mass, in blobs, is The mass, in lbm, is M 6.85 slug M 0.571 blob M 220.5 lb

Note: Mathcad uses lbf for pound-force, and lb for pound-mass. 2. To determine the weight of the given mass, multiply the mass value by the acceleration due to gravity, g. The weight, in lbf, is The weight, in N, is W M g W M g W 220.5 lbf W 980.7 N

MACHINE DESIGN - An Integrated Approach, 4th Ed.

1-7-1

PROBLEM 1-7
Statement: Prepare an interactive computer program (using, for example, Excell, Mathcad, or TKSolver) from which the cross-sectional properties for the shapes shown in the inside front cover can be calculated. Arrange the program to deal with both ips and SI unit systems and convert the results between those systems. See the inside front cover and Mathcad file P0107.

Solution: 1.

Rectangle, let: b 3 in Area h 4 in A b h A 12.000 in A 7742 mm Moment about x-axis Ix b h 12 h b 12


3 3 2

2 4 6 4

Ix 16.000 in

Ix 6.660 10 mm Iy 9.000 in
4 6

Moment about y-axis

Iy

Iy 3.746 10 mm Radius of gyration about x-axis kx Ix A Iy A kx 1.155 in kx 29.329 mm ky 0.866 in ky 21.997 mm Jz 25.000 in
4 7

Radius of gyration about y-axis

ky

Polar moment of inertia

Jz Ix Iy

Jz 1.041 10 mm 2. Solid circle, let: D 3 in Area A

D
4

A 7.069 in

2 2

A 4560 mm
4

Moment about x-axis

Ix

D
64

Ix 3.976 in

4 6 4

Ix 1.655 10 mm
4

Moment about y-axis

Iy

D
64

Iy 3.976 in

4 6 4

Iy 1.655 10 mm Radius of gyration about x-axis kx Ix A kx 0.750 in kx 19.05 mm

MACHINE DESIGN - An Integrated Approach, 4th Ed.

1-7-2

Radius of gyration about y-axis

ky

Iy A

ky 0.750 in ky 19.05 mm
4

Polar moment of inertia

Jz

D
32

Jz 7.952 in

4 6 4

Jz 3.310 10 mm

3.

Hollow circle, let: D 3 in Area d 1 in A

D d

A 6.283 in

2 2

A 4054 mm D d
4 4

Moment about x-axis

Ix

64

64

Ix 3.927 in

4 6 4

Ix 1.635 10 mm Moment about y-axis Iy D d

Iy 3.927 in

4 6 4

Iy 1.635 10 mm Radius of gyration about x-axis kx Ix A Iy A kx 0.791 in kx 20.08 mm ky 0.791 in ky 20.08 mm

Radius of gyration about y-axis

ky

Polar moment of inertia

Jz

32

D d

Jz 7.854 in

4 6 4

Jz 3.269 10 mm

4.

Solid semicircle, let: D 3 in Area R 0.5 D A R 1.5 in A 3.534 in


2 2

D
8

A 2280 mm Moment about x-axis Ix 0.1098 R


4

Ix 0.556 in

4 5 4

Ix 2.314 10 mm Moment about y-axis Iy

R
8

Iy 1.988 in

4 5 4

Iy 8.275 10 mm

MACHINE DESIGN - An Integrated Approach, 4th Ed.

1-7-3

Radius of gyration about x-axis

kx

Ix A Iy A

kx 0.397 in kx 10.073 mm ky 0.750 in ky 19.05 mm Jz 2.544 in


4 6 4

Radius of gyration about y-axis

ky

Polar moment of inertia

Jz Ix Iy

Jz 1.059 10 mm Distances to centroid a 0.4244 R b 0.5756 R a 0.637 in a 16.17 mm b 0.863 in b 21.93 mm

5.

Right triangle, let: b 2 in Area h 1 in A b h 2 A 1.000 in A 645 mm


3 2

Moment about x-axis

Ix

b h 36 h b 36

Ix 0.056 in

4 4 4

Ix 2.312 10 mm
3

Moment about y-axis

Iy

Iy 0.222 in

4 4 4

Iy 9.250 10 mm Radius of gyration about x-axis kx Ix A kx 0.236 in kx 5.987 mm ky 0.471 in ky 11.974 mm Jz 0.278 in
4 5

Radius of gyration about y-axis

ky

Iy A

Polar moment of inertia

Jz Ix Iy

Jz 1.156 10 mm

MACHINE DESIGN - An Integrated Approach, 4th Ed.

1-8-1

PROBLEM 1-8
Statement: Prepare an interactive computer program (using, for example, Excell, Mathcad, or TKSolver) from which the mass properties for the solids shown in the page opposite the inside front cover can be calculated. Arrange the program to deal with both ips and SI unit systems and convert the results between those systems. blob lbf sec in
3 2

Units: Solution: 1.

See the page opposite the inside front cover and Mathcad file P0108. a 2 in b 3 in V a b c c 4 in

Rectangular prism, let: Volume

0.28 lbf in
V 24.000 in
3

V 393290 mm Mass M V g M 0.017 blob M 3.048 kg


2

Moment about x-axis

Ix

M a b 12

2 2 2

Ix 0.019 blob in

2 2

Moment about y-axis

Iy

M a c 12

Ix 2130.4 kg mm Iy 0.029 blob in

2 2

Iy 3277.6 kg mm Iz M b c 12
2

Moment about z-axis

Iz 0.036 blob in

2 2

Iz 4097.0 kg mm kx Ix M Iy M Iz M
3

Radius of gyration about x-axis

kx 1.041 in kx 26.437 mm ky 1.291 in ky 32.791 mm kz 1.443 in kz 36.662 mm

Radius of gyration about y-axis

ky

Radius of gyration about z-axis

kz

2.Cylinder, let:

r 2 in Volume

L 3 in

0.30 lbf in
V r L V g
2

V 37.699 in

3 3

V 617778 mm Mass M M 0.029 blob M 5.13 kg

MACHINE DESIGN - An Integrated Approach, 4th Ed.


Moment about x-axis Ix M r 2 M 3 r L 12 M 3 r L 12
2

1-8-2
Ix 0.059 blob in
2 2

Moment about y-axis

Iy

Ix 6619.4 kg mm Iy 0.051 blob in

2 2

Iy 5791.9 kg mm Iz 0.051 blob in

Moment about z-axis

Iz

2 2

Iz 5791.9 kg mm Radius of gyration about x-axis kx Ix M Iy M Iz M kx 1.414 in kx 35.921 mm ky 1.323 in ky 33.601 mm kz 1.323 in kz 33.601 mm

Radius of gyration about y-axis

ky

Radius of gyration about z-axis

kz

3.

Hollow cylinder, let: a 2 in Volume b 3 in L 4 in

0.28 lbf in

3 3 3

V b a L

V 62.832 in

V 1029630 mm Mass M V g M 0.046 blob M 7.98 kg

Moment about x-axis

Ix

M 2 M 12 M 12

a b

2 2

Ix 0.296 blob in
4

2 2

Ix 3.3 10 kg mm 3 a 3 b L
2

Moment about y-axis

Iy

Iy 0.209 blob in
4

2 2

Iy 2.4 10 kg mm 3 a 3 b L
2 2 2

Moment about z-axis

Iz

Iz 0.209 blob in
4

2 2

Iz 2.4 10 kg mm kx 2.550 in kx 64.758 mm ky 2.141 in ky 54.378 mm

Radius of gyration about x-axis

kx

Ix M Iy M

Radius of gyration about y-axis

ky

MACHINE DESIGN - An Integrated Approach, 4th Ed.


Radius of gyration about z-axis kz Iz M kz 2.141 in kz 54.378 mm

1-8-3

4.

Right circular cone, let: r 2 in Volume h 5 in


2

0.28 lbf in
V

3 3 3

r h
3 V g

V 20.944 in

V 343210 mm Mass M M 0.015 blob M 2.66 kg


2

Moment about x-axis

Ix

3 10

M r

Ix 0.018 blob in

2 2

Moment about y-axis

Iy M

12r2 3h2
80

Ix 2059.4 kg mm Iy 0.023 blob in

2 2

Iy 2638.5 kg mm Iz M

Moment about z-axis

12r2 3h2
80

Iz 0.023 blob in

2 2

Iz 2638.5 kg mm Radius of gyration about x-axis kx Ix M Iy M Iz M kx 1.095 in kx 27.824 mm ky 1.240 in ky 31.495 mm kz 1.240 in kz 31.495 mm

Radius of gyration about y-axis

ky

Radius of gyration about z-axis

kz

5.

Sphere, let: r 3 in Volume V 4 3 r


3

V 113.097 in

3 3

V 1853333 mm M 0.082 blob M 14.364 kg


2

Mass

V g 2 5

Moment about x-axis

Ix

M r

Ix 0.295 blob in

2 2

Ix 33362 kg mm

MACHINE DESIGN - An Integrated Approach, 4th Ed.


Moment about y-axis Iy 2 5 2 5 M r
2

1-8-4
Iy 0.295 blob in
2 2

Iy 33362 kg mm Moment about z-axis Iz M r


2

Iz 0.295 blob in

2 2

Iz 33362 kg mm Ix M Iy M Iz M kx 1.897 in kx 48.193 mm ky 1.897 in ky 48.193 mm kz 1.897 in kz 48.193 mm

Radius of gyration about x-axis

kx

Radius of gyration about y-axis

ky

Radius of gyration about z-axis

kz

MACHINE DESIGN - An Integrated Approach, 4th Ed.

1-9-1

PROBLEM 1-9
Statement: Convert the template in Problem 1-7 to have and use a set of functions or subroutines that can be called from within any program in that language to solve for the cross-sectional properties of the shapes shown on the inside front cover. See inside front cover and Mathcad file P0109. Area Moment about x-axis A ( b h ) b h Ix( b h ) Iy( b h ) b h 12 h b 12
2 3 3

Solution: 1. Rectangle:

Moment about y-axis

2. Solid circle:

Area

A ( D) Ix( D) Iy( D)

D
4

Moment about x-axis

D
64

Moment about y-axis

D
64

3. Hollow circle: Area

A ( D d ) Ix( D d )

D d

Moment about x-axis

64

D d

Moment about y-axis

Iy( D d )

64

D d

4. Solid semicircle: Area A ( D)

D
8

Moment about x-axis

Ix( R) 0.1098 R Iy( R)

Moment about y-axis 5. Right triangle: Area

R
8

A ( b h )

b h 2 b h 36 h b 36
3 3

Moment about x-axis

Ix( b h ) Iy( b h )

Moment about y-axis

MACHINE DESIGN - An Integrated Approach, 4th Ed.

1-10-1

PROBLEM 1-10
Statement: Convert the template in Problem 1-8 to have and use a set of functions or subroutines that can be called from within any program in that language to solve for the cross-sectional properties of the shapes shown on the page opposite the inside front cover. See the page opposite the inside front cover and Mathcad file P0110.

Solution: 1

Rectangular prism: Volume Mass V ( a b c) a b c M ( a b c ) V ( a b c) g M ( a b c ) a b 12 M ( a b c ) a c 12 M ( a b c ) b c 12

Moment about x-axis

Ix( a b c ) Iy( a b c )

2 2 2

Moment about y-axis

Moment about z-axis 2. Cylinder: Volume Mass

Iz( a b c )

V ( r L) r L M ( r L ) V ( r L) g M ( r L ) r 2
2

Moment about x-axis

Ix( r L ) Iy( r L ) Iz( r L )

Moment about y-axis

M ( r L ) 3 r L 12 M ( r L ) 3 r L 12

Moment about z-axis

3.

Hollow cylinder: Volume Mass V ( a b L) b a L M ( a b L ) Ix( a b L ) Iy( a b L ) Iz( a b L ) V ( a b L) g M ( a b L ) 2 M ( a b L ) 12 M ( a b L ) 12 a b

Moment about x-axis

2 2

Moment about y-axis

3 a 3 b L 3 a 3 b L
2 2

Moment about z-axis

MACHINE DESIGN - An Integrated Approach, 4th Ed. 4. Right circular cone:


Volume V ( r h )

1-10-2
2

r h
3 V ( r h ) g 3 10 M ( r h ) r
2

Mass

M ( r h )

Moment about x-axis

Ix( r h )

Moment about y-axis

Iy( r h ) M ( r h )

12r2 3h2
80

Moment about z-axis 5. Sphere: Volume

Iz( r h ) M ( r h )

12r2 3h2
80

V ( r)

4 3

Mass

M ( r )

V ( r) g 2 5 2 5 2 5 M ( r ) r M ( r ) r M ( r ) r
2

Moment about x-axis

Ix( r ) Iy( r ) Iz( r )

Moment about y-axis

Moment about z-axis

MACHINE DESIGN - An Integrated Approach, 4th Ed.

2-1-1

PROBLEM 2-1
Statement: Figure P2-1 shows stress-strain curves for three failed tensile-test specimens. All are plotted on the same scale. (a) Characterize each material as brittle or ductile. (b) Which is the stiffest? (c) Which has the highest ultimate strength? (d) Which has the largest modulus of resilience? (e) Which has the largest modulus of toughness? See Figure P2-1 and Mathcad file P0201.

Solution: 1.

The material in Figure P2-1(a) has a moderate amount of strain beyond the yield point, P2-1(b) has very little, and P2-1(c) has considerably more than either of the other two. Based on this observation, the material in Figure P2-1(a) is mildly ductile, that in P2-1(b)is brittle, and that in P2-1(c) is ductile. The stiffest material is the one with the grearesr slope in the elastic range. Determine this by dividing the rise by the run of the straight-line portion of each curve. The material in Figure P2-1(c) has a slope of 5 stress units per strain unit, which is the greatest of the three. Therefore, P2-1(c) is the stiffest. Ultimate strength corresponds to the highest stress that is achieved by a material under test. The material in Figure P2-1(b) has a maximum stress of 10 units, which is considerably more than either of the other two. Therefore, P2-1(b) has the highest ultimate strength. The modulus of resilience is the area under the elastic portion of the stress-starin curve. From observation of the three graphs, the stress and strain values at the yield points are: P2-1(a) P2-1(b) P2-1(c)

2.

3.

4.

ya := 5 yb := 9 yc := 5

ya := 5 yb := 2 yc := 1

Using equation (2.7), the modulus of resiliency for each material is, approximately, P21a := 1 2 1 2 1 2 ya ya P21a = 12.5

P21b :=

yb yb

P21b = 9

P21c :=

yc yc

P21c = 2.5

P2-1 (a) has the largest modulus of resilience 5. The modulus of toughness is the area under the stress-starin curve up to the point of fracture. By inspection, P2-1 (c) has the largest area under the stress-strain curve therefore, it has the largest modulus of toughness.

MACHINE DESIGN - An Integrated Approach, 4th Ed.

2-2-1

PROBLEM 2-2
Statement: Determine an approximate ratio between the yield strength and ultimate strength for each material shown in Figure P2-1. See Figure P2-1 and Mathcad file P0202.

Solution: 1.

The yield strength is the value of stress at which the stress-strain curve begins to be nonlinear. The ultimate strength is the maximum value of stress attained during the test. From the figure, we have the following values of yield strength and tensile strength: Figure P2-1(a) Figure P2-1(b) Figure P2-1(c) S ya := 5 S yb := 9 S yc := 5 S ua := 6 S ub := 10 S uc := 8

2.

The ratio of yield strength to ultimate strength for each material is: Figure P2-1(a) ratioa := S ya S ua S yb S ub S yc S uc ratioa = 0.83

Figure P2-1(b)

ratiob :=

ratiob = 0.90

Figure P2-1(c)

ratioc :=

ratioc = 0.63

MACHINE DESIGN - An Integrated Approach, 4th Ed.

2-3-1

PROBLEM 2-3
Statement: Which of the steel alloys shown in Figure 2-19 would you choose to obtain (a) Maximum strength (b) Maximum modulus of resilience (c) Maximum modulus of toughness (d) Maximum stiffness Young's modulus for steel E 207 GPa

Given: Solution: 1.

See Figure 2-19 and Mathcad file P0203.

Determine from the graph: values for yield strength, ultimate strength and strain at fracture for each material. Steel AISI 1020: AISI 1095: AISI 4142: Yield Strength Sy1020 300 MPa Sy1095 550 MPa Sy4142 1600 MPa Ultimate Strength Sut 1020 400 MPa Sut 1095 1050 MPa Sut 4142 2430 MPa Fracture Strain

f 1020 0.365 f 1095 0.11 f 4142 0.06

Note: The 0.2% offset method was used to define a yield strength for the AISI 1095 and the 4142 steels. 2. 3. From the values of Sut above it is clear that the AISI 4142 has maximum strength. Using equation (2-7) and the data above, determine the modulus of resilience. 1 Sy1020 UR1020 2 E 1 Sy1095 UR1095 2 E 1 Sy4142 UR4142 2 E
2

UR1020 0.22
2

MN m m
3

UR1095 0.73
2

MN m m
3

UR4142 6.18

MN m
3

m Even though the data is approximate, the AISI 4142 clearly has the largest modulus of resilience. 4. Using equation (2-8) and the data above, determine the modulus of toughness. UT1020 UT1095 UT4142 1 2 1 2 1 2 Sy1020 Sut 1020 f 1020 Sy1095 Sut 1095 f 1095 Sy4142 Sut 4142 f 4142 UT1020 128 UT1095 88 MN m m
3 3

MN m m MN m m
3

UT4142 121

Since the data is approximate, there is no significant difference between the 1020 and 4142 steels. Because of the wide difference in shape and character of the curves, one should also determine the area under the curves by graphical means. When this is done, the area under the curve is about 62 square units for 1020 and 66 for 4142. Thus, they seem to have about equal toughness, which is about 50% greater than that for the 1095 steel. 5. All three materials are steel therefore, the stiffnesses are the same.

MACHINE DESIGN - An Integrated Approach, 4th Ed.

2-4-1

PROBLEM 2-4
Statement: Which of the aluminum alloys shown in Figure 2-21 would you choose to obtain (a) Maximum strength (b) Maximum modulus of resilience (c) Maximum modulus of toughness (d) Maximum stiffness Young's modulus for aluminum E 71.7 GPa

Given: Solution: 1.

See Figure 2-21 and Mathcad file P0204.

Determine, from the graph, values for yield strength, ultimate strength and strain at fracture for each material. Alum 1100: 2024-T351: 7075-T6: Yield Strength Sy1100 120 MPa Sy2024 330 MPa Sy7075 510 MPa Ultimate Strength Sut 1100 130 MPa Sut 2024 480 MPa Sut 7075 560 MPa Fracture Strain

f 1100 0.170 f 2024 0.195 f 7075 0.165

Note: The 0.2% offset method was used to define a yield strength for all of the aluminums. 2. 3. From the values of Sut above it is clear that the 7075-T6 has maximum strength. Using equation (2-7) and the data above, determine the modulus of resilience. 1 Sy1100 UR1100 2 E UR2024 1 Sy2024 2 E
2

UR1100 0.10
2

MN m m
3

UR2024 0.76
2

MN m m
3

1 Sy7075 UR7075 2 E

UR7075 1.81

MN m m
3

Even though the data is approximate, the 7075-T6 clearly has the largest modulus of resilience. 4. Using equation (2-8) and the data above, determine the modulus of toughness. UT1100 UT2024 UT7075 1 2 1 2 1 Sy1100 Sut 1100 f 1100 Sy2024 Sut 2024 f 2024 Sy7075 Sut 7075 f 7075 UT1100 21 UT2024 79 UT7075 88 MN m m
3

MN m m
3

MN m

2 3 m Even though the data is approximate, the 7075-T6 has the largest modulus of toughness. 5. All three materials are aluminum therefore, the stiffnesses are the same.

MACHINE DESIGN - An Integrated Approach, 4th Ed.

2-5-1

PROBLEM 2-5
Statement: Which of the thermoplastic polymers shown in Figure 2-22 would you choose to obtain (a) Maximum strength (b) Maximum modulus of resilience (c) Maximum modulus of toughness (d) Maximum stiffness See Figure 2-22 and Mathcad file P0205.

Solution: 1.

Determine, from the graph, values for yield strength, ultimate strength, strain at fracture, and modulus of elasticity for each material. Plastic Nylon 101: HDPE: PTFE: Yield Strength SyNylon 63 MPa SyHDPE 15 MPa SyPTFE 8.3 MPa Ultimate Strength Sut Nylon 80 MPa Sut HDPE 23 MPa Sut PTFE 13 MPa Fracture Strain Mod of Elasticity ENylon 1.1 GPa EHDPE 0.7 GPa EPTFE 0.8 GPa

f Nylon 0.52 f HDPE 3.0 f PTFE 0.51

2. 3.

From the values of Sut above it is clear that the Nylon 101 has maximum strength. Using equation (2-7) and the data above, determine the modulus of resilience. 1 SyNylon URNylon 2 ENylon
2

URNylon 1.8
2

MN m m
3

1 SyHDPE URHDPE 2 EHDPE 1 SyPTFE URPTFE 2 EPTFE


2

URHDPE 0.16

MN m m
3

URPTFE 0.04

MN m m
3

Even though the data is approximate, the Nylon 101 clearly has the largest modulus of resilience. 4. Using equation (2-8) and the data above, determine the modulus of toughness. UTNylon 1 2 1 2 1 2 SyNylon Sut Nylon f Nylon SyHDPE SutHDPE f HDPE UTNylon 37 MN m m UTHDPE 57
3

UTHDPE

MN m m
3

UTPTFE

SyPTFE SutPTFE f PTFE

UTPTFE 5

MN m m
3

Even though the data is approximate, the HDPE has the largest modulus of toughness. 5. The Nylon 101 has the steepest slope in the (approximately) elastic range and is, therefore, the stiffest of the three materials..

MACHINE DESIGN - An Integrated Approach, 4th Ed.

2-6-1

PROBLEM 2-6
Statement: A metal has a strength of 414 MPa at its elastic limit and the strain at that point is 0.002. What is the modulus of elasticity? What is the strain energy at the elastic limit? Assume that the test speimen is 12.8-mm dia and has a 50-mm gage length. Can you define the type of metal based on the given data? Elastic limit: Strength S el 414 MPa Strain

Given:

el 0.002

Test specimen: Diameter d o 12.8 mm Solution: 1. See Mathcad file P0206.

Length Lo 50 mm

The modulus of elasticity is the slope of the stress-strain curve, which is a straight line, in the elastic region. Since one end of this line is at the origin, the slope (modulus of elasticity) is E S el E 207 GPa

el

2.

The strain energy per unit volume at the elastic limit is the area under the stress-strain curve up to the elastic limit. Since the curve is a straight line up to this limit, the area is one-half the base times the height, or U'el 1 2 S el el U'el 414 kN m m
3

The total strain energy in the specimen is the strain energy per unit volume times the volume,

Uel U'el

d o
4

Lo

Uel 2.7 N m

3.

Based on the modulus of elasticity and using Table C-1, the material is steel.

MACHINE DESIGN - An Integrated Approach, 4th Ed.

2-7-1

PROBLEM 2-7
Statement: A metal has a strength of 41.2 kpsi (284 MPa) at its elastic limit and the strain at that point is 0.004. What is the modulus of elasticity? What is the strain energy at the elastic limit? Assume that the test speimen is 0.505-in dia and has a 2-in gage length. Can you define the type of metal based on the given data? Elastic limit: Strength S el 41.2 ksi Strain

Given:

el 0.004

S el 284 MPa

Test specimen: Diameter d o 0.505 in Solution: 1. See Mathcad file P0207.

Length Lo 2.00 in

The modulus of elasticity is the slope of the stress-strain curve, which is a straight line, in the elastic region. Since one end of this line is at the origin, the slope (modulus of elasticity) is E S el E 10.3 10 psi
6

el

E 71 GPa

2.

The strain energy per unit volume at the elastic limit is the area under the stress-strain curve up to the elastic limit. Since the curve is a straight line up to this limit, the area is one-half the base times the height, or 1 2 lbf in in
2 3

U'el

S el el

U'el 82.4

U'el 568

kN m m
3

The total strain energy in the specimen is the strain energy per unit volume times the volume, Uel U'el 3.

d o
4

Lo

Uel 33.0 in lbf

Based on the modulus of elasticity and using Table C-1, the material is aluminum.

MACHINE DESIGN - An Integrated Approach, 4th Ed.

2-8-1

PROBLEM 2-8
Statement: A metal has a strength of 134 MPa at its elastic limit and the strain at that point is 0.006. What is the modulus of elasticity? What is the strain energy at the elastic limit? Assume that the test speimen is 12.8-mm dia and has a 50-mm gage length. Can you define the type of metal based on the given data? Elastic limit: Strength S el 134 MPa Strain

Given:

el 0.003

Test specimen: Diameter d o 12.8 mm Solution: 1. See Mathcad file P0208.

Length Lo 50 mm

The modulus of elasticity is the slope of the stress-strain curve, which is a straight line, in the elastic region. Since one end of this line is at the origin, the slope (modulus of elasticity) is E S el E 45 GPa

el

2.

The strain energy per unit volume at the elastic limit is the area under the stress-strain curve up to the elastic limit. Since the curve is a straight line up to this limit, the area is one-half the base times the height, or U'el 1 2 S el el U'el 201 kN m m
3

The total strain energy in the specimen is the strain energy per unit volume times the volume,

Uel U'el

d o
4

Lo

Uel 1.3 N m

3.

Based on the modulus of elasticity and using Table C-1, the material is magnesium.

MACHINE DESIGN - An Integrated Approach, 4th Ed.

2-9-1

PROBLEM 2-9
Statement: A metal has a strength of 100 kpsi (689 MPa) at its elastic limit and the strain at that point is 0.006. What is the modulus of elasticity? What is the strain energy at the elastic limit? Assume that the test speimen is 0.505-in dia and has a 2-in gage length. Can you define the type of metal based on the given data? Elastic limit: Strength S el 100 ksi S el 689 MPa Test specimen: Diameter d o 0.505 in Solution: 1. See Mathcad file P0209. Length Lo 2.00 in Strain

Given:

el 0.006

The modulus of elasticity is the slope of the stress-strain curve, which is a straight line, in the elastic region. Since one end of this line is at the origin, the slope (modulus of elasticity) is E S el E 16.7 10 psi
6

el

E 115 GPa

2.

The strain energy per unit volume at the elastic limit is the area under the stress-strain curve up to the elastic limit. Since the curve is a straight line up to this limit, the area is one-half the base times the height, or U'el 1 2 S el el U'el 300 lbf in in
3

U'el 2 10

3 kN m

The total strain energy in the specimen is the strain energy per unit volume times the volume,

Uel U'el

d o
4

Lo

Uel 120.18 in lbf

3.

Based on the modulus of elasticity and using Table C-1, the material is titanium.

MACHINE DESIGN - An Integrated Approach, 4th Ed.

2-10-1

PROBLEM 2-10
Statement: A material has a yield strength of 689 MPa at an offset of 0.6% strain. What is its modulus of resilience? MJ 10 joule Yield strength Yield strain Solution: 1. S y 689 MPa
6

Units: Given:

y 0.006

See Mathcad file P0210.

The modulus of resilience (strain energy per unit volume) is given by Equation (2.7) and is approximately 1 2 MJ m
3

UR

S y y

UR 2.067

UR 2.1 MPa

MACHINE DESIGN - An Integrated Approach, 4th Ed.

2-11-1

PROBLEM 2-11
Statement: A material has a yield strength of 60 ksi (414 MPa) at an offset of 0.2% strain. What is its modulus of resilience? MJ 10 joule Yield strength Yield strain Solution: 1. S y 60 ksi S y 414 MPa
6

Units: Given:

y 0.002

See Mathcad file P0211.

The modulus of resilience (strain energy per unit volume) is given by Equation (2.7) and is approximately 1 2 in lbf in
3

UR

S y y

UR 60

UR 0.414

MJ m
3

UR 0.414 MPa

MACHINE DESIGN - An Integrated Approach, 4th Ed.

2-12-1

PROBLEM 2-12
Statement: A steel has a yield strength of 414 MPa, an ultimate tensile strength of 689 MPa, and an elongation at fracture of 15%. What is its approximate modulus of toughness? What is the approximate modulus of resilience? S y 414 MPa See Mathcad file P0212. S ut 689 MPa

Given: Solution: 1.

f 0.15

Determine the modulus of toughness using Equation (2.8).

UT

Sy S ut f 2

UT 82.7

MN m m
3

UT 82.7 MPa

2.

Determine the modulus of resilience using Equation (2.7) and Young's modulus for steel: E 207 GPa 1 Sy UR 2 E
2

UR 414

kN m m
3

UR 0.41 MPa

MACHINE DESIGN - An Integrated Approach, 4th Ed.

2-13-1

PROBLEM 2-13
Statement: The Brinell hardness of a steel specimen was measured to be 250 HB. What is the material's approximate tensile strength? What is the hardness on the Vickers scale? The Rockwell scale? Brinell hardness of specimen See Mathcad file P0213. HB 250

Given: Solution:

1. Determine the approximate tensile strength of the material from equations (2.10), not Table 2-3. S ut 0.5 HB ksi S ut 125 ksi S ut 862 MPa

2. From Table 2-3 (using linear interpolation) the hardness on the Vickers scale is HV HB 241 277 241 ( 292 253 ) 253 HV 263

3. From Table 2-3 (using linear interpolation) the hardness on the Rockwell C scale is HRC HB 241 277 241 ( 28.8 22.8) 22.8 HRC 24.3

MACHINE DESIGN - An Integrated Approach, 4th Ed.

2-14-1

PROBLEM 2-14
Statement: The Brinell hardness of a steel specimen was measured to be 340 HB. What is the material's approximate tensile strength? What is the hardness on the Vickers scale? The Rockwell scale? Brinell hardness of specimen See Mathcad file P0214. HB 340

Given: Solution:

1. Determine the approximate tensile strength of the material from equations (2.10), not Table 2-3. S ut 0.5 HB ksi S ut 170 ksi S ut 1172 MPa

2. From Table 2-3 (using linear interpolation) the hardness on the Vickers scale is HV HB 311 341 311 ( 360 328 ) 328 HV 359

3. From Table 2-3 (using linear interpolation) the hardness on the Rockwell C scale is HRC HB 311 341 311 ( 36.6 33.1) 33.1 HRC 36.5

MACHINE DESIGN - An Integrated Approach, 4th Ed.

2-15-1

PROBLEM 2-15
Statement: Solution: What are the principal alloy elements of an AISI 4340 steel? How much carbon does it have? Is it hardenable? By what techniques? See Mathcad file P0215.

1. Determine the principal alloying elements from Table 2-5 for 43xx steel.. 1.82% Nickel 0.50 or 0.80% Chromium 0.25% Molybdenum 2. From "Steel Numbering Systems" in Section 2.6, the carbon content is From the last two digits, the carbon content is 0.40%. 3. Is it hardenable? Yes, all of the alloying elements increase the hardenability. By what techniques? It can be through hardened by heating, quenching and tempering; and it can also be case hardened (See Section 2.4).

MACHINE DESIGN - An Integrated Approach, 4th Ed.

2-16-1

PROBLEM 2-16
Statement: Solution: What are the principal alloy elements of an AISI 1095 steel? How much carbon does it have? Is it hardenable? By what techniques? See Mathcad file P0216.

1. Determine the principal alloying elements from Table 2-5 for 10xx steel. Carbon only, no alloying elements 2. From "Steel Numbering Systems" in Section 2.6, the carbon content is From the last two digits, the carbon content is 0.95%. 3. Is it hardenable? Yes, as a high-carbon steel, it has sufficient carbon content for hardening. By what techniques? It can be through hardened by heating, quenching and tempering; and it can also be case hardened (See Section 2.4).

MACHINE DESIGN - An Integrated Approach, 4th Ed.

2-17-1

PROBLEM 2-17
Statement: Solution: What are the principal alloy elements of an AISI 6180 steel? How much carbon does it have? Is it hardenable? By what techniques? See Mathcad file P0217.

1. Determine the principal alloying elements from Table 2-5 for 61xx steel.. 0.15% Vanadium 0.60 to 0.95% Chromium 2. From "Steel Numbering Systems" in Section 2.6, the carbon content is From the last two digits, the carbon content is 0.80%. 3. Is it hardenable? Yes, all of the alloying elements increase the hardenability. By what techniques? It can be through hardened by heating, quenching and tempering; and it can also be case hardened (See Section 2.4).

MACHINE DESIGN - An Integrated Approach, 4th Ed.

2-18-1

PROBLEM 2-18
Statement: Solution: Which of the steels in Problems 2-15, 2-16, and 2-17 is the stiffest? See Mathcad file P0218.

1. None. All steel alloys have the same Young's modulus, which determines stiffness.

MACHINE DESIGN - An Integrated Approach, 4th Ed.

2-19-1

PROBLEM 2-19
Statement: Calculate the specific strength and specific stiffness of the following materials and pick one for use in an aircraft wing spar. Material Steel Code st 0 al 1 ti 2 Ultimate Strength Sut 80 ksi
st

Given:

Young's Modulus E 30 10 psi


st 6

Weight Density

0.28
st

lbf in
3

Aluminum

Sut

al

60 ksi

al

10.4 10 psi
6

0.10
al

lbf in
3

Titanium

Sut 90 ksi
ti

E 16.5 10 psi
ti

0.16
ti

lbf in
3

Index Solution: 1.

i 0 1 2

See Mathcad file P0219.

Specific strength is the ultimate tensile strength divided by the weight density and specific stiffness is the modulus of elasticity divided by the weight density. The text does not give a symbol to these quantities. Sut Specific strength
i 1

E Specific stiffness Steel Aluminum Titanium

i 1

in

in
i

286103 600103 563103

107106 104106 103106

2.

Based on the results above, all three materials have the same specific stiffness but the aluminum has the largest specific strength. Aluminum for the aircraft wing spar is recommended.

MACHINE DESIGN - An Integrated Approach, 4th Ed.

2-20-1

PROBLEM 2-20
Statement: Solution: If maximum impact resistance were desired in a part, which material properties would you look for? See Mathcad file P0220.

1. Ductility and a large modulus of toughness (see "Impact Resistance" in Section 2.1).

MACHINE DESIGN - An Integrated Approach, 4th Ed.

2-21-1

PROBLEM 2-21
Statement:

_____

Refer to the tables of material data in Appendix A and determine the strength-to-weight ratios of the following material alloys based on their tensile yield strengths: heat-treated 2024 aluminum, SAE 1040 cold-rolled steel, Ti-75A titanium, type 302 cold-rolled stainless steel. Material
1

Given:

Yield Strength
1

Specific Weight

Mat "2024 Aluminum, HT" Sy 290 MPa Mat "1040 CR Steel"


2

0.10 lbf in
1

27.14
1

kN m
3

Sy 490 MPa
2

0.28 lbf in
2

76.01
2

kN m
3

Mat "Ti-75A Titanium"


3

Sy 517 MPa
3

0.16 lbf in
3

43.43
3

kN m
3

Mat "Type 302 CR SS"


4

Sy 1138 MPa
4

0.28 lbf in
4

76.01
4

kN m
3

i 1 2 4 Solution: 1. See Mathcad file P0221.

Calculate the strength-to-weight ratio for each material as described in Section 2.1. Sy SWR
i

SWR
4

10 m

"2024 Aluminum, HT" "1040 CR Steel" Mat i "Ti-75A Titanium" "Type 302 CR SS"

1.068 0.645 1.190 1.497

MACHINE DESIGN - An Integrated Approach, 4th Ed.

2-22-1

PROBLEM 2-22
Statement:

_____

Refer to the tables of material data in Appendix A and determine the strength-to-weight ratios of the following material alloys based on their ultimate tensile strengths: heat-treated 2024 aluminum SAE 1040 cold-rolled steel, unfilled acetal plastic, Ti-75A titanium, type 302 cold-rolled stainless steel. Material
1 2 3 4 5

Given:

Tensile Strength
1 2 3 4 5

Specific Weight

Mat "2024 Aluminum, HT" Sut 441 MPa Mat "1040 CR Steel" Mat "Acetal, unfilled" Mat "Ti-75A Titanium" Mat "Type 302 CR SS" i 1 2 5 Solution: 1. See Mathcad file P0222. Sut 586 MPa Sut 60.7 MPa Sut 586 MPa Sut 1310 MPa

0.10 lbf in
1 2 3 4 5

3 3 3

27.14 kN m
1 2 3 4 5

3 3 3 3 3

0.28 lbf in

76.01 kN m 13.84 kN m 43.43 kN m 76.01 kN m

0.051 lbf in 0.16 lbf in 0.28 lbf in

3 3

Calculate the strength-to-weight ratio for each material as described in Section 2.1.

Sut SWR
i

SWR
4

10 m

"2024 Aluminum, HT" "1040 CR Steel" Mat "Acetal, unfilled" i "Ti-75A Titanium" "Type 302 CR SS"

1.625 0.771 0.438 1.349 1.724

MACHINE DESIGN - An Integrated Approach, 4th Ed.

2-23-1

PROBLEM 2-23
Statement:

_____

Refer to the tables of material data in Appendix A and calculate the specific stiffness of aluminum, titanium, gray cast iron, ductile iron, bronze, carbon steel, and stainless steel. Rank them in increasing order of this property and discuss the engineering significance of these data. Mg 10 kg Material Mat "Aluminum"
1 2 3 4 5 6 7 3

Units: Given:

Modulus of Elasticity E 71.7 GPa


1 2 3 4 5 6 7 1 2 3 4 5 6 7

Density

2.8 Mg m 4.4 Mg m 7.2 Mg m 6.9 Mg m 8.6 Mg m 7.8 Mg m 7.8 Mg m

3 3 3 3 3 3 3

Mat "Titanium" Mat "Gray cast iron" Mat "Ductile iron" Mat "Bronze" Mat "Carbon steel" Mat "Stainless steel" i 1 2 7 Solution: 1. See Mathcad file P0223.

E 113.8 GPa E 103.4 GPa E 168.9 GPa E 110.3 GPa E 206.8 GPa E 189.6 GPa

Calculate the specific stiffness for each material as described in Section 2.1. E E'
i

i i

E'

i 6

2 2

"Aluminum" "Titanium" "Gray cast iron" Mat "Ductile iron" i "Bronze" "Carbon steel" "Stainless steel"
E'
2 5 s 6

10

25.6 25.9 14.4 24.5 12.8 26.5 24.3

2.

Rank them in increasing order of specific stiffness. Mat "Bronze"


5

10 E' Mat "Gray cast iron"


3

12.8

2 3 s 6

10 E' Mat "Stainless steel"


7

14.4

2 7 s 6

10

24.3

MACHINE DESIGN - An Integrated Approach, 4th Ed.

2-23-2

E' Mat "Ductile iron"


4

2 4 s 6

10 E' Mat "Aluminum"


1

24.5

2 1 s 6

10 E' Mat "Titanium"


2

25.6

2 2 s 6

10 E' Mat "Carbon steel"


6

25.9

2 6 s 6

10 3.

26.5

Bending and axial deflection are inversely proportional to the modulus of elasticity. For the same shape and dimensions, the material with the highest specific stiffness will give the smallest deflection. Or, put another way, for a given deflection, using the material with the highest specific stiffness will result in the least weight.

MACHINE DESIGN - An Integrated Approach, 4th Ed.

2-24-1

PROBLEM 2-24
Statement: Call your local steel and aluminum distributors (consult the Yellow Pages) and obtain current costs per pound for round stock of consistent size in low-carbon (SAE 1020) steel, SAE 4340 steel, 2024-T4 aluminum, and 6061-T6 aluminum. Calculate a strength/dollar ratio and a stiffness/dollar ratio for each alloy. Which would be your first choice on a cost-efficiency basis for an axial-tension-loaded round rod (a) If maximum strength were needed? (b) If maximum stiffness were needed?

Solution:

Left to the student as data will vary with time and location.

MACHINE DESIGN - An Integrated Approach, 4th Ed.

2-25-1

PROBLEM 2-25
Statement: Call your local plastic stock-shapes distributors (consult the Yellow Pages) and obtain current costs per pound for round rod or tubing of consistent size in plexiglass, acetal, nylon 6/6, and PVC. Calculate a strength/dollar ratio and a stiffness/dollar ratio for each alloy. Which would be your first choice on a cost-efficiency basis for an axial-tension-loaded round rod or tube of particular diameters. (a) If maximum strength were needed? (b) If maximum stiffness were needed?

Solution:

Left to the student as data will vary with time and location.

MACHINE DESIGN - An Integrated Approach, 4th Ed.

2-26-1

PROBLEM 2-26
Statement: A part has been designed and its dimensions cannot be changed. To minimize its deflections under the same loading in all directions irrespective of stress levels, which material woulod you choose among the following: aluminum, titanium, steel, or stainless steel? See Mathcad file P0226.

Solution: 1.

Choose the material with the highest modulus of elasticity because deflection is inversely proportional to modulus of elasticity. Thus, choose steel unless there is a corrosive atmosphere, in which case, choose stainless steel.

MACHINE DESIGN - An Integrated Approach, 4th Ed.

2-27-1

PROBLEM 2-27
Statement: Assuming that the mechanical properties data given in Appendix Table A-9 for some carbon steels represents mean values, what is the value of the tensile yield strength for 1050 steel quenched and tempered at 400F if a reliability of 99.9% is required? Mean yield strength See Mathcad file P0227. S y 117 ksi S y 807 MPa

Given: Solution: 1.

From Table 2-2 the reliability factor for 99.9% is Re 0.753. Applying this to the mean tensile strength gives S y99.9 S y Re S y99.9 88.1 ksi S y99.9 607 MPa

MACHINE DESIGN - An Integrated Approach, 4th Ed.

2-28-1

PROBLEM 2-28
Statement: Assuming that the mechanical properties data given in Appendix Table A-9 for some carbon steels represents mean values, what is the value of the ultimate tensile strength for 4340 steel quenched and tempered at 800F if a reliability of 99.99% is required? Mean ultimate tensile strength See Mathcad file P0228. S ut 213 ksi S ut 1469 MPa

Given: Solution: 1.

From Table 2-2 the reliability factor for 99.99% is Re 0.702. Applying this to the mean ultimate tensile strength gives S ut99.99 S ut Re S ut99.99 150 ksi S ut99.99 1031 MPa

MACHINE DESIGN - An Integrated Approach, 4th Ed.

2-29-1

PROBLEM 2-29
Statement: Assuming that the mechanical properties data given in Appendix Table A-9 for some carbon steels represents mean values, what is the value of the ultimate tensile strength for 4130 steel quenched and tempered at 400F if a reliability of 90% is required? Mean ultimate tensile strength See Mathcad file P0229. S ut 236 ksi S ut 1627 MPa

Given: Solution: 1.

From Table 2-2 the reliability factor for 90% is Re 0.897. Applying this to the mean ultimate tensile strength gives S ut99.99 S ut Re S ut99.99 212 ksi S ut99.99 1460 MPa

MACHINE DESIGN - An Integrated Approach, 4th Ed.

2-30-1

PROBLEM 2-30
Statement: Assuming that the mechanical properties data given in Appendix Table A-9 for some carbon steels represents mean values, what is the value of the tensile yield strength for 4140 steel quenched and tempered at 800F if a reliability of 99.999% is required? Mean yield strength See Mathcad file P0230. S y 165 ksi S y 1138 MPa

Given: Solution:

1. From Table 2-2 the reliability factor for 99.999% is Re 0.659. Applying this to the mean tensile strength gives S y99.9 S y Re S y99.9 109 ksi S y99.9 750 MPa

MACHINE DESIGN - An Integrated Approach, 4th Ed.

2-31-1

PROBLEM 2-31
Statement: A steel part is to be plated to give it better corrosion resistance. Two materials are being considered: cadmium and nickel. Considering only the problem of galvanic action, which would you chose? Why? See Mathcad file P0231.

Solution: 1.

From Table 2-4 we see that cadmium is closer to steel than nickel. Therefore, from the standpoint of reduced galvanic action, cadmium is the better choice. Also, since cadmium is less noble than steel it will be the material that is consumed by the galvanic action. If nickel were used the steel would be consumed by galvanic action.

MACHINE DESIGN - An Integrated Approach, 4th Ed.

2-32-1

PROBLEM 2-32
Statement: A steel part with many holes and sharp corners is to be plated with nickel. Two processes are being considered: electroplating and electroless plating. Which process would you chose? Why? See Mathcad file P0232.

Solution: 1.

Electroless plating is the better choice since it will give a uniform coating thickness in the sharp corners and in the holes. It also provides a relatively hard surface of about 43 HRC.

MACHINE DESIGN - An Integrated Approach, 4th Ed.

2-33-1

PROBLEM 2-33
Statement: What is the common treatment used on aluminum to prevent oxidation? What other metals can also be treated with this method? What options are available with this method? See Mathcad file P0233.

Solution: 1.

Aluminum is commonly treated by anodizing, which creates a thin layer of aluminum oxide on the surface. Titanium, magnesium, and zinc can also be anodized. Common options include tinting to give various colors to the surface and the use of "hard anodizing" to create a thicker, harder surface.

MACHINE DESIGN - An Integrated Approach, 4th Ed.

2-34-1

PROBLEM 2-34
Statement: Steel is often plated with a less nobel metal that acts as a sacrificial anode that will corrode instead of the steel. What metal is commonly used for this purpose (when the finished product will not be exposed to saltwater), what is the coating process called, and what are the common processes used to obtain the finished product? See Mathcad file P0234.

Solution: 1.

The most commonly used metal is zinc. The process is called "galvanizing" and it is accomplished by electroplating or hot dipping.

MACHINE DESIGN - An Integrated Approach, 4th Ed.

2-35-1

PROBLEM 2-35
Statement: A low-carbon steel part is to be heat-treated to increase its strength. If an ultimate tensile strength of approximately 550 MPa is required, what mean Brinell hardness should the part have after treatment? What is the equivalent hardness on the Rockwell scale? Approximate tensile strength See Mathcad file P0235. S ut 550 MPa

Given: Solution: 1.

Use equation (2.10), solving for the Brinell hardness, HB. S ut = 3.45 HB HB S ut 3.45 MPa HB 159

2.

From Table 2-3, the equivalent hardness on the Rocwell scale is 83.9HRB.

MACHINE DESIGN - An Integrated Approach, 4th Ed.

2-36-1

PROBLEM 2-36
Statement: A low-carbon steel part has been tested for hardness using the Brinell method and is found to have a hardness of 220 HB. What are the approximate lower and upper limits of the ultimate tensile strength of this part in MPa? Hardness HB 220

Given: Solution: 1.

See Mathcad file P0236.

Use equation (2.10), solving for ultimate tensile strength. Minimum: S utmin ( 3.45 HB 0.2 HB) MPa S utmax ( 3.45 HB 0.2 HB) MPa S utmin 715 MPa S utmax 803 MPa

Maximum:

MACHINE DESIGN - An Integrated Approach, 4th Ed.

2-37-1

PROBLEM 2-37
Statement: Figure 2-24 shows "guide lines" for minimum weight design when failure is the criterion. The guide line, or index, for minimizing the weight of a beam in bending is f2/3/, where f is the yield strength of a material and is its mass density. For a given cross-section shape the weight of a beam with given loading will be minimized when this index is maximized. The following materials are being considered for a beam application: 5052 aluminum, cold rolled; CA-170 beryllium copper, hard plus aged; and 4130 steel, Q & T @ 1200F. The use of which of these three materials will result in the least-weight beam? Units: Given: Mg kg
3

5052 Aluminum

S ya 255 MPa

a 2.8 Mg m b 8.3 Mg m s 7.8 Mg m

3 3

CA-170 beryllium copper S yb 1172 MPa 4130 steel Solution: 1. 2. See Mathcad file P0237. S ys 703 MPa

The values for the mass density are taken from Appendix Table A-1 and the values of yield strength come from from Tables A-2, A-4, and A-9 for aluminum, beryllium copper, and steel, respectively. Calculate the index value for each material. Index S y Sy
0.667

Mg m MPa

0.667

Aluminum Beryllium copper Steel

Ia Index S ya a Ib Index S yb b Is Index S ys s

Ia 14.4 Ib 13.4 Is 10.2

The 5052 aluminum has the highest value of the index and would be the best choice to minimize weight.

MACHINE DESIGN - An Integrated Approach, 4th Ed.

2-38-1

PROBLEM 2-38
Statement: Figure 2-24 shows "guide lines" for minimum weight design when failure is the criterion. The guide line, or index, for minimizing the weight of a member in tension is f/, where f is the yield strength of a material and is its mass density. The weight of a member with given loading will be minimized when this index is maximized. For the three materials given in Problem 2-37, which will result in the least weight tension member? Units: Given: Mg kg
3

5052 Aluminum

S ya 255 MPa

a 2.8 Mg m b 8.3 Mg m s 7.8 Mg m

3 3

CA-170 beryllium copper S yb 1172 MPa 4130 steel Solution: 1. 2. See Mathcad file P0238. S ys 703 MPa

The values for the mass density are taken from Appendix Table A-1 and the values of yield strength come from from Tables A-2, A-4, and A-9 for aluminum, beryllium copper, and steel, respectively. Calculate the index value for each material. Index S y S y Mg m 3 MPa Ia 91.1 Ib 141.2 Is 90.1

Aluminum Beryllium copper Steel

Ia Index S ya a Ib Index S yb b Is Index S ys s

The beryllium copper has the highest value of the index and would be the best choice to minimize weight.

MACHINE DESIGN - An Integrated Approach, 4th Ed.

2-39-1

PROBLEM 2-39
Statement: Figure 2-23 shows "guide lines" for minimum weight design when stiffness is the criterion. The guide line, or index, for minimizing the weight of a beam in bending is E1/2/, where E is the modulus of elasticity of a material and is its mass density. For a given cross-section shape the weight of a beam with given stiffness will be minimized when this index is maximized. The following materials are being considered for a beam application: 5052 aluminum, cold rolled; CA-170 beryllium copper, hard plus aged; and 4130 steel, Q & T @ 1200F. The use of which of these three materials will result in the least-weight beam? Units: Given: Mg kg
3

5052 Aluminum

Ea 71.7 GPa

a 2.8 Mg m b 8.3 Mg m s 7.8 Mg m

3 3

CA-170 beryllium copper Eb 127.6 GPa 4130 steel Solution: 1. 2. See Mathcad file P0239. Es 206.8 GPa

The values for the mass density and modulus are taken from Appendix Table A-1. Calculate the index value for each material. E
0.5

Index( E )

Mg m GPa

0.5

Aluminum Beryllium copper Steel

Ia Index Ea a Ib Index Eb b Is Index Es s

Ia 3.0 Ib 1.4 Is 1.8

The 5052 aluminum has the highest value of the index and would be the best choice to minimize weight.

MACHINE DESIGN - An Integrated Approach, 4th Ed.

2-40-1

PROBLEM 2-40
Statement: Figure 2-24 shows "guide lines" for minimum weight design when stiffness is the criterion. The guide line, or index, for minimizing the weight of a member in tension is E/, where E is the modulus of elasticity of a material and is its mass density. The weight of a member with given stiffness will be minimized when this index is maximized. For the three materials given in Problem 2-39, which will result in the least weight tension member? Mg kg
3

Units: Given:

5052 Aluminum

Ea 71.7 GPa

a 2.8 Mg m b 8.3 Mg m s 7.8 Mg m

3 3

CA-170 beryllium copper Eb 127.6 GPa 4130 steel Solution: 1. 2. See Mathcad file P0240. Es 206.8 GPa

The values for the mass density and modulus are taken from Appendix Table A-1. Calculate the index value for each material. E Mg m GPa
3

Index( E )

Aluminum Beryllium copper Steel

Ia Index Ea a Ib Index Eb b Is Index Es s

Ia 25.6 Ib 15.4 Is 26.5

The steel has the highest value of the index and would be the best choice to minimize weight.

MACHINE DESIGN - An Integrated Approach, 4th Ed.

3-1-1

PROBLEM 3-1
Statement: Which load class from Table 3-1 best suits these systems? (a) Bicycle frame (b) Flag pole (c) Boat oar (d) Diving board (e) Pipe wrench (f) Golf club. See Mathcad file P0301.

Solution:

1. Determine whether the system has stationary or moving elements, and whether the there are constant or time-varying loads. (a) Bicycle frame Class 4 (Moving element, time-varying loads) (b) Flag pole (c) Boat oar Class 2 (Stationary element, time-varying loads) Class 2 (Low acceleration element, time-varying loads)

(d) Diving board Class 2 (Stationary element, time-varying loads) (e) Pipe wrench Class 2 (Low acceleration elements, time-varying loads) (f) Golf club Class 4 (Moving element, time-varying loads)

MACHINE DESIGN - An Integrated Approach, 4th Ed.

3-2a-1

PROBLEM 3-2a
Statement: Draw free-body diagrams for the system of Problem 3-1a (bicycle frame). Assumptions: 1. A two-dimensional model is adequate. 2. The lower front-fork bearing at C takes all of the thrust load from the front forks. 3. There are no significant forces on the handle bars. Solution: 1. See Mathcad file P0302a.

A typical bicycle frame is shown in Figure 3-2a. There are five points on the frame where external forces and moments are present. The rider's seat is mounted through a tube at A. This is a rigid connection, capable of transmitting two force components and a moment. The handle bars and front-wheel forks are supported by the frame through two bearings, located at B and C. These bearings are capable of transmitting radial and axial loads. The pedal-arm assembly is supported by bearings at D. These bearings are capable of transmitting radial loads. The rear wheel-sprocket assembly is supported by bearings mounted on an axle fixed to the frame at E.
Ra Rb Fbr Rc Fay Rd Re Fey C

Ma Fax

B Fct Fcr

Fex Fdx D

Fdy

FIGURE 3-2a
Free Body Diagram for Problem 3-2a

2. The loads at B and C can be determined by analyzing a FBD of the front wheel-front forks assembly. The loads at D can be determined by analyzing a FBD of the pedal-arm and front sprocket (see Problem 3-3), and the loads at E can be determined by analyzing a FBD of the rear wheel-sprocket assembly. 3. With the loads at B, C, D, and E known, we can apply equations 3.3b to the FBD of the frame and solve for Fax , Fay , and Ma.

Fx : Fy : Mz:

Fax Fbr cos( ) + Fcr cos( ) Fct sin( ) Fdx + Fex = 0 Fay Fbr sin( ) + Fcr sin( ) + Fct cos( ) Fdy + Fey = 0

(1) (2) (3)

Ma + ( Rbx Fby Rby Fbx ) + ( Rcx Fcy Rcy Fcx) ... = 0 + ( R F R F ) + ( R F R F ) ex ey ey ex dx dy dy dx

MACHINE DESIGN - An Integrated Approach, 4th Ed.

3-2e-1

PROBLEM 3-2e
Statement: Draw free-body diagrams for the system of Problem 3-1e (pipe wrench). Assumptions: A two-dimensional model is adequate. Solution: See Mathcad file P0302e.

1. A typical pipe wrench with a pipe clamped in its jaw is shown in Figure 3-2e(a). When a force Fhand is applied on the wrench, the piping system provides an equal and opposite force and a resisting torque, Tpipe.

Fhand

Tpipe Fhand a

(a) FBD of pipe wrench and pipe

Fbt Fbn
Fax

A Fay d b

(b) FBD of pipe wrench only FIGURE 3-2e


Free Body Diagrams for Problem 3-2e

2. The pipe reacts with the wrench at the points of contact A and B. The forces here will be directed along the common normals and tangents. The jaws are slightly tapered and, as a result, the action of Fhand tends to drive the wrench further into the taper, increasing the normal forces. This, in turn, allows for increasing tangential forces. It is the tangential forces that produce the turning torque. 3. Applying equations 3.3b to the FBD of the pipe wrench,

Fx : Fy : M A:
4.

Fax + Fbn cos( ) Fbt sin( ) = 0 Fay + Fbn sin( ) + Fbt cos( ) Fhand = 0 d ( Fbt cos( ) + Fbn sin( ) ) ( d + a ) Fhand = 0

(1) (2) (3)

These equations can be solved for the vertical forces if we assume is small so that sin() = 0 and cos () = 1.

MACHINE DESIGN - An Integrated Approach, 4th Ed.

3-3-1

PROBLEM 3-3
Statement: Draw a free-body diagram of the pedal-arm assembly from a bicycle with the pedal arms in the horizontal position and dimensions as shown in Figure P3-1. (Consider the two arms, pedals and pivot as one piece). Assuming a rider-applied force of 1500 N at the pedal, determine the torque applied to the chain sprocket and the maximum bending moment and torque in the pedal arm. a 170 mm b 60 mm Frider 1.5 kN

Given:

Assumptions: The pedal-arm assembly is supprted by bearings at A and at B. Solution: See Figure 3-3 and Mathcad file P0303.

1. The free-body diagram (FBD) of the pedal-arm assembly (including the sprocket) is shown in Figure 3-3a. The rider-applied force is Frider and the force applied by the chain (not shown) is Fchain. The radial bearing reactions are Fax, Faz, Fbx, and Fbz. Thus, there are five unknowns Fchain, Fax, Faz, Fbx, and Fbz. In general, we can write six equilibrium equations for a three-dimensional force system, but in this system there are no forces in the y-direction so five equations are available to solve for the unknowns.
z Fchain Faz a Frider b Arm Fax Fbx Pedal x y A B Arm (sectioned) Fbz Sprocket

(a) FBD of complete pedal-arm assembly


z

a Frider b Mc Arm

Tc

Fc Pedal x

(b) FBD of pedal and arm with section through the origin FIGURE 3-3
Dimensions and Free Body Diagram of the pedal-arm assembly for Problem 3-3

2.

The torque available to turn the sprocket is found by summing moments about the sprocket axis. From Figure 3-3a, it is

MACHINE DESIGN - An Integrated Approach, 4th Ed.

3-3-2

Ty-axis:

a Frider r Fchain = a Frider Tsprocket = 0 Tsprocket a Frider Tsprocket 255 N m

where r is the sprocket pitch radius. 3. In order to determine the bending moment and twisting torque in the pedal arm, we will cut the arm with a section plane that goes through the origin and is parallel to the y-z plane, removing everything beyond that plane and replacing it with the internal forces and moments in the pedal arm at the section. The resulting FBD is shown in Figure 3-3b. The internal force at section C is Fc the internal bending moment is Mc, and the internal twisting moment (torque) is Tc. We can write three equilibrium equations to solve for these three unknowns: Shear force in pedal arm at section C

Fz : My-axis: Mx-axis:

Fc Frider = 0

Fc Frider

Fc 1.5 kN

Bending moment in pedal arm at section C a Frider Mc = 0 Mc a Frider Mc 255 N m

Twisting moment in pedal arm at section C b Frider Tc = 0 Tc b Frider Tc 90 N m

MACHINE DESIGN - An Integrated Approach,4th Ed.

3-4-1

PROBLEM 3-4
Statement: The trailer hitch from Figure 1-1 has loads applied as shown in Figure P3-2. The tongue weight of 100 kg acts downward and the pull force of 4905 N acts horizontally. Using the dimensions of the ball bracket in Figure 1-5 (p. 15), draw a free-body diagram of the ball bracket and find the tensile and shear loads applied to the two bolts that attach the bracket to the channel in Figure 1-1. a 40 mm Mtongue 100 kg b 31 mm Fpull 4.905 kN c 70 mm t 19 mm d 20 mm

Given:

Assumptions: 1. The nuts are just snug-tight (no pre-load), which is the worst case. 2. All reactions will be concentrated loads rather than distributed loads or pressures. Solution: 1. See Figure 3-4 and Mathcad file P0304.

The weight on the tongue is Wtongue Mtongue g Wtongue 0.981 kN

2. The FBD of the hitch and bracket assembly is shown in Figure 3-4. The known external forces that act on the ball are Fpull and Wtongue . The reactions on the bracket are at points C and D. The bolts at C provide tensile (Fc2x) and shear (Fc2y) forces, and the bracket resists rotation about point D where the reaction force Fd2 is applied by the channel to which the bracket is bolted. 3. Solving for the reactions by summing the horizontal and vertical forces and the moments about D:

Fx : Fy : MD :

Fpull Fc2x Fd2 = 0


Fc2y Wtongue = 0 Fc2x d Fpull ( a t b d ) Wtongue c = 0

(1) (2) (3)

W tongue 70 = c

F pull

40 = a 2 A

19 = t 31 = b Fc2x

C 20 = d D

C D

Fd2 F c2y

FIGURE 3-4
Dimensions and Free Body Diagram for Problem 3-4

4.

Solving equation (3) for Fc2x

MACHINE DESIGN - An Integrated Approach,4th Ed.


Fpull ( a t b d ) Wtongue c d

3-4-2

Fc2x 5.

Fc2x 30.41 kN

(4)

Substituting into (1) and solving for Fd2 Fd2 Fc2x Fpull Fd2 25.505 kN (5)

6.

Solving (2) for Fc2y Fc2y Wtongue Fc2y 0.981 kN (6)

7.

The loads applied to the two bolts that attach the bracket to the channel are: Axial force on two bolts Shear force on two bolts Fc2x 30.4 kN Fc2y 0.98 kN

We assume that each bolt would carry one half of these loads.

MACHINE DESIGN - An Integrated Approach, 4th Ed.

3-5-1

PROBLEM 3-5
Statement: Given: For the trailer hitch of Problem 3-4, determine the horizontal force that will result on the ball from accelerating a 2000-kg trailer to 60 m/sec in 20 sec. Mass of trailer Final velocity Time to reach velocity Mtrailer 2000 kg vf 60 m sec

20 sec

Assumptions: 1. Acceleration is constant. 2. The rolling resistance of the tires and the wheel bearings is negligible. Solution: 1. See Mathcad file P0305.

From elementary kinematics, the acceleration required is a vf a 3.00 m sec


2

(1)

2.

Using Newton's second law to find the force required to accelerate the trailer, Fhitch Mtrailer a Fhitch 6.00 kN (2)

MACHINE DESIGN - An Integrated Approach, 4th Ed.

3-6-1

PROBLEM 3-6
Statement: For the trailer hitch of Problem 3-4, determine the horizontal force that will result on the ball from an impact between the ball and the tongue of the 2000-kg trailer if the hitch deflects 2.8 mm dynamically on impact. The tractor weighs 1000 kg. The velocity at impact is 0.3 m/sec. Mass of trailer Dynamic deflection Mass of tractor Impact velocity Mtrailer 2000 kg

Given:

i 2.8 mm
Mtractor 1000 kg vi 0.3 m sec

Assumptions: 1. The tractor is the "struck member" because the hitch is on the tractor and it is the hitch that deflects. 2. Equations (3.9) and (3.10) can be used to model the impact. Solution: 1. See Mathcad file P0306.

The weight of the trailer (the "striking member") is Wtrailer Mtrailer g Wtrailer 19.613 kN

2.

The correction factor, from equation (3-15), is

1 Mtractor 3 Mtrailer

0.857

3.

Eliminating E from equations (3.9a) and (3.10) and solving for the horizontal force on the ball Fi yields 1 2 Fi i = Mtrailer vi 2 2 1

Fi

Mtrailer vi i

Fi 55.1 kN

MACHINE DESIGN - An Integrated Approach, 4th Ed.

3-7-1

PROBLEM 3-7
Statement: The piston of an internal-combustion engine is connected to its connecting rod with a "wrist pin." Find the force on the wrist pin if the 0.5-kg piston has an acceleration of 2 500 g. Mass of piston Acceleration of piston Mpiston 0.5 kg a piston 2500 g

Given:

Assumptions: The force on the wrist pin due to the weight of the piston is very small compared with the acceleration force. Solution: 1. See Mathcad file P0307. a piston 2.452 10
4

The acceleration in m/s is

m sec
2

2.

Using Newton's Second Law expressed in equation (3.1a), the force on the wrist pin is Fwristpin Mpiston a piston Fwristpin 12.258 kN

MACHINE DESIGN - An Integrated Approach, 4th Ed.

3-8-1

PROBLEM 3-8
Statement:

_____

A cam-follower system similar to that shown in Figure 3-15 has a mass m = 1 kg, a spring constant k = 1000 N/m, and a damping coefficient d = 19.4 N-s/m. Find the undamped and damped natural frequencies of this system. cps := 2 rad sec Mass
1 1

Units: Given:

M := 1 kg,

Spring constant
1

k := 1000 N m

Damping coefficient d := 19.4 N s m Solution: 1.

See Figure 3-15 and Mathcad file P0308.

Calculate the undamped natural frequency using equation 3.4. k M rad sec

n :=
2.

n = 31.6

n = 5.03 cps

Calculate the undamped natural frequency using equation 3.7.


2

d :=

k M

d 2 M

d = 30.1

rad sec

d = 4.79 cps

MACHINE DESIGN - An Integrated Approach, 4th Ed.

3-9-1

PROBLEM 3-9
Statement: A ViseGrip plier-wrench is drawn to scale in Figure P3-3. Scale the drawing for dimensions. Find the forces acting on each pin and member of the assembly for an assumed clamping force of P = 4000 N in the position shown. What force F is required to keep it in the clamped position shown? Clamping force Dimensions P 4.00 kN a 50.0 mm b 55.0 mm c 39.5 mm d 22.0 mm e 28.0 mm f 26.9 mm g 2.8 mm h 21.2 mm

Given:

21.0 deg 129.2 deg

Assumptions: Links 3 and 4 are in a toggle position, i.e., the pin that joins links 3 and 4 is in line with the pins that join 1 with 4 and 2 with 3. Solution: 1. See Figure 3-9 and Mathcad file P0309.

The FBDs of the assembly and each individual link are shown in Figure 3-9. The dimensions, as scaled from Figure P3-3 in the text, are given above and are shown on the link FBDs.
F 4 P 1

3 F
55.0 = b 50.0 = a 22.0 = d

2 P

F14

39.5 = c

129.2

4 F34 P

F41

F21

28.0 = e

2.8 = g

F43 3 F23 F

F12 F32 2

21.2 = h

26.9 = f

FIGURE 3-9
Free Body Diagrams for Problem 3-9

2.

Looking first at Part 3, we see that it is a three-force body. Therefore, the lines of action of the three forces must intersect at a point. But, since Parts 3 and 4 are in a toggle position, F43 and F23 are colinear, which means that their x- and y-components must be equal and opposite, leading to the conclussion that F = 0.

MACHINE DESIGN - An Integrated Approach, 4th Ed.

3-9-2

3.

Now, looking at Part 1, we see that (for F = 0) it is also a three-force body, as is Part 2. In fact, the forces on Part 1 are identical to those on Part 2. Solving for the unknown reactions on Parts 1 and 2, Fx: Fy: F41 cos( 180 deg ) F21 cos( 180 deg) = 0 F41 sin( 180 deg ) F21 sin( 180 deg) P = 0 (a) (b)

Solving equation (a) for F21 F21 = F41 cos( 180 deg ) cos( 180 deg) (c)

Substituting equation (c) into (b) F41 sin( 180 deg ) Solving equation (d) for F41 F41 sin( 180 deg ) P cos( 180 deg ) cos( 180 deg) sin( 180 deg) F41 cos( 180 deg ) cos( 180 deg) sin( 180 deg) P = 0 (d)

F21

F41 cos( 180 deg ) cos( 180 deg)

F41 5.1 kN F21 7.5 kN

Checking moment balance on Part 1, F41 sin( ) c F21 sin( 90 deg) d P a 0 kN m The result is, within the accuracy of the scaled dimensions, zero as it must be. 4. The x and y components of the pin forces on Part 1 are F41x F41 cos( 180 deg ) F41y F41 sin( 180 deg ) F21x F21 cos( 180 deg) F21y F21 sin( 180 deg) 5. The forces on the pins at the ends of Part 4 are F14 F41 F34 F14 6. The forces on the pins at the ends of Part 3 are F43 F34 F23 F43 F43 5.1 kN F23 5.1 kN F14 5.1 kN F34 5.1 kN F41x 4.749 kN F41y 1.823 kN F21x 4.749 kN F21y 5.823 kN

MACHINE DESIGN - An Integrated Approach, 4th Ed.

3-9-3

7.

The forces on the pins at the ends of Part 2 are F12 F21 F32 F23 Checking moment equilibrium on Part 2, F12 ( e cos( 90 deg) g sin( 90 deg) ) 0 kN m F32 ( h cos( ) f sin( ) ) which is zero, as it must be. F12 7.5 kN F32 5.1 kN

MACHINE DESIGN - An Integrated Approach, 4th Ed.

3-10-1

PROBLEM 3-10
Statement: An overhung diving board is shown in Figure P3-4a. Find the reaction forces and construct the shear and moment diagrams for this board with a 100 kg person standing at the free end. Determine the maximum shear force, maximum moment and their locations. Beam length Distance to support Mass at free end L 2000 mm a 700 mm M 100 kg
2000 = L R1 P

Given:

Assumptions: The weight of the beam is negligible compared to the applied load and so can be ignored.
700 = a

R2

Solution:

See Figure 3-10 and Mathcad file P0310. FIGURE 3-10A


Free Body Diagram for Problem 3-10

1. From inspection of Figure 3-10, write the load function equation q(x) = -R1<x - 0>-1 + R2<x - a>-1 - P<x - L >-1 2. Integrate this equation from - to x to obtain shear, V(x) V(x) = -R1<x - 0>0 + R2<x - a>0 - P<x - L >0 3. Integrate this equation from - to x to obtain moment, M(x) M(x) = -R1<x - 0>1 + R2<x - a>1 - P<x - L >1 4. Determine the magnitude of the force, P P M g P 980.7 N

5. Solve for the reactions by evaluating the shear and moment equations at a point just to the right of x = L, where both are zero. At x = L+, V = M = 0 R1 P La a V = R1 R2 P = 0 R1 1821 N R2 2802 N M = R1 L R2 ( L a ) = 0

R2 P R1 6. Define the range for x

x 0 in 0.005 L L

7. For a Mathcad solution, define a step function S. This function will have a value of zero when x is less than z, and a value of one when it is greater than or equal to z. S ( x z) if ( x z 1 0 ) 8. Write the shear and moment equations in Mathcad form, using the function S as a multiplying factor to get the effect of the singularity functions. V ( x) R1 S ( x 0 in) R2 S ( x a ) P S ( x L) M ( x) R1 S ( x 0 in) x R2 S ( x a ) ( x a ) P S ( x L) ( x L)

MACHINE DESIGN - An Integrated Approach, 4th Ed.


9. Plot the shear and moment diagrams. Shear Diagram
1000

3-10-2

0 V ( x) N 1000

2000

500

1000 x mm

1500

2000

Moment Diagram

375 M ( x) Nm

750

1125

1500

500

1000 x mm

1500

2000

FIGURE 3-10B
Shear and Moment Diagrams for Problem 3-10

10. The maximum value of the shear force ocuurs throughout the distance from x = 0 to x = a and is R1 1821 N 11. Find the maximum value of the bending moment by determining the value of x where the shear is zero. Inspection of the shear diagram shows that this occurs at x = a. Mmax M ( a ) Mmax 1275 N m

MACHINE DESIGN - An Integrated Approach, 4th Ed.

3-11-1

PROBLEM 3-11
Statement: Determine the impact force and dynamic deflection that will result when the 100-kg person in Problem 3-10 jumps up 250 mm and lands back on the board. Assume that the board weighs 29 kg and deflects 131 mm statically when the person stands on it. Find the reaction forces and construct the shear and moment diagrams for this dynamic loading. Determine the maximum shear force, maximum moment and their locations along the length of the board. Beam length Distance to support Mass of person Mass of board Static deflection Height of jump L 2000 mm a 700 mm mpers 100 kg mboard 29 kg
R1 2000 = L Fi

Given:

st 131 mm
h 250 mm
700 = a

R2

Assumptions: Equation (3.15) can be used to approximate a mass correction factor. Solution: See Figure 3-11 and Mathcad file P0311.

FIGURE 3-11A
Free Body Diagram for Problem 3-11

1. The person is the moving object and the board is the struck object. The mass ratio to be used in equation (3.15) for the correction factor is massratio mpers mboard massratio 3.448

2. From equation (3.15), the correction factor is

1 1 3 massratio

0.912

3. The weight of the moving mass is

Wpers mpers g

Wpers 0.981 kN

4. The dynamic force is found by solving equation (3.14) for Fi. Fi Wpers 1

2 h

st
Fi Wpers 3.12

Fi 3.056 kN

From this we see that the dynamic force ratio is

5. From inspection of Figure 3-11, write the load function equation q(x) = -R1<x - 0>-1 + R2<x - a>-1 - Fi<x - L >-1 6. Integrate this equation from - to x to obtain shear, V(x) V(x) = -R1<x - 0>0 + R2<x - a>0 - Fi<x - L >0 7. Integrate this equation from - to x to obtain moment, M(x) M(x) = -R1<x - 0>1 + R2<x - a>1 - Fi<x - L >1

MACHINE DESIGN - An Integrated Approach, 4th Ed.

3-11-2

8. Solve for the reactions by evaluating the shear and moment equations at a point just to the right of x = L, where both are zero. At x = L+, V = M = 0 R1 Fi L a a V = R1 R2 Fi = 0 R1 5676 N R2 8733 N M = R1 L R2 ( L a ) = 0

R2 Fi R1 9. Define the range for x

x 0 in 0.005 L L

10. For a Mathcad solution, define a step function S. This function will have a value of zero when x is less than z, and a value of one when it is greater than or equal to z. S ( x z) if ( x z 1 0 ) 11. Write the shear and moment equations in Mathcad form, using the function S as a multiplying factor to get the effect of the singularity functions. V ( x) R1 S ( x 0 in) R2 S ( x a ) Fi S ( x L) M ( x) R1 S ( x 0 in) x R2 S ( x a ) ( x a ) Fi S ( x L) ( x L) 12. Plot the shear and moment diagrams. Shear Diagram
4 2 0 2 4 6

Moment Diagram
0

1 M ( x)

V ( x) kN

2 kN m 3

0.5

1 x m

1.5

0.5

1 x m

1.5

FIGURE 3-11B
Shear and Moment Diagrams for Problem 3-11

13. The maximum value of the shear force ocuurs throughout the distance from x = 0 to x = a and is R1 5676 N 14. Find the maximum value of the bending moment by determining the value of x where the shear is zero. Inspection of the shear diagram shows that this occurs at x = a. Mmax M ( a ) Mmax 3973 N m

MACHINE DESIGN - An Integrated Approach, 4th Ed.

3-12-1

PROBLEM 3-12
Statement: An overhung diving board is shown in Figure P3-4b. Find the reaction forces and construct the shear and moment diagrams for this board with a 100 kg person standing at the free end. Determine the maximum shear force, maximum moment and their locations. Beam length Mass at free end L 1300 mm M 100 kg
2000 1300 = L P

Given:

Assumptions: 1. The weight of the beam is negligible compared to the applied load and so can be ignored. Solution: See Figure 3-12 and Mathcad file P0312.

M1 700

R1

1. From inspection of Figure 3-12, write the load function equation q(x) = -M1<x - 0>-2 + R1<x - a>-1 - P<x - L >-1 2. Integrate this equation from - to x to obtain shear, V(x) V(x) = -M1<x - 0>-1 + R1<x - a>0 - P<x - L >0 3. Integrate this equation from - to x to obtain moment, M(x) M(x) = -M1<x - 0>0 + R1<x - a>1 - P<x - L >1 4. Determine the magnitude of the force, P P M g

FIGURE 3-12A
Free Body Diagram for Problem 3-12

P 980.7 N

5. Solve for the reactions by evaluating the shear and moment equations at a point just to the right of x = L, where both are zero. At x = L+, V = M = 0 R1 P M1 R1 L 6. Define the range for x V = R1 P = 0 R1 981 N M1 1275 m N M = M1 R1 L = 0

x 0 in 0.005 L L

7. For a Mathcad solution, define a step function S. This function will have a value of zero when x is less than z, and a value of one when it is greater than or equal to z. S ( x z) if ( x z 1 0 ) 8. Write the shear and moment equations in Mathcad form, using the function S as a multiplying factor to get the effect of the singularity functions. V ( x) R1 S ( x 0 mm) P S ( x L) M ( x) M1 S ( x 0 mm) R1 S ( x 0 mm) ( x 0 mm) P S ( x L) ( x L) 9. Plot the shear and moment diagrams.

MACHINE DESIGN - An Integrated Approach, 4th Ed.

3-12-2

Shear Diagram

1000 800 600 400 200 0

V ( x) N

0.5

1 x m

1.5

Moment Diagram

0 300 600 900 1200 1500

M ( x) Nm

0.5

1 x m

1.5

FIGURE 3-12B
Shear and Moment Diagrams for Problem 3-12

10. The maximum value of the shear force ocuurs throughout the distance from x = 0 to x = L and is R1 981 N 11. Find the maximum value of the bending moment by determining the value of x where the shear is zero. Inspection of the shear diagram shows that this occurs at x = 0. Mmax M ( 0 mm) Mmax 1275 N m

MACHINE DESIGN - An Integrated Approach, 4th Ed.

3-13-1

PROBLEM 3-13
Statement: Determine the impact force and dynamic deflection that will result when a 100-kg person jumps up 25 cm and lands back on the board. Assume the board weighs 19 kg and deflects 8.5 cm statically when the person stands on it. Find the reaction forces and construct the shear and moment diagrams for this dynamic loading. Determine the maximum shear force, maximum moment, and their locations along the length of the board. Total board length Supported length Mass of board Static board deflection Mass of person Height of jump b 2000 mm a 700 mm mboard 19 kg
2000 1300 = L Fi

Given:

stat 85 mm
mperson 100 kg h 250 mm
M1 700 R1

Assumptions: 1. The board can be modelled as a cantilever beam with maximum shear and moment at the edge of the support. Solution: 1. See Figure 3-13 and Mathcad file P0313.

FIGURE 3-13A
Free Body Diagram for Problem 3-13

The person impacts the board upon landing. Thus, the board is the struck object and the person is the striking object. To determine the force exerted by the person we will first need to know the impact correction factor from equation (3.15).

1 mboard 3 mperson

0.94

(1)

2.

We can now use equation (3.14) to determine the impact force, Fi, Fi mperson g 1

2 h

stat

Fi 3.487 kN

(2)

3.

Write an equation for the load function in terms of equations 3.17 and integrate the resulting function twice using equations 3.18 to obtain the shear and moment functions. Note use of the unit doublet function to represent the moment at the wall. For the beam in Figure 3-13, q(x) = -M1<x - 0>-2 + R1<x - 0>-1 - Fi<x - l>-1 V(x) = -M1<x - 0>-1 + R1<x - 0>0 - Fi<x - l>0 + C1 M(x) = -M1<x - 0>0 + R1<x - 0>1 - Fi<x - l>1 + C1x+ C2 The reaction moment M1 at the wall is in the z direction and the forces R1 and Fi are in the y direction in equation (4). All moments in equation (5) are in the z direction. (3) (4) (5)

4. 5.

Because the reactions have been included in the loading function, the shear and moment diagrams both close to zero at each end of the beam, making C1 = C2 = 0. The reaction force R1 and the reaction moment M1 can be calculated from equations (4) and (5) respectively

by substituting the boundary conditions x = l+, V = 0, M = 0. Note that we can substitute l for l+ since their difference is vanishingly small.

MACHINE DESIGN - An Integrated Approach, 4th Ed.


Unsupported beam length l b a l 1300 mm

3-13-2

V(l) = -M1<l - 0>-1 + R1<l - 0>0 - Fi<l - l>0 = 0 V = R1 Fi = 0 R1 Fi M(l) = -M1<l - 0>0 + R1<l - 0>1 - Fi<l - l>1 = 0 M = M1 R1 l Fi ( l l) = 0 M1 R1 l M1 4533 N m (7) R1 3.487 kN (6)

6. To generate the shear and moment functions over the length of the beam, equations (4) and (5) must be evaluated for a range of values of x from 0 to l, after substituting the above values of C1, C2, R1, and M1 in them. For a Mathcad solution, define a step function S. This function will have a value of zero when x is less than the dummy variable z, and a value of one when it is greater than or equal to z. It will have the same effect as the singularity function. Range of x Unit step function x 0 in 0.005 l l S ( x z) if ( x z 1 0 )

Write the shear and moment equations in Mathcad form, using the function S as a multiplying factor to get the effect of the singularity functions. V ( x) R1 S ( x 0 in) ( x 0 ) Fi S ( x l) ( x l)
0 0 0 1 1

(8)

M ( x) M1 S ( x 0 in) ( x 0 ) R1 S ( x 0 in) ( x 0 ) Fi S ( x l ) ( x l) Plot the shear and moment diagrams (see below). Shear Diagram
0 1 2

Moment Diagram

3 V ( x) kN 2 M ( x)

kN m 3 4 5

0.5 x m

0.5 x m

1.5

7. The graphs show that the shear force and the moment are both largest at x = 0. The function values of these points can be calculated from equations (4) and (5) respectively by substituting x = 0 and evaluating the singularity functions: Vmax = V(0) = R1<0 - 0>0 - Fi<0 - l>0 = R1 (9)

MACHINE DESIGN - An Integrated Approach, 4th Ed.


Vmax R1 Vmax 3.49 kN

3-13-3

M.max = M(0) = -M1<0 - 0>0 + R1<0 - 0>1 - Fi<0 - l>1 = -M1 Mmax M1 Mmax 4533 N m

(10)

MACHINE DESIGN - An Integrated Approach, 4th Ed.

3-14-1

PROBLEM 3-14
Statement: Figure P3-5 shows a child's toy called a pogo stick. The child stands on the pads, applying half her weight on each side. She jumps off the ground, holding the pads up against her feet, and bounces along with the spring cushioning the impact and storing energy to help each rebound. Find the natural frequency of the system, the static deflection of the spring with the child standing still, and the dynamic force and deflection when the child lands after jumping 2 in off the ground.
2

Units: Given:

blob :=

lbf sec in

Child's weight Spring constant Pogo stick weight Height of drop

Wc := 60 lbf k := 100 lbf in Wp := 5 lbf h := 2 in


1

Assumptions: 1. An approximate energy method will be acceptable. 2. The correction factor for energy dissipation will be applied. Solution: See Figure 3-14 and Mathcad file P0314.

1. Find the natural frequency of the (child/spring) system. Mass of child (striker) m := Wc g Wp g k m m = 0.155 blob

Fi /2

Fi /2

Mass of stick (struck)

mb :=

mb = 0.013 blob

Natural frequency

:=

= 25.367

rad sec

f :=

P
f = 4.037 Hz FIGURE 3-14
Free Body Diagram for Problem 3-14

2. The static deflection of the spring with the child standing still is Static deflection of spring

st :=

Wc k

st = 0.6 in

3. Determine the mass ratio correction factor from equation (3.15): Correction factor

:=
1+

1 mb 3 m

= 0.973

4. Using equation (3.14), determine the dynamic force.

Fi := Wc 1 +

1+

2 h

st

Fi = 224 lbf

MACHINE DESIGN - An Integrated Approach, 4th Ed.

3-15-1

PROBLEM 3-15
Statement: A pen plotter imparts a constant acceleration of 2.5 m/sec2 to the pen assembly, which travels in a straight line across the paper. The moving pen assembly weighs 0.5 kg. The plotter weighs 5 kg. What coefficient of friction is needed between the plotter feet and the table top on which it sits to prevent the plotter from moving when the pen accelerates? Acceleration of pen ass'y a 2.5 m sec Mass of pen ass'y mpen 0.5 kg Mass of plotter Solution: 1. See Mathcad file P0315. mplot 5 kg
2

Given:

The force imparted to the pen assembly by the internal drive mechanism must be reacted at the table top by the plotter feet. The horizontal force at the feet will be equal to the force on the pen assembly and must be less than or equal to the maximum friction force, which is the product of the coefficient of friction and the normal force, which is the weight of the plotter. Horizontal driving force on pen ass'y Weight of plotter Minimum coefficient of friction Fpen mpen a Wplot mplot g Fpen 1.25 N Wplot 49.033 N

Fpen Wplot

0.025

MACHINE DESIGN - An Integrated Approach, 4th Ed.

3-16-1

PROBLEM 3-16
Statement: A track to guide bowling balls is designed with two round rods as shown in Figure P3-6. The rods are not parallel to one another but have a small angle between them. The balls roll on the rods until they fall between them and drop onto another track. The angle between the rods is varied to cause the ball to drop at different locations. Each rod's unsupported length is 30 in and the angle between them is 3.2 deg. The balls are 4.5 in dia and weigh 2.5 lb. The center distance between the 1-in-dia rods is 4.2 in at the narrow end. Find the distance from the narrow end at which the ball drops through and determine the worst-case shear and moment maximum for the rods as the ball rolls a distance from the narrow end that is 98% of the distance to drop. Assume that the rods are simply supported at each end and have zero deflection under the applied loading. (Note that assuming zero deflection is unrealistic. This assumption will be relaxed in the next chapter after deflection has been discussed.) Unsupported rod length Half-angle between rods Bowling ball diameter L 30 in 1.6 deg D 4.5 in Bowling ball weight Rod diameter Half width of rod gap W 2.5 lbf d 1.0 in c 2.1 in

Given:

Solution: 1.

See Figure 3-16 and Mathcad file P0316.

Calculate the distance between the ball and rod centers. Distance between centers h D d 2 h 2.75 in

c
TOP VIEW

u x
W/2 F

SECTION A-A

width(x)
(a) Distance between the roll axis and the rod axis. (b) Partial FBD of the bowling ball.

FIGURE 3-16
Dimensions and Free Body Diagrams for Problem 3-16

MACHINE DESIGN - An Integrated Approach, 4th Ed.


2.

3-16-2

Let x be the distance along the roll axis, and u be the corresponding distance to the point of contact between the ball and rods, measured along the rods. Then the distance from the center plane of the ball to the center of a rod as shown in Figure 3-16(a) is, width( x) c cos( ) x sin( ) And the distance from the narrow end to the point at which the ball drops (assuming rigid rods) is xdrop h c cos( ) sin( ) xdrop 23.31 in (1)

The distance along the rod corresponding to xdrop is u drop xdrop h sin( ) cos( ) u drop 23.24 in

3.

The angle made by a line through the ball-rod centers and the horizontal plane (see Figure 3-16b) is

( x) acos

width( x) h


0 ( 0 in) 98% 0.98 xdrop 0 40.241 deg 98% 5.577 deg

When x = 0, this is When x = 0.98xdrop, this is 4.

The loading on the ball is symmetric about its center plane along the x-axis. Figure 3-16(b) shows a FBD of one half of the ball with the internal forces along the plane of symmetry due to the reaction at the other rod omitted. With these forces omitted we may only sum forces in the vertical direction.

Fy :

F sin( ) F cos( ) W

W 2

=0

(2)

F=

2 ( sin( ) cos( ) )

(3)

5.

The ball will drop through the rods when is zero. If there were no friction force present ( = 0) then F would become very large as approached zero. The presence of the friction term in the denominator of equation (3) limits F to finite values. However, with the assumption that the rods are rigid, there is no way for the rods to provide a normal force when reaches zero. Thus, we will need to limit the range of for this analysis. Let

and

min 98%

Then

xmax

h cos min c cos( ) sin( ) xmax h sin( ) cos( ) 2 sin min W

xmax 22.84 in

u max Fmax

u max 22.77 in Fmax 12.86 lbf

(4)

MACHINE DESIGN - An Integrated Approach, 4th Ed.

3-16-3

6.

Determine the worst-case shear and moment maximum for the rods as the ball rolls along their length from Figure B-2(a) in Appendix B where a in the figure is u max. Then, Mmax Fmax u max 1

u max L

Mmax 70.6 in lbf

(5)

For the shear, we must find the reactions, which are R1 Fmax 1 R2 Fmax R1

u max L

R1 3.10 lbf R2 9.76 lbf

The maximum absolute value of shear is the larger of these two. Thus Vmax R2 Vmax 9.8 lbf (6)

MACHINE DESIGN - An Integrated Approach, 4th Ed.

3-17-1

PROBLEM 3-17
Statement: A pair of ice tongs is shown in Figure P3-7. The ice weighs 50 lb and is 10 in wide across the tongs. The distance between the handles is 4 in, and the mean radius r of a tong is 6 in. Draw free-body diagrams of the two tongs and find all forces acting on them. Determine the bending moment at point A. Weight of ice Distances W 50 lbf a x 11.0 in b x 5.0 in cx 2.0 in a y 6.0 in b y 12.0 in cy 3.5 in (1)

Given:

Assumptions: Assume that the horizontal force at C (the handle) is zero, thus Fc 0 lbf Solution: See Figure 3-17 and Mathcad file P0317.

F C FC O
11.0 = ax 3.5 = cy

FO
2.0 = cx 12.0 = by 5.0 = bx

FB B W/2

W
FIGURE 3-17A
Free Body Diagrams for Problem 3-17

1.

Summing forces and moments on a single tong (see FBD above right).

Fx Fy MC
2. 3.

FO FB FC = 0 W 2 F=0 W 2 b x cx = 0

(2) (3) (4) (5)

FO cy FB b y cy

From equations (1) and (2), FO = FB Eliminating FO from equations (4) and (5) and solving for FB FB W b x cx 2 by FB 14.58 lbf

MACHINE DESIGN - An Integrated Approach, 4th Ed.

3-17-2

F
4. From equation (3), the vertical force on one handle is F W 2 F 25 lbf

C FC O
11.0 = ax 3.5 = cy

FO
2.0 = cx

5. From Figure 3-17B we see that, at any section that we might take through the tong, there will be an internal moment, shear force, and axial force present. The bending moment will be a maximum at point A because it is the fartherest point from the centroid of the system. Summing forces and moments:

Fx Fy MO

-FDs cos + FDn sin (6) + FO = 0 -FDs sin - FDn cos +F=0 (7)

FDs M D FDn
FIGURE 3-17B
Free Body Diagram with section at D for Problem 3-17

F cx - M D - (FDs cos + FDn sin )(ay + rc sin ) + (FDs sin + FDn cos )[ax - rc (1 - cos)] = 0 (8)

6.

Solving equations (6) and (7) for FDs and FDn FDn = F cos( ) FO sin( ) FDs = FDn sin( ) FO cos( )

7.

The maximum value of MD will occur at = 0 deg. At = 0 deg, FO FB FDn F FDs FO MD F cx FDs a y FDn a x FDn 25 lbf FDs 14.58 lbf MD 237.5 lbf in

MACHINE DESIGN - An Integrated Approach, 4th Ed.

3-18-1

PROBLEM 3-18
Statement: A tractor-trailer tipped over while negotiating an on-ramp to the New York Thruway. The road has a 50-ft radius at that point and tilts 3 deg toward the outside of the curve. The 45-ft-long by 8-ft-wide by 8.5-ft-high trailer box (13 ft from ground to top) was loaded 44 415 lb of paper rolls in two rows by two high as shown in Figure P3-8. The rolls are 40-in-dia by 38-in-long and weigh about 900 lb each. They are wedged against rolling backward but not against sliding sidewards. The empty trailer weighed 14 000 lb. The driver claims that he was traveling at less than 15 mph and that the load of paper shifted inside the trailer, struck the trailer sidewall, and tipped the truck. The paper company that loaded the truck claims the load was properly stowed and would not shift at that speed. Independent test of the coefficient of friction between similar paper rolls and a similar trailer floor give a value of 0.43 +/- 0.08. The composite center of gravity of the loaded trailer is estimated to be 7.5 ft above the road. Determine the truck speed that would cause the truck to just begin to tip and the speed at which the rolls will just begin to slide sidways. What do you think caused the accident? Weight of paper Weight of trailer Radius of curve Nominal coefficient of friction Coefficient of friction uncertainty Trailer width Height of CG from pavement Wp := 44415 lbf Wt := 14000 lbf r := 50 ft nom := 0.43 u := 0.08 w := 8 ft h := 7.5 ft

Given:

Assumptions: 1. The paper rolls act as a monolith since they are tightly strapped together with steel bands. 2. The tractor has about 15 deg of potential roll freedom versus the trailer due to their relative angle in plan during the turn combined with the substantial pitch freedom in the fifth wheel. So the trailer can tip independently of the tractor. 3. The outside track width of the trailer tires is equal to the width of the trailer. Solution: See Figure 3-18 and Mathcad file P0318.

1. First, calculate the location of the trailer's CG with respect to the outside wheel when it is on the reverse-banked curve. From Figure 3-18A, Tilt angle a := h tan( ) b := w 2 a

:= 3 deg
a = 0.393 ft b = 3.607 ft

3 7.500'
ybar

xbar := b cos( ) ybar := b sin( ) + ybar = 7.699 ft

xbar = 3.602 ft h cos( )

b xbar 4.000'

The coordinates of the CG of the loaded trailer with respect to the lower outside corner of the tires are: xbar = 3.602 ft ybar = 7.699 ft

FIGURE 3-18A
Location of CG for Problem 3-18

MACHINE DESIGN - An Integrated Approach, 4th Ed. 3-18-2 2. The trailer is on the verge of tipping over when the copule due to centrifugal force is equal to the couple formed by the weight of the loaded trailer acting through its CG and the vertical reaction at the outside edge of the tires. At this instant, it is assumed that the entire weight of the trailer is reacted at the outside tires with the inside tires carrying none of the weight. Any increase in tangential velocity of the tractor and trailer will result in tipping. Summing moments about the tire edge (see Figure 3-18B),

Fw xbar Fc ybar = 0

(1)

where Fc is the centrifugal force due to the normal acceleration as the tractor and trailer go through the curve. The normal acceleration is a tip = vtip r
2

Fc
(2)

Fw ybar
(3)

and the force necessary to keep the tractor trailer following a circular path is Fc = mtot a tip where mtot is the total mass of the trailer and its payload. Combining equations (2) and (3) and solving for vtip, we have vtip = Fc r mtot

Rx Ry xbar
(4) FIGURE 3-18B
FBD of Trailer on the Verge of Tipping

or, vtip = Fc r g Fw (5)

3.

Calculate the minimum tipping velocity of the tractor/trailer. From equations (1) and (5), Total weight Centrifugal force required to tip the trailer Minimum tipping speed Fw := Wt + Wp Fc := xbar ybar Fw Fw = 58415 lbf Fc = 27329 lbf

vtip :=

Fc r g Fw

vtip = 18.7 mph

Thus, with the assumptions that we have made, the trailer would not begin to tip over until it reached a speed of vtip = 18.7 mph 4. The load will slip when the friction force between the paper rolls and the trailer floor is no longer sufficient to react the centrifugal force on the paper rolls. Looking at the FBD of the paper rolls in Figure 3-18C, we see that Normal force between paper and floor Fn = Wp cos( ) Fcp sin( ) (6)

MACHINE DESIGN - An Integrated Approach, 4th Ed.


Tangential force tending to slide the paper Ft = Wp sin( ) + Fcp cos( ) Centrifugal force on the paper W p vs = as = g g r Wp
2

3-18-3

(7)

Fcp Wp

Fcp

(8)

Ft Fn

But, the maximum friction force is Ff = Fn = Ft (9) FIGURE 3-18C


FBD of Paper on the Verge of Sliding

Substituting equation (9) into (7), then combining (6) and (7) to eliminate Fn, and solving for Fcp yields Wp ( cos( ) sin( ) ) sin( ) + cos( )

Fcp =

(10)

Substituting equation (10) into (8), to eliminate Fcp , and solving for vs yields

vs =

( cos( ) sin( ) ) sin( ) + cos( ) r g

(11)

5.

Use the upper and lower limit on the coefficient of friction to determine an upper and lower limit on the speed necessary to cause sliding. Maximium coefficient Minimium coefficient
max min

:= :=

nom nom

+u u

max min

= 0.51 = 0.35

Maximum velocity to cause sliding

vsmax :=

max cos( ) max sin( )

sin( ) ) + cos( )

r g

vsmax = 18.3 mph

Minimum velocity to cause sliding

vsmin :=

min cos( ) min sin( )

sin( ) ) + cos( )

r g

vsmin = 14.8 mph

6.

This very rough analysis shows that , if the coefficient of friction was at or near the low end of its measured value, the paper load could slide at a tractor/trailer speed of 15 mph, which would lead to the trailer tipping over. In any case, it appears that the paper load would slide before the truck would tip with the load in place.

MACHINE DESIGN - An Integrated Approach, 4th Ed.

3-19-1

PROBLEM 3-19
Statement: Given:
Assume that the CG of the paper rolls in Problem 3-18 is 2.5 ft above the floor of the trailer. At what speed on the same curve will the pile of rolls tip over (not slide) with respect to the trailer?

Weight of paper Radius of curve Paper roll length Height of CG from floor

Wp := 44415 lbf r := 50 ft L := 38 in h := 2.5 ft L = 3.167 ft

Assumptions: The paper rolls act as a single, lumped mass and tip about one corner where they are braced against sliding. The brace provides no moment support. Solution: See Figure 3-19 and Mathcad file P0319.

1. First, calculate the location of the paper's CG with respect to the outside corner when it is on the reverse-banked curve. From Figure 3-19, Tilt angle a := h tan ( ) b := L a xbar := b cos ( ) ybar := b sin ( ) + ybar = 2.662 ft The coordinates of the CG of the paper with respect to the lower outside corner are: xbar = 3.031 ft 2. ybar = 2.662 ft

:= 3 deg
a = 0.131 ft b = 3.036 ft xbar = 3.031 ft h cos ( ) FIGURE 3-19

Fcp 2.500' Wp a Rx b xbar R y 3.167' ybar

FBD of Paper on the Verge of Tipping

The paper is on the verge of tipping over when the couple due to centrifugal force is equal to the couple formed by the weight of the paper acting through its CG and the vertical reaction at the outside edge of the rolls. At this instant, it is assumed that the entire weight of the paper is reacted at the outside corner. Any increase in tangential velocity of the tractor and trailer will result in tipping. Summing moments about the outside corner nearest the floor (see Figure 3-19),

Wp xbar Fcp ybar = 0

(1)

where Fcp is the centrifugal force due to the normal acceleration as the tractor and trailer go through the curve. The normal acceleration is a tip = vtip r
2

(2)

and the force necessary to keep the tractor trailer following a circular path is Fcp = mp a tip where mp is the mass of the paper. Combining equations (2) and (3) and solving for vtip, we have (3)

MACHINE DESIGN - An Integrated Approach, 4th Ed.


vtip = or, vtip = Fcp r g Wp Fcp r mp

3-19-2
(4)

(5)

3.

Calculate the minimum paper tipping velocity of the tractor/trailer. From equations (1) and (5), Centrifugal force required to tip the paper Minimum tipping speed Fcp := xbar ybar Wp Fcp = 50574 lbf

vtip :=

Fcp r g Wp

vtip = 29.2 mph

Thus, with the assumptions that we have made, the paper would not begin to tip over until the tractor/trailor reached a speed of vtip = 29.2 mph

MACHINE DESIGN - An Integrated Approach, 4th Ed.

3-20-1

PROBLEM 3-20
Statement: Assume that the load of paper rolls in Problem 3-18 will slide sideways at a truck speed of 20 mph on the curve in question. Estimate the impact force of the cargo against the trailer wall. The force-deflection characteristic of the trailer wall has been measured as approximately 400 lb/in. Weight of paper Weight of trailer Speed of tractor/trailer Radius of curve Trailer width Paper roll length Trailer wall stiffness Wp := 44415 lbf Wt := 14000 lbf vt := 20 mph r := 50 ft w := 8 ft L := 38 in lbf k := 400 in

Given:

L = 3.167 ft

Assumptions: 1. The paper rolls act as a monolith since they are tightly strapped together with steel bands. 2. The worst case will result if friction between the floor and the paper is neglected. Solution: 1. See Figure P3-8 and Mathcad file P0320.

Calculate the distance that the rolls will slide before impacting the wall. s := 1 2 ( w 2 L) s = 10 in

2.

Determine the centripetal acceleration at 20 mph. a p := vt


2

a p = 206.507

in sec
2

3.

From elementary particle dynamics, estimate the velocity at impact due to the centripetal acceleration vi := 2 a p s vi = 64.266 in sec

4.

With the paper as the moving mass and the trailer as the stationary or struck mass, calculate the correction factor using equation (3.15)

:=
1+

1 Wt 3 Wp

= 0.905

5.

Calculate the static deflection caused by the paper against the trailer wall.

st :=
6.

Wp k

st = 111.037 in

Using equation (3.12), estimate the dynamic force of the paper rolls impacting the trailer wall.

Fi := Wp vi

g st

Fi = 13114 lbf

MACHINE DESIGN - An Integrated Approach, 4th Ed.

3-21-1

PROBLEM 3-21
Statement: Figure P3-9 shows an automobile wheel with two common styles of lug wrench being used to tighten the wheel nuts, a single-ended wrench in (a), and a double-ended wrench in (b). In each case two hands are required to provide forces respectively at A and B as shown. The distance between points A and B is 1 ft in both cases. The wheel nuts require a torque of 70 ft-lb. Draw free body diagrams for both wrenches and determine the magnitudes of all forces and moments on each wrench. Is there any difference between the way these two wrenches perform their assigned task? Is one design better than the other? If so, why? Explain. Distance between A and B Tightening torque d AB := 1 ft T := 70 ft lbf

Given:

Assumptions: 1. The forces exerted by the user's hands lie in a plane through the wrench that is also parallel to the plane of the wheel. 2. The applied torque is perpendicular to the plane of the forces. 3. By virtue of 1 and 2 above, this is a planar problem that can be described in a 2D FBD. Solution: See Figure 3-21 and Mathcad file P0321.
12" = dAB F

1. Summing moments about the left end of the wrench (for either case) T F d AB = 0 2. Solving for F
T

F :=

T d AB

F = 70 lbf

F (a) Single-ended Wrench

3. This result is the same for both wrenches.


12" = dAB

Is there any difference between the way these two wrenches perform their assigned task? No, they both require the same two-handed exertion of 70 lb from each hand. Is one design better than the other? If so, why? Explain. Design (b) has advantages over (a) because it is balanced about the wheel nut. This allows the user to spin the wrench once the nut is loosened. It is also slightly easier to apply the upward and downward forces (F) in a plane with design (b).
F

6"

T (b) Double-ended Wrench

FIGURE 3-21
Free Body Diagrams for Problem 3-21

MACHINE DESIGN - An Integrated Approach, 4th Ed.

3-22-1

PROBLEM 3-22
Statement: A roller-blade skate is shown in Figure P3-10. The polyurethane wheels are 72 mm dia. The skate-boot-foot combination weighs 2 kg. The effective "spring rate" of the person-skate subsystem is 6000 N/m. Find the forces on the wheels' axles for a 100-kg person landing a 0.5-m jump on one foot. (a) Assume all 4 wheels land simultaneously. (b) Assume that one wheel absorbs all the landing force. Mass of struck member Stiffness of struck member Mass of striking member Height of drop Assumptions: Equation (3.14) applies in this case. Solution: 1. See Figure P3-10 and Mathcad file P0322. Msys 2 kg k 6000 N m Mperson 100 kg h 0.5 m

Given:

The weight of the striking mass is Wperson Mperson g Wperson 980.7 N

2.

The static deflection of the subsystem is

st
3.

Wperson k

st 163.444 mm

The correction factor is

1 Msys 3 Mperson

0.993

4.

From equation (3.14), the force of impact is

Fi 1

2 h

st

Wperson

Fi 3.59 kN

(a) If this will be absorbed by 4 wheel axles, the force per axle is Fa Fi 4 Fb Fi Fa 897 N

(b) If one wheel absorbs all force

Fb 3.59 kN

MACHINE DESIGN - An Integrated Approach, 4th Ed.

3-23a-1

PROBLEM 3-23a
Statement: A beam is supported and loaded as shown in Figure P3-11a. Find the reactions, maximum shear, and maximum moment for the data given in row a from Table P3-1. Beam length Distance to distributed load L 1 m a 0.4 m
a b L

Given:

Distance to concentrated load b 0.6 m Distributed load magnitude Concentrated load Solution: w 200 N m F 500 N
R1

R2

See Figures 3-23 and Mathcad file P0323a. FIGURE 3-23A


Free Body Diagram for Problem 3-23

1. From inspection of Figure P3-11a, write the load function equation q(x) = R1<x - 0>-1 - w<x - 0>0 + w<x - a>0 - F<x - b>-1 + R2<x - L>-1 2. Integrate this equation from - to x to obtain shear, V(x) V(x) = R1<x - 0>0 - w<x - 0>1 + w<x - a>1 - F<x - b>0 + R2<x - L>0 3. Integrate this equation from - to x to obtain moment, M(x) M(x) = R1<x - 0>1 - w<x - 0>2/2 + w<x - a>2/2 - F<x - b>1 + R2<x - L>1 4. Solve for the reactions by evaluating the shear and moment equations at a point just to the right of x = L, where both are zero. At x = L+, V = M = 0 V = R1 w ( L) w ( L a ) F R2 = 0 M = R 1 L R1 w 2 L w 2 L
2

w 2

( L a) F ( L b) = 0 w 2 L ( L a)
2

F L

( L b)

R1 264 N R2 316 N

R2 w a F R1 5. Define the range for x x 0 m 0.005 L L

6. For a Mathcad solution, define a step function S. This function will have a value of zero when x is less than z, and a value of one when it is greater than or equal to z. S ( x z) if ( x z 1 0 ) 7. Write the shear and moment equations in Mathcad form, using the function S as a multiplying factor to get the effect of the singularity functions. V ( x) R1 S ( x 0 m) w S ( x 0 m) ( x) w S ( x a ) ( x a ) F S ( x b ) R2 S ( x L) M ( x) R1 S ( x 0 m) x w 2 S ( x 0 m) x
2

w 2

S ( x a ) ( x a ) F S ( x b ) ( x b )

MACHINE DESIGN - An Integrated Approach, 4th Ed.


8. Plot the shear and moment diagrams. Shear Diagram
400 200 V ( x) N 0 200 400

3-23a-2

0.2

0.4 x m

0.6

0.8

Moment Diagram

150

100 M ( x) Nm 50

0.2

0.4 x m

0.6

0.8

FIGURE 3-23aB
Shear and Moment Diagrams for Problem 3-23a

9. Determine the maximum shear and maximum moment from inspection of the diagrams. Maximum shear: Vmax V ( b ) Vmax 316 N

Maximum moment occurs where V is zero, which is x = b: Mmax M ( b ) Mmax 126.4 N m

MACHINE DESIGN - An Integrated Approach, 4th Ed.

3-24a-1

PROBLEM 3-24a
Statement: Given: A beam is supported and loaded as shown in Figure P3-11b. Find the reactions, maximum shear, and maximum moment for the data given in row a from Table P3-1. Beam length Distance to distributed load Distributed load magnitude Concentrated load Solution: L 1 m a 0.4 m w 200 N m F 500 N
M1 R1 a L

1
w

See Figures 3-24 and Mathcad file P0324a.

1. From inspection of Figure P3-11b, write the load function equation

FIGURE 3-24A
Free Body Diagram for Problem 3-24

q(x) = -M1<x - 0>-2 + R1<x - 0>-1 - w<x - a>0 - F<x - L>-1 2. Integrate this equation from - to x to obtain shear, V(x) V(x) = -M1<x - 0>-1 + R1<x - 0>0 - w<x - a>1 - F<x - L>0 3. Integrate this equation from - to x to obtain moment, M(x) M(x) = -M1<x - 0>0 + R1<x - 0>1 - w<x - a>2/2 - F<x - L>1 4. Solve for the reactions by evaluating the shear and moment equations at a point just to the right of x = L, where both are zero. At x = L+, V = M = 0 V = R1 [ w ( L a ) F ] = 0 M = M1 R1 L R1 w ( L a ) F M1 w 2 ( L a ) R1 L x 0 m 0.005 L L
2

w 2

( L a) = 0 R1 620 N M1 584 N m

5. Define the range for x

6. For a Mathcad solution, define a step function S. This function will have a value of zero when x is less than z, and a value of one when it is greater than or equal to z. S ( x z) if ( x z 1 0 ) 7. Write the shear and moment equations in Mathcad form, using the function S as a multiplying factor to get the effect of the singularity functions. V ( x) R1 S ( x 0 m) w S ( x a ) ( x a ) F S ( x L) M ( x) M1 R1 S ( x 0 m) x w 2 S ( x a ) ( x a ) F S ( x L) ( x L)
2

MACHINE DESIGN - An Integrated Approach, 4th Ed.


8. Plot the shear and moment diagrams. Shear Diagram
600 V ( x) N 400 200 0

3-24a-2

0.2

0.4 x m

0.6

0.8

Moment Diagram

150 M ( x) Nm

300

450

600

0.2

0.4 x m

0.6

0.8

FIGURE 3-24aB
Shear and Moment Diagrams for Problem 3-24a

9. Determine the maximum shear and maximum moment from inspection of the diagrams. Maximum shear: Vmax V ( 0 m) Vmax 620 N

Maximum moment occurs where V is zero, which is x = 0: Mmax M ( 0 m) Mmax 584 N m

MACHINE DESIGN - An Integrated Approach, 4th Ed.

3-25a-1

PROBLEM 3-25a
Statement: A beam is supported and loaded as shown in Figure P3-11d. Find the reactions, maximum shear, and maximum moment for the data given in row a from Table P3-1. Beam length Distance to distributed load L 1 m a 0.4 m
a b F w L

Given:

Distance to concentrated load b 0.6 m Distributed load magnitude Concentrated load Solution: w 200 N m F 500 N
R1

R2

See Figures 3-25 and Mathcad file P0325a. FIGURE 3-25A


Free Body Diagram for Problem 3-25

1. From inspection of Figure P3-11c, write the load function equation q(x) = R1<x - 0>-1 - w<x - a>0 + R2<x - b>-1 - F<x - L>-1 2. Integrate this equation from - to x to obtain shear, V(x) V(x) = R1<x - 0>0 - w<x - a>1 + R2<x - b>0 - F<x - L>0 3. Integrate this equation from - to x to obtain moment, M(x) M(x) = R1<x - 0>1 - w<x - a>2/2 + R2<x - b>1 - F<x - L>1 4. Solve for the reactions by evaluating the shear and moment equations at a point just to the right of x = L, where both are zero. At x = L+, V = M = 0 V = R1 w ( L a ) R2 F = 0 M = R 1 L R1 1 w 2 ( L a ) R2 ( L b ) = 0 R1 353 N R2 973 N
2

w 2 ( L a ) F ( L b ) w ( L a ) ( L b ) 2 b

R2 w ( L a ) F R1 5. Define the range for x x 0 m 0.005 L L

6. For a Mathcad solution, define a step function S. This function will have a value of zero when x is less than z, and a value of one when it is greater than or equal to z. S ( x z) if ( x z 1 0 ) 7. Write the shear and moment equations in Mathcad form, using the function S as a multiplying factor to get the effect of the singularity functions. V ( x) R1 S ( x 0 m) w S ( x a ) ( x a ) R2 S ( x b ) F S ( x L) M ( x) R1 S ( x 0 m) x w 2 S ( x a ) ( x a ) R2 S ( x b ) ( x b )
2

MACHINE DESIGN - An Integrated Approach, 4th Ed.


8. Plot the shear and moment diagrams. Shear Diagram
1 0.5 V ( x) kN 0 0.5 1

3-25a-2

0.2

0.4 x m

0.6

0.8

Moment Diagram

75 M ( x) Nm

150

225

300

0.2

0.4 x m

0.6

0.8

FIGURE 3-25aB
Shear and Moment Diagrams for Problem 3-25a

9. Determine the maximum shear and maximum moment from inspection of the diagrams. Maximum shear: Vmax V ( b ) Vmax 580.0 N

Maximum moment occurs where V is zero, which is x = a: Mmax M ( b ) Mmax 216 N m

MACHINE DESIGN - An Integrated Approach, 4th Ed.

3-26a-1

PROBLEM 3-26a
Statement: Given: A beam is supported and loaded as shown in Figure P3-11d. Find the reactions, maximum shear, and maximum moment for the data given in row a from Table P3-1. Beam length Distance to distributed load Distance to reaction load Distributed load magnitude Concentrated load Solution: L 1 m a 0.4 m b 0.6 m w 200 N m F 500 N
R1 R2 L b a

F w

See Figures 3-26 and Mathcad file P0326a.

FIGURE 3-26A
Free Body Diagram for Problem 3-26

1. From inspection of Figure 3-26aA, write the load function equation q(x) = R1<x - 0>-1 - w<x - a>0 + R2<x - b>-1 - F<x - a>-1 2. Integrate this equation from - to x to obtain shear, V(x) V(x) = R1<x - 0>0 - w<x - a>1 + R2<x - b>0 - F<x - a>0 3. Integrate this equation from - to x to obtain moment, M(x) M(x) = R1<x - 0>1 - w<x - a>2/2 + R2<x - b>1 - F<x - a>1 4. Solve for the reactions by evaluating the shear and moment equations at a point just to the right of x = L, where both are zero. At x = L+, V = M = 0 V = R1 w ( L a ) R2 F = 0 M = R 1 L R1 1 w 2 ( L a ) R2 ( L b ) F ( L a ) = 0 R1 147 N R2 473 N
2

w 2 ( L a ) F ( b a ) w ( L a ) ( L b ) 2 b

R2 w ( L a ) F R1 5. Define the range for x x 0 m 0.005 L L

6. For a Mathcad solution, define a step function S. This function will have a value of zero when x is less than z, and a value of one when it is greater than or equal to z. S ( x z) if ( x z 1 0 ) 7. Write the shear and moment equations in Mathcad form, using the function S as a multiplying factor to get the effect of the singularity functions. V ( x) R1 S ( x 0 m) w S ( x a ) ( x a ) R2 S ( x b ) F S ( x a ) M ( x) R1 S ( x 0 m) x w 2 S ( x a ) ( x a ) R2 S ( x b ) ( x b ) F S ( x a ) ( x a )
2

MACHINE DESIGN - An Integrated Approach, 4th Ed.


8. Plot the shear and moment diagrams. Shear Diagram
500 250 V ( x) N 0 250 500

3-26a-2

0.2

0.4 x m

0.6

0.8

Moment Diagram

60

40 M ( x) Nm 20

20

0.2

0.4 x m

0.6

0.8

FIGURE 3-26aB
Shear and Moment Diagrams for Problem 3-26a

9. Determine the maximum shear and maximum moment from inspection of the diagrams. Maximum shear: Vmax V ( b 0.001 mm) Vmax 393 N

Maximum moment occurs where V is zero, which is x = a: Mmax M ( a ) Mmax 58.7 N m

MACHINE DESIGN - An Integrated Approach, 4th Ed.

3-27-1

PROBLEM 3-27
Statement: A storage rack is to be designed to hold the paper roll of Problem 3-8 as shown in Figure P3-12. Determine the reactions and draw the shear and moment diagrams for the mandrel that extends 50% into the roll. Paper roll dimensions OD 1.50 m ID 0.22 m Lroll 3.23 m

Given:

Roll density

984 kg m

Assumptions: 1. The paper roll's weight creates a concentrated load acting at the tip of the mandrel. 2. The mandrel's root in the stanchion experiences a distributed load over its length of engagemen Solution: See Figure 3-27 and Mathcad file P0327.

W
1. Determine the weight of the roll and the length of the mandrel. W

OD ID Lroll g

W 53.9 kN Lm 0.5 Lroll Lm 1.615 m

M1 R1
FIGURE 3-27

Lm

Free Body Diagram for Problem 3-27

2. From inspection of Figure 3-27, write the load function equation q(x) = -M1<x - 0>-2 + R1<x - 0>-1 - W<x - L>-1 3. Integrate this equation from - to x to obtain shear, V(x) V(x) = -M1<x - 0>-1 + R1<x - 0>0 - W<x - L>0 4. Integrate this equation from - to x to obtain moment, M(x) M(x) = -M1<x - 0>0 + R1<x - 0>1 - W<x - L>1 5. Solve for the reactions by evaluating the shear and moment equations at a point just to the right of x = L, where both are zero. At x = L+, V = M = 0 V = R1 W = 0 R1 W M1 R1 Lm 6. Define the range for x M = M1 R1 L = 0 R1 53.895 kN M1 87.040 kN m x 0 m 0.005 Lm Lm

MACHINE DESIGN - An Integrated Approach, 4th Ed.

3-27-2

7. For a Mathcad solution, define a step function S. This function will have a value of zero when x is less than z, and a value of one when it is greater than or equal to z. S ( x z) if ( x z 1 0 ) 8. Write the shear and moment equations in Mathcad form, using the function S as a multiplying factor to get the effect of the singularity functions. V ( x) R1 S ( x 0 m) W S x Lm M ( x) M1 R1 S ( x 0 m) x W S x Lm x Lm 9. Plot the shear and moment diagrams. Shear Diagram
40 V ( x) kN 20

0.5

1 x m

1.5

Moment Diagram

20

1.615

10 M ( x) kN m

40

70

100

0.5

1 x m

1.5

FIGURE 3-27B
Shear and Moment Diagrams for Problem 3-27

MACHINE DESIGN - An Integrated Approach, 4th Ed.

3-28-1

PROBLEM 3-28
Statement: Figure P3-13 shows a forklift truck negotiating a 15 deg ramp to to drive onto a 4-ft-high loading platform. The truck weighs 5 000 lb and has a 42-in wheelbase. Determine the reactions and draw the shear and moment diagrams for the worst case of loading as the truck travels up the ramp. Ramp angle Platform height Truck weight Truck wheelbase

Given:

15 deg
h 4 ft W 5000 lbf Lt 42 in h 48 in

Assumptions: 1. The worst case is when the truck CG is located at the center of the beam's span. 2. Use a coordinate frame that has the x-axis along the long axis of the beam. 3. Ignore traction forces and the weight components along the x-axis of the beam. 4. There are two ramps, one for each side of the forklift. 5. The location of the CG in Figure P3-13 is 32 in from the front wheel and 10 in from the rear wheel. CGa 32 in Solution: See Figure 3-28 and Mathcad file P0328. CGb 10 in

L b a CG a
CG b

R1 Fa Wa Fb Wb R2 x

FIGURE 3-28A
Dimensions and Free Body Diagram for Problem 3-28

1. Determine the length of the beam between supports and the distances a and b. Length of beam L h sin( ) L 2 a b L 2 L 2 CGa CGb L 15.455 ft a 5.061 ft b 8.561 ft

With the CG at midspan, we have

a CGa =

and

MACHINE DESIGN - An Integrated Approach, 4th Ed.

3-28-2

2. The weight distribution on the wheels is determined from the distance from the front wheel to the CG. Each wheel weight is divided by 2 to get the weight on a single ramp. Weight on front wheel Wa Wb CGb W Lt 2 W 2 Wa Wa 595 lbf Wb 1905 lbf

Weight on rear wheel

3. The normal force on the ramp at each wheel is adjusted for the ramp angle. Load at front wheel Load at rear wheel Fa Wa cos( ) Fb Wb cos( ) Fa 575 lbf Fb 1840 lbf

4. From inspection of Figure 3-28A, write the load function equation q(x) = R1<x - 0>-1 - Fa<x - a>-1 - Fb<x - b>-1 + R2<x - L>-1 5. Integrate this equation from - to x to obtain shear, V(x) V(x) = R1<x - 0>0 - Fa<x - a>0 - Fb<x - b>0 + R2<x - L>0 6. Integrate this equation from - to x to obtain moment, M(x) M(x) = R1<x - 0>1 - Fa<x - a>1 - Fb<x - b>1 + R2<x - L>1 7. Solve for the reactions by evaluating the shear and moment equations at a point just to the right of x = L, where both are zero. At x = L+, V = M = 0 V = R1 Fa Fb R2 = 0 M = R1 L Fa ( L a ) Fb ( L b ) = 0 R1 1 L Fa ( L a ) Fb ( L b ) R1 1207 lbf R2 1207 lbf

R2 Fa Fb R1 8. Define the range for x x 0 m 0.005 L L

9. For a Mathcad solution, define a step function S. This function will have a value of zero when x is less than z, and a value of one when it is greater than or equal to z. S ( x z) if ( x z 1 0 ) 10. Write the shear and moment equations in Mathcad form, using the function S as a multiplying factor to get the effect of the singularity functions. V ( x) R1 S ( x 0 m) Fa S ( x a ) Fb S ( x b ) R2 S ( x L) M ( x) R1 S ( x 0 m) x Fa S ( x a ) ( x a ) Fb S ( x b ) ( x b ) R2 S ( x L) ( x L)

MACHINE DESIGN - An Integrated Approach, 4th Ed.


11. Plot the shear and moment diagrams. Shear Diagram
2000 1000 V ( x) lbf 0 1000 2000

3-28-3

8 x ft

10

12

14

16

Moment Diagram

10000 8000 6000 4000 2000 0

15.455

M ( x) ft lbf

8 x ft

10

12

14

16

FIGURE 3-28B
Shear and Moment Diagrams for Problem 3-28

MACHINE DESIGN - An Integrated Approach, 4th Ed.

3-29-1

PROBLEM 3-29
Statement:

_____

Run the TKSolver or Mathcad model for Case Study 1A and move the point of application of the hand force along the lever by changing the values of Rb2, recalculate and observe the changes to the forces and moments. Determine the forces on the elements of the bicycle brake lever assembly shown in Figure 3-1 during braking. The geometry of each element is known. The average human's hand can develop a grip force of about 267 N (60 lb) in the lever position shown. Magnitude of handle force Fb2 Direction of handle force Fb2 Direction of cable force Fc2 Direction of cable force Fcable Fb2 := 267 N

Problem: Given:

b2 := 270 deg c2 := 184 deg cable := 180 deg

Position vector components (Change the value of Rb2x and note the results) Rb2x := 19 mm Rb2y := 4 mm R21x := 7 mm R21y := 19 mm Rc2x := 25 mm Rc2y := 0 mm Rb1x := 47.5 mm Rb1y := 14 mm R12x := 12 mm R12y := 7 mm R31x := 27 mm R31y := 30 mm

Assumptions: The accelerations are negligible. All forces are coplanar and two-dimensional. A class 1 load model is appropriate and a static analysis is acceptable. The higher applied load will be used as a worst case, assuming that it can be reached before bottoming the tip of the handle on the handgrip. If that occurs, it will change the beam's boundary conditions and the analysis. Solution: 1. See Figures 3-1, 3-2, and Mathcad file P0329.

Figure 3-1 shows the hand brake lever assembly, which consists of three subassemblies: the handlebar (1), the lever (2), and the cable (3). The lever is pivoted to the handlebar and the cable is connected to the lever. The cable runs within a plastic-lined sheath (for low friction) down to the brake caliper assembly at the bicycle's wheel rim. The user's hand applies equal and opposite forces at some point on the lever and handgrip. These forces are transformed to a larger force in the cable by reason of the lever ratio of part 2. Figure 3-1 is a free-body diagram of the entire assembly since it shows all the forces and moments acting on it except for its weight, which is small compared to the applied forces and is thus neglected for this analysis. The "broken away" portion of the handlebar provides internal x and y force components and a moment. These are arbitrarily shown as positive in sign. Their actual signs will "come out in the wash" in the calculations. The known applied forces are shown in their actual directions and senses.

2.

Figure 3-2 shows the three subassembly elements separated and drawn as free-body diagrams with all relevant forces and moments applied to each element, again neglecting the weights of the parts. The lever (part 2) has three forces on it, Fb2, Fc2, and F12. The two-character subscript notation used here should be read as, force of element 1 on 2 (F12) or force at B on 2 (Fb2), etc. This defines the source of the forces (first subscript) and the element on which it acts (second subscript). This notation will be used consistently throughout this text for both forces and position vectors such as Rb2, Rc2, and R12 in Figure 3-2, which serve to locate the above three forces in a local, non rotating coordinate system whose origin is at the center of gravity (CG) of the element or subassembly being analyzed. (See foot note on page 83 of the text). On this brake lever, Fb2 is an applied force whose magnitude and direction are known. Fc2 is the force in the

MACHINE DESIGN - An Integrated Approach, 4th Ed.

3-29-2

cable. Its direction is known but not its magnitude. Force F12 is provided by part 1 on part 2 at the pivot pin. Its magnitude and direction are both unknown. We can write equations 3.3b for this element to sum forces in the x and y directions and sum moments about the CG. Note that all unknown reactive forces and moments are initially assumed positive in the equations. Their true signs will come out in the calculation. (See foot note on page 84 of the text).

Fx = Fb2x + Fc2x + F12x = 0 Fy = Fb2y + Fc2y + F12y = 0 Mz = ( R12 F12) + ( Rb2 Fb2) + ( Rc2 Fc2 ) = 0
The cross products in the moment equation represent the "turning forces" or moments created by the application of these forces at points remote from the CG of the element. Recall that these cross products can be expanded to (a)

Mz = ( R12x F12y R12y F12x) ... = 0

+ ( Rb2x Fb2y Rb2y Fb2x) ... + ( R F R F ) c2x c2y c2y c2x

(b)

We have three equations and four unknowns (F12x, F12y, Fc2x, Fc2y) at this point, so we need another equation. It is available from the fact that the direction of Fc2 is known. (The cable can pull only along its axis). We can express one component of the cable force Fc2 in terms of its other component and the known angle c2 of the cable. (c) Fc2y = Fc2x tan( c2 ) We will now use a Mathcad solve block to solve equations a through c. Calculate components of Fb2 Fb2x := Fb2 cos( b2) Fb2y := Fb2 sin( b2) Guess Given F12x := 1000 N Fb2x + Fc2x + F12x = 0 Fb2y + Fc2y + F12y = 0 Fb2x = 0 N Fb2y = 267 N Fc2x := 1000 N F12y := 1000 N Fc2y := 1000 N

( R12x F12y R12y F12x) ... = 0 + ( Rb2x Fb2y Rb2y Fb2x) ... + ( R F R F ) c2x c2y c2y c2x
Fc2y = Fc2x tan( c2 )

MACHINE DESIGN - An Integrated Approach, 4th Ed.

3-29-3

F12x F 12y := Find ( F , F , F , F ) 12x 12y c2x c2y Fc2x Fc2y


Components of the unknown forces F12, and Fc2 F12x = 1047 N 3. Fc2x = 1047 N F12y = 340 N Fc2y = 73.2 N

Part 3 in Figure 3-2 is the cable that passes through a hole in part 1. This hole is lined with a low friction material, which allows us to assume no friction at the joint between parts 1 and 3. We will further assume that the three forces F13, Fc3, and Fcable form a concurrent system of forces acting through the CG and thus create no moment. With this assumption, only a summation of forces is necessary for this element.

Fx = Fcablex + F13x + Fc3x = 0


(d)

Fy = Fcabley + F13y + Fc3y = 0


Using Newton's third law, we have Fc3x := Fc2x and Fc3y := Fc2y. We also assume that the cable entering from the left is horizontal and that the reaction F13 is vertical, thus Fcabley := 0 N and F13x := 0 N (e)

We can now solve for the forces on part 3 directly, Fcablex := F13x Fc3x F13y := Fcabley Fc3y Fcablex = 1047 N F13y = 73.2 N

The assembly of elements labeled part 1 in Figure 3-2 has both force and moments on it (i.e., it is not a concurrent system), so the three equations 3.3b are needed.

Fx = F21x + Fb1x + F31x + Px + Fsheathx = 0 Fy = F21y + Fb1y + F31y + Py = 0 Mz = Mh + ( R21 F21) + ( Rb1 Fb1) + ( R31 F31) ... = 0
+ ( Rp Fp) + ( Rd Fsheath ) Expanding cross products in the moment equation gives the moment magnitude as (f)

Mz = Mh + ( R21x F21y R21y F21x) ... = 0

+ ( Rb1x Fb1y Rb1y Fb1x) ... + ( R31x F31y R31y F31x) ... + ( R F R F ) ... Px Py Py Px + ( 0 Rdy Fsheathx)

(g)

Using Newton's third law, we have F31x := F13x F31y := F13y F21x := F12x F21y := F12y Fb1x := Fb2x (h) Fb1y := Fb2y

MACHINE DESIGN - An Integrated Approach, 4th Ed.


Fsheathx := Fcablex

3-29-4

Given

RPx := 27 mm Rdx := 41 mm

RPy := 0 mm Rdy := 27 mm

We will now use a Mathcad solve block to solve equations (f) through (h). Guess Given Px := 1000 N Mh := 100 N m Py := 0 N

F21x + Fb1x + F31x + Px + Fsheathx = 0 F21y + Fb1y + F31y + Py = 0

Mh + ( R21x F21y R21y F21x) ... = 0 + ( Rb1x Fb1y Rb1y Fb1x) ... + ( R31x F31y R31y F31x) ... + ( R P R P ) ... Px y Py x + ( 0 N m Rdy Fsheathx)

Px Py := Find( Px , Py , Mh) M h
Summarizing, the results obtained for a grip force Fb2 = 267 N are: Handlebar (1) Fb1x = 0 N F21x = 1047 N F31x = 0 N Px = 1 10
6

Fb1y = 267 N F21y = 340 N F31y = 73.2 N N Py = 0 N

Mh = 0.0 N m Lever (2) Fc2x = 1047 N F12x = 1047 N Cable (3) Fc3x = 1047 N F13x = 0 N Fcablex = 1047 N Fc2y = 73.2 N F12y = 340 N Fc3y = 73.2 N F13y = 73.2 N Fcabley = 0 N

MACHINE DESIGN - An Integrated Approach, 4th Ed.

3-30-1

PROBLEM 3-30
Statement:

_____

Run the TKSolver or Mathcad model for Case Study 2A and move the point of application of the crimp force along the jaw by changing the values of Rhand, recalculate and observe the changes to the forces and moments. Determine the forces on the elements of the crimping tool shown in Figure 3-3 during a crimp operation. The geometry is known and the tool develops a crimp force of 2000 lb (8896 N) at closure in the position shown. Applied crimp force Fc4x := 1956.30 lbf Fc4y := 415.82 lbf

Problem: Given:

Position vector components (Change the value of Rhand and note the results) Rc4x := 0.454 in Rc4y := 0.337 in R23x := 0.602 in R23y := 0.127 in R34x := 0.161 in R12x := 1.399 in R12y := 0.049 in R43x := 0.602 in R43y := 0.127 in R34y := 0.758 in R32x := 2.199 in R32y := 0.077 in R14x := 0.161 in R14y := 0.758 in Rhand := 4.40 in

Assumptions: The accelerations are negligible. All forces are coplanar and two-dimensional. A class 1 load model is appropriate and a static analysis is acceptable. Solution: 1. See Figures 3-3, 3-4, and Mathcad file P0330.

Figure 3-3 shows the tool in the closed position, in the process of crimping a metal connector onto a wire. The user's hand provides the input forces between links 1 and 2, shown as the reaction pair Fhand. The user can grip the handle anywhere along its length but we are assuming a nominal moment arm of Rhand for the application of the resultant of the user's grip force (see Figure 3-4). The high mechanical advantage of the tool transforms the grip force to a large force at the crimp. Figure 3-3 is a free-body diagram of the entire assembly, neglecting the weight of the tool, which is small compared to the crimp force. There are four elements, or links, in the assembly, all pinned together. Link 1 can be considered to be the "ground" link, with the other links moving with respect to it as the jaw is closed. The desired magnitude of the crimp force Fc is defined and its direction will be normal to the surfaces at the crimp.

2.

Figure 3-4 shows the elements of the crimping tool assembly separated and drawn as free-body diagrams with all forces applied to each element, again neglecting their weights as being insignificant compared to the applied forces. The centers of gravity of the respective elements are used as the origins of the local, non rotating coordinate systems in which the points of application of all forces on the element are located. (See footnote on page 116 of the text). We will consider link 1 to be the ground plane and analyze the remaining moving links. Note that all unknown forces and moments are initially assumed positive. Link 4 has three forces acting on it: Fc4 is the known (desired) force at the crimp, and F14 and F34 are the reaction forces from links 1 and 3, respectively. The magnitudes of these two forces are unknown as is the direction of F14. The direction of F34 will be the same as link 3, since it is a two-force member. Writing equations 3.3b for this element:

3.

Fx = F14x + F34x + Fc4x = 0 Fy = F14y + F34y + Fc4y = 0 Mz = ( R14x F14y R14y F14x) ... = 0
(a)

+ ( R34x F34y R34y F34x) ... + ( R F R F ) c4x c4y c4y c4x

MACHINE DESIGN - An Integrated Approach, 4th Ed.

3-30-2

We have three equations and four unknowns (F14x, F14y, F34x, F34y) at this point, so we need another equation. It is available from the fact that the direction of F34 is known. We can express one component of the force F34 in terms of its other component and the known angle 3 of link 3. (b) F34y = F34x tan( 3) where Guess Given

3 := 168 deg
F14x := 500 lbf F14x + F34x + Fc4x = 0 F34x := 1000 lbf F14y := 100 lbf F14y + F34y + Fc4y = 0 F34y := 100 lbf

(c)

( Rc4x Fc4y Rc4y Fc4x) ... = 0 + ( R14x F14y R14y F14x) ... + ( R F R F ) 34x 34y 34y 34x
F34y = F34x tan( 3)

F14x F 14y := Find ( F , F , F , F ) 14x 14y 34x 34y F34x F34y


Components of the unknown forces F14, and F34 F14x = 442.9 lbf 4. F14y = 94.1 lbf F34x = 1513.4 lbf F34y = 321.7 lbf

Link 3 has two forces on it, F23 and F43. Because this is a two-force link, these two forces are equal in magnitude and opposite in direction. Also, from Newton's third law, F43 = - F34. Thus, F43x := F34x F43x = 1513.4 lbf F43y := F34y F43y = 321.7 lbf F23x := F43x F23x = 1513.4 lbf F23y := F43y F23y = 321.7 lbf (d)

5.

Link 2 has three forces acting on it: Fhand is the unknown force from the hand, and F12 and F32 are the reaction forces from links 1 and 3, respectively. Force F12 is provided by part 1 on part 2 at the pivot pin and force F32 is provided by part 3 acting on part 2 at their pivot pin. The magnitude and direction of F32 is known and the direction of Fhand is known. Using equations 3.3b, we can solve for the magnitude of Fhand and the two components of F12. From the third law, F32x := F23x F32y := F23y F32x = 1513.4 lbf F32y = 321.7 lbf

Fx = F12x + F32x = 0 Fy = Fhand + F12y + F32y = 0 Mz = ( R12 F12) + ( R32 F32) ... = 0
+ ( Rhand Fhand ) Guess F12x := 1500 lbf F12y := 100 lbf Fhand := 100 lbf (e)

MACHINE DESIGN - An Integrated Approach, 4th Ed.


Given F12x + F32x = 0 F12y + F32y + Fhand = 0

3-30-3

( R12x F12y R12y F12x) ... = 0 + ( R32x F32y R32y F32x) ... + R F hand hand F12x F12y := Find( F12x , F12y , Fhand ) F hand
F12x = 1513.4 lbf 6. F12y = 373.4 lbf Fhand = 51.7 lbf

The four forces on link 1 can now be determined using the third law. F21x := F12x F21x = 1513.4 lbf Fc1x := Fc4x F21y := F12y F21y = 373.4 lbf Fc1y := Fc4y F41x := F14x F41x = 442.9 lbf Fc1x = 1956.3 lbf F41y := F14y F41y = 94.1 lbf Fc1y = 415.8 lbf

7.

The solution to this problem for the scaled dimensions in Figure 3-3 assuming a 2000-lb (8896-N) force applied at the crimp, normal to the crimp surface, is given above. The total forces at the pivot points are: Pivot A Pivot B Pivot C Pivot D F12 := F12x + F12y
2 2 2 2 2 0.5

0.5

F12 = 1559 lbf F32 = 1547 lbf


0.5

F32 := F32x + F32y F43 := F43x + F43y F14 := F14x + F14y


0.5

2 2

F43 = 1547 lbf F14 = 453 lbf

The moment that must be applied to the handles to generate the crimp force of Crimp force Moment Fc4 := Fc4x + Fc4y
2 2 0.5

Fc4 = 2000 lbf Mh = 227 lbf in

Mh := Rhand Fhand

This moment can be obtained with a force of Fhand = 52 lbf applied at mid-handle. This force is within the physiological grip-force capacity of the average human.

MACHINE DESIGN - An Integrated Approach, 4th Ed.

3-31-1

PROBLEM 3-31
Statement:

_____

Run the TKSolver or Mathcad model for Case Study 2A and move the point of application of P along the x direction by changing the values of Rpx, recalculate and observe the changes to the forces and moments. What happens when the vertical force P is centered on link 3? Also, change the angle of the applied force P to create an x component and observe the effects on the forces and moments on the elements. Determine the forces on the elements of the scissors-jack in the position shown in Figure 3-5. The geometry is known and the jack supports a force of 1000 lb (4448 N) in the position shown. Support force Px := 0.0 lbf Py := 1000 lbf

Problem: Given:

Position vector components (Change the value of Rpx and note the results) Rpx := 0.50 in Rpy := 0.87 in R42x := 2.71 in R42y := 1.00 in R14x := 3.12 in R14y := 1.80 in R12x := 3.12 in R12y := 1.80 in R23x := 0.78 in R23y := 0.78 in R24x := 2.58 in R24y := 1.04 in R32x := 2.08 in R32y := 1.20 in R43x := 0.78 in R43y := 0.78 in R34x := 2.08 in R34y := 1.20 in

Angle of gear teeth common normal

:= 45.0 deg

Assumptions: The accelerations are negligible. The jack is on level ground. The angle of the elevated car chassis does not impart an overturning moment to the jack. All forces are coplanar and two-dimensional. A class 1 load model is appropriate and a static analysis is acceptable. Solution: 1. See Figures 3-5 through 3-8, and Mathcad file P0331.

Figure 3-5 shows a schematic of a simple scissors jack used to raise a car. It consists of six links that are pivoted and/or geared together and a seventh link in the form of a lead screw that is turned to raise the jack. While this is clearly a three-dimensional device, it can be analyzed as a two-dimensional one if we assume that the applied load (from the car) and the jack are exactly vertical (in the z direction). If so, all forces will be in the xy plane. This assumption is valid if the car is jacked from a level surface. If not, then there will be some forces in the yz and xz planes as well. The jack designer needs to consider the more general case, but for our simple example we will initially assume two-dimensional loading. For the overall assembly as shown in Figure 3-5, we can solve for the reaction force Fg, given force P, by summing forces: Fg = -P. Figure 3-6 shows a set of free-body diagrams for the entire jack. Each element or subassembly of interest has been separated from the others and the forces and moments shown acting on it (except for its weight, which is small compared to the applied forces and is thus neglected for this analysis). The forces and moments can be either internal reactions at interconnections with other elements or external loads from the "outside world." The centers of gravity of the respective elements are used as the origins of the local, non rotating coordinate systems in which the points of application of all forces on the element are located. In this design, stability is achieved by the mating of two pairs of crude (non involute) gear segments acting between links 2 and 4 and between links 5 and 7. These interactions are modeled as forces acting along a common normal shared by the two teeth. This common normal is perpendicular to the common tangent at the contact point. There are 3 second-law equations available for each of the seven elements allowing 21 unknowns. An additional 10 third-law equations will be needed for a total of 31. This is a cumbersome system to solve for such a simple device, but we can use its symmetry to advantage in order to simplify the problem.

2.

3.

Figure 3-7 shows the upper half of the jack assembly. Because of the mirror symmetry between the upper and lower portions, the lower half can be removed to simplify the analysis. The forces calculated for this half will be duplicated in the other. If we wished, we could solve for the reaction forces at A and B using equations 3.3b from this free-body diagram of the half-jack assembly.

MACHINE DESIGN - An Integrated Approach, 4th Ed.


4.

3-31-2

Figure 3-8a shows the free-body diagrams for the upper half of the jack assembly, which are essentially the same as those of Figure 3-6. We now have four elements but can consider the subassembly labeled 1 to be the "ground," leaving three elements on which to apply equations 3.3. Note that all forces and moments are initially assumed positive in the equations. Link 2 has three forces acting on it: F42 is the unknown force at the gear tooth contact with link 4; F12 and F32 are the unknown reaction forces from links 1 and 3, respectively. Force F12 is provided by part 1 on part 2 at the pivot pin and force F32 is provided by part 3 acting on part 2 at their pivot pin. The magnitudes and the directions of these pin forces and the magnitude of F42 are unknown. The direction of F42 is along the common normal shown in Figure 3-8b. Write equations 3.3b for this element to sum the forces in the x and y directions and sum moments about the CG (with the cross products expanded). Fx = F12x + F32x + F42x = 0

5.

Fy = F12y + F32y + F42y = 0 Mz = ( R12x F12y R12y F12x) ... = 0

(a)

+ ( R32x F32y R32y F32x) ... + ( R F R F ) 42x 42y 42y 42x

6.

Link 3 has three forces acting on it: P, F23 and F43. Only P is known. Writing equations 3.3b for this element gives

Fx = F23x + F43x + Px = 0 Fy = F23y + F43y + Py = 0 Mz = ( R23x F23y R23y F23x) ... = 0


(b)

+ ( R43x F43y R43y F43x) ... + ( R P R P ) px y py x

7.

Link 4 has three forces acting on it: F24 is the unknown force from link 2; F14 and F34 are the unknown reaction forces from links 1 and 3, respectively.

Fx = F14x + F24x + F34x = 0 Fy = F14y + F24y + F34y = 0 Mz = ( R14x F14y R14y F14x) ... = 0
(c)

+ ( R24x F24y R24y F24x) ... + ( R F R F ) 34x 34y 34y 34x

8.

The nine equations in sets a through c have 16 unknowns in them, F12x, F12y, F32x, F32y, F23x, F23y, F43x, F43y, F14x , F14y, F34x, F34y, F24x, F24y, F42x, F42y. We can write the third-law relationships between action-reaction pairs at each of the joints to obtain six of the seven additional equations needed: F32x = F23x F34x = F43x F42x = F24x F32y = F23y F34y = F43y F42y = F24y (d)

9.

The last equation needed comes from the relationship between the x and y components of the force F24 (or F42) at the tooth/tooth contact point. Such a contact (or half) joint can transmit force (excepting friction force) only along the common normal , which is perpendicular to the joint's common tangent as shown in Figure 3-8b. The common normal is also called the axis of transmission.

MACHINE DESIGN - An Integrated Approach, 4th Ed.


The tangent of the angle of this common normal relates the two components of the force at the joint: F24y = F24x tan( ) 10. Equations (d) and (e) will be substituted into equations (a) through (c) to create a set of nine simultaneous equations for solution. Guess F12x := 500 lbf F23x := 500 lbf F43x := 500 lbf Given F12y := 500 lbf F23y := 500 lbf F43y := 500 lbf F14x := 500 lbf F24x := 500 lbf F14y := 500 lbf

3-31-3

(e)

( R12x F12y R12y F12x) ... + ( R32x F23y + R32y F23x) ... + ( R F tan( ) + R F ) 42x 24x 42y 24x ( R23x F23y R23y F23x) ... = 0 + ( R43x F43y R43y F43x) ... + ( R P R P ) px y py x

=0

F12x F23x F24x = 0 F12y F23y F24x tan( ) = 0

F23x + F43x + Px = 0 F23y + F43y + Py = 0

( R14x F14y R14y F14x) ... =0 + ( R24x F24x tan( ) R24y F24x) ... + ( R F + R F ) 34x 43y 34y 43x

F14x + F24x F43x = 0 F14y + F24x tan( ) F43y = 0

F12x F12y F14x F14y F23x := Find ( F , F , F , F , F , F , F , F , F ) 12x 12y 14x 14y 23x 23y 24x 43x 43y F23y F 24x F43x F43y
Results: F14x = 877.8 lbf F24x = 290.1 lbf F34x := F43x F23x = 587.7 lbf F12x = 877.8 lbf F42x := F24x F14y = 469.6 lbf F24y := F24x tan( ) F34y := F43y F23y = 820.5 lbf F12y = 530.4 lbf F42y := F24y F43x = 587.7 lbf F32x := F23x F43y = 179.5 lbf F32y := F23y F24y = 290.1 lbf

MACHINE DESIGN - An Integrated Approach, 4th Ed.

3-32-1

PROBLEM 3-32
Statement:

_____

Figure P3-14 shows a cam-follower arm. If the load P = 200 lb, what spring force is needed at the right end to maintain a minimum load between cam and follower of 25 lb? Find the maximum shear force and bending moment in the follower arm. Plot the shear and moment diagrams. Load at left end of beam Load at cam follower P := 200 lbf Pcam := 25 lbf a := 10 in b := 22 in c := 29 in

Given:

Distance from left end to: Pivot point Cam follower Spring Solution: 1. See Figure P3-14 and Mathcad file P0332.

Draw a FBD of the cam-follower arm (beam).


c b

Pcam

Fspring

2.

From inspection of the FBD, write the load function equation q(x) = -P<x - 0>-1 + R<x - a>-1 + Pcam<x - b >-1 - Fspring<x - 0>-1

3.

Integrate this equation from - to x to obtain shear, V(x) V(x) = -P<x - 0>0 + R<x - a>0 + Pcam<x - b >0 - Fspring<x - 0>0

4.

Integrate this equation from - to x to obtain moment, M(x) M(x) = -P<x - 0>1 + R<x - a>1 + Pcam<x - b >1 - Fspring<x - 0>1

5.

Solve for the reactions by evaluating the shear and moment equations at a point just to the right of x = c, where both are zero. At x = c+, V = M = 0 V = P + R + Pcam Fspring = 0 M = P c + R ( c a ) + Pcam ( c b ) = 0 P a + Pcam ( b a ) ca

Fspring :=

Fspring = 121.05 lbf R = 296.05 lbf

R := Fspring + P Pcam 6. Define the range for x x := 0 in , 0.002 c .. c

MACHINE DESIGN - An Integrated Approach, 4th Ed.


7.

3-32-2

For a Mathcad solution, define a step function S. This function will have a value of zero when x is less than z, and a value of one when it is greater than or equal to z. S ( x , z) := if ( x z , 1 , 0 )

8.

Write the shear and moment equations in Mathcad form, using the function S as a multiplying factor to get the effect of the singularity functions. V ( x) := P S ( x , 0 in) + R S ( x , a ) + Pcam S ( x , b ) Fspring S ( x , c) M ( x) := P S ( x , 0 in) x + R S ( x , a ) ( x a) + Pcam S ( x , b ) ( x b ) Fspring S ( x , c) ( x c)

9.

Plot the shear and moment diagrams and find the maximum shear force and bending moment.

SHEAR DIAGRAM
200 100 0 100 200 300

V ( x) lbf

10 x in

20

30

Vmax := V ( 0 in)

Vmax = 200 lbf

MOMENT DIAGRAM
0

500 M ( x) in lbf 1000

1500

2000

10 x in

20

30

Mmax := M ( a )

Mmax = 2000 in lbf

MACHINE DESIGN - An Integrated Approach, 4th Ed.

3-33-1

PROBLEM 3-33
Statement:

_____

Write a computer program or equation solver model to calculate all the singularity functions listed in equations 3.17. Set them up as functions that can be called from within any other program or model. See Mathcad file P0333.

Solution: 1.

No solution is provided for this programming problem.

MACHINE DESIGN - An Integrated Approach, 4th Ed.

3-34a-1

PROBLEM 3-34a
Statement: A beam is supported and loaded as shown in Figure P3-15. Find the reactions, maximum shear, and maximum moment for the data given in row a from Table P3-2. Beam length Distance to RH bearing Distance to concentrated load Concentrated load L := 20 in a := 16 in b := 18 in P := 1000 lbf
R1
a

Given:

R2

FIGURE 3-34aA Solution: 1. See Figure 3-34 and Mathcad file P0334a.
Free Body Diagram for Problem 3-34

From inspection of Figure 3-34, write the load function equation q(x) = R1<x - 0>-1 + R2<x - b>-1 - P<x - L>-1

2.

Integrate this equation from - to x to obtain shear, V(x) V(x) = R1<x - 0>0 + R2<x - b>0 - P<x - L>0

3.

Integrate this equation from - to x to obtain moment, M(x) M(x) = R1<x - 0>1 + R2<x - b>1 - P<x - L>1

4.

Solve for the reactions by evaluating the shear and moment equations at a point just to the right of x = b, where both are zero. At x = b +, V = M = 0 V = R1 + R2 P = 0 M = R1 b + R2 ( b a ) = 0 R1 := P a (a b) R1 = 125 lbf R2 = 1125 lbf

R2 := P R1 5. 6. Define the range for x x := 0 m , 0.002 L .. L

For a Mathcad solution, define a step function S. This function will have a value of zero when x is less than z, and a value of one when it is greater than or equal to z. S ( x , z) := if ( x z , 1 , 0 )

7.

Write the shear and moment equations in Mathcad form, using the function S as a multiplying factor to get the effect of the singularity functions. V ( x) := R1 S ( x , 0 in) + R2 S ( x , a ) P S ( x , b ) M ( x) := R1 S ( x , 0 in) x + R2 S ( x , a ) ( x a ) P S ( x , b ) ( x b)

8.

Plot the shear and moment diagrams.

MACHINE DESIGN - An Integrated Approach, 4th Ed.

3-34a-2

SHEAR DIAGRAM
1000 1000

MOMENT DIAGRAM

0 500 V ( x) lbf 0 2000 M ( x) in lbf 1000

500

10 x in

15

20

3000

10 x in

15

20

FIGURE 3-34aB
Shear and Moment Diagrams for Problem 3-34a

9.

Determine the maximum shear and maximum moment from inspection of the diagrams. Maximum shear: Vmax := V ( a ) Vmax = 1000 lbf

Maximum moment occurs where V is zero, which is x = a: Mmax := M ( a ) Mmax = 2000 in lbf

MACHINE DESIGN - An Integrated Approach, 4th Ed.

3-35a-1

PROBLEM 3-35a
Statement: A beam is supported and loaded as shown in Figure P3-15. Write a computer program or equation solver model to find the reactions and calculate and plot the loading, shear, and moment functions. Test the program with the data given in row a from Table P3-2. Enter data in highlighted areas Beam length Distance to RH bearing Distance to concentrated load Concentrated load Solution: 1. L := 20 in a := 16 in b := 18 in F := 1000 lbf FIGURE 3-34aA See Figures 3-35 and Mathcad file P0335a.
Free Body Diagram for Problem 3-34
R1
a

Input data:

R2

From inspection of Figure 3-35, write the load function equation q(x) = R1<x - 0>-1 + R2<x - b>-1 - F<x - L>-1

2.

Integrate this equation from - to x to obtain shear, V(x) V(x) = R1<x - 0>0 + R2<x - b>0 - F<x - L>0

3.

Integrate this equation from - to x to obtain moment, M(x) M(x) = R1<x - 0>1 + R2<x - b>1 - F<x - L>1

4.

Solve for the reactions by evaluating the shear and moment equations at a point just to the right of x = b, where both are zero. At x = b +, V = M = 0 V = R1 + R2 F = 0 M = R1 b + R2 ( b a ) = 0 R1 := F a ( a b) R1 = 125 lbf R2 = 1125 lbf

R2 := F R1 5. 6. Define the range for x x := 0 m , 0.002 L .. L

For a Mathcad solution, define a step function S. This function will have a value of zero when x is less than z, and a value of one when it is greater than or equal to z. S ( x , z) := if ( x z , 1 , 0 )

7.

Write the shear and moment equations in Mathcad form, using the function S as a multiplying factor to get the effect of the singularity functions. V ( x) := R1 S ( x , 0 in) + R2 S ( x , a ) F S ( x , b ) M ( x) := R1 S ( x , 0 in) x + R2 S ( x , a ) ( x a ) F S ( x , b ) ( x b )

8.

Plot the shear and moment diagrams.

MACHINE DESIGN - An Integrated Approach, 4th Ed.

3-35a-2

SHEAR DIAGRAM
1000 1000

MOMENT DIAGRAM

0 500 V ( x) lbf 0 2000 M ( x) in lbf 1000

500

10 x in

15

20

3000

10 x in

15

20

FIGURE 3-34aB
Shear and Moment Diagrams for Problem 3-35a

9.

Determine the maximum shear and maximum moment from inspection of the diagrams. Maximum shear: Vmax := V ( a ) Vmax = 1000 lbf

Maximum moment occurs where V is zero, which is x = a: Mmax := M ( a ) Mmax = 2000 in lbf

MACHINE DESIGN - An Integrated Approach, 4th Ed.

3-36a-1

PROBLEM 3-36a
Statement: Given: A beam is supported and loaded as shown in Figure P3-16. Find the reactions, maximum shear, and maximum moment for the data given in row a from Table P3-2. Beam length Distance to RH bearing Distance to start of load Distance to end of load Distributed load Solution: 1. L := 20 in L := 20 in a := 16 in b := 18 in p := 1000 lbf in
R1 L R2 b a p

FIGURE 3-36aA
Free Body Diagram for Problem 3-36

See Figures 3-36 and Mathcad file P0336a.

From inspection of Figure 3-36, write the load function equation q(x) = R1<x - 0>-1 - p<x - a>0 + p<x - b>0 + R2<x - L>-1

2.

Integrate this equation from - to x to obtain shear, V(x) V(x) = R1<x - 0>0 - p<x - a>1 + p<x - b>1 + R2<x - L>0

3.

Integrate this equation from - to x to obtain moment, M(x) M(x) = R1<x - 0>1 - p<x - a>2/2 + p<x - b>2/2 + R2<x - L>1

4.

Solve for the reactions by evaluating the shear and moment equations at a point just to the right of x = L, where both are zero. At x = L+, V = M = 0 V = R1 p ( L a ) + p ( L b ) + R2 = 0 M = R1 L p 2 L p 2 (L a) +
2

p 2
2

( L b ) + R2 ( L b) = 0
2

R1 :=

2 ( b a ) L + a b

R1 = 300 lbf R2 = 1700 lbf

R2 := p ( b a ) R1 5. 6. Define the range for x x := 0 in , 0.002 L .. L

For a Mathcad solution, define a step function S. This function will have a value of zero when x is less than z, and a value of one when it is greater than or equal to z. S ( x , z) := if ( x z , 1 , 0 )

7.

Write the shear and moment equations in Mathcad form, using the function S as a multiplying factor to get the effect of the singularity functions. V ( x) := R1 S ( x , 0 in) p S ( x , a ) ( x a) + p S ( x , b ) ( x b ) + R2 S ( x , L) M ( x) := R1 S ( x , 0 in) x p 2 S(x , a) ( x a) +
2

p 2

S ( x , b ) ( x b ) + R2 S ( x , L) ( x L)

MACHINE DESIGN - An Integrated Approach, 4th Ed.


8. Plot the shear and moment diagrams.

3-36a-2

SHEAR DIAGRAM
1000 5000

MOMENT DIAGRAM

4000 0 V ( x) lbf 1000 1000 M ( x) in lbf 2000 3000

2000

10 x in

15

20

10 x in

15

20

FIGURE 3-36aB
Shear and Moment Diagrams for Problem 3-36a

9.

Determine the maximum shear and maximum moment from inspection of the diagrams. Maximum shear: Vmax := V ( b ) Vmax = 1700 lbf

Maximum moment occurs where V is zero, which is x = c, where: R1 b + R2 a R1 + R2

c :=

c = 16.3 in

Mmax := M ( c)

Mmax = 4845 in lbf

MACHINE DESIGN - An Integrated Approach, 4th Ed.

3-37a-1

PROBLEM 3-37a
Statement: A beam is supported and loaded as shown in Figure P3-16. Write a computer program or equation solver model to find the reactions and calculate and plot the loading, shear, and moment functions. Test the program with the data given in row a from Table P3-2. Enter data in highlighted areas Beam length Distance to RH bearing Distance to start of load Distance to end of load Distributed load Solution: 1. L := 20 in L := 20 in a := 16 in b := 18 in p := 1000 lbf in
R1 L R2 b a p

Input data:

FIGURE 3-37aA
Free Body Diagram for Problem 3-37

See Figures 3-37 and Mathcad file P0337a.

From inspection of Figure 3-37, write the load function equation q(x) = R1<x - 0>-1 - p<x - a>0 + p<x - b>0 + R2<x - L>-1

2.

Integrate this equation from - to x to obtain shear, V(x) V(x) = R1<x - 0>0 - p<x - a>1 + p<x - b>1 + R2<x - L>0

3.

Integrate this equation from - to x to obtain moment, M(x) M(x) = R1<x - 0>1 - p<x - a>2/2 + p<x - b>2/2 + R2<x - L>1

4.

Solve for the reactions by evaluating the shear and moment equations at a point just to the right of x = L, where both are zero. At x = L+, V = M = 0 V = R1 p ( L a ) + p ( L b ) + R2 = 0 M = R1 L p 2 L p 2 (L a) +
2

p 2
2

( L b ) + R2 ( L b) = 0
2

R1 :=

2 ( b a ) L + a b

R1 = 300 lbf R2 = 1700 lbf

R2 := p ( b a ) R1 5. 6. Define the range for x x := 0 in , 0.002 L .. L

For a Mathcad solution, define a step function S. This function will have a value of zero when x is less than z, and a value of one when it is greater than or equal to z. S ( x , z) := if ( x z , 1 , 0 )

7.

Write the shear and moment equations in Mathcad form, using the function S as a multiplying factor to get the effect of the singularity functions. V ( x) := R1 S ( x , 0 in) p S ( x , a ) ( x a) + p S ( x , b ) ( x b ) + R2 S ( x , L) M ( x) := R1 S ( x , 0 in) x p 2 S(x , a) ( x a) +
2

p 2

S ( x , b ) ( x b ) + R2 S ( x , L) ( x L)

MACHINE DESIGN - An Integrated Approach, 4th Ed.


8. Plot the shear and moment diagrams.

3-37a-2

SHEAR DIAGRAM
1000 5000

MOMENT DIAGRAM

4000 0 V ( x) lbf 1000 1000 M ( x) in lbf 2000 3000

2000

10 x in

15

20

10 x in

15

20

FIGURE 3-37aB
Shear and Moment Diagrams for Problem 3-37a

9.

Determine the maximum shear and maximum moment from inspection of the diagrams. Maximum shear: Vmax := V ( b ) Vmax = 1700 lbf

Maximum moment occurs where V is zero, which is x = c, where: R1 b + R2 a R1 + R2

c :=

c = 16.3 in

Mmax := M ( c)

Mmax = 4845 in lbf

MACHINE DESIGN - An Integrated Approach, 4th Ed.

3-38a-1

PROBLEM 3-38a
Statement: A beam is supported and loaded as shown in Figure P3-17. Find the reactions, maximum shear, and maximum moment for the data given in row a from Table P3-2. Beam length Distance to RH bearing Distance to concentrated load Concentrated load Distributed load Solution: 1. L := 20 in a := 16 in b := 18 in P := 1000 lbf p := 1000 lbf in
1
R1
a

Given:

b p

R2

FIGURE 3-38aA See Figure 3-38 and Mathcad file P0338a.


Free Body Diagram for Problem 3-38

Determine the distance from the origin to the left and right ends of the roller. Distance to left end Distance to right end e := 0.1 a f := 0.9 a e = 1.600 in f = 14.400in

2.

From inspection of Figure 3-38, write the load function equation q(x) = R1<x - 0>-1 - p<x - e>0 + p<x - f>0 + R2<x - a>-1 - P<x - b>-1

3.

Integrate this equation from - to x to obtain shear, V(x) V(x) = R1<x - 0>0 - p<x - e>1 + p<x - f>1 + R2<x - a>0 - P<x - b>0

4.

Integrate this equation from - to x to obtain moment, M(x) M(x) = R1<x - 0>1 - p<x - e>2/2 + p<x - f>2/2 + R2<x - a>1 - P<x - b>1

5.

Solve for the reactions by evaluating the shear and moment equations at a point just to the right of x = b, where both are zero. At x = b +, V = M = 0 V = R1 p ( b e ) + p ( b f ) + R2 P = 0 M = R1 b p 2 ( b e) +
2

p 2

( b f ) + R2 ( b a) = 0

R1 :=

e2 f 2 b a P + f e p 2 a a

R1 = 6275 lbf R2 = 7525 lbf

R2 := p ( f e) R1 + P 6. 7. Define the range for x x := 0 m , 0.002 L .. L

For a Mathcad solution, define a step function S. This function will have a value of zero when x is less than z, and a value of one when it is greater than or equal to z. S ( x , z) := if ( x z , 1 , 0 )

8.

Write the shear and moment equations in Mathcad form, using the function S as a multiplying factor to get the effect of the singularity functions. V ( x) := R1 S ( x , 0 m) p S ( x , e) ( x e) + p S ( x , f ) ( x f ) + R2 S ( x , a ) P S ( x , b ) M ( x) := R1 S ( x , 0 m) x p S ( x , e) ( x e) +
2

2 2 + R2 S ( x , a ) ( x a) P S ( x , b ) ( x b )

S ( x , f ) ( x f ) ...

MACHINE DESIGN - An Integrated Approach, 4th Ed.

3-38a-2

9.

Plot the shear and moment diagrams.

SHEAR DIAGRAM
10000 30000

MOMENT DIAGRAM

5000 V ( x) lbf 0 M ( x) in lbf

20000

10000

5000

10000

10 x in

15

20

10000

10 x in

15

20

FIGURE 3-38aB
Shear and Moment Diagrams for Problem 3-38a

9.

Determine the maximum shear and maximum moment from inspection of the diagrams. Maximum shear: Vmax := V ( f ) Vmax = 6525 lbf

Maximum moment occurs where V is zero, which is x = c: ce R1 = f c R2 P c := f R1 + e R2 e P R1 + R2 P c = 7.875 in

Mmax := M ( c)

Mmax = 29728 in lbf

MACHINE DESIGN - An Integrated Approach, 4th Ed.

3-39a-1

PROBLEM 3-39a
Statement: A beam is supported and loaded as shown in Figure P3-17. Write a computer program or equation solver model to find the reactions and calculate and plot the loading, shear, and moment functions. Test the program with the data given in row a from Table P3-2. Enter data in highlighted areas Beam length Distance to RH bearing Distance to concentrated load Concentrated load Distributed load Solution: 1. L := 20 in a := 16 in b := 18 in P := 1000 lbf p := 1000 lbf in
1
R1
a

Input data:

b p

R2

FIGURE 3-39aA Free Body Diagram for Problem 3-39

See Figure 3-39 and Mathcad file P0339a.

Determine the distance from the origin to the left and right ends of the roller. Distance to left end Distance to right end e := 0.1 a f := 0.9 a e = 40.64 mm f = 365.76mm

2.

From inspection of Figure 3-39, write the load function equation q(x) = R1<x - 0>-1 - p<x - e>0 + p<x - f>0 + R2<x - a>-1 - P<x - b>-1

3.

Integrate this equation from - to x to obtain shear, V(x) V(x) = R1<x - 0>0 - p<x - e>1 + p<x - f>1 + R2<x - a>0 - P<x - b>0

4.

Integrate this equation from - to x to obtain moment, M(x) M(x) = R1<x - 0>1 - p<x - e>2/2 + p<x - f>2/2 + R2<x - a>1 - P<x - b>1

5.

Solve for the reactions by evaluating the shear and moment equations at a point just to the right of x = b, where both are zero. At x = b +, V = M = 0 V = R1 p ( b e ) + p ( b f ) + R2 P = 0 M = R1 b p 2 ( b e) +
2

p 2

( b f ) + R2 ( b a) = 0

R1 :=

e2 f 2 b a P + f e p 2 a a

R1 = 6275 lbf R2 = 7525 lbf

R2 := p ( f e) R1 + P 6. 7. Define the range for x x := 0 m , 0.002 L .. L

For a Mathcad solution, define a step function S. This function will have a value of zero when x is less than z, and a value of one when it is greater than or equal to z. S ( x , z) := if ( x z , 1 , 0 )

8.

Write the shear and moment equations in Mathcad form, using the function S as a multiplying factor to get the effect of the singularity functions. V ( x) := R1 S ( x , 0 m) p S ( x , e) ( x e) + p S ( x , f ) ( x f ) + R2 S ( x , a ) P S ( x , b )

MACHINE DESIGN - An Integrated Approach, 4th Ed.

3-39a-2

M ( x) := R1 S ( x , 0 m) x

2 2 + R2 S ( x , a ) ( x a) P S ( x , b ) ( x b )

S ( x , e) ( x e) +

S ( x , f ) ( x f ) ...

9.

Plot the shear and moment diagrams.

SHEAR DIAGRAM
10000 30000

MOMENT DIAGRAM

5000 V ( x) lbf 0 M ( x) in lbf

20000

10000

5000

10000

10 x in

15

20

10000

10 x in

15

20

FIGURE 3-39aB
Shear and Moment Diagrams for Problem 3-39a

9.

Determine the maximum shear and maximum moment from inspection of the diagrams. Maximum shear: Vmax := V ( f ) Vmax = 6525 lbf

Maximum moment occurs where V is zero, which is x = c: ce R1 = f c R2 P c := f R1 + e R2 e P R1 + R2 P c = 7.875 in

Mmax := M ( c)

Mmax = 29728 in lbf

MACHINE DESIGN - An Integrated Approach, 4th Ed.

3-40a-1

PROBLEM 3-40a
Statement: A beam is supported and loaded as shown in Figure P3-18. Find the reactions, maximum shear, and maximum moment for the data given in row a from Table P3-2.
a

Given:

Distance to gear 2 Distance to gear 1 Distance to RH bearing Concentrated load at gear 2 Concentrated load at gear 1

L := 20 in a := 16 in b := 18 in P2 := 1000 lbf P1 := 0.4 P2


R1

P1

P2

R2 b
L

FIGURE 3-40a Solution: 1. See Figure 3-40 and Mathcad file P0340a.
Free Body Diagram for Problem 3-40

From inspection of Figure 3-40, write the load function equation q(x) = R1<x - 0>-1 - P1<x - a>-1 + R2<x - b>-1 - P2<x - L>-1

2.

Integrate this equation from - to x to obtain shear, V(x) V(x) = R1<x - 0>0 - P1<x - a>0 + R2<x - b>0 - P2<x - L>0

3.

Integrate this equation from - to x to obtain moment, M(x) M(x) = R1<x - 0>1 - P1<x - a>1 + R2<x - b>1 - P<x - L>1

4.

Solve for the reactions by evaluating the shear and moment equations at a point just to the right of x = L, where both are zero. At x = L+, V = M = 0 V = R1 P1 + R2 P2 = 0 M = R1 L P1 ( L a ) + R2 ( b a ) = 0

R1 := P1 1

a b

+ P2 1

L b

R1 = 67 lbf R2 = 1467 lbf

R2 := P1 + P2 R1 5. 6. Define the range for x x := 0 m , 0.002 L .. L

For a Mathcad solution, define a step function S. This function will have a value of zero when x is less than z, and a value of one when it is greater than or equal to z. S ( x , z) := if ( x z , 1 , 0 )

7.

Write the shear and moment equations in Mathcad form, using the function S as a multiplying factor to get the effect of the singularity functions. V ( x) := R1 S ( x , 0 in) P1 S ( x , a ) + R2 S ( x , b ) P2 S ( x , L) M ( x) := R1 S ( x , 0 mm) ( x 0 mm) P1 S ( x , a ) ( x a ) ... + R2 S ( x , b ) ( x b) P2 S ( x , L) ( x L)

8.

Plot the shear and moment diagrams.

MACHINE DESIGN - An Integrated Approach, 4th Ed.

3-40a-2

SHEAR DIAGRAM
1000 0

MOMENT DIAGRAM

500 V ( x) lbf 0 M ( x) in lbf

1000

2000

500

10 x in

15

20

3000

10 x in

15

20

FIGURE 3-40aB
Shear and Moment Diagrams for Problem 3-40a

9.

Determine the maximum shear and maximum moment from inspection of the diagrams. Maximum shear: Vmax := V ( b ) Vmax = 1000 lbf

Maximum moment occurs where V is zero, which is x = b: Mmax := M ( b ) Mmax = 2000 in lbf

MACHINE DESIGN - An Integrated Approach, 4th Ed.

3-41a-1

PROBLEM 3-41a
Statement: A beam is supported and loaded as shown in Figure P3-18. Write a computer program or equation solver model to find the reactions and calculate and plot the loading, shear, and moment functions. Test the program with the data given in row a from Table P3-2. Enter data in highlighted areas Distance to gear 2 Distance to gear 1 Distance to RH bearing Concentrated load at gear 2 Concentrated load at gear 1 Solution: 1. L := 20 in a := 16 in b := 18 in P2 := 1000 lbf P1 := 0.4 P2 FIGURE 3-41aA See Figure 3-41 and Mathcad file P0341a.
Free Body Diagram for Problem 3-41
R1 b
L

Input data:

a P1
P2

R2

From inspection of Figure 3-41, write the load function equation q(x) = R1<x - 0>-1 - P1<x - a>-1 + R2<x - b>-1 - P2<x - L>-1

2.

Integrate this equation from - to x to obtain shear, V(x) V(x) = R1<x - 0>0 - P1<x - a>0 + R2<x - b>0 - P2<x - L>0

3.

Integrate this equation from - to x to obtain moment, M(x) M(x) = R1<x - 0>1 - P1<x - a>1 + R2<x - b>1 - P<x - L>1

4.

Solve for the reactions by evaluating the shear and moment equations at a point just to the right of x = L, where both are zero. At x = L+, V = M = 0 V = R1 P1 + R2 P2 = 0 M = R1 L P1 ( L a ) + R2 ( b a ) = 0

R1 := P1 1

a b

+ P2 1

L b

R1 = 67 lbf R2 = 1467 lbf

R2 := P1 + P2 R1 5. 6. Define the range for x x := 0 m , 0.002 L .. L

For a Mathcad solution, define a step function S. This function will have a value of zero when x is less than z, and a value of one when it is greater than or equal to z. S ( x , z) := if ( x z , 1 , 0 )

7.

Write the shear and moment equations in Mathcad form, using the function S as a multiplying factor to get the effect of the singularity functions. V ( z) := R1 S ( z , 0 in) P1 S ( z , a) + R2 S ( z , b) P2 S ( z , L) M ( z) := R1 S ( z , 0 mm) ( z 0 mm ) P1 S ( z , a) ( z a ) ... + R2 S ( z , b) ( z b ) P2 S ( z , L) ( z L)

MACHINE DESIGN - An Integrated Approach, 4th Ed.


8. Plot the shear and moment diagrams.

3-41a-2

SHEAR DIAGRAM
1000 0

MOMENT DIAGRAM

500 V ( x) lbf 0 M ( x) in lbf

1000

2000

500

10 x in

15

20

3000

10 x in

15

20

FIGURE 3-41aB
Shear and Moment Diagrams for Problem 3-41a

9.

Determine the maximum shear and maximum moment from inspection of the diagrams. Maximum shear: Vmax := V ( b ) Vmax = 1000 lbf

Maximum moment occurs where V is zero, which is x = b: Mmax := M ( b ) Mmax = 2000 in lbf

MACHINE DESIGN - An Integrated Approach, 4th Ed.

3-42-1

PROBLEM 3-42
Statement:

_____

A 1000 kg speedboat reaches a speed of 16 kph at the instant it takes up the slack in a 100 m-long tow rope attached to a surfboard carrying a 100 kg passenger. If the rope has k = 5 N/m, what is the dynamic force exerted on the surfboard? Mass of speedboat Speed of boat Mass of passenger Rope stiffness ms := 1000 kg vi := 16 kph mp := 100 kg k := 5 N m
1

Given:

Assumptions: 1. The water does not influence the dynamic force. 2. An impact model can be used to estimate the dynamic force. Solution: 1. See Mathcad file P0342.

For the impact model, the passenger is the "struck" mass and the speedboat is the "striking mass". Thus, from equation 3.15, the energy correction factor is:

:=
1+

1 mp 3 ms

= 0.97

2.

Use equation 3.11 to estimate the dynamic force on the surfboard/passenger.

Fi := vi ms k

Fi = 309 N

MACHINE DESIGN - An Integrated Approach, 4th Ed.

3-43-1

PROBLEM 3-43
Statement: Figure P3-19 shows an oil-field pump jack. For the position shown, draw free-body diagrams of the crank (2), connecting rod (3) and walking beam (4) using variable names similar to those used in Case Studies 1A and 2A. Assume that the crank turns slowly enough that accelerations can be ignored. Include the weight acting at the CG of the walking beam and the crank but not the connecting rod.

Assumptions: 1. A two-dimensional model is adequate. 2. Inertia forces may be ignored. Solution: 1. See Mathcad file P0343.

Isolate each of the elements to be analyzed, starting with the walking beam, since the external forces on it are known. Place the known force, Fcable, at the point P and the known weight at the CG. Assume the forces at the interfaces O4 and B to be positive. The position vectors R14, R34, and Rp will be known as will the angle, 3,that the connecting rod makes with the horizontal axis.

F34 R34 R 14 head end P 4 O4 F


14x

y 3 B x
counterweight

RP F
14y

F cable
2. The connecting rod is a two-force member with the forces acting at the interfaces A and B along the line joining points A and B. The assumption made in step 1 is that these are compressive forces on link 3.

W4

F43 y B R 43

3
x

R23

F 23

MACHINE DESIGN - An Integrated Approach, 4th Ed.


3. The crank is acted on by forces at A and O2, its weight at its CG, and a torque which we will assume to be positive (CCW). As in step 1, assume that the unknown reaction force at O2 is positive.

3-43-2

F32 y

12y

3 x

2 O2 T2 W2

12x

MACHINE DESIGN - An Integrated Approach, 4th Ed.

3-44-1

PROBLEM 3-44
Statement: Given: For the pump jack of Problem 3-43 and the data of Table P3-3, determine the pin forces on the walking beam, connecting rod, and crank and the reaction torque on the crank. R12 := 13.2 in R32 := 0.80 in Fcable := 2970 lbf

12 := 135 deg 32 := 45 deg

R14 := 79.22 in R34 := 32.00 in W2 := 598 lbf

14 := 196 deg 34 := 169 deg


W4 := 2706 lbf

3 := 98.5 deg
Solution: 1.

RP := 124.44 in P := 185 deg

See Mathcad files P0343 and P0344.

Draw free-body diagrams of each element (see Problem 3-43).

F34 R34 R 14 head end P 4 O4 F


14x

y 3 B x
counterweight

RP F
14y

F cable

W4

F43 y

F32 y

B R 43

12y

3 x

3
x

2 O2 T2 W2

12x

R23

F 23

MACHINE DESIGN - An Integrated Approach, 4th Ed. 2. Calculate the x- and y-components of the position vectors.
R12x := R12 cos( 12) R14x := R14 cos( 14) R32x := R32 cos( 32) R34x := R34 cos( 34) RPx := RP cos( P) 3. R12x = 9.334 in R14x = 76.151 in R32x = 0.566 in R34x = 31.412 in RPx = 123.966 in R12y := R12 sin( 12) R14y := R14 sin( 14) R32y := R32 sin( 32) R34y := R34 sin( 34) RPy := RP sin( P) R12y = 9.334 in R14y = 21.836 in R32y = 0.566 in R34y = 6.106 in RPy = 10.846 in

3-44-2

Write equations 3(b) for link 4, the walking beam.

Fx: Fy: Mz:


4.

F14x + F34x = 0 Fcable + F14y + F34y W4 = 0 Rpx Fcable + ( R14x F14y R14y F14x) + ( R34x F34y R34y F34x) = 0

(1) (2) (3)

The direction (but not the sense) of F34 is known so write the equation that relates the x- and y-components of this force. F34y F34x tan( 3) = 0 (4)

5.

There are four unknowns in the four equations above. Solving for F14x, F14y, F34x, and F34y, 0 1 0 1 0 1 0 1 A := R14y R14x R34y R34x in in in in 0 tan( 3) 1 0 F14x = 2446 lbf F14y = 10687 lbf 0 Fcable + W4 lbf B := RPx Fcable in lbf 0 F34x = 2446 lbf

F14x F 14y := A 1 B lbf F34x F34y


F34y = 16363 lbf

6.

From Newton's thrid law and, since the connecting rod (3) is a two-force member F43x := F34x F23x := F43x F43x = 2446 lbf F23x = 2446 lbf F43y := F34y F23y := F43y F43y = 16363 lbf F23y = 16363 lbf

7.

Write equations 3(b) for link 2, the crank.

Fx: Fy: Mz:


8. F32x := F23x

F12x + F32x = 0 F12y + F32y W2 = 0 T2 + ( R12x F12y R12y F12x) + ( R32x F32y R32y F32x) = 0 (7)

(5) (6)

There are three unknowns in the three equations above. Solving for F12x, F12y, and T2, since F32x = 2446 lbf F32y := F23y F32y = 16363 lbf

MACHINE DESIGN - An Integrated Approach, 4th Ed.

3-44-3

0 0 1 1 0 0 A := R12y R12x 1 in in

F32x lbf W2 F32y B := lbf ( R32x F32y R32y F32x) in lbf

F12x 1 F12y := A B T 2

F12x = 2446 T2 = 146128

lbf in-lbf

F12y = 16961

lbf

MACHINE DESIGN - An Integrated Approach, 4th Ed.

3-45-1

PROBLEM 3-45
Statement: Figure P3-20 shows an aircraft overhead bin mechanism in end view. For the position shown, draw free-body diagrams of links 2 and 4 and the door (3) using variable names similar to those used in Case Studies 1A and 2A. There are stops that prevent further clockwise motion of link 2 (and the identical link behind it at the other end of the door) resulting in horizontal forces being applied to the door at points A. Assume that the mechanism is symmetrical so that each set of links 2 and 4 carry one half of the door weight. Ignore the weight of links 2 and 4 as they are negligible.

Assumptions: 1. A two-dimensional model is adequate. 2. Inertia forces may be ignored as the mechanism is at rest against stops. Solution: 1. See Mathcad file P0345.

Isolate each of the elements to be analyzed, starting with the door. Place the force, Fstop, at the point A and the known weight at the CG. Assume the forces in links 2 and 4 to be positive (tensile). The position vectors R43 and R23 will be known as will the angles 2 and 4 that links 2 and 4 make with the horizontal axis.

F 23 4 2 F stop A R23 F 43 y R43 3 B x

W3 2

F 12

2.

Links 2 and 4 are two-force members with the forces acting at the pinned ends along the line joining the pin centers. The assumption made in step 1 is that these are tensile forces on links 2 and 4.

y O2 R12 2

x
4

F 14 O4 R14 x 4 R34 B F 34

R32 A

F 32

MACHINE DESIGN - An Integrated Approach, 4th Ed.

3-46-1

PROBLEM 3-46
Statement: Given: For the overhead bin mechanism of Problem 3-45 and the data of Table P3-4, determine the pin forces on the door (3), and links 2 & 4 and the reaction force on each of the two stops. R23 := 180.0 mm 23 := 160.345 deg W3 := 45 N Solution: 1. R43 := 180.0 mm

43 := 27.862 deg

2 := 85.879 deg

4 := 172.352 deg

See Mathcad files P0345 and P0346.

Draw free-body diagrams of each element (see Problem 3-45).

F 12

y O2 R12 2

2
4

F 14

x R32 A
F 23

O4

R14

x 4 R34 B F 34

F 32
F stop A

2 R23

F 43 y R43 3 B x

W3 2
2. Calculate the x- and y-components of the position vectors on the door (3). R23x := R23 cos( 23) R43x := R43 cos( 43) 3. R23x = 169.512 mm R43x = 159.134 mm R23y := R23 sin( 23) R43y := R43 sin( 43) R23y = 60.544 mm R43y = 84.122 mm

Write equations 3(b) for link 3, the door.

Fx:

Fstop + F23x + F43x = 0

(1)

MACHINE DESIGN - An Integrated Approach, 4th Ed. F23y + F43y 0.5 W3 = 0 Fy:

3-46-2
(2)

Mz:
4.

R23x Fstop + ( R23x F23y R23y F23x) + ( R43x F43y R43y F43x) = 0

(3)

The directions (but not the sense) of F 23 and F43 are known so write the equations that relates the x- and y-components of these forces. F23y F23x tan( 2) = 0 F43y F43x tan( 4) = 0 (4) (5)

5.

There are five unknowns in the five equations above. Solving for F23x, F23y, F43x, F43y, and Fstop:

0 1 0 1 1 1 0 1 0 0 R23y R23x R43y R43x R23x A := mm mm mm mm mm 0 0 0 tan( 2) 1 0 0 tan( 4) 1 0

0 0.5 W3 N B := 0 0 0

F23x F23y F43x := A 1 B N F43y F stop

F23x = 1.49 N

F23y = 20.63 N

F43x = 13.96 N

F43y = 1.87 N

The pin forces at A and B are: F23 := F23x + F23y


2 2

F23 = 20.68 N Fstop = 12.47 N

F43 :=

F43x + F43y

F43 = 14.08 N

The force on each stop is: 6.

From Newton's thrid law and, since links 2 and 4 are two-force members F34x := F43x F32x := F23x F34x = 13.96 N F32x = 1.49 N F34y := F43y F32y := F23y F34y = 1.87 N F32y = 20.63 N

The pin forces at O2 and O4 are numerically equal to those at A and B, respectively.

MACHINE DESIGN - An Integrated Approach, 4th Ed.

3-47-1

PROBLEM 3-47
Statement: A particular automobile front suspension consists of two A-arms , the wheel (with tire), a coil spring, and a shock absorber (damper). The effective stiffness of the suspension (called the "ride rate") is a function of the coil spring stiffness and the tire stiffness. The A-arms are designed to give the wheel a nearly vertical displacement as the tire rides over bumps in the road. The entire assembly can be modeled as a spring-mass-damper system as shown in Figure 3-15(b). If the sprung mass (mass of the portion of the vehicle supported by the suspension system) weighs 675 lb, determine the ride rate that is required to achieve an undamped natural frequency of 1.4 Hz. What is the static deflection of the suspension for the calculated ride rate? Hz := 2 rad sec Sprung mass
1

Units: Given: Solution: 1.

Ws := 675 lbf

Natural frequency n := 1.4 Hz

See Figure 3-15(b) and Mathcad file P0347. Ws g Ms = 1.748 lbf sec in
2 1

Calculate the sprung mass Ms :=

2.

Using equation 3.4, calculate the required ride rate Ride rate k := n Ms
2

k = 135.28

lbf in

3.

Calculate the static deflection using equation 3.5 Static deflection

:=

Ws k

= 4.99in

MACHINE DESIGN - An Integrated Approach, 4th Ed.

3-48-1

PROBLEM 3-48
Statement: The independent suspension system of Problem 3-47 has an unsprung weight (the weight of the axle, wheel, A-arms, etc.) of 106 lb. Calculate the natural frequency (hop resonance) of the unsprung mass if the sum of the tire and coil spring stiffnesses is 1100 lb/in. Hz := 2 rad sec Unsprung mass
1

Units: Given:

Wu := 106 lbf

Stiffness

k := 1100

lbf in

Solution: 1.

See Figure 3-15(b) and Mathcad file P0348. Wu g Mu = 0.275 lbf sec in
2 1

Calculate the unsprung mass Mu :=

2.

Using equation 3.4, calculate the natural frequency Natural frequency

n :=

k Mu

n = 10.1 Hz

MACHINE DESIGN - An Integrated Approach, 4th Ed.

3-49-1

PROBLEM 3-49
Statement: The independent suspension system of Problem 3-47 has a sprung weight of 675 lb and a ride rate of 135 lb/in. Calculate the damped natural frequency of the sprung mass if the damping coefficient of the shock absorber is a constant 12 lb-sec/in. Hz := 2 rad sec Sprung mass
1

Units: Given:

Ws := 675 lbf d := 12

Ride rate lbf sec in

k := 135

lbf in

Damping coefficient

Solution: 1.

See Figure 3-15(b) and Mathcad file P0349. Ws g Ms = 1.748lbf sec in


2 1

Calculate the sprung mass Ms :=

2.

Using equation 3.7, calculate the damped natural frequency Damped natural frequency

d :=

k Ms

2 Ms
d

d = 1.29Hz

MACHINE DESIGN - An Integrated Approach, 4th Ed.

3-50-1

PROBLEM 3-50_______________________________________________________
Statement: Figure P3-22 shows a powder compaction mechanism. For the position shown, draw free-body diagrams of the input arm (2), connecting rod (3) and compacting ram (4) using variable names similar to those used in Case Studies 1A and 2A. Assume that the input arm turns slowly enough that accelerations can be ignored. Ignore the weights of the arm, connecting rod, and compacting ram. Neglect friction.

Assumptions: 1. A two-dimensional model is adequate. 2. Inertia forces may be ignored. 3. The reactions at slider bearings E and F can be modeled as concentrated forces acting horizontally at the center of each bearing. Solution: 1. See Mathcad file P0350.

y E D F34
3

Isolate each of the elements to be analyzed, starting with the compacting rod, since the external force on it is known. Place the known force, Fcom , at the point P. The position vectors R14E, R14F, and R p will be known as will the angle, q3,that the compacting ram makes with the vertical axis.

F14E

R14E R34 x F R14F RP

2.

3.

The connecting rod is a two-force member with the forces acting at the interfaces B and D along the line joining points B and D. The assumption made in step 1 is that these are tensile forces on link 3. The input arm is acted on by forces at A, B, and C. Assume that the unknown reaction force at A is positive.

F14F

F43

P Fcom
Compacting Ram (4)

D y R43 x R23 B
Fin Rin B R32 R12
Input Arm (2)

C y F32 x F12y A F12x

F23

Connecting Rod (3)

MACHINE DESIGN - An Integrated Approach, 4th Ed.

3-51-1

PROBLEM 3-51______________________________________________________
Statement: Given:
For the compaction mechanism of Problem 3-50 and the data of Table P3-5, determine the pin forces on the compacting ram, connecting rod, and input arm.

R12 := 148.4 mm 12 := 45 deg R14F := 62.9 mm 14F := 270 deg R23 := 87.6 mm R32 := 42.9 mm Fcom := 100 N

R14E := 57.0 mm 14E := 90 deg R34 := 32.00 in R34 := 15.0 mm

34 := 105.64 deg 43 := 74.36 deg 34 := 90 deg

23 := 254.36 deg 32 := 74.36 deg 3 := 254.36 deg

R43 := 87.6 mm

Rin := 152.6 mm in := 225 deg

RP := 105.0 mm P := 270 deg

Solution: 1.

See Mathcad files P0350 and P0351.

Draw free-body diagrams of each element (see Problem 3-50).

y R14E R34 x F
3

F43

F14E

E D

D y R43 x R23 B
C

F34 F14F

R14F RP

P Fcom

Compacting Ram (4)

F23
y
Connecting Rod (3)

Fin Rin B R32


Input Arm (2)

F32 x F12y A R12 F12x

MACHINE DESIGN - An Integrated Approach, 4th Ed.


2. Calculate the x- and y-components of the position vectors. R12x := R12 cos( 12) R14Ex := R14E cos( 14E) R14Fx := R14F cos( 14F ) R23x := R23 cos( 23) R32x := R32 cos( 32) R34x := R34 cos( 34) R43x := R43 cos( 43) RPx := RP cos( P) Rinx := Rin cos( in) 3. R12x = 104.935 mm R14Ex = 0 mm R14Fx = 0.000 mm R23x = 23.616 mm R32x = 11.566 mm R34x = 0.000 mm R43x = 23.616 mm RPx = 0.000 mm Rinx = 107.904 mm R12y := R12 sin( 12) R14Ey := R14E sin( 14E) R14Fy := R14F sin( 14F ) R23y := R23 sin( 23) R32y := R32 sin( 32) R34y := R34 sin( 34) R43y := R43 sin( 43) RPy := RP sin( P) Riny := Rin sin( in)

3-51-2

R12y = 104.935 mm R14Ey = 57.000 mm R14Fy = 62.900 mm R23y = 84.357 mm R32y = 41.312 mm R34y = 15.000 mm R43y = 84.357 mm RPy = 105.000 mm Riny = 107.904 mm

Write equations 3(b) for link 4, the compacting ram.

Fx: Fy: Mz:


4.

F14E + F14F + F34x = 0 Fcom + F34y = 0

(1) (2) (3)

(R14Ey F14E) + (R14Fy F14F ) + (R34x F34y R34y F34x) = 0

The direction (but not the sense) of F34 is known so write the equation that relates the x- and y-components of this force. F34y F34x tan( 3) = 0 (4)

5.

There are four unknowns in the four equations above. Solving for F14x, F14y, F34x, and F34y, 1 1 0 1 0 0 0 1 A := R14Ey R14Fy R34y R34x mm mm mm mm 0 tan( 3) 1 0 F14E = 18.2 N F14F = 9.8 N

0 F com B := N 0 0
F34x = 28.0 N

F14E F 14F := A 1 B N F34x F34y


F34y = 100.0 N

6.

From Newton's thrid law and, since the connecting rod (3) is a two-force member F43x := F34x F23x := F43x F43x = 28.0 N F23x = 28.0 N F43y := F34y F23y := F43y F43y = 100.0 N F23y = 100.0 N

7.

Write equations 3(b) for link 2, the input arm.

Fx: Fy: Mz:

F12x + F32x + Finx = 0 F12y + F32y + Finy = 0

(5) (6) (7)

(R12x F12y R12y F12x) + (R32x F32y R32y F32x) + (Rinx Finy Riny Finx) = 0

MACHINE DESIGN - An Integrated Approach, 4th Ed.


8. The direction (but not the sense) of Fin is known so write the equation that relates the x- and y-components of this force. Finy Finx tan( in) = 0 9. There are four unknowns in the four equations above. Solving for F12x, F12y, Finx, and Finy, since F32x := F23x F32x = 28 N F32y := F23y F32x N F32y B := N ( R32x F32y R32y F32x) N mm 0 Finx = 64.0 N Fin := F32y = 100 N

3-51-3

(8)

0 1 0 1 0 1 0 1 Riny Rinx A := R12y R12x mm mm mm mm 0 tan( in) 1 0

F12x F 12y := A 1 B N Finx Finy

F12x = 36.0 N F12 :=

F12y = 36.0 N

Finx = 64.0 N

F12x2 + F12y2

Finx2 + Finy2

F12 = 51 N

Fin = 91 N

MACHINE DESIGN - An Integrated Approach, 4th Ed.

3-52-1

PROBLEM 3-52
Statement: Figure P3-23 shows a drag link slider crank mechanism. For the position shown, draw free-body diagrams of links 2 through 6 using variable names similar to those used in Case Studies 1A and 2A. Assume that the crank turns slowly enough that accelerations can be ignored. Ignore the weights of the links and any friction forces or torques.

Assumptions: 1. A two-dimensional model is adequate. 2. Inertia forces may be ignored. 3. Links 4 and 6 are three-force bodies. Solution: 1. See Figure P3-23 and Mathcad file P0352.

Isolate each of the elements to be analyzed, starting with the slider, link 6, since the external forces on it are known. Place the known force, FP, at the point P. This is a three-force member so the forces are coincident at point D and there is no turning moiment on the link. The angle, 5,that link 5 makes with the horizontal axis is known.

y
F 56

5
D

F P

2.

Link 5 is a two-force member with the forces acting at the interfaces C and D along the line joining points C and D. The assumption made in step 1 is that these are compressive forces on link 5.

F 16
Slider block 6

F45

x R45

5
D

R65
Link 5

F65

3.

Link 4 is a three-force body with the three forces meeting at a point. The position vectors R 14, R34, and R54 will be known as will the angles, 3 and 5,that links 3 and 5, respectively, make with the horizontal axis.

MACHINE DESIGN - An Integrated Approach, 4th Ed.

3-52-2

y R34
E

F54 x F14y
O4

R54

F14x F34 R14


Link 4

F43 y
B

R43 x

4.

Link 3 is a two-force member with the forces acting at the interfaces A and B along the line joining points A and B.

R23
5. The crank is acted on by forces at A and O2, and a torque which we will assume to be positive (CCW). As in step 1, assume that the unknown reaction force at O2 is positive.

F23
Link 3

F y R32 T

12y

F12x
O2

x R12

F32
Link 2

MACHINE DESIGN - An Integrated Approach, 4th Ed.

3-53-1

PROBLEM 3-53
Statement: Given:
For the drag link slider crank mechanism of Problem 3-52 and the data of Table P3-6, determine the pin forces on the slider, connecting rods, and crank and the reaction torque on the crank.

R12 := 63.5 mm R23 := 63.5 mm R43 := 63.5 mm FP := 85 N

12 := 45.38 deg 23 := 267.8 deg 43 := 87.80 deg 5 := 156.65deg

R14 := 93.6 mm R32 := 63.5 mm

14 := 55.89 deg 32 := 225.38 deg

R34 := 103.5 mm 34 := 202.68 deg R45 := 190.5 mm 45 := 156.65 deg R65 := 190.5 mm 65 := 23.35 deg R54 := 103.5 mm 54 := 45.34 deg

3 := 87.80 deg

Solution: 1.

See Mathcad files P0352 and P0353.

Draw free-body diagrams of each element (see Problem 3-52).

y
F 56

Slider block 6

5
D

F P

F 16

F45

x R45

5
D

Link 5

R65

F65

2.

Calculate the x- and y-components of the position vectors. R12x := R12 cos( 12) R14x := R14 cos( 14) R23x := R23 cos( 23) R32x := R32 cos( 32) R34x := R34 cos( 34) R43x := R43 cos( 43) R12x = 44.602 mm R14x = 52.489 mm R23x = 2.438 mm R32x = 44.602 mm R34x = 95.497 mm R43x = 2.438 mm R12y := R12 sin( 12) R14y := R14 sin( 14) R23y := R23 sin( 23) R32y := R32 sin( 32) R34y := R34 sin( 34) R43y := R43 sin( 43) R12y = 45.198 mm R14y = 77.497 mm R23y = 63.453 mm R32y = 45.198 mm R34y = 39.908 mm R43y = 63.453 mm

MACHINE DESIGN - An Integrated Approach, 4th Ed.


R45x := R45 cos( 45) R54x := R54 cos( 54) R65x := R65 cos( 65) R45x = 174.898 mm R54x = 72.75 mm R65x = 174.898 mm R45y := R45 sin( 45) R54y := R54 sin( 54) R65y := R65 sin( 65)

3-53-2
R45y = 75.504 mm R54y = 73.619 mm R65y = 75.504 mm

F43 y

y R34
E

F54 x F14y
O4

R43 x
B

R54

F14x
R23
A

F34

R14
Link 4

F23
Link 3

F y R32 T

12y

F12x
O2

x R12

F32
Link 2 3. Write equations 3(b) for link 5, the slider.

Fx: Fy:
4.

F56x FP = 0 F16 + F56y = 0

(1) (2)

The direction (but not the sense) of F56 is known so write the equation that relates the x- and y-components of this force. F56y F56x tan( 5) = 0 (3)

5.

There are three unknowns in the three equations above. Solving for F56x, F56y, and F16,

MACHINE DESIGN - An Integrated Approach, 4th Ed.


0 0 1 0 1 1 A := tan( 5) 1 0

3-53-3

FP N B := 0 0
F56y = 36.7 N F16 = 36.7 N

F56x 1 F56y := A B N F 16

F56x = 85.0 N 6.

From Newton's thrid law and, since the connecting rod (5) is a two-force member F65x := F56x F45x := F65x and, for link 4 F54x := F45x F54x = 85 N F54y := F45y F54y = 36.7 N F65x = 85 N F45x = 85 N F65y := F56y F45y := F65y F65y = 36.7 N F45y = 36.7 N

7.

Write equations 3(b) for link 4, the rocker.

Fx: Fy: Mz:


8.

F34x + F54x + F14x = 0 F34y + F54y + F14y = 0

(4) (5)

(R14x F14y R14y F14x) + (R34x F34y R34y F34x) + (R54x F54y R54y F54x) = 0

The direction (but not the sense) of F34 is known so write the equation that relates the x- and y-components of this force. F34y F34x tan( 3) = 0 (7)

9.

There are four unknowns in the four equations above. Solving for F34x, F34y, F14x, and F14y, F54x N F54y B := N ( R54x F54y R54y F54x) N mm 0 F14x = 81.5 N

0 1 0 1 0 1 0 1 A := R34y R34x R14y R14x mm mm mm mm 0 0 tan( 3) 1

F34x F 34y := A 1 B N F14x F14y

F34x = 3.5 N

F34y = 90.9 N

F14y = 127.6 N

10. From Newton's thrid law and, since the connecting rod (3) is a two-force member F43x := F34x F23x := F43x and, for link 2 F32x := F23x F32x = 3.5 N F32y := F23y F32y = 90.9 N F43x = 3.5 N F23x = 3 N F43y := F34y F23y := F43y F43y = 90.9 N F23y = 90.9 N

MACHINE DESIGN - An Integrated Approach, 4th Ed.


11. Write equations 3(b) for link 2, the crank.

3-53-4

Fx: Fy: Mz:

F12x + F32x = 0 F12y + F32y = 0 T2 + ( R12x F12y R12y F12x) + ( R32x F32y R32y F32x) = 0

(8) (9) (10)

12. There are three unknowns in the three equations above. Solving for F12x, F12y, and T2 F32x N F32y B := N ( R32x F32y R32y F32x) N mm

0 0 1 1 0 0 A := R12y R12x mm mm 1

F12x 1 F12y := A B T 2

F12x = 3.5 N

F12y = 90.9

T2 = 7796

N*mm

MACHINE DESIGN - An Integrated Approach, 4th Ed.

4-1a-1

PROBLEM 4-1a
Statement: A differential stress element has a set of applied stresses on it as indicated in each row of Table P4-1. For row a, draw the stress element showing the applied stresses, find the principal stresses and maximum shear stress using Mohr's circle diagram, and draw the rotated stress element showing the principal stresses.

Given:

x := 1000 xy := 500

y := 0 yz := 0

z := 0 zx := 0
500 y x 1000

Solution:

See Figure 4-1a and Mathcad file P0401a.

1. Draw the stress element, indicating the x and y axes. 2. Draw the Mohr's circle axes, indicating the and axes with CW up and CCW down. 3. Plot the positive x-face point, which is (+1000, -500), and label it with an "x." 4. Plot the positive y-face point, which is (0, +500), and label it with a "y."

FIGURE 4-1aA
Stress Element for Problem 4-1a

5. Draw a straight line from point x to point y. Using the point where this line intersects the -axis as the center of the Mohr circle, draw a circle that goes through points x and y. 6. The center of the circle will be at

c :=

x + y
2

c = 500

7. The circle will intersect the -axis at two of the principal stresses. In this case, we see that one is positive and the other is negative so they will be 1 and 3. The third principal stress is 2 = 0.

8. Calculate the radius of the circle

x y 2 R := + xy 2

R = 707.1

CW

CW

1-3 1-2 500 y 500

-500 3 0

500 2

1000 1

1500

-500 3

2-3 0 2

500

1000 1

1500

500

500

CCW

CCW

FIGURE 4-1aB
2D and 3D Mohr's Circle Diagrams for Problem 4-1a

MACHINE DESIGN - An Integrated Approach, 4th Ed.


9. Calculate the principal stresses

4-1a-2

1 := c + R 3 := c R

1 = 1207 3 = 207

2 := 0

10. Draw the three Mohr's circles to represent the complete 3D stress state. 11. Calculate the principal shear stresses
y 207 1207 22.5 x

12 := 0.5 ( 1 2) 23 := 0.5 ( 2 3) 13 := 0.5 ( 1 3)

12 = 603.6 23 = 103.6 13 = 707.1

The maximum principal stress is always 13. 12. Determine the orientation of the principal normal stress (1) with respect to the x-axis. From the 2D Mohr's circle diagram, we see that the angle 2 from x to 1 is CCW and is given by FIGURE 4-1aC

x c := acos 2 R
1

= 22.5 deg

Rotated Stress Element for Problem 4-1a

13. Draw the rotated 2D stress element showing the two nonzero principal stresses.

MACHINE DESIGN - An Integrated Approach, 4th Ed.

4-1h-1

PROBLEM 4-1h
Statement: A differential stress element has a set of applied stresses on it as indicated in each row of Table P4-1. For row h, draw the stress element showing the applied stresses, find the principal stresses and maximum shear stress and draw the Mohr's circle diagram.

Given:

x := 750 xy := 500

y := 500 yz := 0

z := 250 zx := 0
z 250

Solution:

See Figures 4-1h and Mathcad file P0401h.

1. Calculate the coefficients (stress invariants) of equation (4.4c). C2 := x + y + z C2 = 1.500 10


3

C1 :=

x xy x zx y yz + + xy y zx z yz z
750

C1 = 4.375 10

500

500

500
y

x xy zx C0 := xy y yz zx yz z

C0 = 3.125 10

FIGURE 4-1hA
Stress Element for Problem 4-1h

2. Find the roots of the triaxial stress equation:

C2 + C 1 C0 = 0

C0 C1 v := C2 1

r := polyroots ( v)

110 250 r= 1140


CW

3. Extract the principal stresses from the vector r by inspection.

1 := r 2 := r 3 := r

3 2 1

1 = 1140
500

1-3 1-2

2 = 250 3 = 110
-500

2-3
0

500

1000

1500

3 2 1

4. Using equations (4.5), evaluate the principal shear stresses.

13 := 12 := 23 :=

1 3
2

13 = 515 12 = 445 23 = 70

500

1 2
2

2 3
2

CCW

FIGURE 4-1hB 5. Draw the three-circle Mohr diagram.


The Three Mohr's Circles for Problem 4-1h

MACHINE DESIGN - An Integrated Approach, 4th Ed.

4-2-1

PROBLEM 4-2
Statement: A 400-lb chandelier is to be hung from two 10-ft-long solid steel cables in tension. Choose a suitable diameter for the cable such that the stress will not exceed 5000 psi. What will be the deflection of the cables? State all assumptions. Weight of chandelier Length of cable Allowable stress Number of cables Young's modulus W := 400 lbf L := 10 ft L = 120 in allow := 5000 psi N := 2 E := 30 10 psi
6

Given:

Assumptions: The cables share the load equally. Solution: 1. 2. See Mathcad file P0402. P := W N P = 200 lbf

Determine the load on each cable

The stress in each cable will be equal to the load on the cable divided by its cross-sectional area. Using equation (4.7), and setting the stress equal to the allowable stress, we have

allow =
3.

4 P

d
Solve this equation for the unknown cable diameter. d := 4 P

allow

d = 0.226 in

4. 5.

Round this up to the next higher decimal equivalent of a common fractional size: Using equation (4.8), determine the deflection in each cable. Cross-section area A :=

d := 0.250 in

d
4

A = 0.049 in

Cable deflection

s :=

P L A E

s = 0.016 in

MACHINE DESIGN - An Integrated Approach, 4th Ed.

4-3-1

PROBLEM 4-3
Statement: For the bicycle pedal-arm assembly in Figure 4-1 with rider-applied force of 1500 N at the pedal, determine the maximum principal stress in the pedal arm if its cross-section is 15 mm in dia. The pedal attaches to the pedal arm with a 12-mm screw thread. What is the stress in the pedal screw? Distances (see figure) Rider-applied force Pedal arm diameter Screw thread diameter a 170 mm Frider 1.5 kN d pa 15 mm d sc 12 mm
z

Given:

b 60 mm

Solution:

See Figure 4-3 and Mathcad file P0403.


a C Mc Arm y Tc Frider b

1. From the FBD in Figure 4-3A (and on the solution for Problem 3-3), we see that the force from the rider is reacted in the pedal arm internally by a moment, a torque, and a vertical shear force. There are two points at section C (Figure 4-3B) that we should investigate, one at z = 0.5 d pa (point A), and one at y = 0.5 d pa (point B). 2. Refering to the FBD resulting from taking a section through the arm at C, the maximum bending moment Mc is found by summing moments about the y-axis, and the maximum torque Tc is found by summing moments about the x-axis. M y: M x: Frider a Mc = 0 Frider b Tc = 0

Fc Pedal x

FIGURE 4-3A
Free Body Diagram for Problem 4-3

A
Maximum bending moment: Mc Frider a Mc 255 N m

Section C B

Arm

Maximum torque: Tc Frider b Vertical shear: Fc Frider Fc 1.500 kN Tc 90 N m

x y
FIGURE 4-3B
Points A and B at Section C

3.

Determine the stress components at point A where we have the effects of bending and torsion, but where the transverse shear due to bending is zero because A is at the outer fiber. Looking down the z-axis at a stress element on the surface at A, Distance to neutral axis cpa 0.5 d pa cpa 7.5 mm

MACHINE DESIGN - An Integrated Approach, 4th Ed.


Moment of inertia of pedal-arm

4-3-2

Ipa

d pa
64

Ipa 2.485 10 mm

Bending stress (x-direction) Stress in y-direction Torsional stress due to Tc

Mc cpa Ipa

x 769.6 MPa

y 0 MPa xy
Tc cpa 2 Ipa

xy 135.8 MPa

CW

Principal stresses at A, equation (4.6a)

1A

x y
2

2 x y 2 xy 2 2 x y 2 xy 2

3A

x y
2

1A 793 MPa
4.

2A 0 MPa

3A 23 MPa

Determine the stress components at point B where we have the effects of transverse shear and torsion, but where the bending stress is zero because B is on the neutral plane. Looking down the y-axis at a stress element at B, Cross-section area of pedal-arm Torsional stress due to Tc and shear stress due to Fc Normal stresses

Apa

d pa
4

Apa 176.7 mm

zx

4 Fc xy 3 Apa

zx 124.5 MPa

CW

x 0 MPa x z
2

z 0 MPa
2 x z 2 zx 2 2 x z 2 zx 2

Principal stresses at B

1B

3B 1B 124 MPa

x z
2

2B 0 MPa

3B 124 MPa

5.

The maximum principal stress is at point A and is

1A 793 MPa

MACHINE DESIGN - An Integrated Approach, 4th Ed.


6. Determine the stress in the pedal screw. Bending moment Distance to neutral axis Moment of inertia of pedal screw Msc Frider b csc 0.5 d sc Isc Msc 90 N m csc 6 mm Isc 1.018 10 mm
3 4

4-3-3

d sc
64

Bending stress (y-direction) Stress in z-direction Torsional stress

Msc csc Isc

y 530.5 MPa

z 0 MPa xy 0 MPa

Since there is no shear stress present at the top of the screw where the bending stress is a maximum, the maximum principal stress in the pedal screw is

1 y

1 530.5 MPa

MACHINE DESIGN - An Integrated Approach, 4th Ed.

4-4-1

PROBLEM 4-4
Statement: The trailer hitch shown in Figure P4-2 and Figure 1-1 (p. 12) has loads applied as defined in Problem 3-4. The tongue weight of 100 kg acts downward and the pull force of 4905 N acts horizontally. Using the dimensions of the ball bracket shown in Figure 1-5 (p. 15), determine: (a) The principal stresses in the shank of the ball where it joins the ball bracket. (b) The bearing stress in the ball bracket hole. (c) The tearout stress in the ball bracket. (d) The normal and shear stresses in the 19-mm diameter attachment holes. (e) The principal stresses in the ball bracket as a cantilever. a 40 mm b 31 mm Mtongue 100 kg Fpull 4.905 kN c 70 mm d sh 26 mm d 20 mm t 19 mm

Given:

Assumptions: 1. The nuts are just snug-tight (no pre-load), which is the worst case. 2. All reactions will be concentrated loads rather than distributed loads or pressures. Solution: See Figure 4-4 and Mathcad file P0404. Wtongue Mtongue g Wtongue 0.981 kN

1. The weight on the tongue is

2. Solving first for the reactions on the ball by summing the horizontal and vertical forces and the moments about A.
W tongue 70 = c

F pull

40 = a 2 A B A F b1 B F a1y 20 = d D F a2y Fa2x 2 F b2 C D Fd2 F c2y F a1x

19 = t 31 = b

Fc2x

FIGURE 4-4A
Dimensions and Free Body Diagram for Problem 4-4

Fx : Fy :

Fpull Fa1x Fb1 = 0 Fa1y Wtongue = 0

(1) (2)

MACHINE DESIGN - An Integrated Approach, 4th Ed.


MA: Fb1 t Fpull a = 0 Fb1 Fpull a t Fb1 10.326 kN Fa1x 15.231 kN Fa1y 0.981 kN

4-4-2
(3)

3. Solving equation (3) for Fb1 4. Substituting into (1) and solving for Fa1x 5. Solving (2) for Fa1y

Fa1x Fpull Fb1 Fa1y Wtongue

6. Now, refering to the FBD of the bracket, we can apply the equations of equilibrium to determine the reactions at C and D on the bracket. Fx : Fy : MC: Fa2x Fb2 Fc2x Fd2 = 0 Fc2y Fa2y = 0 Fd2 d Fb2 b Fa2x ( b t) Fa2y c = 0 (4) (5) (6)

7. Note also that the interface forces between part 1 (ball) and part 2 (bracket) have been drawn on their respective FBDs in opposite senses. Therefore, Fa2x Fa1x Fa2y Fa1y Fa2x ( b t) Fa2y c Fb2 b d Fc2x Fa2x Fb2 Fd2 Fc2y Fa2y Fb2 Fb1

8. Solving equation (6) for Fd2

Fd2

Fd2 25.505 kN Fc2x 30.41 kN Fc2y 0.981 kN

9. Substituting into (4) and solving for Fc2x 10. Solving (5) for Fa1y

11. Determine the principal stresses in the shank of the ball where it joins the ball bracket. The internal bending moment at A on the FBD of the ball is M Fpull a Distance to neutral axis csh 0.5 d sh Ish M 196.2 N m csh 13 mm
4

Moment of inertia of shank

d sh
64 M csh Ish

Ish 2.243 10 mm

Bending stress (x-direction)

x 113.7 MPa

Stress in y-direction Shear stress at A

y 0 MPa xy 0 MPa

Since the shear stress is zero, x is the maximum principal stress, thus

1 x

1 114 MPa

2 0 MPa

3 0 MPa

MACHINE DESIGN - An Integrated Approach, 4th Ed.


12. Determine the bearing stress in the ball bracket hole. Bearing area Bearing stress Abearing d sh t Abearing 494 mm
2

4-4-3

bearing

Fpull Abearing

bearing 9.93 MPa


Tearout length

13. Determine the tearout stress in the ball bracket. Shear area (see Figure 4-4B) Atear = 2 t R ( 0.5 d )
2 2

Atear 2 t ( 32 mm) 0.5 d sh


2

Atear 1111 mm Stress

tear

Fpull Atear FIGURE 4-4B


Tearout Diagram for Problem 4-4

tear 4.41 MPa


14. Determine the normal and shear stresses in the attachment bolts if they are 19-mm dia. Bolt cross-section area (2 bolts) d bolt 19 mm

Abolt 2

d bolt
4

Abolt 567.1 mm

Normal stress (tension)

bolt

Fc2x Abolt

bolt 53.6 MPa

MACHINE DESIGN - An Integrated Approach, 4th Ed. Shear stress

4-4-4
W tongue

bolt

Fc2y Abolt

bolt 1.7 MPa


F pull 1

15. Determine the principal stresses in the ball bracket as a cantilever (see Figure 4-4C). Bending moment M Fpull a Wtongue c Width of bracket Moment of inertia Total tensile stress w 64 mm I w t
3
2

M 264.8 N m
M R c

12

I 36581 mm M t 2 I Fpull w t

FIGURE 4-4C
Cantilever FBD for Problem 4-4

72.8 MPa

Since there are no shear stress at the top and bottom of the bracket where the bending stresses are maximum, they are also the principal stresses, thus

1 max
2

1 72.8 MPa max 36.4 MPa

2 0 MPa

3 0 MPa

MACHINE DESIGN - An Integrated Approach, 4th Ed.

4-5-1

PROBLEM 4-5
Statement: Repeat Problem 4-4 for the loading conditions of Problem 3-5, i.e., determine the stresses due to a horizontal force that will result on the ball from accelerating a 2000-kg trailer to 60 m/sec in 20 se Assume a constant acceleration. From Problem 3-5, the pull force is 6000 N. Determine: (a) The principal stresses in the shank of the ball where it joins the ball bracket. (b) The bearing stress in the ball bracket hole. (c) The tearout stress in the ball bracket. (d) The normal and shear stresses in the 19-mm diameter attachment holes. (e) The principal stresses in the ball bracket as a cantilever. a 40 mm b 31 mm Mtongue 100 kg Fpull 6 kN c 70 mm d sh 26 mm d 20 mm t 19 mm

Given:

Assumptions: 1. The nuts are just snug-tight (no pre-load), which is the worst case. 2. All reactions will be concentrated loads rather than distributed loads or pressures. Solution: See Figure 4-5 and Mathcad file P0405. Wtongue Mtongue g Wtongue 0.981 kN

1. The weight on the tongue is

2. Solving first for the reactions on the ball by summing the horizontal and vertical forces and the moments about A.
W tongue 70 = c

F pull

40 = a 2 A B A F b1 B F a1y 20 = d D F a2y Fa2x 2 F b2 C D Fd2 F c2y F a1x

19 = t 31 = b

Fc2x

FIGURE 4-5A
Dimensions and Free Body Diagram for Problem 4-5

Fx : Fy :

Fpull Fa1x Fb1 = 0 Fa1y Wtongue = 0

(1) (2)

MACHINE DESIGN - An Integrated Approach, 4th Ed.


MA: Fb1 t Fpull a = 0 Fb1 Fpull a t Fb1 12.632 kN Fa1x 18.632 kN Fa1y 0.981 kN

4-5-2
(3)

3. Solving equation (3) for Fb1 4. Substituting into (1) and solving for Fa1x 5. Solving (2) for Fa1y

Fa1x Fpull Fb1 Fa1y Wtongue

6. Now, refering to the FBD of the bracket, we can apply the equations of equilibrium to determine the reactions at C and D on the bracket. Fx : Fy : MC: Fa2x Fb2 Fc2x Fd2 = 0 Fc2y Fa2y = 0 Fd2 d Fb2 b Fa2x ( b t) Fa2y c = 0 (4) (5) (6)

7. Note also that the interface forces between part 1 (ball) and part 2 (bracket) have been drawn on their respective FBDs in opposite senses. Therefore, Fa2x Fa1x Fa2y Fa1y Fa2x ( b t) Fa2y c Fb2 b d Fc2x Fa2x Fb2 Fd2 Fc2y Fa2y Fb2 Fb1

8. Solving equation (6) for Fd2

Fd2

Fd2 30.432 kN Fc2x 36.432 kN Fc2y 0.981 kN

9. Substituting into (4) and solving for Fc2x 10. Solving (5) for Fa1y

11. Determine the principal stresses in the shank of the ball where it joins the ball bracket. The internal bending moment at A on the FBD of the ball is M Fpull a Distance to neutral axis csh 0.5 d sh Ish M 240 N m csh 13 mm
4

Moment of inertia of shank

d sh
64 M csh Ish

Ish 2.243 10 mm

Bending stress (x-direction)

x 139.1 MPa

Stress in y-direction Shear stress at A

y 0 MPa xy 0 MPa

Since the shear stress is zero, x is the maximum principal stress, thus

1 x

1 139 MPa

2 0 MPa

3 0 MPa

12. Determine the bearing stress in the ball bracket hole.

MACHINE DESIGN - An Integrated Approach, 4th Ed.


Bearing area Bearing stress Abearing d sh t Abearing 494 mm
2

4-5-3

bearing

Fpull Abearing

bearing 12.15 MPa

13. Determine the tearout stress in the ball bracket. Shear area (see Figure 4-4B) Atear = 2 t R ( 0.5 d )
2 2

Tearout length

Atear 2 t ( 32 mm) 0.5 d sh


2

Atear 1111 mm Stress

tear

Fpull Atear FIGURE 4-5B


Tearout Diagram for Problem 4-5

tear 5.4 MPa

14. Determine the normal and shear stresses in the attachment bolts if they are 19-mm dia. Bolt cross-section area (2 bolts)

d bolt 19 mm

Abolt 2

d bolt
4

Abolt 567.1 mm

Normal stress (tension) Shear stress

bolt

Fc2x Abolt

bolt 64.2 MPa


W tongue

bolt

Fc2y Abolt

bolt 1.7 MPa


F pull 1

15. Determine the principal stresses in the ball bracket as a cantilever (see Figure 4-4C). Bending moment M Fpull a Wtongue c Width of bracket Moment of inertia w 64 mm I w t
3
2

M 308.6 N m
M R c

12

I 36581 mm M t 2 I Fpull w t

FIGURE 4-5C
Cantilever FBD for Problem 4-5

Total tensile stress

85.1 MPa

MACHINE DESIGN - An Integrated Approach, 4th Ed.

4-5-4

Since there are no shear stress at the top and bottom of the bracket where the bending stresses are maximum, they are also the principal stresses, thus

1 max
2

1 85.1 MPa max 42.5 MPa

2 0 MPa

3 0 MPa

MACHINE DESIGN - An Integrated Approach, 4th Ed.

4-6-1

PROBLEM 4-6
Statement: Repeat Problem 4-4 for the loading conditions of Problem 3-6, i.e., determine the stresses due to a horizontal force that will results from an impact between the ball and the tongue of the 2000-kg trailer if the hitch deflects 2.8 mm dynamically on impact. The tractor weighs 1000 kg and the velocity at impact is 0.3 m/sec. Determine: (a) The principal stresses in the shank of the ball where it joins the ball bracket. (b) The bearing stress in the ball bracket hole. (c) The tearout stress in the ball bracket. (d) The normal and shear stresses in the 19-mm diameter attachment holes. (e) The principal stresses in the ball bracket as a cantilever. a 40 mm b 31 mm Mtongue 100 kg Fpull 55.1 kN c 70 mm d sh 26 mm d 20 mm t 19 mm

Given:

Assumptions: 1. The nuts are just snug-tight (no pre-load), which is the worst case. 2. All reactions will be concentrated loads rather than distributed loads or pressures. Solution: See Figure 4-6 and Mathcad file P0406. Wtongue Mtongue g Wtongue 0.981 kN

1. The weight on the tongue is

2. Solving first for the reactions on the ball by summing the horizontal and vertical forces and the moments about A.
W tongue 70 = c

F pull

40 = a 2 A B A F b1 B F a1y 20 = d D F a2y Fa2x 2 F b2 C D Fd2 F c2y F a1x

19 = t 31 = b

Fc2x

FIGURE 4-6A
Dimensions and Free Body Diagram for Problem 4-6

Fx :

Fpull Fa1x Fb1 = 0

(1)

MACHINE DESIGN - An Integrated Approach, 4th Ed.


Fy : MA: Fa1y Wtongue = 0 Fb1 t Fpull a = 0 Fb1 Fpull a t Fb1 116 kN Fa1x 171.1 kN Fa1y 0.981 kN

4-6-2
(2) (3)

3. Solving equation (3) for Fb1 4. Substituting into (1) and solving for Fa1x 5. Solving (2) for Fa1y

Fa1x Fpull Fb1 Fa1y Wtongue

6. Now, refering to the FBD of the bracket, we can apply the equations of equilibrium to determine the reactions at C and D on the bracket. Fx : Fy : MC: Fa2x Fb2 Fc2x Fd2 = 0 Fc2y Fa2y = 0 Fd2 d Fb2 b Fa2x ( b t) Fa2y c = 0 (4) (5) (6)

7. Note also that the interface forces between part 1 (ball) and part 2 (bracket) have been drawn on their respective FBDs in opposite senses. Therefore, Fa2x Fa1x Fa2y Fa1y Fa2x ( b t) Fa2y c Fb2 b d Fc2x Fa2x Fb2 Fd2 Fc2y Fa2y Fb2 Fb1

8. Solving equation (6) for Fd2

Fd2

Fd2 251.382 kN Fc2x 306.482 kN Fc2y 0.981 kN

9. Substituting into (4) and solving for Fc2x 10. Solving (5) for Fa1y

11. Determine the principal stresses in the shank of the ball where it joins the ball bracket. The internal bending moment at A on the FBD of the ball is M Fpull a Distance to neutral axis csh 0.5 d sh Ish M 2.204 10 N m csh 13 mm
4 3

Moment of inertia of shank

d sh
64 M csh Ish

Ish 2.243 10 mm

Bending stress (x-direction)

x 1277 MPa

Stress in y-direction Shear stress at A

y 0 MPa xy 0 MPa

Since the shear stress is zero, x is the maximum principal stress, thus

1 x

1 1277 MPa

2 0 MPa

3 0 MPa

MACHINE DESIGN - An Integrated Approach, 4th Ed.


12. Determine the bearing stress in the ball bracket hole. Bearing area Bearing stress Abearing d sh t Abearing 494 mm
2

4-6-3

bearing

Fpull Abearing

bearing 111.54 MPa

13. Determine the tearout stress in the ball bracket. Shear area (see Figure 4-4B) Atear = 2 t R ( 0.5 d )
2 2

Tearout length

Atear 2 t ( 32 mm) 0.5 d sh


2

Atear 1111 mm Stress

tear

Fpull Atear FIGURE 4-6B


Tearout Diagram for Problem 4-6

tear 49.59 MPa


14. Determine the normal and shear stresses in the attachment bolts if they are 19-mm dia. Bolt cross-section area (2 bolts) d bolt 19 mm

Abolt 2

d bolt
4

Abolt 567.1 mm

Normal stress (tension) Shear stress

bolt

Fc2x Abolt

bolt 540 MPa


W tongue

bolt

Fc2y Abolt

bolt 1.7 MPa


F pull 1

15. Determine the principal stresses in the ball bracket as a cantilever (see Figure 4-4C). Bending moment M Fpull a Wtongue c Width of bracket Moment of inertia w 64 mm I w t
3 3
2

M 2.3 10 N m
M R c

12

I 36581 mm M t 2 I Fpull w t

FIGURE 4-6C
Cantilever FBD for Problem 4-6

Total tensile stress

635.5 MPa

MACHINE DESIGN - An Integrated Approach, 4th Ed.

4-6-4

Since there are no shear stress at the top and bottom of the bracket where the bending stresses are maximum, they are also the principal stresses, thus

1 max
2

1 636 MPa max 318 MPa

2 0 MPa

3 0 MPa

MACHINE DESIGN - An Integrated Approach, 4th Ed.

4-7-1

PROBLEM 4-7
Statement: Design the wrist pin of Problem 3-7 for a maximum allowable principal stress of 20 ksi if the pin is hollow and loaded in double shear. Force on wrist pin Allowable stress Fwristpin 12.258 kN Fwristpin 2756 lbf

Given:

allow 20 ksi
od 0.375 in

Assumptions: Choose a suitable outside diameter, say Solution:

See Figure 4-12 in the text and Mathcad file P0407. F Fwristpin 2 F 1378 lbf

1. The force at each shear plane is

2. With only the direct shear acting on the plane, the Mohr diagram will be a circle with center at the origin and radius equal to the shear stress. Thus, the principal normal stress is numerically equal to the shear stress, which in this case is also the principal shear stress, so we have = 1 = allow. F A 4 F
2

3. The shear stress at each shear plane is

od id

= allow

4. Solving for the inside diameter,

id

od

4 F

allow

id 0.230 in

MACHINE DESIGN - An Integrated Approach, 4th Ed.

4-8-1

PROBLEM 4-8
Statement: A paper mill processes rolls of paper having a density of 984 kg/m3. The paper roll is 1.50 m outside dia (OD) by 0.22 m inside dia (ID) by 3.23 m long and is on a simply supported, hollow, steel shaft. Find the shaft ID needed to obtain a maximum deflection at the center of 3 mm if the shaft OD is 22 cm. Paper density Roll dimensions Outside diameter Inside diameter Lemgth

Given:

984

kg m
3

OD 1.50 m ID 0.22 m L 3.23 m

Shaft outside dia Young's modulus Allowable deflection

od 220 mm E 207 GPa 3 mm

Assumptions: The shaft (beam) supporting the paper roll is simply-supported at the ends and is the same length as the paper roll. The paper acts as a distributed load over the length of the shaft. Solution: 1. See Mathcad file P0408.

The weight of the paper roll is equal to its volume times the paper density times g. Wroll

OD ID L g Wroll L

Wroll 53.89 kN w 16.686 N mm

2.

The intensity of the distributed load is w

3.

Using Figure B-2(b) in Appendix B with a = 0, the maximum deflection is at the midspan and is y = w x 24 E I 2 L x x L

For x = L/2, this reduces to

y =

5 w L

384 E I I 5 w L
4

Letting = -y and solving for I, we have

384 E

I 3.808 10 mm

4.

The area moment of inertia for a hollow circular cross-section is I =

64

od id

Solving this for the id yields

4 od4 64 I id

id 198.954 mm

Round this down (for slightly less deflection) to

id 198 mm

MACHINE DESIGN - An Integrated Approach, 4th Ed.

4-9-1

PROBLEM 4-9
Statement: A ViseGrip plier-wrench is drawn to scale in Figure P4-3, and for which the forces were analyzed in Problem 3-9, find the stresses in each pin for an assumed clamping force of P = 4000 N in the position shown. The pins are 8-mm dia and are all in double shear. Pin forces as calculated in Problem 3-9: Member 1 F21 7.5 kN Member 2 Member 3 Member 4 Pin diameter F12 7.5 kN F23 5.1 kN F14 5.1 kN d 8 mm F41 5.1 kN F32 5.1 kN F43 5.1 kN F34 5.1 kN

Given:

Assumptions: Links 3 and 4 are in a toggle position, i.e., the pin that joins links 3 and 4 is in line with the pins that join 1 with 4 and 2 with 3. Solution: 1. See Figure 4-9 and Mathcad file P0409.

The FBDs of the assembly and each individual link are shown in Figure 4-9. The dimensions, as scaled from Figure P4-3 in the text, are shown on the link FBDs.
F 4 P 1

3 F
55.0 = b 50.0 = a 22.0 = d

2 P

F14

39.5 = c

129.2

4 F34 P

F41

F21

28.0 = e

2.8 = g

F43 3 F23 F

F12 F32 2

21.2 = h

26.9 = f

FIGURE 4-9
Free Body Diagrams for Problem 4-9

2.

The cross-sectional area for all pins is the same and is

d
4

A 50.265 mm

MACHINE DESIGN - An Integrated Approach, 4th Ed.


3.

4-9-2

The pin that joins members 1 and 2 is the most highly stressed while the stress on each of the remaining pins is the same. Since the pins are in double shear, we will divide the pin load by 2 in each case. F12 2 A F14 2 A

Pin joining 1 and 2

12

12 74.6 MPa

All other pins

14

14 50.7 MPa

MACHINE DESIGN - An Integrated Approach, 4th Ed.

4-10-1

PROBLEM 4-10
Statement: The over-hung diving board of problem 3-10 is shown in Figure P4-4a. Assume cross-section dimensions of 305 mm x 32 mm. The material has E = 10.3 GPa. Find the largest principal stress at any location in the board when a 100-kg person is standing at the free end. Weight of person Board dimensions Distance to support Length of board Cross-section a 0.7 m L 2 m w 305 mm t 32 mm
700 = a R2

Given:

W 100 kgf
R1

2000 = L P

FIGURE 4-10 Assumptions: The weight of the beam is negligible compared to the applied load and so can be ignored. Solution: 1. See Figure 4-10 and Mathcad file P0410.
Free Body Diagram for Problem 4-10

From the FBD of the diving board and Figure B-3 (Appendix B), the reactions at the supports are R1 W 1

R1 1821 N R2 2802 N

R2 W 2.

Also from Figure D-3, the maximum bending moment occurs at the right-hand support where, in the FBD above, x = a. Mmax R1 a Mmax 1275 N m

3.

The maximum bending stress will occur on the top and bottom surfaces of the board at the section where the maximum bending moment occurs which, in this case, is at x = a. The only stress present on the top or bottom surface of the board is the bending stress x. Therefore, on the top surface where the stress is tensile, x. is the principal stress 1 . Thus, Distance to extreme fiber c I t 2 w t
3

c 16 mm I 8.329 10 mm
5 4

Moment of inertia

12 Mmax c I

Bending stress

x 24.492 MPa 1 24.5 MPa

Maximum principal stress

1 x

MACHINE DESIGN - An Integrated Approach, 4th Ed.

4-11-1

PROBLEM 4-11
Statement: Repeat Problem 4-10 using the loading conditions of Problem 3-11. Assume the board weighs 29 kg and deflects 13.1 cm statically when the person stands on it. Find the largest principal stress at any location in the board when the 100-kg person in Problem 4-10 jumps up 25 cm and lands back on the board. Find the maximum deflection. Beam length Distance to support Mass of person Mass of board Static deflection Height of jump Cross-section L 2000 mm a 700 mm mpers 100 kg mboard 29 kg
R1 2000 = L Fi

Given:

st 131 mm
h 250 mm w 305 mm t 32 mm
700 = a

R2

FIGURE 4-11
Free Body Diagram for Problem 4-11

Assumptions: The apparent Young's modulus for fiberglas is E 1.03 10 MPa Solution: See Figure 4-11 and Mathcad file P0411.
4

1. From Problem 3-11, the dynamic load resulting from the impact of the person with the board is i 3.056 kN F 2. From the FBD of the diving board and Figure B-3(a) (Appendix B), the reactions at the supports are R1 Fi 1 L a

R1 5.675 kN R2 8.731 kN

R2 Fi

3. Also from Figure D-3(a), the maximum bending moment occurs at the right-hand support where, in the FBD above, x = a. Mmax R1 a Mmax 3.973 kN m

4. The maximum bending stress will occur on the top and bottom surfaces of the board at the section where the maximum bending moment occurs which, in this case, is at x = a. The only stress present on the top or bottom surface of the board is the bending stress x. Therefore, on the top surface where the stress is tensile, x is the principal stress 1 . Thus, Distance to extreme fiber c I t 2 w t
3

c 16 mm I 8.329 10 mm
5 4

Moment of inertia

12 Mmax c I

Bending stress Maximum principal stress

x 76.322 MPa 1 76.3 MPa

1 x

5. Calculate the maximum deflection from the equation given in Figure D-3(a) at x = L. Let b in the figure be a in our problem and let a in the figure be equal to L, then ymax Fi 6 a E I ( a L) L L ( L a ) a ( L a ) L
3 3 2

ymax 401.4 mm

MACHINE DESIGN - An Integrated Approach, 4th Ed.

4-12-1

PROBLEM 4-12
Statement: Given: Repeat Problem 4-10 using the cantilevered diving board design in Figure P4-4b. Beam length Weight at free end Cross-section L 1300 mm P 100 kgf w 305 mm t 32 mm Assumptions: The apparent Young's modulus for fiberglas is E 1.03 10 MPa Solution: See Figure 4-12 and Mathcad file P0412.
4

2000 1300 = L P

M1 700

R1

FIGURE 4-12
Free Body Diagram for Problem 4-12

1.

From the FBD of the diving board and Figure B-1(a) (Appendix B), the reactions at the supports are R1 P M1 P L R1 981 N M1 1275 N m

2.

Also from Figure D-1, the maximum bending moment occurs at the support where, in the FBD above, x = 0. Mmax M1 Mmax 1275 N m

3. The maximum bending stress will occur on the top and bottom surfaces of the board at the section where the maximum bending moment occurs which, in this case, is at x = 0. The only stress present on the top or bottom surface of the board is the bending stress x. Therefore, on the top surface where the stress is tensile, x is the principal stress 1 . Thus, Distance to extreme fiber c I t 2 w t
3

c 16 mm I 8.329 10 mm
5 4

Moment of inertia

12 Mmax c I

Bending stress

x 24.492 MPa 1 24.5 MPa

Maximum principal stress

1 x

MACHINE DESIGN - An Integrated Approach, 4th Ed.

4-13-1

PROBLEM 4-13
Statement: Repeat Problem 4-11 using the diving board design shown in Figure P4-4b. Assume the board weighs 19 kg and deflects 8.5 cm statically when the person stands on it. Unsupported length Mass of board Static board deflection Mass of person Height of jump Cross-section L 1300 mm mboard 19 kg
2000 1300 = L Fi

Given:

stat 85 mm
mperson 100 kg h 250 mm w 305 mm t 32 mm
M1 700 R1

Assumptions: The apparent Young's modulus for fiberglas is E 1.03 10 MPa Solution: 1. 2. See Figure 4-13 and Mathcad file P0413.
4

FIGURE 4-13
Free Body Diagram for Problem 4-13

From Problem 3-13, the dynamic load resulting from the impact of the person with the board is i 3.487 kN F From the FBD of the diving board and Figure B-1(a) (Appendix B), the reactions at the supports are R1 Fi M1 Fi L R1 3487 N M1 4533 N m

3.

Also from Figure D-1, the maximum bending moment occurs at the support where, in the FBD above, x = 0. Mmax M1 Mmax 4533 N m

4. The maximum bending stress will occur on the top and bottom surfaces of the board at the section where the maximum bending moment occurs which, in this case, is at x = 0. The only stress present on the top or bottom surface of the board is the bending stress x. Therefore, on the top surface where the stress is tensile, x is the principal stress 1 . Thus, Distance to extreme fiber c I t 2 w t
3

c 16 mm I 8.329 10 mm
5 4

Moment of inertia

12 Mmax c I

Bending stress

x 87.086 MPa 1 87.1 MPa

Maximum principal stress 5.

1 x

Calculate the maximum deflection from the equation given in Figure D-3(a) at x = L. Let b in the figure be a in our problem and let a in the figure be equal to L, then Fi L
3

ymax

3 E I

ymax 297.7 mm

MACHINE DESIGN - An Integrated Approach, 4th Ed.

4-14-1

PROBLEM 4-14
Statement: Figure P4-5 shows a child's toy called a pogo stick. The child stands on the pads, applying half her weight on each side. She jumps off the ground, holding the pads up against her feet, and bounces along with the spring cushioning the impact and storing energy to help each rebound. Design the aluminum cantilever beam sections on which she stands to survive jumping 2 in off the ground. Assume an allowable stress of 20 ksi. Define and size the beam shape. Allowable stress Young's modulus

Given:

allow 20 ksi
E 10.3 10 psi
6

Assumptions: The beam will have a rectangular cross-section with the load applied at a distance of 5 in from the central support. L 5 in Solution: See Figure 4-14 and Mathcad file P0414.

1. From Problem 3-14, the total dynamic force on both foot supports is Fi 224 lbf Therefore, the load on each support is P Fi 2 P 112 lbf

Fi /2

Fi /2

2. To give adequate support to the childs foot, let the width of the support beam be w 1.5 in 3. From Figure B-1(a) in Appendix B, the maximum bending moment at x = 0 is M P L 4. M 560 in lbf FIGURE 4-14
Free Body Diagram for Problem 4-14

We can now calculate the minimum required section modulus, Z = I/c. M Z

Bending stress Solving for Z,

=
Z

= allow M Z 458.8 mm
3

allow
3

5.

For a rectangular cross-section, I =

w t

12

and c =

t 2

so Z =

w t 6

Solving for t,

6 Z w

t 0.335 in t 0.375 in

Round this up to the next higher decimal equivalent of a common fraction,

MACHINE DESIGN - An Integrated Approach, 4th Ed.

4-15-1

PROBLEM 4-15
Statement: Design a shear pin for the propeller shaft of an outboard motor if the shaft through which the pin is placed is 25-mm diameter, the propeller is 20-cm diameter, and the pin must fail when a force > 400 N is applied to the propeller tip. Assume an ultimate shear strength for the pin material of 100 MPa. Propeller shaft dia Propeller dia Max propeller tip force Ultimate shear strength d 25 mm D 200 mm Fmax 400 N S us 100 MPa
T Fpin

Given:

Propeller Hub

Shear Pin

Assumptions: A shear pin is in direct, double shear. Solution: See Figure 4-15 and Mathcad file P0415.
Fpin d Propeller Shaft

1. Calculate the torque on the propeller shaft that will result from a tip force on the propeller of Fmax. T Fmax 2. D 2 T 40000 N mm

FIGURE 4-15
Free Body Diagram for Problem 4-15

This will be reacted by the shear pin's couple on the shaft. Determine the magnitude of the direct shear force. Fpin T d Fpin 1600 N

3.

Determine the maximum pin diameter that will shear at this force. Fpin A 4 Fpin

Direct shear stress

d pin
4 Fpin

= S us

Solving for the pin diameter

d pin

S us

d pin 4.514 mm

Round this to

d pin 4.5 mm

MACHINE DESIGN - An Integrated Approach, 4th Ed.

4-16-1

PROBLEM 4-16
Statement: A track to guide bowling balls is designed with two round rods as shown in Figure P4-6. The rods are not parallel to one another but have a small angle between them. The balls roll on the rods until they fall between them and drop onto another track. The angle between the rods is varied to cause the ball to drop at different locations. Find the maximum stress and deflection in the rods assuming that they are (a) simply supported at each end, and (b) fixed at each end. Rod length Rod diameter Distance to load Young's modulus L 30 in d 1.00 in a 23.15 in E 30 10 psi
6
R1 L R2 a Fball

Given:

Assumptions: The analysis of Problem 3-16 yielded the following for a simply supported beam: Max ball load Max moment Reactions Fball 13.89 lbf Mmax 73.4 in lbf R1 3.17 lbf R2 10.72 lbf Solution: See Figure 4-16 and Mathcad file P0416.

FIGURE 4-16A
Free Body Diagram for Problem 4-16(a), taken on a plane through the rod axis and ball center

1. The maximum bending stress will occur at the outer fibers of the rod at the section where the maximum bending moment occurs which, in this case, is at x = a. The only stress present on the top or bottom surface of the rod is the bending stress x. Therefore, on the bottom surface where the stress is tensile, x is the principal stress 1 . Thus, for a simply supported rod, Distance to extreme fiber c I d 2 c 0.5 in
4

Moment of inertia

d
64

I 0.0491 in

Bending stress Maximum principal stress 2.

Mmax c I

x 748 psi 1 748 psi

1 x

Calculate the maximum deflection for the simply supported case from the equation given in Figure D-2(a), ymax Fball 6 E I 2 a
3

L a

ymax 0.0013 in
a Fball

3. For the case where the rod is built in at each end, the beam is statically indeterminate. As seen in Figure 4-16B, there are four unknown reactions and only two equilibrium equations can be written using statics. We will find the reactions using Example 4-7 as a model.

M1

R1

R 2 M2

FIGURE 4-16B
Free Body Diagram for Problem 4-16(b), taken on a plane through the rod axis and ball center

MACHINE DESIGN - An Integrated Approach, 4th Ed.

4-16-2

4. Write an equation for the load function in terms of equations 3.17 and integrate the resulting function four times using equations 3.18 to obtain the shear and moment functions. Note use of the unit doublet function to represent the moment at the wall. For the beam in Figure 4-16B, q(x) = -M1<x - 0>-2 + R1<x - 0>-1 - F<x - a>-1 + R2<x - L>-1 + M2<x - L>-2 V(x) = -M1<x - 0>-1 + R1<x - 0>0 - F<x - a>0
+ R2<x

- L>0 + M2<x - L>-1 + C1

M(x) = -M1<x - 0>0 + R1<x - 0>1 - F<x - a>1 + R2<x - L>1 + M2<x - L>0 + C1x+ C2 (x) = ( -M1<x - 0>1 + R1<x - 0>2/2 - F<x - a>2/2
+ R2<x

- L>2/2 + M2<x - L>1 + C1x2/2 + C2x + C3) / EI

y(x) = ( -M1<x - 0>2/2 + R1<x - 0>3/6 - F<x - a>3/6 + R2<x - L>3/6 + M2<x - L>2 /2+ C1x3/6 + C2x2/2 + C3x + C4) / EI 5. Because the reactions have been included in the loading function, the shear and moment diagrams both close to zero at each end of the beam, making C1 = C2 = 0. This leaves six unknowns; the four reactions and the constants of integration, C3 and C4. There are four boundary conditions that we can use and two equilibrium equations. The boundary conditions are: at x = 0, = 0 and y = 0; and at x = L, = 0 and y = 0. Applying the boundary conditions at x = 0 results in C3 = C4 = 0. Applying the BCs at x = L results in the following two equations, which are solved for R1 and M1. At x = L,

=0

0=

R1 2 R1 6

L M 1 L L
3

F 2

( L a) F 6

y =0

0=

M1 2

( L a)

Solving these two equations simultaneously for R1 and M1, M1 Fball L M1 L ( L a )


2

( L a) L
2

M1 16.765 in lbf

R1 2

Fball

( L a) L
2

R1 1.842 lbf

6.

The remaing two reactions can be found by using the equations of equilibrium.

Fy = 0: M = 0:

R1 Fball R2 = 0 M1 Fball a R2 L M2 = 0

Solving these two equations simultaneously for R2 and M2, R2 Fball R1 M2 M1 Fball a R2 L 7. Define the range for x, x 0 in 0.005 L L R2 12.048 lbf M2 56.657 in lbf

8. For a Mathcad solution, define a step function S. This function will have a value of zero when x is less than z, and a value of one when it is greater than or equal to z. S ( x z) if ( x z 1 0 ) 9. Write the shear, moment, slope, and deflection equations in Mathcad form, using the function S as a multiplying factor to get the effect of the singularity functions.

MACHINE DESIGN - An Integrated Approach, 4th Ed.


V ( x) R1 S ( x 0 mm) Fball S ( x a ) R2 S ( x L) M ( x) M1 S ( x 0 mm) R1 S ( x 0 mm) ( x 0 mm) Fball S ( x a ) ( x a ) M2 S ( x L) R2 S ( x L) ( x L) 1 E I

4-16-3

( x)

M1 S ( x 0 mm) x

R1 2

S ( x 0 mm) ( x 0 mm)

Fball 2

R M2 S( x L) ( x L) 2 S( x L) ( x L) 2 2
S ( x 0 mm) x
2

S ( x a ) ( x a )

y ( x)

1 E I

M1
2

R1 6

S ( x 0 mm) ( x 0 mm)

Fball 6

M R 2 S ( x L) ( x L) 2 2 S ( x L) ( x L) 3 6 2

S ( x a ) ( x a )

10. Plot the shear, moment, slope, and deflection diagrams. (a) Shear Diagram
5

(b) Moment Diagram


40 20 Moment, M - lb in 0 20 40 60

0 Shear, V - lb

10

15

10

20

30

10

20

30

Distance along beam, x - in

Distance along beam, x - in

(c) Slope Diagram


0.1 Deflection - thousandths of in Slope - Thousands of Rad

(d) Deflection Diagram


0

0.2

0.4

0.6

0.1

10

20

30

0.8

10

20

30

Distance along beam, x - in

Distance along beam, x - in

FIGURE 4-16C
Shear, Moment, Slope, and Deflection Diagrams for Problem 4-16(b)

MACHINE DESIGN - An Integrated Approach, 4th Ed.


11 The maximum moment occurs at x = L and is Mmax M2 Mmax 56.7 in lbf

4-16-4

12 Calculate the maximum bending and principal stresses. Bending stress Maximum principal stress

Mmax c I

x 577 psi 1 577 psi

1 x

13. To find the maximum deflection, first determine at what point on the beam the slope is zero. Let this be at x = e. From the slope diagram, we see that e < a. Using the slope equation and setting it equal to zero, we have For = 0 0 = M1 e e 2 M1 R1 R1 2 e 2 e 18.204 in ymax 0.00063 in

Solving for e

Maximum deflection

ymax y ( e)

MACHINE DESIGN - An Integrated Approach, 4th Ed.

4-17-1

PROBLEM 4-17
Statement: A pair of ice tongs is shown in Figure P4-7. The ice weighs 50 lb and is 10 in wide across the tongs. The distance between the handles is 4 in, and the mean radius r of the tong is 6 in. The rectangular cross-sectional dimensions are 0.75 x 0.312 in. Find the stress in the tongs. Mean radius of tong Tong width Tong depth Assumptions: beam. rc 6.00 in w 0.312 in h 0.75 in

Given:

F C FC O
11.0 = ax 3.5 = cy

The tong can be analyzed as a curved

FO
2.0 = cx 12.0 = by 5.0 = bx

See Problem 3-17, Figure 4-17, Solution: and Mathcad file P0417. 1. The maximum bending moment and axial force in the tong were found in Problem 3-17 at point A. They are

FB B

Maximum moment Axial force at D

MA 237.5 in lbf FAn 25 lbf

W/2
FIGURE 4-17
Free Body Diagram for Problem 4-17

2.

Calculate the section area, inside radius and outside radus. Area of section Inside and outside radii of section A h w ri rc 0.5 h ro rc 0.5 h A 0.234 in ri 5.625 in ro 6.375 in
2

3.

Use the equation in the footnote on page 195 of the text to calculate the radius of the neutral axis. Radius of neutral axis rn ro ri

ro ln ri

rn 5.992 in

4.

Calculate the eccentricty and the distances from the neutral axis to the extreme fibers. Eccentricity Distances from neutral axis to extreme fibers e rc rn ci rn ri co ro rn e 0.007821 in ci 0.3672 in co 0.3828 in

Stresses at inner and outer radii

MA c i FAn e A ri A

i 8.58 ksi

o
5.

MA co FAn A e A ro

o 7.69 ksi

The shear stress is zero at the outer fibers. Therefore, these are the principal stresses. At the inner surface

1 i

1 8.58 ksi

2 0 ksi

3 0 ksi

MACHINE DESIGN - An Integrated Approach, 4th Ed.

4-18-1

PROBLEM 4-18
Statement: A set of steel reinforcing rods is to be stretched axially in tension to create a tensile stress of 30 ksi prior to being cast in concrete to form a beam. Determine how much force will be required to stretch them the required amount and how much deflection is required. There are 10 rods; each is 0.75-in diameter and 30 ft long. Desired stress Number of rods Rod length

Given:

30 ksi
Nrods 10 L 30 ft

Rod diameter Young's modulus

d 0.75 in E 30 10 psi
6

Assumptions: The rods share the load equally. Solution: 1. 2. See Mathcad file P0418.

Calculate the cross-sectional area of one rod. A

d
4

A 0.442 in

Determine the force required to achieve the desired stress level in one rod.

=
3.

F A

F A

F 13.254 kip

Determine the total force required to achieve the desired stress level in all rods. Ftotal Nrods F Ftotal 132.5 kip

4.

Determine the amount the rods will deflect under the applied load.

F L A E

0.360 in

MACHINE DESIGN - An Integrated Approach, 4th Ed.

4-19-1

PROBLEM 4-19
Statement: The clamping fixture used to pull the rods in Problem 4-18 is conected to the hydraulic ram by a clevis like that shown in Figure P4-8. Determine the size of the clevis pin needed to withstand the applied force. Assume an allowable shear stress of 20 000 psi and an allowable normal stress of 40 000 psi. Determine the required outside radius of the clevis end to not exceed the above allowable stresses in either tear out or bearing if the clevis flanges are each 0.8 in thick. Desired rod stress rod 30 ksi Number of rods Rod length Clevis strength Nrods 10 L 30 ft S sallow 20 ksi S ballow 40 ksi Assumptions: The rods share the load equally, and there is one clevis for all ten rods. Solution: 1. 2. See Figures 4-12 and 4-13 in the text, Figure 4-19, and Mathcad file P0419. A Rod diameter d 0.75 in E 30 10 psi t 0.8 in
6

Given:

Young's modulus Clevis flange thickness

Calculate the cross-sectional area of one rod.

d
4

A 0.442 in

Determine the force required to achieve the desired stress level in one rod. F rod = F rod A F 13.254 kip A Determine the total force required to achieve the desired stress level in all rods. Ftotal Nrods F Ftotal 132.5 kip

3.

This force is transmitted through the clevis pin, which is in double shear. 4. Calculate the minimum required clevis pin diameter for the allowable shear stress. Divide the load by 2 because of the double shear loading.

pin =

Ftotal 2 Apin

2 Ftotal

= S sallow

Solving for the pin diameter

2 Ftotal

S sallow

d 2.054 in

Round this up to the next higher decimal equivalent of a common fraction ( 2 1/8) 5. Check the bearing stress in the clevis due to the pin on one side of the clevis. Bearing stress area Bearing force Ab d t Fb Ftotal 2 Fb Ab Ab 1.700 in
2

d 2.125 in

Fb 66.268 kip

Bearing stress

b 39.0 ksi

Since this is less than S ballow, this pin diameter is acceptable.

MACHINE DESIGN - An Integrated Approach, 4th Ed.


6. Determine the tearout stress in the clevis. Shear area (see Figure 4-19) Shear force Ftear Ftotal 2 Ftear 66.268 kip Atear = 2 t R ( 0.5 d )
2 2
Tearout length

4-19-2

Shear stress and strength


d R

Ftear Atear

Ftear 2 t R ( 0.5 d )
2 2

= S sallow

FIGURE 4-19
Tearout Diagram for Problem 4-19

Solving for the clevis radius, R

Ftear 2 R ( 0.5 d) 2 t S sallow

R 2.328 in

Round this up to the next higher decimal equivalent of a common fraction ( 2 3/8)

R 2.375 in

MACHINE DESIGN - An Integrated Approach, 4th Ed.

4-20-1

PROBLEM 4-20
Statement: Repeat Problem 4-19 for 12 rods, each 1 cm in diameter and 10 m long. The desired rod stress is 20 MPa. The allowable normal stress in the clevis and pin is 280 MPa and their allowable shear stress is 140 MPa. Each clevis flange is 2 cm wide. kN 10 newton
3

Units: Given:

MPa 10 Pa Nrods 12 L 10 m S sallow 140 MPa S ballow 280 MPa

GPa 10 Pa Rod diameter Young's modulus Clevis flange thickness d 10 mm E 207 GPa t 20 mm

Desired rod stress rod 200 MPa Number of rods Rod length Clevis strength

Assumptions: The rods share the load equally, and there is one clevis for all twelve rods. Solution: 1. 2. See Figures 4-12 and 4-13 in the text, Figure 4-20, and Mathcad file P0420. A

Calculate the cross-sectional area of one rod.

d
4

A 78.54 mm

Determine the force required to achieve the desired stress level in one rod.

rod =
3.

F A

F rod A

F 15.708 kN

Determine the total force required to achieve the desired stress level in all rods. Ftotal Nrods F Ftotal 188.5 kN

This force is transmitted through the clevis pin, which is in double shear. 4. Calculate the minimum required clevis pin diameter for the allowable shear stress. Divide the load by 2 because of the double shear loading.

pin =

Ftotal 2 Apin

2 Ftotal

= S sallow

Solving for the pin diameter

2 Ftotal

S sallow
d 30 mm

d 29.277 mm

Round this up to the next higher even mm 5.

Check the bearing stress in the clevis due to the pin on one side of the clevis. Bearing stress area Bearing force Ab d t Fb Ftotal 2 Fb Ab Ab 600 mm
2

Fb 94.248 kN

Bearing stress

b 157.1 MPa

Since this is less than S ballow, this pin diameter is acceptable.

MACHINE DESIGN - An Integrated Approach, 4th Ed.


6. Determine the tearout stress in the clevis. Shear area (see Figure 4-19) Shear force Ftear Ftotal 2 Ftear 94.248 kN Atear = 2 t R ( 0.5 d )
2 2
Tearout length

4-20-2

Shear stress and strength


d R

Ftear Atear

Ftear 2 t R ( 0.5 d )
2 2

= S sallow

FIGURE 4-20
Tearout Diagram for Problem 4-20

Solving for the clevis radius, R

Ftear 2 R ( 0.5 d) 2 t S sallow


R 24 mm

R 22.544 mm

Round this up to the next higher even mm

MACHINE DESIGN - An Integrated Approach, 4th Ed.

4-21-1

PROBLEM 4-21
Statement: Figure P4-9 shows an automobile wheel with two common styles of lug wrench being used to tighten the wheel nuts, a single-ended wrench in (a), and a double-ended wrench in (b). In each case two hands are required to provide forces respectively at A and B as shown. The distance between points A and B is 1 ft in both cases and the handle diameter is 0.625 in. The wheel nuts require a torque of 70 ft-lb. Find the maximum principle stress and maximum deflection in each wrench design. Distance between A and B Tightening torque Wrench diameter d AB 1 ft T 70 ft lbf d 0.625 in

Given:

Assumptions: 1. The forces exerted by the user's hands lie in a plane through the wrench that is also parallel to the plane of the wheel. 2. The applied torque is perpendicular to the plane of the forces. 3. By virtue of 1 and 2 above, this is a planar problem that can be described in a 2D FBD. Solution: See Figure 4-21 and Mathcad file P0421.
12" = dAB F

1. In Problem 3-21 we found that for both cases F 70 lbf 2. From examination of the FBDs, we see that, in both cases, the arms are in bending and the stub that holds the socket wrench is in pure torsion. The maximum bending stress in the arm will occur near the point where the arm transitions to the stub. The stress state at this transition is very complicated, but we can find the nominal bending stress there by treating the arm as a cantilever beam, fixed at the transition point. For both cases the torque in the stub is the same.

T F (a) Single-ended Wrench

12" = dAB 6" F

Case (a)
T

2. The bending moment at the transition is


F

Ma F d AB

Ma 840 lbf in FIGURE 4-21

(b) Double-ended Wrench

3. The tensile stress at this point is found from

Free Body Diagrams for Problem 4-21 4

Moment of inertia Dist to extreme fibre Stress 4.

d
64

I 0.00749 in c 0.313 in

c 0.5 d

M a c I

x 35.05 ksi

There are no other stress components present at this point, so x is the maximum principle stress here and

1 x
5. 6. The torque in the stub is

1 35.0 ksi
T 840 in lbf

2 0 psi

3 0 psi

The shear stress at any point on the outside surface of the stub is found from

MACHINE DESIGN - An Integrated Approach, 4th Ed.


Polar moment of inertia Shear stress 7. J 2 I J 0.0150 in J
4

4-21-2

xy

Tc

xy 17.52 ksi

There are no other stress components present along the outside surface of the stub, so

1 xy
8.

1 17.5 ksi

2 0 psi

3 1

Thus, the maximum principle stress for case (a) is on the upper surface of the handle (arm) near the point where it transitions to the stub. There will be two deflection components that we can calculate separately and then add (superposition). One will come from the bending of the arm and one will come from the twisting of the stub, projected out to the end of the arm.

9.

Deflection of the arm due to bending only for a stub length of stub 3 in: Assuming that the wrenches are made from steel E 30 10 psi yarm F d AB
3 6

G 11.7 10 psi yarm 0.179 in

From Figure B-1(a), Appendix B, From equation (4.24), the angular twist of the stub is The deflection at the end of the arm due to the stub twist is So, the total deflection is

3 E I T stub J G

stub

stub 0.014 rad


ystub 0.173 in ya 0.352 in

ystub d AB stub ya yarm ystub

Case (b) 10. The bending moment at the transition is 11. The tensile stress at this point is found from Stress Mb F d AB 2 Mb 420 lbf in

M b c I

x 17.52 ksi

12. There are no other stress components present at this point, so x is the maximum principle stress here and

1 x
13. The torque in the stub is 14.

1 17.5 ksi
T 840 in lbf

2 0 psi

3 0 psi

The shear stress at any point on the outside surface of the stub is found from Shear stress

xy

Tc J

xy 17.52 ksi

15. There are no other stress components present along the outside surface of the stub, so

1 xy

1 17.5 ksi

2 0 psi

3 1

MACHINE DESIGN - An Integrated Approach, 4th Ed.

4-21-3

16. Thus, the maximum principle stress for case (b) is the same on the upper surface of the handle (arm) near the point where it transitions to the stub, and on the outside surface of the stub. There will be two deflection components that we can calculate separately and then add (superposition). One will come from the bending of the arm and one will come from the twisting of the stub, projected out to the end of the arm. Deflection of the arm due to bending only: From Figure B-1(a), Appendix B, yarm F 0.5 d AB 3 E I T stub J G d AB 2 stub
3

yarm 0.022 in

From equation (4.24), the angular twist of the stub is The deflection at the end of the arm due to the stub twist is So, the total deflection is

stub

stub 0.014 rad

ystub

ystub 0.086 in

yb yarm ystub

yb 0.109 in

MACHINE DESIGN - An Integrated Approach, 4th Ed.

4-22-1

PROBLEM 4-22
Statement: A roller-blade skate is shown in Figure P4-10. The polyurethane wheels are 72 mm dia. The skate-boot-foot combination weighs 2 kg. The effective "spring rate" of the person-skate subsystem is 6000 N/m. The axles are 10-mm-dia steel pins in double shear. Find the stress in the pins for a 100-kg person landing a 0.5-m jump on one foot. (a) Assume all 4 wheels land simultaneously. (b) Assume that one wheel absorbs all the landing force. Axle pin diameter d 10 mm

Given: Solution: 1. 2.

See Figure P4-10 and Mathcad file P0422. Fa 897 N Fb 3.59 kN

From Problem 3-22, we have the forces for cases (a) and (b):

In both cases, this is the force on one axle. The shear force will be one half of these forces because the pins are in double shear. Shear area Shear stress Case (a) all wheels landing As

d
4

As 78.54 mm

Fa 2 As Fb 2 As

a 5.71 MPa

Case (b) one wheel landing

b 22.9 MPa

MACHINE DESIGN - An Integrated Approach, 4th Ed.

4-23a-1

PROBLEM 4-23a
Statement: A beam is supported and loaded as shown in Figure P4-11a. Find the reactions, maximum shear, maximum moment, maximum slope, maximum bending stress, and maximum deflection for the data given in row a from Table P4-2. Beam length Distance to distributed load L 1 m a 0.4 m w 200 N m F 500 N I 2.85 10
8 1
a w L b F

Given:

Distance to concentrated load b 0.6 m Distributed load magnitude Concentrated load Moment of inertia

R1

R2

Distance to extreme fiber c 2.00 10 Solution: 1.

FIGURE 4-23A
Free Body Diagram for Problem 4-23

See Figures 4-23 and Mathcad file P0423a.

The reactions, maximum shear and maximum moment were all found in Problem 3-23a. Those results are summarized here. Load function Shear function Moment function Modulus of elasticity Reactions Maximum shear Maximum moment q(x) = R1<x - 0>-1 - w<x - 0>0 + w<x - a>0 - F<x - b>-1 + R2<x - L>-1 V(x) = R1<x - 0>0 - w<x - 0>1 + w<x - a>1 - F<x - b>0 + R2<x - L>0 M(x) = R1<x - 0>1 - w<x - 0>2/2 + w<x - a>2/2 - F<x - b>1 + R2<x - L>1 E 207 GPa R1 264.0 N Vmax 316 N Mmax 126.4 N m R2 316.0 N (negative, from x = b to x =L) (at x = b)

2.

Integrate the moment function, multiplying by 1/EI, to get the slope. (x) = [R1<x>2/2 - w<x>3/6 + w<x - a>3/6 - F<x - b>2/2 + R2<x - L>2/2 + C3]/EI

3.

Integrate again to get the deflection. y(x) = [R1<x>3/6 - w<x>4/24 + w<x - a>4/24 - F<x - b>3/6 + R2<x-L>3/6 + C3x +C4]/EI

4.

Evaluate C3 and C4 At x = 0 and x = L, y = 0, therefore, C4 = 0. 0= R1 6 L


3

w 24

w 24

( L a)

F 6

( L b ) C 3 L

C3 5. 6.

R1 3 w 4 w F 4 3 L L ( L a) ( L b) L 6 24 24 6
1 x 0 m 0.005 L L

C3 31.413 N m

Define the range for x

For a Mathcad solution, define a step function S. This function will have a value of zero when x is less than z, and a value of one when it is greater than or equal to z. S ( x z) if ( x z 1 0 )

7.

Write the slope and deflection equations in Mathcad form, using the function S as a multiplying factor to get the effect of the singularity functions. See Figure 4-23aB where these functions are plotted.

MACHINE DESIGN - An Integrated Approach, 4th Ed.

4-23a-2
w 6 S ( x a ) ( x a )
3

( x)

1 E I

R1
2

S ( x 0 in) x

w 6

S ( x 0 in) x

R 2 S ( x L) ( x L) 2 F S( x b) ( x b ) 2 C3 2 2
S ( x 0 in) x
3

y ( x)

1 E I

R1
6

w 24

S ( x 0 in) x

w 24

R 2 S( x L) ( x L) 3 F S ( x b ) ( x b) 3 C3 x 6 6
max ( L)

S ( x a ) ( x a )

8. Maximum slope occurs at x = L

max 0.335 deg

9. Maximum deflection occurs at x = c, where = 0 and c < b.

0 =
Solving for c, A R1 2

R1 2 w 3 w 3 c c ( c a ) C3 = 0 E I 2 6 6
1 w 6 a B 3 w 6 a
2

C C3

w 6

3 2

A 92.000 N c B B 4 A C 2 A
2

B 16.000 N m c 0.523 m ymax y ( c)

C 33.547 N m

Substituting c into the deflection equation, SLOPE, radians


0.01

ymax 1.82 mm DEFECTION, mm


0 0.5

0.005 y ( x) 0 0.005 0.01 mm

( x)

1 1.5 2

0.2

0.4 x m

0.6

0.8

0.2

0.4 x m

0.6

0.8

FIGURE 4-23aB
Slope and Deflection Diagrams for Problem 4-23a

10. The maximum bending stress occurs at x = b, where the moment is a maximum. For c 2.00 10
2

c 20 mm

max

Mmax c I

max 88.7 MPa

MACHINE DESIGN - An Integrated Approach, 4th Ed.

4-24a-1

PROBLEM 4-24a
Statement: A beam is supported and loaded as shown in Figure P4-11b. Find the reactions, maximum shear, maximum moment, maximum slope, maximum bending stress, and maximum deflection for the data given in row a from Table P4-2. Beam length Distance to distributed load L 1 m a 0.4 m w 200 N m F 500 N I 2.85 10
8 1
M1 a F w L

Given:

Distance to concentrated load b 0.6 m Distributed load magnitude Concentrated load Moment of inertia

R1

Distance to extreme fiber c 2.00 10 Solution: 1.

FIGURE 4-24A
Free Body Diagram for Problem 4-24

See Figures 4-24 and Mathcad file P0424a.

The reactions, maximum shear and maximum moment were all found in Problem 3-24a. Those results are summarized here. Load function Shear function Moment function Modulus of elasticity Reactions Maximum shear Maximum moment q(x) = -M1<x - 0>-2 + R1<x - 0>-1 - w<x - a>0 - F<x - L>-1 V(x) = -M1<x - 0>-1 + R1<x - 0>0 - w<x - a>1 - F<x - L>0 M(x) = -M1<x - 0>0 + R1<x - 0>1 - w<x - a>2/2 - F<x - L>1 E 207 GPa R1 620.0 N Vmax 620 N Mmax 584 N m M1 584.0 N m (positive, at x = 0) (negative, at x = 0)

2.

Integrate the moment function, multiplying by 1/EI, to get the slope. (x) = [-M1<x-0>1 + R1<x - 0>2/2 - w<x - a>3/6 - F<x - L>2/2 + C3]/EI

3.

Integrate again to get the deflection. y(x) = [-M1<x-0>2/2 + R1<x - 0>3/6 - w<x - a>4/24 - F<x - L>3/6 + C3x +C4]/EI

4. 5. 6.

Evaluate C3 and C4. At x = 0, = 0 and y = 0, therefore, C3 = 0 and C4 = 0. Define the range for x x 0 m 0.005 L L

For a Mathcad solution, define a step function S. This function will have a value of zero when x is less than z, and a value of one when it is greater than or equal to z. S ( x z) if ( x z 1 0 )

7.

Write the slope and deflection equations in Mathcad form, using the function S as a multiplying factor to get the effect of the singularity functions. See Figure 4-24aB where these functions are plotted.

( x)

1 E I

M1 S ( x 0 in) x

R1

2 F S( x L) ( x L) 2 2

S ( x 0 in) x

w 6

S ( x a ) ( x a )

MACHINE DESIGN - An Integrated Approach, 4th Ed.


y ( x) 1 E I

4-24a-2
3

M1

6 2F 3 S ( x L) ( x L) 6

S ( x 0 in) x

R1

S ( x 0 in) x

w 24

S ( x a ) ( x a )

8. Maximum slope occurs at x = L 9. Maximum deflection occurs at x = L

max ( L)
ymax y ( L)

max 2.73 deg


ymax 32.2 mm c 20 mm

10. The maximum bending stress occurs at x = 0, where the moment is a maximum. For

max
SLOPE, radians
0 0.01 0.02

M1 c I

max 410 MPa


DEFLECTION, mm
0

10 y ( x) mm

( x)
0.03 0.04 0.05

20

30

0.2

0.4 x m

0.6

0.8

40

0.2

0.4 x m

0.6

0.8

FIGURE 4-24aB
Slope and Deflection Diagrams for Problem 4-24a

MACHINE DESIGN - An Integrated Approach, 4th Ed.

4-25a-1

PROBLEM 4-25a
Statement: A beam is supported and loaded as shown in Figure P4-11c. Find the reactions, maximum shear, maximum moment, maximum slope, maximum bending stress, and maximum deflection for the data given in row a from Table P4-2. Beam length Distance to distributed load Distance to reaction load Distributed load magnitude Concentrated load Moment of inertia L 1 m a 0.4 m b 0.6 m w 200 N m F 500 N I 2.85 10
8 1
a w b F L

Given:

R1

R2

Distance to extreme fiber c 2.00 10 Solution: 1.

FIGURE 4-25A
Free Body Diagram for Problem 4-25

See Figures 4-25 and Mathcad file P0425a.

The reactions, maximum shear and maximum moment were all found in Problem 3-25a. Those results are summarized here. Load function Shear function Moment function Modulus of elasticity Reactions Maximum shear Maximum moment q(x) = R1<x - 0>-1 - w<x - a>0 + R2<x - b>-1 - F<x - L>-1 V(x) = R1<x - 0>0 - w<x - a>1 + R2<x - b>0 - F<x - L>0 M(x) = R1<x - 0>1 - w<x - a>2/2 + R2<x - b>1 - F<x - L>1 E 207 GPa R1 353.3 N Vmax 580 N Mmax 216 N m R2 973.3 N (positive, at x = b) (negative, at x = b)

2.

Integrate the moment function, multiplying by 1/EI, to get the slope. (x) = [R1<x - 0>2/2 - w<x - a>3/6 + R2<x - b>2/2 - F<x - L>2/2 + C3]/EI

3.

Integrate again to get the deflection. y(x) = [R1<x - 0>3/6 - w<x - a>4/24 + R2<x-b>3/6 - F<x - L>3/6 + C3x +C4]/EI

4.

Evaluate C3 and C4 At x = 0 and x = b, y = 0, therefore, C4 = 0. 0= R1 6 b


3

w 24

( b a ) C3 b C3 21.22 N m
2

1 R1 3 w 4 C3 b ( b a) b 6 24 5. 6. Define the range for x x 0 m 0.005 L L

For a Mathcad solution, define a step function S. This function will have a value of zero when x is less than z, and a value of one when it is greater than or equal to z. S ( x z) if ( x z 1 0 )

7.

Write the slope and deflection equations in Mathcad form, using the function S as a multiplying factor to get the effect of the singularity functions. See Figure 4-25aB where these functions are plotted.

MACHINE DESIGN - An Integrated Approach, 4th Ed.


1 E I

4-25a-2

( x)

R1
2

R2 F 2 2 S ( x b ) ( x b ) S ( x L) ( x L) C3 2 2
S ( x 0 in) x
2

w 6

S ( x a ) ( x a )

y ( x)

E I 6 24 R 2 S( x b) ( x b ) 3 F S ( x L) ( x L) 3 C3 x 6 6
1 S ( x 0 in) x
3

R1

S ( x a ) ( x a )

8. Maximum slope occurs at x = L 9. Maximum deflection occurs at x = L.

max ( L)
ymax y ( L)

max 0.823 deg


ymax 4.81 mm c 20 mm

10. The maximum bending stress occurs at x = b, where the moment is a maximum. For

max

Mmax c I

max 152 MPa


DEFLECTION, mm
2

SLOPE, radians
0.005

0 y ( x) mm

( x) 0.005

0.01

0.015

0.2

0.4 x m

0.6

0.8

0.2

0.4 x m

0.6

0.8

FIGURE 4-25aB
Slope and Deflection Diagrams for Problem 4-25a

MACHINE DESIGN - An Integrated Approach, 4th Ed.

4-26a-1

PROBLEM 4-26a
Statement: A beam is supported and loaded as shown in Figure P4-11d. Find the reactions, maximum shear, maximum moment, maximum slope, maximum bending stress, and maximum deflection for the data given in row a from Table P4-2. Beam length Distance to distributed load Distance to R2 Distributed load magnitude Concentrated load Moment of inertia L 1 m a 0.4 m b 0.6 m w 200 N m F 500 N I 2.85 10
8 1
a L b F w

Given:

R1

R2

R3

Distance to extreme fiber c 2.00 10 Modulus of elasticity Solution:

FIGURE 4-26A
Free Body Diagram for Problem 4-26

E 207 GPa

See Figures 4-26 and Mathcad file P0426a.

1. From inspection of Figure P4-11d, write the load function equation q(x) = R1<x>-1 - F<x - a>-1 - w<x - a>0 + R2<x - b>-1 - R3<x - L>-1 2. Integrate this equation from - to x to obtain shear, V(x) V(x) = R1<x>0 - F<x - a>0 - w<x - a>1 + R2<x - b>0 - R3<x - L>0 3. Integrate this equation from - to x to obtain moment, M(x) M(x) = R1<x>1 - F<x - a>1 - w<x - a>2/2 + R2<x - b>1 - R3<x - L>1 4. Integrate the moment function, multiplying by 1/EI, to get the slope. (x) = [R1<x>2/2 - F<x - a>2/2 - w<x - a>3/6 + R2<x - b>2/2 + R3<x - L>2/2 + C3]/EI 5. Integrate again to get the deflection. y(x) = [R1<x>3/6 - F<x - a>3/6 - w<x - a>4/24 + R2<x - b>3/6 + R3<x - L>3/6 + C3x + C4]/EI 6. Evaluate R1, R2, R3, C3 and C4 At x = 0, x = b, and x = L; y = 0, therefore, C4 = 0. At x = L+, V = M = 0 Guess Given R1 6 R1 6 b
3

R1 100 N

R2 100 N

R3 100 N

C3 5 N m

F 6 F 6

( b a)

w 24 w 24

( b a ) C3 b = 0 N m R2 6

( L a)

( L a)

( L b ) C 3 L = 0 N m

R1 F w ( L a ) R2 R3 = 0 N R 1 L F ( L a ) w 2 ( L a ) R 2 ( L b ) = 0 N m
2

MACHINE DESIGN - An Integrated Approach, 4th Ed.

4-26a-2

R1 R 2 Find R R R C 1 2 3 3 R3 C3
R1 112.33 N 7. Define the range for x R2 559.17 N R3 51.50 N C3 5.607 N m
2

x 0 in 0.002 L L

8. For a Mathcad solution, define a step function S. This function will have a value of zero when x is less than z, and a value of one when it is greater than or equal to z. S ( x z) if ( x z 1 0 ) 9. Write the shear and moment equations in Mathcad form, using the function S as a multiplying factor to get the effect of the singularity functions. V ( x) R1 S ( x 0 in) F S ( x a ) w S ( x a ) ( x a ) R2 S ( x b ) R3 S ( x L) M ( x) R1 S ( x 0 in) x F S ( x a ) ( x a ) R2 S ( x b ) ( x b ) 10. Plot the shear and moment diagrams. w 2 S ( x a ) ( x a )
2

SHEAR, N
200 60

MOMENT, N-m

0 V ( x) N M ( x) Nm

35

200

10

400

15

600 0 200 400 x mm 600 800 1 10


3

40 0 200 400 x mm 600 800 1 10


3

FIGURE 4-26aB
Shear and Moment Diagrams for Problem 4-26a

11. From the diagram, we see that maximum shear occurs at x = b -, Vmax V ( b 0.001 mm) 12. The maximum moment occurs at x = a, Mmax M ( a ) Mmax 44.9 N m Vmax 428 N

13. Write the slope and deflection equations in Mathcad form, using the function S as a multiplying factor to get t effect of the singularity functions. See Figure 4-26aB where these functions are plotted.

MACHINE DESIGN - An Integrated Approach, 4th Ed.

4-26a-3
2

( x)

1 E I

R1
2

S ( x 0 in) x

F 2

S ( x a ) ( x a )

w 6

R R 2 S ( x b ) ( x b) 2 3 S ( x L) ( x L) 2 C3 2 2
S ( x 0 in) x
3

S ( x a ) ( x a )

y ( x)

1 E I

R1
6

F 6

S ( x a ) ( x a )

w 24

R R 2 S( x b) ( x b ) 3 3 S( x L) ( x L) 3 C3 x 6 6

S ( x a ) ( x a )

14. Maximum slope occurs between x = a and x = b 15. Maximum deflection occurs between x = 0 and x = a

max 0.0576 deg


ymax 0.200 mm c 20 mm

16. The maximum bending stress occurs at x = a, where the moment is a maximum. For

max

Mmax c I

max 31.5 MPa


DEFLECTION, mm
0.1

SLOPE, deg.
0.1

0.05

0 y ( x)

( x)
deg 0

mm

0.1

0.05

0.2

0.1

0.2

0.4 x m

0.6

0.8

0.3

0.2

0.4 x m

0.6

0.8

FIGURE 4-26aC
Slope and Deflection Diagrams for Problem 4-26a

MACHINE DESIGN - An Integrated Approach, 4th Ed.

4-27-1

PROBLEM 4-27
Statement: A storage rack is to be designed to hold the paper roll of Problem 4-8 as shown in Figure P4-12. Determine suitable values for dimensions a and b in the figure. Consider bending, shear, and bearing stresses. Assume an allowable tensile/compressive stress of 100 MPa and an allowable shear stress of 50 MPa for both stanchion and mandrel, which are steel. The mandrel is solid and inserts halfway into the paper roll. Balance the design to use all of the material strength. Calculate the deflection at the end of the roll. Paper roll dimensions OD 1.50 m ID 0.22 m Lroll 3.23 m Roll density
3

Given:

Material properties

S y 100 MPa S ys 50 MPa E 207 GPa

984 kg m

Assumptions: 1. The paper roll's weight creates a concentrated load acting at the tip of the mandrel. 2. The mandrel's root in the stanchion experiences a distributed load over its length of engagement Solution: See Figures 4-27 and Mathcad file P0427.
y W

1. In Problem 3-27, we were concerned only with the portion of the mandrel outside of the stanchion. Therefore, we modeled it as a cantilever beam with a shear and moment reaction at the stanchion. Unfortunately, this tells us nothing about the stress or force distributions in the portion of the mandrel that is inside the stanchion. To do this we need to modify the model by replacing the concentrated moment (and possibly the concentrated shear force) with a force system that will yield information about the stress distribution in the mandrel on that portion that is inside the stanchion. Figure 4-27A shows the FBD used in Problem 3-27. Figure 4-27B is a simple model, but is not representative of a built-in condition. It would be appropriate if the hole in the stanchion did not fit tightly around the mandrel. Figure 4-27C is an improvement that will do for our analysis. 2. Determine the weight of the roll and the length of the mandrel. W

M1 R1

Lm

FIGURE 4-27A
Free Body Diagram for Problem 3-27
y R1 W

Lm R2

OD ID Lroll g

W 53.9 kN Lm 1.615 m

FIGURE 4-27B
Simplified Free Body Diagram, not used

Lm 0.5 Lroll

3. From inspection of Figure 4-27C, write the load function equation q(x) = -w<x>0 + w<x - b>0 + R<x - b>-1 - W<x - b -Lm>-1 4. Integrate this equation from - to x to obtain shear, V(x) V(x) = -w<x>1 + w<x - b>1 + R<x - b>0 - W<x - b -Lm>0 5. Integrate this equation from - to x to obtain moment, M(x)
b a

y w

Lm R

FIGURE 4-27C
Free Body Diagram used in Problem 4-27

MACHINE DESIGN - An Integrated Approach, 4th Ed.


M(x) = -(w/2)<x>2 + (w/2)<x - b>2 + R<x - b>1 - W<x - b -Lm>1

4-27-2

6. Solve for the reactions by evaluating the shear and moment equations at a point just to the right of x = b + Lm, where both are zero. At x = (b + Lm)+ , V = M = 0 0 = w b Lm w Lm R W w w 2 w 2 2 w 2 0 = b Lm Lm R Lm = b Lm Lm ( W w b ) Lm 2 2 2 2 Note that R is inversely proportional to b and w is inversly proportional to b 2. 7. To see the value of x at which the shear and moment are maximum, let b 400 mm then w 2 W Lm b 8. Define the range for x
2

R = W w b w= 2 W Lm b
2

and

R W w b

L b Lm

x 0 mm 0.002 L L

9. For a Mathcad solution, define a step function S. This function will have a value of zero when x is less than z, and a value of one when it is greater than or equal to z. S ( x z) if ( x z 1 0 ) 10. Write the shear and moment equations in Mathcad form, using the function S as a multiplying factor to get the effect of the singularity functions. V ( x) w S ( x 0 mm) x w S ( x b ) ( x b ) R S ( x b ) W S ( x L) M ( x) w 2 S ( x 0 mm) x
2

w 2

S ( x b ) ( x b ) R S ( x b ) ( x b ) W S ( x L) ( x L)

11. Plot the shear and moment diagrams. Shear Diagram


200 0 200 400 600 800 100 0 M ( x) kN m 50 50

Moment Diagram

V ( x) kN

400

800 x

1200 mm

1600

2000

400

800 x

1200 mm

1600

2000

FIGURE 4-27D
Shear and Moment Diagram Shapes for Problem 4-27

12. From Figure 4-27D, the maximum internal shear and moment occur at x = b and are Vmax = 2 W Lm b Mmax W Lm Mmax 87.04 kN m

MACHINE DESIGN - An Integrated Approach, 4th Ed.

4-27-3

13. The bending stress will be a maximum at the top or bottom of the mandrel at a section through x = b.

max =

Mmax a 2 I

where
1

I=

a
64

so,

max =

32 Mmax

a
a 206.97 mm

= Sy

Solving for a,

32 W Lm a S y
a 210 mm

Round this to

14. Using this value of a and equation 4.15c, solve for the shear stress on the neutral axis at x = b.

max =

4 Vmax 3 A

8 W Lm

a 2 b 3 4

= S ys

Solving for b

8 W Lm

a 2 Sys 3 4

b 134.026 mm

Round this to

b 134 mm

15. These are minimum values for a and b. Using them, check the bearing stress. Magnitude of distributed load w 2 W Lm b Bearing stress
2

w 9695

N mm

bear

w b a b

bear 46.2 MPa

Since this is less than S y, the design is acceptable for a 210 mm and b 134 mm 16. Assume a cantilever beam loaded at the tip with load W and a mandrel diameter equal to a calculated above. Moment of inertia I

a
64

I 9.547 10 mm W Lm
3

Deflection at tip (Appendix B)

ymax

3 E I

ymax 3.83 mm

This can be accomodated by the 220-mm inside diameter of the paper roll.

MACHINE DESIGN - An Integrated Approach, 4th Ed.

4-28-1

PROBLEM 4-28
Statement: Figure P4-13 shows a forklift truck negotiating a 15 deg ramp to to drive onto a 4-ft-high loading platform. The truck weighs 5 000 lb and has a 42-in wheelbase. Design two (one for each side) 1-ft-wide ramps of steel to have no more than 1-in deflection in the worst case of loading as the truck travels up them. Minimize the weight of the ramps by using a sensible cross-sectional geometry. Ramp angle Platform height Truck weight Truck wheelbase

Given:

15 deg
h 4 ft W 5000 lbf Lt 42 in

Ramp width Allowable deflection Young's modulus

w 12 in max 1.0 in E 30 10 psi


6

Assumptions: 1. The worst case is when the truck CG is located at the center of the beam's span. 2. Use a coordinate frame that has the x-axis along the long axis of the beam. 3. Ignore traction forces and the weight components along the x-axis of the beam. 4. There are two ramps, one for each side of the forklift. Solution: See Figure 4-28 and Mathcad file P0428.

L b a CG a
CG b

R1 Fa Wa Fb Wb R2 x

FIGURE 4-28A
Dimensions and Free Body Diagram for Problem 4-28

1. Determine the length of the beam between supports and the distances a and b for the worst-case loading. Length of beam From Problem 3-28, L h sin( ) L 15.455 ft b 8.561 ft

a 5.061 ft

2. The load distribution of the wheels on a single ramp is given in Problem 3-28 as Fa 575.0 lbf Fb 1839.9 lbf

3. From inspection of Figure 4-28A, write the load function equation q(x) = R1<x - 0>-1 - Fa<x - a>-1 - Fb<x - b>-1 + R2<x - L>-1

MACHINE DESIGN - An Integrated Approach, 4th Ed.


4. Integrate this equation from - to x to obtain shear, V(x) V(x) = R1<x - 0>0 - Fa<x - a>0 - Fb<x - b>0 + R2<x - L>0 5. Integrate this equation from - to x to obtain moment, M(x) M(x) = R1<x - 0>1 - Fa<x - a>1 - Fb<x - b>1 + R2<x - L>1 6. The reactions are given in Problem 3-28 as R1 1207.4 lbf R2 1207.4 lbf

4-28-2

7. Integrate the moment function, multiplying by 1/EI, to get the slope. (x) = [R1<x>2/2 - Fa<x - a>2/2 - Fb<x - b>2/2 + R2<x - L>2/2 + C3]/EI 8. Integrate again to get the deflection. y(x) = [R1<x>3/6 - Fa<x - a>3/6 - Fb<x - b>3/6 + R2<x-L>3/6 + C3x +C4]/EI 9. Evaluate C3 and C4 At x = 0 and x = L, y = 0, therefore, C4 = 0. 0 = R1 L Fa ( L a ) Fb ( L b ) 6 C3 L C3 1 6 L R1 L Fa ( L a ) Fb ( L b )
3 3 3 3 3 3

C3 4.983 10 lbf in

8. Define the range for x

x 0 m 0.005 L L

9. For a Mathcad solution, define a step function S. This function will have a value of zero when x is less than z, and a value of one when it is greater than or equal to z. S ( x z) if ( x z 1 0 ) 10. Write the slope and deflection equations in Mathcad form, using the function S as a multiplying factor to get the effect of the singularity functions. Use an assumed value of I so that the value of x that corresponds to ymax can be found. Let I 10 in
4 2

( x)

1 E I

R1
2

S ( x 0 m) x

Fa 2

S ( x a ) ( x a )

Fb 2

R 2 S ( x L) ( x L) 2 C3 2
S ( x 0 m) x
3

S ( x b ) ( x b )

y ( x)

1 E I

R1
6

Fa 6

S ( x a ) ( x a )

Fb 6

R 2 S( x L) ( x L) 3 C3 x 6

S ( x b ) ( x b )

11. Plot the shear and moment diagrams using the assumed value of I.

MACHINE DESIGN - An Integrated Approach, 4th Ed.


SLOPE, radians
0.02 0

4-28-3
DEFLECTION, in

0.01 0.5 y ( x) 0 in 1 0.01

( x)

0.02

8 x ft

12

16

1.5

8 x ft

12

16

FIGURE 4-28B
Slope and Deflection Diagrams for Problem 4-28, Using an Assumed Value for I

12. Maximum deflection occurs at x = c, where = 0 and c < b.

0 =
Solving for c, A R1 2

R1 2 Fa 2 c ( c a ) C3 = 0 E I 2 2
1 B a Fa B 3.492 10 lbf in c 7.804 ft
4

Fa 2

C C3

a Fa 2
6 2

A 316.200 lbf c B B 4 A C 2 A
2

C 6.043 10 in lbf

13. The maximum deflection occurs at x = c and is Solving for I

ymax

R 1 c 3 F a 3 = ( c a ) C3 c = max E I 6 6
1

R 1 c 3 F a 3 I ( c a ) C3 c E max 6 6
1 This is the minimum allowable value of the moment of inertia.

I 10.159 in

14. Assume a channel section such as that shown in Figure 4-28C. To keep it simple, let the thickness of the flanges and web be the same. Choose 3/8-in thick plate, which is readily available. Then, t 0.375 in 15. The cross-sectional area of the ramp is 16. The distance to the CG is cg( h ) A ( h ) w t 2 t ( h t) 1 A (h)

w t 2 2

t h t

MACHINE DESIGN - An Integrated Approach, 4th Ed.


17. The moments of inertia of the web and a flange are Iweb( h ) w t
3

4-28-4

12

w t cg( h )

2 h t
2

Flange Web

Ifl ( h )

t ( h t) 12

h t cg( h )

18. Using the known moment of inertia, solve for the unknown flange height, h. Guess h 1 in Given I = Iweb( h ) 2 Ifl ( h ) h Find ( h ) Round this up to h 4.00 in h 3.988 in FIGURE 4-28C
Channel Section for Problem 4-28

19. Summarizing, the ramp design dimensions are: Length Width L 185.5 in w 12.00 in Flange height Thickness h 4.00 in t 0.375 in Shape Material channel steel

MACHINE DESIGN - An Integrated Approach, 4th Ed.

4-29a-1

PROBLEM 4-29a
Statement: Find the spring rate of the beam in Problem 4-23 at the applied concentrated load for row a in Table P4-2. Beam length Distance to distributed load L 1 m a 0.4 m w 200 N m Fb 500 N I 2.85 10
8 1
R1 b a w F L

Given:

Distance to concentrated load b 0.6 m Distributed load magnitude Concentrated load Moment of inertia Modulus of elasticity Solution:

R2

E 207 GPa

FIGURE 4-29
Free Body Diagram for Problem 4-23

See Figure 4-29 and Mathcad file P0429a.

1. The deflection equation was found in Problem 4-23. Those results are summarized here. Load function Shear function Moment function Slope function Deflection function q(x) = R1<x - 0>-1 - w<x - 0>0 + w<x - a>0 - F<x - b>-1 + R2<x - L>-1 V(x) = R1<x - 0>0 - w<x - 0>1 + w<x - a>1 - F<x - b>0 + R2<x - L>0 M(x) = R1<x - 0>1 - w<x - 0>2/2 + w<x - a>2/2 - F<x - b>1 + R2<x - L>1 (x) = [R1<x>2/2 - w<x>3/6 + w<x - a>3/6 - F<x - b>2/2 + R2<x - L>2/2 + C3]/EI y(x) = [R1<x>3/6 - w<x>4/24 + w<x - a>4/24 - F<x - b>3/6 + R2<x-L>3/6 + C3x +C4]/EI

2. To determine the stiffness under the load F we will need to find the incremental beam deflection due to F alone. The procedure will be to find the deflection at x = b when F = 0, and then find it when Fb 500 N . The stiffness will then be the force divided by the incremental deflection. 3. For a Mathcad solution, define a step function S. This function will have a value of zero when x is less than z, and a value of one when it is greater than or equal to z. S ( x z) if ( x z 1 0 ) 4. Write the reactions (from Problem 3-23), integration constant, and deflection (from problem 4-23) equations in Mathcad form, using the function S as a multiplying factor to get the effect of the singularity functions. R1( F ) w 2 L F L ( L b) w 2 L ( L a)
2

R2( F ) w a F R1( F ) C3( F ) y ( x F )

R1( F ) 3 w 4 w F 4 3 L L ( L a) ( L b) 6 L 24 24 6
1 1 E I

R1( F )
6

S ( x 0 in) x

w 24

S ( x 0 in) x

w 24

R (F ) 2 S( x L) ( x L) 3 F S ( x b ) ( x b) 3 C3( F ) x 6 6
y0 y ( b F ) yF y ( b F )

S ( x a ) ( x a )

5. The deflection at x = b for F 0 N is 6. The deflection at x = b for F Fb is 7. The deflection due to F alone is

y0 0.137 mm yF 1.765 mm

y yF y0
k F

y 1.627 mm y
k 307 N mm

8. The stiffness of the beam under the load F at x = b is

MACHINE DESIGN - An Integrated Approach, 4th Ed.

4-30a-1

PROBLEM 4-30a
Statement: Find the spring rate of the beam in Problem 4-24 at the applied concentrated load for row a in Table P4-2. Beam length Distance to distributed load Distributed load magnitude Concentrated load Moment of inertia Modulus of elasticity Solution: L 1 m a 0.4 m w 200 N m FL 500 N I 2.85 10
8 1
w a F L

Given:

4
M1 R1

E 207 GPa

See Figure 4-30 and Mathcad file P0430a.

FIGURE 4-30
Free Body Diagram for Problem 4-24

1. The deflection equation was found in Problem 4-24. Those results are summarized here. Load function Shear function Moment function Slope function Deflection function

q(x) = -M1<x - 0>-2 + R1<x - 0>-1 - w<x - a>0 - F<x - L>-1 V(x) = -M1<x - 0>-1 + R1<x - 0>0 - w<x - a>1 - F<x - L>0 M(x) = -M1<x - 0>0 + R1<x - 0>1 - w<x - a>2/2 - F<x - L>1 (x) = [-M1<x-0>1 + R1<x - 0>2/2 - w<x - a>3/6 - F<x - L>2/2 + C3]/EI y(x) = [-M1<x-0>2/2 + R1<x - 0>3/6 - w<x - a>4/24 - F<x - L>3/6 + C3x +C4]/EI

2. To determine the stiffness under the load F we will need to find the incremental beam deflection due to F alone. The procedure will be to find the deflection at x = L when F = 0, and then find it when FL 500 N . The stiffness will then be the force divided by the incremental deflection. 3. For a Mathcad solution, define a step function S. This function will have a value of zero when x is less than z, and a value of one when it is greater than or equal to z. S ( x z) if ( x z 1 0 ) 4. Write the reaction (from Problem 3-24) and deflection (from problem 4-24) equations in Mathcad form, using the function S as a multiplying factor to get the effect of the singularity functions. R1( F ) w ( L a ) F M1( F ) w 2 1 E I ( L a ) R 1( F ) L
2

y ( x F )

M1( F )
2

S ( x 0 in) x

R1( F ) 6

S ( x 0 in) x

w 24

F S( x L) ( x L) 3 6

S ( x a ) ( x a )

5. The deflection at x = L for F 0 N is 6. The deflection at x = L for F FL is 7. The deflection due to F alone is

y0 y ( L F ) yF y ( L F )

y0 3.912 mm yF 32.163 mm

y yF y0
k F

y 28.251 mm
k 17.7 N mm

8. The stiffness of the beam under the load F at x = L is

MACHINE DESIGN - An Integrated Approach, 4th Ed.

4-31a-1

PROBLEM 4-31a
Statement: Find the spring rate of the beam in Problem 4-25 at the applied concentrated load for row a in Table P4-2. Beam length Distance to distributed load L 1 m a 0.4 m w 200 N m FL 500 N I 2.85 10
8 1
R1 b a w F L

Given:

Distance to concentrated load b 0.6 m Distributed load magnitude Concentrated load Moment of inertia Modulus of elasticity Solution:

R2

E 207 GPa

FIGURE 4-31
Free Body Diagram for Problem 4-25

See Figure 4-31 and Mathcad file P0431a.

1. The deflection equation was found in Problem 4-25. Those results are summarized here. Load function Shear function Moment function Slope function Deflection function q(x) = R1<x - 0>-1 - w<x - a>0 + R2<x - b>-1 - F<x - L>-1 V(x) = R1<x - 0>0 - w<x - a>1 + R2<x - b>0 - F<x - L>0 M(x) = R1<x - 0>1 - w<x - a>2/2 + R2<x - b>1 - F<x - L>1 (x) = [R1<x - 0>2/2 - w<x - a>3/6 + R2<x - b>2/2 - F<x - L>2/2 + C3]/EI y(x) = [R1<x - 0>3/6 - w<x - a>4/24 + R2<x-b>3/6 - F<x - L>3/6 + C3x +C4]/EI

2. To determine the stiffness under the load F we will need to find the incremental beam deflection due to F alone. The procedure will be to find the deflection at x = L when F = 0, and then find it when FL 500 N . The stiffness will then be the force divided by the incremental deflection. 3. For a Mathcad solution, define a step function S. This function will have a value of zero when x is less than z, and a value of one when it is greater than or equal to z. S ( x z) if ( x z 1 0 ) 4. Write the reactions (from Problem 3-25), integration constant, and deflection (from problem 4-25) equations in Mathcad form, using the function S as a multiplying factor to get the effect of the singularity functions. R1( F ) w 2 ( L a ) F ( L b ) w ( L a ) ( L b ) 2 b 1

R2( F ) w ( L a ) F R1( F ) 1 R1( F ) 3 w 4 C3( F ) b ( b a) b 6 24 y ( x F ) 1 E I

R1( F )

6 24 R2( F ) F 3 3 S ( x b ) ( x b ) S ( x L) ( x L) C3( F ) x 6 6
S ( x 0 in) x
3

S ( x a ) ( x a )

5. The deflection at x = L for F 0 N is 6. The deflection at x = L for F FL is

y0 y ( L F ) yF y ( L F )

y0 0.288 mm yF 4.808 mm

MACHINE DESIGN - An Integrated Approach, 4th Ed.


7. The deflection due to F alone is

4-31a-2

y yF y0
k F

y 4.52 mm
k 111 N mm

8. The stiffness of the beam under the load F at x = L is

MACHINE DESIGN - An Integrated Approach, 4th Ed.

4-32a-1

PROBLEM 4-32a
Statement: Find the spring rate of the beam in Problem 4-26 at the applied concentrated load for row a in Table P4-2. Beam length Distance to distributed load L 1 m a 0.4 m w 200 N m Fa 500 N I 2.85 10
8 1
R1 b a F w L

Given:

Distance to concentrated load b 0.6 m Distributed load magnitude Concentrated load Moment of inertia Modulus of elasticity Solution:

R2

R3

E 207 GPa

FIGURE 4-32
Free Body Diagram for Problem 4-26

See Figure 4-32 and Mathcad file P0432a.

1. The deflection equation was found in Problem 4-26. Those results are summarized here. Load function Shear function Moment function Slope function q(x) = R1<x>-1 - F<x - a>-1 - w<x - a>0 + R2<x - b>-1 - R3<x - L>-1 V(x) = R1<x>0 - F<x - a>0 - w<x - a>1 + R2<x - b>0 - R3<x - L>0 M(x) = R1<x>1 - F<x - a>1 - w<x - a>2/2 + R2<x - b>1 - R3<x - L>1 (x) = [R1<x>2/2 - F<x - a>2/2 - w<x - a>3/6 + R2<x - b>2/2 + R3<x - L>2/2 + C3]/EI

Deflection function y(x) = [R1<x>3/6 - F<x - a>3/6 - w<x - a>4/24 + R2<x - b>3/6 + R3<x - L>3/6 + C3x + C4]/EI 2. To determine the stiffness under the load F we will need to find the incremental beam deflection due to F alone. The procedure will be to find the deflection at x = a when F = 0, and then find it when Fa 500 N . The stiffness will then be the force divided by the incremental deflection. 3. For a Mathcad solution, define a step function S. This function will have a value of zero when x is less than z, and a value of one when it is greater than or equal to z. S ( x z) if ( x z 1 0 ) 4. Write the reactions, integration constant, and deflection (from problem 4-26) equations in Mathcad form, using the function S as a multiplying factor to get the effect of the singularity functions. Let f1 ( F ) F 6 ( b a) w 2
3

w 24

( b a)
2

f2 ( F )

F 6

( L a)

w 24

( L a)

f3 ( F ) F ( L a ) then

( L a)

2 L f1 ( F ) f2 ( F ) ( L b ) f3 ( F ) R1( F ) L b ( L b ) b 6

R1 Fa 112.333 N R2 Fa 559.167 N R3 Fa 51.500 N

R2( F )

1 ( L b)

f3 ( F ) L R1( F )

R3( F ) F w ( L a ) R1( F ) R2( F ) C3( F ) 1 b f1 ( F ) b


2

R 1( F )

MACHINE DESIGN - An Integrated Approach, 4th Ed.

4-32a-2

y ( x F )

1 E I

R1( F )

w 3 F 3 4 S ( x 0 in) x S ( x a ) ( x a ) S ( x a ) ( x a ) 6 6 24 R (F ) R (F ) 2 S( x b) ( x b ) 3 3 S ( x L) ( x L) 3 C3( F ) x 6 6

5. The deflection at x = a for F 0 N is 6. The deflection at x = a for F Fa is 7. The deflection due to F alone is

y0 y ( a F ) yF y ( a F )

y0 0.00126 mm yF 0.177 mm

y yF y0
k F

y 0.176 mm
k 2844 N mm

8. The stiffness of the beam under the load F at x = a is

MACHINE DESIGN - An Integrated Approach, 4th Ed.

4-33a-1

PROBLEM 4-33a
Statement: For the bracket shown in Figure P4-14 and the data in row a of Table P4-3, determine the bending stress at point A and the shear stress due to transverse loading at point B. Also the torsional shear stress at both points. Then determine the principal stresses at points A and B. Tube length Arm length Arm thickness Arm depth Applied force Tube OD Tube ID Modulus of elasticity Solution: L 100 mm a 400 mm t 10 mm h 20 mm F 50 N OD 20 mm ID 14 mm E 207 GPa
R T M L y A B T x F

Given:

FIGURE 4-33
Free Body Diagram of Tube for Problem 4-33

See Figure 4-33 and Mathcad file P0433a.

1. Determine the bending stress at point A. From the FBD of the tube in Figure 4-33 we see that Reaction force Reaction moment Distance from NA to outside of tube Moment of inertia Bending stress at point A R F M F L ct 0.5 OD It R 50.0 N M 5.00 N m ct 10.0 mm
4 4

64

OD ID

It 5968 mm

xA

M ct It

xA 8.38 MPa

2. Determine the shear stress due to transverse loading at B. Cross-section area Maximum shear Maximum shear stress (Equation 4.15d) A

OD ID

A 160.2 mm

V R

Vmax 2

V A

Vmax 0.624 MPa

3. Determine the torsional shear stress at both points. Using equation 4.23b and the FBD above Torque on tube Polar moment of inertia Maximum torsional stress at surface T F a J T 20.0 N m
4 4

32

OD ID T ct J

J 11936 mm

Tmax

Tmax 16.76 MPa

4. Determine the principal stress at point A. Stress components

xA 8.378 MPa xz Tmax

zA 0 MPa xz 16.76 MPa

MACHINE DESIGN - An Integrated Approach, 4th Ed.


Principal stresses

4-33a-2

xA zA
2

2 xA zA 2 xz 2

1 21.46 MPa

2 0 MPa xA zA
2
2 xA zA 2 xz 2

3 13.08 MPa

13

1 3
2

13 17.27 MPa

5. Determine the principal stress at point B. Stress components

xB 0 MPa xy Tmax Vmax

yB 0 MPa xy 16.13 MPa

Principal stresses

xB yB
2

2 xB yB 2 xy 2

1 16.13 MPa

2 0 MPa xB yB
2
2 xB yB 2 xy 2

3 16.13 MPa

13

1 3
2

13 16.13 MPa

MACHINE DESIGN - An Integrated Approach, 4th Ed.

4-34a-1

PROBLEM 4-34a
Statement: Given: For the bracket shown in Figure P4-14 and the data in row a of Table P4-3, determine the deflection at load F. Tube length Arm length Arm thickness Arm depth L 100 mm a 400 mm t 10 mm h 20 mm Applied force Tube OD Tube ID Modulus of elasticity Modulus of rigidity F 50 N OD 20 mm ID 14 mm E 207 GPa G 80.8 GPa

Solution: 1. 2.

See Figure 4-34 and Mathcad file P0434a.

The deflection at load F can be determined by superimposing the rigid-body deflection of the arm due to the twisting of the tube with the beam deflection of the tube and the arm alone. Determine the rigid-body deflection due to twisting of the tube. Refering to Figure 4-34, the torque in the tube is Torque on tube Polar moment of inertia Tube angle of twist T F a Jt T 20.0 N m
4 4

32

TL J t G

OD ID

Jt 11936 mm

4 3

2.07368 10 0.119 deg

rad

Deflection at F due to 3.

a
Jt 2 F L
3

0.829 mm

Determine the rigid-body deflection due to bending of the tube. Moment of inertia Deflection of tube end and arm end (see Appendix B)
F y A a

It

It 5968 mm

tb

3 E It

tb 0.013 mm
F

T M

z T

L R F

FIGURE 4-34
Free Body Diagrams of Tube and Arm for Problem 4-34

4.

Determine the beam bending of arm alone. Moment of inertia Ia t h


3

12 F a
3

Ia 6667 mm

Deflection at F 5. Determine the total deflection by superposition.

3 E Ia

a 0.773 mm

tot tb a

tot 1.616 mm

downward

MACHINE DESIGN - An Integrated Approach, 4th Ed.

4-35a-1

PROBLEM 4-35a
Statement: For the bracket shown in Figure P4-14 and the data in row a of Table P4-3, determine the spring rate of the tube in bending, the spring rate of the arm in bending, and the spring rate of the tube in torsion. Combine these into an overall spring rate in terms of the force F and the linear deflection at F. Tube length Arm length Arm thickness Arm depth L 100 mm a 400 mm t 10 mm h 20 mm Applied force Tube OD Tube ID Modulus of elasticity Modulus of rigidity F 50 N OD 20 mm ID 14 mm E 207 GPa G 80.8 GPa

Given:

Solution: 1.

See Figure 4-35 and Mathcad file P0435a.

Determine the spring rate due to bending of the tube. Moment of inertia Deflection of tube end and arm end (see Appendix B) Spring rate due to bending in tube It

64

OD ID
3

It 5968 mm

tb

F L

3 E It F

tb 0.013 mm
N mm

ktb

tb

ktb 3706

2.

Determine the spring rate due to beam bending of arm alone. Moment of inertia Ia t h
3

12 F a
3

Ia 6667 mm

Deflection at F

3 E Ia

a 0.773 mm

Spring rate due to bending in arm


F

ka

ka 64.7

N mm
F

y A T M L R B

z T F

FIGURE 4-35
Free Body Diagrams of Tube and Arm for Problem 4-35

3.

Determine the spring rate of the tube in torsion. Refering to Figure 4-35, the torque in the tube is Torque on tube Polar moment of inertia T F a Jt

32

OD ID

T 20.0 N m Jt 11936 mm
4

MACHINE DESIGN - An Integrated Approach, 4th Ed.


Tube angle of twist

4-35a-2

TL J t G

2.07368 10 0.119 deg

rad

Deflection at F due to q Spring rate due to torsion in tube

a
k F

0.829 mm
k 60.28 N mm

1 koa 1 k

4.

Determine the overall spring rate. The springs are in series, thus k ktb ka ktb ka k ka k ktb

1 ktb

1 ka N mm

koa

koa 30.9

Checking,

tot

F koa

tot 1.616 mm

which is the same total deflection gotten in Problem 4-34.

MACHINE DESIGN - An Integrated Approach, 4th Ed.

4-36a-1

PROBLEM 4-36a
Statement: For the bracket shown in Figure P4-14 and the data in row a of Table P4-3, redo Problem 4-33 considering the stress concentration at points A and B. Assume a stress concentration factor of 2.5 in both bending and torsion. Tube length Arm length Arm thickness Arm depth Applied force Tube OD Tube ID Modulus of elasticity Stress-concentration factors Solution: 1. L 100 mm a 400 mm t 10 mm h 20 mm F 50 N OD 20 mm ID 14 mm E 207 GPa Ktb 2.5 Kts 2.5
R T M L y A B T x F

Given:

FIGURE 4-36
Free Body Diagram of Tube for Problem 4-36

See Figure 4-36 and Mathcad file P0436a.

Determine the bending stress at point A. From the FBD of the tube in Figure 4-36 we see that Reaction force Reaction moment Distance from NA to outside of tube Moment of inertia Bending stress at point A R F M F L ct 0.5 OD It R 50.0 N M 5.00 N m ct 10.0 mm
4 4

64

OD ID M ct It

It 5968 mm

xA Ktb

xA 20.94 MPa

2.

Determine the shear stress due to transverse loading at B. Cross-section area Maximum shear Maximum shear stress (Equation 4.15d) A

OD ID

A 160.2 mm

V R

Vmax 2

V A

Vmax 0.624 MPa

3.

Determine the torsional shear stress at both points. Using equation 4.23b and the FBD above Torque on tube Polar moment of inertia Maximum torsional stress at surface T F a J T 20.0 N m

32

OD ID T ct J

J 11936 mm

Tmax Kts

Tmax 41.89 MPa

4.

Determine the principal stress at point A. Stress components

xA 20.944 MPa

zA 0 MPa

MACHINE DESIGN - An Integrated Approach, 4th Ed.

4-36a-2

xz Tmax
Principal stresses

xz 41.89 MPa

xA zA
2

2 xA zA 2 xz 2

1 53.6 MPa

2 0 MPa xA zA
2
2 xA zA 2 xz 2

3 32.71 MPa

13
5.

1 3
2

13 43.18 MPa

Determine the principal stress at point B. Stress components

xB 0 MPa xy Tmax Vmax

yB 0 MPa xy 41.26 MPa

Principal stresses

xB yB
2

2 xB yB 2 xy 2

1 41.26 MPa

2 0 MPa xB yB
2
2 xB yB 2 xy 2

3 41.26 MPa

13

1 3
2

13 41.26 MPa

MACHINE DESIGN - An Integrated Approach, 4th Ed.

4-37-1

PROBLEM 4-37
Statement: A semicircular, curved beam as shown in Figure 4-37 has the dimensions given below. For the load pair applied along the diameter and given below, find the eccentricity of the neutral axis and the stress at the inner and outer fibers. Outside diameter Inside diameter Width of beam Load Solution: od 150 mm id 100 mm w 25 mm F 14 kN
od id F F w

Given:

See Figure 4-37 and Mathcad file P0437.

1. Calculate the section depth, area, inside radius and outside radus. Section depth Area of section Centroid radius Inside and outside radii of section h od id 2 od id 4 h 25 mm A 625 mm
2
F M F rc (b) Critical Section (a) Entire Beam

A h w rc

rc 62.5 mm ri 50 mm ro 75 mm

ri rc 0.5 h ro rc 0.5 h

2. The critical section is the one that is along the horizontal centerline. There, the bending moment is Bending moment 3. M F rc

FIGURE 4-37
Free Body Diagrams for Problem 4-37

M 0.875 kN m

Use the equation in the footnote of the text to calculate the radius of the neutral axis. Radius of neutral axis rn ro ri

ro ln ri

rn 61.658 mm

4.

Calculate the eccentricty and the distances from the neutral axis to the extreme fibers. Eccentricity Distances from neutral axis to extreme fibers e rc rn ci rn ri co ro rn e 0.8424 mm ci 11.66 mm co 13.34 mm F A

Stresses at inner and outer radii

e A ri

ci

i 409.9 MPa

M co F e A ro A

o 273.2 MPa

MACHINE DESIGN - An Integrated Approach, 4th Ed.

4-38-1

PROBLEM 4-38
Statement: Design a solid, straight, steel torsion bar to have a spring rate of 10 000 in-lb per radian per foot of length. Compare designs of solid round and solid square cross-sections. Which is more efficient in terms of material use? Length of rod Spring rate Solution: 1. See Mathcad file P0438. L 12 in k 10000 in lbf rad Modulus of rigidity G 11.7 10 psi
6

Given:

Determine the rod diameter and volume for a round rod. Spring rate k= J G L
1

J =

d
32

Rod diameter

d V

32 L k G
d
4
2

d 0.569 in V 3.046 in
3

Volume of rod 2.

Determine the rod width and volume for a square rod using equation 4.26b and Table 4-6. Spring rate k= K G L
1 4

K = 2.25 a

Rod half-width Volume of rod

L k a 2.25 G V ( 2 a ) L
2

a 0.260 in V 3.241 in
3

2 a 0.520 in

3.

Even though the square rod width is less than the round rod diameter, it takes slightly more material when a square rod is used than when a round rod is used. Thus, the round rod is more efficient.

MACHINE DESIGN - An Integrated Approach, 4th Ed.

4-39-1

PROBLEM 4-39
Statement: Design a 1-ft-long steel, end-loaded cantilever spring for a spring rate of 10 000 lb/in. Compare designs of solid round and solid square cross-sections. Which is more efficient in terms of material use? Length of rod Spring rate Solution: 1. L 12 in k 10000 lbf in Modulus of rigidity E 30 10 psi
6

Given:

See Figure B-1(a) in Appendix B and Mathcad file P0439.

Determine the rod diameter and volume for a round rod. Spring rate k= 3 E I L
3 1 4

I=

d
64

Rod diameter

64 L3 k d 3 E
V

d 1.406 in V 18.64 in
3

Volume of rod 2.

d
4

Determine the rod width and volume for a square rod using equation 4.26b and Table 4-6. Spring rate k= 3 E I L Rod width Volume of rod
3 1 4

I=

12

4 L3 k a E
V a L
2

a 1.232 in V 18.215 in
3

3.

It takes slightly more material when a round rod is used than when a square rod is used. Thus, the square rod is more efficient.

MACHINE DESIGN - An Integrated Approach, 4th Ed.

4-40-1

PROBLEM 4-40
Statement: Redesign the roll support of Problem 4-8 to be like that shown in Figure P4-16. The stub mandrels insert to 10% of the roll length at each end. Choose appropriate dimensions a and b to fully utilize the mandrel's strength, which is the same as in Problem 4-27. See Problem 4-8 for additional data. Paper roll dimensions OD 1.50 m ID 0.22 m Lroll 3.23 m Roll density
3

Given:

Material properties

S y 100 MPa S ys 50 MPa E 207 GPa

984 kg m

Assumptions: 1. The paper roll's weight creates a concentrated load acting at the tip of the mandrel. 2. The mandrel's root in the stanchion experiences a distributed load over its length of engagement Solution: See Figures 4-40 and Mathcad file P0440.
y w a x F

1. Model the support in such a way that stresses in the portion of the mandrel that is inside the stanchion can be determined. There are several assumptions that can be made about the loads on this portion of the mandrel. Figure 4-40A shows the one that will be used for this design. 2. Determine the weight of the roll, the load on each support, and the length of the mandrel. W

b R

Lm

OD ID Lroll g

W 53.9 kN F 26.95 kN Lm 323 mm

FIGURE 4-40A
Free Body Diagram used in Problem 4-40

F 0.5 W Lm 0.1 Lroll

3. From inspection of Figure 4-40A, write the load function equation q(x) = -w<x>0 + w<x - b>0 + R<x - b>-1 - F<x - b -Lm>-1 4. Integrate this equation from - to x to obtain shear, V(x) V(x) = -w<x>1 + w<x - b>1 + R<x - b>0 - F<x - b -Lm>0 5. Integrate this equation from - to x to obtain moment, M(x) M(x) = -(w/2)<x>2 + (w/2)<x - b>2 + R<x - b>1 - F<x - b -Lm>1 6. Solve for the reactions by evaluating the shear and moment equations at a point just to the right of x = b + Lm, where both are zero. At x = (b + Lm)+ , V = M = 0 0 = w b Lm w Lm R F w w 2 w 2 2 w 2 0 = b Lm Lm R Lm = b Lm Lm ( F w b ) Lm 2 2 2 2 Note that R is inversely proportional to b and w is inversly proportional to b 2. 7. To see the value of x at which the shear and moment are maximum, let R = F w b w= 2 F Lm b
2

MACHINE DESIGN - An Integrated Approach, 4th Ed.


b 200 mm then w 2 F Lm b 8. Define the range for x
2

4-40-2
and R F w b L b Lm

x 0 mm 0.002 L L

9. For a Mathcad solution, define a step function S. This function will have a value of zero when x is less than z, and a value of one when it is greater than or equal to z. S ( x z) if ( x z 1 0 ) 10. Write the shear and moment equations in Mathcad form, using the function S as a multiplying factor to get the effect of the singularity functions. V ( x) w S ( x 0 mm) x w S ( x b ) ( x b ) R S ( x b ) F S ( x L) M ( x) w 2 S ( x 0 mm) x
2

w 2

S ( x b ) ( x b ) R S ( x b ) ( x b ) F S ( x L) ( x L)

11. Plot the shear and moment diagrams. Shear Diagram


200

Moment Diagram
2

100 V ( x) kN 0 M ( x) kN m

100

200

100

200

300 x mm

400

500

600

10

100

200

300 x mm

400

500

600

FIGURE 4-40B
Shear and Moment Diagram Shapes for Problem 4-40

12. From Figure 4-40B, the maximum internal shear and moment occur at x = b and are Vmax = 2 F Lm b Mmax F Lm Mmax 8.704 kN m

13. The bending stress will be a maximum at the top or bottom of the mandrel at a section through x = b.

max =

Mmax a 2 I

where
1

I=

a
64

so,

max =

32 Mmax

a
a 121.037 mm

= Sy

Solving for a,

32 W Lm a S y
a 125 mm

Round this to

MACHINE DESIGN - An Integrated Approach, 4th Ed.

4-40-3

14. Using this value of a and equation 4.15c, solve for the shear stress on the neutral axis at x = b.

max =

4 Vmax 3 A

8 F Lm

a 2 b 3 4

= S ys

Solving for b

8 F Lm

a 2 Sys 3 4

b 37.828 mm

Round this to

b 38 mm

15. These are minimum values for a and b. Using them, check the bearing stress. Magnitude of distributed load w 2 F Lm b Bearing stress
2

w 12055

N mm

bear

w b a b

bear 96.4 MPa

Since this is less than S y, the design is acceptable for a 125 mm and b 38 mm

MACHINE DESIGN - An Integrated Approach, 4th Ed.

4-41-1

PROBLEM 4-41
Statement: Given: Assumption: Solution: A 10-mm ID steel tube carries liquid at 7 MPa. Determine the principal stresses in the wall if its thickness is: a) 1 mm, b) 5 mm. Tubing ID ID 10 mm Inside pressure p i 7 MPa

The tubing is long therefore the axial stress is zero. See Mathcad file P0441. t 1 mm

(a) Wall thickness is

1. Check wall thickness to radius ratio to see if this is a thick or thin wall problem. ratio t 0.5 ID ratio 0.2

Since the ratio is greater than 0.1, this is a thick wall problem. 2. Using equations 4.48a and 4.48b, the stresses are maximum at the inside wall where Inside radius Outside radius ri 0.5 ID ro ri t
2 1 ro t 2 2 2 ri ro ri

ri 5 mm ro 6 mm

Tangential stress

ri p i

t 38.82 MPa

Radial stress

2 1 ro r 2 2 2 ri ro ri

ri p i

r 7.00 MPa

3. Determine the principal stresses (since, for this choice of coordinates, the shear stress is zero),

1 t 2 0 MPa 3 r
The maximum shear stress is

1 38.82 MPa

3 7.00 MPa 1 3
2 t 5 mm

max
(b) Wall thickness is

max 22.91 MPa

1. Check wall thickness to radius ratio to see if this is a thick or thin wall problem. ratio t 0.5 ID ratio 1

Since the ratio is greater than 0.1, this is a thick wall problem. 2. Using equations 4.48a and 4.48b, the stresses are maximum at the inside wall where Inside radius Outside radius ri 0.5 ID ro ri t ri 5 mm ro 10 mm

MACHINE DESIGN - An Integrated Approach, 4th Ed.


2 1 ro t 2 2 2 ri ro ri

4-41-2
2

Tangential stress

ri p i

t 11.67 MPa

Radial stress

ri p i
2 2

ro ri

ro ri

r 7.00 MPa

3. Determine the principal stresses (since, for this choice of coordinates, the shear stress is zero),

1 t 2 0 MPa 3 r
The maximum shear stress is

1 11.67 MPa

3 7.00 MPa 1 3
2

max

max 9.33 MPa

MACHINE DESIGN - An Integrated Approach, 4th Ed.

4-42-1

PROBLEM 4-42
Statement: A cylindrical tank with hemispherical ends is required to hold 150 psi of pressurized air at room temperature. Find the principal stresses in the 1-mm-thick wall if the tank diameter is 0.5 m and its length is 1 m. Tank ID Wall thickness Inside pressure ID 500 mm t 1 mm p i 150 psi

Given:

p i 1034 kPa

Solution: 1.

See Mathcad file P0442.

Check wall thickness to radius ratio to see if this is a thick or thin wall problem. ratio t 0.5 ID ratio 4 10
3

Since the ratio is less than 0.1, this is a thin wall problem. 2. Using equations 4.49a, 4.49b and 4.49c, the stresses are Radius Tangential stress Radial stress Axial stress 3. r 0.5 ID r 250 mm

pi r t

t 258.55 MPa r 0.00 MPa a 129.28 MPa

r 0 MPa a
pi r 2 t

Determine the principal stresses (since, for this choice of coordinates, the shear stress is zero),

1 t 2 a 3 0 MPa
The maximum shear stress is

1 259 MPa 2 129 MPa 3 0.00 MPa

1 37.5 ksi 2 18.75 ksi 3 0.00 MPa

max

1 3
2

max 129 MPa

MACHINE DESIGN - An Integrated Approach, 4th Ed.

4-43-1

PROBLEM 4-43
Statement: Figure P4-17 shows an off-loading station at the end of a paper rolling machine. The finished paper rolls are 0.9-m OD by 0.22-m ID by 3.23-m long and have a density of 984 kg/m3. The rolls are transfered from the machine conveyor (not shown) to the forklift truck by the V-linkage of the off-load station, which is rotated through 90 deg by an air cylinder. The paper then rolls onto the waiting forks of the truck. The forks are 38-mm thick by 100-mm wide by 1.2-m long and are tipped at a 3-deg angle from the horizontal. Find the stresses in the two forks on the truck when the paper rolls onto it under two different conditions (state all assumptions): (a) The two forks are unsupported at their free end. (b) The two forks are contacting the table at point A. Paper roll dimensions OD 0.90 m ID 0.22 m Lroll 3.23 m Fork dimensions t 38 mm w 100 mm Lfork 1200 mm

Given:

Roll density

984 kg m

fork 3 deg

Assumptions: 1. The greatest bending moment will occur when the paper roll is at the tip of the fork for case (a) and when it is midway between supports for case (b). 2. Each fork carries 1/2 the weight of a paper roll. 3. For case (a), each fork acts as a cantilever beam (see Appendix B-1(a)). 4. For case (b), each fork acts as a beam that is built-in at one end and simply-supported at the other. Solution: See Figure 4-43 and Mathcad file P0443.
F L fork

1. Determine the weight of the roll and the load on each fork. W

OD ID Lroll g

W 18.64 kN F 9.32 kN
Case (a), Cantilever Beam R1 M1

F 0.5 W

2. The moment of inertia and the distance to the extreme fiber for a fork are I c w t t 2
3

12

I 4.573 10 mm c 19 mm

0.5 L fork t

L fork R1 R2

M2

Case (a) 3. From Figure D-1(a), the moment is a maximum at the support and is Mmax F Lfork Mmax 11.186 kN m

Case (b), Fixed-Simply Supported Beam

FIGURE 4-43A
Free Body Diagrams used in Problem 4-43

4. The bending stress is maximum at the support and is Case (b)

Mmax c I

a 464.8 MPa

5. This beam is statically indeterminate. However, using singularity functions and the method shown in Example 4-7, we can determine the reactions and find the maximum moment. 6. Calculate the distance from the left support to the load and the distance between supports.

MACHINE DESIGN - An Integrated Approach, 4th Ed.

4-43-2

a 0.5 Lfork L Lfork

a 600 mm L 1200 mm

7. From inspection of Figure 4-43A, write the load function equation q(x) = R1<x>-1 - F<x - a>-1 + R2<x - L>-1 + M2<x - L>-2 8. Integrate this equation from - to x to obtain shear, V(x) V(x) = R1<x>0 - F<x - a>0 + R2<x - L>0 + M2<x - L>-1 9. Integrate this equation from - to x to obtain moment, M(x) M(x) = R1<x>1 - F<x - a>1 + R2<x - L>1 + M2<x - L>0 10. Integrate the moment function, multiplying by 1/EI, to get the slope. (x) = [R1<x>2/2 - F<x - a>2/2 + R2<x - L>2/2 + M2<x - L>1 + C3]/EI 11. Integrate again to get the deflection. y(x) = [R1<x>3/6 - F<x - a>3/6 + R2<x - L>3/6 + M2<x - L>2/2 + C3x + C4]/EI 12. Evaluate R1, R2, M2, C3 and C4 At x = 0 and x = L; y = 0, therefore, C4 = 0. At x = L, = 0 At x = L+, V = M = 0 Guess Given R 1 L 6 R 1 L 2
2 3

R1 1 kN

R2 1 kN
3

M2 1 kN m

C3 1 kN m

F ( L a) 6 F ( L a) 2

C3 L = 0 kN m

C3 = 0 kN m

R1 R2 F = 0 kN R1 L F ( L a ) M2 = 0 kN m

R1 R 2 Find R R M C 1 2 2 3 M2 C3
R1 2.913 kN 13. Define the range for x R2 6.409 kN M2 2.097 kN m C3 0.419 kN m
2

x 0 in 0.002 L L

14. For a Mathcad solution, define a step function S. This function will have a value of zero when x is less than z, and a value of one when it is greater than or equal to z. S ( x z) if ( x z 1 0 )

MACHINE DESIGN - An Integrated Approach, 4th Ed.

4-43-3

15. Write the shear and moment equations in Mathcad form, using the function S as a multiplying factor to get the effect of the singularity functions. V ( x) R1 S ( x 0 in) F S ( x a ) R2 S ( x L) M ( x) R1 S ( x 0 in) x F S ( x a ) ( x a ) R2 S ( x L) ( x L) 16. Plot the shear and moment diagrams. Shear Diagram
10

Moment Diagram
2 1 0

5 V ( x) kN 0 M ( x)

kN m 1 2 3

10

200

400

600 x mm

800

1000 1200

200

400

600 x mm

800

1000 1200

FIGURE 4-43B
Shear and Moment Diagrams for Problem 4-43

17. The maximum moment occurs at x = L,

Mmax M ( L)

Mmax 2.097 kN m

18. The bending stress is maximum at the support and is

Mmax c I

a 87.2 MPa

MACHINE DESIGN - An Integrated Approach, 4th Ed.

4-44-1

PROBLEM 4-44
Statement: Determine a suitable thickness for the V-links of the off-loading station of Figure P4-17 to limit their deflections at the tips to 10-mm in any position during their rotation. Two V-links support the roll, at the 1/4 and 3/4 points along the roll's length, and each of the V arms is 10-cm wide by 1-m long. The V arms are welded to a steel tube that is rotated by the air cylinder. See Problem 4-43 for more information. Roll OD Roll ID Roll length Roll density OD 0.90 m ID 0.22 m Lroll 3.23 m
3

Given:

Arm width Arm length Max tip deflection Mod of elasticity

wa 100 mm La 1000 mm

tip 10 mm
E 207 GPa

984 kg m

Assumptions: 1. The maximum deflection on an arm will occur just after it begins the transfer and just before it completes it, i.e., when the angle is either zero or 90 deg., but after the tip is no longer supported by the base unit. 2. At that time the roll is in contact with both arms ("seated" in the V) and will remain in that state throughout the motion. When the roll is in any other position on an arm the tip will be supported. 3. The arm can be treated as a cantilever beam with nonend load. 4. A single arm will never carry more than half the weight of a roll. 5. The pipe to which the arms are attached has OD = 160 mm. Solution: See Figure 4-44 and Mathcad file P0444.
450

1. Determine the weight of the roll and the load on each V-arm. W

OD ID Lroll g

W 18.64 kN F 9.32 kN

F 0.5 W

2. From Appendix B, Figure B-1, the tip deflection of a cantilever beam with a concentrated load located at a distance a from the support is ymax = F a
2

6 E I

( a 3 L)
370 = a

1000 = L F

where L is the beam length and I is the cross-section moment of inertia. In this case w a t a 12 and a 370 mm
3
M F

I= 3. Setting

FIGURE 4-44
Free Body Diagram used in Problem 4-44

ymax = tip

substituting for I and solving for ta


1

2 F a2 3 La a ta E tip wa
Let the arm thickness be

ta 31.889 mm ta 32 mm

MACHINE DESIGN - An Integrated Approach, 4th Ed.

4-45-1

PROBLEM 4-45
Statement: Determine the critical load on the air cylinder rod in Figure P4-17 if the crank arm that rotates it is 0.3 m long and the rod has a maximum extension of 0.5 m. The 25-mm-dia rod is solid steel with a yield strength of 400 MPa. State all assumptions. Rod length Rod diameter L 500 mm d 25 mm Young's modulus Yield strength E 207 GPa S y 400 MPa

Given:

Assumptions: 1. The rod is a fixed-pinned column. 2. Use a conservative value of 1 for the end factor (see Table 4-7 in text). Solution: 1. See Mathcad file P0445.

Calculate the slenderness ratio that divides the unit load vs slenderness ratio graph into Johnson and Euler regions. S rD 2 E Sy S rD 101.07

2.

Calculate the cross-section area and the moment of inertia. Area Moment of inertia A I

2
4 d d

A 490.87 mm
4

2 4

64

I 1.92 10 mm

3.

Using Table 4-7, calculate the effective column length. Leff 1 L Leff 500 mm

4.

Calculate the slenderness ratio for the column. Radius of gyration k I A Leff k k 6.25 mm

Slenderness ratio

S r

S r 80.00

Since the Sr for this column is less than SrD, it is a Johnson column. 5. Calculate the critical load using the Johnson equation.
2 S S Sy 1 y r Pcr A E 2

Pcr 134.8 kN

MACHINE DESIGN - An Integrated Approach, 4th Ed.

4-46-1

PROBLEM 4-46
Statement: The V-links of Figure P4-17 are rotated by the crank arm through a shaft that is 60 mm dia by 3.23 m long. Determine the maximum torque applied to this shaft during motion of the V-linkage and find the maximum stress and deflection for the shaft. See Problem 4-43 for more information. Paper roll dimensions OD 900 mm ID 220 mm Lroll 3230 mm Roll density
3

Given:

Shaft dims

d 60 mm Lshaft 3230 mm G 79 GPa

984 kg m

Modulus of rigidity

Assumptions: The greatest torque will occur when the link is horizontal and the paper roll is located as shown in Figure P4-17 or Figure 4-46. Solution: See Figure 4-46 and Mathcad file P0446.
y

1. Determine the weight of the roll on the V-arms. W

OD ID Lroll g

W 18.64 kN 2. Summing moments about the shaft center, T OD 2 W T 8.390 kN m


W

3. Calculate the polar moment of inertia. J

d
32

J 1.272 10 mm

4
Ry 60-mm-dia shaft 450.0

4. The maximum torsional stress will be at the outside diameter of the shaft. The radius of the OD is, r d 2 r 30 mm

FIGURE 4-46
Free Body Diagram used in Problem 4-46

5. Determine the maximum torsional stress using equation (4.23b).

max

Tr J

max 197.8 MPa

6. Use equation (4.24) to determine the angular shaft deflection.

T Lshaft J G

15.447 deg

MACHINE DESIGN - An Integrated Approach, 4th Ed.

4-47-1

PROBLEM 4-47
Statement: Given: Determine the maximum forces on the pins at each end of the air cylinder of Figure P4-17. Determine the stress in these pins if they are 30-mm dia and in single shear. Paper roll dimensions OD 0.90 m ID 0.22 m Lroll 3.23 m Pin diameter d 30 mm

Roll density

984 kg m

Assumptions: 1. The maximum force in the cylinder rod occurs when the transfer starts. 2. The cylinder and rod make an angle of 8 deg to the horizontal at the start of transfer. 3. The crank arm is 300 mm long and is 45 deg from vertical at the start of transfer. 4. The cylinder rod is fully retracted at the start of the transfer. At the end of the transfer it will have extended 500 mm from its initial position. Solution: See Figure 4-47 and Mathcad file P0447.
y

1. Determine the weight of the roll on the forks. W

OD ID Lroll g

W 18.64 kN 2. From the assumptions and Figure 4-47, the x and y distances from the origin to point A are, Rax 300 cos( 45 deg) mm Ray 300 sin( 45 deg) mm Rax 212.132 mm Ray 212.132 mm
212.1 W Rx x 212.1 A F 450.0 8

Ry

3. From Figure 4-47, the x distance from the origin to point where W is applied is,

FIGURE 4-47
Free Body Diagram at Start of Transfer for V-link of Problem 4-47

Rwx 4.

OD 2

Rwx 450 mm

Sum moments about the pivot point and solve for the compressive force in the cylinder rod. W Rwx F Rax sin( 8 deg) F Ray cos( 8 deg) = 0 F W Rwx Ray cos( 8 deg) Rax sin( 8 deg) F 46.469 kN

This is the shear force in the pins

MACHINE DESIGN - An Integrated Approach, 4th Ed.


5. Determine the cross-sectional area of the pins and the direct shear stress. Shear area A

4-47-2

d
4 F A

A 706.858 mm

Shear stress

65.7 MPa

MACHINE DESIGN - An Integrated Approach, 4th Ed.

4-48-1

PROBLEM 4-48
Statement: A 100-kg wheelchair marathon racer wants an exerciser that will allow indoor practicing in any weather. The design shown in Figure P4-18 is proposed. Two free-turning rollers on bearings support the rear wheels. A platform supports the front wheels. Design the 1-m-long rollers as hollow tubes of aluminum to minimize the height of the platform and also limit the roller deflections to 1 mm in the worst case. The wheelchair has 650-mm-dia drive wheels separated by a 700-mm track width. The flanges shown on the rollers limit the lateral movement of the chair while exercising and thus the wheels can be anywhere between those flanges. Specify suitable sized steel axles to support the tubes on bearings. Calculate all significant stresses. Mass of chair M 100 kg Wheel diameter d w 650 mm Track width Roller length T 700 mm Lr 1000 mm Maximum deflection Modulus elasticity Aluminum Steel

Given:

1 mm
Ea 71.7 GPa Es 207 GPa

Assumptions: 1. The CG of the chair with rider is sufficiently close to the rear wheel that all of the weight is taken by the two rear wheels. 2. The small camber angle of the rear wheels does not significantly affect the magnitude of the forces on the rollers. 3. Both the aluminum roller and the steel axle are simply supported. The steel axles that support the aluminum tube are fixed in the mounting block and do not rotate. The aluminum tube is attached to them by two bearings (one on each end of the tubes, one for each axle). The bearings' inner race is fixed, and the outer race rotates with the aluminum tube. Each steel axle is considered to be loaded as a simply supported beam. Their diameter must be less than the inner diameter of the tubes to fit the roller bearings between them. Solution: See Figures 4-48 and Mathcad file P0448.

W/2

F
FIGURE 4-48A

1. Calculate the weight of the chair with rider. Weight of chair W M g W 980.7 N

Free Body Diagram of One Wheel used in Problem 4-48

2. Calculate the forces exerted by the wheels on the rollers (see Figure 4-48A). From the FBD of a wheel, summing vertical forces 2 F cos( ) Let W 2 =0 then F W 4 cos( ) F 260.9 N

20 deg

3.

The worst condition (highest moment at site of a stress concentration) will occur when the chair is all the way to the left or right. Looking at the plane through the roller that includes the forces exerted by the wheels (the FBD is shown in Figure 4-48B) the reactions R1 and R2 come from the bearings, which are inside the hollow roller and are, themselves, supported by the steel axle.

4. Solving for the reactions. Let the distance from R1 to F be a 15 mm

MACHINE DESIGN - An Integrated Approach, 4th Ed.


M1 Fy R2 R2 Lr F ( a T ) F a = 0 R1 2 F R2 = 0 F (2 a T ) Lr R2 190.5 N
15 F 700 F

4-48-2

R2 1000

R1 2 F R2

R1 331.3 N

R1

FIGURE 4-48B
Free Body Diagram of One Tube used in Problem 4-48

5. The maximum bending moment will be at the right-hand load and will be Mrmax R2 Lr ( a T ) Mrmax 54.3 N m

Note, if the chair were centered on the roller the maximum moment would be Mc F Lr T 2 Mc 39.1 N m

and this would be constant along the axle between the two loads, F. 6. Note that the bearing positions are fixed regardless of the position of the chair on the roller. Because of symmetry, Ra1 R1 Ra2 R2 Ra1 331.3 N Ra2 190.5 N
R a1 1130 R a2 65 R1 1000 R2

7. The maximum bending moment occurs at R1 and is for b 65 mm Mamax Ra1 b Mamax 21.5 N m

FIGURE 4-48C
Free Body Diagram of One Axle used in Problem 4-48

8. Determine a suitable axle diameter. Let the factor of safety against yielding in the axle be Nsa 3 9. Tentatively choose a low-carbon steel for the axle, say AISI 1020, cold rolled with S y 393 MPa 10. At the top of the axle under the load R1 there is only a bending stress. Set this stress equal to the yield strength divided by the factor of safety.

x =

32 Mamax

Sy Nsa
1

d a

Solving for the axle diameter, d a

32 Nsa Mamax d a S y
d a 15 mm

d a 11.875 mm

Let the axle diameter be

made from cold-rolled AISI 1020 steel.

MACHINE DESIGN - An Integrated Approach, 4th Ed.


11. Suppose that bearing 6302 from Chapter 10, Figure 10-23. It has a bore of 15 mm and an OD of 42 mm. Thus, the inside diameter of the roller away from the bearings where the moment is a maximum will be d i 40 mm. This will provide a 1-mm shoulder for axial location of the bearings. 12. The maximum deflection of the roller will occur when the chair is in the center of the roller. For this case the reactions are both equal to the loads, F (see Figure 4-48D). The maximum deflection is at the center of the roller.

4-48-3

150 F

700 F

F 15 1000 F

FIGURE 4-48D
Free Body Diagram of Roller with Chair in the Center.

13. Write the load function and then integrate four times to get the deflection function. q(x) = F<x>-1 - F<x - a>-1 - F<x - b>-1 + F<x - L>-1 y(x) = F[<x>3 - <x - a>3 - <x - b>3 + <x - L>3 + C3x]/(6EI) where C3 = 1 L ( L a ) a L
3 3 3

14. Write the deflection function at x = L/2 for a 150 mm ymax =

L 3 6 Ea I 2
F

3 L a 1 ( L a) 3 a3 L3 2 2

15. Set this equation equal to the allowed deflection and solve for the required moment of inertia, I.

Lr 3 6 Ea 2
F

3 Lr 1 3 3 3 a Lr a a Lr 2 2

I 6.618 10 mm

16. Knowing the inside diameter of the tube, solve for the outside diameter.
1

I=

4 4 do di 64
d o 46 mm

d o

64 I d 4 i

d o 44.463 mm

Round this up to DESIGN SUMMARY Axles Material Diameter Length

Rollers AISI 1020 steel, cold-rolled d a 15 mm 1220 mm Material Outside diameter Inside diameter Length Spacing 2024-T4 aluminum d o 46 mm d i 40 mm 1040 mm c d w d o sin( ) c 238 mm

MACHINE DESIGN - An Integrated Approach, 4th Ed.

4-49a-1

PROBLEM 4-49a
Statement: A hollow, square column has the dimensions and properties below. Determine if it is a Johnson or an Euler column and find the critical load: (a) If its boundary conditions are pinned-pinned. (b) If its boundary conditions are fixed-pinned. (c) If its boundary conditions are fixed-fixed. (d) If its boundary conditions are fixed-free. Length of column Outside dimension Inside dimension Solution: 1. See Mathcad file P0449a. Material L 100 mm Yield strength so 4 mm si 3 mm Modulus of elasticity Steel S y 300 MPa E 207 GPa

Given:

Calculate the slenderness ratio that divides the unit load vs slenderness ratio graph into Johnson and Euler regions. S rD 2 E Sy S rD 116.7

2.

Calculate the cross-section area and the moment of inertia. Area Moment of inertia A so si I 1 12
2 2 4

A 7.00 mm

2 4

so si

I 14.58 mm

(a) pinned-pinned ends 3. Using Table 4-7, calculate the effective column length. Leff 1 L 4. Leff 100 mm

Calculate the slenderness ratio for the column. Radius of gyration k I A Leff k k 1.443 mm

Slenderness ratio

S r

S r 69.28

Since the S r for this column is less than S rD, it is a Johnson column. 5. Calculate the critical load using the Johnson equation.
2 1 S y S r Pcr A S y E 2

Pcr 1.73 kN

(b) fixed-pinned ends 6. Using Table 4-7, calculate the effective column length. Leff 0.8 L 7. Leff 80 mm

Calculate the slenderness ratio for the column. Radius of gyration k I A k 1.443 mm

MACHINE DESIGN - An Integrated Approach, 4th Ed.


Slenderness ratio S r Leff k S r 55.43

4-49a-2

Since the S r for this column is less than S rD, it is a Johnson column. 8. Calculate the critical load using the Johnson equation. Pcr A S y

2 Sy S r E 2

Pcr 1.86 kN

(c) fixed-fixed ends 9. Using Table 4-7, calculate the effective column length. Leff 0.65 L Leff 65 mm

10. Calculate the slenderness ratio for the column. Radius of gyration k S r I A Leff k k 1.443 mm S r 45.03

Slenderness ratio

Since the S r for this column is less than S rD, it is a Johnson column. 11. Calculate the critical load using the Johnson equation.
2 S S Sy 1 y r Pcr A E 2

Pcr 1.94 kN

(d) fixed-free ends 12. Using Table 4-7, calculate the effective column length. Leff 2.1 L Leff 210 mm

13. Calculate the slenderness ratio for the column. Radius of gyration k S r I A Leff k k 1.443 mm S r 145.49

Slenderness ratio

Since the S r for this column is greater than S rD, it is an Euler column. 14. Calculate the critical load using the Euler equation.

Pcr A

E
Sr
2

Pcr 676 N

MACHINE DESIGN - An Integrated Approach, 4th Ed.

4-50a-1

PROBLEM 4-50a
Statement: A hollow, round column has the dimensions and properties below. Determine if it is a Johnson or an Euler column and find the critical load: (a) If its boundary conditions are pinned-pinned. (b) If its boundary conditions are fixed-pinned. (c) If its boundary conditions are fixed-fixed. (d) If its boundary conditions are fixed-free. Length of column Outside diameter Inside diameter Solution: 1. See Mathcad file P0450a. L 1500 mm Material Yield strength od 20 mm id 14 mm Modulus of elasticity Steel S y 300 MPa E 207 GPa

Given:

Calculate the slenderness ratio that divides the unit load vs slenderness ratio graph into Johnson and Euler regions. S rD 2 E Sy S rD 116.7

2.

Calculate the cross-section area, moment of inertia, and the radius of gyration. Area A I

od id od id
4

A 160.22 mm I 5968 mm
4

Moment of inertia

64

I A

Radius of gyration

k 6.103 mm

3.

Define functions to determine column type and critical load. Type type S r "Euler" if S r S rD "Johnson" otherwise Pcr S r return A

Critical load

E
Sr
2

if type S r = "Euler"

2 S S S y 1 y r otherwise A E 2

(a) pinned-pinned ends 4. Using Table 4-7, calculate the effective column length. Leff 1 L 5. Leff 1500 mm

Calculate the slenderness ratio for the column. Slenderness ratio S r Leff k S r 245.77

6.

Determine the type and critical load using the functions defined above. type S r "Euler" Pcr S r 5.42 kN

MACHINE DESIGN - An Integrated Approach, 4th Ed.


(b) fixed-pinned ends 7. Using Table 4-7, calculate the effective column length. Leff 0.8 L 8. Leff 1200 mm

4-50a-2

Calculate the slenderness ratio for the column. Slenderness ratio S r Leff k S r 196.62

9.

Determine the type and critical load using the functions defined above. type S r "Euler" Pcr S r 8.47 kN

(c) fixed-fixed ends 10. Using Table 4-7, calculate the effective column length. Leff 0.65 L Leff 975 mm

11. Calculate the slenderness ratio for the column. Slenderness ratio S r Leff k S r 159.75

12. Determine the type and critical load using the functions defined above. type S r "Euler" (d) fixed-free ends 13. Using Table 4-7, calculate the effective column length. Leff 2.1 L Leff 3150 mm Pcr S r 12.8 kN

14. Calculate the slenderness ratio for the column. Slenderness ratio S r Leff k S r 516.12

15. Determine the type and critical load using the functions defined above. type S r "Euler" Pcr S r 1.23 kN

MACHINE DESIGN - An Integrated Approach, 4th Ed.

4-51a-1

PROBLEM 4-51a
Statement: A solid, rectangular column has the dimensions and properties below. Determine if it is a Johnson or an Euler column and find the critical load: (a) If its boundary conditions are pinned-pinned. (b) If its boundary conditions are fixed-pinned. (c) If its boundary conditions are fixed-fixed. (d) If its boundary conditions are fixed-free. Length of col. Thickness Height Solution: 1. See Mathcad file P0451a. L 100 mm t 10 mm h 20 mm Material Yield strength Modulus of elasticity Steel S y 300 MPa E 207 GPa

Given:

Calculate the slenderness ratio that divides the unit load vs slenderness ratio graph into Johnson and Euler regions. S rD 2 E Sy S rD 116.7

2.

Calculate the cross-section area, moment of inertia, and the radius of gyration. Area A h t I h t
3

A 200.00 mm I 1667 mm
4

Moment of inertia

12 I A

Radius of gyration

k 2.887 mm

3.

Define functions to determine column type and critical load. Type type S r "Euler" if S r S rD "Johnson" otherwise Pcr S r return A

Critical load

E
Sr
2

if type S r = "Euler"

2 1 S y S r A S y otherwise E 2

(a) pinned-pinned ends 4. Using Table 4-7, calculate the effective column length. Leff 1 L 5. Leff 100 mm

Calculate the slenderness ratio for the column. Slenderness ratio S r Leff k S r 34.64

6.

Determine the type and critical load using the functions defined above. type S r "Johnson" Pcr S r 57.36 kN

MACHINE DESIGN - An Integrated Approach, 4th Ed.


(b) fixed-pinned ends 7. Using Table 4-7, calculate the effective column length. Leff 0.8 L 8. Leff 80 mm

4-51a-2

Calculate the slenderness ratio for the column. Slenderness ratio S r Leff k S r 27.71

9.

Determine the type and critical load using the functions defined above. type S r "Johnson" Pcr S r 58.31 kN

(c) fixed-fixed ends 10. Using Table 4-7, calculate the effective column length. Leff 0.65 L Leff 65 mm

11. Calculate the slenderness ratio for the column. Slenderness ratio S r Leff k S r 22.52

12. Determine the type and critical load using the functions defined above. type S r "Johnson" (d) fixed-free ends 13. Using Table 4-7, calculate the effective column length. Leff 2.1 L Leff 210 mm Pcr S r 58.9 kN

13. Calculate the slenderness ratio for the column. Slenderness ratio S r Leff k S r 72.75

14. Determine the type and critical load using the functions defined above. type S r "Johnson" Pcr S r 48.34 kN

MACHINE DESIGN - An Integrated Approach, 4th Ed.

4-52a-1

PROBLEM 4-52a
Statement: A solid, circular column, loaded eccentrically, has the dimensions and properties below. Find the critical load: (a) If its boundary conditions are pinned-pinned. (b) If its boundary conditions are fixed-pinned. (c) If its boundary conditions are fixed-fixed. (d) If its boundary conditions are fixed-free. Length of column Outside diameter Eccentricity (t) Solution: 1. See Mathcad file P0452a. L 100 mm od 20 mm e 10 mm Material Yield strength Modulus of elasticity Steel S y 300 MPa E 207 GPa

Given:

Calculate the cross-section area, distance to extreme fiber, and the moment of inertia. Area Distance to extreme fiber Moment of inertia A

od

A 314.16 mm c 10 mm I 7854 mm
4

c 0.5 od I

64

od

4.

Calculate the radius of gyration and eccentricity ratio for the column. Radius of gyration k Er I A e c k
2

k 5.00 mm Er 4.0

Eccentricity ratio (a) pinned-pinned ends 3.

Using Table 4-7, calculate the effective column length. Leff 1 L Leff 100 mm

4.

Calculate the slenderness ratio for the column. Slenderness ratio S r Leff k S r 20.00

5.

Calculate the critical load using the Secant equation. Guess Given P= P 1 kN S y A 1 Er sec S r Pcr Find ( P)

4 E A Pcr 18.63 kN

(b) fixed-pinned ends 6. Using Table 4-7, calculate the effective column length. Leff 0.8 L 7. Leff 80 mm

Calculate the slenderness ratio for the column.

MACHINE DESIGN - An Integrated Approach, 4th Ed.


Slenderness ratio 8. S r Leff k S r 16.00

4-52a-2

Calculate the critical load using the Secant equation. Guess Given P= P 1 kN S y A 1 Er sec S r Pcr Find ( P)

4 E A Pcr 18.71 kN

(c) fixed-fixed ends 9. Using Table 4-7, calculate the effective column length. Leff 0.65 L Leff 65 mm

10. Calculate the slenderness ratio for the column. Slenderness ratio S r Leff k S r 13.00

11. Calculate the critical load using the Secant equation. Guess Given P= P 1 kN S y A 1 Er sec S r Pcr Find ( P) (d) fixed-free ends 12. Using Table 4-7, calculate the effective column length. Leff 2.1 L Leff 210 mm

4 E A
P Pcr 18.76 kN

13. Calculate the slenderness ratio for the column. Slenderness ratio S r Leff k S r 42

14. Calculate the critical load using the Secant equation. Guess Given P= P 1 kN S y A 1 Er sec S r Pcr Find ( P)

4 E A
P Pcr 17.93 kN

MACHINE DESIGN - An Integrated Approach, 4th Ed.

4-53-1

PROBLEM 4-53
Statement: Given: Design an aluminum, hollow, circular column for the conditions given below for (a) pinned-pinned ends and (b) fixed-free ends. Length of column Wall thickness Load supported Solution: 1. See Mathcad file P0453. Factor of safety L 3 m Yield strength t 5 mm F 900 N Modulus of elasticity FS 3 S yc 150 MPa E 71.7 GPa

Start by calculating the slenderness ratio that divides the unit load vs slenderness ratio graph into Johnson and Euler regions. S rD 2 E S yc S rD 97.136

2.

Using Table 4-7, calculate the effective column length. Leff 1 L Leff 3000 mm

3.

To start the iterative process, assume that the final design will be an Euler column with the critical load equal to FS*F. From equation 4.38b, Pcr =

E A k
L
2

and

k =

I A

Substituting for k2

Pcr =

E I
L
2 2

= FS F

Solving for I

Leff FS F

E
The required moment of inertia, assuming an Euler column is I 34339 mm 4.
4

Using the relationships given on the inside cover, solve for the outside diameter of the tube. Guess Given D 20 mm I= 64

4 4 D ( D 2 t)
D 30.64 mm

D Find ( D) 5.

Using this diameter, calculate the slenderness ratio and compare to S rD. If it is greater than S rD the assumption o an Euler column is correct, if not, recalculate using the Johnson equation. Inside diameter Area d D 2 t Ar d 20.64 mm
2

D d

Ar 402.7 mm

Radius of gyration

kr

I Ar Leff kr

kr 9.234 mm

Slenderness ratio

S r

S r 324.9

MACHINE DESIGN - An Integrated Approach, 4th Ed.

4-53-2

Since this is greater than S rD, the assumption of an Euler column is correct and the minimum outside diameter is D 30.64 mm (b) fixed-free ends 6. Using Table 4-7, calculate the effective column length. Leff 2.1 L 7. Leff 6300 mm

To start the iterative process, assume that the final design will be an Euler column with the critical load equal to FS*F. From equation 4.38b, Pcr =

E A k
L
2

and

k =

I A

Substituting for k2

Pcr =

E I
L
2 2

= FS F

Solving for I

Leff FS F

E
The required moment of inertia, assuming an Euler column is I 2 10 mm 8.
5 4

Using the relationships given on the inside cover, solve for the outside diameter of the tube. Guess Given D 20 mm I= 64

4 4 D ( D 2 t)
D 47.37 mm

D Find ( D) 9.

Using this diameter, calculate the slenderness ratio and compare to S rD. If it is greater than S rD the assumption of an Euler column is correct, if not, recalculate using the Johnson equation. Inside diameter Area d D 2 t Ar d 37.37 mm
2

D d

Ar 665.6 mm

Radius of gyration

kr

I Ar Leff kr

kr 15.084 mm

Slenderness ratio

S r

S r 417.7

Since this is greater than S rD, the assumption of an Euler column is correct and the minimum outside diameter is D 47.37 mm

MACHINE DESIGN - An Integrated Approach, 4th Ed.

4-54-1

PROBLEM 4-54
Statement: Three round, 1.25-in-dia bars are made of SAE 1030 hot-rolled steel but are of different lengths, 5 in, 30 in, and 60 in, respectively. They are loaded axially in compression. Compare the load supporting capability of the three bars if the ends are assumed to be: (a) Pinned-pinned. (b) Fixed-pinned. (c) Fixed-fixed. (d) Fixed-free. Outside diameter Lengths Material d 1.25 in L 5 in
1

Given:

L 30 in
2

L 60 in
3

i 1 2 3

SAE 1030 Steel

Yield strength Modulus of elasticity

S y 38 ksi E 30 10 psi
6

Solution: 1.

See Mathcad file P0454.

Calculate the slenderness ratio that divides the unit load vs. slenderness ratio graph into Johnson and Euler regions. S rD 2 E Sy S rD 124.8

2.

Calculate the cross-section area, moment of inertia, and the radius of gyration. Area A I

2
4 d d

A 791.73 mm I 49882 mm
4

Moment of inertia

64

Radius of gyration 3.

I A

k 7.938 mm

Define functions to determine column type and critical load. Type type S r "Euler" if S r S rD "Johnson" otherwise Pcr S r return A

Critical load

E
Sr
2

if type S r = "Euler"

2 S S S y 1 y r otherwise A E 2

(a) pinned-pinned ends 4. Using Table 4-7, calculate the effective column length.

Leff 1 L 5.

5 Leff 30 in 60

Calculate the slenderness ratio for the column.

MACHINE DESIGN - An Integrated Approach, 4th Ed.

4-54-2

Slenderness ratio

S r

Leff k

16 S r 96 192

6.

Determine the type and critical load using the functions defined above.

Type type S r
i

"Johnson" Type "Johnson" i "Euler"

Pcr S r lbf

46250 32844 9857

(b) fixed-pinned ends 7. Using Table 4-7, calculate the effective column length.

Leff 0.8 L

4.0 Leff 24.0 in 48.0

8.

Calculate the slenderness ratio for the column.

Slenderness ratio

S r

Leff k

12.8 S r 76.8 153.6

9.

Determine the type and critical load using the functions defined above.

Type type S r
i

"Johnson" Type "Johnson" i "Euler"

Pcr S r lbf

46388 37808 15401

(c) fixed-fixed ends 10. Using Table 4-7, calculate the effective column length.

Leff 0.65 L

3.3 Leff 19.5 in 39.0

11.

Calculate the slenderness ratio for the column.

Slenderness ratio

S r

Leff k

10.4 S r 62.4 124.8

MACHINE DESIGN - An Integrated Approach, 4th Ed.

4-54-3

12.

Determine the type and critical load using the functions defined above.

Type type S r
i

"Johnson" Type "Johnson" i "Johnson"

Pcr S r lbf

46471 40807 23329

(d) fixed-free ends 13. Using Table 4-7, calculate the effective column length.

Leff 2.1 L

10.5 Leff 63.0 in 126.0

14.

Calculate the slenderness ratio for the column.

Slenderness ratio

S r

Leff k

33.6 S r 201.6 403.2

15.

Determine the type and critical load using the functions defined above.

Type type S r
i

"Johnson" Type "Euler" i "Euler"

Pcr S r lbf

44944 8940 2235

MACHINE DESIGN - An Integrated Approach, 4th Ed.

4-55-1

PROBLEM 4-55
Statement:

_____

Figure P4-19 shows a 1.5-in-dia, 30-in-long steel rod subjected to tensile loads P = 10000 lb applied at each end of the rod, acting along its longitudinal Y axis and through the centroid of its circular cross section. Point A is 12 in below the upper end and point B is 8 in below A. For this bar with its loading, find: (a) All components of the stress tensor matrix (equation 4.1a) for a point midway between A and B. (b) The displacement of point B relative to point A. (c) The elastic strain in the section between A and B. (d) The total strain in the section between A and B. Tensile load Diameter Lengths P 10000 lbf Modulus of elasticity d 1.50 in L 30 in LA 12 in LAB 8 in E 30 10 psi
6

Given:

Solution: 1.

See Mathcad file P0455.

Calculate the cross-section area of the rod. A

d
4

A 1.767 in

2.

(a) The loading is simple axial tension so all components of the stress tensor are zero except yy, which is found using equation 4.7.

yy

P A

yy 5659 psi

This stress is uniform across the rod and has the same value at any cross section along the longitudinal axis except close to the ends where the load is applied. 3. (b) The displacement of point B relative to A can be found using equation 4.8. P LAB A E
3

sBA
4.

sBA 1.509 10

in

(c) The elastic strain in the rod can be found using Hooke's law (equation 2.2)

yy
E

1.886 10

5.

(d) Assuming that the yield strength of this steel is greater than yy, the strain calculated in step 4 is the total strain.

MACHINE DESIGN - An Integrated Approach, 4th Ed.

4-56-1

PROBLEM 4-56
Statement:

_____

The rod in Figure P4-19, with the loading of Problem 4-55, is subjected to a reduction of temperature from 80F to 20F after the load is applied. The coefficient of thermal expansion for steel is approximately 6 in/in/degF. Find: (a) All components of the stress tensor matrix (equation 4.1a) for a point midway between A and B. (b) The displacement of point B relative to point A. (c) The elastic strain in the section between A and B. (d) The total strain in the section between A and B. Temperature scale F 1 Tensile load Diameter Lengths Temperatures P 10000 lbf Modulus of elasticity d 1.50 in L 30 in LA 12 in LAB 8 in T1 80 F T2 20 F E 30 10 psi
6

Units: Given:

Solution: 1.

Coefficient of thermal expansion See Mathcad file P0456.

6 10

Calculate the cross-section area of the rod. A

d
4

A 1.767 in

2.

(a) The loading is simple axial tension so all components of the stress tensor are zero except yy, which is found using equation 4.7.

yy

P A

yy 5659 psi

This stress is uniform across the rod and has the same value at any cross section along the longitudinal axis except close to the ends where the load is applied. The change in temperature does not affect the stress since the ends are free. 3. (b) The displacement of point B relative to A can be found by summing equation 4.8 for the elastic portion and the thermal expansion equation from elementary mechanics of materials for the thermal portion.

sBA
4.

P LAB A E

T2 T1 LAB

sBA 1.371 10

in

(c) The elastic strain in the rod can be found using Hooke's law (equation 2.2)


5.

yy
E

1.886 10

(d) Assuming that there is no plastic strain in the rod, the total strain is the sum of the elastic strain found in step 4 plus the thermal strain.

tot T2 T1

tot 1.714 10

MACHINE DESIGN - An Integrated Approach, 4th Ed.

4-57-1

PROBLEM 4-57
Statement:

_____

Figure P4-20 shows a steel bar fastened to a rigid ground plane with two 0.25-in-dia hardened steel dowel pins. For P = 1500 lb, find: (a) The shear stress in each pin. (b) The direct bearing stress in each pin and hole. (c) The minimum value of dimension h to prevent tearout failure if the steel bar has a shear strength of 32500 psi. Pin diameter Distance between pins Thickness of bar d 0.250 in a 2.0 in t 0.25 in Applied load Shear strength of bar Distance from right pin to load P 1500 lbf S s 32.5 ksi b 4.0 in

Given:

Solution: 1.

See Mathcad file P0457.

Draw a free-body diagram and find the shear forces (reactions) on each pin.

a RL

RR P
Write equations 3.3b for the bar and solve for the reactions.

F:
RL 2.

RL RR P 0 b a P RL 3000 lbf

M:

RL a P b 0 RR 4500 lbf

RR P RL

Calculate the cross-section area of a pin. A

d
4

A 0.0491 in

3.

(a) Use equation 4.9 to determine the shear stress in each pin. Left pin

L R

RL A RR A

L 61.1 ksi R 91.7 ksi

Right pin 4.

(b) Calculate the bearing area from equation 4.10 and use it to determine the bearing stress in each pin. Bearing area Abear d t Abear 0.0625 in
2

RL Abear

L 48.0 ksi

MACHINE DESIGN - An Integrated Approach, 4th Ed.

4-57-2

RR Abear

R 72.0 ksi

5.

(c) The tearout area is

Atear 2

t , where (h - d)/2 is the distance from the edge of the hole to 2

h d

the outside of the bar. Substitute this area in equation 4.9 for the shear area and substitute the shear strength for xy, solving then for the unknown distance h. Left pin h L RL S s t RR S s t d h L 0.619 in

Right pin

h R

h R 0.804 in h min 0.804 in

Minimum value of h

h min h R

MACHINE DESIGN - An Integrated Approach, 4th Ed.

4-58-1

PROBLEM 4-58
Statement:

_____

Figure P4-20 shows a steel bar fastened to a rigid ground plane with two 0.25-in-dia hardened steel dowel pins. For P = 2200 lb, find: (a) The shear stress in each pin. (b) The direct bearing stress in each pin and hole. (c) The minimum value of dimension h to prevent tearout failure if the steel bar has a shear strength of 32500 psi. Pin diameter Distance between pins Thickness of bar d 0.250 in a 2.0 in t 0.25 in Applied load Shear strength of bar Distance from right pin to load P 2200 lbf S s 32.5 ksi b 4.0 in

Given:

Solution: 1.

See Mathcad file P0458.

Draw a free-body diagram and find the shear forces (reactions) on each pin.

a RL

RR P
Write equations 3.3b for the bar and solve for the reactions.

F:
RL 2.

RL RR P 0 b a P RL 4400 lbf

M:

RL a P b 0 RR 6600 lbf

RR P RL

Calculate the cross-section area of a pin. A

d
4

A 0.0491 in

3.

(a) Use equation 4.9 to determine the shear stress in each pin. Left pin

L R

RL A RR A

L 89.6 ksi R 134.5 ksi

Right pin 4.

(b) Calculate the bearing area from equation 4.10 and use it to determine the bearing stress in each pin. Bearing area Abear d t Abear 0.0625 in
2

MACHINE DESIGN - An Integrated Approach, 4th Ed.


RL Abear RR Abear

4-58-2

L R

L 70.4 ksi R 105.6 ksi

5.

t , where (h - d)/2 is the distance from the edge of the hole to 2 the outside of the bar. Substitute this area in equation 4.9 for the shear area and substitute the shear strength for xy, solving then for the unknown distance h.
(c) The tearout area is Left pin h L RL S s t RR S s t d h L 0.792 in

Atear 2

h d

Right pin

h R

h R 1.062 in h min 1.062 in

Minimum value of h

h min h R

MACHINE DESIGN - An Integrated Approach, 4th Ed.

4-59-1

PROBLEM 4-59
Statement:

_____

Figure P4-21 shows a rectangular section aluminum bar subjected to off-center forces P = 4000 N applied as shown. (a) Solve for the maximum normal stress in the mid-region of the bar well away from the eyes where the loads are applied. (b) Plot the normal stress distribution across the cross section at this mid-region. (c) Sketch a "reasonable" plot of the normal stress distribution across the cross section at the ends, close to the applied loads. Depth of bar h 40 mm Thickness of bar t 10 mm See Mathcad file P0459. Applied loads Location of eye P 4000 N d 35 mm (from bottom edge)

Given: Solution: 1.

Draw a free-body diagram of the bar, cut at any section along the length of the bar.
FACE OF CUT SURFACE P d 0.5h
M

SECTION CENTROIDAL AXIS

Equilibrium requires that there be a force directed along the centroidal axis of the cross section that is equal and opposite to the applied force and a bending moment to react the couple formed by the applied force and the reaction force. Thus, since the reaction moment is clockwise, M ( d 0.5 h ) P 2. M 60.000 N m

Calculate the cross-section area, moment of inertia, and distance from the centroid to the outer surface. A h t I t h
3

A 400.0 mm

2 4 4

12

I 5.333 10 mm c 20.000 mm

c 0.5 h 2.

(a) The normal stress on a section well away from the ends is a combination of uniform tension, as given by equation 4.7, and bending, as given by equation 4.11a.

( y )

M y I

P A

This will be a maximum at y = c. max ( c) 3.

max 32.5 MPa

(b) Plot the normal stress distribution across the cross section at the mid-region of the bar for y c c 1 mm c

MACHINE DESIGN - An Integrated Approach, 4th Ed.

4-59-2

NORMAL STRESS ON SECTION


40 30 Stress, MPa 20

( y )
MPa 10 0 10 20 20 10

0 y mm

10

20

30

Distance from neutral axis, mm

4.

(c) Sketch a "reasonable" plot of the normal stress distribution across the cross section at the ends, close to the applied loads. Use the "force flow" analogy show in Figures 4-37 and 4-38 as a guide to the stress distribution. Near the applied load the stress will be highly concentrated. As the distance from the point of load application increases the stress will become more evenly distributed.

MACHINE DESIGN - An Integrated Approach, 4th Ed.

4-60-1

PROBLEM 4-60
Statement:

_____

Figure P4-22 shows a bracket machined from 0.5-in-thick steel flat stock. It is rigidly attached to a support and loaded with P = 5000 lb at point D. Find: (a) The magnitude, location, and the plane orientation of the maximum normal stress at section A-A. (b) The magnitude, location, and the plane orientation of the maximum shear stress at section A-A. (c) The magnitude, location, and the plane orientation of the maximum normal stress at section B-B. (d) The magnitude, location, and the plane orientation of the maximum shear stress at section B-B. Distance from support to: Section A-A Point D d 8 in Depth of section h 3 in Applied load P 5000 lbf Centroid of B-B b 18.5 in a 10 in Thickness of section t 0.5 in

Given:

Assumptions: The bracket remains flat and does not buckle (out-of-plane) under the applied load. Solution: 1. See Mathcad file P0460.

Calculate the cross-section area and moment of inertia at sections A-A and B-B, which are the same. A h t A 1.500 in
2

t h

12

I 1.1250 in

2.

For parts (a) and (b), draw a free-body diagram of the portion of the bracket that is to the right of section A-A.

V a A M A h y

P
3. Use the equilibrium equations 3.3a to calculate the shear force and bending moment on section A-A.

F:

V P 0 V P V 5000 lbf

M:

P ( a d ) M 0 M P ( a d ) M 10000 in lbf

4.

(a) The maximum normal stress in the bracket at section A-A is determined using equation 4.11b. It is located a the bottom of the section and is oriented in the positive x direction, i.e., it is tensile. Distance from neutral axis to extreme fiber c 0.5 h c 1.500 in

MACHINE DESIGN - An Integrated Approach, 4th Ed.


M c I

4-60-2

Maximum normal stress 5.

max

max 13.33 ksi

(b) The maximum shear stress in the bracket at section A-A is either at the neutral axis (due to the transverse shear, which is a maximum at the NA) or it is at the top or bottom of the section (due to the bending stress at those points, which is numerically the same). At the neutral axis, using equation 4.14b

max

3 V 2 A

max 5.000 ksi

At the bottom edge the stress state is: x max, y 0 ksi, xy 0 ksi. Using equation 4.6a, the principal stresses are
2 x y 2 xy 2 2 x y 2 xy 2

x y
2

1 13.333 ksi

x y
2

2 0.000 ksi

3 0 ksi
And, from equation 4.6b, the maximum shear stress is

max

1 3
2

max 6.667 ksi

As seen from the Mohr's Circle in Figure 4-8, this stress is oriented 45 degrees from the positive x axis. 6. For parts (c) and (d), draw a free-body diagram of the portion of the bracket that is below section B-B.

F M

7.

Use the equilibrium equations 3.3a to calculate the normal force and bending moment on section B-B.

F:

F P 0

M:

P ( b d ) M 0

MACHINE DESIGN - An Integrated Approach, 4th Ed.


F P 8. F 5000 lbf M P ( b d ) M 52500 in lbf

4-60-3

(c) The maximum normal stress in the bracket at section B-B is a combination of uniform tension and bending and is determined by summing equations 4.7 and 4.11b. It is located at the left edge of the section and is oriented in the positive y direction, i.e., it is tensile. Distance from neutral axis to extreme fiber Maximum normal stress c 0.5 h F A c 1.500 in

max

M c I

max 73.33 ksi

9.

(d) The maximum shear stress in the bracket at section B-B is at the left edge of the section (due to the combined tensile and bending stresses). Since there is no transverse shear on this section, the shear stress at the neutral axis is zero. At the left edge the stress state is: x 0 ksi, y max, xy 0 ksi. Using equation 4.6a, the principal stresses are
2 x y 2 xy 2 2 x y 2 xy 2

x y
2

1 73.333 ksi

x y
2

2 0.000 ksi

3 0 ksi
And, from equation 4.6b, the maximum shear stress is

max

1 3
2

max 36.667 ksi

As seen from the Mohr's Circle in Figure 4-8, this stress is oriented 45 degrees from the positive x axis.

MACHINE DESIGN - An Integrated Approach, 4th Ed.

4-61-1

PROBLEM 4-61
Statement: Given: Distance from support to: Point D d 8 in Depth of section h 3 in Applied load P 5000 lbf

_____

For the bracket of Problem 4-60, solve for the deflection of point C. Point C a 18.5 in Thickness of section t 0.5 in Modulus of elasticity E 30 10 psi
6

Assumptions: 1. The bracket remains flat and does not buckle (out-of-plane) under the applied load. 2. The bracket can be modeled using its centroidal axis length dimensions. Solution: 1. See Mathcad file P0461.

Calculate the moment of inertia along the segment AC. I t h


3

12

I 1.1250 in

2.

Draw idealized free-body diagrams of the portions of the bracket from the support to point C and from point C to point D.

P a

MA A

MC

C P

d P C

MC

D P
3. Calculate the magnitude of the moments on segment AC using equilibrium equation 3.3a. MC P ( a d ) MA P d MC 52500 in lbf MA 40000 in lbf

MACHINE DESIGN - An Integrated Approach, 4th Ed.


4. From inspection of the FBD, write the load function equation q(x) = -MA<x - 0>-2 + P<x - 0>-1 - P<x - a>-1 - MC<x - a>-2 5. Integrate this equation from - to x to obtain shear, V(x) V(x) = -MA<x - 0>-1 + P<x - 0>0 - P<x - a>0 - MC<x - a>-1 6. Integrate this equation from - to x to obtain moment, M(x) M(x) = -MA<x - 0>0 + P<x - 0>1 - P<x - a>1 - MC<x - a>0 7. Integrate the moment function, multiplying by 1/EI, to get the slope. (x) = [-MA<x-0>1 + P<x - 0>2/2 - P<x - a>2/2 - MC<x-a>1 + C3]/EI 8. Integrate again to get the deflection. y(x) = [-MA<x-0>2/2 + P<x - 0>3/6 - P<x - a>3/6 - MC<x-a>2/2 + C3x +C4]/EI 9. Evaluate C3 and C4. At x = 0, = 0 and y = 0, therefore, C3 = 0 and C4 = 0.

4-61-2

10. Evaluate and y at x = a using the equations in steps 7 and 8, respectively.

1 E I 1

MA a

P 2

C 0.196 deg

yC

MA 2 P 3 a a E I 2 6

yC 0.0465 in

MACHINE DESIGN - An Integrated Approach, 4th Ed.

4-62-1

PROBLEM 4-62
Statement:

_____

Figure P4-23 shows a 1-in-dia steel bar supported and subjected to the applied load P = 500 lb. Solve for the deflection at the load and the slope at the roller support. Diameter Applied load Modulus of elasticity d 1.00 in Dimensions: P 500 lbf a 20 in E 30 10 psi L 40 in
6

Given: Solution: 1.

See Mathcad file P0462.

Draw a free-body diagram.


L a R2

M1 R1 P

2.

This is a statically indeterminate beam because there are three unknown reactions, R1, M1, and R2. To solve for these unknowns, follow the method presented in Example 4-7. First, calculate the moment of inertia for the round section. I

d
64

I 0.0491 in

3.

From inspection of the FBD, write the load function equation q(x) = -M1<x>-2 + R1<x>-1 - R2<x - a>-1 + P<x - L>-1

4.

Integrate this equation from - to x to obtain shear, V(x) V(x) = -M1<x>-1 + R1<x>0 - R2<x - a>0 + P<x - L>0

5.

Integrate this equation from - to x to obtain moment, M(x) M(x) = -M1<x>0 + R1<x>1 - R2<x - a>1 + P<x - L>1

6.

Integrate the moment function, multiplying by 1/EI, to get the slope. (x) = [ -M1<x>1 + R1<x>2/2 - R2<x - a>2/2 + P<x - L>2/2 + C3]/EI

7.

Integrate again to get the deflection. y(x) = [-M1<x>2/2 + R1<x>3/6 - R2<x - a>3/6 + P<x - L>3/6 + C3x + C4]/EI

8.

Evaluate R1, M1, R2, C3 and C4 At x = 0, y = 0 and = 0, therefore, C3 = 0 and C4 = 0. At x = a, y = 0 At x = L+, V = M = 0 Guess Given M1 1000 in lbf y(a) = 0: M1 2 a
2

R1 500 lbf R1 6 a = 0 lbf in


3 3

R2 1000 lbf

V(L) = 0:

R1 R2 P = 0 lbf

MACHINE DESIGN - An Integrated Approach, 4th Ed.

4-62-2

M(L) = 0:

M1 R1 L R2 ( L a ) = 0 lbf in

M1 R1 Find M1 R1 R2 R 2
9.

M1 5000 in lbf

R1 750 lbf R2 1250 lbf

Evaluate y at x = L to get the deflection at the load.

yL

M1 2 R1 3 R2 3 L L ( L a) E I 2 6 6
1

yL 1.584 in

10. Evaluate at x = a to get the slope at the roller support.

E I
1

M1 a

R1 2

A 0.0340 rad

MACHINE DESIGN - An Integrated Approach, 4th Ed.

4-63-1

PROBLEM 4-63
Statement:

_____

Figure P4-24 shows a 1.25-in-dia solid steel shaft with several twisting couples applied in the directions shown. For TA = 10000 lb-in, TB = 20000 lb-in, TC = 30000 lb-in, find: (a) The magnitude and location of the maximum shear stress in the shaft. (b) The corresponding principal stresses for the location determined in part (a). (c) The magnitude and location of the maximum shear strain in the shaft. Modulus of rigidity Shaft diameter d 1.25 in Torque magnitudes: TA 10 kip in TB 20 kip in Segment lengths: LAB 18 in LBC 12 in See Mathcad file P0463. G 11.7 10 psi TC 30 kip in LCD 10 in
6

Given:

Solution: 1.

Looking at the shaft from the left end (A), TA and TC are clockwise (negative) and TB is counterclockwise (positive). For the shaft to be in equilibrium, the applied torques must sum to zero. Write the equilibrium equation and solve for the unknown reaction TD. TA TB TC TD 0 TD TA TB TC TD 20 kip in

2.

The net torque on each shaft segment is now TAB TA TBC TAB TB TCD TBC TC TAB 10 kip in TBC 10 kip in TCD 20 kip in

3.

Calculate the outside radius and the polar moment of inertia of the shaft. r d 2 r 0.625 in J

d
32

J 0.240 in

4.

(a) Since the shaft is uniform in cross-section, the maximum shear stress will occur in that segment that has the largest absolute value of torque applied to it. In this case, that is segment CD. Use equation 4.23b to calculate the maximum shear stress in segment CD.

max
5.

TCD r J

max 52.2 ksi

(b) Mohr's circle for pure shear is centered at 0,0 and has a radius equal to the shear stress on the stress element. Thus, for this case, the two nonzero principal stresses are

1 max

1 52.2 ksi

3 max

3 52.2 ksi

The third principal stress is zero, 2 0 ksi 6. (c) The shear strain in any given segment is proportional to the shear stress so the maximum shear strain will occur in segment CD, where the shear stress is a maximum. Hooke's law for shear is similar to that given in equation 2.2.

max

max
G

max 4.46 10

rad

MACHINE DESIGN - An Integrated Approach, 4th Ed.

4-64-1

PROBLEM 4-64
Statement:

_____

If the shaft of Problem 4-63 were rigidly attached to fixed supports at each end (A and D) and loaded only by the couples TB and TC, then find: (a) The reactions TA and TD at each end of the shaft. (b) The rotation of section B with respect to section C. (c) The magnitude and location of the maximum shear strain. Shaft diameter Torque magnitudes: Segment lengths: d 1.25 in TB 20 kip in LAB 18 in Modulus of rigidity TC 30 kip in LBC 12 in G 11.7 10 psi
6

Given:

LCD 10 in

Solution: 1.

See Mathcad file P0464.

Calculate the outside radius and the polar moment of inertia of the shaft. r d 2 r 0.625 in J

d
32

J 0.240 in

2.

(a) Looking at the shaft from the left end (A), TC is clockwise (negative) and TB is counterclockwise (positive). For the shaft to be in equilibrium, the applied torques must sum to zero. Since there are two unknown reactions in the equilibrium equation, we cannot solve for them without another equation. An equation that expresses the fact that the total rotational deflection from A to D is zero is called the compatibility equation. Write the equilibrium and compatibility equations and solve for the unknown reactions TA and TD. TA TB TC TD 0

AB BC CD 0
Guess TA 10 kip in

TA LAB J G

TA TB LBC
J G

TD LCD J G

TD 30 kip in

Given

TA TB TC TD = 0 kip in TA LAB J G

TA TB LBC
J G

TD LCD J G

= 0 rad

TA Find TA TD TD

TA 3.50 kip in TD 13.50 kip in

clockwise counterclockwise

3.

The net torque on each shaft segment is now TAB TA TBC TAB TB TCD TBC TC TAB 3.5 kip in TBC 16.5 kip in TCD 13.5 kip in

4.

(b) Use equation 4.24 to calculate the rotation of section B with respect to C.

MACHINE DESIGN - An Integrated Approach, 4th Ed.


TBC LBC J G

4-64-2

BC
5.

BC 0.0706 rad

BC 4.045 deg

(c) Since the shaft is uniform in cross-section, the maximum shear stress will occur in that segment that has the largest absolute value of torque applied to it. In this case, that is segment BC. Use equation 4.23b to calculate the maximum shear stress in segment BC.

max

TBC r J

max 43.0 ksi

The shear strain in any given segment is proportional to the shear stress so the maximum shear strain will occur in segment BC, where the shear stress is a maximum. Hooke's law for shear is similar to that given in equation 2.2.

max

max
G

max 3.68 10

rad

MACHINE DESIGN - An Integrated Approach, 4th Ed.

4-65-1

PROBLEM 4-65
Statement: Given:

_____

Figure P4-25 shows a pivot pin that is press-fit into part A and is slip fit in part B. If F = 100 lb and l = 1.50 in, what pin diameter is needed to limit the maximum stress in the pin to 50 kpsi? Applied force Total length, l F 100 lbf l 1.50 in Maximum stress 50 ksi Beam length L 0.5 l

Assumptions: 1. Since there is a slip fit between the pin and part B, part B offers no resistance to bending of the pin and, since the pin is press-fit into part A, it can be modeled as a cantilever beam of length l/2. 2. Part B distributes the concentrated force F so that, at the pin, it is uniformly distributed over the exposed length of the pin. Solution: 1. See Mathcad file P0465.

Calculate the intensity of the uniformly distributed load acting over the length of the pin. w F L w 133.3 lbf in

2.

A cantilever beam with uniform loading is shown in Figure B-1(b) in Appendix B. In this case, the dimension a in the figure is zero. As shown in the figure, when a = 0, the maximum bending moment occurs at the support and is Mmax w L 2
2

Mmax 37.50 lbf in

3.

The bending stress in a beam is given in equation 4.11c, which can be solved for the required section modulus, Z. Z Mmax Z 7.500 10 z= I c =
4

in

where, for a round cross-section


1

d
32

Solving for d,

d min

32 Z

d min 0.197 in

MACHINE DESIGN - An Integrated Approach, 4th Ed.

4-66-1

PROBLEM 4-66
Statement: Given:

_____

Figure P4-25 shows a pivot pin that is press-fit into part A and is slip fit in part B. If F = 100 N and l = 64 mm, what pin diameter is needed to limit the maximum stress in the pin to 250 MPa? Applied force Total length, l F 100 N l 64 mm Maximum stress 250 MPa Beam length L 0.5 l

Assumptions: 1. Since there is a slip fit between the pin and part B, part B offers no resistance to bending of the pin and, since the pin is press-fit into part A, it can be modeled as a cantilever beam of length l/2. 2. Part B distributes the concentrated force F so that, at the pin, it is uniformly distributed over the exposed length of the pin. Solution: 1. See Mathcad file P0466.

Calculate the intensity of the uniformly distributed load acting over the length of the pin. w F L w 3.125 N mm

2.

A cantilever beam with uniform loading is shown in Figure B-1(b) in Appendix B. In this case, the dimension a in the figure is zero. As shown in the figure, when a = 0, the maximum bending moment occurs at the support and is Mmax w L 2
2

Mmax 1600.0 N mm

3.

The bending stress in a beam is given in equation 4.11c, which can be solved for the required section modulus, Z. Z Mmax Z 6.400 mm z= I c =
3

where, for a round cross-section


1

d
32

Solving for d,

d min

32 Z

d min 4.0 mm

MACHINE DESIGN - An Integrated Approach, 4th Ed.

4-67-1

PROBLEM 4-67
Statement: Figure P4-25 shows a pivot pin that is press-fit into part A and is slip fit in part B. Determine the l/d ratio that will make the pin equally strong in shear and bending if the shear strength is equal to one-half the bending strength.

Assumptions: 1. Since there is a slip fit between the pin and part B, part B offers no resistance to bending of the pin and, since the pin is press-fit into part A, it can be modeled as a cantilever beam of length l/2. 2. Part B distributes the concentrated force F so that, at the pin, it is uniformly distributed over the exposed length of the pin. Solution: 1. See Mathcad file P0467.

The intensity of the uniformly distributed load acting over the exposed length of the pin is w 2 F l

2.

A cantilever beam with uniform loading is shown in Figure B-1(b) in Appendix B. In this case, the dimension a in the figure is zero. As shown in the figure, when a = 0, the maximum bending moment for a beam of length L occurs at the support and is Mmax = w L 2
2

1 2 F l F l = 2 l 2 4

3.

From equation 4.11c, the bending stress is

max =

M Z

F l 32 = 8 F l 4 3 d d 3

4.

Figure B-1(b) in Appendix B shows that the maximum shear occurs at the support and, for a = 0, is Vmax = w L =

2 F l = F l 2

5.

From equation 4.15c, the maximum shear stress due to the transverse loading is

max =

4 V 4 4 16 F = F = 3 A 3 2 2 d 3 d

6.

For equal shear and bending strength, let the shear stress equal one half the bending stress. 16 F 3 d Solving for l/d,
2

1 8 F l 2 3 d l d = 4 3

MACHINE DESIGN - An Integrated Approach, 4th Ed.

4-69-1

PROBLEM 4-69
Statement: Figure P4-26a shows a C-clamp with an elliptical body dimensioned as shown. The clamp has a T-section with a uniform thickness of 3.2 mm at the throat as shown in Figure P4-26b. Find the bending stress at the inner and outer fibers of the throat if the clamp force is 2.7 kN. Clamping force F 2.7 kN Distance from center of screw to throat Section dimensions: Solution: 1. ri 63.5 mm Web h 31.8 mm t 3.2 mm

Given:

Flange b 28.4 mm

See Figure P4-26 and Mathcad file P0469.

Determine the location of the CG of the T-section and the distance from the centerline of the screw to the centroid of the section at the throat. yCG 0.5 t ( b t) 0.5 ( h t) ( h t) t b t ( h t) t yCG 9.58 mm rc 73.08 mm

rc ri yCG 2.

Using equation 4.12a and Figure 4-16, calculate the distance to the neutral axis, rn, and the distance from the centroidal axis to the neutral axis, e. Distance from the screw centerline to the outside fiber Cross section area A b t ( h t) t rn A r
ri t

ro ri h A 182.4 mm
2

ro 95.30 mm

Distance to neutral axis

t dr dr r r r t
i

ro

rn 71.86 mm

Distance from centroidal to neutral axis 3.

e rc rn

e 1.21 mm

Take a section through the throat area and draw a FBD. There will be a vertical axial force through the section CG (at a distance rc from the screw centerline) which will form a couple of magnitude rc x F. This couple will be balanced by an internal moment of equal magnitude. Internal moment M rc F M 1.97 10 N mm
5

4.

Calculate the distances from the neutral axis to the inner and outer fibers. ci rn ri ci 8.364 mm co ro rn co 23.436 mm

5.

Using equations 4.12d and 4.12e, calculate the stresses at the inner and outer fibers of the throat section.

ci F e A ri A
M

i 132.2 MPa

co F e A ro A
M

o 204.3 MPa

MACHINE DESIGN - An Integrated Approach, 4th Ed.

4-70-1

PROBLEM 4-70
Statement: Given: A C-clamp as shown in Figure P4-26a has a rectangular cross section as in Figure P4-26c. Find the bending stress at the inner and outer fibers of the throat if the clamping force is 1.6 kN. Clamping force F 1.6 kN Distance from center of screw to throat Section dimensions: Solution: 1. ri 63.5 mm Depth h 31.8 mm

Width b 6.2 mm

See Figure P4-26 and Mathcad file P0470.

Determine the distance from the centerline of the screw to the centroid of the section at the throat. rc ri h 2 rc 79.40 mm

2.

Using equation 4.12a and Figure 4-16, calculate the distance to the neutral axis, rn, and the distance from the centroidal axis to the neutral axis, e. Distance from the screw centerline to the outside fiber Cross section area A b h rn A r
ro

ro ri h A 197.160 mm
2

ro 95.30 mm

Distance to neutral axis

rn 78.33 mm dr

b r

Distance from centroidal to neutral axis 3.

e rc rn

e 1.07 mm

Take a section through the throat area and draw a FBD. There will be a vertical axial force through the section CG (at a distance rc from the screw centerline) which will form a couple of magnitude rc x F. This couple will be balanced by an internal moment of equal magnitude. Internal moment M rc F M 1.27 10 N mm
5

4.

Calculate the distances from the neutral axis to the inner and outer fibers. ci rn ri ci 14.827 mm co ro rn co 16.973 mm

5.

Using equations 4.12d and 4.12e, calculate the stresses at the inner and outer fibers of the throat section.

ci F e A ri A
M

i 148.3 MPa

co F e A ro A
M

o 98.8 MPa

MACHINE DESIGN - An Integrated Approach, 4th Ed.

4-71-1

PROBLEM 4-71
Statement: A C-clamp as shown in Figure P4-26a has an elliptical cross section as in Figure P4-26d. Dimensions of the major and minor axes of the ellipse are given. Determine the bending stress at the inner and outer fibers of the throat if the clamping force is 1.6 kN. Clamping force F 1.6 kN Distance from center of screw to throat ri 63.5 mm Section dimensions: Solution: 1. Width b 9.6 mm Depth h 31.8 mm

Given:

See Figure P4-26 and Mathcad file P0471.

Determine the distance from the centerline of the screw to the centroid of the section at the throat. rc ri h 2 rc 79.40 mm

2.

Using equation 4.12a and Figure 4-16, calculate the distance to the neutral axis, rn, and the distance from the centroidal axis to the neutral axis, e. Distance from the screw centerline to the outside fiber Cross section area b h A 2 2 rn A o 0.5 r rc 2 2 b 1 4 2 h dr r r
i

ro ri h A 239.766 mm
2

ro 95.30 mm

Distance to neutral axis

rn 78.595 mm

Distance from centroidal to neutral axis 3.

e rc rn

e 0.805 mm

Take a section through the throat area and draw a FBD. There will be a vertical axial force through the section CG (at a distance rc from the screw centerline) which will form a couple of magnitude rc x F. This couple will be balanced by an internal moment of equal magnitude. Internal moment M rc F M 1.27 10 N mm
5

4.

Calculate the distances from the neutral axis to the inner and outer fibers. ci rn ri ci 15.095 mm co ro rn co 16.705 mm

5.

Using equations 4.12d and 4.12e, calculate the stresses at the inner and outer fibers of the throat section.

ci F e A ri A
M

i 163.2 MPa

co F e A ro A
M

o 108.7 MPa

MACHINE DESIGN - An Integrated Approach, 4th Ed.

4-72-1

PROBLEM 4-72
Statement: A C-clamp as shown in Figure P4-26a has a trapezoidal cross section as in Figure P4-26e. Determine the bending stress at the inner and outer fibers of the throat if the clamping force is 1.6 kN. Clamping force F 1.6 kN Distance from center of screw to throat ri 63.5 mm Section dimensions: Solution: 1. Width b i 9.6 mm b o 3.2 mm Depth h 31.8 mm

Given:

See Figure P4-26 and Mathcad file P0472.

Determine the distance from the centerline of the screw to the centroid of the section at the throat. rc ri h bi 2 bo 3 bi bo rc 76.75 mm

2.

Using equation 4.12a and Figure 4-16, calculate the distance to the neutral axis, rn, and the distance from the centroidal axis to the neutral axis, e. Distance from the screw centerline to the outside fiber Cross section area A bi bo 2 h A o bi bo r ri bi h dr r r
i

ro ri h A 203.520 mm
2

ro 95.30 mm

Distance to neutral axis

rn

rn 75.771 mm

Distance from centroidal to neutral axis 3.

e rc rn

e 0.979 mm

Take a section through the throat area and draw a FBD. There will be a vertical axial force through the section CG (at a distance rc from the screw centerline) which will form a couple of magnitude rc x F. This couple will be balanced by an internal moment of equal magnitude. Internal moment M rc F M 1.228 10 N mm
5

4.

Calculate the distances from the neutral axis to the inner and outer fibers. ci rn ri ci 12.271 mm co ro rn co 19.529 mm

5.

Using equations 4.12d and 4.12e, calculate the stresses at the inner and outer fibers of the throat section.

ci F e A ri A
M

i 126.9 MPa

co F e A ro A
M

o 118.4 MPa

MACHINE DESIGN - An Integrated Approach, 4th Ed.

4-73-1

PROBLEM 4-73
Statement: We want to design a C-clamp with a T-section similar to the one shown in Figure P4-26. The depth of the section will be 31.8 mm as shown but the width of the flange (shown as 28.4 mm) is to be determined. Assuming a uniform thickness of 3.2 mm and a factor of safety against static yielding of 2, determine a suitable value for the width of the flange if the C-clamp is to be made from 60-40-18 ductile iron and the maximum design load is 1.6 kN. Maximum clamping force F 1.6 kN Distance from center of screw to throat ri 63.5 mm t 3.2 mm

Given:

Section dimensions: Web h 31.8 mm Factor of safety N 2 Yield strength S y 324 MPa Solution: 1. See Figure P4-26 and Mathcad file P0473.

Determine the location of the CG of the T-section and the distance from the centerline of the screw to the centroid of the section at the throat as functions of the unknown flange width, b. yCG ( b ) 0.5 t ( b t) 0.5 ( h t) ( h t) t b t ( h t) t

rc( b ) ri yCG ( b ) 2. Using equation 4.12a and Figure 4-16, calculate the distance to the neutral axis, rn, and the distance from the centroidal axis to the neutral axis, e,as functions of b. Distance from the screw centerline to the outside fiber Cross section area A ( b ) b t ( h t) t rn( b ) A (b) i r
i

ro ri h

ro 95.3 mm

Distance to neutral axis

r t

o t dr dr r r r t
i

Distance from centroidal to neutral axis 3.

e( b ) rc( b ) rn( b )

Take a section through the throat area and draw a FBD. There will be a vertical axial force through the section CG (at a distance rc from the screw centerline) which will form a couple of magnitude rc x F. This couple will be balanced by an internal moment of equal magnitude. Internal moment M ( b ) rc( b ) F

4.

Calculate the distances from the neutral axis to the inner and outer fibers. ci( b ) rn( b ) ri co( b ) ro rn( b )

5.

Using equations 4.12d and 4.12e, calculate the stresses at the inner and outer fibers of the throat section.

i( b )
6.

ci( b ) F A (b) e( b ) A ( b ) ri
M (b)

Set the tensile stress on the inner fiber equal to the yield strength divided by the factor of safety and solve for the flange width, b.

MACHINE DESIGN - An Integrated Approach, 4th Ed.


Guess Given b 12 mm

4-73-2

i( b ) =

Sy N

b Find ( b )

b 10.13 mm

7.

Using the calculated value of b, check the stresses at the inner and outer fibers..

i( b )

ci( b ) F e( b ) A ( b ) ri A ( b )
M (b)

i( b ) 162 MPa

o( b )

c o( b ) F A ( b) e( b ) A ( b ) ro
M ( b)

o( b ) 149.4 MPa

A suitable minimum value for the flange width is b 10.1 mm

MACHINE DESIGN - An Integrated Approach, 4th Ed.

4-74-1

PROBLEM 4-74
Statement: A round steel bar is 10 in long and has a diameter of 1 in. (a) Calculate the stress in the bar when it is subjected to a 1000-lb force in tension. (b) Calculate the bending stress in the bar if it is fixed at one end (as a cantilever beam) and has a 1000-lb transverse load at the other end. (c) Calculate the transverse shear stress in the bar of part (b). (d) Calculate the torsional shear stress when the 1000-lb force is displaced 10 inches radially from the centerline (axis) of the cantilever beam. (e) Calculate the maximum bending stress in the bar if it is formed into a semicircle with a centroidal radius of 10/ in and 1000-lb opposing forces are applied at the ends in the plane of the of the ends. Assume that there is no distortion of the cross section during bending. (f) Calculate the direct bearing stress that would result on the bar of (a) if it were the pin in a pin-and-clevis connection that is subjected to a 1000-lb pull if the center part (the eye or tongue) is 1-in wide. (g) Determine how short the bar must be when loaded as a cantilever beam for its maximum flexural bending stress and its maximum transverse shear stress to provide equal tendency to failure. Find the length as a fraction of the diameter if the failure stress in shear is half the failure stress in bending. (h) If the force on the cantilever beam in (f) is eccentric, inducing torsional as well as bending stress, what fraction of the diameter would the eccentricity need to be in order to give a torsional stress equal to the transverse shear stress? Length of bar L 10 in Force F 1000 lbf See Mathcad file P0474. Diameter Load Radius d 1.00 in R 10 in

Given:

Solution:

(a) Use equation 4.7 to calculate the axial stress. Cross sectional area A d
2

A 0.785 in

Axial stress

F A

1.27 ksi

(b) The beam loading diagram is shown in Appendix Figure B-1a with the concentrated load at a = L. The maximum bending stress occurs at x = 0 and is given by Equation 4.11b. Bending moment Radius of bar M L F c d 2 d
4

M 10000 in lbf c 0.5 in I 0.049 in M c I


4

Moment of inertia

64

Maximum bending stress

101.9 ksi

(c) The maximum transverse shear stress occurs at y = 0 and is given by Equation 4.15c and in Figure 4-20b. Maximum transverse shear stress in a solid, round bar

max

4 F 3 A

max 1.70 ksi

(d) The maximum torsional shear shear stress occurs at y = d/2 and is given by Equation 4.23b.

MACHINE DESIGN - An Integrated Approach, 4th Ed.


Twisting torque Polar moment of inertia T F R J d
4

4-74-2
T 10000 in lbf J 0.098 in
4

32

Max torsional shear stress

Td 2 J

50.93 ksi

(e) The maximum bending stress for a curved beam occurs at r = ri and is given by Equation 4.12d. Centroidal radius Inside radius Outside radius Cross section area rc 10 in rc 3.183 in ri 2.683 in ro 3.683 in A 0.785 in
2

ri rc 0.5 d ro rc 0.5 d A d
2

4 A

Distance to neutral axis

rn

r Distance from centroid to neutral axis Internal moment M rc F

ro

rn 3.163 in

d r r 2 c 2 dr
r

e rc rn M 3183 in lbf co ro rn

e 0.020 in

Distances from the neutral axis to the inner and outer fibers ci rn ri ci 0.480 in co 0.520 in

Stress at the inner fibers of the throat section

ci F e A ri A
M l 1 in

i 37.9 ksi

(f) The direct bearing stress is given in Equations 4.7 and 4.10. Given length of bearing contact Projected area of contact Bearing stress

Abearing l d

Abearing 1 in F

bearing

Abearing

bearing 1.0 ksi

(g) Determine how short the bar must be when loaded as a cantilever beam for its maximum flexural bending stress and its maximum transverse shear stress to provide equal tendency to failure. Find the length as a fraction of the diameter if the failure stress in shear is half the failure stress in bending.

MACHINE DESIGN - An Integrated Approach, 4th Ed.


M c I 32 F L

4-74-3

Bending stress

Transverse shear

4 F 16 F = 3 A 2 3 d 32 F L = 32 3 F

Equating

= 2

Solving for L

L=

d 3

(h) If the force on the cantilever beam in (f) is eccentric, inducing torsional as well as bending stress, what fraction of the diameter would the eccentricity need to be in order to give a torsional stress equal to the transverse shear stress? Torsional shear stress

tor =

Tc J

16 F e

Transverse shear

trans =

4 F 16 F = 3 A 2 3 d 16 F e 16 F 3 d
2

Equating

tor = trans

d
d 3

Solving for the eccentricity, e

e=

MACHINE DESIGN - An Integrated Approach, 4th Ed.

4-75a-1

PROBLEM 4-75a
Statement: For a filleted flat bar in tension similar to that shown in Appendix Figure C-9 and the data from row a from Table P4-4, determine the nominal stress, the geometric stress concentration factor, and the maximum axial stress in the bar. Widths Thickness Force D 40 mm h 10 mm P 8000 N d 20 mm Radius r 4 mm

Given:

Solution: 1.

See Appendix Figure C-9 and Mathcad file P0475a.

Determine the nominal stress in the bar using equation 4.7.

nom
2.

P h d

nom 40.0 MPa

Determine the geometric stress concentration factor using Appendix Figure C-9. Width ratio From Figure E-9 SCF D d 2.00 b 0.32077
b

A 1.0966 r Kt A d

Kt 1.838

3.

Determine the maximum stress in the bar using equation 4.31.

max Kt nom

max 73.5 MPa

MACHINE DESIGN - An Integrated Approach, 4th Ed.

4-76a-1

PROBLEM 4-76a
Statement: For a filleted flat bar in bending similar to that shown in Appendix Figure C-10 and the data from row a from Table P4-4, determine the nominal stress, the geometric stress concentration factor, and the maximum bending stress in the bar. Widths Thickness Moment D 40 mm h 10 mm M 80 N m d 20 mm Radius r 4 mm

Given:

Solution: 1.

See Appendix Figure C-10 and Mathcad file P0476a.

Determine the nominal stress in the bar using equation 4.11b. c d 2 M c I c 10 mm I h d


3

12

I 6.667 10 mm

nom
2.

nom 120.0 MPa

Determine the geometric stress concentration factor using Appendix Figure C-10. Width ratio From Figure E-9 SCF D d 2.00 b 0.30304 Kt 1.518

A 0.93232 Kt A

3.

Determine the maximum stress in the bar using equation 4.31.

max Kt nom

max 182.2 MPa

MACHINE DESIGN - An Integrated Approach, 4th Ed.

4-77a-1

PROBLEM 4-77a
Statement: For a shaft, with a shoulder fillet, in tension similar to that shown in Appendix Figure C-1 and the data from row a from Table P4-4, determine the nominal stress, the geometric stress concentration factor, and the maximum axial stress in the shaft. Widths Radius Force D 40 mm r 4 mm P 8000 N d 20 mm

Given:

Solution: 1.

See Appendix Figure C-1 and Mathcad file P0477a.

Determine the nominal stress in the bar using equation 4.7.

nom
2.

4 P

nom 25.5 MPa

Determine the geometric stress concentration factor using Appendix Figure C-1. Width ratio From Figure E-1 SCF D d 2.00 b 0.30035 Kt 1.645

A 1.01470 r Kt A d
b

3.

Determine the maximum stress in the bar using equation 4.31.

max Kt nom

max 41.9 MPa

MACHINE DESIGN - An Integrated Approach, 4th Ed.

4-78a-1

PROBLEM 4-78a
Statement: For a shaft, with a shoulder fillet, in bending similar to that shown in Appendix Figure C-2 and the data from row a from Table P4-4, determine the nominal stress, the geometric stress concentration factor, and the maximum bending stress in the shaft. Widths Radius Moment D 40 mm r 4 mm M 80 N m d 20 mm

Given:

Solution: 1.

See Appendix Figure C-2 and Mathcad file P0478a.

Determine the nominal stress in the bar using equation 4.11b. c d 2 M c I c 10 mm I

d
64

I 7.854 10 mm

nom
2.

nom 101.9 MPa

Determine the geometric stress concentration factor using Appendix Figure C-2. Width ratio From Figure E-2 SCF D d 2.00 b 0.28598 Kt 1.44

A 0.90879 Kt A

3.

Determine the maximum stress in the bar using equation 4.31.

max Kt nom

max 146.7 MPa

MACHINE DESIGN - An Integrated Approach, 4th Ed.

4-79-1

PROBLEM 4-79
Statement: A differential stress element has a set of applied stresses on it as shown in Figure 4-1. For x = 850, y = -200, z = 300, xy = 450, yz = -300, and zx = 0; find the principal stresses and maximum shear stress and draw the Mohr's circle diagram for this three-dimensional stress state.

Given:

x 850 xy 450

y 200 yz 300

z 300 zx 0

Solution:

See Figure 4-1 and Mathcad file P04079.

1. Calculate the coefficients (stress invariants) of equation (4.4c). C2 x y z C2 950.000


5

C1

x xy x zx y yz xy y zx z yz z

C1 2.675 10

x xy zx C0 xy y yz zx yz z
2. Find the roots of the triaxial stress equation:
3 2

C0 1.882 10

C2 C1 C0 = 0

C0 C1 v C2 1
3. Extract the principal stresses from the vector r by inspection.

r polyroots ( v)

470 r 388 1032


CW

1 r 2 r 3 r

3 2 1

1 1032 2 388 3 470


-500
3

1-3

500
2-3 1-2

500 0
2

1000
1

1500

4. Using equations (4.5), evaluate the principal shear stresses.

13 12 23

1 3
2

13 751
500

1 2
2

12 322 23 429

2 3
2

CCW

5. Draw the three-circle Mohr diagram.

FIGURE 4-79
The Three Mohr's Circles for Problem 4-79

MACHINE DESIGN - An Integrated Approach, 4th Ed.

4-80-1

PROBLEM 4 - 80
Statement: Write expressions for the normalized (stress/pressure) tangential stress as a function of the normalized wall thickness (wall thickness/outside radius) at the inside wall of a thick-wall cylinder and for a thin-wall cylinder, both with internal pressure only. Plot the ratio of these two expressions and determine the range of the wall thickness to outside radius-ratio for which the stress predicted by the thin-wall expression is at least 5% greater than that predicted by the thick-wall expression. See Mathcad file P0480.

Solution: 1.

Let the t/p ratio be S' and the t/ro ratio be t', then For the thick-wall cylinder at the inside wall, using equation 4.48a S'thick ( t') 2 2 t' t' 2 t' t'
2 2

and, for the thin-wall cylinder, using equation 4.49a S'thin ( t') 2. 3. 1 t'

Choose a range for the normalized thickness ratio, t' 0.01 0.02 0.99 Plot the difference between the two functions. ( t')

S'thin( t') S'thick( t')


S'thick ( t')

25

20

15

( t')
% 10

0.1

0.2

0.3

0.4

0.5 t'

0.6

0.7

0.8

0.9

MACHINE DESIGN - An Integrated Approach, 4th Ed.

4-80-2

4.

Determine the values of t' for which the difference is 5%.

( 0.10) 5.0 %
5.

( 0.946 ) 5.1 %

The range of the normalized thickness for which the thin-wall stress is at least 5% greater than the thick-wall stress is from 0.10 to 0.946.

MACHINE DESIGN - An Integrated Approach, 4th Ed.

4-81-1

PROBLEM 4 - 81
Statement: A hollow square torsion bar such as that shown in Table 4-3 has dimensions a = 25 mm, t = 3 mm, and l = 300 mm. If it is made of steel with a modulus of rigidity of G = 80.8 GPa, determine the maximum shear stress in the bar and the angular deflection under a torsional load of 500 N-m. Dimensions Modulus a 25 mm G 80.8 GPa t 3 mm Load l 300 mm T 500 N m

Given: Solution: 1.

See Table 4-3 Mathcad file P0481.

Calculate the factors K and Q for a hollow square from Table 4-3. 2 t ( a t) 2 a t 2 t Q 2 t ( a t)
2 4 4

K 31944 mm

Q 2904 mm

2.

Using equation 4.26a, calculate the maximum shear stress.

max
3.

T Q

max 172.2 MPa

Using equation 4.26b, calculate the angular deflection. Tl K G

0.058 radians 3.33 deg

MACHINE DESIGN - An Integrated Approach, 4th Ed.

4-82-1

PROBLEM 4 - 82
Statement: Design a hollow rectangular torsion bar such as that shown in Table 4-3 that has dimensions a = 45 mm, b = 20 mm, and l = 500 mm. It is made of steel with a shear yield strength of 90 MPa and has an applied torsional load of 135 N-m. Use a factor of safety against yielding of 2. Dimensions a 45 mm b 20 mm l 500 mm Modulus G 80.8 GPa Load T 135 N m Shear yield strength S sy 90 MPa Factor of safety N 2 See Table 4-3 Mathcad file P0482.

Given:

Solution: 1.

Calculate the Q-factor for a hollow rectangle from Table 4-3. Q( t) 2 t ( a t) ( b t)

2.

Calculate the maximum shear stress as a function of thickness, t, using equation 4.26a.

( t)
3.

T Q( t)

Define a function that relates the maximum shear stress to the shear strength divided by the factor of safety and solve for the thickness, t. Guess a value of t t 3 mm f ( t) ( t) S sy N

Define the design function

t root( f ( t) t)

t 1.927 mm

Let t = 2 mm (note that this solution does not check for buckling under the applied load)

MACHINE DESIGN - An Integrated Approach, 4th Ed.

4-83-1

PROBLEM 4 - 83
Statement: A pressure vessel with closed ends has the following dimensions: outside diameter, OD = 450 mm, and wall thickness, t = 6 mm. If the internal pressure is 690 kPa, find the principal stresses on the inside surface away from the ends. What is the maximum shear stress at the point analyzed? Dimensions Pressure OD 450 mm p 690 kPa t 6 mm

Given: Solution: 1.

See Mathcad file P0483.

Convert the given dimensions to inside and outside radii. ro 0.5 OD ri ro t ro 225 mm ri 219 mm

2.

Determine whether to use thick-wall or thin-wall theory. ro 10 22.5 mm

Since the wall thickness, t 6 mm, is much less than one tenth the outside radius, use thin wall theory. 3. Calculate the principal stresses using equations 4.49. Tangential (y-direction) Radial (x-direction) Axial (z-direction) The principal stresses are:

p ro t

t 25.9 MPa r 0.0 MPa a 12.9 MPa

r 0 MPa a
p ro 2 t

1 t 2 a 3 r
4.

1 25.9 MPa 2 12.9 MPa 3 0.0 MPa

Using equation 4.6b, calculate the maximum shear stress.

max

1 3
2

max 12.9 MPa

MACHINE DESIGN - An Integrated Approach, 4th Ed.

4-84-1

PROBLEM 4-84
Statement: A simply supported steel beam of length, l, with a concentrated load, F, acting at midspan has a rectangular cross-section with width, b, and depth, h. If the strain energy due to transverse shear loading is Us and that due to bending loading is Ub, derive an expression for the ratio Us/Ub and plot it as a function of h/l over the range 0 to 0.10. See Mathcad file P0484. 2 3 V Us = dx 5 G A
0 l

Solution:

1.

From equation 4.22e, the strain energy in transverse loading is:

2.

From equation 4.22d, the strain energy in bending loading is:

2 1 M Ub = dx 2 E I
0

3.

Let

U' =

Us Ub

, then:

U' =

0 6 E I 5 G A l 2 M dx 0 3

2 V dx

4.

For a rectangular cross-section:

A = b h

and

I=

b h 12

5.

And, for steel:

E G

5 2

therefore

2 V dx 2 0 h U' = l 4 2 M dx
0

6.

For the given loading: For x between 0 and l/2, V= F 2 F 2 and and M= M= F x 2 F x 2 F l 2

For x between l/2 and l, 7.

V=

Substituting these expressions into the equation for U' and integrating gives:
l 0.5 l 2 2 F F 12.0 2 12.0 2 dx 2 dx h 2 2 2 2 h 0 0.5 l l 6.0 h l 4 4 0.5 l l 2 l 2 2 F x dx F x F l dx 2 2 2 0 0.5 l

MACHINE DESIGN - An Integrated Approach, 4th Ed.


8. Let h' = h l then U'( h') 6 h'
2

4-84-2

9.

Plotting the strain energy ratio over the range:

h' 0 0.001 0.10

STRAIN ENERGY RATIO vs DEPTH TO LENGTH RATIO


6

5 Strain Energy Ratio - Percent

U'( h' ) % 3

0.02

0.04 h'

0.06

0.08

0.1

Depth to Length Ratio

MACHINE DESIGN - An Integrated Approach, 4th Ed.

4-85a-1

PROBLEM 4-85a
Statement: Given: A beam is supported and loaded as shown in Figure P4-27(a). Find the reactions for the data given in row a from Table P4-2. Beam length Distance to R2 Distributed load magnitude L 1 m a 0.4 m w 200 N m
1

a w

R1

R2
FIGURE 4-85A
Free Body Diagram for Problem 4-85

R3

Solution: 1.

See Figure P4-27(a) and Mathcad file P0485a.

Consider the reaction force R1 to be redundant and remove it temporarily. The beam will then be statically determinant and will deflect at x = 0. Now consider the reaction force R1 to be an unkown applied load that will force the deflection to be zero. Write an equation for the deflection at x = 0 in terms of the force R1 with the deflection set to zero. Write equation 4.21 for the deflection y1 at the unknown applied load R1 in terms of the strain energy in the beam at that point: y1 =

2.

U R1
M E I

3.

Substitute equation 4.22d and differentiate:

y1 =

R1

M dx

(a)

4.

Write an expression for the bending moment and its partial derivative with respect toR1 as a function of x. For x between 0 and a, M = R 1 x w x 2 w x 2
2 2

M =x R1
R 2 ( x a )

(b)

For x between a and l, 5.

M = R 1 x

M =x R1

(c)

Substitute equations (b) and (c) into (a), set equal to zero and integrate.

MACHINE DESIGN - An Integrated Approach, 4th Ed.

4-85a-2

6.

2 R1 x w x x dx 2

2 R1 x w x R2 ( x a) x dx = 0 2

Solving for R1 and R2 and summing forces and moments about x = 0: R1 3 Summing forces m
3 4 L3 a L2 a 3 a3 R2 w L = 0 2 8 3 2 3

From strain energy

R1 R2 R3 w L = 0 R 2 a R 3 L w L 2
2

Summing moments 7.

=0 R1 65 N R2 70 N R3 65 N

Use these three equations to solve for R1, R2, and R3. Guess Given R1 3 m
3

4 L3 a L2 a 3 a3 R2 w L = 0 2 8 3 2 3

R1 R2 R3 w L = 0 R 2 a R 3 L w L 2
2

=0

R Find R1 R2 R3

10.714 R 148.81 N 40.476


R 40.5 N
3

R 10.7 N
1

R 148.8 N
2

MACHINE DESIGN - An Integrated Approach, 4th Ed.

4-86a-1

PROBLEM 4-86a
Statement: Given: A beam is supported and loaded as shown in Figure P4-27(b). Find the reactions for the data given in row a from Table P4-2. Beam length Distance to R2 Distributed load magnitude Distance to concentrated load Concentrated load L 1.0 m a 0.4 m w 200 N m b 0.6 m F 500 N
1

L b a F w

R1

R2
FIGURE 4-86A
Free Body Diagram for Problem 4-86

R3

Solution: 1.

See Figure P4-27(b) and Mathcad file P0485a.

Consider the reaction force R1 to be redundant and remove it temporarily. The beam will then be statically determinant and will deflect at x = 0. Now consider the reaction force R1 to be an unkown applied load that will force the deflection to be zero. Write an equation for the deflection at x = 0 in terms of the force R1 with the deflection set to zero. Write equation 4.21 for the deflection y1 at the unknown applied load R1 in terms of the strain energy in the beam at that point: y1 =

2.

U R1
M E I

3.

Substitute equation 4.22d and differentiate:

y1 =

R1

M dx

(a)

4.

Write an expression for the bending moment and its partial derivative with respect toR1 as a function of x. For x between 0 and a, M = R 1 x w x 2 a
2

M =x R1
2

(b)

For x between a and b,

M = R 1 x w a x

R 2 ( x a )

M =x R1

(c)

MACHINE DESIGN - An Integrated Approach, 4th Ed.


M = R 1 x w a x a

4-86a-2

For x between b and L,

R 2 ( x a ) F ( x b )
M =x R1

(d)

5.

Substitute equations (b), (c) and (d) into (a), set equal to zero and integrate.
a 2 R1 x w x x dx 2 L

R x w a x 1

R2 ( x a ) x dx = 0

6.

R x w a x 1

a 2

R2 ( x a ) F ( x b ) x dx

Solving for R1 and R2 and summing forces and moments about x = 0:


2 3 L3 3 R1 L a L a R2 =0 2 6 3 3 2 3 a2 L2 a L3 a4 3 w L b L b F 3 2 24 6 6 3

From strain energy

Summing forces Summing moments 7.

R1 R2 R3 w a F = 0 R 2 a R 3 L w a 2
2

F b = 0 R2 400 N R3 200 N

Use these three equations to solve for R1, R2, and R3. Guess R1 100 N Given

2 3 L3 3 R1 L a L a R2 =0 2 6 3 3 2 3 a2 L2 a L3 a4 3 w L b L b F 3 2 24 6 6 3

R1 R2 R3 w a F = 0 R 2 a R 3 L w a 2
2

F b = 0

R Find R1 R2 R3

81.143 R 575.238 N 85.905


R 85.9 N
3

R 81.1 N
1

R 575.2 N
2

MACHINE DESIGN - An Integrated Approach, 4th Ed.

5-1a-1

PROBLEM 5-1a
Statement: A differential stress element has a set of applied stresses on it as indicated in row a of Table P5-1. For row a, draw the stress element showing the applied stresses. Find the principal stresses and the von Mises stress.

Given:

x 1000 xy 500

y 0 yz 0

z 0 zx 0

Solution:

See Figure 5-1a and Mathcad file P0501a.

1. Draw the stress element, indicating the x and y axes. 2. From Problem 4-1a, the principal stresses are

500 y x 1000

1 1207

2 0

3 207

3. Using equatoion 5.7c, the von Mises stress is

'

1 1 3 3

' 1323

FIGURE 5-1aA
Stress Element for Problem 5-1a

MACHINE DESIGN - An Integrated Approach, 4th Ed.

5-1h-1

PROBLEM 5-1h
Statement: A differential stress element has a set of applied stresses on it as indicated in each row of Table P5-1. For row h, draw the stress element showing the applied stresses, find the principal stresses and the von Mises stress.

Given:

x 750 xy 500

y 500 yz 0

z 250 zx 0

Solution:

See Figure 5-1h and Mathcad file P0501h.


z 250

1. Draw the stress element (see Figure 5-1h). 2. From Problem 4-1h, the principal stresses are

1 1140

2 250

3 110

3. Using equation 5.7, the von Mises stress is

'

1 2

1 2 2 3 1 3
2 2

750 x

500

500

500
y

' 968

FIGURE 5-1h
Stress Element for Problem 5-1h

MACHINE DESIGN - An Integrated Approach, 4th Ed.

5-2-1

PROBLEM 5-2
Statement: Given: A 400-lb chandelier is to be hung from two 10-ft-long solid, low- carbon steel cables in tension. Size the cables for a safety factor of 4. State all assumptions. Weight of chandelier Length of cable Design Safety factor Number of cables Young's modulus W 400 lbf L 10 ft Nd 4 N 2 E 30 10 psi
6

L 120 in

Assumptions: The material is AISI 1010 hot-rolled steel with S y 26 ksi Solution: 1. See Mathcad file P0502. P W N Nd = Sy P 200 lbf

Determine the load on each cable

2. 3.

Using the distortion-energy failure theory,

'

In this case, the only stress in the axial direction is the tensile stress. Therefore, this is the principal stress and also the von Mises stress. 4 P ' = 1 = = 2 d Substitute the equation in step 3 into the design equation in step 2 and solve for the minimum diameter, d.
1

4.

4 P N d d S y
5.

d 0.198 in

Round up to an available size (see Table 13-2) and check the actual factor of safety against static failure.

d 0.207 in

Ns

d S y
4 P

Ns 4.4

MACHINE DESIGN - An Integrated Approach, 4th Ed.

5-3-1

PROBLEM 5-3
Statement: For the bicycle pedal-arm assembly in Figure P5-1 with rider-applied force of 1500 N at the pedal, determine the von Mises stress in the 15-mm-dia pedal arm. The pedal attaches to the arm with a 12-mm thread. Find the von Mises stress in the screw. Find the safety factor against static failure if the material has S y = 350 MPa. Distances (see figure) Rider-applied force Pedal arm diameter Solution: a 170 mm Frider 1.5 kN d pa 15 mm b 60 mm Screw thread diameter Material yield strength
z

Given:

d sc 12 mm S y 350 MPa

See Figures 5-3 and Mathcad file P0503.

1. From problem 4-3, the maximum principal stresses in the pedal arm are at point A and are

a Frider b

1 793 MPa 3 23 MPa

2 0 MPa

C Mc Arm

Tc

2. Using equation 5.7c, the von Mises stress is

Fc Pedal x

'

1 1 3 3

FIGURE 5-3A

' 805 MPa


3. The factor of safety for the pedal arm is N Sy N 0.43

Free Body Diagram for Problem 5-3

'

4. From Problem 4-3 solution, the stresses at the top of the screw where it joins the pedal arm are

A Arm

Section C B

x 530.5 MPa zx 0 MPa

z 0 MPa

5. From this, we see that the principal stresses are

x y
FIGURE 5-3B

1 x 3 0 MPa
6. The von Mises stress is

2 0 MPa
Points A and B at Section C

' 1
N Sy

' 530.5 MPa


N 0.66

7. The factor of safety for the screw is

'

MACHINE DESIGN - An Integrated Approach, 4th Ed.

5-4-1

PROBLEM 5-4
Statement: The trailer hitch shown in Figure P4-2 and Figure 1-1 (p. 12) has loads applied as defined in Problem 3-4. The tongue weight of 100 kg acts downward and the pull force of 4905 N acts horizontally. Using the dimensions of the ball bracket shown in Figure 1-5 (p. 15) and S y = 300 MPa ductile steel, determine static safety factors for: (a) The shank of the ball where it joins the ball bracket. (b) Bearing failure in the ball bracket hole. (c) Tearout failure in the ball bracket. (d) Tensile failure in the 19-mm diameter attachment holes. (e) Bending failure in the ball bracket as a cantilever. a 40 mm b 31 mm c 70 mm d 20 mm Mtongue 100 kg Fpull 4.905 kN d sh 26 mm t 19 mm S y 300 MPa Assumptions: 1. The nuts are just snug-tight (no pre-load), which is the worst case. 2. All reactions will be concentrated loads rather than distributed loads or pressures. Solution: See Figure 5-4 and Mathcad file P0504.
W tongue 70 = c

Given:

F pull

40 = a 2 A B A F b1 B F a1y 20 = d D F a2y Fa2x 2 F b2 C D Fd2 F c2y F a1x

19 = t 31 = b

Fc2x

FIGURE 5-4A
Dimensions and Free Body Diagram for Problem 5-4

1.

From Problem 4-4, the principal stresses in the shank of the ball where it joins the ball bracket are:

1 114 MPa

2 0 MPa

3 0 MPa

2. Since 1 is the only nonzero principal stress, it is also the von Mises stress.The factor of safety against a static failure at the shank of the ball is

MACHINE DESIGN - An Integrated Approach, 4th Ed.

5-4-2
Na 2.6

' 1
3.

Na

Sy

'

From Problem 4-4, the principal stresses at the bearing area in the ball bracket hole are:

1 9.93 MPa

2 0 MPa

3 0 MPa

4. Since 1 is the only nonzero principal stress, it is also the von Mises stress.The factor of safety against a static bearing failure in the ball bracket hole is

' 1

Nb

Sy

'

Nb 30.2

5. From Problem 4-4, the shear stress in the tearout area in the ball bracket is:

Tearout length

4.41 MPa
6. For pure shear, the von Mises stress is ' 3 and the factor of safety against a static tearout failure is Nc Sy Nc 39.3
2

'

7. From Problem 4-4, the principal stresses in the attachment bolts if they are 19-mm diameter are:

d
FIGURE 5-4B

x 53.6 MPa xy 1.7 MPa


8.

y 0 MPa

Tearout Diagram for Problem 5-4

The von Mises stress and the factor of safety against a static failure in the attachment bolts are:

'

x y x y 3 xy

' 53.7 MPa

Nd

Sy

'

Nd 5.6

9.

From Problem 4-4, the principal stresses in the bracket due to bending in the ball bracket as a cantilever are:

1 72.8 MPa

2 0 MPa

3 0 MPa

10. Since 1 is the only nonzero principal stress, it is also the von Mises stress.The factor of safety against a static bearing failure in the ball bracket hole is

' 1

Ne

Sy

'

Ne 4.1

MACHINE DESIGN - An Integrated Approach, 4th Ed.

5-5-1

PROBLEM 5-5
Statement: Repeat Problem 5-4 for the loading conditions of Problem 3-5, i.e., determine the horizontal force that will result on the ball from accelerating a 2000-kg trailer to 60 m/sec in 20 sec. Assume a constant acceleration. From Problem 3-5, the pull force is 6000 N. Determine static safety factors for: (a) The shank of the ball where it joins the ball bracket. (b) Bearing failure in the ball bracket hole. (c) Tearout failure in the ball bracket. (d) Tensile failure in the 19-mm diameter attachment holes. (e) Bending failure in the ball bracket as a cantilever. a 40 mm b 31 mm Mtongue 100 kg Fpull 6 kN S y 300 MPa Assumptions: 1. The nuts are just snug-tight (no pre-load), which is the worst case. 2. All reactions will be concentrated loads rather than distributed loads or pressures. Solution: See Figures 5-5 and Mathcad file P0505.
W tongue 70 = c

Given:

c 70 mm d 20 mm d sh 26 mm t 19 mm

F pull

40 = a 2 A B A F b1 B F a1y 20 = d D F a2y Fa2x 2 F b2 C D Fd2 F c2y F a1x

19 = t 31 = b

Fc2x

FIGURE 5-5A
Dimensions and Free Body Diagram for Problem 5-5

1. From Problem 4-5, the principal stresses in the shank of the ball where it joins the ball bracket are:

1 139 MPa

2 0 MPa

3 0 MPa

2. Since 1 is the only nonzero principal stress, it is also the von Mises stress.The factor of safety against a static failure at the shank of the ball is

MACHINE DESIGN - An Integrated Approach, 4th Ed.

5-5-2
Na 2.2

' 1

Na

Sy

'

3. From Problem 4-5, the principal stresses at the bearing area in the ball bracket hole are:

1 12.15 MPa

2 0 MPa

3 0 MPa

4. Since 1 is the only nonzero principal stress, it is also the von Mises stress.The factor of safety against a static bearing failure in the ball bracket hole is

' 1

Nb

Sy

'

Nb 24.7

5. From Problem 4-5, the shear stress in the tearout area in the ball bracket is:

Tearout length

5.4 MPa
6. For pure shear, the von Mises stress is ' 3 and the factor of safety against a static tearout failure is Nc Sy Nc 32.1
2

'

7. From Problem 4-5, the principal stresses in the attachment bolts if they are 19-mm diameter are:

d
FIGURE 5-5B

x 64.2 MPa xy 1.7 MPa

y 0 MPa

Tearout Diagram for Problem 5-5

8. The von Mises stress and the factor of safety against a static failure in the attachment bolts are:

'

x y x y 3 xy

' 64.3 MPa

Nd

Sy

'

Nd 4.7

9. From Problem 4-5, the principal stresses in the bracket due to bending in the ball bracket as a cantilever are:

1 85.1 MPa

2 0 MPa

3 0 MPa

10. Since 1 is the only nonzero principal stress, it is also the von Mises stress.The factor of safety against a static bearing failure in the ball bracket hole is

' 1

Ne

Sy

'

Ne 3.5

MACHINE DESIGN - An Integrated Approach, 4th Ed.

5-6-1

PROBLEM 5-6
Statement: Repeat Problem 5-4 for the loading conditions of Problem 3-6, i.e., determine the horizontal force that will results from an impact between the ball and the tongue of the 2000-kg trailer if the hitch deflects 2.8 mm dynamically on impact. The tractor weighs 1000 kg and the velocity at impact is m/sec. Determine static safety factors for: (a) The shank of the ball where it joins the ball bracket. (b) Bearing failure in the ball bracket hole. (c) Tearout failure in the ball bracket. (d) Tensile failure in the 19-mm diameter attachment holes. (e) Bending failure in the ball bracket as a cantilever. a 40 mm b 31 mm c 70 mm d 20 mm Mtongue 100 kg Fpull 55.1 kN d sh 26 mm t 19 mm S y 300 MPa Assumptions: 1. The nuts are just snug-tight (no pre-load), which is the worst case. 2. All reactions will be concentrated loads rather than distributed loads or pressures. Solution: See Figures 5-6 and Mathcad file P0506.

Given:

W tongue 70 = c

F pull

40 = a 2 A B A F b1 B F a1y 20 = d D F a2y Fa2x 2 F b2 C D Fd2 F c2y F a1x

19 = t 31 = b

Fc2x

FIGURE 5-6A
Dimensions and Free Body Diagram for Problem 5-6

1. From Problem 4-6, the principal stresses in the shank of the ball where it joins the ball bracket are:

1 1277 MPa

2 0 MPa

3 0 MPa

2. Since 1 is the only nonzero principal stress, it is also the von Mises stress.The factor of safety against a static failure at the shank of the ball is

MACHINE DESIGN - An Integrated Approach, 4th Ed.

5-6-2
Na 0.23

' 1

Na

Sy

'

3. From Problem 4-6, the principal stresses at the bearing area in the ball bracket hole are:

1 111.5 MPa

2 0 MPa

3 0 MPa

4. Since 1 is the only nonzero principal stress, it is also the von Mises stress.The factor of safety against a static bearing failure in the ball bracket hole is

' 1

Nb

Sy

'

Nb 2.7

5. From Problem 4-6, the shear stress in the tearout area in the ball bracket is:

Tearout length

49.6 MPa
6. For pure shear, the von Mises stress is ' 3 and the factor of safety against a static tearout failure is Nc Sy Nc 3.5
2

'

7. From Problem 4-6, the principal stresses in the attachment bolts if they are 19-mm diameter are:

d
FIGURE 5-6B

x 540.5 MPa xy 1.7 MPa

y 0 MPa

Tearout Diagram for Problem 5-6

8. The von Mises stress and the factor of safety against a static failure in the attachment bolts are:

'

x y x y 3 xy

' 540.5 MPa

Nd

Sy

'

Nd 0.56

9. From Problem 4-6, the principal stresses in the bracket due to bending in the ball bracket as a cantilever are:

1 635.5 MPa

2 0 MPa

3 0 MPa

10. Since 1 is the only nonzero principal stress, it is also the von Mises stress.The factor of safety against a static bearing failure in the ball bracket hole is

' 1

Ne

Sy

'

Ne 0.47

MACHINE DESIGN - An Integrated Approach, 4th Ed.

5-7-1

PROBLEM 5-7
Statement: Design the wrist pin of Problem 3-7 for a safety factor of 3 and S y = 100 ksi if the pin is hollow and loaded in double shear. Force on wrist pin Yield strength Design safety factor Fwristpin 12.258 kN S y 100 ksi Nd 3 od 0.375 in Fwristpin 2756 lbf

Given:

Assumptions: Choose a suitable outside diameter, say Solution:

See Figure 4-12 in the text and Mathcad file P0507. F Fwristpin 2 F 1378 lbf

1. The force at each shear plane is

2. With only the direct shear acting on the plane, the Mohr diagram will be a circle with center at the origin and radius equal to the shear stress. Thus, the principal normal stress is numerically equal to the shear stress, which in this case is also the principal shear stress, so we have = 1 = '. F A 4 F

3. The shear stress at each shear plane is

2 2 od id

= '

4. Using the distortion-energy failure theory,

Nd =

Sy

'

od id S y
4 F

5. Solving for the inside diameter,

id

od

4 F Nd

S y

id 0.297 in

6. Round this down to the decimal equivalent of a common fraction (9/32),

id 0.281 in

7. The realized factor of safety is,

od id S y
4 F

N 3.5

MACHINE DESIGN - An Integrated Approach, 4th Ed.

5-8-1

PROBLEM 5-8
Statement: A paper mill processes rolls of paper having a density of 984 kg/m3. The paper roll is 1.50-m outside diameter (OD) x 0.22-m inside diameter (ID) x 3.23-m long and is on a simple supported, hollow, steel shaft with S y = 300 MPa. Find the shaft ID needed to obtain a static safety factor of 5 if the shaft OD is 22 cm. Paper roll: Density Outside dia. Inside dia. Length Shaft: Strength Outside dia. Factor of safety S y 300 MPa od 220 mm Ns 5
V R L/2 0 L x -R M

Given:

984

kg
3

y w x R L R

m OD 1500 mm ID 220 mm L 3230 mm

Assumptions: 1. The shaft is stiffer than the paper roll so the weight of the roll on the shaft can be modelled as a uniformly distributed load. 2. The bearings that support the shaft are close to the ends of the paper roll and are thin with respect to the length of the roll so we can consider the distance between the shaft supports to be the same as the length of the roll. Solution: See Figure 5-8 and Mathcad file P0508.

wL /8

0 L/2 L

FIGURE 5-8
Load, Shear, and Moment Diagrams for Problem 5-8

1. The weight of the paper roll is, Volume Weight V

OD ID L

V 5.585 m

(1) (2)

W g V

W 53.895 kN

2. From Figure 5-8, we see that the bending moment in the shaft is a maximum at the center of the span. First, determine the magnitude of the distributed load, then find the maximum bending moment using Figure D-2(b) in Appendix B with a = 0 and x = L/2. Distributed load Maximum moment w W L w L 8
2

w 16.686

newton mm
7

(3) (4)

Mmax

Mmax 2.176 10 newton mm

3. Using equation 4.11b, find the maximum bending stress as a function of the unkown shaft inside diameter, id. Bending stress at midspan M c I 32 Mmax od

max =

od id

(5)

4. This is the only stress element present at this point on the shaft and there is no shear stress at this point so max = 1 and 2 = 3 = 0. Furthermore, since 2 and 3 are zero, max = '. Equation 5.8a can be used to find the unknown id,

MACHINE DESIGN - An Integrated Approach, 4th Ed.


Sy

5-8-2

Factor of safety

Ns =

'
1

(6)

Substituting equation 5 into 6 and solving for id, we have

Shaft id

Sy od4 32 Ns Mmax od id S y

id 198 mm

(7)

MACHINE DESIGN - An Integrated Approach, 4th Ed.

5-9-1

PROBLEM 5-9
Statement: A ViseGrip plier-wrench is drawn to scale in Figure P5-3, and for which the forces were analyzed in Problem 3-9 and the stresses in Problem 4-9, find the safety factors for each pin for an assumed clamping force of P = 4000 N in the position shown. The pins are 8-mm dia, S y = 400 MPa, and are all in double shear. Pin stresses as calculated in Problem 4-9: Pin 1-2 12 74.6 MPa Pin 1-4 Pin 2-3 Pin 3-4 Yield strength

Given:

14 50.7 MPa 23 50.7 MPa 34 50.7 MPa


S y 400 MPa

Assumptions: Links 3 and 4 are in a toggle position, i.e., the pin that joins links 3 and 4 is in line with the pins th join 1 with 4 and 2 with 3. Solution: 1. See Figure 5-9 and Mathcad file P0509.

The FBDs of the assembly and each individual link are shown in Figure 5-9. The dimensions, as scaled from Figure P5-3 in the text, are shown on the link FBDs.
F 4 P 1

3 F
55.0 = b 50.0 = a 22.0 = d

2 P

F14

39.5 = c

129.2

4 F34 P

F41

F21

28.0 = e

2.8 = g

F43 3 F23 F

F12 F32 2

21.2 = h

26.9 = f

FIGURE 5-9
Free Body Diagrams for Problem 5-9

2.

The pins are in pure shear, so the principal stresses are

MACHINE DESIGN - An Integrated Approach, 4th Ed.


Pin joining 1 and 2 All other pins

5-9-2

'12 '14

3 12 3 14

'12 129.211 MPa '14 87.815 MPa

3.

Using the distortion-energy failure theory, the factors of safety are Pin joining 1 and 2 N12 Sy N12 3.1

'12
Sy

All other pins

N14

'14

N14 4.6

MACHINE DESIGN - An Integrated Approach, 4th Ed.

5-10-1

PROBLEM 5-10
Statement: An over-hung diving board is shown in Figure P5-4a. Assume cross-section dimensions of 305 mm x 32 mm. Find the largest principal stress in the board when a 100-kg person is standing at the free end. What is the static safety factor if the material is brittle fiberglass with S ut = 130 MPa in the longitudinal direction? Maximum principal stresses due to bending at R2 from Problem 4-10
2000 = L R1 P

Given:

1 24.5 MPa 2 0 MPa 3 0 MPa


Ultimate strength S ut 130 MPa
700 = a R2

FIGURE 5-10
Free Body Diagram for Problem 5-10

Solution: 1.

See Figure 5-10 and Mathcad file P0510.

The diving board will be in tension at the top of the board and compression along the bottom. At the top, over the right-hand support, the nonzero principal stress is positive and the load line on the 1-3 diagram is along the 1 axis. Using the Modified-Mohr failure theory, the static safety factor is Ns S ut

Ns 5.3

MACHINE DESIGN - An Integrated Approach, 4th Ed.

5-11-1

PROBLEM 5-11
Statement: Repeat Problem 5-10 assuming the 100-kg person in Problem 5-10 jumps up 25 cm and lands back on the board. Assume the board weighs 29 kg and deflects 13.1 cm statically when the person stands on it. What is the static safety factor if the material is brittle fiberglass with S ut = 130 MPa i the longitudinal direction? Maximum principal stresses due to bending at R2 from Problem 4-11
2000 = L R1 P

Given:

1 76.3 MPa 2 0 MPa 3 0 MPa


Ultimate strength S ut 130 MPa FIGURE 5-11
Free Body Diagram for Problem 5-11
700 = a R2

Solution: 1.

See Figure 5-11 and Mathcad file P0511.

The diving board will be in tension at the top of the board and compression along the bottom. At the top, ove the right-hand support, the nonzero principal stress is positive and the load line on the 1-3 diagram is along the 1 axis. Using the Modified-Mohr failure theory, the static safety factor is Ns S ut

Ns 1.7

MACHINE DESIGN - An Integrated Approach, 4th Ed.

5-12-1

PROBLEM 5-12
Statement: Given: Repeat Problem 5-10 using the cantilevered diving board design in Figure P5-4b. Maximum principal stresses due to bending at support from Problem 4-12
2000 1300 = L P

1 24.5 MPa 2 0 MPa 3 0 MPa


Ultimate strength Solution: S ut 130 MPa
M1 700 R1

See Figure 5-12 and Mathcad file P0512.

FIGURE 5-12
Free Body Diagram for Problem 5-12

1.

The diving board will be in tension at the top of the board and compression along the bottom. At the top, at the built-in support, the nonzero principal stress is positive and the load line on the 1-3 diagram is along the 1 axis. Using the Modified-Mohr failure theory, the static safety factor is Ns S ut

Ns 5.3

MACHINE DESIGN - An Integrated Approach, 4th Ed.

5-13-1

PROBLEM 5-13
Statement: Repeat Problem 5-11 using the cantilevered diving board design in Figure P5-4b. Assume the board weighs 19 kg and deflects 8.5 cm statically when the person stands on it. Maximum principal stresses due to bending at support from Problem 4-13
2000 1300 = L P

Given:

1 87.1 MPa 2 0 MPa 3 0 MPa


Ultimate strength Solution: S ut 130 MPa
M1 700 R1

See Figure 5-13 and Mathcad file P0513.

FIGURE 5-13
Free Body Diagram for Problem 5-13

1.

The diving board will be in tension at the top of the board and compression along the bottom. At the top, at the built-in support, the nonzero principal stress is positive and the load line on the 1-3 diagram is along the 1 axis. Using the Modified-Mohr failure theory, the static safety factor is Ns S ut Ns 1.5

MACHINE DESIGN - An Integrated Approach, 4th Ed.

5-14-1

PROBLEM 5-14
Statement: Figure P4-5 shows a child's toy called a pogo stick. The child stands on the pads, applying half he weight on each side. She jumps off the ground, holding the pads up against her feet, and bounces along with the spring cushioning the impact and storing energy to help each rebound. Design the aluminum cantilever beam sections on which she stands to survive jumping 2 in off the ground with a safety factor of 2. Use 1100 series aluminum. Define and size the beam shape. Cold rolled 1100 aluminum: Yield strength Safety factor S y 22 ksi Ns 2

Given:

Assumptions: The beam will have a rectangular cross-section with the load applied at a distance of 5 in from the central support. L 5 in Solution: See Figure 5-14 and Mathcad file P0514.

1. From Problem 3-14, the total dynamic force on both foot supports is Fi 224 lbf Therefore, the load on each support is P Fi 2 P 112 lbf

Fi /2

Fi /2

2. To give adequate support to the childs foot, let the width of the support beam be w 1.5 in 3. From Figure B-1(a) in Appendix B, the maximum bending moment at x = 0 is M P L M 560 in lbf FIGURE 5-14
Free Body Diagram for Problem 5-14

4. We can now calculate the minimum required section modulus, Z = I/c. Using the distortion-energy failure theor the bending stress will also be the only nonzero principal stress, which will also be the von Mises stress. Design equation Ns = Sy

'
M Z = ' = Sy Ns Z 834.3 mm
3

Bending stress

=
Z

Solving for Z,

N s M Sy w t 6 Z w
3

5. For a rectangular cross-section,

I= t

and

12

c=

t 2

so

Z=

w t 6

Solving for t,

t 0.451 in t 0.500 in

Round this up to the next higher decimal equivalent of a common fraction,

MACHINE DESIGN - An Integrated Approach, 4th Ed.

5-15-1

PROBLEM 5-15
Statement: Solution: Any part whose stress equals its strength has a safety factor of 1 by definition. What is the safety factor for the shear pin as defined in Problem 4-15?

MACHINE DESIGN - An Integrated Approach, 4th Ed.

5-16-1

PROBLEM 5-16
Statement: A track to guide bowling balls is designed with two round rods as shown in Figure P5-6. The rods are not parallel to one another but have a small angle between them. The balls roll on the rods unt they fall between them and drop onto another track. The angle between the rods is varied to cause the ball to drop at different locations. Find the static safety factor for the 1-in dia SAE 1045 normalized steel rods. (a) Assume rods are simply supported at each end. (b) Assume rods are fixed at each end. Yield strength S y 58 ksi
Fball

Given: Solution:

See Figure 5-16 and Mathcad file P0516.


R1 L R2

1. The maximum bending stress will occur at the outer fibers of the rod at the section where the maximum bending moment occurs which, in this case, is at x = a. The only stress present on the top or bottom surface of the rod is the bending stress x. Therefore, on the bottom surface where the stress is tensile, x is the principal stress 1 . Thus, from Problem 4-16, for a simply supported rod, Maximum principal stress

FIGURE 5-16A
Free Body Diagram for Problem 5-16(a), taken on a plane through the rod axis and ball center

1 748 psi

'a 1

2. Using the distortion-energy failure theory, the safety factor against a static failure is Nsa Sy Nsa 78
a Fball

'a

3. For the built-in case, the maximum bending stress will occur at the outer fibers of the rod at the section where the maximum bending moment occurs which, in this case, is at x = L. The only stress present on the top or bottom surface of the rod is the bending stress x. Therefore, on the bottom surface where the stress is tensile, x is the principal stress 1 . Thus, from Problem 4-16, for a simply supported rod, Maximum principal stress

M1

R1

R 2 M2

FIGURE 5-16B
Free Body Diagram for Problem 5-16(b), taken on a plane through the rod axis and ball center

1 577 psi

'b 1

4. Using the distortion-energy failure theory, the safety factor against a static failure is Nsb Sy Nsb 101

'b

MACHINE DESIGN - An Integrated Approach, 4th Ed.

5-17-1

PROBLEM 5-17
Statement: A pair of ice tongs is shown in Figure P5-7. The ice weighs 50 lb and is 10 in wide across the tongs. The distance between the handles is 4 in, and the mean radius r of the tong is 6 in. The rectangular cross-sectional dimensions are 0.75 x 0.312 in. Find the safety factor for the tongs if their S y = 30 ksi. F Yield strength S y 30 ksi

Given:

C FC O
11.0 = ax 3.5 = cy

See Problem 4-17, Figure 5-17, and Solution: Mathcad file P0517. 1. The maximum bending stress in the tong was found in Problem 4-17 at point A. Vertical direction

FO
2.0 = cx 12.0 = by 5.0 = bx

i 8.58 ksi
FB B

All other components are zero 2. There are no other stress components present so

1 i
and

2 0 ksi ' 1

3 0 ksi ' 8.58 ksi

W/2
FIGURE 5-17
Free Body Diagram for Problem 5-17

3. The factor of safety is (using the distortion energy theory)

Sy

'

N 3.5

MACHINE DESIGN - An Integrated Approach, 4th Ed.

5-18-1

PROBLEM 5-18
Statement: A pair of ice tongs is shown in Figure P5-7. The ice weighs 50 lb and is 10 in wide across the tongs. The distance between the handles is 4 in, and the mean radius r of the tong is 6 in. The rectangular cross-sectional dimensions are 0.75 x 0.312 in. Find the safety factor for the tongs if they are made from Class 20 gray cast iron.

F
Given: Tensile strength Compressive strength S ut 22 ksi S uc 83 ksi

C FC O
3.5 = cy

See Problem 4-18, Figure 5-18, and Solution: Mathcad file P0518. 1. The maximum bending stress in the tong was found in Problem 4-17 at point A. Vertical direction

FO
2.0 = cx 12.0 = by 5.0 = bx

11.0 = ax

i 8.58 ksi
B W/2

FB

All other components are zero 2. Therefore, the principal stresses are

1 i

2 0 ksi

3 0 ksi

FIGURE 5-18
Free Body Diagram for Problem 5-18

3. The load line on the 1-3 diagram is along the 1 axis. Using the Modified-Mohr failure theory, the static safety factor is S ut N N 2.6

MACHINE DESIGN - An Integrated Approach, 4th Ed.

5-19-1

PROBLEM 5-19
Statement: Determine the size of the clevis pin, shown in Figure P5-8, needed to withstand an applied force of 130 000 lb. Also determine the required outside radius of the clevis end to not fail in either tear out or bearing if the clevis flanges are each 2.5 in thick. Use a safety factor of 3 for all modes of failure. Assume S y = 89.3 ksi for the pin and S y = 35.5 ksi for the clevis. Applied force Clevis strength Safety factor Solution: 1. P 130 kip S yclevis 35.5 ksi Ns 3 Clevis flange thickness t 2.50 in Pin strength S ypin 89.3 ksi

Given:

See Figures P5-8 in the text and Mathcad file P0519.

Determine the force carried by each of the two flanges of the clevis. F 0.5 P F 65 kip

This force is transmitted through each end of the clevis pin, which is in double shear. 2. The pin is in direct (pure) shear. Therefore, the von Mises stress is

'pin = 3 pin =

4 3 F

d
3. Calculate the minimum required clevis pin diameter using the distortion-energy failure theory. Ns = S ypin =

d S ypin
4 3 F 4 3 F Ns

'pin

Solving for the pin diameter

S ypin

d 2.194 in

Round this up to the next higher decimal equivalent of a common fraction ( 2 1/4) 4. Check the bearing stress in the clevis due to the pin on one side of the clevis. Bearing stress area Bearing force Bearing stress Ab d t Fb F Ab 5.625 in Fb 65 kip
2

d 2.250 in

Fb Ab

b 11.6 ksi
Tearout length

5. Determine the safety factor against a static bearing failure. Nbear S yclevis Nbear 3.1

Since this is greater than 3, the pin diameter is acceptable. 6. Determine the tearout stress in the clevis. Shear area (see Figure 5-19) Shear force Ftear F Ftear 65 kip Atear = 2 t R ( 0.5 d )
2 2
d R

FIGURE 5-19
Tearout Diagram for Problem 5-19

MACHINE DESIGN - An Integrated Approach, 4th Ed.

5-19-2

Shear stress and distortion-energy equation

tear =

Ftear Atear

Ftear 2 t R ( 0.5 d )
2 2

Ns =

S yclevis

'tear

S yclevis 3 tear

2 t S yclevis R ( 0.5 d ) 3 Ftear

Solving for the clevis radius, R

3 Ftear Ns 2 R ( 0.5 d ) 2 t S yclevis

R 2.211 in

Round this up to the next higher decimal equivalent of a common fraction ( 2 1/4)
2 2

R 2.250 in
2

The tearout area for each flange is

Atear 2 t R ( 0.5 d )

Atear 9.743 in

7. Design summary: Pin diameter d 2.250 in Clevis flange radius R 2.25 in

MACHINE DESIGN - An Integrated Approach, 4th Ed.

5-20-1

PROBLEM 5-20
Statement: Given: A 100 N-m torque is applied to a 1-m-long, solid, round shaft. Design it to limit its angular deflection to 2 deg and select a steel alloy to have a yielding safety factor of 2. Applied torque Maximum deflection Safety factor T 100 N m max 2 deg Ns 2 Shaft length Modulus of rigidity L 1000 mm G 79 GPa

Assumptions: A ductile steel will be chosen. Solution: 1. See Mathcad file P0520.

Using the angular deflection requirement and equation (4.24), determine the required polar moment of inertia an the minimum diameter.

TL J G

TL

max G
1 4

J 3.626 10 mm

J =

d
32

32 J d

d 24.653 mm

Round this up to 2.

d 25 mm

Determine the shear stress at the outside diameter of the shaft using equation (4.23b). T

max
3.

2
J

max 34.47 MPa

For this case of pure shear, use the distortion-energy theory and equations (5.8) and (5.9) to solve for the minimum required yield strength. Sy Sy 3 max

Ns =

'

S y

3 max Ns

S y 119.4 MPa

4.

Using this value of S y, choose a steel from Table A-9 in Appendix A. Any of the steels listed in Table A-9 will be adequate. The least expensive is AISI 1010, hot rolled.

MACHINE DESIGN - An Integrated Approach, 4th Ed.

5-21-1

PROBLEM 5-21
Statement: Figure P5-9 shows an automobile wheel with two common styles of lug wrench being used to tighten the wheel nuts, a single-ended wrench in (a), and a double-ended wrench in (b). The distance between points A and B is 1 ft in both cases and the handle diameter is 0.625 in. What is the maximum force possible before yielding the handle if the material S y = 45 ksi? Distance between A and B Wrench diameter Yield strength d AB 1 ft d 0.625 in S y 45 ksi

Given:

Assumptions: 1. The forces exerted by the user's hands lie in a plane through the wrench that is also parallel to the plane of the wheel. 2. The applied torque is perpendicular to the plane of the forces. 3. By virtue of 1 and 2 above, this is a planar problem that can be described in a 2D FBD. Solution: See Figure 5-21 and Mathcad file P0521.
12" = dAB F

1. From examination of the FBDs, we see that, in both cases, the arms are in bending and the stub that holds the socket wrench is in pure torsion. The maximum bending stress in the arm will occur near the point where the arm transitions to the stub. The stress state at this transition is very complicated, but we can find the nominal bending stress there by treating the arm as a cantilever beam, fixed at the transition point. For both cases the torque in the stub is the same.

T F (a) Single-ended Wrench

Case (a)
6"

12" = dAB F

2. The bending moment at the transition is Ma = Fa d AB


T

3. The tensile stress at this point is found from


F

Moment of inertia I

(b) Double-ended Wrench

d
64

I 0.00749 in

FIGURE 5-21
Free Body Diagrams for Problem 5-21

Dist to extreme fibre Stress

c 0.5 d

c 0.313 in

x =

Ma c I

4. There are no other stress components present at this point, so x is the maximum principle stress here and

1 = x

2 0 psi

3 0 psi ' = 1 = x =
Ma c I = Fa d AB c I

5. Since there is only one nonzero principal stress, the von Mises stress is

6. Using the distortion-energy theory, solve for the maximum applied force. Ns = Sy

'

I Sy Fa d AB c

=1

Fa

I Sy d AB c

Fa 89.882 lbf

MACHINE DESIGN - An Integrated Approach, 4th Ed.


Fa d AB c I T 1079 in lbf

5-21-2

7. The von Mises stress in the handle at the transition point is

'

' 45 ksi

8. Determine the torque in the stub.

T Fa d AB

9. The shear stress at any point on the outside surface of the stub is found from Polar moment of inertia Shear stress J 2 I J 0.0150 in J
4

xy

Tc

xy 22.5 ksi

10. There are no other stress components present along the outside surface of the stub, so

1 xy
and

1 22.5 ksi
2 2

2 0 psi ' 39.0 ksi

3 1

'

1 1 3 3

11. Thus, the maximum von Mises stress for case (a) is on the upper surface of the handle (arm) near the point where it transitions to the stub, and the maximum force that can be applied to the handle without yielding is Fa 89.9 lbf Case (b) 12. The bending moment at the transition is Mb = Fb d AB 2 Mb c I

11. The tensile stress at this point is found from

x =

12. There are no other stress components present at this point, so x is the maximum principle stress here and

1 = x

2 0 psi

3 0 psi
Mb c I = Fb d AB c 2 I

13. Since there is only one nonzero principal stress, the von Mises stress is ' = 1 = x = 14. Using the distortion-energy theory, solve for the maximum applied force. Ns = Sy = 2 I Sy Fb d AB c =1 Fb 2 I Sy d AB c Fb 179.763 lbf

'

15. The von Mises stress in the handle at the transition point is 16. The torque in the stub is T Fb d AB

'

Fb d AB c 2 I T 2157 in lbf

' 45 ksi

14. The shear stress at any point on the outside surface of the stub is found from Shear stress

xy

Tc J

xy 45 ksi

15. There are no other stress components present along the outside surface of the stub, so

1 xy

1 45.0 ksi

2 0 psi

3 1

MACHINE DESIGN - An Integrated Approach, 4th Ed.

5-21-3

and

'

1 1 3 3

' 77.9 ksi

16. Since the von Mises stress in the stub due to torsion is greater than the yield strength, the force in the handle will be limited by the shear stress in the stib and by the bending stress in the handle. Ns = Sy = Sy 3 xy = J Sy 3 T c = J Sy 3 Fb d AB c =1

'

Fb

J Sy 3 d AB c

Fb 103.8 lbf

17. Thus, the maximum von Mises stress for case (b) is on the stub, and the maximum force that can be applied to the handles without yielding is Fb 103.8 lbf

MACHINE DESIGN - An Integrated Approach, 4th Ed.

5-22-1

PROBLEM 5-22
Statement: A roller-blade skate is shown in Figure P5-10. The polyurethane wheels are 72 mm dia and spaced on 104-mm centers. The skate-boot-foot combination weighs 2 kg. The effective "spring rate" of the person-skate subsystem is 6000 N/m. The axles are 10-mm-dia steel pins in double shear with S y = 400 MPa. Find the safety factor for the pins when a 100-kg person lands a 0.5-m jump on one foot. (a) Assume all 4 wheels land simultaneously. (b) Assume that one wheel absorbs all the landing force. Axle pin diameter d 10 mm Yield strength S y 400 MPa

Given: Solution: 1. 2.

See Figure P5-10 and Mathcad file P0522.

From Problem 4-22, we have the stresses for cases (a) and (b): Using the distortion-energy failure theory, Case (a) all wheels landing Nsa Sy 3 a Sy 3 b

a 5.71 MPa

b 22.9 MPa

Nsa 40.4

Case (b) one wheel landing

Nsb

Nsb 10.1

MACHINE DESIGN - An Integrated Approach, 4th Ed.

5-23a-1

PROBLEM 5-23a
Statement: A beam is supported and loaded as shown in Figure P5-11a. For the data given in row a from Table P5-2, find the static safety factor: (a) If the beam is a ductile material with S y = 300 MPa, (b) If the beam is a cast-brittle material with S ut = 150 MPa, S uc = 570 MPa. Ductile yield strength S y 300 MPa
a w L b F

Given:

Brittle ultimate tensile strength S ut 150 MPa

Solution:

See Figure 5-23 and Mathcad file P0523a.

R1

R2

FIGURE 5-23
Free Body Diagram for Problem 5-23

1.

The maximum bending stress occurs under the concentrated load F at x = b. It was determined in Problem 4-23a as

x 88.7 MPa
2. Since this is the only stress component present in the given coordinate frame, x is equal to 1 and the other two principal stresses are zero.

1 x
3.

2 0 MPa

3 0 MPa

For case (a), use the distortion-energy failure theory. With only one nonzero principal stress, the von Mises stress is the same as 1. von Mises stress Safety factor, case (a)

' 1
Nsa Sy

' 88.7 MPa


Nsa 3.4

'

4.

For case (b), use the Modified Mohr failure theory. The nonzero principal stress is positive and the load line on the s1-s3 diagram is along the 1 axis. Safety factor, case (b) Nsb S ut Nsb 1.7

MACHINE DESIGN - An Integrated Approach, 4th Ed.

5-24a-1

PROBLEM 5-24a
Statement: A beam is supported and loaded as shown in Figure P5-11b. For the data given in row a from Tabl P5-2, find the static safety factor: (a) If the beam is a ductile material with S y = 300 MPa, (b) If the beam is a cast-brittle material with S ut = 150 MPa, S uc = 570 MPa.
L

Given:

Ductile yield strength Brittle ultimate strength S ut 150 MPa

S y 300 MPa

a F w

M1

Solution:

See Figure 5-24 and Mathcad file P0524a.

R1

FIGURE 5-24
Free Body Diagram for Problem 5-24

1.

The maximum bending stress occurs at the support where x = 0. It was determined in Problem 4-24a as

x 410 MPa
2. Since this is the only stress component present in the given coordinate frame, x is equal to 1 and the other two principal stresses are zero.

1 x
3.

2 0 MPa

3 0 MPa

For case (a), use the distortion-energy failure theory. With only one nonzero principal stress, the von Mises stress is the same as 1. von Mises stress Safety factor, case (a)

' 1
Nsa Sy

' 410 MPa


Nsa 0.73

'

4.

For case (b), use the Modified Mohr failure theory. The nonzero principal stress is positive and the load line on the 1-3 diagram is along the 1 axis. Safety factor, case (b) Nsb S ut Nsb 0.37

MACHINE DESIGN - An Integrated Approach, 4th Ed.

5-25a-1

PROBLEM 5-25a
Statement: A beam is supported and loaded as shown in Figure P5-11c. For the data given in row a from Table P5-2, find the static safety factor: (a) If the beam is a ductile material with S y = 300 MPa, (b) If the beam is a cast-brittle material with S ut = 150 MPa, S uc = 570 MPa.
L

Given:

Ductile yield strength Brittle ultimate strength S ut 150 MPa

S y 300 MPa
a

b F w

Solution:

See Figure 5-25 and Mathcad file P0525a.

R1

R2

FIGURE 5-25
Free Body Diagram for Problem 5-25

1.

The maximum bending stress occurs at the right-hand support where x = b. It was determined in Problem 4-25a as x 151.6 MPa Since this is the only stress component present in the given coordinate frame, x is equal to 1 and the other two principal stresses are zero.

2.

1 x
3.

2 0 MPa

3 0 MPa

For case (a), use the distortion-energy failure theory. With only one nonzero principal stress, the von Mises stress is the same as 1. von Mises stress Safety factor, case (a)

' 1
Nsa Sy

' 151.6 MPa


Nsa 2.0

'

4.

For case (b), use the Modified Mohr failure theory. The nonzero principal stress is positive and the load line on the 1-3 diagram is along the 1 axis. Safety factor, case (b) Nsb S ut Nsb 0.99

MACHINE DESIGN - An Integrated Approach, 4th Ed.

5-26a-1

PROBLEM 5-26a
Statement: A beam is supported and loaded as shown in Figure P5-11d. For the data given in row a from Tabl P5-2, find the static safety factor: (a) If the beam is a ductile material with S y = 300 MPa, (b) If the beam is a cast-brittle material with S ut = 150 MPa, S uc = 570 MPa.
L

Given:

Ductile yield strength Brittle ultimate strength S ut 150 MPa

S y 300 MPa
a

b F w

Solution: 1.

See Figure 5-26 and Mathcad file P0526a.

R1

R2

R3

The maximum bending stress occurs under the concentrated load F, where x = a. It was determined in Problem 4-26a as

FIGURE 5-26
Free Body Diagram for Problem 5-26

x 31.5 MPa
2. Since this is the only stress component present in the given coordinate frame, x is equal to 1 and the other two principal stresses are zero.

1 x
3.

2 0 MPa

3 0 MPa

For case (a), use the distortion-energy failure theory. With only one nonzero principal stress, the von Mises stress is the same as 1. von Mises stress Safety factor, case (a)

' 1
Nsa Sy

' 31.5 MPa


Nsa 9.5

'

4.

For case (b), use the Modified Mohr failure theory. The nonzero principal stress is positive and the load line on the 1-3 diagram is along the 1 axis. Safety factor, case (b) Nsb S ut Nsb 4.8

MACHINE DESIGN - An Integrated Approach, 4th Ed.

5-27-1

PROBLEM 5-27
Statement: A storage rack is to be designed to hold the paper roll of Problem 5-8 as shown in Figure P5-12. Determine suitable values for dimensions a and b in the figure. Make the static factor of safety at least 1.5. The mandrel is solid and inserts halfway into the paper roll. (a) The beam is a ductile material with Sy = 300 MPa (b) The beam is a cast-brittle material with Sut = 150 MPa, S uc = 570 MPa.
3

Given:

Paper roll dimensions

OD 1.50 m ID 0.22 m Lroll 3.23 m S y 300 MPa

Roll density Factor of safety

984 kg m

Ns 1.5 S ut 150 MPa

Ductile yield strength

Brittle ultimate strength

Assumptions: The paper roll's weight creates a concentrated load acting at the tip of the y mandrel. The mandrel's root in the w stanchion experiences a distributed load a over the length of engagement (see the solution to Problem 3-27 for further discussion of this point). The required diameter a of the mandrel root section b (over the length b) will be sized to use the allowable tensile strength in bending. R The length b will be sized to use the FIGURE 5-27 allowable transverse shear strength.

Lm

Free Body Diagram used in Problem 5-27

Solution: 1.

See Figure 5-27 and Mathcad file P0527.

Determine the weight of the roll and the length of the mandrel. Weight Length W

OD ID Lroll g

W 53.9 kN Lm 1.615 m

Lm 0.5 Lroll

2.

The maximum internal shear and moment occur at a section where the mandrel root leaves the stanchion. and are Vmax = 2 W Lm b Mmax W Lm Mmax 87.04 kN m

3.

Part (a) - The bending stress will be a maximum at the top or bottom of the mandrel at a section where the mandrel root leaves the stanchion.

max =
4.

Mmax a 2 I

where

I=

a
64

so,

max =

32 Mmax

At this point the only nonzero stress component is max therefore

1 = max
5.

2 0 MPa

3 0 MPa

All three of the ductile failure theories have the same fail/safe boundary for this condition (slope of load line is zero) Ns = Sy or Ns 1 = S y

MACHINE DESIGN - An Integrated Approach, 4th Ed.


1

5-27-2

6.

Solving for a,

32 Ns W Lm a S y
a 166 mm

a 164.272 mm

Round this to 7.

Using this value of a and equation (4.15c), solve for the shear stress on the neutral axis at the same section.

max =

4 Vmax 3 A

8 W Lm 3

a 2 b 4
2 0 MPa 3 = max

8.

At this point, this is the only nonzero stress component therefore, the principal stresses are

1 = max
9

Using the distortion energy theory, the von Mises stress is

' = 3 max
Solving for b b

and

Ns =

Sy

'

or

Ns max = b 92.9 mm

Sy 3

8 Ns W Lm

a 2 S y 3 4 3
a 166 mm

Rounding to higher even values, let

b 94 mm

for case (a).

10. Part (b) - The bending stress will be a maximum at the top or bottom of the mandrel at a section where the mandrel root leaves the stanchion.

max =

Mmax a 2 I

where

I=

a
64

so,

max =

32 Mmax

11. At this point the only nonzero stress component is max therefore

1 = max

2 0 MPa

3 0 MPa

12. All three of the brittle failure theories have the same fail/safe boundary for this condition (slope of load line is zero) Ns = S ut = S ut = 2 I S ut Mmax a =

a S ut
32 Mmax
1

max

13. Solving for a, Round this to

32 Ns Mmax a Sut
a 208 mm

a 206.97 mm

14. Using this value of a and equation (4.15c), solve for the shear stress on the neutral axis at the same section.

MACHINE DESIGN - An Integrated Approach, 4th Ed.


4 Vmax 3 A 8 W Lm 3

5-27-3

max =

a 2 b 4
2 0 MPa 3 = max

15. At this point, this is the only nonzero stress component therefore, the principal stresses are

1 = max
16. Using the Modified Mohr theory, 3 =

Ns =

S ut

S ut

a 2 b S ut 4
8 W Lm 8 Ns W Lm

max

Solving for b

a 2 Sut 3 4
a 208 mm

b 68.3 mm

Rounding to higher even values, let

b 70 mm

for case (b).

MACHINE DESIGN - An Integrated Approach, 4th Ed.

5-28-1

PROBLEM 5-28
Statement: Figure P5-13 shows a forklift truck negotiating a 15 deg ramp to to drive onto a 4-ft-high loading platform. The truck weighs 5 000 lb and has a 42-in wheelbase. Design two (one for each side) 1-ft-wide ramps of steel to have a safety factor of 3 in the worst case of loading as the truck travels up them. Minimize the weight of the ramps by using a sensible cross-sectional geometry. Choose an appropriate steel or aluminum alloy. Ramp angle 15 deg Platform height h 4 ft Truck wheelbase Lt 42 in Ramp width Truck weight w 12 in W 5000 lbf

Given:

Assumptions: 1. The worst case is when the truck CG is located at the center of the beam's span. 2. Use a coordinate frame that has the x-axis along the long axis of the beam. 3. Ignore traction forces and the weight components along the x-axis of the beam. 4. There are two ramps, one for each side of the forklift. See Figure 5-28 and Mathcad file P0528. Solution:

L b a CG a
CG b

R1 Fa Wa Fb Wb R2 x

FIGURE 5-28A
Dimensions and Free Body Diagram for Problem 5-28

1. From Problem 3-28 the maximum bending moment in the ramp occurs at the rear wheel of the truck and is Mmax 8324 ft lbf Mmax 99888 in lbf

2. The bending stress is the only stress component present and is, therefore, also the only nonzero principal stress and is also the von Mises stress. The governing design equations then are

' =

Mmax Z

and

Ns =

Sy

'

3. The approach will be to 1) choose a suitable factor of safety, 2) choose a suitable material and determine its yiel strength, 3) from the equations above determine the required value of the section modulus, 4) choose an appropriate cross-section for the ramp, and 5) determine the dimensions of the cross-section. 4. The following design choices have been made for this problem:

MACHINE DESIGN - An Integrated Approach, 4th Ed.


Design factor of safety Material Yield strength Nsd 3 7075 Aluminum, heat treated S y 73 ksi

5-28-2

5. Solve the design equations for the minimum section modulus, Z. Z Nsd Mmax Sy Z 4.105 in
3

This is the minimum allowable value of the section modulus. 6. Assume a channel section such as that shown in Figure 5-28B. To keep it simple, let the thickness of the flanges and web be the same. Choose 1/2-in thick plate, which is readily available. Then, t 0.50 in 7. The cross-sectional area of the ramp is 8. The distance to the CG is cg( h ) A ( h ) w t 2 t ( h t) 1 A (h)

w t 2 2

t h t

Flange Web

9. The moments of inertia of the web and a flange are Iweb( h ) w t


3

12

w t cg( h )

2 t
2

Ifl ( h )

t ( h t) 12

h h t cg( h ) 2

I ( h ) Iweb( h ) 2 Ifl ( h ) 11. The maximum stress will occur in the flange at the top and is compressive. The distance from the centroid up to the top of the flange is c( h ) h cg( h ) 12. Using the known section modulus, solve for the unknown flange height, h. Guess h 1 in Given Z= I (h) c( h ) h Find ( h ) h 3.843 in Round this up to h 4.00 in FIGURE 5-28B
Channel Section for Problem 5-28

13. Summarizing, the ramp design dimensions are: Width w 12.00 in Flange height Thickness h 4.00 in t 0.5 in Shape Material channel 7075 aluminum

MACHINE DESIGN - An Integrated Approach, 4th Ed.

5-29-1

PROBLEM 5-29
Statement: A differential element is subected to the stresses given below and a ductile material has the strengths given below. Calculate the safety factor and draw 1-3 diagrams of each theory showing the stress state using: (a) Maximum shear-stress theory, and (b) Distortion-energy theory. Principal stresses Material properties Solution:

Given:

1 10 ksi
S ut 50 ksi

2 0 ksi
S y 40 ksi

3 20 ksi
S uc 50 ksi

See Figure 5-29 and Mathcad file P0529.

1. Calculate the slope of load line. (The load line is the line from the origin through the stress point.) m

3 1

m 2

2. The safety factor equation for the distortion-enrgy theory is the same regardless of which quadrant the load line falls in. However, the equation for the maximum shear-stress factor of safety is different for each of the three quadrants that the load line (1st, 3rd, or 4th) can fall in. In this case, the load line falls in the 4th quadrant. The factors of safety are: (a) Maximum shear-stress theory Na Sy Na 1.3

1 3

(b) Distortion energy theory


40

3
2
MINIMUM NONZERO PRINCIPAL STRESS, KSI

'

1 1 3 3

(a) Maximum shear stress boundary 30 (b) Distortion energy boundary 20 10 0 -10 (10,-20) -20 -30 -40 -50 -60 -40 -sy

' 26.5 ksi


Nb Sy Nb 1.5

'

sy

3. Plot the 1-3 diagram showing the safe-fail boundaries, the stress state point (10 ksi, -20 ksi) and the load line. Note that if 1 > 3 , then only that area on the graph that is to the right of and below the diagonal line can contain valid stress points. The factor of safety is the distance along the load line from the origin to the intersection of the load line with the failure boundary, divided by the distance from the origin to the stress point. Since the distance from the origin to the distortion-energy boundary is greater than the distance to the maximum shear-stress baoundary, its factor of safety is greater.

Stress states at which failure will occur Load Line

-30

-20

-10

10

20

30

40

50

MAXIMUM PRINCIPAL STRESS, KSI

FIGURE 5-29
1 - 3 Diagram for Problem 5-29

MACHINE DESIGN - An Integrated Approach, 4th Ed.

5-30-1

PROBLEM 5-30
Statement: A differential element is subected to the stresses and strengths given below. Calculate the safety factor and draw 1-3 diagrams of each theory showing the stress state using: (a) Coulomb-Mohr theory, and (b) Modified Mohr theory. Principal stresses Material properties Solution:

Given:

1 10 ksi
S ut 50 ksi

2 0 ksi
S y 40 ksi

3 20 ksi
S uc 90 ksi

See Figure 5-30 and Mathcad file P0530.

1. Calculate the slope of load line. (The load line is the line from the origin through the stress point.) m

3 1

m 2

2. The safety factor equation for both theories is different for each quadrant the load line falls in. The equation for the modified Mohr factor of safety is different for each of the two regions in the 4th quadrant that the load line can fall in. In this case, the load line falls in the 4th quadrant, below the -1 slope line.. The factors of safety are: (a)Coulomb-Mohr theory Na S uc S ut S uc 1 S ut 3 Na 2.4

(b) Modified Mohr theory Nb S uc


MINIMUM NONZERO PRINCIPAL STRESS, KSI

3
50 40 30 20 10 0 -10 (10,-20) -20 -30 -40 -50 -60 -70 -80 -90 -S uc 0 10 20 30 40 50 Stress states at which failure will occur (b) Modified Mohr boundary -S ut (a) Coulomb-Mohr boundary

S uc S ut S 1 3 ut

Nb 3.2 3. Plot the 1-3 diagram showing the safe-fail boundaries, the stress state point (10 ksi, -20 ksi) and the load line. Note that if 1 > 3 , then only that area on the graph that is to the right of and below the diagonal line can contain valid stress points. The factor of safety is the distance along the load line from the origin to the intersection of the load line with the failure boundary, divided by the distance from the origin to the stress point. Since the distance from the origin to the modified Mohr boundary is greater than the distance to the Coulomb-Mohr boundary, its factor of safety is greater.

Load Line

-100 -100 -90 -80 -70 -60 -50 -40 -30 -20 -10

MAXIMUM PRINCIPAL STRESS, KSI

FIGURE 5-30
1 - 3 Diagram for Problem 5-30

MACHINE DESIGN - An Integrated Approach, 4th Ed.

5-31-1

PROBLEM 5-31
Statement: Design a jack-stand in a tripod configuration that will support 2 tons of load with a safety factor of 3. Use SAE 1020 steel and minimize its weight. This open-ended design problem has many valid solutions that are left to the student.

Solution:

MACHINE DESIGN - An Integrated Approach, 4th Ed.

5-32-1

PROBLEM 5-32
Statement: Given: A part has the combined stress state and strengths given below. Choose an appropriate failure theory based on the given data, find the effective stress and factor of safety against static failure. Stresses: x 10 ksi Strengths: Solution: 1. 2.

y 5 ksi

xy 4.5 ksi
S uc 80 ksi

S y 18 ksi

S ut 20 ksi

See Mathcad file P0532.

Because S uc is greater than S ut, this is an uneven material, which is characteristic of a brittle material. Therefore, use the modified Mohr theory. Find the maximum shear stress and principal stresses that result from this combination of applied stresses using equations 4.6.
2 x y 2 max xy 2

Maximum shear stress

max 5.148 ksi

Principal stresses

1 2

x y
2

max max

1 12.648 ksi 2 2.352 ksi

x y
2

3 0 psi
3. Find the Dowling factors C1, C2, C3 using equations 5.12b: C1 1 2 1 2 1 2 1 2

S uc 2 S ut S uc S uc 2 S ut S uc S uc 2 S ut S uc

1 2

C1 8.898 ksi

C2

2 3

2 3 3 1

C2 1.764 ksi

C3 4.

3 1

C3 9.486 ksi

Then find the largest of the six stresses C1, C2, C3 , 1, 2, 3:

C 1 C2 C 3 eff max 1 2 3
which is the modified-Mohr effective stress. 5.

eff 12.6 ksi

The safety factor can now be found using equation 5.12d.

S ut

eff

N 1.6

MACHINE DESIGN - An Integrated Approach, 4th Ed.

5-33a-1

PROBLEM 5-33a
Statement: Solution: 1. For the bracket shown in Figure P5-14 and the data in row a of Table P5-3, determine the von Mises stresses at points A and B. See Mathcad file P0533a.

From Problem 4-33a the principal stresses at point A are

1 21.46 MPa
2.

2 0 MPa

3 13.08 MPa

Use equation (5.7c) to find the von Mises stress at point A.

'A
3.

1 1 3 3

'A 30.2 MPa

From Problem 4-33a the principal stresses at point B are

1 16.13 MPa
4.

2 0 MPa

3 16.13 MPa

Use equation (5.7c) to find the von Mises stress at point B.

'B

1 1 3 3

'B 27.9 MPa

y A T M L R B

FIGURE 5-33
Free Body Diagram of Tube for Problem 5-33

MACHINE DESIGN - An Integrated Approach, 4th Ed.

5-34a-1

PROBLEM 5-34a
Statement: Calculate the safety factor for the bracket in Problem 5-33 using the distortion energy, the maximum shear stress, and the maximum normal-stress theories. Comment on their appropriateness. Assume a ductile material strength as given below. Yield strength See Mathcad file P0534a. S y 400 MPa

Given: Solution: 1.

From Problem 4-33a the principal stresses at point A are

1A 21.46 MPa
2.

2A 0 MPa

3A 13.08 MPa

Using the two nonzero stresses, the slope of the load line on a 1-3 graph is m

3A 1A

m 0.61

This intersects the failure boundaries in the fourth quadrant. 3. Calculate the von Mises effective stress at point A using equation (5.7c).

'A
4.

1A 1A 3A 3A

'A 30.205 MPa

Determine the factor of safety at point A Distortion energy NADE Sy NADE 13.2

'A
Sy

Maximum shear stress

NAMS

1A 3A
Sy

NAMS 11.6

Maximum normal stress

NANS

1A

NANS 18.6

5.

From Problem 4-33a, the principal stresses at Point B are

1B 16.13 MPa
6.

2B 0 MPa

3B 16.13 MPa

Using the two nonzero stresses, the slope of the load line on a 1-3 graph is m

3B 1B

m 1

This intersects the failure boundaries in the fourth quadrant. 7. Calculate the von Mises effective stress at point A using equation (5.7c).

'B
8.

1B 1B 3B 3B

'B 27.938 MPa

Determine the factor of safety at point B

MACHINE DESIGN - An Integrated Approach, 4th Ed.


Sy

5-34a-2

Distortion energy

NBDE

'B
Sy

NBDE 14.3

Maximum shear stress

NBMS

1B 3B
Sy

NBMS 12.4

Maximum normal stress

NBNS

1B

NBNS 24.8

9.

Whichever theory is used, the critical point (lowest factor of safety) is point A. The distortion energy theory should be used because experimental data follows its failure boundary more nearly than the maximum shear stress in all quadrants. Using the maximum normal stress theory would give an overestimate of the actual safety factor.

MACHINE DESIGN - An Integrated Approach, 4th Ed.

5-35a-1

PROBLEM 5-35a
Statement: Calculate the safety factor for the bracket in Problem 5-33 using the Coulomb-Mohr and the modified Mohr effective stress theories. Comment on their appropriateness. Assume a brittle material strength as given below. Tensile strength See Mathcad file P0535a. S ut 350 MPa Compressive strength S uc 1000 MPa

Given: Solution: 1.

From Problem 4-33a the principal stresses at point A are

1A 21.46 MPa
2.

2A 0 MPa

3A 13.08 MPa

Using the two nonzero stresses, the slope of the load line on a 1-3 graph is m

3A 1A

m 0.61

This intersects the failure boundaries in the fourth quadrant. For the Coulomb-Mohr diagram (see Figure 5-9 in the text) there is a single, straight line in this quadrant. For the modified-Mohr theory, the load line will intersect the boundary at a point similar to B' in Figure 5-11 in the text. 3. Determine the factor of safety at point A Coulomb-Mohr NACM S ut S uc S uc 1A S ut 3A S ut NACM 13.4

Modified-Mohr

NAMM

1A

NAMM 16.3

4.

From Problem 4-33a, the principal stresses at Point B are

1B 16.13 MPa
5.

2B 0 MPa

3B 16.13 MPa

Using the two nonzero stresses, the slope of the load line on a 1-3 graph is m

3B 1B

m 1

This intersects the failure boundaries in the fourth quadrant. For the Coulomb-Mohr diagram (see Figure 5-9 in the text) there is a single, straight line in this quadrant. For the modified-Mohr theory, the load line will intersect the boundary at the point (S ut -S ut) Figure 5-11 in the text. 6. Determine the factor of safety at point B Coulomb-Mohr NBCM S ut S uc S uc 1B S ut 3B S ut NBCM 16.1

Modified-Mohr 7.

NBMM

1B

NBMM 21.7

Whichever theory is used, the critical point (lowest factor of safety) is point A. The modified-Mohr theory should be used because experimental data follows its failure boundary more nearly than the Coulomb-Mohr when the slope of the load line is in the fourth quadrant. Using the Coulomb-Mohr would give an underestimat of the actual safety factor.

MACHINE DESIGN - An Integrated Approach, 4th Ed.


8. Calculating factor of safety using Modified Mohr and equations (5.12c, d, and e) Point A C1 1 2 1 2 1 2 1A 2A

5-35a-2

S uc 2 S ut S uc S uc 2 S ut S uc S uc 2 S ut S uc

1A 2A

C1 13.9 MPa

C2

2A 3A

2A 3A

C2 4.6 MPa

C3

3A 1A

3A 1A

C3 18.5 MPa

1 is maximum so

S ut

1A

N 16.3

Point B

C1

1 2 1 2 1 2

1B 2B

S uc 2 S ut S uc S uc 2 S ut S uc S uc 2 S ut S uc

1B 2B

C1 10.5 MPa

C2

2B 3B

2B 3B

C2 5.6 MPa

C3

3B 1B

3B 1B

C3 16.1 MPa

1 is maximum so

S ut

1B

N 21.7

MACHINE DESIGN - An Integrated Approach, 4th Ed.

5-36a-1

PROBLEM 5-36a
Statement: For the bracket shown in Figure P5-14 and the data in row a of Table P5-3, redo Problem 5-33 considering the stress concentration at points A and B. Assume a stress concentration factor of 2.5 in both bending and torsion. Factors of safety: Bending See Mathcad file P0536a. Kf 2.5 Torsion Kfs 2.5

Given:

Solution: 1. 2.

From Problem 4-36a the principal stresses at point A are 1 53.6 MPa 2 0 MPa Use equation (5.7c) to find the von Mises stress at point A.

3 32.7 MPa 'A 75.5 MPa 3 41.3 MPa

'A
3.

1 1 3 3

From Problem 4-36a the principal stresses at point B are 1 41.3 MPa 2 0 MPa Use equation (5.7c) to find the von Mises stress at point B.

4.

'B

1 1 3 3

'B 71.5 MPa


F

y A T M L R B

FIGURE 5-36
Free Body Diagram of Tube for Problem 5-36

MACHINE DESIGN - An Integrated Approach, 4th Ed.

5-37-1

PROBLEM 5-37
Statement: A semicircular, curved beam as shown in Figure 5-37 has the dimensions given below. For a load pair F = 14 kN applied along the diameter, find the safety factor at the inner and outer fibers: (a) If the beam is a ductile material with Sy = 700 MPa, (b) If the beam is a cast-brittle material with Sut = 420 MPa, Suc = 1200 MPa. (a) Yield strength (b) Tensile strength Solution: 1. S ut 420 MPa
F od id F w

Given: S y 700 MPa

Compressive strength S uc 1200 MPa See Figure 5-37 and Mathcad file P0537.

From Problem 4-37, the stresses at the inside radius and outside radius are: Inside Outside

i 409.9 MPa o 273.2 MPa


(a) Entire Beam

These are the only stress components present on their respective surfaces so they are also principal stresses. Thus,

1i 409.9 MPa 1o 0 MPa


Part (a) 2.

2i 0 MPa 2o 0 MPa

3i 0 MPa 3o 273.2 MPa

F M F rc (b) Critical Section

Use the distortion energy theory for the ductile material.

3. Since 1 is the only nonzero principal stress, it is also the von Mises effective stress,

FIGURE 5-37
Free Body Diagrams for Problem 5-37

'i 1i 'o 3o
4.

'i 409.9 MPa 'o 273.2 MPa

The factor of safety against a static failure for this ductile material is Inside surface Nai Nao Sy Nai 1.7 Nao 2.6

'i
Sy

Outside surface Part (b) 5. 6.

'o

Use the modified-Mohr theory for the brittle material. The load line on the 1-3 graph for the inside surface is along the positive 1 axis. In this case, the factor of safety equation simplifies to Inside surface Nbi S ut Nbi 1.0

1i

7.

The load line on the 1-3 graph for the outside surface is along the negative 3 axis. In this case, the factor of safety equation simplifies to Outside surface Nbo S uc Nbo 4.4

3o

MACHINE DESIGN - An Integrated Approach, 4th Ed.

5-38-1

PROBLEM 5-38
Statement: Given: Assume that the curved beam of Problem 5-37 has a crack on its inside surface of half-width a = 2 mm and a fracture toughness of 50 MPa-m0.5. What is its safety factor against sudden fracture? Outside diameter Width of section Half crack length Solution: 1. od 150 mm t 25 mm a 2 mm Inside diameter Load Fracture toughness id 100 mm F 14 kN Kc 50 MPa m

See Figure 5-38 and Mathcad file P0538.

From Problem 4-37, the nominal stress at the inside radius is: Nominal inside stress i 409.9 MPa Calculate the half-width of the beam. b 0.5 t b 12.5 mm

2. 3.

Calculate the geometry and stress intensity factors. a sec 1.016 2 b K i a K 33.01 MPa m NFM
w

4.

Determine the factor of safety against sudden fracture failure

Kc K

NFM 1.5

F od id F

(a) Entire Beam

F M F rc (b) Critical Section

FIGURE 5-38
Free Body Diagrams for Problem 5-38

MACHINE DESIGN - An Integrated Approach, 4th Ed.

5-39-1

PROBLEM 5-39
Statement: Consider the failed 260-in dia by 0.73-in wall rocket case of Figure 5-14. The steel had S y = 240 k and a fracture toughness Kc = 79.6 ksi-in 0.5. It was designed for an internal pressure of 960 psi but failed at 542 psi. Failure was attributed to a small crack that precipitated a sudden, brittle, fracture-mechanics failure. Find the nominal stress in the wall and the yielding safety factor at the failure conditions and estimate the size of the crack that caused it to explode. Assume b = 1.0. Given: Case diameter Wall thickness Yield strength Solution: 1. See Mathcad file P0539. d 260 in t 0.73 in S y 240 ksi Fracture toughness Design pressure Failure pressure Kc 79.6 ksi in p d 960 psi p f 542 psi

Find the nominal stress in the wall. The ratio of the wall thickness to the radius of the case is such that we can use thin-wall theory. Thus Case radius Tangential stress r 0.5 d r 130 in pd r t pd r 2 t

t a

t 171.0 ksi a 85.5 ksi

Axial stress Radial stress 2.

r 0 psi

Find the yielding safety factor at the failure conditions. Since, for these directions, there are no shear stresses present, these are the principal stresses. The von Mises stress is Von Mises stress Factor of safety against yielding

'
Ns

t t a a
Sy

' 148.1 ksi


Ns 1.6

'

3.

Estimate the size of the crack that caused it to explode. Tangential stress

t a

pf r t pf r 2 t

t 96.5 ksi a 48.3 ksi

Axial stress

(a) Assume that the crack was longitudinal (growing in the axial direction) Nominal stress Stress intensity factor

nom t
K = nom a

nom 96.5 ksi

Setting the stress intensity factor equal to the fracture toughness of the material and solving for the crack length

Half-length

Kc2 a 2 nom
2 a 0.433 in

a 0.216 in

Crack length

MACHINE DESIGN - An Integrated Approach, 4th Ed.


(b) Assume that the crack was tangential (growing in the tangential direction) Nominal stress Stress intensity factor

5-39-2

nom a
K = nom a

nom 48.3 ksi

Setting the stress intensity factor equal to the fracture toughness of the material and solving for the crack length

Half-length

Kc2 2 nom

a 0.866 in

Crack length

2 a 1.732 in

MACHINE DESIGN - An Integrated Approach, 4th Ed.

5-40-1

PROBLEM 5-40
Statement: Redesign the roll support of Problem 5-8 to be like that shown in Figure P5-16. The stub mandrels insert to 10% of the roll length at each end. Design dimension a for a factor of safety of 2. See Problem 5-8 for additional data. (a) The beam is a ductile material with S y = 300 MPa (b) The beam is a cast-brittle material with S ut = 150 MPa, S uc = 570 MPa. Paper roll dimensions: OD 1.50 m ID 0.22 m Lroll 3.23 m Material properties: Yield strength Comp strength Roll density

Given:

S y 300 MPa S uc 570 MPa Ns 2

Tensile strength S ut 150 MPa

984 kg m

Factor of safety

Assumptions: 1. The paper roll's weight creates a concentrated load acting at the tip of the mandrel. 2. The base of the mandrel (the portion that inserts into the stanchion) is solid and fits tightly into the stanchion. Therefore, the mandrel can be treated as a cantilever beam. 3. The length of ther mandrel base is b 100 mm. Solution: 1. See Figure 5-40 and Mathcad file P0540.

For the assumptions made, it is not necessary to determine the stress distribution on the mandrel base inside the stanchion. From Figure 5-40, we see that we can determine the diameter a by applying the beam stress equation at the section where the mandrel transitions from the base to the full diameter. Determine the weight of the roll, the load on each support, and the length of the mandrel. W

F y

a M1

2.

OD 4

ID L
2

b R

Lm

roll g

W 53.9 kN F 26.95 kN Lm 323 mm FIGURE 5-40

F 0.5 W Lm 0.1 Lroll 3.

Free Body Diagram used in Problem 5-40

From Figure 5-40, the maximum internal bending moment occurs at x = 0 and is Mmax F Lm Mmax 8.704 kN m

4.

The bending stress will be a maximum at the top or bottom of the mandrel at a section through x = 0.

max =

Mmax a 2 I

where

I=

a
64

There are no other stress components at this point so max = 1 and

2 0 MPa
5.

3 0 MPa

For the ductile material of part (a), the maximum principal stress is also the von Mises stress so

max = ' =

32 Mmax

Sy Ns

MACHINE DESIGN - An Integrated Approach, 4th Ed.


1

5-40-2

Solving for a,

32 Ns F Lm a S y
a 84 mm

a 83.922 mm

Round this to 5.

for the ductile material of part (a)

For the brittle material of part (b), the load line on the 1-3 diagram is along the positive 1 axis where both brittle material failure theories have the same boundary, which is 1 = S ut. Thus, for the brittle case of part (b),

max = 1 =

32 Mmax

S ut Ns

a
1

Solving for a,

32 Ns F Lm S ut

a 105.735 mm

Round this to

a 106 mm

for the brittle material of part (b)

MACHINE DESIGN - An Integrated Approach, 4th Ed.

5-41-1

PROBLEM 5-41
Statement: A 10-mm ID steel tube carries liquid at 7 MPa. The steel has S y = 400 MPa Determine the safety factor for the wall if its thickness is: a) 1 mm, b) 5 mm. Yield strength S y 400 MPa

Given: Assumption: Solution:

The tubing is long therefore the axial stress is zero. See Mathcad file P0541. t 1 mm

(a) Wall thickness is 1.

From Problem 4-41, this is a thick wall cylinder and the principal stresses are:

1a 38.82 MPa
2.

2a 0 MPa

3a 7.00 MPa

Calculate the von Mises effective stress using equation (5.7c).


2 2

'a
3.

1a 1a 3a 3a

'a 42.752 MPa

Using the distortion energy theory, the factor of safety is Sy

Na

'a
t 5 mm

Na 9.4

(b) Wall thickness is 4.

From Problem 4-41, this is a thick wall cylinder and the principal stresses are:

1b 11.67 MPa
5.

2b 0 MPa

3b 7.00 MPa

Calculate the von Mises effective stress using equation (5.7c).


2 2

'b
6.

1b 1b 3a 3b

'b 16.336 MPa

Using the distortion energy theory, the factor of safety is Sy

Nb

'b

Nb 24.5

MACHINE DESIGN - An Integrated Approach, 4th Ed.

5-42-1

PROBLEM 5-42
Statement: A cylindrical tank with hemispherical ends is required to hold 150 psi of pressurized air at room temperature. The steel has S y = 400 MPa. Determine the safety factor if the tank diameter is 0.5 m with 1 mm wall thickness, and its length is 1 m. Yield strength S y 400 MPa

Given: Solution: 1.

See Mathcad file P0542.

From Problem 4-42, the maximum principal stresses in the wall are

1 259 MPa
2. The von Mises stress is

2 129 MPa ' 1 1 2 2


2 2

3 0 MPa ' 224.301 MPa

3.

Using the distortion-energy theory, the factor of safety against a static failure is Ns Sy Ns 1.8

'

MACHINE DESIGN - An Integrated Approach, 4th Ed.

5-43-1

PROBLEM 5-43
Statement: The paper rolls in Figure P5-17 are 0.9-m OD by 0.22-m ID by 3.23-m long and have a density of 984 kg/m3. The rolls are transfered from the machine conveyor (not shown) to the forklift truck by the V-linkage of the off-load station, which is rotated through 90 deg by an air cylinder. The paper then rolls onto the waiting forks of the truck. The forks are 38-mm thick by 100-mm wide by 1.2-m long and are tipped at a 3-deg angle from the horizontal and have Sy = 600MPa. Find the safety factor for the two forks on the truck when the paper rolls onto it under two different conditions (state all assumptions): (a) The two forks are unsupported at their free end. (b) The two forks are contacting the table at point A.
F L fork

Given:

Yield strength

S y 600 MPa

Assumptions: 1. The greatest bending moment will occur when the paper roll is at the tip of the fork for case (a) and when it is midway between supports for case (b). 2. Each fork carries 1/2 the weight of a paper roll. 3. For case (a), each fork acts as a cantilever beam (see Appendix B-1(a)). 4. For case (b), each fork acts as a beam that is built-in at one end and simply-supported at the other. Solution: 1. See Figure 5-43 and Mathcad file P0543.

R1 Case (a), Cantilever Beam

M1

0.5 L fork t

L fork R1 R2

M2

From Problem 4-43, the maximum stresses in the forks are: Case (a)

Case (b), Fixed-Simply Supported Beam

a 464.8 MPa

FIGURE 5-43
Free Body Diagrams used in Problem 5-43

at the base of the fork. Case (b)

b 87.2 MPa

also at the base of the fork.

Since there are no other stress components present, these are also the maximum principal stresses and the von Mises stresses. Thus, 'a a and 'b b. Case (a) 2. The factor of safety against a static failure is Nsa Sy

'a

Nsa 1.3

Case (b) 3. The factor of safety against a static failure is Nsb Sy

'b

Nsb 6.9

MACHINE DESIGN - An Integrated Approach, 4th Ed.

5-44-1

PROBLEM 5-44
Statement: Determine a suitable thickness for the V-links of the off-loading station of Figure P5-17 to limit their deflections at the tips to 10-mm in any position during their rotation. Two V-links support the roll, at the 1/4 and 3/4 points along the roll's length, and each of the V arms is 10-cm wide by 1-m long. What is their safety factor against yielding when designed to limit deflection as above? Roll OD Roll ID Roll length Roll density Yield strength OD 0.90 m ID 0.22 m Lroll 3.23 m
3

Given:

Arm width Arm length Max tip deflection Mod of elasticity

wa 100 mm La 1000 mm

tip 10 mm
E 207 GPa

984 kg m

S y 400 MPa

Assumptions: 1. The maximum deflection on an arm will occur just after it begins the transfer and just before it completes it, i.e., when the angle is either zero or 90 deg., but after the tip is no longer supported b the base unit. 2. At that time the roll is in contact with both arms ("seated" in the V) and will remain in that state throughout the motion. When the roll is in any other position on an arm the tip will be supported. 3. The arm can be treated as a cantilever beam with nonend load. 4. A single arm will never carry more than half the weight of a roll. 5. The pipe to which the arms are attached has OD = 160 mm. Solution: See Figure 5-44 and Mathcad file P0544.
450

1. Determine the weight of the roll and the load on each V-arm. W

OD ID Lroll g

W 18.64 kN F 9.32 kN

F 0.5 W

2. From Appendix B, Figure B-1, the tip deflection of a cantilever beam with a concentrated load located at a distance a from the support is ymax = F a
2

6 E I

( a 3 L)
370 = a

1000 = L F

where L is the beam length and I is the cross-section moment of inertia. In this case I= 3. Setting w a t a 12 and a 370 mm
3
M F

ymax = tip

FIGURE 5-44
Free Body Diagram used in Problem 5-44

substituting for I and solving for ta


1

2 F a2 3 La a ta E tip wa
Let the arm thickness be

ta 31.889 mm ta 32 mm

4. The maximum bending stress in the arm will be at its base where it joins the 160-mm-dia pipe. The bending moment, moment of inertia, and distance to the outside fiber at that point are:

MACHINE DESIGN - An Integrated Approach, 4th Ed.

5-44-2

Bending moment Moment of inertia Distance to outer fiber

M a F I wa ta 12
3

M 3449 N m I 2.731 10 mm c 16 mm
5 4

c 0.5 t a

5. The bending stress, which is also the von Mises stress, is

'

M c I

' 202.1 MPa

6. Using the distortion-energy theory, the factor of safety against a static failure is Ns Sy Ns 2.0

'

MACHINE DESIGN - An Integrated Approach, 4th Ed.

5-45-1

PROBLEM 5-45
Statement: Determine the safety factor based on critical load on the air cylinder rod in Figure P5-17 if the crank arm that rotates it is 0.3 m long and the rod has a maximum extension of 0.5 m. The 25-mm-dia rod is solid steel with a yield strength of 400 MPa. State all assumptions. Rod length Rod diameter L 500 mm d 25 mm Young's modulus Yield strength E 207 GPa S y 400 MPa

Given:

Assumptions: 1. The rod is a fixed-pinned column. 2. Use a conservative value of 1 for the end factor (see Table 4-7 in text). Solution: 1. 2. 3. See Problems 4-45, 4-47, and Mathcad file P0545. Pcr 134.8 kN F 46.47 kN Nbuck 2.9

From Problem 4-45, the critical load on the air cylinder rod is From Problem 4-47, the maximum load on the air cylinder rod is The factor of safety against a buckling failure is Nbuck Pcr F

MACHINE DESIGN - An Integrated Approach, 4th Ed.

5-46-1

PROBLEM 5-46
Statement: The V-links of Figure P5-17 are rotated by the crank arm through a shaft that is 60 mm dia by 3.23 m long. Determine the maximum torque applied to this shaft during motion of the V-linkage and find the static safety factor against yielding for the shaft if its S y = 400 MPa. See Problem 5-43 for more information.
y

Given:

Yield strength

S y 400 MPa

Assumptions: The greatest torque will occur when the link is horizontal and the paper roll is located as shown in Figure P5-17 or Figure 5-46. Solution: P0546. See Figure 5-46 and Mathcad file

1. From Problem 4-46, the maximum torsional stress in the shaft is

W T

max 197.88 MPa


2. Using the distortion-energy theory, the factor of safety against static yielding is
60-mm-dia shaft Ry 450.0

Ns

Sy 3 max

Ns 1.2

FIGURE 5-46
Free Body Diagram used in Problem 5-46

MACHINE DESIGN - An Integrated Approach, 4th Ed.

5-47-1

PROBLEM 5-47
Statement: Determine the maximum forces on the pins at each end of the air cylinder of Figure P4-17. Determine the safety factor for these pins if they are 30-mm dia and in single shear. S y = 400 MPa. Paper roll dimensions OD 0.90 m ID 0.22 m Lroll 3.23 m Roll density
3

Given:

Pin diameter Yield strength

d 30 mm S y 400 MPa

984 kg m

Assumptions: 1. The maximum force in the cylinder rod occurs when the transfer starts. 2. The cylinder and rod make an angle of 8 deg to the horizontal at the start of transfer. 3. The crank arm is 300 mm long and is 45 deg from vertical at the start of transfer. 4. The cylinder rod is fully retracted at the start of the transfer. At the end of the transfer it will have extended 500 mm from its initial position. Solution: See Figure 4-47 and Mathcad file P0447.
y

1. Determine the weight of the roll on the forks. W

OD ID Lroll g

W 18.64 kN 2. From the assumptions and Figure 4-47, the x and y distances from the origin to point A are, Rax 300 cos( 45 deg) mm Ray 300 sin( 45 deg) mm Rax 212.132 mm Ray 212.132 mm
212.1 W Rx x 212.1 A F 450.0 8

Ry

3. From Figure 4-47, the x distance from the origin to point where W is applied is,

FIGURE 4-47
Free Body Diagram at Start of Transfer for V-link of Problem 4-47

Rwx

OD 2

Rwx 450 mm

4. Sum moments about the pivot point and solve for the compressive force in the cylinder rod. W Rwx F Rax sin( 8 deg) F Ray cos( 8 deg) = 0 F W Rwx Ray cos( 8 deg) Rax sin( 8 deg) F 46.469 kN

This is the shear force in the pins 5. Determine the cross-sectional area of the pins and the direct shear stress.

MACHINE DESIGN - An Integrated Approach, 4th Ed.


2

5-47-2

Shear area

d
4 F A

A 706.858 mm

Shear stress

65.7 MPa

6. Using the distortion-energy theory, the factor of safety against a static yielding failure is Ns Sy 3 Ns 3.5

MACHINE DESIGN - An Integrated Approach, 4th Ed.

5-48-1

PROBLEM 5-48
Statement: Figure P5-18 shows an exerciser for a 100-kg wheelchair racer. The wheel chair has 65 cm dia drive wheels separated by a 70-cm track width. Two free-turning rollers on bearings support the rear wheels. The lateral movement of the chair is limited by the flanges. Design the 1-m-long rollers as hollow tubes of aluminum (select alloy) to minimize the height of the platform and also limit the roller deflections to 1 mm in the worst case. Specify suitable sized steel axles to support the tubes on bearings. Calculate all significant stresses. Mass of chair M 100 kg Wheel diameter d w 650 mm Track width Roller length T 700 mm Lr 1000 mm Maximum deflection Modulus elasticity: Aluminum Steel

Given:

1 mm
Ea 71.7 GPa Es 207 GPa

Assumptions: 1. The CG of the chair with rider is sufficiently close to the rear wheel that all of the weight is taken by the two rear wheels. 2. The small camber angle of the rear wheels does not significantly affect the magnitude of the forces on the rollers. 3. Both the aluminum roller and the steel axle are simply supported. The steel axles that support the aluminum tube are fixed in the mounting block and do not rotate. The aluminum tube is attached to them by two bearings (one on each end of the tubes, one for each axle). The bearings' inner race is fixed, and the outer race rotates with the aluminum tube. Each steel axle is considered to be loaded as a simply supported beam. Their diameter must be less than the inner diameter of the tubes to fit the roller bearings between them. Solution: See Figures 5-48 and Mathcad file P0548.

W/2

F
FIGURE 5-48A

Free Body Diagram of One Wheel used in Problem 5-48

1. Calculate the weight of the chair with rider. Weight of chair W M g W 980.7 N

2. Calculate the forces exerted by the wheels on the rollers (see Figure 5-48A). From the FBD of a wheel, summing vertical forces 2 F cos( ) Let W 2 =0 then F W 4 cos( ) F 260.9 N

20 deg

3. The worst condition (highest moment at site of a stress concentration) will occur when the chair is all the way to the left or right. Looking at the plane through the roller that includes the forces exerted by the wheels (the FBD is shown in Figure 5-48B) the reactions R1 and R2 come from the bearings, which are inside the hollow roller and are, themselves, supported by the steel axle. 4. Solving for the reactions. Let the distance from R1 to F be a 15 mm

MACHINE DESIGN - An Integrated Approach, 4th Ed.

5-48-2
700

M1 Fy
R2

R2 Lr F ( a T ) F a = 0 R1 2 F R2 = 0 F (2 a T ) Lr R2 190.5 N

15 R1 1000

R2

R1 2 F R2

R1 331.3 N FIGURE 5-48B


Free Body Diagram of One Tube used in Problem 5-48

5. The maximum bending moment will be at the right-hand load and will be Mrmax R2 Lr ( a T ) Lr T 2

Mrmax 54.3 N m

Note, if the chair were centered on the roller the maximum moment would be Mc F Mc 39.1 N m

and this would be constant along the axle between the two loads, F. 6. Note that the bearing positions are fixed regardless of the position of the chair on the roller. Because of symmetry, Ra1 R1 Ra2 R2 Ra1 331.3 N Ra2 190.5 N
R a1 1130 R a2 65 R1 1000 R2

7. The maximum bending moment occurs at R1 and is for b 65 mm Mamax Ra1 b Mamax 21.5 N m

FIGURE 5-48C
Free Body Diagram of One Axle used in Problem 5-48

8. Determine a suitable axle diameter. Let the factor of safety against yielding in the axle be Nsa 3 9. Tentatively choose a low-carbon steel for the axle, say AISI 1020, cold rolled with S y 393 MPa 10. At the top of the axle under the load R1 there is only a bending stress, which is also the von Mises stress. Set th stress equal to the yield strength divided by the factor of safety.

' =

32 Mamax

Sy Nsa
1

d a

Solving for the axle diameter, d a

d a

32 Nsa Mamax S y

d a 11.875 mm

Let the axle diameter be

d a 15 mm

made from cold-rolled AISI 1020 steel.

MACHINE DESIGN - An Integrated Approach, 4th Ed.

5-48-3

11. Suppose that bearing 6302 from Chapter 10, Figure 10-23, page 684 is used. It has a bore of 15 mm and an OD of 42 mm. Thus, the inside diameter of the roller away from the bearings where the moment is a maximum will be d i 40 mm. This will provide a 1-mm shoulder for axial location of the bearings. 12. The maximum deflection of the roller will occur when the chair is in the center of the roller. For this case the reactions are both equal to the loads, F (see Figure 4-48D). The maximum deflection is at the center of the roller.
150 F 700 F

F 15 F 1000

13. Write the load function and then integrate four times to get the deflection function.

FIGURE 5-48D
Free Body Diagram of Roller with Chair in the Center.

q(x) = F<x>-1 - F<x - a>-1 - F<x - b>-1 + F<x - L>-1 y(x) = F[<x>3 - <x - a>3 - <x - b>3 + <x - L>3 + C3x]/(6EI) where C3 = 1 L ( L a ) a L
3 3 3

14. Write the deflection function at x = L/2 for a 150 mm

ymax

L 3 = 6 Ea I 2
F

3 L a 1 ( L a) 3 a3 L3 2 2

15. Set this equation equal to the allowed deflection and solve for the required moment of inertia, I.

Lr 3 I 6 Ea 2
F

3 Lr 1 3 3 3 a Lr a a Lr 2 2

I 6.618 10 mm

16. Knowing the inside diameter of the tube, solve for the outside diameter.
1

4 4 I= do di 64
Round this up to DESIGN SUMMARY Axles Material Diameter Length d o 46 mm

64 I 4 d o di

d o 44.463 mm

Rollers AISI 1020 steel, cold-rolled d a 15 mm 1220 mm Material Outside diameter Inside diameter Length Spacing 2024-T4 aluminum d o 46 mm d i 40 mm 1040 mm c d w d o sin( ) c 238 mm

MACHINE DESIGN - An Integrated Approach, 4th Ed.

5-49-1

PROBLEM 5-49
Statement:

_____

A part made of ductile steel with Sy = 40 ksi is subjected to a three-dimensional stress state of 1 = -80 ksi, 2 = -80 ksi, 3 = -80 ksi. What is the maximum shear stress? Will the part fail?

Solution: 1.

See Mathcad file P0549.

This is a case of hydrostatic stress. As explained in Section 5.1, the maximum shear stress is zero. Parts loaded hydrostatically can withstand stresses well in excess of their yield strength. One example of this is that parts on the ocean floor such as those retrieved from the Titanic are intact and undistorted even though they are surrounded by water at great pressure.

MACHINE DESIGN - An Integrated Approach, 4th Ed.

5-50-1

PROBLEM 5-50
Statement:

_____

A component in the shape of a large sheet is to be fabricated from 7075-T651 aluminum, which has a fracture toughness Kc = 24.2 MPa-m0.5 and a tensile yield strength of 495 MPa. Determine the largest edge crack that could be tolerated in the sheet if the nominal stress does not exceed one half the yield strength. Fracture toughness Yield strength Kc 24.2 MPa m S y 495 MPa
0.5

Given:

Solution: 1.

Mathcad file P0550.

Calculate the nominal stress based on the yield strength and the stress level given in the problem statement.

nom
2.

Sy 2

nom 247.5 MPa

Determine the value of the geometry factor from the discussion in Section 5.3 for a plate with an edge crack.

1.12
3. Using equation 5.14b, calculate the critical crack length for this material under the given stress condition.

Kc a nom
1

a 2.4 mm

MACHINE DESIGN - An Integrated Approach, 4th Ed.

5-51-1

PROBLEM 5-51
Statement:

_____

A component in the shape of a large sheet is to be fabricated from 4340 steel, which has a fracture toughness Kc = 98.9 MPa-m0.5 and a tensile yield strength of 860 MPa. The sheets are inspected for crack flaws after fabrication, but the inspection device cannot detect flaws smaller than 5 mm. The part is too heavy as designed. An engineer has suggested that the thickness be reduced and the material be heat-treated to increase its tensile strength to 1515 MPa, which would result in decreasing the fracture toughness to 60.4 MPa-m0.5. Assuming that the stress level does not exceed one half the yield strength, is the suggestion feasible? If not, why not. Fracture toughness Yield strength Kc1 98.9 MPa m S y1 860 MPa
0.5

Given:

Kc2 60.4 MPa m S y2 1515 MPa

0.5

Solution: 1.

See Mathcad file P0551.

Calculate the nominal stress for the two material conditions based on the yield strength and the stress level given in the problem statement.

nom1

S y1 2 S y2 2

nom1 430 MPa

nom2

nom2 757.5 MPa

2.

Determine the value of the geometry factor from the discussion in Section 5.3 for a large plate.

1
3. Using equation 5.14b, calculate the critical crack length for each material condition under the given stress condition.

Kc1 a 1 nom1
1

a 1 16.8 mm

2 a 1 33.7 mm

Kc2 a 2 nom2
1 4.

a 2 2.0 mm

2 a 2 4.0 mm

The suggestion to increase the strength of the material so that its thickness can be decreased to save weight is not feasible because the critical crack size of the material in the second condition is less than that which can be detected by the inspection equipment.

MACHINE DESIGN - An Integrated Approach, 4th Ed.

5-52-1

PROBLEM 5-52
Statement:

_____

A large plate is subjected to a nominal tensile stress of 350 MPa. The plate has a central crack that is 15.9 mm long. Calculate the stress intensity factor at the tip of the crack. Nominal stress Crack length

Given:

nom 350 MPa


lcrack 15.9 mm

Solution: 1.

See Mathcad file P0552.

Calculate the half-width of the crack a 0.5 l crack a 7.95 mm

2.

Determine the value of the geometry factor from the discussion in Section 5.3 for a plate with an edge crack.

1
3. Using equation 5.14b, calculate the stress intensity factor. K nom a K 55.3 MPa m
0.5

MACHINE DESIGN - An Integrated Approach, 4th Ed.

5-53-1

PROBLEM 5-53
Statement:

_____

A movie scene calls for a stuntman to hang from a rope that is suspended 3 m above a pit of poisonous spiders. The rope is attached to a glass sheet that is 3000 mm long by 100 mm wide and 1.27 mm thick. The stuntman knows that the glass sheet contains a central crack with total length of 16.2 mm that is oriented parallel to the ground. The fracture toughness of the glass is 0.83 MPa-m0.5. Should he do the stunt? Show all assumptions and calculations in support of your answer. Fracture toughness Glass dimensions Total crack length Kc 0.83 MPa m
0.5

Given:

L 3000 mm W 100 mm lcrack 16.2 mm Weight 900 N NFMd 3

t 1.27 mm

Assumptions: Weight of stuntman Desired safety factor Solution: 1. See Mathcad file P0553.

Calculate the nominal stress based on the assumed weight of the stuntman and the glass dimensions. Cross-section area Nominal stress A W t A 127 mm
2

nom

Weight A

nom 7.087 MPa

2.

Determine the value of the geometry factor from equation 5.14c for a plate with a central crack. Crack half-width Glass half-width a 0.5 l crack b 0.5 W a 8.1 mm b 50 mm

sec

2 b

1.017

3.

Using equation 5.14b, calculate the stress intensity factor for the given assumptions. K nom a K 1.149 MPa m
0.5

4.

Using equation 5.15, calculate the safety factor against sudden failure for the given assumptions. NFM Kc K NFM 0.72

5.

The stuntman should definitely not do the stunt since the factor of safety is not only less than the desired value, but is less than one.

MACHINE DESIGN - An Integrated Approach, 4th Ed.

5-54-1

PROBLEM 5-54
Statement:

_____

A material has a fracture toughness of 50 MPa-m0.5 and a yield strength of 1000 MPa and is to be made into a large panel. If the panel is stressed to one-half the yield stress, what is the maximum central crack size that can be tolerated without catastrophic failure? Fracture toughness Yield strength Kc 50 MPa m S y 1000 MPa
0.5

Given:

Solution: 1.

See Mathcad file P0554.

Calculate the nominal stress based on the yield strength and the stress level given in the problem statement.

nom
2.

Sy 2

nom 500 MPa

Determine the value of the geometry factor from the discussion in Section 5.3 for a large plate with a central crack.

1
3. Using equation 5.14b, calculate the critical crack length for this material under the given stress condition.

Kc a nom
1 lcritical 2 a

a 3.18 mm

lcritical 6.4 mm

MACHINE DESIGN - An Integrated Approach, 4th Ed.

5-55-1

PROBLEM 5-55
Statement:

_____

A material that has a fracture toughness of 33 MPa-m0.5 is to be made into a large panel that is 2000 mm long by 250 mm wide and 4 mm thick. If the minimum allowable total crack length is 4 mm, what is the maximum tensile load in the long direction that can be applied without catastrophic failure with a safety factor of 2.5? Fracture toughness Kc 33 MPa m
0.5

Given:

Panel dimensions L 2000 mm Total allow. crack length lcrack 4 mm Safety factor Solution: 1. See Mathcad file P0555. NFM 2.5

W 250 mm

t 4 mm

Calculate the allowable stress intensity factor using equation 5.15. Kallow Kc NFM Kallow 13.2 MPa m
0.5

2.

Determine the value of the geometry factor from equation 5.14c for a plate with a central crack. Crack half-width Panel half-width a 0.5 l crack b 0.5 W a 2 mm b 125 mm

sec

2 b

1.00

3.

Using equation 5.14b, calculate the allowable nominal stress in the panel.

allow

Kallow

allow 166.5 MPa

4.

Calculate the allowable load for the given conditions. Fallow allow W t Fallow 167 kN

MACHINE DESIGN - An Integrated Approach, 4th Ed.

5-56-1

PROBLEM 5-56
Statement:

_____

Figure P5-19 shows an SAE 1020 cold-rolled steel bar fastened to a rigid ground plane with two 0.25-in-dia A8 steel dowel pins, hardened to HRC52. For P = 1500 lb and t = 0.25 in, find: (a) The safety factor for each pin. (b) The safety factor for direct bearing stress in each hole. (c) The safety factor for tearout failure if h = 1 in. Pin diameter Applied load Distance between pins Thickness of bar See Mathcad file P0556. d 0.250 in P 1500 lbf a 2.0 in t 0.25 in Depth of section Distance from right pin to load Yield strength of bar Yield strength of pin h 1.0 in b 4.0 in S yb 57 ksi S yp 225 ksi

Given:

Solution: 1.

Draw a free-body diagram and find the shear forces (reactions) on each pin.

a RL

RR P
Write equations 3.3b for the bar and solve for the reactions.

F:
RL 2.

RL RR P 0 b a P RL 3000 lbf

M:

RL a P b 0 RR 4500 lbf

RR P RL

Calculate the cross-section area of a pin. A

d
4

A 0.0491 in

3.

Use equation 4.9 to determine the shear stress in each pin. Left pin

L R

RL A RR A

L 61.1 ksi R 91.7 ksi

Right pin 4.

(a) From equations 5.8c and 5.9b, the safety factor against failure in the pins is 0.577 S yp

Left pin

NL

NL 2.1

MACHINE DESIGN - An Integrated Approach, 4th Ed.

5-56-2

Right pin

NR

0.577 S yp

NR 1.4

5.

Calculate the bearing area from equation 4.10 and use it to determine the bearing stress in each hole. Bearing area Abear d t Abear 0.0625 in
2

L R
These are principal stresses 1. 6.

RL Abear RR Abear

L 48.0 ksi R 72.0 ksi

(b) Calculate the safety factor for direct bearing from equation 5.8c where 2 and 3 are both zero. Left hole NL S yb NL 1.2

L
S yb

Right hole

NR

NR 0.8

7.

The tearout area is

Atear 2

outside of the bar. Substitute this area in equation 4.9 for the shear area and solve for the shear strength xy. Atear 2

t , where (h - d)/2 is the distance from the edge of the hole to the 2

h d

t 2
L
RL Atear RR Atear

h d

Atear 0.187 in

Left hole

L 16.00 ksi

Right hole

R 24.00 ksi

8.

(c) From equations 5.8c and 5.9b, the safety factor against tearout failure in the holes is Left hole NL 0.577 S yb NL 2.1

L
0.577 S yb

Right hole

NR

NR 1.4

MACHINE DESIGN - An Integrated Approach, 4th Ed.

5-57-1

PROBLEM 5-57
Statement:

_____

Figure P5-19 shows a class 50 cast iron bar fastened to a rigid ground plane with two 0.25-in-dia A8 steel dowel pins, hardened to HRC52. For P = 1500 lb and t = 0.25 in, find: (a) The safety factor for each pin. (b) The safety factor for direct bearing stress in each hole. (c) The safety factor for tearout failure if h = 1 in. Pin diameter Applied load Distance between pins Thickness of bar d 0.250 in P 1500 lbf a 2.0 in t 0.25 in Depth of section Distance from right pin to load Tensile strength of bar Yield strength of pin h 1.0 in b 4.0 in S utb 52 ksi S yp 225 ksi

Given:

Solution: 1.

See Mathcad file P0557.

Draw a free-body diagram and find the shear forces (reactions) on each pin.

a RL

RR P
Write equations 3.3b for the bar and solve for the reactions.

F:
RL 2.

RL RR P 0 b a P RL 3000 lbf

M:

RL a P b 0 RR 4500 lbf

RR P RL

Calculate the cross-section area of a pin. A

d
4

A 0.0491 in

3.

Use equation 4.9 to determine the shear stress in each pin. Left pin

L R

RL A RR A

L 61.1 ksi R 91.7 ksi

Right pin 4.

(a) From equations 5.8c and 5.9b, the safety factor against failure in the pins is Left pin NL 0.577 S yp NL 2.1

MACHINE DESIGN - An Integrated Approach, 4th Ed.

5-57-2

Right pin

NR

0.577 S yp

NR 1.4

5.

Calculate the bearing area from equation 4.10 and use it to determine the bearing stress in each hole. Bearing area Abear d t Abear 0.0625 in
2

L R
These are principal stresses 1. 6.

RL Abear RR Abear

L 48.0 ksi R 72.0 ksi

(b) Calculate the safety factor for direct bearing from equation 5.12a where 2 and 3 are both zero. Left hole NL S utb NL 1.1

L
S utb

Right hole

NR

NR 0.7

7.

The tearout area is

Atear 2

outside of the bar. Substitute this area in equation 4.9 for the shear area and solve for the shear strength xy. Atear 2

t , where (h - d)/2 is the distance from the edge of the hole to the 2

h d

t 2
L
RL Atear RR Atear

h d

Atear 0.187 in

Left hole

L 16.00 ksi

Right hole

R 24.00 ksi

8.

(c) For pure shear the Mohr circle is centered at 0,0 and has a radius equal to the shear stress. This results in 1 = . Using the Modified-Mohr failure theory and Figure 5-11, we see that we can use equation 5.12a for the safety factor against tearout. NL S utb NL 3.3 NR S utb NR 2.2

MACHINE DESIGN - An Integrated Approach, 4th Ed.

5-58-1

PROBLEM 5-58
Statement:

_____

Figure P5-20 shows a bracket machined from 0.5-in-thick SAE 1045 cold-rolled steel flat stock. It is rigidly attached to a support and loaded with P = 5000 lb at point D. Find: (a) The safety factor against static failure at point A. (b) The safety factor against static failure at point B. Distance from support to: Point D d 8 in Depth of section h 3 in Applied load P 5000 lbf Points B and C b 17 in Thickness of section t 0.5 in Tensile yield strength S y 77 ksi

Given:

Assumptions: The bracket remains flat and does not buckle (out-of-plane) under the applied load. Solution: 1. See Mathcad file P0558.

Calculate the cross-section area and moment of inertia at A, B, and C, which are the same. A h t A 1.500 in
2

t h

12

I 1.1250 in

2.

For part (a), draw a free-body diagram of the entire bracket.

V A M h B h C d D h x y

P
3. Use the equilibrium equations 3.3a to calculate the shear force and bending moment at the support.

F:
4.

V P 0 V P V 5000 lbf

M:

P ( d ) M 0 MA P ( d ) c 0.5 h MA 40000 in lbf c 1.500 in

The normal stress in the bracket at point A is determined using equation 4.11b. Distance from neutral axis to extreme fiber Normal stress at point A MA c I

A 53.33 ksi

5.

(a) The transverse shear stress in the bracket at point A is zero, therefore A is a principal stress. There are no stress components in the y or z directions so this is a case of uniaxial stress. Thus, equations 5.7 reduce to

'

' 53.3 ksi

Use equation 5.8a to calculate the factor of safety against a static failure at point A.

MACHINE DESIGN - An Integrated Approach, 4th Ed.


NA Sy NA 1.4

5-58-2

'

6.

For part (b), draw a free-body diagram of the portion of the bracket that is below point B.

b F y B M x

P
7. Use the equilibrium equations 3.3a to calculate the normal force and bending moment on the section shown.

F:
8.

F P 0 F P F 5000 lbf

M:

P ( b 0.5 h d ) M 0 MB1 52500 in lbf

MB1 P ( b 0.5 h d )

The normal stress in the bracket at point B in the y direction is a combination of uniform tension and bending and is determined by summing equations 4.7 and 4.11b. Normal stress at B in y direction

By

MB1 c I

F A

By 73.33 ksi

9.

The normal stress in the bracket at point B in the x direction is bending and is determined from equation 4.11b, using the FBD from part (a). MB2 V b MA Normal stress at B in x direction MB2 45000 in lbf

Bx

MB2 c I

Bx 60.00 ksi

10. (b) The transverse shear at B due to the shear force V is zero so Bx and By are the only stress components at B. Use equations 5.7d and 5.8a to determine the factor of safety against a static failure at B (ignoring the stress concentration there).

'
NB

Bx By Bx By
Sy

' 67.66 ksi


NB 1.1

'

MACHINE DESIGN - An Integrated Approach, 4th Ed.

5-59-1

PROBLEM 5-59
Statement:

_____

Figure P5-20 shows a bracket machined from 1-in-thick class 60 cast iron flat stock. It is rigidly attached to a support and loaded with P = 5000 lb at point D. Find: (a) The safety factor against static failure at point A. (b) The safety factor against static failure at point B. Distance from support to: Point D d 8 in Depth of section h 3 in Applied load P 5000 lbf Points B and C b 17 in Thickness of section t 0.5 in Ultimate tensile strength S ut 62 ksi Ultimate comp. strength S uc 187 ksi

Given:

Assumptions: The bracket remains flat and does not buckle (out-of-plane) under the applied load. Solution: 1. See Mathcad file P0559. t h
3

Calculate the cross-section area and moment of inertia at A, B, and C, which are the same. A h t A 1.500 in
2

12

I 1.1250 in

2.

For part (a), draw a free-body diagram of the entire bracket.

V A M h B h C d D h x y

P
3. Use the equilibrium equations 3.3a to calculate the shear force and bending moment at the support.

F:
4.

V P 0 V P V 5000 lbf

M:

P ( d ) M 0 MA P ( d ) c 0.5 h MA 40000 in lbf c 1.500 in

The normal stress in the bracket at point A is determined using equation 4.11b. Distance from neutral axis to extreme fiber Normal stress at point A MA c I

A 53.33 ksi

5.

(a) The transverse shear stress in the bracket at point A is zero, therefore A is a principal stress. There are no stress components in the y or z directions so this is a case of uniaxial stress. Thus, use equation 5.12a (adapted to a compressive stress state) to calculate the factor of safety against a static failure at point A. NA S uc NA 3.5

MACHINE DESIGN - An Integrated Approach, 4th Ed.

5-59-2

6.

For part (b), draw a free-body diagram of the portion of the bracket that is below point B.

b F y B M x

P
7. Use the equilibrium equations 3.3a to calculate the normal force and bending moment on the section shown.

F:
8.

F P 0 F P F 5000 lbf

M:

P ( b 0.5 h d ) M 0 MB1 52500 in lbf

MB1 P ( b 0.5 h d )

The normal stress in the bracket at point B in the y direction is a combination of uniform tension and bending and is determined by summing equations 4.7 and 4.11b. Normal stress at B in y direction

By

MB1 c I

F A

By 73.33 ksi

9.

The normal stress in the bracket at point B in the x direction is bending and is determined from equation 4.11b, using the FBD from part (a). MB2 V b MA Normal stress at B in x direction MB2 45000 in lbf

Bx

MB2 c I

Bx 60.00 ksi

10. (b) The transverse shear at B due to the shear force V is zero so Bx and By are the only stress components at B. Use equations 4.6 to determine the principal stresses and 5.12a to determine the factor of safety against a static failure at B (ignoring the stress concentration there).

Bx By
2

2 Bx By 2 2 Bx By 2

1 73.333 ksi

Bx By
2

2 60.000 ksi

3 0 ksi

NB

S ut

NB 0.85

MACHINE DESIGN - An Integrated Approach, 4th Ed.

5-60-1

PROBLEM 5-60
Statement:

_____

Figure P5-21 shows a 1-in-dia SAE 1040 hot-rolled, normalized steel bar supported and subjected to the applied load P = 500 lb. Find the safety factor against static failure. Diameter Applied load Dimensions: d 1.00 in P 500 lbf a 20 in Modulus of elasticity E 30 10 psi Yield strength S y 54 ksi
6

Given:

L 40 in

Solution: 1.

See Mathcad file P0560.

Draw a free-body diagram.


L a R2

M1 R1 P

2.

This is a statically indeterminate beam because there are three unknown reactions, R1, M1, and R2. To solve for these unknowns, follow the method presented in Example 4-7. First, calculate the moment of inertia and distance to the extreme fiber for the round section. I

d
64

I 0.0491 in

c 0.5 d

c 0.500 in

3.

From inspection of the FBD, write the load function equation q(x) = -M1<x>-2 + R1<x>-1 - R2<x - a>-1 + P<x - L>-1

4.

Integrate this equation from - to x to obtain shear, V(x) V(x) = -M1<x>-1 + R1<x>0 - R2<x - a>0 + P<x - L>0

5.

Integrate this equation from - to x to obtain moment, M(x) M(x) = -M1<x>0 + R1<x>1 - R2<x - a>1 + P<x - L>1

6.

Integrate the moment function, multiplying by 1/EI, to get the slope. (x) = [ -M1<x>1 + R1<x>2/2 - R2<x - a>2/2 + P<x - L>2/2 + C3]/EI

5.

Integrate again to get the deflection. y(x) = [-M1<x>2/2 + R1<x>3/6 - R2<x - a>3/6 + P<x - L>3/6 + C3x + C4]/EI

7.

Evaluate R1, M1, R2, C3 and C4 At x = 0, y = 0 and = 0, therefore, C3 = 0 and C4 = 0. At x = a, y = 0 At x = L+, V = M = 0 Guess Given M1 1000 in lbf y(a) = 0: V(L) = 0: M1 2 a
2

R1 500 lbf R1 6 a = 0 lbf in


3 3

R2 1000 lbf

R1 R2 P = 0 lbf

MACHINE DESIGN - An Integrated Approach, 4th Ed.

5-60-2

M(L) = 0:

M1 R1 L R2 ( L a ) = 0 lbf in

M1 R1 Find M1 R1 R2 R 2
8. 9. Define the range for x x 0 in 0.02 L L

M1 5000 in lbf

R1 750 lbf R2 1250 lbf

For a Mathcad solution, define a step function S. This function will have a value of zero when x is less than z, and a value of one when it is greater than or equal to z. S ( x z) if ( x z 1 0 )

10. Write the moment equation in Mathcad form, using the function S as a multiplying factor to get the effect of the singularity functions. M ( x) M1 R1 S ( x 0 in) x R2 S ( x a ) ( x a ) P S ( x L) ( x L) 11. Plot the moment equation and determine the maximum bending moment.

As expected, the maximum bending moment occurs under the support at x = a. Mmax M ( a ) Mmax 10.0 kip in
M ( x) kip in

MOMENT DIAGRAM
10

10

20 x in

30

40

12. Use equation 4.11b to calculate the maximum bending stress in the bar.

max

Mmax c I

max 101.9 ksi

13. There are no other stress components present (the transverse shear is zero at the extreme fiber) so this is a principal stress and the other two principal stresses are zero. Thus, this is a case of uniaxial stress. Determine the safety factor against static failure using equations 5.7c and 5.8a, which reduce to N Sy N 0.53

max

MACHINE DESIGN - An Integrated Approach, 4th Ed.

5-61-1

PROBLEM 5-61
Statement:

_____

Figure P5-21 shows a 1.5-in-dia class 60 cast iron bar supported and subjected to the applied load = 500 lb. Find the safety factor against static failure. Diameter Applied load Dimensions: d 1.50 in P 500 lbf a 20 in Modulus of elasticity E 30 10 psi Tensile strength S ut 54 ksi
6

Given:

L 40 in

Solution: 1.

See Mathcad file P0561.

Draw a free-body diagram.


L a R2

M1 R1 P

2.

This is a statically indeterminate beam because there are three unknown reactions, R1, M1, and R2. To solve for these unknowns, follow the method presented in Example 4-7. First, calculate the moment of inertia and distance to the extreme fiber for the round section. I

d
64

I 0.2485 in

c 0.5 d

c 0.750 in

3.

From inspection of the FBD, write the load function equation q(x) = -M1<x>-2 + R1<x>-1 - R2<x - a>-1 + P<x - L>-1

4.

Integrate this equation from - to x to obtain shear, V(x) V(x) = -M1<x>-1 + R1<x>0 - R2<x - a>0 + P<x - L>0

5.

Integrate this equation from - to x to obtain moment, M(x) M(x) = -M1<x>0 + R1<x>1 - R2<x - a>1 + P<x - L>1

6.

Integrate the moment function, multiplying by 1/EI, to get the slope. (x) = [ -M1<x>1 + R1<x>2/2 - R2<x - a>2/2 + P<x - L>2/2 + C3]/EI

5.

Integrate again to get the deflection. y(x) = [-M1<x>2/2 + R1<x>3/6 - R2<x - a>3/6 + P<x - L>3/6 + C3x + C4]/EI

7.

Evaluate R1, M1, R2, C3 and C4 At x = 0, y = 0 and = 0, therefore, C3 = 0 and C4 = 0. At x = a, y = 0 At x = L+, V = M = 0 Guess Given M1 1000 in lbf y(a) = 0: V(L) = 0: M1 2 a
2

R1 500 lbf R1 6 a = 0 lbf in


3 3

R2 1000 lbf

R1 R2 P = 0 lbf

MACHINE DESIGN - An Integrated Approach, 4th Ed.

5-61-2

M(L) = 0:

M1 R1 L R2 ( L a ) = 0 lbf in

M1 R1 Find M1 R1 R2 R 2
8. 9. Define the range for x x 0 in 0.02 L L

M1 5000 in lbf

R1 750 lbf R2 1250 lbf

For a Mathcad solution, define a step function S. This function will have a value of zero when x is less than z, and a value of one when it is greater than or equal to z. S ( x z) if ( x z 1 0 )

10. Write the moment equation in Mathcad form, using the function S as a multiplying factor to get the effect of the singularity functions. M ( x) M1 R1 S ( x 0 in) x R2 S ( x a ) ( x a ) P S ( x L) ( x L) 11. Plot the moment equation and determine the maximum bending moment.

As expected, the maximum bending moment occurs under the support at x = a. Mmax M ( a ) Mmax 10.0 kip in
M ( x) kip in

MOMENT DIAGRAM
10

10

20 x in

30

40

12. Use equation 4.11b to calculate the maximum bending stress in the bar.

max

Mmax c I

max 30.2 ksi

13. There are no other stress components present (the transverse shear is zero at the extreme fiber) so this is a principal stress and the other two principal stresses are zero. Thus, this is a case of uniaxial stress. Determine the safety factor against static failure using equation 5.12a. N S ut N 1.8

max

MACHINE DESIGN - An Integrated Approach, 4th Ed.

5-62-1

PROBLEM 5-62
Statement:

_____

Figure P5-22 shows a pivot pin that is press-fit into part A and is slip fit in part B. If F = 100 lb, l = 2 in, and d = 0.5 in, what is the pin's safety factor against yielding when made of SAE 1020 cold-rolled steel? Applied force Total length, l Pin dia F 100 lbf l 2.00 in d 0.5 in Yield strength Beam length S y 57 ksi L 0.5 l L 1.000 in

Given:

Assumptions: 1. Since there is a slip fit between the pin and part B, part B offers no resistance to bending of the pin and, since the pin is press-fit into part A, it can be modeled as a cantilever beam of length l/2. 2. Part B distributes the concentrated force F so that, at the pin, it is uniformly distributed over the exposed length of the pin. Solution: 1. See Mathcad file P0562.

Calculate the intensity of the uniformly distributed load acting over the length of the pin. w F L w 100.0 lbf in

2.

A cantilever beam with uniform loading is shown in Figure B-1(b) in Appendix B. In this case, the dimension a the figure is zero. As shown in the figure, when a = 0, the maximum bending moment occurs at the support and is Mmax w L 2
2

Mmax 50.00 lbf in

3.

Calculate the moment of inertia and distance to the extreme fiber of the pin. The bending stress in the beam is then found using equation 4.11b. I

d
64

I 3.068 10 c 0.250 in

in

c 0.5 d

Mmax c I

4074 psi

4.

There are no other stress components present (the transverse shear is zero at the extreme fiber) so this is a principal stress and the other two principal stresses are zero. Thus, this is a case of uniaxial stress. Determine the safety factor against static failure using equations 5.7c and 5.8a, which reduce to N Sy N 14

MACHINE DESIGN - An Integrated Approach, 4th Ed.

5-63-1

PROBLEM 5-63
Statement:

_____

Figure P5-22 shows a pivot pin that is press-fit into part A and is slip fit in part B. If F = 100 N, l = 50 mm, and d = 16 mm, what is the pin's safety factor against yielding when made of class 50 cast iron? Applied force Total length, l Pin dia F 100 N l 50 mm d 16 mm Tensile strength Beam length S ut 359 MPa L 0.5 l L 25 mm

Given:

Assumptions: 1. Since there is a slip fit between the pin and part B, part B offers no resistance to bending of the pin and, since the pin is press-fit into part A, it can be modeled as a cantilever beam of length l/2. 2. Part B distributes the concentrated force F so that, at the pin, it is uniformly distributed over the exposed length of the pin. Solution: 1. See Mathcad file P0563.

Calculate the intensity of the uniformly distributed load acting over the length of the pin. w F L w 4.0 N mm

2.

A cantilever beam with uniform loading is shown in Figure B-1(b) in Appendix B. In this case, the dimension a the figure is zero. As shown in the figure, when a = 0, the maximum bending moment occurs at the support and is Mmax w L 2
2

Mmax 1250 N mm

3.

Calculate the moment of inertia and distance to the extreme fiber of the pin. The bending stress in the beam is then found using equation 4.11b. I

d
64

I 3.217 10 mm c 8.000 mm

c 0.5 d


4.

Mmax c I

3.108 MPa

There are no other stress components present (the transverse shear is zero at the extreme fiber) so this is a principal stress and the other two principal stresses are zero. Thus, this is a case of uniaxial stress. Determine the safety factor against static failure using equation 5.12a. N S ut N 115

MACHINE DESIGN - An Integrated Approach, 4th Ed.

5-64-1

PROBLEM 5-64
Statement: A differential element is subjected to the stresses (in ksi): x = 10, y = -20, xy = -20. The material is uneven and has strengths (in ksi) of S ut = 50, S y = 40, and S uc = 90. Calculate the safety factor and draw a a-b diagram showing the boundary for each theory with the stress state and load line using: (a) Coulomb-Mohr theory, and (b) Modified Mohr theory. Stress components Material properties Solution: 1.

Given:

x 10 ksi
S ut 50 ksi

y 20 ksi
S y 40 ksi

xy 20 ksi
S uc 90 ksi

See Figure 5-62 and Mathcad file P0564.

Calculate the nonzero principal stresses using equation 4.6a.

x y
2

2 x y 2 xy 2 2 x y 2 xy 2

a 20 ksi

b
2.

x y
2

b 30 ksi

Calculate the slope of load line. (The load line is the line from the origin through the stress point.) m

b a

m 1.5

3.

The safety factor equation for both theories is different for each quadrant the load line falls in. The equation for the modified Mohr factor of safety is different for each of the two regions in the 4th quadrant that the load line can fall in. In this case, the load line falls in the 4th quadrant, below the -1 slope line.. The factors of safety are: (a) Coulomb-Mohr theory Na S uc S ut S uc a S ut b Na 1.4

(b) Modified Mohr theory

Nb

S uc

S uc S ut S a b ut

Nb 2

4.

Plot the a-b diagram showing the safe-fail boundaries, the stress state point (20 ksi, -30 ksi) and the load line. Note that if a > b , then only that area on the graph that is to the right of and below the diagonal line can contain valid stress points. The factor of safety is the distance along the load line from the origin to the intersection of the load line with the failure boundary, divided by the distance from the origin to the stress poin Since the distance from the origin to the modified Mohr boundary is greater than the distance to the Coulomb-Mohr boundary, its factor of safety is greater. See Figure 5-62 on the following page.

MACHINE DESIGN - An Integrated Approach, 4th Ed.

5-64-2

b
50 40 30 MINIMUM NONZERO PRINCIPAL STRESS, KSI 20 10 0 -10 -20 (20,-30) -30 -40 -50 -60 -70 -80 -90 -S uc 0 10 20 30 40 50 Stress states at which failure will occur (b) Modified Mohr boundary -S ut (a) Coulomb-Mohr boundary

Load Line

-100 -100 -90 -80 -70 -60 -50 -40 -30 -20 -10

MAXIMUM PRINCIPAL STRESS, KSI

FIGURE 5-64
a - b Diagram for Problem 5-64

MACHINE DESIGN - An Integrated Approach, 4th Ed.

5-65-1

PROBLEM 5-65
Statement: A differential element is subjected to the stresses (in ksi): x = 10, y = -5, xy = 15. The material is uneven and has strengths (in ksi) of S ut = 50, S y = 40, and S uc = 90. Calculate the safety factor and draw a a-b diagram showing the boundary for each theory with the stress state and load line using: (a) Coulomb-Mohr theory, and (b) Modified Mohr theory. Stress components Material properties Solution: 1.

Given:

x 10 ksi
S ut 50 ksi

y 5 ksi
S y 40 ksi

xy 15 ksi
S uc 90 ksi

See Figure 5-65 and Mathcad file P0565.

Calculate the nonzero principal stresses using equation 4.6a.

x y
2

2 x y 2 xy 2 2 x y 2 xy 2

a 19.3 ksi

b
2.

x y
2

b 14.3 ksi

Calculate the slope of load line. (The load line is the line from the origin through the stress point.) m

b a

m 0.741

3.

The safety factor equation for both theories is different for each quadrant the load line falls in. The equation for the modified Mohr factor of safety is different for each of the two regions in the 4th quadrant that the load line can fall in. In this case, the load line falls in the 4th quadrant, above the -1 slope line.. The factors of safety are: (a) Coulomb-Mohr theory Na S uc S ut S uc a S ut b Na 1.8

(b) Modified Mohr theory

Nb

S ut

Nb 2.6

4.

Plot the a-b diagram showing the safe-fail boundaries, the stress state point (19.3 ksi, -14.3 ksi) and the load line. Note that if a > b , then only that area on the graph that is to the right of and below the diagonal line can contain valid stress points. The factor of safety is the distance along the load line from the origin to the intersection of the load line with the failure boundary, divided by the distance from the origin to the stress poin Since the distance from the origin to the modified Mohr boundary is greater than the distance to the Coulomb-Mohr boundary, its factor of safety is greater. See Figure 5-63 on the following page.

MACHINE DESIGN - An Integrated Approach, 4th Ed.

5-65-2

b
50 40 30 MINIMUM NONZERO PRINCIPAL STRESS, KSI 20 10 0 -10 -20 -30 -40 -50 -60 -70 -80 -90 -S uc 0 10 20 30 40 50 Stress states at which failure will occur (b) Modified Mohr boundary Load Line -S ut (19.3,-14.3) (a) Coulomb-Mohr boundary

-100 -100 -90 -80 -70 -60 -50 -40 -30 -20 -10

MAXIMUM PRINCIPAL STRESS, KSI

FIGURE 5-65
a - b Diagram for Problem 5-65

MACHINE DESIGN - An Integrated Approach, 4th Ed.

5-66-1

PROBLEM 5-66
Statement: A differential element is subjected to the stresses (in ksi): x = -20, y = -15, xy = 15. The material is uneven and has strengths (in ksi) of S ut = 50, S y = 40, and S uc = 90. Calculate the safety factor and draw a a-b diagram showing the boundary for each theory with the stress state and load line using: (a) Coulomb-Mohr theory, and (b) Modified Mohr theory. Stress components Material properties Solution: 1.

Given:

x 20 ksi
S ut 50 ksi

y 15 ksi
S y 40 ksi

xy 15 ksi
S uc 90 ksi

See Figure 5-66 and Mathcad file P0566.

Calculate the nonzero principal stresses using equation 4.6a.

x y
2

2 x y 2 xy 2 2 x y 2 xy 2

a 2.29 ksi

b
2.

x y
2

b 32.7 ksi

Calculate the slope of load line. (The load line is the line from the origin through the stress point.) m

b a

m 14.263

(third quadrant since both principal stresses are negative)

3.

The safety factor equation for both theories is the same when the load line falls in the third quadrant. The factors of safety are:

(a) Coulomb-Mohr theory

Na

S uc

Na 2.8

(b) Modified Mohr theory

Nb

S uc

Nb 2.8

4.

Plot the a-b diagram showing the safe-fail boundaries, the stress state point (-2.29 ksi, -32.7 ksi) and the load line. Note that if a > b , then only that area on the graph that is to the right of and below the diagonal line can contain valid stress points. The factor of safety is the distance along the load line from the origin to the intersection of the load line with the failure boundary, divided by the distance from the origin to the stress point. Since the distance from the origin to the modified Mohr boundary is the same as the distance to the Coulomb-Mohr boundary, its factor of safety is the same. See Figure 5-63 on the following page.

MACHINE DESIGN - An Integrated Approach, 4th Ed.

5-66-2

b
50 40 30 MINIMUM NONZERO PRINCIPAL STRESS, KSI 20 10 0 -10 -20 -30 (-2.29,-32.7) -40 -50 -60 -70 -80 -90 -S uc (b) Modified Mohr boundary Load Line 0 10 20 30 40 50 Stress state at which failure will occur for both theories -S ut (a) Coulomb-Mohr boundary

-100 -100 -90 -80 -70 -60 -50 -40 -30 -20 -10

MAXIMUM PRINCIPAL STRESS, KSI

FIGURE 5-66
a - b Diagram for Problem 5-66

MACHINE DESIGN - An Integrated Approach, 4th Ed.

5-67-1

PROBLEM 5-67
Statement: Solution: 1.

_____

Derive the von Mises effective stress equation 5.7d for the two-dimensional case. See Mathcad file P0567.

Start with equation 5.7c, which gives the von Mises stress in terms of the two nonzero principal stresses.

'
2.

1 1 3 3

(a)

Define 1 and 3 in terms of x, y, and xy using equations 4.6a.

x y
2

2 x y 2 xy 2 2

(b)

3
3.

x y
2

x y 2 xy 2

(c)

To make the manipulations easier, define:

c
4.

x y
2

and

2 x y 2 xy 2

(d)

Substitute equations d into b and c.

1 c R
5.

3 c R

(e)

Substitute equations e into a, expand, collect terms and simplify.

'

c R 2 c R c R c R 2
c 2 R c R c R c 2 R c R
2 2 2 2 2 2

'

(f)

'
6.

c 3 R

Substitute equations d into f, expand, collect terms and simplify to obtain the derived equation.

2 2 x y x y 2 ' 3 3 xy 2 2

'

x y x y 3 xy

(5.7d)

MACHINE DESIGN - An Integrated Approach, 4th Ed.

5-68-1

PROBLEM 5-68
Statement: Figure P5-23 shows an oil-field pump jack. The crank drive shaft at O2 is loaded in torsion and bending with maximum values of 6500 in-lb and 9800 in-lb, respectively. The point on the shaft with maximum stress is located away from the key that connects the shaft to the crank. Using a factor of safety of 2 against static yielding, determine a suitable diameter for the shaft if it is to be made of SAE 1040 cold-rolled steel. Yield strength S y 71 ksi Factor of safty Ns 2

Given:

Torque T 6500 in lbf Solution: 1.

Bending moment M 9800 in lbf

See Figure P5-23 and Mathcad file P0568.

Express the torsional and bending stresses as a functions of the unknown shaft diameter, d 32 M

Bending stress

x( d )

d
Torsional stress

xy( d )

16 T

2.

Use these two stresses in an expression for the von Mises effective stress, equation 5.7d withy = 0.

von Mises effective stress 3.

'( d )

x( d ) 3 xy( d )

Use equation 5.8a as a design relationship to solve for the diameter, d. Design equation

x( d ) 3 xy( d ) =
2

Sy Ns
2 2

32 M 3 16 T = S y 3 3 Ns d d
Solving for d
1

( 32 M ) 2 3 ( 16 T ) 2 d 2 Sy 2 Ns

d 1.480 in

A suitable diameter for the given design requirements is

d 1.500 in

MACHINE DESIGN - An Integrated Approach, 4th Ed.

5-69-1

PROBLEM 5-69
Statement: Figure P5-24a shows a C-clamp with an elliptical body dimensioned as shown. The clamp has a T-section with a uniform thickness of 3.2 mm at the throat as shown in Figure P5-24b. Find the static factor of safety if the clamping force is 2.7 kN and the material is class 40 gray cast iron. Clamping force F 2.7 kN Distance from center of screw to throat Section dimensions: t 3.2 mm Material properties Solution: 1. ri 63.5 mm Web h 31.8 mm S uc 965 MPa

Given:

Flange b 28.4 mm S ut 290 MPa

See Figure P4-26 and Mathcad file P0569.

Determine the location of the CG of the T-section and the distance from the centerline of the screw to the centroid of the section at the throat. yCG 0.5 t ( b t) 0.5 ( h t) ( h t) t b t ( h t) t yCG 9.578 mm rc 73.078 mm

rc ri yCG 2.

Using equation 4.12a and Figure 4-16, calculate the distance to the neutral axis, rn, and the distance from the centroidal axis to the neutral axis, e. Distance from the screw centerline to the outside fiber Cross section area A b t ( h t) t rn A r
ri t

ro ri h A 182.4 mm
2

ro 95.3 mm

Distance to neutral axis

t dr dr r r r t
i

ro

rn 71.864 mm

Distance from centroidal to neutral axis 3.

e rc rn

e 1.214 mm

Take a section through the throat area and draw a FBD. There will be a vertical axial force through the section CG (at a distance rc from the screw centerline) which will form a couple of magnitude rc x F. This couple will be balanced by an internal moment of equal magnitude. Internal moment M rc F M 197 N m

4.

Calculate the distances from the neutral axis to the inner and outer fibers. ci rn ri ci 8.364 mm co ro rn co 23.436 mm

5.

Using equations 4.12d and 4.12e, calculate the stresses at the inner and outer fibers of the throat section. Inner radius

ci F e A ri A
M

i 132.2 MPa

Outer radius

co F e A ro A
M

o 204.3 MPa

6.

These are the principal stresses so,

MACHINE DESIGN - An Integrated Approach, 4th Ed.


Inner radius Outer radius 7.

5-69-2

1i i 1o 0 MPa

2i 0 MPa 2o 0 MPa

3i 0 MPa 3o o

Calculate the factor of safety using equations 5.12c, 5.12d, and 5.12e. Inner radius 1 2 1 2 1 2

C1i

1i 2i

S uc 2 S ut S uc S uc 2 S ut S uc S uc 2 S ut S uc

1i 2i

C1i 92.46 MPa

C2i

2i 3i

2i 3i

C2i 0.00 MPa

C3i

3i 1i

3i 1i

C3i 92.46 MPa

eff max C1i C2i C3i 1i 2i 3i


Ni Outer radius 1 2 1 2 1 2 S ut Ni 2.2

eff 132.182 MPa

eff

C1o

1o 2o

S uc 2 S ut S uc S uc 2 S ut S uc S uc 2 S ut S uc

1o 2o

C1o 0.00 MPa

C2o

2o 3o

2o 3o

C2o 61.41 MPa

C3o

3o 1o

3o 1o

eff 61.41 MPa

C3o 61.41 MPa

eff max C1o C2o C3o 1o 2o 3o


No S ut No 4.7

eff

MACHINE DESIGN - An Integrated Approach, 4th Ed.

5-70-1

PROBLEM 5-70
Statement: Given: A C-clamp as shown in Figure P5-24a has a rectangular cross section as in Figure P5-24c. Find the static factor of safety if the clamping force is 1.6 kN and the material is class 50 gray cast iron. Clamping force F 1.6 kN Distance from center of screw to throat Section dimensions: Material properties Solution: 1. ri 63.5 mm Depth h 31.8 mm S uc 1131 MPa

Width b 6.4 mm S ut 359 MPa

See Figure P5-24 and Mathcad file P0570.

Determine the distance from the centerline of the screw to the centroid of the section at the throat. rc ri h 2 rc 79.4 mm

2.

Using equation 4.12a and Figure 4-16, calculate the distance to the neutral axis, rn, and the distance from the centroidal axis to the neutral axis, e. Distance from the screw centerline to the outside fiber Cross section area A b h rn A o b dr r r
i

ro ri h A 203.520 mm
2

ro 95.300 mm

Distance to neutral axis

rn 78.327 mm

Distance from centroidal to neutral axis 3.

e rc rn

e 1.073 mm

Take a section through the throat area and draw a FBD. There will be a vertical axial force through the section CG (at a distance rc from the screw centerline) which will form a couple of magnitude rc x F. This couple will be balanced by an internal moment of equal magnitude. Internal moment M rc F M 127 N m

4.

Calculate the distances from the neutral axis to the inner and outer fibers. ci rn ri ci 14.827 mm co ro rn co 16.973 mm

5.

Using equations 4.12d and 4.12e, calculate the stresses at the inner and outer fibers of the throat section.

ci F e A ri A
M

i 143.7 MPa

o
6.

co F e A ro A
M

o 95.8 MPa

These are the principal stresses so, Inner radius Outer radius

1i i 1o 0 MPa

2i 0 MPa 2o 0 MPa

3i 0 MPa 3o o

MACHINE DESIGN - An Integrated Approach, 4th Ed.

5-70-2

7.

Calculate the factor of safety using equations 5.12c, 5.12d, and 5.12e. Inner radius 1 2 1 2 1 2

C1i

1i 2i

S uc 2 S ut S uc S uc 2 S ut S uc S uc 2 S ut S uc

1i 2i

C1i 98.09 MPa

C2i

2i 3i

2i 3i

C2i 0.00 MPa

C3i

3i 1i

3i 1i

C3i 98.09 MPa

eff max C1i C2i C3i 1i 2i 3i


Ni S ut Ni 2.5

eff 143.707 MPa

eff

Outer radius 1 2 1 2 1 2

C1o

1o 2o

S uc 2 S ut S uc S uc 2 S ut S uc S uc 2 S ut S uc

1o 2o

C1o 0.00 MPa

C2o

2o 3o

2o 3o

C2o 30.39 MPa

C3o

3o 1o

3o 1o

C3o 30.39 MPa

eff max C1o C2o C3o 1o 2o 3o


No S ut No 11.8

eff 30.394 MPa

eff

MACHINE DESIGN - An Integrated Approach, 4th Ed.

5-71-1

PROBLEM 5-71
Statement: A C-clamp as shown in Figure P5-24a has an elliptical cross section as in Figure P5-24d. Dimensions of the major and minor axes of the ellipse are given. Find the static factor of safety if the clamping force is 1.6 kN and the material is class 60 gray cast iron. Clamping force F 1.6 kN Distance from center of screw to throat Section dimensions: Material properties Solution: 1. ri 63.5 mm Depth h 31.8 mm S uc 1289 MPa

Given:

Width b 9.6 mm S ut 427 MPa

See Figure P5-24 and Mathcad file P0571.

Determine the distance from the centerline of the screw to the centroid of the section at the throat. rc ri h 2 rc 79.4 mm

2.

Using equation 4.12a and Figure 4-16, calculate the distance to the neutral axis, rn, and the distance from the centroidal axis to the neutral axis, e. Distance from the screw centerline to the outside fiber Cross section area b h A 2 2 rn A o 0.5 r rc 2 2 b 1 4 2 h dr r r
i

ro ri h A 239.766 mm
2

ro 95.300 mm

Distance to neutral axis

rn 78.595 mm

Distance from centroidal to neutral axis 3.

e rc rn

e 0.805 mm

Take a section through the throat area and draw a FBD. There will be a vertical axial force through the section CG (at a distance rc from the screw centerline) which will form a couple of magnitude rc x F. This couple will be balanced by an internal moment of equal magnitude. Internal moment M rc F M 127 N m

4.

Calculate the distances from the neutral axis to the inner and outer fibers. ci rn ri ci 15.095 mm co ro rn co 16.705 mm

5.

Using equations 4.12d and 4.12e, calculate the stresses at the inner and outer fibers of the throat section.

ci F e A ri A
M

i 163.2 MPa

o
6.

co F e A ro A
M

o 108.7 MPa

These are the principal stresses so,

MACHINE DESIGN - An Integrated Approach, 4th Ed.


Inner radius Outer radius 7.

5-71-2

1i i 1o 0 MPa

2i 0 MPa 2o 0 MPa

3i 0 MPa 3o o

Calculate the factor of safety using equations 5.12c, 5.12d, and 5.12e. Inner radius 1 2 1 2 1 2

C1i

1i 2i

S uc 2 S ut S uc S uc 2 S ut S uc S uc 2 S ut S uc

1i 2i

C1i 109.12 MPa

C2i

2i 3i

2i 3i

C2i 0.00 MPa

C3i

3i 1i

3i 1i

C3i 109.12 MPa

eff max C1i C2i C3i 1i 2i 3i


Ni S ut Ni 2.6

eff 163.169 MPa

eff

Outer radius 1 2 1 2 1 2

C1o

1o 2o

S uc 2 S ut S uc S uc 2 S ut S uc S uc 2 S ut S uc

1o 2o

C1o 0.00 MPa

C2o

2o 3o

2o 3o

C2o 36.02 MPa

C3o

3o 1o

3o 1o

C3o 36.02 MPa

eff max C1o C2o C3o 1o 2o 3o


No S ut No 11.9

eff 36.016 MPa

eff

MACHINE DESIGN - An Integrated Approach, 4th Ed.

5-72-1

PROBLEM 5-72
Statement: Given: A C-clamp as shown in Figure P5-24a has a trapezoidal cross section as in Figure P5-24e. Find the static factor of safety if the clamping force is 350 lb and the material is class 40 gray cast iron. Clamping force F 1.6 kN Distance from center of screw to throat Section dimensions: Material properties Solution: 1. S ut 290 MPa ri 63.5 mm b o 3.2 mm S uc 965 MPa Depth h 31.8 mm

Width b i 9.6 mm

See Figure P5-24 and Mathcad file P0572.

Determine the distance from the centerline of the screw to the centroid of the section at the throat. rc ri h bi 2 bo 3 bi bo rc 76.75 mm

2.

Using equation 4.12a and Figure 4-16, calculate the distance to the neutral axis, rn, and the distance from the centroidal axis to the neutral axis, e. Distance from the screw centerline to the outside fiber Cross section area A bi bo 2 h A o bi bo r ri bi h dr r r
i

ro ri h A 203.520 mm
2

ro 95.300 mm

Distance to neutral axis

rn

rn 75.771 mm

Distance from centroidal to neutral axis 3.

e rc rn

e 0.979 mm

Take a section through the throat area and draw a FBD. There will be a vertical axial force through the section CG (at a distance rc from the screw centerline) which will form a couple of magnitude rc x F. This couple will be balanced by an internal moment of equal magnitude. Internal moment M rc F M 123 N m

4.

Calculate the distances from the neutral axis to the inner and outer fibers. ci rn ri ci 12.271 mm co ro rn co 19.529 mm

5.

Using equations 4.12d and 4.12e, calculate the stresses at the inner and outer fibers of the throat section.

ci F e A ri A
M

i 126.9 MPa

o
6.

co F e A ro A
M

o 118.4 MPa

These are the principal stresses so, Inner radius Outer radius

1i i 1o 0 MPa

2i 0 MPa 2o 0 MPa

3i 0 MPa 3o o

MACHINE DESIGN - An Integrated Approach, 4th Ed.


7. Calculate the factor of safety using equations 5.12c, 5.12d, and 5.12e. Inner radius 1 2 1 2 1 2

5-72-2

C1i

1i 2i

S uc 2 S ut S uc S uc 2 S ut S uc S uc 2 S ut S uc

1i 2i

C1i 88.77 MPa

C2i

2i 3i

2i 3i

C2i 0.00 MPa

C3i

3i 1i

3i 1i

C3i 88.77 MPa

eff max C1i C2i C3i 1i 2i 3i


Ni S ut Ni 2.3

eff 126.907 MPa

eff

Outer radius 1 2 1 2 1 2

C1o

1o 2o

S uc 2 S ut S uc S uc 2 S ut S uc S uc 2 S ut S uc

1o 2o

C1o 0.00 MPa

C2o

2o 3o

2o 3o

C2o 35.58 MPa

C3o

3o 1o

3o 1o

C3o 35.58 MPa

eff max C1o C2o C3o 1o 2o 3o


No S ut No 8.2

eff 35.577 MPa

eff

MACHINE DESIGN - An Integrated Approach, 4th Ed.

5-73-1

PROBLEM 5-73
Statement: The connecting rod (3) on the oil-field pump jack shown in Figure P5-23 is, in fact, made up of two rods, one connecting on each side of the walking beam (4). Determine a suitable width of 1/2-inch-thick SAE 1020 cold-rolled bar stock to use if the maximum tensile load on the bars is 3500 lb each. Use a factor of safety of 4 against static yielding. Yield strength Thickness Solution: 1. S y 57 ksi t 0.50 in Factor of safty Tensile force Ns 4 F 3500 lbf

Given:

See Figure P5-23 and Mathcad file P0573.

Express the tensile stress as a functions of the unknown width, w. Tensile stress

x( w)

F w t

2.

The tensile stress is the only stress present so it is also the von Mises effective stress. F w t

von Mises effective stress 3.

'( w)

Use equation 5.8a as a design relationship to solve for the diameter, d. Design equation F w t = Sy Ns

Solving for w

N s F t Sy

w 0.491 in

A suitable size for the given design requirements is

w 0.500 in

MACHINE DESIGN - An Integrated Approach, 4th Ed.

5-74-1

PROBLEM 5-74
Statement: A work platform is elevated on the end of a boom that has the ability to extend its length and vary its angle with respect to ground. The platform width is large compared to the boom diameter so that it is possible to load the boom eccentrically resulting in a combination of bending, torsion and direct compression in the base of the boom. At the base the boom is a hollow tube with an outside diameter of 8 in and a wall thickness of 0.75 in. It is made from SAE 1030 CR steel. Determine the factor of safety against static failure if the loading at a point at the base of the boom is: M = 600 kip-in, T = 76 kip-in, and an axial compression of 4800 lb. Yield strength SAE 1030 CR steel S y 64 ksi Boom dimensions Loading Solution: 1. See Mathcad file P0574. D 8.00 in M 600 kip in twall 0.75 in T 76 kip in F 4800 lbf

Given:

Calculate the bending stress at the point of interest. Inside diameter Moment of inertia Distance to outer fiber Bending stress d D 2 twall I d 6.500 in
4

64

D d

I 113.438 in c 4.000 in

c 0.5 D

bend

M c I

bend 21.157 ksi

2.

Calculate the axial stress due to the compressive load at the point of interest. Cross-section area Axial stress A

D d F A

A 17.082 in

axial

axial 0.281 ksi

3.

Combine the bending and axial stresses to get the maximum normal stress on the compressive side of the boom. Max. normal stress

x bend axial

x 21.438 ksi

4.

Calculate the torsional stress at the point of interest. Polar moment Torsional stress J 2 I J 226.876 in J
4

xy

Tc

xy 1.34 ksi

5.

Calculate the von Mises effective stress using equation 5.7d. von Mises stress

'

x 3 xy

' 21.563 ksi

6.

Calculate the factor of safety using equation 5.8a. Factor of safety N Sy

'

N 2.97

MACHINE DESIGN - An Integrated Approach, 4th Ed.

5-75-1

PROBLEM 5-75
Statement: Given: Repeat Problem 5-74 for a boom that is made from class 20 gray cast iron. At the base the boom is hollow tube with an outside diameter of 10 in and a wall thickness of 1.00 in. Strength Class 20 gray cast iron Boom dimensions Loading Solution: 1. See Mathcad file P0575. S ut 22 ksi D 10.00 in M 600 kip in S uc 83 ksi twall 1.00 in T 76 kip in F 4800 lbf

Calculate the bending stress at the point of interest. Inside diameter Moment of inertia Distance to outer fiber Bending stress d D 2 twall I d 8.000 in
4

64

D d

I 289.812 in c 5.000 in

c 0.5 D

bend

M c I

bend 10.352 ksi

2.

Calculate the axial stress due to the compressive load at the point of interest. Cross-section area Axial stress A

D d F A

A 28.274 in

axial

axial 0.17 ksi

3.

Combine the bending and axial stresses to get the maximum normal stress on the tensile and compressive sides of the boom. Max compressive Max tensile

xc bend axial xt bend axial

xc 10.521 ksi xt 10.182 ksi

4.

Calculate the torsional stress at the point of interest. Polar moment Torsional stress J 2 I J 579.624 in J
4

xy

Tc

xy 0.656 ksi

5.

Calculate the principal stresses on the tensile and compressive sides of the boom.

Compressive side

xc 2 maxc xy 2
1c xc
2 maxc

maxc 5.301 ksi

1c 0.041 ksi

2c 0 ksi 3c xc
2 maxc

3c 10.562 ksi

MACHINE DESIGN - An Integrated Approach, 4th Ed.

5-75-2

Tensile side

xt 2 maxt xy 2
1t xt
2 maxc

maxt 5.133 ksi

1t 10.392 ksi

2t 0 ksi 3t
6.

xt
2

maxc

3t 0.210 ksi

Calculate the factor of safety using equations 5.12c, 5.12d, and 5.12e. Compressive side 1 2 1 2 1 2

C1c

1c 2c

S uc 2 S ut S uc S uc 2 S ut S uc S uc 2 S ut S uc

1c 2c


eff 2.829 ksi

C1c 0.03 ksi

C2c

2c 3c

2c 3c

C2c 2.80 ksi

C3c

3c 1c

3c 1c

C3c 2.83 ksi

eff max C1c C2c C3c 1c 2c 3c


Nc Tensile side C1t 1 2 1 2 1 2 1t 2t S ut Nc 7.8

eff

S uc 2 S ut S uc S uc 2 S ut S uc S uc 2 S ut S uc

1t 2t

C1t 7.64 ksi

C2t

2t 3t

2t 3t

C2t 0.06 ksi

C3t

3t 1t

3t 1t

eff 10.392 ksi

C3t 7.69 ksi

eff max C1t C2t C3t 1t 2t 3t


Nt S ut Nt 2.1

eff

MACHINE DESIGN - An Integrated Approach, 4th Ed.

5-76-1

PROBLEM 5-76
Statement: Assume that the curved beam of Problem 5-70 has a crack on its inside surface of half-width a = 1.5 mm and a fracture toughness of 35 MPa-m0.5. What is its safety factor against sudden fracture? Width of section Half crack length Solution: 1. t 31.8 mm a 1.5 mm Fracture toughness Kc 35 MPa m

Given:

See Figure 5-38 and Mathcad file P0538.

From Problem 5-70, the nominal stress at the inside radius is: Nominal inside stress i 143.7 MPa Calculate the half-width of the beam. b 0.5 t b 15.9 mm

2. 3.

Calculate the geometry and stress intensity factors. a sec 1.006 2 b K i a K 9.92 MPa m NFM Kc K NFM 3.5

4.

Determine the factor of safety against sudden fracture failure

MACHINE DESIGN - An Integrated Approach, 4th Ed.

5-77-1

PROBLEM 5-77
Statement:

_____

A large aircraft panel is to be made from 7075-T651 aluminum bar. From test data it is found that the nominal tensile stress in the panel is 200 MPa. What is the average maximum central crack size that can be tolerated without catastrophic failure? Fracture toughness Nominal stress Kc 22 ksi in
0.5

Given:

Kc 24.2 MPa m

0.5

nom 200 MPa

Solution: 1.

See Mathcad file P0577.

Determine the value of the geometry factor from the discussion in Section 5.3 for a large plate with a central crack.

1
2. Using equation 5.14b, calculate the critical crack length for this material under the given stress condition.
2 Kc nom

a 4.65 mm

lcritical 2 a

lcritical 9.3 mm

MACHINE DESIGN - An Integrated Approach, 4th Ed.

5-78-1

PROBLEM 5-78
Statement: Design the connecting rod (link 3) of Problem 3-50 for a safety factor of 4 if the link is made from SAE 1010 hot-rolled steel sheet, the pin hole diameter at each end is 6 mm, and the maximum applied tensile load is 2000 N. There are two links carrying the load. Force on links Yield strength Design safety factor Pin hole diameter Ftotal 2000 N S y 179 MPa Nd 4 d 6 mm w 3 d w 18 mm

Given:

Assumptions: Choose a suitable width, say Solution:

See Figure P3-22 and Mathcad file P0577. F Ftotal 2 F 1000 N

1. The force on each link is

2. With only a tensile force acting on the link, the tensile stress will be the principal stress and it will also be the von Mises effective stress, so we have x = 1 = '. 3. The tensile stress on each link is

x =

F A Sy

F t w

= '

4. Using the distortion-energy failure theory,

Nd =

'

t w Sy F t 1.241 mm t 2 mm

5. Solving for the thickness,t,

F Nd w S y

6. Round this up to the next higher integer value, 7. The realized factor of safety against tensile failure is, 8. N t w S y F

N 6.4

Check the factor of safety against bearing failure in the pin holes. Bearing area Abear w t Abear 36.0 mm
2

bear
This is the principal stresses 1. 9.

F Abear

bear 27.8 MPa

Calculate the safety factor for direct bearing from equation 5.8c where 2 and 3 are both zero. Pin hole Nbear Sy Nbear 6.4

bear
2

10. The tearout area is

Atear = 2 t R ( 0.5 d ) , where R 0.5 w (see figure below). Substitute this area in

equation 4.9 for the shear area and solve for the shear strength xy.

MACHINE DESIGN - An Integrated Approach, 4th Ed.


Tearout length

5-78-2

Shear area

Atear 2 t R ( 0.5 d ) Atear 33.941 mm


2

Shear stress

xy

F Atear

xy 29.46 MPa
d R

8.

From equations 5.8c and 5.9b, the safety factor against tearout failure in the holes is 0.577 S y

Ntear

xy

Ntear 3.5

This is slightly less than the design FS of Nd 4 so, choose t = 2.5 mm or increase w to 4*d.

MACHINE DESIGN - An Integrated Approach, 4th Ed.

5-79-1

PROBLEM 5-79
Statement: Design the compacting ram (link 4) of Problem 3-50 for a safety factor of 4 if the ram is made from SAE 1010 hot-rolled steel bar, the pin hole diameter at the joint where link 3 attaches is 6 mm, and the applied load Fcom = 2000 N. The piston has a diameter of 35 mm. Force at point P Yield strength Design safety factor Pin hole diameter Fcom 2000 N S y 179 MPa Nd 4 d 6 mm

Given:

Assumptions: The points of maximum stress on a plane through point D are sufficiently removed from point D that there is no stress concentration at those points. Solution: See Figure P3-22 and Mathcad file P0579.

1. From Problem 3-51 the forces and reactions on the ram are: F34x 553 N F14E 357 N F34y 2000 N F14F 196 N

42.5

F14E F34x

E D
120.0

2. The maximum bending moment is at point D and is: M 42.5 mm F14E M 15172.5 N mm

F34y 77.5 F 14F F P Fcom


Compacting Ram (4)

The section modulus and area for the ram are Z ( D)

D
32

A ( D)

D
4

3. Between points D and P there is a compressive force of Fcom 2000 N. Thus, there is a compressive stress due to this force in addition to the bending stress at point D. On the left side of the ram at the section through point D

bL ( D)

M Z ( D)

a( D)

Fcom A ( D)

L ( D) bL ( D) a( D)

On the right side of the ram at the section through point D

bR( D)

M Z ( D)

a( D)

4 Fcom

R( D) bR( D) a( D)

The compressive stress on the right side will be numerically greater than that on the left side. 4. Since the shear stress due to bending is zero at these points, the axial stress will be the principal stress and it will also be the von Mises effective stress, so we have x = 1 = '. 5. Using the distortion-energy failure theory, 6. Solving for the diameter, D, Sy Sy

Nd =

'

R( D)

MACHINE DESIGN - An Integrated Approach, 4th Ed.


Guess D 10 mm D root( f ( D) D) D 16.368 mm D 18 mm N 5.2

5-79-2

f ( D) Nd R( D) S y

7. Round this up to the next higher even integer value, say 8. The realized factor of safety against axial yeilding is, N Sy

R( D)

9. The axial stress on each side of the ram on a section through D is:

L ( D) 18.6 MPa

R( D) 34.4 MPa

MACHINE DESIGN - An Integrated Approach, 4th Ed.

5-80-1

PROBLEM 5-80
Statement: A differential element is subected to the stresses given below and a ductile material has the strengths given below. Calculate the safety factor and draw 1-3 diagrams of each theory showing the stress state using: (a) Maximum shear-stress theory, and (b) Distortion-energy theory. Principal stresses Material properties Solution:

Given:

1 70 MPa

2 0 ksi

3 140 MPa
S uc 350 MPa

S ut 350 MPa S y 280 MPa

See Figure 5-80 and Mathcad file P0580.

1. Calculate the slope of load line. (The load line is the line from the origin through the stress point.) m

3 1

m 2

2. The safety factor equation for the distortion-enrgy theory is the same regardless of which quadrant the load line falls in. However, the equation for the maximum shear-stress factor of safety is different for each of the three quadrants that the load line (1st, 3rd, or 4th) can fall in. In this case, the load line falls in the 4th quadrant. The factors of safety are: (a) Maximum shear-stress theory Na Sy Na 1.3

1 3
3
(a) Maximum shear stress boundary

(b) Distortion energy theory


280

'

1 1 3 3

2
210

' 26.9 ksi


Nb Sy Nb 1.5
MINIMUM NONZERO PRINCIPAL STRESS, MPa

(b) Distortion energy boundary 140

70

'

3. Plot the 1-3 diagram showing the safe-fail boundaries, the stress state point (70MPa, -140 MPa) and the load line. Note that if 1 > 3 , then only that area on the graph that is to the right of and below the diagonal line can contain valid stress points. The factor of safety is the distance along the load line from the origin to the intersection of the load line with the failure boundary, divided by the distance from the origin to the stress point. Since the distance from the origin to the distortion-energy boundary is greater than the distance to the maximum shear-stress baoundary, its factor of safety is greater.

0 sy -70 (70,-140) -140

-210

-280

-s y

Stress states at which failure will occur Load Line

-350

-420 -280

-210

-140

-70

70

140

210

280

350

MAXIMUM PRINCIPAL STRESS, MPa

FIGURE 5-80
1 - 3 Diagram for Problem 5-80

MACHINE DESIGN - An Integrated Approach, 4th Ed.

5-81-1

PROBLEM 5-81
Statement: Given: A part has the combined stress state and strengths given below. Using the Distortion-Energy failure theory, find the von Mises effective stress and factor of safety against static failure. Stresses: x 70 MPa Strengths: Solution: 1.

y 35 MPa

xy 31.5 MPa

S y 126 MPa

S ut 140 MPa S uc 140 MPa

See Mathcad file P0581.

Find the maximum shear stress and principal stresses that result from this combination of applied stresses using equations 4.6.
2 x y 2 max xy 2

Maximum shear stress

max 36.0 MPa

Principal stresses

1 2

x y
2

max max

1 88.5 MPa 2 16.5 MPa

x y
2

3 0 psi
2. Find the von Mises effective stress using equation 5.7d:

'

x x y y 3 xy

' 81.6 MPa


Sy

3.

The safety factor can now be found using equation 5.8a.

'

N 1.5

MACHINE DESIGN - An Integrated Approach, 4th Ed.

5-82-1

PROBLEM 5-82
Statement: Given: Repeat Problem 5-78 for the connecting rod made from class 20 cast iron. Force on links Strength Design safety factor Pin hole diameter Ftotal 2000 N S ut 152 MPa Nd 4 d 6 mm w 4 d w 24 mm S uc 572 MPa

Assumptions: Choose a suitable width, say Solution:

See Figure P3-22 and Mathcad file P0582. F Ftotal 2 F 1000 N

1. The force on each link is

2. With only a tensile force acting on the link, the tensile stress will be the principal stress so we have x = 1.

3. The tensile stress on each link is

x =

F A

F t w

= 1

4. Using the modified-Mohr failure theory,

Nd =

S ut

t w S ut F t 1.096 mm t 2 mm

5. Solving for the thickness,t,

F Nd w S ut

6. Round this up to the next higher integer value, 7. The realized factor of safety against tensile failure is, 8. N t w S ut F

N 7.3

Check the factor of safety against bearing failure in the pin holes. Bearing area Abear w t Abear 48.0 mm
2

bear
This is the principal stresses 1. 9.

F Abear

bear 20.8 MPa

Calculate the safety factor for direct bearing from equation 5.8c where 2 and 3 are both zero. Pin hole Nbear
2

S uc

bear
2

Nbear 27.5

10. The tearout area is

Atear = 2 t R ( 0.5 d ) , where R 0.5 w (see figure below). Substitute this area in

equation 4.9 for the shear area and solve for the shear strength xy.

MACHINE DESIGN - An Integrated Approach, 4th Ed.

5-82-2

Tearout length

Shear area

Atear 2 t R ( 0.5 d ) Atear 46.476 mm


2

Shear stress

xy

F Atear

xy 21.52 MPa
Principal stress 8.

1 xy

For pure shear the Mohr circle is centered at 0,0 and has a radius equal to the shear stress. This results in 1 = . Using the Modified-Mohr failure theory and Figure 5-11, we see that we can use equation 5.12a for the safety factor against tearout. Ntear S ut Ntear 7.1

MACHINE DESIGN - An Integrated Approach, 4th Ed.

5-83-1

PROBLEM 5-83
Statement: Given: Repeat Problem 5-79 for the part made from class 20 cast iron. Force at point P Tensile strength Design safety factor Pin hole diameter Fcom 2000 N S ut 152 MPa Nd 4 d 6 mm

Assumptions: The points of maximum stress on a plane through point D are sufficiently removed from point D that there is no stress concentration at those points. Solution: See Figure P3-22 and Mathcad files P0579 and P0583.

1. From Problem 3-51 the forces and reactions on the ram are: F34x 553 N F14E 357 N F34y 2000 N F14F 196 N

42.5

F14E F34x

E D
120.0

2. The maximum bending moment is at point D and is: M 42.5 mm F14E M 15172.5 N mm

F34y 77.5 F 14F F P Fcom


Compacting Ram (4)

The section modulus and area for the ram are Z ( D)

D
32

A ( D)

D
4

3. Between points D and P there is a compressive force of Fcom 2000 N. Thus, there is a compressive stress due to this force in addition to the bending stress at point D. On the left side of the ram at the section through point D

bL ( D)

M Z ( D)

a( D)

Fcom A ( D)

L ( D) bL ( D) a( D)

On the right side of the ram at the section through point D

bR( D)

M Z ( D)

a( D)

4 Fcom

R( D) bR( D) a( D)

The tensile stress on the left side will be critical for an uneven, brittle material. 4. With only a tensile stress acting on the ram at this point, it will be the principal stress so we have L = 1. 5. Using the modified-Mohr failure theory, 6. Solving for the diameter, D, Guess D 10 mm D root( f ( D) D) D 14.567 mm Nd = S ut = S ut

f ( D) Nd L ( D) S ut

MACHINE DESIGN - An Integrated Approach, 4th Ed.

5-83-2

7. Round this up so that 8. The realized factor of safety against axial failure is,

D 3 d N S ut

D 18 mm N 8.2

L ( D)

9. The axial stress on each side of the ram on a section through D is:

L ( D) 18.6 MPa

R( D) 34.4 MPa

MACHINE DESIGN - An Integrated Approach, 4th Ed.

5-84-1

PROBLEM 5-84
Statement: A differential element is subected to the stresses and strengths given below. Calculate the safety factor and draw 1-3 diagrams of each theory showing the stress state using: (a) Coulomb-Mohr theory, and (b) Modified Mohr theory. Principal stresses Material properties Solution:

Given:

1 70 MPa

2 0 MPa

3 140 MPa

S ut 350 MPa S uc 630 MPa

See Figure 5-84 and Mathcad file P05384.

1. Calculate the slope of load line. (The load line is the line from the origin through the stress point.) m

3 1

m 2

2. The safety factor equation for both theories is different for each quadrant the load line falls in. The equation for the modified Mohr factor of safety is different for each of the two regions in the 4th quadrant that the load line can fall in. In this case, the load line falls in the 4th quadrant, below the -1 slope line.. The factors of safety are: (a)Coulomb-Mohr theory Na S uc S ut S uc 1 S ut 3 Na 2.4

(b) Modified Mohr theory Nb S uc

350 280 210 140 MINIMUM NONZERO PRINCIPAL STRESS, MPa 70 0 -70 (a) Coulomb-Mohr boundary

S uc S ut S 1 3 ut

Nb 3.2 3. Plot the 1-3 diagram showing the safe-fail boundaries, the stress state point (70 MPa,-140 MPa) and the load line. Note that if 1 > 3 , then only that area on the graph that is to the right of and below the diagonal line can contain valid stress points. The factor of safety is the distance along the load line from the origin to the intersection of the load line with the failure boundary, divided by the distance from the origin to the stress point. Since the distance from the origin to the modified Mohr boundary is greater than the distance to the Coulomb-Mohr boundary, its factor of safety is greater.

1
(70,-140)

-140 -210 -280 -350 -420 -490 -560 -630 -S uc 0 70 140 210 280 350 Stress states at which failure will occur (b) Modified Mohr boundary Load Line -S ut

-700 -700 -630 -560 -490 -420 -350 -280 -210 -140 -70

MAXIMUM PRINCIPAL STRESS, MPa

FIGURE 5-84
1 - 3 Diagram for Problem 5-84

MACHINE DESIGN - An Integrated Approach, 4th Ed.

5-85-1

PROBLEM 5-85
Statement: Given: A part has the combined stress state and strengths given below. Using the Modified-Mohr failure theory, find the effective stress and factor of safety against static failure. Stresses: x 70 MPa Strengths: Solution: 1. 2.

y 35 MPa

xy 31.5 MPa

S y 126 MPa

S ut 140 MPa S uc 560 MPa

See Mathcad file P0585.

Because S uc is greater than S ut, this is an uneven material, which is characteristic of a brittle material. Find the maximum shear stress and principal stresses that result from this combination of applied stresses using equations 4.6.
2 x y 2 xy 2

Maximum shear stress

max

max 36.0 MPa

Principal stresses

1 2

x y
2

max max

1 88.5 MPa 2 16.5 MPa

x y
2

3 0 psi
3. Find the Dowling factors C1, C2, C3 using equations 5.12b: C1 1 2 1 2 1 2 1 2

S uc 2 S ut S uc S uc 2 S ut S uc S uc 2 S ut S uc

1 2

C1 62.3 MPa

C2

2 3

2 3 3 1

C2 12.3 MPa

C3 4.

3 1

C3 66.4 MPa

Then find the largest of the six stresses C1, C2, C3 , 1, 2, 3:

C 1 C2 C 3 eff max 1 2 3
which is the modified-Mohr effective stress. 5.

eff 88.5 MPa

The safety factor can now be found using equation 5.12d.

S ut

eff

N 1.6

MACHINE DESIGN - An Integrated Approach, 4th Ed.

6-1a-1

PROBLEM 6-1a
Statement: Given: Solution: 1. For the data in row a in Table P6-1, find the stress range, alternating stress component, mean stress component, stress ratio, and amplitude ratio.

max 1000

min 0

See Mathcad file P0601a.

Use equations (6.1) to calculate the required quantities. Stress range

max min a max min


2

1000 a 500

Alternating stress

Mean stress

max min
2

m 500

Stress ratio

min max a m

R0

Amplitude ratio

A1

MACHINE DESIGN - An Integrated Approach, 4th Ed.

6-2a-1

PROBLEM 6-2a
Statement: Given: Solution: 1. For the strength data in row a in Table P6-2, calculate the uncorrected endurance limit and draw th strength-life (S-N) diagram for the material, assuming it to be steel. Tensile strength See Mathcad file P0602a. S ut 90 ksi

Using equation (6-5a), calculate the uncorrected endurance limit. S'e return 0.5 S ut if S ut 200 ksi 100 ksi otherwise S'e 45.0 ksi

2. 3. 4.

Using equation (6.9), calculate the fatigue strength at N = 10 3 cycles.

S m 0.9 S ut

S m 81.0 ksi S'f = a N


b

The equation for the S-N curve in the HCF region is given by equation (6.10a):

Determine the constants a and b from equations (6.10c) and (6.10a). From Table 6-5, for N = 10 6 , z 3.000 b 1 z log Sm

Sm S'e

b 0.0851

103

a 145.8 ksi

5.

To draw the S-N graph over the range 10 3 <= N <= 10 8, define a piecewise continuous function. S'f ( N ) return a N
b

if N 10

S'e otherwise 6. Plot the S-N curve over the range N 10 10 10


3 5 8

100

S' f ( N ) ksi

10 3 1 10

1 10

1 10

1 10 N

1 10

1 10

FIGURE 6-2a
S-N Diagram for Steel for Problem 6-2a

MACHINE DESIGN - An Integrated Approach,4th Ed.

6-3-1

PROBLEM 6-3
Statement: For the bicycle pedal-arm assembly in Figure P6-1 assume a rider-applied force that ranges from 0 1500 N at the pedal each cycle. Determine the fluctuating stresses in the 15-mm-dia pedal arm. Fi the fatigue safety factor if S ut = 500 MPa. Material yield strength Applied load Pedal arm diameter Solution: 1. S y 350 MPa Fmax 1500 N d 15 mm S ut 500 MPa Fmin 0 N

Given:

See Figures 6-3 and Mathcad file P0603.

From problem 4-3, the maximum principal stresses in the pedal arm due to Fmax are at point A and are

1max 793 MPa


2.

2max 0 MPa

3max 23 MPa

Using equation 5.7c, the maximum von Mises stress is

'max
3. 4.

1max 1max 3max 3max

'max 804.7 MPa

The minimum von Mises stress is zero.

'min 0 MPa

The alternating and mean components of the von Mises stress are:

'a 'm
5.

'max 'min
2

'a 402.4 MPa 'm 402.4 MPa


S'e 0.5 S ut S'e 250 MPa

'max 'min
2

Calculate the unmodified endurance limit.

a Frider b

C Mc Arm

Tc

A Arm

Section C B

Fc Pedal x

x y
FIGURE 6-3B
Points A and B at Section C

FIGURE 6-3A
Free Body Diagram for Problem 6-3

6.

Calculate the endurance limit modification factors for a nonrotating round beam. Load Cload 1

MACHINE DESIGN - An Integrated Approach,4th Ed.

6-3-2

Size

A95 0.010462 d d equiv A95 0.0766

A95 2.354 mm

d equiv 5.544 mm
0.097

d equiv Csize 1.189 mm


Surface A 4.51

Csize 1.007

Csize 1

b 0.265
b

(machined)

Csurf Temperature Reliability 7.

Sut A MPa

Csurf 0.869

Ctemp 1 Creliab 0.753 (R = 99.9%)

Calculate the modified endurance limit. S e Cload Csize Csurf Ctemp Creliab S'e S e 163.56 MPa

8.

Assuming a Case 3 load line, use equation (6.18e) to determine the factor of safety. Nf S e S ut Nf 0.31

'a S ut 'm S e

MACHINE DESIGN - An Integrated Approach

6-4a-1

PROBLEM 6-4a
Statement: For the strength data in row a in Table P6-2, calculate the uncorrected fatigue strength at 5E8 cycles and draw the strength-life (S-N) diagram for the material, assuming it to be an aluminum alloy. Tensile strength See Mathcad file P0604a. S ut 90 ksi

Given: Solution: 1.

Using equation (6-5c), calculate the uncorrected fatigue strength at 5E8 cycles. S'f5E8 return 0.4 S ut if S ut 48 ksi 19 ksi otherwise S'f5E8 19.0 ksi

2. 3. 4.

Using equation (6.9), calculate the fatigue strength at N = 10 3 cycles.

S m 0.9 S ut

S m 81.0 ksi S'f = a N


b

The equation for the S-N curve in the HCF region is given by equation (6.10a):

Determine the constants a and b from equations (6.10c) and (6.10a). From Table 6-5, for N = 5E8 , z 5.699 b 1 z log Sm

Sm S'f5E8

b 0.1105

103

a 173.772 ksi

5. 6.

To draw the S-N graph over the range 10 3 <= N <= 10 8, Plot the S-N curve over the range
3

S'f ( N ) a N
5 8

N 10 1.01 10 10

100

S' f ( N ) ksi

10 3 1 10

1 10

1 10

1 10 N

1 10

1 10

FIGURE 6-4a
S-N Diagram for Aluminum for Problem 6-4a

MACHINE DESIGN - An Integrated Approach, 4th Ed.

6-5a-1

PROBLEM 6-5a
Statement: Given: For the data in row a in Table P6-3, find the corrected endurance strength (or limit), create equations for the S-N line, and draw the S-N diagram. Material Tensile strength Shape Size (diameter) Solution: 1. See Mathcad file P0605a. steel S ut 110 ksi round d 2 in Surface finish Loading Temperature Reliability surface "ground" load "torsion" T 72 R 0.999

Using equation (6-5a), calculate the uncorrected endurance limit. S'e return 0.5 S ut if S ut 200 ksi 100 ksi otherwise S'e 55.0 ksi

2.

Calculate the endurance limit modification factors for a nonrotating round rod. Load Cload return 1 if load = "bending" return 1 if load = "torsion" return 0.7 if load = "axial" Size d equiv d Cload 1

d equiv Csize 0.869 in


Surface A

0.097

Csize 0.812 A 1.34

return 1.34 if surface = "ground" return 2.70 if surface = "machined" return 2.70 if surface = "cold_rolled" return 14.4 if surface = "hot_rolled" return 39.9 if surface = "forged"

return 0.085 if surface = "ground" return 0.265 if surface = "machined" return 0.265 if surface = "cold_rolled" return 0.718 if surface = "hot_rolled" return 0.995 if surface = "forged"

b 0.085

Csurf Temperature

S ut A ksi

Csurf 0.899 Ctemp 1

Ctemp

return 1 if T 840 1 0.0032 ( T 840 ) otherwise

MACHINE DESIGN - An Integrated Approach, 4th Ed.


Reliability Creliab return 1.000 if R = 0.50 return 0.897 if R = 0.90 return 0.814 if R = 0.99 return 0.753 if R = 0.999 return 0.702 if R = 0.9999 return 0.659 if R = 0.99999 3. Calculate the modified endurance limit. S e Cload Csize Csurf Ctemp Creliab S'e 4. Using equation (6.9), calculate the fatigue strength at N = 10 3 cycles. S m return 0.75 S ut if load = "axial" 0.9 S ut otherwise 5. 6. The equation for the S-N curve in the HCF region is given by equation (6.10a): Sf = a N
b

6-5a-2
Creliab 0.753

S e 30.2 ksi

S m 99.0 ksi

Determine the constants a and b from equations (6.10c) and (6.10a). From Table 6-5, for N = 10 6 , z 3.000 b 1 z log Sm

Sm Se

b 0.1717

103

a 324.120 ksi

7.

To draw the S-N graph over the range 10 3 <= N <= 10 8, define a piecewise continuous function. S f ( N ) return a N
b

if N 10

S e otherwise N 10 10 10
3 5 8

8.

Plot the S-N curve over the range

100

Sf ( N ) ksi

10 3 1 10

1 10

1 10

1 10 N

1 10

1 10

FIGURE 6-5a
S-N Diagram for Problem 6-5a

MACHINE DESIGN - An Integrated Approach, 4th Ed.

6-6-1

PROBLEM 6-6
Statement: For the trailer hitch from Problem 3-6 on p. 169 (also see Figures P6-2 and 1-5), find the infinite-lif fatigue safety factors for all modes of failure assuming that the horizontal impact force of the traile on the ball is fully reversed. Use steel with S ut = 600 MPa and S y = 450 MPa. Determine safety factors for: (a) The shank of the ball where it joins the ball bracket. (b) Bearing failure in the ball bracket hole. (c) Tearout failure in the ball bracket. (d) Tensile failure in the 19-mm diameter attachment holes. (e) Bending failure in the ball bracket as a cantilever. a 40 mm b 31 mm Mtongue 100 kg Fpull 55.1 kN S y 300 MPa S ut 600 MPa c 70 mm d sh 26 mm w 64 mm d 20 mm t 19 mm R 32 mm

Given:

Assumptions: 1. The nuts are just snug-tight (no pre-load), which is the worst case. 2. All reactions will be concentrated loads rather than distributed loads or pressures. Solution: See Figures 6-6 and Mathcad file P0606.
W tongue 70 = c

F pull

40 = a 2 A B A F b1 B F a1y 20 = d D F a2y Fa2x 2 F b2 C D Fd2 F c2y F a1x

19 = t 31 = b

Fc2x

FIGURE 6-6A
Dimensions and Free Body Diagram for Problem 6-6

1.

The dynamic loading in this problem is fully reversed so the mean stresses are zero and the alternating stresses are the same as those calculated in Problem 4-6. From Problem 4-6, the alternating components of the principal stresses in the shank of the ball where it joins the ball bracket are:

a1 1277 MPa
2.

a2 0 MPa

a3 0 MPa

Since 1 is the only nonzero principal stress, it is also the von Mises stress.

MACHINE DESIGN - An Integrated Approach, 4th Ed.

6-6-2

'a a1
3. 4. Calculate the unmodified endurance limit.

'a 1277 MPa


S'e 0.5 S ut S'e 300 MPa

Calculate the endurance limit modification factors for a nonrotating round beam. Load Size Cload 1 (bending load)
2

A95 0.010462 d sh d equiv A95 0.0766

A95 7.072 mm

d equiv 9.609 mm
0.097

Csize 1.189 A 4.51 Csurf A Temperature Reliability 5. Ctemp 1 Creliab 0.753

d equiv mm

Csize 0.955

Surface

b 0.265

(machined)

Sut MPa

Csurf 0.828

(R = 99.9%)

Calculate the modified endurance limit. S e Cload Csize Csurf Ctemp Creliab S'e Na Se S e 178.54 MPa Na 0.14

6.

Calculate the factor of safety for the ball shank.

'a

7.

From Problem 4-6, the alternating components of the principal stresses at the bearing area in the ball bracket ho are:

a1 111.5 MPa
8.

a2 0 MPa

a3 0 MPa

Since 1 is the only nonzero principal stress, it is also the von Mises stress.

'a a1
9.

'a 111.5 MPa

Calculate the endurance limit modification factors that are different from those in step 4. Load Size Cload 0.7 Csize 1 (axial load) (axial load)

10. Calculate the modified endurance limit. S e Cload Csize Csurf Ctemp Creliab S'e Nb Se S e 130.91 MPa Nb 1.17

11. Calculate the factor of safety for the bearing.

'a

MACHINE DESIGN - An Integrated Approach, 4th Ed.


12. From Problem 4-6, the alternating components of the von Mises stress at the tearout shear area in the ball bracket hole is:

6-6-3

'a 85.91 MPa


13. Calculate the endurance limit modification factors that are different from those in step 9. Load Size Cload 1 (simulated bending load)
2 2

A95 2 t ( 32 mm) 0.5 d sh A95 0.0766


0.097

A95 1111 mm

d equiv

d equiv 120.439 mm

d equiv Csize 1.189 mm


14. Calculate the modified endurance limit. S e Cload Csize Csurf Ctemp Creliab S'e

Csize 0.747

S e 139.71 MPa Nc Se Nc 1.6

15. Calculate the factor of safety against tearout.

'a

16. From Problem 4-6, the alternating components of the von Mises stress in the attachment bolts is:

'a 540.5 MPa


17. Calculate the endurance limit modification factors that are different from those in step 9. Load Size Cload 0.7 Csize 1 (axial load) (axial load)

18. Calculate the modified endurance limit. S e Cload Csize Csurf Ctemp Creliab S'e Nd Se S e 130.91 MPa Nd 0.24

19. Calculate the factor of safety against bolt tensile failure.

'a

20. From Problem 4-6, the alternating components of the principal stresses in the cantilever beam are:

a1 635.5 MPa

a2 0 MPa

a3 0 MPa

21. Since 1 is the only nonzero principal stress, it is also the von Mises stress.

'a a1

'a 635.5 MPa

22. Calculate the endurance limit modification factors that are different from those in step 17. Load Size Cload 1 A95 0.05 w t (bending load) A95 60.8 mm
2

MACHINE DESIGN - An Integrated Approach, 4th Ed.

6-6-4

d equiv

A95 0.0766
0.097

d equiv 28.173 mm

d equiv Csize 1.189 mm


23. Calculate the modified endurance limit. S e Cload Csize Csurf Ctemp Creliab S'e

Csize 0.86

S e 160.85 MPa Ne Se Ne 0.25

24. Calculate the factor of safety for the cantilever beam.

'a

MACHINE DESIGN - An Integrated Approach, 4th Ed.

6-7-1

PROBLEM 6-7
Statement: Given: Design the wrist pin of Problem 3-7 for infinite life with a safety factor of 1.5 if the 2500-g acceleration is fully reveresed and S ut = 130 ksi. Force on wrist pin Tensile strength Design safety factor Fwristpin 12.258 kN S ut 130 ksi Nd 1.5 od 0.375 in Fwristpin 2756 lbf

Assumptions: Choose a suitable outside diameter, say Solution: 1. 2.

See Figure 4-12 in the text and Mathcad file P0607. F Fwristpin 2 F 1378 lbf

The force at each shear plane is

With only the direct shear acting on the plane, the Mohr diagram will be a circle with center at the origin and radius equal to the shear stress. Thus, the principal normal stress is numerically equal to the shear stress, which in this case is also the principal shear stress, so we have = 1 = '. The shear stress at each shear plane is

3.

F A

od id
Se =

4 F
2

4. 5. 6.

For fully reversed loading the factor of safety is, Nd = Calculate the unmodified endurance limit.

2 2 od id S
2

= '

'

4 F S'e 65 ksi

S'e 0.5 S ut

Calculate the endurance limit modification factors for a nonrotating round pin (uniformly stressed). Load Size Cload 1 A95 ( id)

od id
4

0.097

d equiv( id)

A95 ( id) 0.0766

d equiv( id) Csize( id) 0.869 in


Surface A 1.34

b 0.085
b

(ground)

Csurf Temperature Reliability 7.

S ut A ksi

Csurf 0.886

Ctemp 1 Creliab 1.000 (R = 50%)

Calculate the modified endurance limit. S e( id) Cload Csize( id) Csurf Ctemp Creliab S'e

8.

Solving for the inside diameter, guess id 0.2 in Given Nd =

od id S e( id)
4 F

MACHINE DESIGN - An Integrated Approach, 4th Ed.

6-7-2

id Find ( id) 9.

id 0.299 in id 0.281 in

Round this down to the decimal equivalent of a common fraction (9/32),

10. The realized factor of safety is,

Nf

od id S e( id)
4 F

Nf 1.8

MACHINE DESIGN - An Integrated Approach, 4th Ed.

6-8-1

PROBLEM 6-8
Statement: A paper machine processes rolls of paper having a density of 984 kg/m3. The paper roll is 1.50-m OD x 0.22-m ID x 3.23 m long and is on a simply supported, hollow, steel shaft with S ut = 400 MPa Find the shaft ID needed to obtain a dynamic safety factor of 2 for a 10-year life if the shaft OD is 22 cm and the roll turns at 50 rpm. Paper roll: Density Outside dia. Inside dia. Length Shaft: Strength Outside dia. Design safety factor Design life Shaft speed
y

Given:

984

kg
3

w x R V L R

m OD 1500 mm ID 220 mm L 3230 mm S ut 400 MPa od 220 mm Nfd 2 Life 10 yr 50 rpm

R L/2 0 L x -R M

Assumptions: 1. The shaft is stiffer than the paper roll so the weight of the roll on the shaft can be modelled as a uniformly distributed load. 2. The bearings that support the shaft are close to x 0 the ends of the paper roll and are thin with respect L/2 L to the length of the roll so we can consider the distance between the shaft supports to be the FIGURE 6-8 same as the length of the roll. Load, Shear, and Moment Diagrams 3. There are no stress concentrations near the for Problem 6-8 point of maximum moment on the shaft. 4. The paper mill operates 3 shifts/day, 365 days/year. 5. The shaft is machined and the material reliabilty is 99.9%. See Figure 6-8 and Mathcad file P0608. Solution: 1. This is a case of fully reversed bending. The FBD for this loading case is shown in Appendix B, Figure B-2b, with the dimension a equal to 0. That is, the distributed load starts at the left support and ends at the right support. Calculate the number of stress cycles to see if we will design for finite or infinite life. Nlife Life Nlife 1.652 10
9

wL /8

2.

cycles

This is well beyond 10 6 cycles, so we will design for infinite life. 3. Determine the weight of the paper roll and the magnitude of the distributed load on the shaft. Roll volume Roll weight Distributed load on shaft 4.
2

OD ID L

V 5.585 10 mm W 53.895 kN w 16.686 N mm

W V g w W L

(a

Figure D-2b shows that the maximum bending moment occurs at the center of the shaft and is Mmax w L 8 Mmax 2.176 10 N mm
7

MACHINE DESIGN - An Integrated Approach, 4th Ed.


This is fully reversed bending so 5. Ma Mmax and Mm 0 N mm

6-8-2

The stress in the shaft at the point of maximum bending moment will depend upon the, as yet, unknown id. Tha is, Area moment of inertia I ( id)

64

od id

(b)

Alternating stress 6.

a( id)

Ma od 2 I ( id) (c)

Calculate the modified endurance strength of the shaft. Unmodified endurance limit Modification factors: Load Cload 1 Csize 1.189 S'e 0.5 S ut S'e 200 MPa

Size

mm
od

0.097

Csize 0.705
0.265

Surface

Csurf 4.51 Ctemp 1

Sut MPa

Csurf 0.922

Temperature Reliability

Creliab 0.753

Modified endurance limit S e Cload Csize Csurf Ctemp Creliab S'e 7. S e 97.8 MPa (d)

Use the factor of safety equation as a design equation to solve for the unknown id. For fully reversed bending, the factor of safety is Nf = Se (e)

a
1

Substituting equations b and c into e and solving for id,


4 4 32 Nfd Ma od id od S e

id 191.526 mm

Rounding this, let the shaft ID be This gives a wall thickness of

id 190 mm t 1 2 ( od id) t 15 mm

MACHINE DESIGN - An Integrated Approach, 4th Ed.

6-9-1

PROBLEM 6-9
Statement: For the Vise Grip plier-wrench is drawn to scale in Figure P6-3, and for which the forces were analyzed in Problem 3-9 and the stresses in Problem 4-9, find the safety factors for each pin for an assumed clamping force of P = 4000 N in the position shown. The pins are 8-mm dia, S y = 400 MP S ut = 520 MPa, and are all in double shear. Assume a desired finite life of 5E4 cycles. Pin stresses as calculated in Problem 4-9: Pin 1-2 12 74.6 MPa Pin 1-4 Pin 2-3 Pin 3-4 Yield strength Pin diameter Desired life S y 400 MPa d 8 mm Nlife 5 10
4

Given:

14 50.7 MPa 23 50.7 MPa 34 50.7 MPa

Tensile strength S ut 520 MPa

Assumptions: 1. Links 3 and 4 are in a toggle position, i.e., the pin that joins links 3 and 4 is in line with the pins that join 1 with 4 and 2 with 3. Solution: 1. See Figure 6-9 and Mathcad file P0609.

The FBDs of the assembly and each individual link are shown in Figure 6-9. The dimensions, as scaled from Figure P5-3 in the text, are shown on the link FBDs.
F 4 P 1

3 F
55.0 = b 50.0 = a 22.0 = d

2 P

F14

39.5 = c

129.2

4 F34 P

F41

F21

28.0 = e

2.8 = g

F43 3 F23 F

F12 F32 2

21.2 = h

26.9 = f

FIGURE 6-9 Free Body Diagrams for Problem 6-9 2. The pins are in pure shear, so the principal stresses are

MACHINE DESIGN - An Integrated Approach, 4th Ed.


Pin joining 1 and 2 All other pins 3.

6-9-2

'12 '14

3 12 3 14

'12 129.211 MPa '14 87.815 MPa

This is a case of repeated fatigue loading. The alternating and mean von Mises stress components are: Pin joining 1 and 2 All other pins

'12a 0.5 '12 '14a 0.5 '14


S'e 0.5 S ut

'12m '12a '14m '14a


S'e 260 MPa

4. 5.

Calculate the unmodified endurance limit.

Calculate the endurance limit modification factors for a non rotating round pin (uniformly stressed). Load Size Cload 1 A95

d
4

A95 50.265 mm A95

d equiv

0.0766
0.097

d equiv 25.617 mm

d equiv Csize 1.189 mm


Surface A 4.51

Csize 0.868

b 0.265
b

(machined)

Csurf Temperature Reliability 6.

Sut A MPa

Csurf 0.86

Ctemp 1 Creliab 1.000 (R = 50%)

Calculate the modified endurance limit. S e Cload Csize Csurf Ctemp Creliab S'e S e 194.07 MPa S m 0.9 S ut Sf = a N S m 468 MPa
b

7. 8. 9.

Using equation (6.9), calculate the fatigue strength at N = 10 3 cycles.

The equation for the S-N curve in the HCF region is given by equation (6.10a):

Determine the constants a and b from equations (6.10c) and (6.10a). From Table 6-5, for N = 10 6 , z 3.000 b 1 z log Sm

Sm S'e

b 0.0851

10
3

a 842.4 MPa

MACHINE DESIGN - An Integrated Approach, 4th Ed.

6-9-3

10. Calculate the corrected fatigue strength at Nlife 5 10 cycles. S f a Nlife

S f 335.49 MPa

11. Assuming a Case 3 load line, use equation (6.18e) to determine the factor of safety. Pin joining 1 and 2 Nf S f S ut Nf 3.2

'12a S ut '12m S f
S f S ut

All other pins

Nf

'14a S ut '14m S f

Nf 4.6

MACHINE DESIGN - An Integrated Approach, 4th Ed.

6-10-1

PROBLEM 6-10
Statement: An overhung diving board is shown in Figure P6-4a. A 100-kg person is standing on the free end. Assume cross-sectional dimensions of 305 mm x 32 mm. What is the fatigue safety factor for infinite life if the material is brittle fiberglass with S f = 39 MPa @ N = 5E8 cycles and S ut = 130 MP in the longitudinal direction?
2000 = L

Given:

Mass of person Board dimensions Load dimensions Material properties

M 100 kg w 305 mm t 32 mm b 700 mm L 2000 mm S ut 130 MPa S f5E8 39 MPa

R1

R2

700 = b

FIGURE 6-10
Free Body Diagram for Problem 6-10

Assumptions: 1. The given fatigue strength is fully corrected. 2. There are no stress-concentrations near the point of maximum moment on the diving board. Solution: 1. 2. See Figure 6-10 and Mathcad file P0610.

This is a case of repeated bending. The FBD for this loading case is shown in Appendix B, Figure B-3a, with th dimension a equal to L. That is, the concentrated force F is at the end of the overhung beam. Determine the weight of the person on the end of the board. Weight W M g W 980.7 N (a)

3.

Figure B-3a in Appendix B shows that the maximum bending moment occurs at the right-hand support and is Mmax W ( L b ) This is repeated bending so Ma Mmax 2 w t
3

Mmax 1.275 10 N mm

and

Mm Ma

4.

The stress in the board at the point of maximum bending moment is Area moment of inertia I I 8.329 10 mm
5 4

(b)

12 Ma t 2 I Mm t 2 I

Alternating stress

a m

a 12.2 MPa m 12.2 MPa

(c)

Mean stress 5.

(d)

For repeated (fluctuating) bending, the factor of safety for Case 3 loading is Nf S f5E8 S ut Nf 2.4 (e)

a S ut m S f5E8

MACHINE DESIGN - An Integrated Approach, 4th Ed.

6-11-1

PROBLEM 6-11
Statement: Repeat Problem 6-10 assuming the 100-kg person in Problem 6-10 jumps up 25 cm and lands back on the board. Assume the board weighs 29 kg and deflects 13.1 cm statically when the person stands on it. What is the fatigue safety factor for finite life if the material is brittle fiberglass with S = 39 MPa @ N = 5E8 cycles and S ut = 130 MPa in the longitudinal direction? Maximum principal stresses due to bending at R2 from Problem 4-11 1max 76.3 MPa
2000 = L R1 P

Given:

2max 0 MPa 3max 0 MPa


Ultimate strength Fatigue strength Fatigue life Solution: 1. S ut 130 MPa S f 39 MPa Ncycle 5 10
8
700 = b R2

FIGURE 6-11 See Figure 6-11 and Mathcad file P0611.


Free Body Diagram for Problem 6-11

The dynamic loading in this case is repeated, i.e., the stresses go from zero to the maximum values given above. Thus, the minimum and maximum von Mises stresses are:

'max

1max 1max 3max 3max

'max 76.3 MPa


2. The alternating and mean components of the von Mises stress are:

'min 0 MPa

'a 'm
3.

'max 'min
2

'a 38.1 MPa 'm 38.1 MPa

'max 'min
2

Assuming a Case 3 load line, use equation (6.18e) to calculate the factor of safety. S f S ut

Nf

'a S ut 'm S f

Nf 0.79

MACHINE DESIGN - An Integrated Approach, 4th Ed.

6-12-1

PROBLEM 6-12
Statement: Repeat Problem 6-10 using the cantilevered diving board design in Figure P6-4b.
2000

Given:

Mass of person Board dimensions Load dimensions Material properties

M 100 kg w 305 mm t 32 mm L 1300 mm S ut 130 MPa S f5E8 39 MPa


M1 R1

1300 = L P

Assumptions: 1. The given fatigue strength is fully corrected. 700 2. There are no stress-concentrations near the point of maximum moment on the diving board. FIGURE 6-12
Free Body Diagram for Problem 6-12

Solution: 1. 2.

See Figure 6-12 and Mathcad file P0612.

This is a case of repeated bending. The FBD for this loading case is shown in Appendix B, Figure B-1a, with the dimension a equal to L. That is, the concentrated force F is at the end of the cantilever beam. Determine the weight of the person on the end of the board. Weight W M g W 980.7 N (a)

3.

Figure B-1a in Appendix B shows that the maximum bending moment occurs at the support and is Mmax W L This is repeated bending so Ma Mmax 2 w t
3

Mmax 1.275 10 N mm

and

Mm Ma

4.

The stress in the board at the point of maximum bending moment is Area moment of inertia I I 8.329 10 mm
5 4

(b)

12 Ma t 2 I Mm t 2 I

Alternating stress

a m

a 12.2 MPa m 12.2 MPa

(c)

Mean stress 5.

(d)

For repeated (fluctuating) bending, the factor of safety for Case 3 loading is Nf S f5E8 S ut Nf 2.4 (e)

a S ut m S f5E8

MACHINE DESIGN - An Integrated Approach, 4th Ed.

6-13-1

PROBLEM 6-13
Statement: Repeat Problem 6-11 using the cantilevered diving board design in Figure P6-4b. Assume the board weighs 19 kg and deflects 8.5 cm statically when the person stands on it. Maximum principal stresses due to bending at support from Problem 4-13 1max 87.1 MPa
2000 1300 = L P

Given:

2max 0 MPa 3max 0 MPa


Ultimate strength Fatigue strength Fatigue life Solution: 1. S ut 130 MPa S f 39 MPa Ncycle 5 10
8

M1 700

R1

FIGURE 6-13 See Figure 6-13 and Mathcad file P0613.


Free Body Diagram for Problem 6-13

The dynamic loading in this case is repeated, i.e., the stresses go from zero to the maximum values given above. Thus, the minimum and maximum von Mises stresses are:

'max

1max 1max 3max 3max

'max 87.1 MPa


2. The alternating and mean components of the von Mises stress are:

'min 0 MPa

'a 'm
3.

'max 'min
2

'a 43.5 MPa 'm 43.5 MPa

'max 'min
2

Assuming a Case 3 load line, use equation (6.18e) to calculate the factor of safety. Nf S f S ut Nf 0.69

'a S ut 'm S f

MACHINE DESIGN - An Integrated Approach, 4th Ed.

6-14-1

PROBLEM 6-14
Statement: Figure P6-5 shows a child's toy called a pogo stick. The child stands on the pads, applying half her weight on each side. She jumps off the ground, holding the pads up against her feet, and bounces along with the spring cushioning the impact and storing energy to help each rebound. Design the aluminum cantilever beam sections on which she stands to survive jumping 2 in off the ground with a dynamic safety factor of 2 for a finite life of 5E4 cycles. Use 2000 series aluminum. Define and size the beam shape.

Given:

Heat treated 2024 aluminum: Tensile strength S ut 64 ksi Design safety factor Design life Nfd 2 Nlife 5 10
4

Assumptions: The beam will have a rectangular crosssection with the load applied at a distance of 5 in from the central support. L 5 in Solution: 1. See Figure 6-14 and Mathcad file P0614.

From Problem 3-14, the total dynamic force on both foot supports is Fimax 224 lbf Fimax 2 Fimin 2 Fimin 0 lbf

Fi /2

Fi /2

Therefore, the load on each support is Pmax Pmin 2. 3. Pmax 112 lbf Pmin 0 lbf

To give adequate support to the childs foot, let the width of the support beam be w 1.5 in From Figure B-1(a) in Appendix B, the maximum bending moment at x = 0 is Mmax Pmax L Mmax 560 in lbf

P
FIGURE 5-14
Free Body Diagram for Problem 5-14

Mmin 0 in lbf

4.

Calculate the alternating and mean components of the bending moment. Ma Mm Mmax Mmin 2 Mmax Mmin 2 Ma 280 in lbf Mm 280 in lbf S'e 19 ksi @ 5E8 cycles

5. 6.

Determine the unmodified endurance limit.

Calculate the endurance limit modification factors for a nonrotating rectangular beam. Load Size Cload 1 A95 ( t) 0.05 w t d equiv( t) A95 ( t) 0.0766

MACHINE DESIGN - An Integrated Approach, 4th Ed.

6-14-2
0.097

d equiv( t) Csize( t) 0.869 in


Surface A 2.7

b 0.265
b

(machined)

Csurf Temperature Reliability 7.

S ut A ksi

Csurf 0.897

Ctemp 1 Creliab 1.000 (R = 50%)

Calculate the modified endurance limit. S e( t) Cload Csize( t) Csurf Ctemp Creliab S'e

8. 9.

Using equation (6.9), calculate the fatigue strength at N = 10 3 cycles.

S m 0.9 S ut Sf = a N

S m 57.6 ksi
b

The equation for the S-N curve in the HCF region is given by equation (6.10a):

10. Determine the constants a and b from equations (6.10c) and (6.10a). From Table 6-5, for N = 5E8 , z 5.699 b ( t) 1 z log

Sm Se( t)

a ( t)

Sm

103

b( t )

11. Determine the corrected fatigue strength at Nlife 5 10 cycles.

S f ( t) a ( t) Nlife

b( t )

12. We can now determine the minimum required section depth, t. Using the distortion-energy failure theory with the modified Goodman diagram, the bending stress will also be the only nonzero principal stress, which will also be the von Mises stress. Assuming a Case 3 load line, use equation (6.18e) to determine the factor of safety. Guess t 10 mm. Bending stress Given Nfd =

M c I

t 12 6 M = M = 2 3 2 w t w t
2

w t 6

S f ( t) S ut Ma S ut Mm S f ( t) t 0.304 in t 0.375 in

t Find ( t)

Round this up to the next higher decimal equivalent of a common fraction, Using this value of t, the values of the functions of t are: Csize( t) 0.912 The realized safety factor is Nf w t 6
2

S e( t) 15.545 ksi

S f ( t) 38.981 ksi

S f ( t) S ut Ma S ut Mm S f ( t)

Nf 3.0

MACHINE DESIGN - An Integrated Approach, 4th Ed.

6-15a-1

PROBLEM 6-15a
Statement: For a notched part having a notch dimension r, geometric stress concentration factor Kt, and material strength S ut as shown in row a of Table P6-4, find the Neuber factor a, the material's notch sensitivity q, and the fatigue stress-concentration factor Kf. Ultimate tensile strength Geometric stress-concentration factor Loading "Bending" Solution: See Mathcad file P0615a.
1

Given:

S ut 100 ksi Kt 3.3

Notch radius

r 0.25 in

Material is steel

1.

From Table 6-6, the Neuber constant for S ut 100 ksi is

a 0.062 in q 1 1 a r

a 0.062 in q 0.890

2.

Using equation 6-13, the notch sensitivity is

3.

The fatigue stress-concentration factor, from equation 6.11b, is Kf 1 q Kt 1 Kf 3.05

MACHINE DESIGN - An Integrated Approach, 4th Ed.

6-16-1

PROBLEM 6-16
Statement: A track to guide bowling balls is designed with two round rods as shown in Figure P6-6. The rods are not parallel to one another but have a small angle between them. The balls roll on the rods until they fall between them and drop onto another track. The angle between the rods is varied to cause the ball to drop at different locations. Find the infinite-life safety factor for the 1-in dia SAE 1010 cold-rolled steel rods. (a) Assume rods are simply supported at each end. (b) Assume rods are fixed at each end. Tensile strength Rod diameter Solution: 1. S ut 53 ksi d 1.00 in
a Fball

Given:

See Figure 6-16 and Mathcad file P0616.


R1 L R2

The maximum bending stress will occur at the outer fibers of the rod at the section where the maximum bending moment occurs which, in this case, is at x = a. The only stress present on the top or bottom surface of the rod is the bending stress x. Therefore, on the bottom surface where the stress is tensile, sx is the principal stress 1 . Thus, from Problem 4-16, for a simply supported rod, Maximum principal stress

FIGURE 6-16A
Free Body Diagram for Problem 6-16(a), taken on a plane through the rod axis and ball center

1 748 psi

2.

The dynamic loading is repeated from 0 to 1 for each ball that rolls down the track. The alternating and mean components of the von Mises stress are: Alternating von Mises stress Mean von Mises stress

'a 0.5 1 'm 0.5 1


S'e 0.5 S ut

'a 374 psi 'm 374 psi


S'e 26.5 ksi

3. 4.

Calculate the unmodified endurance limit.

Calculate the endurance limit modification factors for a nonrotating round beam. Load Size Cload 1 A95 0.010462 d
2 0.097

d equiv

A95 0.0766

d equiv Csize 0.869 in


Surface A 2.70

Csize 0.957

b 0.265
b

(machined)

Csurf Temperature Reliability

S ut A ksi

Csurf 0.943

Ctemp 1 Creliab 0.659 (R = 99.999%)

MACHINE DESIGN - An Integrated Approach, 4th Ed.


5. Calculate the modified endurance limit. S e Cload Csize Csurf Ctemp Creliab S'e 6. S e 15.759 ksi

6-16-2

Assuming a Case 3 load line, use equation (6.18e) to determine the factor of safety. Case a Nfa S e S ut Nfa 32
a Fball

'a S ut 'm S e

7.

For the built-in case, the maximum bending stress will occur at the outer fibers of the rod at the section where the maximum bending moment occurs which, in this case, is at x = L. The only stress present on the top or bottom surface of the rod is the bending stress x. Therefore, on the bottom surface where the stress is tensile, sx is the principal stress 1 . Thus, from Problem 4-16, for a simply supported rod, Maximum principal stress

M1

R1

R 2 M2

FIGURE 6-16B
Free Body Diagram for Problem 6-16(b), taken on a plane through the rod axis and ball center

1 577 psi

8.

The dynamic loading is repeated from 0 to 1 for each ball that rolls down the track. The alternating and mean components of the von Mises stress are: Alternating von Mises stress Mean von Mises stress

'a 0.5 1 'm 0.5 1

'a 288.5 psi 'm 288.5 psi

9.

Assuming a Case 3 load line, use equation (6.18e) to determine the factor of safety. Case b Nfb S e S ut Nfb 42

'a S ut 'm S e

MACHINE DESIGN - An Integrated Approach, 4th Ed.

6-17-1

PROBLEM 6-17
Statement: A pair of ice tongs is shown in Figure P6-7. The ice weighs 50 lb and is 10 in wide across the tongs. The distance between the handles is 4 in, and the mean radius r of the tong is 6 in. The rectangular cross-sectional dimensions are 0.75 x 0.312 in. Find the safety factor for the tongs for 5E5 cycles if their S ut = 50 ksi. F Tensile strength Cross-section: Width Depth Life S ut 50 ksi w 0.312 in h 0.75 in Nf 5 10
5
11.0 = ax

Given:

C FC O
3.5 = cy

FO
2.0 = cx 12.0 = by 5.0 = bx

A
Assumptions: The tongs are forged. Use 99.99% reliability. Operating temperature is between 32F and 70F.

FB
Solution: See Problem 4-17, Figure 6-17, and Mathcad file P0617.

B W/2
FIGURE 6-17
Free Body Diagram for Problem 6-17

1. The maximum bending stress in the tong was found in Problem 4-17 at point A. Vertical direction

i 8.58 ksi

All other components are zero. 2. There are no other stress components present so

1max i
3.

1max 8.58 ksi

2max 0 ksi

3max 0 ksi

The dynamic loading in this case is repeated, thus

1min 0 ksi
4.

2min 0 ksi

3min 0 ksi

Even though this is a brittle material, for HCF analysis, determine the von Mises effective stresses. Since there i only one nonzero stress,

'max 1max 'min 1min 'a 'm


5. 6.

'max 8.58 ksi 'min 0 ksi 'a 4.29 ksi 'm 4.29 ksi
S'e 0.5 S ut S'e 25 ksi

'max 'min
2

'max 'min
2

Calculate the unmodified endurance limit.

Calculate the endurance limit modification factors for a nonrotating rectangular beam. Load Size Cload 1 A95 0.05 w h d equiv A95 0.0766 A95 7.548 mm
2

d equiv 9.927 mm

MACHINE DESIGN - An Integrated Approach, 4th Ed.

6-17-2
0.097

Csize 1.189 A 39.9 Csurf A Temperature Reliability 7. Ctemp 1 Creliab 0.702

d equiv mm

Csize 0.952

Surface

b 0.995

(forged)

S ut ksi

Csurf 0.814

(R = 99.99%)

Calculate the modified endurance limit. S e Cload Csize Csurf Ctemp Creliab S'e S e 13.59 ksi S m 0.9 S ut Sf = a N S m 45 ksi
b

8. 9.

Using equation (6.9), calculate the fatigue strength at N = 10 3 cycles.

The equation for the S-N curve in the HCF region is given by equation (6.10a):

10. Determine the constants a and b from equations (6.10c) and (6.10a). From Table 6-5, for N = 10 6 , z 3.000 b 1 z log Sm

Sm Se

b 0.1733

103

a 148.991 ksi

11. Using equation (6.10a), determine the fatigue strength.

S f5E5 a Nf

S f5E5 15.326 ksi

12. Assuming a Case 3 load line, use equation (6.18e) to calculate the factor of safety. S f5E5 S ut

Nf5E5

'a S ut 'm S f5E5

Nf5E5 2.7

MACHINE DESIGN - An Integrated Approach, 4th Ed.

6-18-1

PROBLEM 6-18
Statement: A pair of ice tongs is shown in Figure P6-7. The ice weighs 50 lb and is 10 in wide across the tongs. The distance between the handles is 4 in, and the mean radius r of the tong is 6 in. The rectangular cross-sectional dimensions are 0.75 x 0.312 in. Find the safety factor for the tongs for 5E5 cycles if they are made of Class 40 gray cast iron. Tensile strength Cross-section: Width Depth Life S ut 42 ksi w 0.312 in h 0.75 in Nf 5 10
5
11.0 = ax

Given:

F C FC O
3.5 = cy

FO
2.0 = cx 12.0 = by 5.0 = bx

Assumptions: The tongs are as-cast. Use 99.99% reliability. Operating temperature is between 32F and 70F. Set Csurf to 1 for a cast finish, which does not need a surface factor. See Problem 4-18, Figure 6-18, and Solution: Mathcad file P0618. 1. The maximum bending stress in the tong was found in Problem 4-17 at point A. Vertical direction All other components are zero. 2. There are no other stress components present so

FB B W/2
FIGURE 6-18
Free Body Diagram for Problem 6-18

i 8.58 ksi

1max i
3.

1max 8.58 ksi

2max 0 ksi

3max 0 ksi

The dynamic loading in this case is repeated, thus

1min 0 ksi
4.

2min 0 ksi

3min 0 ksi

Even though this is a brittle material, for HCF analysis, determine the von Mises effective stresses. Since there is only one nonzero stress,

'max 1max 'min 1min 'a 'm


5. 6.

'max 8.58 ksi 'min 0 ksi 'a 4.29 ksi 'm 4.29 ksi
S'e 0.4 S ut S'e 16.8 ksi

'max 'min
2

'max 'min
2

Calculate the unmodified endurance limit.

Calculate the endurance limit modification factors for a nonrotating rectangular beam. Load Size Cload 1 A95 0.05 w h A95 7.548 mm
2

MACHINE DESIGN - An Integrated Approach, 4th Ed.

6-18-2

d equiv

A95 0.0766
0.097

d equiv 9.927 mm

Csize 1.189 Csurf 1 Ctemp 1 Creliab 0.702

d equiv mm

Csize 0.952

Surface Temperature Reliability 7.

(R = 99.99%)

Calculate the modified endurance limit. S e Cload Csize Csurf Ctemp Creliab S'e S e 11.22 ksi S m 0.9 S ut Sf = a N S m 37.8 ksi
b

8. 9.

Using equation (6.9), calculate the fatigue strength at N = 10 3 cycles.

The equation for the S-N curve in the HCF region is given by equation (6.10a):

10. Determine the constants a and b from equations (6.10c) and (6.10a). From Table 6-5, for N = 10 6 , z 3.000 b 1 z log Sm

Sm Se

b 0.1758

103

a 127.305 ksi

11. Using equation (6.10a), determine the fatigue strength.

S f5E5 a Nf

S f5E5 12.678 ksi

12. Assuming a Case 3 load line, use equation (6.18e) to calculate the factor of safety. S f5E5 S ut

Nf5E5

'a S ut 'm S f5E5

Nf5E5 2.3

MACHINE DESIGN - An Integrated Approach, 4th Ed.

6-19-1

PROBLEM 6-19
Statement: Determine the size of the clevis pin shown in Figure P6-8 needed to withstand an applied repeated force of 0 to 130000 lb for infinite life. Also determine the required outside radius of the clevis end to not fail in either tearout or bearing if the clevis flanges are each 2.5 in thick. Use a safety factor of 3. Assume S ut = 140 ksi for the pin and S ut = 80 ksi for the clevis. Minimum force Maximum force Flange thickness Pmin 0 lbf Pmax 130 kip t 2.5 in Nf 3 Material strength: Pin Clevis S utp 140 ksi S utc 80 ksi

Given:

Factor of safety against fatigue failure

Assumptions: The parts are machined. Use 90% reliability and room temperature. Solution: 1. See Figure 6-19 and Mathcad file P0619.

Calculate the alternating and mean components of the forces on the clevis and link. Pa Pm Pmax Pmin 2 Pmax Pmin 2 Pa 65 kip Pm 65 kip

Stress in Pin 2. The pin is in double shear and there is no stress-concentration. The alternating and mean loads at one section on the pin are

a =

1 Pa 2 Apin

m =

1 Pm 2 Apin d 4
2

(1)

3. 4.

The cross-section area of the pin is

Apin ( d )

(2)

The alternating and mean shear stresses and von Mises stresses are

a( d )

Pa 2 Apin ( d ) 3 a( d )

m( d )

Pm 2 Apin ( d ) 3 m( d ) (3)

'a( d )
Pin Strength 5. 6.

'm( d )

(4) S'ep 70 ksi

Calculate the unmodified endurance limit.

S'ep 0.5 S utp

Calculate the endurance limit modification factors for a nonrotating round pin in bending. Load Size Cload 1 A95 ( d ) 0.010462 d Csize( d ) 0.869 Surface A 2.70
2 0.097

d equiv( d )

A95 ( d ) 0.0766

dequiv( d) in

b 0.265

(machined)

MACHINE DESIGN - An Integrated Approach, 4th Ed.

6-19-2

Csurf Temperature Reliability 7.

S utp A ksi

Csurf 0.729

Ctemp 1 Creliab 0.897 (R = 90%)

Calculate the modified endurance limit. S ep( d ) Cload Csize( d ) Csurf Ctemp Creliab S'ep (5)

Design Equation 8. Using the modified-Goodman failure criterion and a case 3 load line, the factor of safety is given by equation 6-18e as Nf = S e S ut

'a S ut 'm S e
Guess d 2.0 in

(6)

9.

Substituting equations 4 and 5 into 6 and solving for d yields Given Nf = S ep( d ) S utp

'a( d ) S utp 'm( d ) S ep( d )


d 2.632 in d 2.750 in

d Find ( d )

Rounding to the next higher eighth of an inch, let With this value of d, we have

'a( d ) 9.5 ksi

'm( d ) 9.5 ksi

S ep( d ) 39.71 ksi

and the realized factor of safety against fatigue failure in the pin is Nf S ep( d ) S utp Nf 3.3
Tearout length

'a( d ) S utp 'm( d ) S ep( d )

Clevis Tearout (See Figure 6-19) 10. Let the outside radius of the clevis be R. Then the tearout area is Atear ( R) 2 t R ( 0.5 d )
2 2

11. The alternating and mean shear stresses and von Mises stresses are

a( R)

Pa 2 Atear ( R)

m( R)

Pm 2 Atear ( R)

(7)

FIGURE 6-19
Tearout Diagram for Problem 6-19

MACHINE DESIGN - An Integrated Approach, 4th Ed.

6-19-3

'a( R)
Clevis Strength

3 a( R)

'm( R)

3 m( R)

(8)

12. Calculate the unmodified endurance limit.

S'ec 0.5 S utc

S'ec 40 ksi

13. Calculate the endurance limit modification factors for a nonrotating rectangular shear area (uniformly stressed). Load Size Cload 1 A95 ( R) Atear ( R) d equiv( R)
0.097

A95 ( R) 0.0766

Csize( R) 0.869 A 2.70 Csurf A Temperature Reliability Ctemp 1 Creliab 0.897

d equiv( R) in

Surface

b 0.265

(machined)

S utc ksi

Csurf 0.845

(R = 90%)

14. Calculate the modified endurance limit. S ec( R) Cload Csize( R) Csurf Ctemp Creliab S'ec Design Equation 15. Using the modified-Goodman failure criterion and a case 3 load line, the factor of safety is given by equation 6-18e as S e S ut (10) Nf = 'a S ut 'm S e 16. Substituting equations 8 and 9 into 10 and solving for R yields Given Nf = S ec( R) S utc Guess R 2 in (9)

'a( R) S utc 'm( R) S ec( R)


R 2.624 in R 2.625 in

R Find ( R) Bearing Stress

The maximum bearing stress in the hole in each flange is

maxbear

Pa Pm
2 d t

maxbear 9.5 ksi

This is small compared to the ultimate strength of the clevis.

MACHINE DESIGN - An Integrated Approach, 4th Ed.

6-20-1

PROBLEM 6-20
Statement: Given: A 100 N-m torque is applied to a 1-m-long, solid round steel shaft. Design it to limit its angular deflection to 2 deg and select a steel alloy to have a fatigue safety factor of 2 for infinite life. Applied torque Shaft length Max deflection Design safety factor Modulus of rigidity Ta 100 N m L 1000 mm max 2 deg Nfd 2 G 80.8 GPa Tm 0 N m

Assumptions: There are no stress-concentrations anywhere on the shaft. The shaft is machined, reliability is 99.9%, and the it is at room temperature. Solution: 1. See Mathcad file P0620.

This is a case of fully reversed torsion. We will use the von Mises effective stress so the load factor will be 1. The maximum torque is Tmax Ta Tm Tmax 100 N m

2.

The diameter of the shaft can be found from equations 4.24 and 4.25 with = max.

max =

Tmax L J G

32 Tmax L

d G
1 4

Solving for d,

32 Tmax L d max G
d 24.5 mm

d 24.514 mm

Rounding, let 3.

Now, we can solve for the stress in the shaft. Polar moment of inertia J

32

J 3.537 10 mm

Torsional stress

Ta d 2 J

a 34.632 MPa

The corresponding von Mises normal stress is von Mises stress 4.

'a

3 a

'a 59.984 MPa

Using the factor of safety equation for reversed loading, calculate the required endurance limit Nf = Se S e Nfd 'a S e 119.967 MPa

'a

5.

This endurance limit is a function of the unknown ultimate tensile strength. Use the endurance limit modificati equation to determine the required S ut. S e = Cload Csize Csurf Ctemp Creliab S'e

6.

Calculate the endurance limit modification factors for a solid, round steel shaft.

MACHINE DESIGN - An Integrated Approach, 4th Ed.


Load Size Cload 1 Csize 1.189 A 4.51 Csurf = A Temperature Reliability Uncorrected endurance strength 7. Ctemp 1 Creliab 0.753 S'e = 0.5 S ut (R = 99.9%)

6-20-2

mm
d

0.097

Csize 0.872

Surface

b 0.265

(machined)

S ut MPa

Substituting these into the equation above and solving for S ut,
1 b 1

S ut

Se 0.5 A C C MPa size reliab

MPa

S ut 395 MPa

Based on this requirement, choose AISI 1020 cold-rolled steel that will be machined to size. 8. Check the actual factor of safety based on the material chosen. For this material, S ut 469 MPa Csurf A

Surface factor Uncorrected endurance strength Corrected endurance strength

Sut MPa

Csurf 0.884 S'e 234.5 MPa

S'e 0.5 S ut S e Cload Csize Csurf Ctemp Creliab S'e S e 136.0 MPa Nf Se

Factor of safety

'a

Nf 2.3

MACHINE DESIGN - An Integrated Approach, 4th Ed.

6-21-1

PROBLEM 6-21
Statement: Figure P6-9 shows an automobile wheel with two common styles of lug wrench being used to tighten the wheel nuts, a single-ended wrench in (a), and a double-ended wrench in (b). The distance between points A and B is 1 ft in both cases and the handle diameter is 0.625 in. How many cycles of tightening can be expected before a fatigue failure if the average tightening torque is 100 ft-lb and the material S ut = 60 ksi? Distance between A and B Wrench diameter Tensile strength d AB 1 ft d 0.625 in S ut 60 ksi Minimum torque Tmin 0 ft lbf Maximum torque Tmax 100 ft lbf

Given:

Assumptions: 1. The forces exerted by the user's hands lie in a plane through the wrench that is also parallel to the plane of the wheel. 2. The applied torque is perpendicular to the plane of the forces. 3. By virtue of 1 and 2 above, this is a planar problem that can be described in a 2D FBD. 4. The surface is as-forged, the reliability is 50%, and the wrench will not be used in extremely ho or cold environments. Solution: 1. See Figure 6-21 and Mathcad file P0621.
12" = dAB F

From examination of the FBDs, we see that, in both cases, the arms are in bending and the stub that holds the socket wrench is in pure torsion. The maximum bending stress in the arm will occur near the point where the arm transitions to the stub. The stress state at this transition is very complicated, but we can find the nominal bending stress there by treating the arm as a cantilever beam, fixed at the transition point. For both cases the torque in the stub is the same.

T F (a) Single-ended Wrench

Case (a) 2. The bending moment at the transition is M = F d AB = T Ma Tmax Tmin 2


F 6"

12" = dAB F

T (b) Double-ended Wrench

Ma 600 in lbf FIGURE 6-21 Mm Ma 3.


Free Body Diagrams for Problem 6-21

The alternating and mean components of the bending stress at this point are found from Moment of inertia I Dist to extreme fibre Alternating stress

I in

64 c 0.5 d

c 0.313 in

xa xm

Ma c I Mm c I

xa 25.033 ksi xm 25.033 ksi

Mean stress

MACHINE DESIGN - An Integrated Approach, 4th Ed.


4.

6-21-2

There are no other stress components present at this point, so x is the maximum principle stress here and

1 = x
5.

2 0 psi

3 0 psi ' = 1 = x
and

Since there is only one nonzero principal stress, the von Mises stress is

'a xa
6.

'm xm

Assuming a Case 3 load line, use equation (6.18e) to solve for the fatigue strength at which the wrench will fail (safety factor of 1). Nf = S f S ut =1 S f

'a S ut
S ut 'm

'a S ut 'm S f

S f 42.954 ksi S'e 30 ksi

7. 8.

Using equation (6-5a), calculate the uncorrected endurance limit. S'e 0.5 S ut Calculate the endurance limit modification factors for a nonrotating round beam. Load Size Cload 1 A95 0.010462 d
2 0.097

d equiv

A95 0.0766

Csize 0.869 A 39.9 Csurf A Temperature Reliability 9. Calculate the modified endurance limit. S e Cload Csize Csurf Ctemp Creliab S'e Ctemp 1 Creliab 1.000

d equiv in

Csize 1.002

Surface

b 0.995

(as forged)

S ut ksi

Csurf 0.679

(R = 50%)

S e 20.398 ksi S m 0.9 S ut Sf = a N S m 54 ksi


b

10. Using equation (6.9), calculate the fatigue strength at N = 10 3 cycles.

11. The equation for the S-N curve in the HCF region is given by equation (6.10a):

12. Determine the constants a and b from equations (6.10c) and (6.10a). From Table 6-5, for N = 10 6 , z 3.000 b 1 z log Sm

Sm Se

b 0.1409

103

a 142.955 ksi

MACHINE DESIGN - An Integrated Approach, 4th Ed.

6-21-3

13. Calculate the number of cycles to failure using equation (6.10a) Case (b) 14. The bending moment at the transition is M= M c I

Na

Sf a
T 2

Na 5.1 10

F d AB 2 Tc 2 I

15. The tensile stress at this point is found from

x =

16. The bending stress in the handle for case (b) is one half that of case (a). However, the torque in the stub is the same in both cases. The shear stress at any point on the outside surface of the stub is found from Polar moment of inertia Maximum shear stress J 2 I J 0.0150 in Tmax c J Tmin c J
4

xymax xymin a m

xymax 25.03 ksi xymin 0 ksi a 12.52 ksi m 12.52 ksi

Minimum shear stress

Alternating shear stress

xymax xymin
2

Mean shear stress

xymax xymin
2

17. There are no other stress components present along the outside surface of the stub, so

1a a

1a 12.5 ksi

2a 0 psi

3a 1a

and

'a

1a 1a 3a 3a

'a 21.7 ksi 'm 21.7 ksi

'm 'a

18. Assuming a Case 3 load line, use equation (6.18e) to solve for the fatigue strength at which the wrench will fail (safety factor of 1). Nf = S f S ut =1 S f

'a S ut
S ut 'm
1

'a S ut 'm S f

S f 33.944 ksi

19. Calculate the number of cycles to failure using equation (6.10a)

Sf Nb a

Nb 2.7 10

MACHINE DESIGN - An Integrated Approach, 4th Ed.

6-22-1

PROBLEM 6-22
Statement: A roller-blade skate is shown in Figure P6-10. The polyurethane wheels are 72 mm dia and spaced on 104-mm centers. The skate-boot-foot combination weighs 2 kg. The effective "spring rate" of the person-skate subsystem is 6000 N/m. The axles are 10-mm-dia steel pins in double shear with S ut = 550 MPa. Find the fatigue safety factor for the pins when a 100-kg person lands a 0.5-m jump on one foot assuming infinite life. (a) Assume all 4 wheels land simultaneously. (b) Assume that one wheel absorbs all the landing force. Axle pin diameter d 10 mm Tensile strength S ut 550 MPa

Given:

Assumptions: Pins are machined and reliability is 99.999%. Solution: 1. 2. See Figure P6-10 and Mathcad file P0622.

From Problem 4-22, we have the stresses for cases (a) and (b):

a 5.71 MPa

b 22.9 MPa

The dynamic loading in this case is repeated so the stresses given in step 1 are the maximum and the minimum stresses are zero. Determine the minimum, maximum, alternating, and mean von Mises stresses. Part (a)

'maxa 'aa 'ma

3 a

'maxa 9.89 MPa

'mina 0 MPa

'maxa 'mina
2

'aa 4.945 MPa 'ma 4.945 MPa 'maxb 39.664 MPa 'minb 0 MPa

'maxa 'mina
2 3 b

Part (b)

'maxb 'ab 'mb

'maxb 'minb
2

'ab 19.832 MPa 'mb 19.832 MPa


S'e 0.5 S ut S'e 275 MPa

'maxb 'minb
2

3. 4.

Calculate the unmodified endurance limit.

Calculate the endurance limit modification factors for a nonrotating round pin. Load Size Cload 1 A95

d
4

A95 78.54 mm A95

d equiv

0.0766
0.097

d equiv 32.021 mm

d equiv Csize 1.189 mm


Surface A 4.51

Csize 0.849

b 0.265
b

(machined)

Csurf

Sut A MPa

Csurf 0.847

MACHINE DESIGN - An Integrated Approach, 4th Ed.


Temperature Reliability 5. Ctemp 1 Creliab 0.659 (R = 99.999%)

6-22-2

Calculate the modified endurance limit. S e Cload Csize Csurf Ctemp Creliab S'e S e 130.42 MPa

6.

Assuming a Case 3 load line, use equation (6.18e) to determine the factor of safety. Part (a) Nfa S e S ut Nfa 21.3

'aa S ut 'ma S e
S e S ut

Part (b)

Nfb

'ab S ut 'mb S e

Nfb 5.3

MACHINE DESIGN - An Integrated Approach, 4th Ed.

6-23a-1

PROBLEM 6-23a
Statement: The beam in Figure P6-11a is subjected to a sinusoidal force-time function with Fmax = F and Fmin = -F/2, where F and the beam's other data are given in row a of Table P6-5. Find the stress state in the beam due to this loading and choose a material specification that will give a safety factor of 3 for N = 5E8 cycles. Beam length Distance to concen. load Concentrated load Moment of inertia
8

Given:

L 1 m b 0.6 m F 500 N

L b F

I 2.85 10

Distance to extreme fiber c 2.00 10 Design safety factor Nd 3


8

R1

R2

FIGURE 6-23
Free Body Diagram for Problem 6-23

Solution:

Cycle life Nf 5 10 See Figure 6-23 and Mathcad file P0623a.

1. The minimum, maximum, alternating, and mean components of the loads are: Fmax F Fa 2. Fmax Fmin 2 Fmax 500 N Fa 375 N Fmin Fm F 2 Fmin 250 N Fm 125 N

Fmax Fmin 2

Calculate the alternating and mean components of the maximum bending moment on the beam using the equati in Figure B-2(a) in Appendix B. Ma Fa b 1 b

L b L

Ma 90 N m Mm 30 N m

Mm Fm b 1

3.

Calculate the alternating and mean components of the maximum bending stress in the beam using equation (4.11b). These are principal stresses and also von Mises stresses.

'a

M a c I Mm c I

'a 63.158 MPa

'm
4.

'm 21.053 MPa

Calculate the beam cross-section dimensions from I and c. Beam depth Beam width h 2 c w 12 I h
3

h 40 mm w 5.344 mm

5.

Calculate the endurance limit modification factors for a nonrotating rectangular beam. Load Cload 1

MACHINE DESIGN - An Integrated Approach, 4th Ed.

6-23a-2

Size

A95 0.05 w h d equiv A95 0.0766


0.097

A95 10.688 mm

d equiv 11.812 mm

d equiv Csize 1.189 mm


Surface Temperature Reliability 6. Csurf 1 Ctemp 1 Creliab 1

Csize 0.936

(R = 50%)

Determine the modified endurance limit as a function of the unknown endurance limit. S e S ut Cload Csize Csurf Ctemp Creliab 0.5 S ut

7.

Assuming a Case 3 load line, use equation (6.18e) as the design equation. S e S ut S ut

Nd =

'a S ut 'm S e S ut

8.

Solve the equations in steps 6 and 7 simultaneously for the desired S ut. S ut Nd 2 'a Cload Csize Csurf Ctemp Creliab 'm Cload Csize Csurf Ctemp Creliab

S ut 468 MPa 9. Choose AISI 1020, cold-rolled steel (see Appendix A, Table A-9).

MACHINE DESIGN - An Integrated Approach, 4th Ed.

6-24a-1

PROBLEM 6-24a
Statement: The beam in Figure P6-11b is subjected to a sinusoidal force-time function with Fmax = F and Fmin = F/2, where F and the beam's other data are given in row a of Table P6-5. Find the stress state in the beam due to this loading and choose a material specification that will give a safety factor of 1.5 for N = 5E8 cycles. Beam length Concentrated load Moment of inertia I 2.85 10
8

Given:

L 1 m F 500 N m
4 2

Distance to extreme fiber c 2.00 10 Design safety factor Nd 1.5 Cycle life Solution: Nf 5 10
8

m
M1 R1

See Figure 6-24 and Mathcad file P0624a.

FIGURE 6-24 1. The minimum, maximum, alternating, and mean components of the loads are: Fmax F Fa Fmax Fmin 2 Fmax 500 N Fa 125 N
Free Body Diagram for Problem 6-24

Fmin Fm

F 2

Fmin 250 N Fm 375 N

Fmax Fmin 2

2.

Calculate the alternating and mean components of the maximum bending moment on the beam using the equati in Figure B-1(a) in Appendix B. Ma Fa L Mm Fm L Ma 125 N m Mm 375 N m

3.

Calculate the alternating and mean components of the maximum bending stress in the beam using equation (4.11b). These are principal stresses and also von Mises stresses.

'a

M a c I Mm c I

'a 87.719 MPa

'm
4.

'm 263.158 MPa

Calculate the beam cross-section dimensions from I and c. Beam depth Beam width h 2 c w 12 I h
3

h 40 mm w 5.344 mm

5.

Calculate the endurance limit modification factors for a nonrotating rectangular beam. Load Size Cload 1 A95 0.05 w h A95 10.688 mm
2

MACHINE DESIGN - An Integrated Approach, 4th Ed.

6-24a-2

d equiv

A95 0.0766
0.097

d equiv 11.812 mm

Csize 1.189 Csurf 1 Ctemp 1 Creliab 1

d equiv mm

Csize 0.936

Surface Temperature Reliability 6.

(R = 50%)

Determine the modified endurance limit as a function of the unknown endurance limit assuming the material is steel. S e S ut Cload Csize Csurf Ctemp Creliab 0.5 S ut

7.

Assuming a Case 3 load line, use equation (6.18e) as the design equation. S e S ut S ut

Nd =

'a S ut 'm S e S ut

8.

Solve the equations in steps 6 and 7 simultaneously for the desired S ut. S ut Nd 2 'a Cload Csize Csurf Ctemp Creliab 'm Cload Csize Csurf Ctemp Creliab

S ut 676 MPa 9. Choose AISI 1060 hot-rolled steel (see Appendix A, Table C-9).

MACHINE DESIGN - An Integrated Approach, 4th Ed.

6-25a-1

PROBLEM 6-25a
Statement: The beam in Figure P6-11c is subjected to a sinusoidal force-time function with Fmax = F and Fmin = 0, where F and the beam's other data are given in row a of Table P6-5. Find the stress state in the beam due to this loading and choose a material specification that will give a safety factor of 2.5 for N = 5E8 cycles. Beam length L 1 m Distance between supports b 0.6 m Concentrated load F 500 N Moment of inertia I 2.85 10
8
L b F

Given:

4 2

Distance to extreme fiber c 2.00 10 Design safety factor Nd 2.5 Cycle life Solution: 1. Nf 5 10
8

m
R1 R2

FIGURE 6-25
Free Body Diagram for Problem 6-25

See Figure 6-25 and Mathcad file P0625a.

The minimum, maximum, alternating, and mean components of the loads are: Fmax F Fa Fmax Fmin 2 Fmax 500 N Fa 250 N Fmin 0 N Fm Fmax Fmin 2 Fmin 0 N Fm 250 N

2.

Calculate the alternating and mean components of the maximum bending moment on the beam using the equati in Figure B-1(a) in Appendix B. Ma Fa ( b L) Mm Fm ( b L) Ma 100 N m Mm 100 N m

3.

Calculate the alternating and mean components of the maximum bending stress in the beam using equation (4.11b). These are principal stresses and also von Mises stresses.

'a

M a c I Mm c I

'a 70.175 MPa

'm
4.

'm 70.175 MPa

Calculate the beam cross-section dimensions from I and c. Beam depth Beam width h 2 c w 12 I h
3

h 40 mm w 5.344 mm

5.

Calculate the endurance limit modification factors for a nonrotating rectangular beam. Load Cload 1

MACHINE DESIGN - An Integrated Approach, 4th Ed.

6-25a-2

Size

A95 0.05 w h d equiv A95 0.0766


0.097

A95 10.688 mm

d equiv 11.812 mm

d equiv Csize 1.189 mm


Surface Temperature Reliability 6. Csurf 1 Ctemp 1 Creliab 1

Csize 0.936

(R = 50%)

Determine the modified endurance limit as a function of the unknown endurance limit assuming the material is steel. S e S ut Cload Csize Csurf Ctemp Creliab 0.5 S ut

7.

Assuming a Case 3 load line, use equation (6.18e) as the design equation. S e S ut S ut

Nd =

'a S ut 'm S e S ut

8.

Solve the equations in steps 6 and 7 simultaneously for the desired S ut. S ut Nd 2 'a Cload Csize Csurf Ctemp Creliab 'm Cload Csize Csurf Ctemp Creliab

S ut 550 MPa 9. Choose AISI 1035 cold-rolled steel (see Appendix A, Table A-9).

MACHINE DESIGN - An Integrated Approach, 4th Ed.

6-26a-1

PROBLEM 6-26a
Statement: The beam in Figure P6-11d is subjected to a sinusoidal force-time function with Fmax = F and Fmin = -F, where F and the beam's other data are given in row a of Table P6-5. Find the stress state in the beam due to this loading and choose a material specification that will give a safety factor of 6 for N = 5E8 cycles. Beam length L 1 m Distance to concentrated load a 0.4 m Distance to 2nd support b 0.6 m Concentrated load F 500 N Moment of inertia I 2.85 10
8
L b a F

Given:

4 2
R1 R2 R3

Distance to extreme fiber c 2.00 10 Design safety factor Nfd 6 Cycle life Solution: 1. Nlife 5 10

m FIGURE 6-26A
Free Body Diagram for Problem 6-26

See Figures 6-26 and Mathcad file P0626a.

To determine the stresses, we must first get the maximum bending moment. From inspection of Figure P6-26, write the load function equation q(x) = R1<x>-1 - F<x - a>-1 + R2<x - b>-1 - R3<x - L>-1

2.

Integrate this equation from - to x to obtain shear, V(x) V(x) = R1<x>0 - F<x - a>0 + R2<x - b>0 - R3<x - L>0

3.

Integrate this equation from - to x to obtain moment, M(x) M(x) = R1<x>1 - F<x - a>1 + R2<x - b>1 - R3<x - L>1

4.

Integrate the moment function, multiplying by 1/EI, to get the slope. (x) = [R1<x>2/2 - F<x - a>2/2 + R2<x - b>2/2 + R3<x - L>2/2 + C3]/EI

5.

Integrate again to get the deflection. y(x) = [R1<x>3/6 - F<x - a>3/6 + R2<x - b>3/6 + R3<x - L>3/6 + C3x + C4]/EI

6.

Evaluate R1, R2, R3, C3 and C4 At x = 0, x = b, and x = L; y = 0, therefore, C4 = 0. At x = L+, V = M = 0 Guess Given R1 6 R1 6 b
3

R1 100 N

R2 100 N

R3 100 N

C3 5 N m

F 6 F 6

( b a ) C 3 b = 0 N m R2 6

( L a)

( L b ) C3 L = 0 N m

R1 F R2 R3 = 0 N R1 L F ( L a ) R2 ( L b ) = 0 N m

MACHINE DESIGN - An Integrated Approach, 4th Ed.

6-26a-2

R1 R 2 Find R R R C 1 2 3 3 R3 C3
R1 111.11 N 7. 8. Define the range for x R2 472.22 N x 0 in 0.002 L L R3 83.33 N C3 5.556 N m
2

For a Mathcad solution, define a step function S. This function will have a value of zero when x is less than z, and a value of one when it is greater than or equal to z. S ( x z) if ( x z 1 0 ) Write the shear and moment equations in Mathcad form, using the function S as a multiplying factor to get the effect of the singularity functions. V ( x) R1 S ( x 0 in) F S ( x a ) R2 S ( x b ) R3 S ( x L) M ( x) R1 S ( x 0 in) x F S ( x a ) ( x a ) R2 S ( x b ) ( x b )

9.

10. Plot the shear and moment diagrams. Shear Diagram


200

Moment Diagram
60

0 V ( x) N M ( x) Nm

35

200

10

400

15

600 0 200 400 x mm 600 800 1 10


3

40 0 200 400 x mm 600 800 1 10


3

FIGURE 6-26aB

Shear and Moment Diagrams for Problem 6-26a

11. From Figure 6-26aB, the maximum moment occurs at x = a. The maximum, minimum, alternating and mean bending moments at x = a are: Mmax M ( a ) Ma Mm Mmax Mmin 2 Mmax Mmin 2 Mmax 44.4 N m Ma 44.444 N m Mm 0 N m Mmin Mmax

MACHINE DESIGN - An Integrated Approach, 4th Ed.

6-26a-3

12. Calculate the alternating and mean components of the maximum bending stress in the beam using equation (4.11b). These are principal stresses and also von Mises stresses.

'a

M a c I Mm c I

'a 31.189 MPa

'm

'm 0 MPa

13. Calculate the beam cross-section dimensions from I and c. Beam depth Beam width h 2 c w 12 I h
3

h 40 mm w 5.344 mm

14. Calculate the endurance limit modification factors for a nonrotating rectangular beam. Load Size Cload 1 A95 0.05 w h d equiv A95 0.0766
0.097

A95 10.688 mm

d equiv 11.812 mm

d equiv Csize 1.189 mm


Surface Temperature Reliability A 4.51 Ctemp 1 Creliab 1

Csize 0.936

b 0.265

S ut Csurf S ut A MPa

(R = 50%)

15. Determine the modified endurance limit as a function of the unknown endurance limit. S e S ut Cload Csize Csurf S ut Ctemp Creliab 0.5 S ut 16. Assuming a Case 3 load line, use equation (6.18e) as the design equation and solve for S ut. Guess Given S e S ut S ut S ut Find S ut Csurf S ut 0.895 18. Choose AISI 1020, cold-rolled steel (see Appendix A, Table A-9). S ut 447 MPa S ut 100 MPa

Nfd =

'a S ut 'm S e S ut

MACHINE DESIGN - An Integrated Approach, 4th Ed.

6-27-1

PROBLEM 6-27
Statement: A storage rack is to be designed to hold the paper roll of Problem 6-8 as shown in Figure P6-12. Determine a suitable value for dimension a in the figure for an infinite-life fatigue safety factor of 2. Assume dimension b = 100 mm and that the mandrel is solid and inserts halfway into the paper roll. (a) The beam is a ductile material with S ut = 600 MPa. (b) The beam is a cast-brittle material with S ut = 300 MPa. Paper roll dimensions OD 1.50 m ID 0.22 m Lroll 3.23 m Ductile tensile strength S uta 600 MPa
W

Given:

Roll density Design safety factor Brittle tensile strength

984 kg m
Nfd 2

S utb 300 MPa

Assumptions: The paper roll's weight creates a concentrated load acting at the tip of the mandrel. The mandrel's root fits tightly in the stanchion so it can be modeled as a cantilever beam. The mandrel is machined, reliability is 90%, and it operates at room temperature. Solution: 1. See Figure 6-27 and Mathcad file P0627.

a M1 R1

Lm

FIGURE 6-27
Free Body Diagram used in Problem 6-27

Determine the weight of the roll and the length of the mandrel. Weight Length W

OD ID Lroll g

W 53.9 kN Lm 1.615 m

Lm 0.5 Lroll

2.

The maximum moment occurs at a section where the mandrel root leaves the stanchion and is Mmax W Lm Mmax 87.04 kN m

3. 4.

The dynamic loading is repeated from 0 to Mmax on each stress cycle, thus Mmin 0 kN m Part (a) - Calculate the alternating and mean components of the bending moment. Ma Mm Mmax Mmin 2 Mmax Mmin 2 Ma 43520 N m Mm 43520 N m S'e 0.5 S uta S'e 300 MPa

5. 6.

Determine the unmodified endurance limit.

Calculate the endurance limit modification factors for a nonrotating rectangular beam. Load Cload 1 A95 ( a ) 0.010462 a
2 0.097

Size

d equiv( a )

A95 ( a ) 0.0766

dequiv( a) Csize( a ) 1.189 mm

MACHINE DESIGN - An Integrated Approach, 4th Ed.


Surface A 4.51 b 0.265
b

6-27-2
(machined)

Csurf Temperature Reliability 7.

S uta A MPa

Csurf 0.828

Ctemp 1 Creliab 0.897 (R = 90%)

Calculate the modified endurance limit. S e( a ) Cload Csize( a ) Csurf Ctemp Creliab S'e

8.

We can now determine the minimum required diameter, a. Using the distortion-energy failure theory with the modified Goodman diagram, the bending stress will also be the only nonzero principal stress, which will also be the von Mises stress. Assuming a Case 3 load line, use equation (6.18e) to determine the factor of safety. Guess a 100 mm. Bending stress Given Nfd =

M c I

a 64 32 M = M = 2 4 3 a a
3

a
32

S e( a ) S uta Ma S uta Mm S e( a ) a 186.864 mm a 190 mm

a Find ( a ) Round this up to the next higher even value Using this value of a, the values of the functions of a are: Csize( a ) 0.787 The realized safety factor is Nfa S e( a ) 175.371 MPa

a
32

S e( a ) S uta Ma S uta Mm S e( a ) S'e 0.4 S utb

Nfa 2.1

9.

Part (b) - Determine the unmodified endurance limit.

S'e 120 MPa

10. Calculate the endurance limit size modification factor for a nonrotating rectangular beam. Size A95 ( a ) 0.010462 a
2 0.097

d equiv( a )

A95 ( a ) 0.0766

dequiv( a) Csize( a ) 1.189 mm


11. Calculate the modified endurance limit. S e( a ) Cload Csize( a ) Csurf Ctemp Creliab S'e

MACHINE DESIGN - An Integrated Approach, 4th Ed.

6-27-3

12. We can now determine the minimum required diameter, a. Using the distortion-energy failure theory with the modified Goodman diagram, the bending stress will also be the only nonzero principal stress, which will also be the von Mises stress. Assuming a Case 3 load line, use equation (6.18e) to determine the factor of safety. Guess a 100 mm. Bending stress Given Nfd =

M c I

a 64 32 M = M = 2 4 3 a a
3

a
32

S e( a ) S utb Ma S utb Mm S e( a ) a 251.687 mm a 252 mm

a Find ( a ) Round this up to the next higher even value Using this value of a, the values of the functions of a are: Csize( a ) 0.766 The realized safety factor is Nfb S e( a ) 68.253 MPa

a
32

S e( a ) S utb Ma S utb Mm S e( a )

Nfb 2.0

MACHINE DESIGN - An Integrated Approach, 4th Ed.

6-28-1

PROBLEM 6-28
Statement: Figure P6-13 shows a forklift truck negotiating a 15 deg ramp to to drive onto a 4-ft-high loading platform. The truck weighs 5 000 lb and has a 42-in wheelbase. Design two (one for each side) 1-ft-wide ramps of steel to have a safety factor of 2 for infinite life in the worst case of loading as the truck travels up them. Minimize the weight of the ramps by using a sensible cross-sectional geometry. Choose an appropriate steel or aluminum alloy. Ramp angle 15 deg Platform height h 4 ft Truck wheelbase Lt 42 in Ramp width Truck weight w 12 in W 5000 lbf

Given:

Assumptions: 1. The worst case is when the truck CG is located at the center of the beam's span. 2. Use a coordinate frame that has the x-axis along the long axis of the beam. 3. Ignore traction forces and the weight components along the x-axis of the beam. 4. There are two ramps, one for each side of the forklift. See Figures 6-28 and Mathcad file P0628. Solution:

L b a CG a
CG b

R1 Fa Wa Fb Wb R2 x

FIGURE 6-28A
Dimensions and Free Body Diagram for Problem 6-28

1.

From Problem 3-28 the maximum bending moment in the ramp occurs at the rear wheel of the truck and is Mmax 8324 ft lbf Mmax 99888 in lbf Mmin 0 in lbf

The alternating and mean components of the bending moment are: Ma Mmax Mmin 2 Mmax Mmin 2 Ma 49944 in lbf

Mm 2.

Mm 49944 in lbf

The bending stress is the only stress component present and is, therefore, also the only nonzero principal stress and is also the von Mises stress. The governing design equations then are

MACHINE DESIGN - An Integrated Approach, 4th Ed.


S e S ut

6-28-2

'a =

Ma Z

'm =

Mm Z

Nfd =

'a S ut 'm S e
Z S e S ut Ma S ut Mm S e

Combining these into a single equation

Nfd =

3.

The approach will be to 1) choose a suitable factor of safety, 2) choose a suitable material and determine its tensile strength and endurance limit, 3) from the equation above determine the required value of the section modulus, 4) choose an appropriate cross-section for the ramp, and 5) determine the dimensions of the cross-section. The following design choices have been made for this problem: Design factor of safety Material Tensile strength Nfd 2 AISI 1095 steel, hot-rolled S ut 120 ksi S'e 0.5 S ut S'e 60 ksi

4.

5. 6.

Determine the unmodified endurance limit.

Calculate the endurance limit modification factors for a nonrotating rectangular beam. Load Size Surface Cload 1 Csize 0.732 A 14.4 Csurf A Temperature Reliability Ctemp 1 Creliab 0.897 (R = 90%) (initially guessed, and then found by iteration) b 0.718 (hot-rolled)

S ut ksi

Csurf 0.463

7.

Calculate the modified endurance limit. S e Cload Csize Csurf Ctemp Creliab S'e S e 18.237 ksi

8.

Solve the design equation for the minimum section modulus, Z. Z Nfd

Ma Sut Mm Se
S e S ut

Z 6.309 in

This is the minimum allowable value of the section modulus. 9. Assume a channel section such as that shown in Figure 6-28B. To keep it simple, let the thickness of the flanges and web be the same. Choose 5/8-in thick plate, which is readily available. Then, t 0.625 in

10. The cross-sectional area of the ramp is A ( h ) w t 2 t ( h t) 11. The distance to the CG is cg( h ) 1 A (h)

w t 2 2

t h t

MACHINE DESIGN - An Integrated Approach, 4th Ed.

6-28-3

12. The moments of inertia of the web and a flange are Iweb( h ) w t
3

Flange Web

12

w t cg( h )

2 t
2

Ifl ( h )

t ( h t) 12

h h t cg( h ) 2

I ( h ) Iweb( h ) 2 Ifl ( h ) 13. The maximum stress will occur in the flange at the top and is compressive. The distance from the centroid up to the top of the flange is c( h ) h cg( h ) 14. Using the known section modulus, solve for the unknown flange height, h. Guess h 1 in Given Z= I (h) c( h ) h Find ( h ) h 4.304 in FIGURE 6-28B
Channel Section for Problem 6-28

Round this to

h 4.25 in

15. Check the size modification factor. A95 0.05 w cg( h ) t ( h cg( h ) ) d equiv A95 0.0766
0.097

A95 2.628 in

d equiv 5.858 in

d equiv Csize 0.869 in

Csize 0.732

16. Summarizing, the ramp design dimensions are: Width w 12.00 in Flange height Thickness h 4.25 in t 0.625 in Shape Material channel 1095 steel

MACHINE DESIGN - An Integrated Approach, 4th Ed.

6-29-1

PROBLEM 6-29
Statement: A bar, 22 mm x 30 mm in cross-section, is loaded axially in tension with Fmin = -8 kN and Fmax = +8 kN. A 10-mm hole passes through the center of the 30-mm side. Find the safety factor for infinite life if the material has S ut = 500 MPa. Bar width Bar thickness Hole diameter Tensile strength w 30 mm h 22 mm d 10 mm S ut 500 MPa Maximum load Minimum load Infinite life Fmax 8 kN Fmin 8 kN

Given:

Assumptions: Machined surfaces, temperature of 37C, and reliability of 99.999%. Solution: 1. N = See Figure 6-29 and Mathcad file P0629.

For completely reversed loading, the factor of safety is Se Kf the Since 'a uniform axial stress is the only stress component present,

30

'a = a
2.

and

N =

Se Kf a

10

Calculate the unmodified endurance limit. S'e 0.5 S ut S'e 250 MPa

3.

Calculate the endurance limit modification factors for an axial bar. Load Size Surface Cload 0.7 Csize 1 A 4.51 (axial loading) (axial loading) b 0.265
b

22

Csurf Temperature Reliability 4.

Sut A MPa

Csurf 0.869

F
FIGURE 6-29 (R = 99.999%)
Free Body Diagram used in Problem 6-29

Ctemp 1 Creliab 0.659

Calculate the modified endurance limit. S e Cload Csize Csurf Ctemp Creliab S'e S e 100.2 MPa

5.

Determine the nominal (not increased by a stress concentration factor) alternating component of stress at the hole. Area Alternating load A ( w d ) h Fa Fmax Fmin 2 Fa A A 440 mm Fa 8 kN
2

Alternating stress

a 18.182 MPa

MACHINE DESIGN - An Integrated Approach, 4th Ed.


6. Determine the geometric stress concentration factor from Appendix C, Figure C-13. Kt 3.0039 3.753

6-29-2

w 4 5 d d 1.8145 2.9684 w w
w

7.9735

9.2659

w
Kt 2.33

7.

Determine the notch sensitivity of the material. Note from Figure 6-35 that the Neuber constant for steel in tension is slightly lower that for torsional loading. However, comparison of values of a 1/2 obtained from the dashed red curve with those in Table 6-6 indicates that, for tension as well as torsion, a value of 20 ksi should be added to S ut to obtain a 1/2 from Table 6-6. Lookup value of S ut Neuber constant Notch radius Notch sensitivity S'ut S ut 20 ksi a 0.068 in r 0.5 d q 1 1 a r
2

S'ut 93 ksi a 0.068 in r 5 mm q 0.867


0.5

8.

Determine the fatigue stress concentration factor from equation (6.11b). Kf 1 q Kt 1 Kf 2.153

9.

Determine the factor of safety against fatigue failure for the assumptions made. Se Kf a

Nf

Nf 2.6

MACHINE DESIGN - An Integrated Approach, 4th Ed.

6-30-1

PROBLEM 6-30
Statement: A bar, 22 mm x 30 mm in cross-section, is loaded axially in tension with Fmin = 0 kN and Fmax = 16 kN. A 10-mm hole passes through the center of the 30-mm side. Find the safety factor for infinite life if the material has S ut = 500 MPa. Bar width Bar thickness Hole diameter Tensile strength w 30 mm h 22 mm d 10 mm S ut 500 MPa Maximum load Minimum load Infinite life Fmax 16 kN Fmin 0 kN

Given:

Assumptions: Machined surfaces, temperature of 37C, and reliability of 99.999%. Solution: 1. See Figure 6-29 and Mathcad file P0629.

F
For fluctuating loading, the factor of safety is S e S ut Kf 'a S ut Kfm 'm S e

N =

30
Since the uniform axial stress is the only stress component present,

'a = a
2.

and

'm = m

10

Calculate the unmodified endurance limit. S'e 0.5 S ut S'e 250 MPa

3.

Calculate the endurance limit modification factors for an axial bar. Load Size Surface Cload 0.7 Csize 1 A 4.51 (axial loading) (axial loading) b 0.265
b

22

Csurf Temperature Reliability 4.

Sut A MPa

Csurf 0.869

F
FIGURE 6-30 (R = 99.999%)
Free Body Diagram used in Problem 6-30

Ctemp 1 Creliab 0.659

Calculate the modified endurance limit. S e Cload Csize Csurf Ctemp Creliab S'e S e 100.2 MPa

5.

Determine the nominal (not increased by a stress concentration factor) alternating and mean components of stress at the hole. Area A ( w d ) h A 440 mm
2

MACHINE DESIGN - An Integrated Approach, 4th Ed.


Alternating load Fa Fm Fmax Fmin 2 Fmax Fmin 2 Fa A Fm A Fa 8 kN Fm 8 kN

6-30-2

Mean load

Alternating stress

a 18.182 MPa

Mean stress 6.

m 18.182 MPa

Determine the geometric stress concentration factor from Appendix C, Figure C-13. Kt 3.0039 3.753

w 4 5 d d 1.8145 2.9684 w w
w

7.9735

9.2659

Kt 2.33

7.

Determine the notch sensitivity of the material. Note from Figure 6-35 that the Neuber constant for steel in tension is slightly lower that for torsional loading. However, comparison of values of a 1/2 obtained from the dashed red curve with those in Table 6-6 indicates that, for tension as well as torsion, a value of 20 ksi should be added to S ut to obtain a 1/2 from Table 6-6. Lookup value of S ut Neuber constant Notch radius Notch sensitivity S'ut S ut 20 ksi a 0.068 in r 0.5 d q 1 1 a r
2

S'ut 93 ksi a 0.068 in r 5 mm q 0.867


0.5

8.

Determine the fatigue stress concentration factors from equations (6.11b) and (6.17). Kf 1 q Kt 1 Kf 2.153 Kf m a 78 MPa

Assuming the yield strength for this material is about 400 MPa, we can use the first of equations (6.17) and Kfm Kf 9. Kfm 2.153

Determine the factor of safety against fatigue failure for the assumptions made. S e S ut Kf a S ut Kfm m S e

Nf

Nf 2.1

MACHINE DESIGN - An Integrated Approach, 4th Ed.

6-31-1

PROBLEM 6-31
Statement: A bar, 22 mm x 30 mm in cross-section, is loaded axially in tension with Fmin = 8 kN and Fmax = 24 kN. A 10-mm hole passes through the center of the 30-mm side. Find the safety factor for infinite life if the material has S ut = 500 MPa. Bar width Bar thickness Hole diameter Tensile strength w 30 mm h 22 mm d 10 mm S ut 500 MPa Maximum load Minimum load Infinite life Fmax 24 kN Fmin 8 kN

Given:

Assumptions: Machined surfaces, temperature of 37C, and reliability of 99.999%. Solution: 1. See Figure 6-29 and Mathcad file P0629.

For fluctuating loading, the factor of safety is S e S ut Kf 'a S ut Kfm 'm S e

N =

30

Since the uniform axial stress is the only stress component present,

'a = a
2.

and

'm = m

10

Calculate the unmodified endurance limit. S'e 0.5 S ut S'e 250 MPa

3.

Calculate the endurance limit modification factors for an axial bar. Load Size Surface Cload 0.7 Csize 1 A 4.51 (axial loading) (axial loading) b 0.265
b

22

Csurf Temperature Reliability

Sut A MPa

Csurf 0.869

F
FIGURE 6-31
Free Body Diagram used in Problem 6-31

Ctemp 1 Creliab 0.659 (R = 99.999%)

4. Calculate the modified endurance limit. S e Cload Csize Csurf Ctemp Creliab S'e 5. S e 100.2 MPa

Determine the nominal (not increased by a stress concentration factor) alternating and mean components of stress at the hole. Area Alternating load A ( w d ) h Fa Fmax Fmin 2 A 440 mm Fa 8 kN
2

MACHINE DESIGN - An Integrated Approach, 4th Ed.


Fmax Fmin 2 Fa A Fm A

6-31-2

Mean load

Fm

Fm 16 kN

Alternating stress

a 18.182 MPa

Mean stress 6.

m 36.364 MPa

Determine the geometric stress concentration factor from Appendix C, Figure C-13. Kt 3.0039 3.753

d 9.2659 w w w 4 5 d d 1.8145 2.9684 w w


d

7.9735

Kt 2.33

7.

Determine the notch sensitivity of the material. Note from Figure 6-35 that the Neuber constant for steel in tension is slightly lower that for torsional loading. However, comparison of values of a 1/2 obtained from the dashed red curve with those in Table 6-6 indicates that, for tension as well as torsion, a value of 20 ksi should be added to S ut to obtain a 1/2 from Table 6-6. Lookup value of S ut Neuber constant Notch radius Notch sensitivity S'ut S ut 20 ksi a 0.068 in r 0.5 d q 1 1 a r
2

S'ut 93 ksi a 0.068 in r 5 mm q 0.867


0.5

8.

Determine the fatigue stress concentration factors from equations (6.11b) and (6.17). Kf 1 q Kt 1 Kf 2.153 Kf m a 117 MPa

Assuming the yield strength for this material is about 400 MPa, we can use the first of equations (6.17) and Kfm Kf 9. Kfm 2.153

Determine the factor of safety against fatigue failure for the assumptions made. S e S ut Kf a S ut Kfm m S e

Nf

Nf 1.8

MACHINE DESIGN - An Integrated Approach, 4th Ed.

6-32-1

PROBLEM 6-32
Statement: A bar, 22 mm x 30 mm in cross-section, is loaded axially in tension with Fmin = -4 kN and Fmax = 12 kN. A 10-mm hole passes through the center of the 30-mm side. Find the safety factor for infinite life if the material has S ut = 500 MPa. Bar width Bar thickness Hole diameter Tensile strength w 30 mm h 22 mm d 10 mm S ut 500 MPa Maximum load Minimum load Infinite life Fmax 12 kN Fmin 4 kN

Given:

Assumptions: Machined surfaces, temperature of 37C, and reliability of 99.999%. Solution: 1. See Figure 6-29 and Mathcad file P0629.

For fluctuating loading, the factor of safety is S e S ut Kf 'a S ut Kfm 'm S e

N =

30

Since the uniform axial stress is the only stress component present,

'a = a
2.

and

'm = m

10

Calculate the unmodified endurance limit. S'e 0.5 S ut S'e 250 MPa

3.

Calculate the endurance limit modification factors for an axial bar. Load Size Surface Cload 0.7 Csize 1 A 4.51 (axial loading) (axial loading) b 0.265
b

22

Csurf Temperature Reliability 4.

Sut A MPa

Csurf 0.869

F
FIGURE 6-32 (R = 99.999%)
Free Body Diagram used in Problem 6-32

Ctemp 1 Creliab 0.659

Calculate the modified endurance limit. S e Cload Csize Csurf Ctemp Creliab S'e S e 100.2 MPa

5.

Determine the nominal (not increased by a stress concentration factor) alternating and mean components of stress at the hole. Area A ( w d ) h Fa Fmax Fmin 2 A 440 mm Fa 8 kN
2

Alternating load

MACHINE DESIGN - An Integrated Approach, 4th Ed.


Fmax Fmin 2 Fa A Fm A

6-32-2

Mean load

Fm

Fm 4 kN

Alternating stress

a 18.182 MPa

Mean stress 6.

m 9.091 MPa

Determine the geometric stress concentration factor from Appendix C, Figure C-13. Kt 3.0039 3.753

w 4 5 d d 1.8145 2.9684 w w
w

7.9735

9.2659

Kt 2.33

7.

Determine the notch sensitivity of the material. Note from Figure 6-35 that the Neuber constant for steel in tension is slightly lower that for torsional loading. However, comparison of values of a 1/2 obtained from the dashed red curve with those in Table 6-6 indicates that, for tension as well as torsion, a value of 20 ksi should be added to S ut to obtain a 1/2 from Table 6-6. Lookup value of S ut Neuber constant Notch radius Notch sensitivity S'ut S ut 20 ksi a 0.068 in r 0.5 d q 1 1 a r
2

S'ut 93 ksi a 0.068 in r 5 mm q 0.867


0.5

8.

Determine the fatigue stress concentration factors from equations (6.11b) and (6.17). Kf 1 q Kt 1 Kf 2.153 Kf m a 59 MPa

Assuming the yield strength for this material is about 400 MPa, we can use the first of equations (6.17) and Kfm Kf 9. Kfm 2.153

Determine the factor of safety against fatigue failure for the assumptions made. S e S ut Kf a S ut Kfm m S e

Nf

Nf 2.3

MACHINE DESIGN - An Integrated Approach, 4th Ed.

6-33a-1

PROBLEM 6-33a
Statement: For the bracket shown in Figure P6-14 subjected to a sinusoidal force-time function with Fmax = F and Fmin = -F, where F and the beam's other data are given in row a of Table P6-6. Find the stress states at points A and B due to this fully reversed loading and choose a ductile steel material specification that will give a safety factor of 2 for infinite life. Assume a geometric stress-concentration factor of 2.5 in bending and 2.8 in torsion. Outside diameter Geometric stress concentration factors Design safety factor od 20 mm Kt 2.5 Kts 2.8 Nd 2
T M L R y A B T x F

Given:

Assumptions: The finish is machined, reliability is 50%, and the bracket operates at room temperature. The notch sensitivity q = 1 so that Kf = Kt. Solution: 1. 2.

See Figure 6-33 and Mathcad file P0633a. FIGURE 6-33


Free Body Diagram of Tube for Problem 6-33

From Problem 4-33a the stress components at point A are x 8.38 MPa Calculate fatigue stresses and principal stresses. Fatigue stresses

zx 16.76 MPa

b Kt x s Kts zx

b 20.95 MPa s 46.93 MPa


2

Principal stresses

b 2 1 s 2 2
b

1 58.56 MPa 2 0 MPa

b 2 3 s 2 2
b
3. Calculate the alternating von Mises effective stress (the mean component is zero).

3 37.61 MPa

'a
4. 5. Calculate the unmodified endurance limit

1 1 3 3

'a 83.94 MPa

S'e S ut 0.5 S ut

Determine the endurance limit modification factors Load Size Cload 1 A95 0.010462 od d eq A95 0.0766
2

(nonrotating round section)

d eq 7.391 mm

MACHINE DESIGN - An Integrated Approach, 4th Ed.

6-33a-2
0.097

deq Csize 1.189 mm


Surface A 4.51 Cs S ut A

Csize 0.979 (cold-drawn tubing)

b 0.265

S ut MPa

Csurf S ut if Cs S ut 1 1 Cs S ut Temperature Reliability Ctemp 1 Creliab 1.0 (R = 50%)

6.

Calculate the modified endurance limit S e S ut Cload Csize Csurf S ut Ctemp Creliab S'e S ut

7.

Use the equation for the factor of safety for fully reversed loading to solve for S ut. Guess S ut 100 MPa S e S ut S ut Find S ut S ut 362 MPa

Given

Nd =

'a

8.

The varables that depend on S ut are:

Csurf S ut 0.946

S e S ut 167.88 MPa

MACHINE DESIGN - An Integrated Approach, 4th Ed.

6-34a-1

PROBLEM 6-34a
Statement: For the bracket shown in Figure P6-14 subjected to a sinusoidal force-time function with Fmax = F and Fmin = 0, where F and the beam's other data are given in row a of Table P6-6. Find the stress states at points A and B due to this repeated loading and choose a ductile steel material specification that will give a safety factor of 2 for infinite life. Assume a geometric stress-concentration factor of 2.8 in bending and 3.2 in torsion. Outside diameter Geometric stress concentration factors Design safety factor od 20 mm Kt 2.8 Kts 3.2 Nd 2
T M L R y A B T x F

Given:

Assumptions: The finish is machined, reliability is 50%, and the bracket operates at room temperature. The notch sensitivity q = 1 so that Kf = Kt. Solution: 1.

See Figure 6-33 and Mathcad file P0633a. FIGURE 6-34


Free Body Diagram of Tube for Problem 6-34

From Problem 4-33a the stress components at point A are

xmax 8.38 MPa xmin 0 MPa


2. Calculate the alternating and mean stress components.

zxmax 16.76 MPa zxmin 0 MPa

xa xm zxa zxm
3.

xmax xmin
2

xa 4.19 MPa xm 4.19 MPa zxa 8.38 MPa zxm 8.38 MPa

xmax xmin
2

zxmax zxmin
2

zxmax zxmin
2

Calculate fatigue stresses and principal stresses. Fatigue stresses

ba Kt xa sa Kts zxa bm Kt xa sm Kts zxa

ba 11.73 MPa sa 26.82 MPa bm 11.73 MPa sm 26.82 MPa

Principal stresses

ba 2 1a sa 2 2
ba

1a 33.32 MPa 2a 0 MPa

MACHINE DESIGN - An Integrated Approach, 4th Ed.

6-34a-2

ba 2 3a sa 2 2
ba bm
2

3a 21.58 MPa

1m

bm 2 sm 2 bm 2 sm 2
2

1m 33.32 MPa 2m 0 MPa

3m
4.

bm
2

3m 21.58 MPa

Calculate the alternating and mean von Mises effective stress components.

'a 'm
5. 6.

1a 1a 3a 3a
2

'a 47.91 MPa


2

1m 1m 3m 3m

'm 47.91 MPa

Calculate the unmodified endurance limit

S'e S ut 0.5 S ut

Determine the endurance limit modification factors Load Size Cload 1 A95 0.010462 od d eq A95 0.0766
0.097 2

(nonrotating round section)

d eq 7.391 mm

deq Csize 1.189 mm


Surface A 4.51 Cs S ut A

Csize 0.979 (machined tubing)

b 0.265

S ut MPa

Csurf S ut if Cs S ut 1 1 Cs S ut Temperature Reliability 7. Calculate the modified endurance limit S e S ut Cload Csize Csurf S ut Ctemp Creliab S'e S ut 8. Use the equation for the factor of safety for repeated loading assuming a Case 3 load line and using equation (6.18e). Guess S ut 100 MPa Ctemp 1 Creliab 1.0 (R = 50%)

MACHINE DESIGN - An Integrated Approach, 4th Ed.

6-34a-3

Given

Nd =

'a S ut 'm S e S ut
S ut 291 MPa S e S ut 142.72 MPa

S e S ut S ut

S ut Find S ut 9. The varables that depend on S ut are: Csurf S ut 1

MACHINE DESIGN - An Integrated Approach, 4th Ed.

6-35a-1

PROBLEM 6-35a
Statement: For the bracket shown in Figure P6-14 subjected to a sinusoidal force-time function with Fmax = F and Fmin = -F, where F and the beam's other data are given in row a of Table P6-6. Find the stress states at points A and B due to this fully reversed loading and choose a cast iron material specification that will give a safety factor of 2 for infinite life. Assume a geometric stress-concentration factor of 2.5 in bending and 2.8 in torsion. Outside diameter Geometric stress concentration factors Design safety factor od 20 mm Kt 2.5 Kts 2.8 Nd 2
T M L R y A B T x F

Given:

Assumptions: The finish is as-cast, reliability is 50%, and the bracket operates at room temperature. There ia a fillet at the wall with radius r 2 mm. However, set the stress concentration factors and Csurf to 1 since cast iron's internal flaws mask these effects. Solution: 1 2. See Figure 6-35 and Mathcad file P0635a.

FIGURE 6-35
Free Body Diagram of Tube for Problem 6-35

From Problem 4-33a the stress components at point A are x 8.38 MPa Calculate fatigue stress concentration factors and principal stresses. Let Fatigue stresses Kf 1

zx 16.76 MPa
Kfs 1

b Kf x s Kfs zx

b 8.38 MPa s 16.76 MPa

Principal stresses

b 2 1 s 2 2
b

1 21.47 MPa

b 2 3 s 2 2
b 2 0 MPa
3. Calculate the alternating von Mises effective stress (the mean component is zero).

3 13.09 MPa

'a
4. 5.

1 1 3 3

'a 30.21 MPa

Calculate the unmodified endurance limit

S'e S ut 0.4 S ut

Determine the endurance limit modification factors Load Size Cload 1 A95 0.010462 od
2

(nonrotating round section)

MACHINE DESIGN - An Integrated Approach, 4th Ed.

6-35a-2

d eq

A95 0.0766
0.097

d eq 7.391 mm

Csize 1.189 Surface Temperature Reliability 6. Calculate the modified endurance limit Csurf 1 Ctemp 1 Creliab 1.0

deq mm

Csize 0.979 ( cast iron)

(R = 50%)

S e S ut Cload Csize Csurf Ctemp Creliab S'e S ut 7. Use the equation for the factor of safety for fully reversed loading to solve for S ut. Guess S ut 100 MPa Given Nd = S e S ut S ut Find S ut S ut 154 MPa

'a

8.

The varables that depend on S ut are:

S e S ut 60.43 MPa

MACHINE DESIGN - An Integrated Approach, 4th Ed.

6-36a-1

PROBLEM 6-36a
Statement: For the bracket shown in Figure P6-14 subjected to a sinusoidal force-time function with Fmax = F and Fmin = 0, where F and the beam's other data are given in row a of Table P6-6. Find the stress states at points A and B due to this repeated loading and choose a cast iron material specification that will give a safety factor of 2 for infinite life. Assume a geometric stress-concentration factor o 2.8 in bending and 3.2 in torsion. Outside diameter Geometric stress concentration factors Design safety factor od 20 mm Kt 2.8 Kts 3.2 Nd 2

Given:

Assumptions: The finish is as-cast, reliability is 50%, and the bracket operates at room temperature. There ia a fillet at the wall with radius r 2 mm. However, set the stress concentration factors and Csurf to 1 since cast iron's internal flaws mask these effects. Solution: 1. See Figure 6-36 and Mathcad file P0636a.

FIGURE 6-36
Free Body Diagram of Tube for Problem 6-36

From Problem 4-33a the stress components at point A are

xmax 8.38 MPa xmin 0 MPa


2. Calculate the alternating and mean stress components.

zxmax 16.76 MPa zxmin 0 MPa

xa xm zxa zxm
3.

xmax xmin
2

xa 4.19 MPa xm 4.19 MPa zxa 8.38 MPa zxm 8.38 MPa

xmax xmin
2

zxmax zxmin
2

zxmax zxmin
2

Calculate fatigue stress concentration factors (set 1 for cast iron) and principal stresses Fatigue stress concentration factors Fatigue stresses Kf 1 Kfm 1 Kfs 1 Kfsm 1

ba Kf xa sa Kfs zxa bm Kfm xa sm Kfsm zxa

ba 4.19 MPa sa 8.38 MPa bm 4.19 MPa sm 8.38 MPa

MACHINE DESIGN - An Integrated Approach, 4th Ed.


Principal stresses

6-36a-2

ba 2 1a sa 2 2
ba

1a 10.73 MPa 2a 0 MPa

ba 2 3a sa 2 2
ba bm
2

3a 6.54 MPa

1m

bm 2 sm 2 bm 2 sm 2
2

1m 10.73 MPa 2m 0 MPa

3m
4.

bm
2

3m 6.54 MPa

Calculate the alternating von Mises effective stress (the mean component is zero).

'a 'm
5. 6.

1a 1a 3a 3a
2

2 2

'a 15.11 MPa 'm 15.11 MPa

1m 1m 3m 3m

Calculate the unmodified endurance limit

S'e S ut 0.4 S ut Cload 1 A95 0.010462 od d eq A95 0.0766


2

Determine the endurance limit modification factors Load Size

(nonrotating round section) d eq 7.391 mm


0.097

Csize 1.189 Surface Temperature Reliability 7. Calculate the modified endurance limit Csurf 1 Ctemp 1 Creliab 1.0

deq mm

Csize 0.979 (cast iron)

(R = 50%)

S e S ut Cload Csize Csurf Ctemp Creliab S'e S ut 8. Use the equation for the factor of safety for repeated loading assuming a Case 3 load line and using equation (6.18e). Guess S ut 100 MPa Given Nd =

'a S ut 'm S e S ut
S ut 107 MPa

S e S ut S ut

S ut Find S ut

MACHINE DESIGN - An Integrated Approach, 4th Ed.

6-37-1

PROBLEM 6-37
Statement: A semicircular, curved beam as shown in Figure 5-37 has the dimensions given below. For a load pair F = 3 kN applied along the diameter, find the safety factor at the inner and outer fibers: (a) If the beam is steel with S ut = 700 MPa, (b) If the beam is cast-iron with S ut = 420 MPa.

Given:

(a) Tensile strength (b) Tensile strength Maximum load Minimum load

S uta 700 MPa S utb 420 MPa Fmax 3 kN Fmin 3 kN

Solution: 1.

See Figure 6-37 and Mathcad file P0637.

From Problem 4-37, the stresses at the inside radius are: Inside

i 409.9 MPa

These are based on a load of 14 kN. Since the stress in a curved beam is directly proportional to the applied load, we can determine the stresses at the inside surface for this problem by applying the ratio 3/14 to this stress. Thus,

max min

3 14 3 14

i i

max 87.836 MPa min 87.836 MPa


FIGURE 6-37
Free Body Diagrams for Problem 6-37

These are the only stress components present on their respective surfaces so they are also von Mises stresses. 2.

The dynamic loading in this problem is fully reversed. Determine the alternating stress component.

'a
Part (a) 3. 4.

max min
2

'a 87.836 MPa

Calculate the unmodified endurance limit.

S'ea 0.5 S uta

S'ea 350 MPa

Calculate the endurance limit modification factors for a nonrotating rectangular beam. Load Size Cload 0.7 Section dims A95 0.05 w h d equiv A95 0.0766
0.097

(combined axial and bending loads) w 25 mm h 25 mm A95 31.25 mm


2

d equiv 20.198 mm

d equiv Csize 1.189 mm


Surface A 4.51

Csize 0.888 (machined)

b 0.265

MACHINE DESIGN - An Integrated Approach, 4th Ed.

6-37-2

Csurf Temperature Reliability 5.

S uta A MPa

Csurf 0.795

Ctemp 1 Creliab 0.897 (R = 90%)

Calculate the modified endurance limit. S ea Cload Csize Csurf Ctemp Creliab S'ea S ea 155.15 MPa

6.

Assuming no stress concentration, the fatigue factor of safety at the inner fiber for fully reversed loading is Nfa S ea Nfa 1.8

'a

Part (b) 7. 8. Calculate the unmodified endurance limit using equation (6.5b). S'eb 160 MPa

Calculate the endurance limit modification factors for a nonrotating rectangular beam. Load Size Cload 0.7 Section dims A95 0.05 w h d equiv A95 0.0766
0.097

(combined axial and bending loads) w 25 mm h 25 mm A95 31.25 mm


2

d equiv 20.198 mm

d equiv Csize 1.189 mm


Surface A 4.51 Csurf A Temperature Reliability 9. Ctemp 1 Creliab 0.897

Csize 0.888 (machined)

b 0.265

S utb MPa

Csurf 0.910

(R = 90%)

Calculate the modified endurance limit. S eb Cload Csize Csurf Ctemp Creliab S'eb S eb 81.21 MPa

10. Assuming no stress concentration, the fatigue factor of safety at the inner fiber for fully reversed loading is Nfb S eb Nfb 0.92

'a

MACHINE DESIGN - An Integrated Approach, 4th Ed.

6-38-1

PROBLEM 6-38
Statement: A 42-mm-dia steel shaft with a 19-mm transverse hole is subjected to a sinusoidal combined loading of = 100 MPa bending stress and steady torsion of 110 MPa. Find the safety factor for infinite life if S ut = 1000 MPa. Shaft diameter Hole diameter Infinite life Assumptions: Stresses given include stress concentration effects. Ground surfaces, temperature of 37C, and reliability of 50%. Solution: 1. See Mathcad file P0638. D 42 mm d 19 mm Max bending stress Min bending stress Steady torsion

Given:

max 100 MPa min 100 MPa m 110 MPa

Tensile strength S ut 1000 MPa

Calculate the alternating and mean von Mises stress components.

'a 'm
2.

max min
2 3 m

'a 100 MPa 'm 190.526 MPa

Calculate the unmodified endurance limit. S'e 0.5 S ut S'e 500 MPa

3.

Calculate the endurance limit modification factors for a rotating, round shaft. Load Size Surface Cload 1 Csize 1.189 A 1.58 D (combined bending and torsion)

mm

0.097

Csize 0.827 b 0.085 (ground)

Csurf Temperature Reliability 4.

Sut A MPa

Csurf 0.878

Ctemp 1 Creliab 1 (R = 50%)

Calculate the modified endurance limit. S e Cload Csize Csurf Ctemp Creliab S'e S e 363.37 MPa

5.

Assuming a Case 3 load line, determine the factor of safety against fatigue failure. S e S ut

Nf

'a S ut 'm S e

Nf 2.1

MACHINE DESIGN - An Integrated Approach, 4th Ed.

6-39-1

PROBLEM 6-39
Statement: A 42-mm-dia steel shaft with a 19-mm transverse hole is subjected to a combined loading of = 100 MPa bending stress and an alternating torsion of 110 MPa, which are 90 deg out of phase. Find the safety factor for infinite life if S ut = 1000 MPa. Shaft diameter Hole diameter Phase angle D 42 mm d 19 mm Max bending stress Min bending stress Max torsional stress Min torsional stress

Given:

max 100 MPa min 100 MPa max 110 MPa min 110 MPa

Tensile strength S ut 1000 MPa

90 deg

Assumptions: Stresses given include stress concentration effects. Ground surfaces, temperature of 37C, and reliability of 50%. Solution: 1. See Mathcad file P0639.

The dynamic loading is fully reversed so both mean stresses are zero. Calculate the alternating SEQA stress component using equation (6.23). Stress ratio Q 2

max max

Q 2.2
1

SEQAa

max
2

3 4

3 2

Q cos( 2 )

9 16

SEQAa 190.526 MPa

2.

Calculate the unmodified endurance limit. S'e 0.5 S ut S'e 500 MPa

3.

Calculate the endurance limit modification factors for a rotating, round shaft. Load Size Surface Cload 1 Csize 1.189 A 1.58 (combined bending and torsion)

mm
D

0.097

Csize 0.827 b 0.085 (ground)

Csurf Temperature Reliability 4.

Sut A MPa

Csurf 0.878

Ctemp 1 Creliab 1 (R = 50%)

Calculate the modified endurance limit. S e Cload Csize Csurf Ctemp Creliab S'e S e 363.37 MPa

5.

Assuming a Case 3 load line, determine the factor of safety against fatigue failure. Nf Se SEQAa Nf 1.9

MACHINE DESIGN - An Integrated Approach, 4th Ed.

6-40-1

PROBLEM 6-40
Statement: Redesign the roll support of Problem 6-8 to be like that shown in Figure P6-16. The stub mandrels insert to 10% of the roll length at each end. Design dimension a for an infinite-life factor of safety of 2. See Problem 6-8 for additional data. (a) The beam is a ductile material with S y = 450 MPa, S ut = 600 MPa (b) The beam is a cast-brittle material with S ut = 300 MPa Paper roll dimensions OD 1.50 m ID 0.22 m Lroll 3.23 m Ductile tensile strength S uta 600 MPa Brittle tensile strength S utb 300 MPa Roll density Design safety factor

Given:

984 kg m
Nfd 2

Assumptions: The paper roll's weight creates a concentrated load acting at the tip of the mandrel. The mandrel's root fits tightly in the stanchion so it can be modeled as a cantilever beam. The mandrel is machined, reliability is 99.999%, and it operates at room temperature. Solution: 1. See Figure 6-40 and Mathcad file P0640.

Determine the weight of the roll, the load on each support, and the length of the mandrel. Weight of paper W

OD ID Lroll g

FIGURE 6-40
Free Body Diagram used in Problem 6-40

W 53.9 kN Load on one mandrel Length of mandrel 2. F 0.5 W Lm 0.1 Lroll F 26.95 kN Lm 0.323 m

The maximum moment occurs at a section where the mandrel root leaves the stanchion and is Mmax F Lm Mmax 8.704 kN m

3. 4.

The dynamic loading is repeated from 0 to Mmax on each stress cycle, thus Mmin 0 kN m Part (a) - Calculate the alternating and mean components of the bending moment. Ma Mm Mmax Mmin 2 Mmax Mmin 2 Ma 4352 N m Mm 4352 N m S'e 0.5 S uta S'e 300 MPa

5. 6.

Determine the unmodified endurance limit.

Calculate the endurance limit modification factors for a nonrotating rectangular beam. Load Cload 1 A95 ( a ) 0.010462 a
2

Size

d equiv( a )

A95 ( a ) 0.0766

MACHINE DESIGN - An Integrated Approach, 4th Ed.

6-40-2
0.097

dequiv( a) Csize( a ) 1.189 mm


Surface A 4.51 Csurf A Temperature Reliability 7. Ctemp 1 Creliab 0.659

b 0.265

(machined)

S uta MPa

Csurf 0.828

(R = 99.999%)

Calculate the modified endurance limit. S e( a ) Cload Csize( a ) Csurf Ctemp Creliab S'e

8.

We can now determine the minimum required diameter, a. Using the distortion-energy failure theory with the modified Goodman diagram, the bending stress will also be the only nonzero principal stress, which will also be the von Mises stress. Assuming a Case 3 load line, use equation (6.18e) to determine the factor of safety. Guess a 100 mm. Bending stress Given Nfd =

M c I

a 64 32 M = M = 2 4 3 a a
3

a
32

S e( a ) S uta Ma S uta Mm S e( a ) a 92.421 mm a 94 mm

a Find ( a ) Round this up to the next higher even value Using this value of a, the values of the functions of a are: Csize( a ) 0.843 The realized safety factor is Nfa S e( a ) 137.942 MPa

a
32

S e( a ) S uta Ma S uta Mm S e( a ) S'e 0.4 S utb

Nfa 2.1

9.

Part (b) - Determine the unmodified endurance limit.

S'e 120 MPa

10. Calculate the endurance limit size modification factor for a nonrotating rectangular beam. Size A95 ( a ) 0.010462 a
2 0.097

d equiv( a )

A95 ( a ) 0.0766

dequiv( a) Csize( a ) 1.189 mm


11. Calculate the modified endurance limit. S e( a ) Cload Csize( a ) Csurf Ctemp Creliab S'e

MACHINE DESIGN - An Integrated Approach, 4th Ed.

6-40-3

12. We can now determine the minimum required diameter, a. Using the distortion-energy failure theory with the modified Goodman diagram, the bending stress will also be the only nonzero principal stress, which will also be the von Mises stress. Assuming a Case 3 load line, use equation (6.18e) to determine the factor of safety. Guess a 100 mm. Bending stress Given Nfd =

M c I

a 64 32 M = M = 2 4 3 a a
3

a
32

S e( a ) S utb Ma S utb Mm S e( a ) a 124.874 mm a 125 mm

a Find ( a ) Round this up to the next higher even value Using this value of a, the values of the functions of a are: Csize( a ) 0.82 The realized safety factor is Nfb S e( a ) 53.672 MPa

a
32

S e( a ) S utb Ma S utb Mm S e( a )

Nfb 2.0

MACHINE DESIGN - An Integrated Approach, 4th Ed.

6-41-1

PROBLEM 6-41
Statement: A 10-mm ID steel tube carries liquid at 7 MPa. The pressure varies periodically from zero to maximum. The steel has S ut = 400 MPa Determine the infinite-life fatigue safety factor for the wall if its thickness is: a) 1 mm, b) 5 mm. Tensile strength S ut 400 MPa

Given:

Assumption: The tubing is long therefore the axial stress is zero. The finish is machined, reliability is 99.999% and the tubing is at room temperature. Solution: See Mathcad file P0641. t 1 mm

(a) Wall thickness is 1.

From Problem 4-41, this is a thick wall cylinder and the maximum principal stresses are:

1maxa 38.82 MPa


2.

2maxa 0 MPa

3maxa 7.00 MPa

Calculate the minimum, maximum, alternating, and mean von Mises effective stress using equation (5.7c).

'min 0 MPa 'maxa 'aa 'ma


3. 4.

1maxa 1maxa 3maxa 3maxa

'maxa 42.752 MPa

'maxa 'min
2

'aa 21.376 MPa 'ma 21.376 MPa


S'e 0.5 S ut S'e 200 MPa

'maxa 'min
2

Calculate the unmodified endurance limit.

Calculate the endurance limit modification factors for axial loading. Load Size Surface Cload 0.7 Csize 1 A 4.51 (axial loading) (axial loading) b 0.265
b

( machined ) Csurf 0.922

Csurf Temperature Reliability 5.

Sut A MPa

Ctemp 1 Creliab 0.659 (R = 99.999%)

Calculate the modified endurance limit. S e Cload Csize Csurf Ctemp Creliab S'e S e 85.04 MPa

6.

Assuming a Case 3 load line, the factor of safety from equation (6.18e) is S e S ut

Na

'aa S ut 'ma S e

Na 3.3

MACHINE DESIGN - An Integrated Approach, 4th Ed.


(b) Wall thickness is 7. t 5 mm

6-41-2

From Problem 4-41, this is a thick wall cylinder and the principal stresses are:

1maxb 11.67 MPa


8.

2maxb 0 MPa

3maxb 7.00 MPa

Calculate the minimum, maximum, alternating, and mean von Mises effective stress using equation (5.7c).
2 2

'maxb

1maxb 1maxb 3maxb 3maxb

'maxb 16.336 MPa

'ab

'maxb 'min
2

'ab 8.168 MPa

'mb
9.

'maxb 'min
2

'mb 8.168 MPa

The endurance limit does not change from part a to b. Assuming a Case 3 load line, the factor of safety from equation (6.18e) is S e S ut

Nb

'ab S ut 'mb S e

Nb 8.6

MACHINE DESIGN - An Integrated Approach, 4th Ed.

6-42-1

PROBLEM 6-42
Statement: A cylindrical tank with hemispherical ends is required to hold 150 psi of pressurized air at room temperature. The pressure cycles from zero to maximum. The steel has S ut = 500 MPa. Determine the infinite-life fatigue safety factor if the tank diameter is 0.5 m with 1 mm wall thickness, and its length is 1 m. Tensile strength S ut 500 MPa

Given: Assumption: Solution: 1.

The finish is machined, reliability is 99.999% and the tank is at room temperature. See Mathcad file P0642.

From Problem 4-42, this is a thin wall cylinder and the maximum principal stresses are:

1max 259 MPa


2.

2max 129 MPa

3max 0 MPa

Calculate the minimum, maximum, alternating, and mean von Mises effective stress using equation (5.7c).

'min 0 MPa 'max 1max 1max 2max 2max


2 2

'max 224.301 MPa

'a

'max 'min
2

'a 112.151 MPa

'm
3. 4.

'max 'min
2

'm 112.151 MPa


S'e 0.5 S ut S'e 250 MPa

Calculate the unmodified endurance limit.

Calculate the endurance limit modification factors for axial loading. Load Size Surface Cload 0.7 Csize 1 A 4.51 (axial loading) (axial loading) b 0.265
b

( machined ) Csurf 0.869

Csurf Temperature Reliability 5.

Sut A MPa

Ctemp 1 Creliab 0.659 (R = 99.999%)

Calculate the modified endurance limit. S e Cload Csize Csurf Ctemp Creliab S'e S e 100.2 MPa

6.

Assuming a Case 3 load line, the factor of safety from equation (6.18e) is Nf S e S ut Nf 0.74

'a S ut 'm S e

MACHINE DESIGN - An Integrated Approach, 4th Ed.

6-43-1

PROBLEM 6-43
Statement: The paper rolls in Figure P6-17 are 0.9-m OD by 0.22-m ID by 3.23-m long and have a density of 984 kg/m3. The rolls are transfered from the machine conveyor (not shown) to the forklift truck by the V-linkage of the off-load station, which is rotated through 90 deg by an air cylinder. The paper then rolls onto the waiting forks of the truck. The forks are 38-mm thick by 100-mm wide by 1.2-m long and are tipped at a 3-deg angle from the horizontal and have S ut = 600MPa. Find the infinite-life safety factor for the two forks on the truck when the paper rolls onto it under two different conditions (state all assumptions): (a) The two forks are unsupported at their free end. (b) The two forks are contacting the table at point A. Tensile strength S ut 600 MPa Fork width Fork thickness w 100 mm t 38 mm
F t L fork

Given:

Assumptions: 1. The greatest bending moment will occur when the paper roll is at the tip of the fork for case (a) and when it is midway between supports for case (b). 2. Each fork carries 1/2 the weight of a paper roll. 3. For case (a), each fork acts as a cantilever beam (see Appendix B-1(a)). 4. For case (b), each fork acts as a beam that is built-in at one end and simply-supported at the other. 5. The forks are machined, the reliability is 90%, and they operate at room temperature. Solution: 1. See Figure 6-43 and Mathcad file P0643.

R1 Case (a), Cantilever Beam

M1

0.5 L fork t

L fork R1 R2

M2

Case (b), Fixed-Simply Supported Beam

FIGURE 6-43 From Problem 4-43, the maximum stresses in the forks are: Case (a) Case (b) Both cases
Free Body Diagrams used in Problem 6-43

maxa 464.8 MPa maxb 87.2 MPa min 0 MPa

at the base of the fork. also at the base of the fork.

Since there are no other stress components present, these are also the maximum principal stresses and the von Mises stresses. This is a repeated load problem.

Case (a) 2. The dynamic loading is repeated from 0 to 1 for each paper roll that is transfered. The alternating and mean components of the von Mises stress are: Alternating von Mises stress Mean von Mises stress 3. 4. Calculate the unmodified endurance limit.

'a 0.5 maxa min 'm 0.5 maxa min


S'e 0.5 S ut

'a 232.4 MPa 'm 232.4 MPa


S'e 300 MPa

Calculate the endurance limit modification factors for a nonrotating rectangular beam. Load Cload 1

MACHINE DESIGN - An Integrated Approach, 4th Ed.


A95 0.0766

6-43-2

Size

A95 0.05 w t

d equiv
0.097

d equiv Csize 0.869 in


Surface A 4.51

Csize 0.814

b 0.265
b

(machined)

Csurf Temperature Reliability 5.

Sut A MPa

Csurf 0.828

Ctemp 1 Creliab 0.897 (R = 90%)

Calculate the modified endurance limit. S e Cload Csize Csurf Ctemp Creliab S'e S e 181.357 MPa

6.

Assuming a Case 3 load line, use equation (6.18e) to determine the factor of safety. Case a Nfa S e S ut Nfa 0.60

'a S ut 'm S e

Case (b) 7. The dynamic loading is repeated from 0 to 1 for each paper roll that is transfered. The alternating and mean components of the von Mises stress are: Alternating von Mises stress Mean von Mises stress 8.

'a 0.5 maxb min 'm 0.5 maxb min

'a 43.6 MPa 'm 43.6 MPa

Assuming a Case 3 load line, use equation (6.18e) to determine the factor of safety. Case a Nfb S e S ut Nfb 3.2

'a S ut 'm S e

MACHINE DESIGN - An Integrated Approach, 4th Ed.

6-44-1

PROBLEM 6-44
Statement: Determine a suitable thickness for the V-links of the off-loading station of Figure P6-17 to limit th deflections at the tips to 10 mm in any position during their rotation. Two V-links support the roll at the 1/4 and 3/4 points along the roll's length and that each of the V-links is 10 cm wide by 1 m long. What is the infinite-life safety factor when designed to limit deflection as above? S ut = 600 MPa. See Problem 4-43 for more information. Roll OD Roll ID Roll length Roll density OD 0.90 m ID 0.22 m Lroll 3.23 m
3

Given:

Arm width Arm length Max tip deflection Mod of elasticity Tensile strength

wa 100 mm La 1000 mm

tip 10 mm
E 207 GPa S ut 600 MPa

984 kg m

Assumptions: 1. The maximum deflection on an arm will occur just after it begins the transfer and just before it completes it, i.e., when the angle is either zero or 90 deg., but after the tip is no longer supported b the base unit. 2. At that time the roll is in contact with both arms ("seated" in the V) and will remain in that state throughout the motion. When the roll is in any other position on an arm the tip will be supported. 3. The arm can be treated as a cantilever beam with nonend load. 4. A single arm will never carry more than half the weight of a roll. 5. The pipe to which the arms are attached has OD = 160 mm. 6. The V-links are machined, reliability is 90%, and they operate at room temperature. Solution: 1. See Figure 6-44 and Mathcad file P0644.

Determine the weight of the roll and the load on each V-arm. W

OD ID Lroll g

W 18.64 kN F 9.32 kN

F 0.5 W 2.

From Appendix B, Figure B-1, the tip deflection of a cantilever beam with a concentrated load located at a distance a from the support is ymax = F a
2

6 E I

( a 3 L)

where L is the beam length and I is the cross-section moment of inertia. In this case I= 3. w a t a 12 FIGURE 6-44
Free Body Diagram used in Problem 6-44 1 3

Setting ymax = tip and a 370 mm,

substituting for I and solving for ta

2 F a2 3 La a ta E tip wa
Let the arm thickness be

ta 31.889 mm ta 32 mm

MACHINE DESIGN - An Integrated Approach, 4th Ed.


4.

6-44-2

The maximum bending stress in the arm will be at its base where it joins the 160-mm-dia pipe. The bending moment, moment of inertia, and distance to the outside fiber at that point are: Bending moment Mmax F a Mmin 0 kN m Ma Mm Distance to n.a. 1 2 1 2 Mmax Mmin Mmax Mmin Ma 1.725 kN m Mm 1.725 kN m c 16 mm I 2.731 10 mm
5 4

Mmax 3.449 kN m

c 0.5 t a I wa ta 12 M a c I Mm c I
3

Moment of inertia

Nom tensile stress

anom mnom

anom 101 MPa mnom 101 MPa

5.

Determine the stress concentration factors. Figure E-10 comes the closest to our situation. Assuming that the effective D/d-ratio is 2 and r/d is about 0.25, Kt 1.4. For a material with ksi 10 psi and, for the assumed value of r/d, The notch sensitivity factor is
3

S ut 87 ksi r 0.25 ta q 1 1 a r

a 0.073 in r 8 mm q 0.885

and the fatigue stress concentration factor is Kf 1 q Kt 1 6. Kf 1.35

Assuming that Kfm Kf , the actual alternating and mean components of stress at the point where the V-link meets the central hub are

a Kf anom m Kfm mnom


7.

a 136.8 MPa m 136.8 MPa

Since there are no other nonzero stress components at this point on the top of the arm, the von Mises stresses are and 'a a 'm m Determine the modified material strength. Unmodified endurance limit Load Size S'e 0.5 S ut Cload 1 A95 0.05 wa ta A95 160 mm
2

8.

S'e 300 MPa

MACHINE DESIGN - An Integrated Approach, 4th Ed.


A95 0.0766
0.097

6-44-3

d eq

d eq 45.703 mm

deq Csize 1.189 mm


Surface Csurf

Csize 0.821
0.265

Sut 4.51 MPa

Csurf 0.828

Temperature Reliability Endurance limit 9.

Ctemp 1 Creliab 0.897 S e Cload Csize Csurf Ctemp Creliab S'e (R = 90%) S e 182.8 MPa

Calculate the factor of safety. Using the distortion energy theory and the modified Goodman theory, the fatigue factor of safety for a V-link thickness of ta 32 mm is Nf S ut S e Nf 1.0

'a S ut 'm S e

MACHINE DESIGN - An Integrated Approach, 4th Ed.

6-45-1

PROBLEM 6-45
Statement: Determine the infinite-life fatigue safety factor based on the tension load on the air-cylinder rod in Figure P6-17. The tension load cycles from zero to maximum (compression loads below the critical buckling load will not affect the fatigue life). The crank arm that it rotates is 0.3 m long and the rod has a maximum extension of 0.5 m. The 25-mm-dia rod is solid steel with S ut = 600 MPa. State all assumptions. Paper roll dimensions OD 0.90 m ID 0.22 m Lroll 3.23 m Roll density
3

Given:

Rod diameter Tensile strength

d 25 mm S ut 600 MPa

984 kg m

Assumptions: 1. The maximum force in the cylinder rod occurs when the transfer starts. 2. The cylinder and rod make an angle of 5.5 deg to the horizontal at the end of transfer. 3. The crank arm is 300 mm long and is 45 deg from vertical at the end of transfer. 4. The finish is machined, reliability is 90%, and the cylinder operates at room temperature. 5. The cylinder rod is fully retracted at the start of the transfer. At the end of the transfer it will have extended 500 mm from its initial position. Solution: 1. See Figure 6-45 and Mathcad file P0645.
y

Determine the weight of the roll on the forks. W

OD ID Lroll g

W 18.64 kN 2. From the assumptions and Figure 6-45, the x and y distances from the origin to point A are, Rax 300 cos( 45 deg) mm Ray 300 sin( 45 deg) mm Rax 212.132 mm Ray 212.132 mm 3. From Figure 6-45, the x distance from the origin to point where W is applied is, Rwx 4. OD 2 Rwx 450 mm FIGURE 6-45
Free Body Diagram at End of Transfer for V-link of Problem 6-45
212.1 A F Rx x Ry 5.5

450.0 W

212.1

Sum moments about the pivot point and solve for the tensile force in the cylinder rod.

W Rwx Fmax Rax sin( 5.5 deg) Fmax Ray cos( 5.5 deg) = 0 Fmax W Rwx Ray cos( 8 deg) Rax sin( 8 deg) Fmin 0 kN Fmax 35.017 kN

tension

5.

Assume that the dynamic load is repeated so

MACHINE DESIGN - An Integrated Approach, 4th Ed.


6. Determine the alternating and mean components of axial stress in the rod. Area A Fa Fm

6-45-2

d
4

A 490.874 mm Fa 17.508 kN Fm 17.508 kN

Alternating load

Fmax Fmin 2 Fmax Fmin 2 Fa A Fm A Nf = S e S ut

Mean load

Alternating stress

a 35.668 MPa

Mean stress

m 35.668 MPa

7.

For fluctuating loading, the factor of safety is

'a S ut 'm S e

8.

Since the uniform axial stress is the only stress component present,

'a = a
9.

and

'm = m
S'e 0.5 S ut S'e 300 MPa

Calculate the unmodified endurance limit.

10. Calculate the endurance limit modification factors for an axial bar. Load Size Surface Cload 0.7 Csize 1 A 4.51 b 0.265 (axial loading) (axial loading) (machined)

Csurf A Temperature Reliability 11. Calculate the modified endurance limit. S e Cload Csize Csurf Ctemp Creliab S'e Ctemp 1

Sut MPa

Csurf 0.828

Creliab 0.897

(R = 90%)

S e 155.95 MPa

12. Determine the factor of safety against fatigue failure for the assumptions made. Nf S e S ut Nf 3.5

a S ut m S e

MACHINE DESIGN - An Integrated Approach, 4th Ed.

6-46-1

PROBLEM 6-46
Statement: The V-links of Figure P6-17 are rotated by the crank arm through a shaft that is 60 mm dia by 3.23 m long. Determine the maximum torque applied to this shaft during motion of the V-linkage and find the infinite-life fatigue safety factor for the shaft if its S ut = 600 MPa. See Problem 6-43 for more information.
y

Given:

Tensile strength S ut 600 MPa Shaft diameter d 60 mm

Assumptions: 1. The greatest torque will occur when the link is horizontal and the paper roll is located as shown in Figure P6-17 or Figure 6-46. 2. The V-links are machined, use a reliability of 90%, and operate at room temperature. Solution: 1. See Figure 6-46 and Mathcad file P0646.

W T

From Problem 4-46, the maximum torsional stress in the shaft is

max 197.88 MPa


60-mm-dia shaft

Ry 450.0

2.

Although not exactly true, assume that the load is fully reversed, then the minimum torque is min 197.88 MPa

FIGURE 6-46
Free Body Diagram used in Problem 6-46

3.

Calculate the alternating component of the torsional stress in the shaft. Alternating stress

max min
2

a 197.88 MPa

4.

Convert this to the corresponding component of the von Mises stress. Alternating stress

'a

3 a

'a 342.738 MPa


S'e 0.5 S ut S'e 300 MPa

5. 6.

Calculate the unmodified endurance limit using equation (6.5a).

Calculate the endurance limit modification factors for a solid, round steel shaft. Load Size Cload 1 d Csize 1.189 mm A 4.51
0.097

Csize 0.799

Surface

b 0.265
b

(machined)

Csurf Temperature Reliability

Sut A MPa

Csurf 0.828

Ctemp 1 Creliab 0.897 (R = 90%)

MACHINE DESIGN - An Integrated Approach, 4th Ed.


7. Calculate the modified endurance limit. S e Cload Csize Csurf Ctemp Creliab S'e S e 178.07 MPa

6-46-2

8.

Calculate the factor of safety for the shaft. Se

Nf

'a

Nf 0.52

MACHINE DESIGN - An Integrated Approach, 4th Ed.

6-47-1

PROBLEM 6-47
Statement: Determine the maximum forces on the pins at each end of the air cylinder of Figure P6-17. Determine the infinite-life fatigue safety factor in these pins if they are 30-mm dia and in single shear. S ut = 600 MPa. See Problem 6-43 for more information. Pin diameter d 30 mm Tensile strength S ut 600 MPa

Given:

Assumptions: 1. The maximum force in the cylinder rod occurs when the transfer starts. 2. The dynamic loading is fully reversed. 3. The finish is machined, reliability is 90%, and the pins are at room temperature. Solution: See Figure 6-47 and Mathcad file P0647.
y

W Rx x 212.1 A F 212.1 450.0 8

Ry

FIGURE 6-47
Free Body Diagram at Start of Transfer for V-link of Problem 6-47

1. 2. 3. 4.

From Problem 4-47 the maximum shear stress on the pins is

max 65.7 MPa


3 max 'a 113.796 MPa

This is the only stress com- ponent so the alternating von Mises stress is 'a Calculate the unmodified endurance limit. S'e 0.5 S ut

S'e 300 MPa

Calculate the endurance limit modification factors for a nonrotating direct shear. Size Load A95

d
4

A95 706.858 mm

Cload 1 d equiv A95 0.0766


0.097

d equiv 96.062 mm

d equiv Csize 1.189 mm


Surface A 4.51

Csize 0.764 (machined)

b 0.265

MACHINE DESIGN - An Integrated Approach, 4th Ed.

6-47-2

Csurf A Temperature Reliability 5. Ctemp 1

Sut MPa

Csurf 0.828

Creliab 0.897

(R = 90%)

Calculate the modified endurance limit. S e Cload Csize Csurf Ctemp Creliab S'e S e 170.12 MPa

6. Assuming a Case 3 load line, use equation (6.14) to determine the factor of safety. Nf Se Nf 1.5

'a

MACHINE DESIGN - An Integrated Approach, 4th Ed.

6-48-1

PROBLEM 6-48
Statement: Figure P6-18 shows an exerciser for a 100-kg wheelchair racer. The wheelchair has 65-cm-dia drive wheels separated by a 70-cm track width. Two free-turning rollers on bearings support the rear wheels. The lateral movement of the chair is limited by the flanges. Design the 1-m-lomg rollers as hollow tubes of aluminum (select alloy) to minimize the height of the platform and also limit the roller deflections to 1 mm in the worst case. Specify suitable sized steel axles to support the tubes on bearings. Calculate the fatigue safety factors at a life of 5E8 cycles. Mass of chair M 100 kg Wheel diameter d w 650 mm Track width Roller length T 700 mm Lr 1000 mm Maximum deflection Modulus elasticity Aluminum Steel

Given:

1 mm
Ea 71.7 GPa Es 207 GPa

Assumptions: 1. The CG of the chair with rider is sufficiently close to the rear wheel that all of the weight is taken by the two rear wheels. 2. The small camber angle of the rear wheels does not significantly affect the magnitude of the forces on the rollers. 3. Both the aluminum roller and the steel axle are simply supported. The steel axles that support the aluminum tube are fixed in the mounting block and do not rotate. The aluminum tube is attached to them by two bearings (one on each end of the tubes, one for each axle). The bearings' inner race is fixed, and the outer race rotates with the aluminum tube. Each steel axle is considered to be loaded as a simply supported beam. Their diameter must be less than the inner diameter of the tubes to fit the roller bearings between them. 4. All surfaces are machined, reliability is 90%, and parts are at room temperature. Solution: 1. 2. See Figures 6-48 and Mathcad file P0648.

W/2

F
FIGURE 6-48A

Free Body Diagram of One Wheel used in Problem 6-48

Calculate the weight of the chair with rider. Weight of chair

W M g

W 980.7 N

Calculate the forces exerted by the wheels on the rollers (see Figure 5-48A). From the FBD of a wheel, summing vertical forces 2 F cos( ) Let W 2 =0 then F W 4 cos( ) F 260.9 N

20 deg

3.

The worst condition (highest moment at site of a stress concentration) will occur when the chair is all the way to the left or right. Looking at the plane through the roller that includes the forces exerted by the wheels (the FBD is shown in Figure 6-48B) the reactions R1 and R2 come from the bearings, which are inside the hollow roller and are, themselves, supported by the steel axle. Solving for the reactions. Let the distance from R1 to F be a 15 mm

4.

MACHINE DESIGN - An Integrated Approach, 4th Ed.

6-48-2

M1 Fy
R2

R2 Lr F ( a T ) F a = 0 R1 2 F R2 = 0 F (2 a T ) Lr R2 190.5 N

700

15

R2 1000

R1 2 F R2 5.

R1 331.3 N

R1

The maximum bending moment will be at the right-hand load and will be Mrmax R2 Lr ( a T ) Lr T 2

FIGURE 6-48B
Free Body Diagram of One Tube used in Problem 6-48

Mrmax 54.3 N m

Note, if the chair were centered on the roller the maximum moment would be Mc F Mc 39.1 N m

and this would be constant along the axle between the two loads, F. 6. Note that the bearing positions are fixed regardless of the position of the chair on the roller. Because of symmetry, Ra1 R1 Ra2 R2 7. Ra1 331.3 N Ra2 190.5 N
R a1 1130 R a2 65 R1 1000 R2

The maximum bending moment occurs at R1 and is for b 65 mm Mamax Ra1 b Mamax 21.5 N m

FIGURE 6-48C
Free Body Diagram of One Axle used in Problem 6-48

8. 9.

Determine a suitable axle diameter. Let the factor of safety against yielding in the axle be Nsa 3 Tentatively choose a low-carbon steel for the axle, say AISI 1020, cold rolled with S y 393 MPa

10. At the top of the axle under the load R1 there is only a bending stress, which is also the von Mises stress. Set this stress equal to the yield strength divided by the factor of safety.

' =

32 Mamax

Sy Nsa
1

d a

Solving for the axle diameter, d a Let the axle diameter be

d a

32 Nsa Mamax S y

d a 11.875 mm

d a 15 mm

made from cold-rolled AISI 1020 steel.

11. Suppose that bearing 6302 from Chapter 10, Figure 10-23, page 684 is used. It has a bore of 15 mm and an OD of 42 mm. Thus, the inside diameter of the roller away from the bearings where the moment is a maximum will be d i 40 mm. This will provide a 1-mm shoulder for axial location of the bearings.

MACHINE DESIGN - An Integrated Approach, 4th Ed.


12. Design for a factor of safety of Nra 3. Tentatively choose 2024-T4 aluminum with S ut 440 MPa and S'e5E8 138 MPa 13. A point on the outside diameter of the roller will see completely reversed bending, which will also be the only nonzero principal stress. Thus,
F 15 1000 F

6-48-3

150 F

700 F

FIGURE 6-48D
Free Body Diagram of Roller with Chair in the Center.

x =

Kf Mrmax Z

= ' =

Se Nfr

where Kf is the fatigue stress concentration at the shoulder and S e is the modified endurance limit. 14. Tentatively choose (these values arrived at by iteration): Outside diameter Shoulder diameter Fillet radius d o 45 mm D 54 mm r 5 mm

15. Determine the fatigue stress concentration factor. From Figure E-2 and Table 6-6 for r do 0.111
0.21796

D do

1.2

r Kt 0.97098 d o S ut 6.38 10 psi q 1 1 0.102 r in


4

Kt 1.57

q 0.813

Kf 1 q Kt 1 16. Calculate the alternating von Mises stress component. I

Kf 1.46

4 4 do di 64

I 7.563 10 mm c 22.5 mm

c 0.5 d o

'a

Kf Mrmax c I

'a 23.6 MPa

MACHINE DESIGN - An Integrated Approach, 4th Ed.


17. Destermine the endurance limit at 5E8 cycles Load factor Cload 1

6-48-4

Size factor

do Csize 1.189 mm
Csurf 4.51

0.097

Csize 0.822
0.265

Surface factor as machined Reliability at 90% Modified endurance limit

Sut MPa

Csurf 0.899

Creliab 0.897

S e5E8 Cload Csize Csurf Creliab S'e5E8 18. Determine the factor of safety for repeated loading. Nfr S e5E8

S e5E8 91.44 MPa

'a

Nfr 3.87

19. The maximum deflection of the roller will occur when the chair is in the center of the roller. For this case the reactions are both equal to the loads, F. Using Figure B-2(a) in Appendix B, the maximum deflection is at the center of the roller and is for a 150 mm E 71.7 GPa ymax 2 a 3 3 x ( x a ) 6 E I Lr a 2 2 a 3 a Lr 2 Lr x Lr F 1 x 0.5 Lr x 500 mm

ymax 0.875 mm

This design meets the deflection requirement and has a reasonable factor of safety against fatigue failure while allowing sufficient space for the bearings. DESIGN SUMMARY Axle Material Diameter Length AISI 1020 steel, cold-rolled d a 15 mm 1220 mm Roller Material Outside diameter Inside diameter Shoulder dia Fillet radius Length Center line spacing 2024-T4 aluminum d o 45 mm d i 40 mm D 54 mm r 5 mm 1040 mm c d w d o sin( ) c 238 mm

MACHINE DESIGN - An Integrated Approach, 4th Ed.

6-49-1

PROBLEM 6-49
Statement:

_____

Figure P6-19 shows a machined pivot pin that is press-fit into part A and is slip fit in part B. If F = 100 lb, l = 2 in, and d = 0.5 in, what is the pin's safety factor against fatigue when made of SAE 1020 cold-rolled steel? The loading is fully reversed and a reliability of 90% is desired. There is a bending stress concentration factor Kt = 1.8 at the section where the pin leaves part A on the right-hand side. Applied force Total length, l Pin dia F 100 lbf l 2.00 in d 0.5 in Material strength S y 57 ksi Beam length L 0.5 l S ut 68 ksi

Given:

Assumptions: 1. Since there is a slip fit between the pin and part B, part B offers no resistance to bending of the pin and, since the pin is press-fit into part A, it can be modeled as a cantilever beam of length l/2. 2. Part B distributes the concentrated force F so that, at the pin, it is uniformly distributed over the exposed length of the pin. Solution: 1. See Mathcad file P0649. Calculate the intensity of the uniformly distributed load acting over the length of the pin. w 2. F L w 100.0 lbf in

A cantilever beam with uniform loading is shown in Figure B-1(b) in Appendix B. In this case, the dimension a in the figure is zero. As shown in the figure, when a = 0, the maximum bending moment occurs at the support and is w L 2
2

Mmax 3.

Mmax 50.00 lbf in

Calculate the moment of inertia and distance to the extreme fiber of the pin. The nominal alternating bending stress in the beam is then found using equation 4.11b. I

d
64

I 3.068 10 c 0.250 in

in

c 0.5 d

anom
4.

Mmax c I

anom 4074 psi

From Table 6-6, the Neuber constant for S ut 68 ksi is


1

a 0.096 in 5.

a 0.096 in

Using equation 6-13, the notch sensitivity for r 0.5 d is q 1 1 a r q 0.839

6.

The fatigue stress-concentration factor for Kt 1.8, from equation 6.11b, is Kf 1 q Kt 1 Kf 1.67

MACHINE DESIGN - An Integrated Approach, 4th Ed.

6-49-2

7.

Because the stress state in the pin is simple, uniaxial stress, the alternating principal stress is equal to the alternating tensile stress and is also equal to the alternating von Mises stress. Thus,

a Kf anom ' a
8. 9. Calculate the unmodified endurance limit.

a 6.81 ksi

S'e 0.5 S ut

S'e 34.0 ksi

Calculate the endurance limit modification factors for a non rotating round pin in bending. Load Size Cload 1 A95 0.010462 d A95 0.0766
2

A95 2.615 10

in

d equiv

d equiv 0.185 in

Surface

This is less than the lower limit in equation 6.7b, so use Csize 1 (machined) A 2.7 b 0.265 Csurf A

S ut ksi

Csurf 0.883

Temperature Reliability

Ctemp 1 Creliab 0.897 (R = 90%)

10. Calculate the modified endurance limit. S e Cload Csize Csurf Ctemp Creliab S'e S e 26.9 ksi

11. Using equation 6.14, calculate the factor of safety against a fatigue failure for this case of fully reversed bending. Se

Nf

'

Nf 4.0

MACHINE DESIGN - An Integrated Approach, 4th Ed.

6-50-1

PROBLEM 6-50
Statement:

_____

Figure P6-19 shows a machined pivot pin that is press-fit into part A and is slip fit in part B. If F = 100 N, l = 50 mm, and d = 16 mm, what is the pin's safety factor against fatigue when made of class 50 cast iron? The loading is fully reversed and a reliability of 90% is desired. There is a bending stress concentration factor Kt = 1.8 at the section where the pin leaves part A on the right-hand side.

Given:

Applied force Total length, l

F 100 N l 50 mm

Tensile strength S ut 359 MPa Beam length L 0.5 l Pin dia d 16 mm

Assumptions: 1. Since there is a slip fit between the pin and part B, part B offers no resistance to bending of the pin and, since the pin is press-fit into part A, it can be modeled as a cantilever beam of length l/2. 2. Part B distributes the concentrated force F so that, at the pin, it is uniformly distributed over the exposed length of the pin. Solution: 1. See Mathcad file P0650.

Calculate the intensity of the uniformly distributed load acting over the length of the pin. w F L w 4.0 N mm

2.

A cantilever beam with uniform loading is shown in Figure B-1(b) in Appendix B. In this case, the dimension a in the figure is zero. As shown in the figure, when a = 0, the maximum bending moment occurs at the support and is Mmax w L 2
2

Mmax 1250 N mm

3.

Calculate the moment of inertia and distance to the extreme fiber of the pin. The nominal alternating bending stress in the beam is then found using equation 4.11b. I

d
64

I 3.217 10 mm c 8.000 mm

c 0.5 d

anom
4.

Mmax c I

anom 3.108 MPa

Since this is a brittle material, so the full value of the geometric stress concentration factor Kt 1.8 will be applied to the nominal stress using equation 4.31.

5.

Because the stress state in the pin is simple, uniaxial stress, the alternating principal stress is equal to the alternating tensile stress and is also equal to the alternating von Mises stress. Thus,

a Kt anom ' a
6. 7. Calculate the unmodified endurance limit.

a 5.60 MPa

S'e 0.5 S ut

S'e 179.5 MPa

Calculate the endurance limit modification factors for a non rotating round pin in bending. Load Cload 1

MACHINE DESIGN - An Integrated Approach, 4th Ed.

6-50-2

Size

A95 0.010462 d A95 0.0766

A95 2.678 mm

d equiv

d equiv 5.913 mm

Surface

This is less than the lower limit in equation 6.7b, so use Csize 1 (machined) A 4.51 b 0.265

Csurf Temperature Reliability 8.

Sut A MPa

Csurf 0.949

Ctemp 1 Creliab 0.897 (R = 90%)

Calculate the modified endurance limit. S e Cload Csize Csurf Ctemp Creliab S'e S e 152.7 MPa

9.

Using equation 6.14, calculate the factor of safety against a fatigue failure for this case of fully reversed bending Nf Se Nf 27

'

MACHINE DESIGN - An Integrated Approach, 4th Ed.

6-51-1

PROBLEM 6-51
Statement:

_____

A component in the shape of a large sheet is to be fabricated from 7075-T651 aluminum, which has a fracture toughness Kc = 24.2 MPa-m0.5 and a tensile yield strength of 495 MPa. Determine the number of loading cycles that can be endured if the nominal stress varies from 0 to one half the yield strength and the initial crack had a total length of 1.2 mm. The values of the coefficient and exponent in equation 6.4 for this material are A = 5 x 10 -11 (mm/cyc) and n = 4. cycle 1 Fracture toughness Yield strength Initial crack length Coeff. and exponent Kc 24.2 MPa m S y 495 MPa lo 1.2 mm A 5 10
11 0.5

Units: Given:

mm cycle

n 4

Solution: 1.

See Mathcad file P0651.

Calculate the minimum and maximum nominal stresses based on the yield strength and the stress level given in the problem statement.

min 0 MPa max


2. Sy 2

max 247.5 MPa

Determine the value of the geometry factor from the discussion in Section 5.3 for a plate with a central crack.

1
3. Using equation 5.14b, calculate the critical crack length for this material at the maximum stress condition.

Kc a c max
1 4.

a c 3.0 mm

Calculate the initial crack half-length. a o 0.5 l o a o 0.60 mm

5.

Using equations 6.3, write the stress intensity factor range as a function of crack half-length.

K ( a ) a max min
6. Integrate equation 6.4 to find the number of cycles to failure. 1 Nc A
ac

K ( a ) 0.5 MPa m

da

Nc 7.2 10 cycle

ao

MACHINE DESIGN - An Integrated Approach, 4th Ed.

6-52-1

PROBLEM 6-52
Statement:

_____

A component in the shape of a large sheet is to be fabricated from 4340 steel, which has a fracture toughness Kc = 98.9 MPa-m0.5. The sheets are inspected for crack flaws after fabrication, but the inspection device cannot detect flaws smaller than 5 mm. Determine the minimum thickness required for the sheet to have a minimum cycle life of 10 6 cycles (using fracture-mechanics criteria if its width is 400 mm and the load normal to the crack varies from 20 to 170 kN. The values of th coefficient and exponent in equation 6.4 for this material are A = 4 x 10 -9 (mm/cyc) and n = 3. cycle 1 Fracture toughness Width of sheet Load Coeff. and exponent Cycles to failure Kc 98.9 MPa m W 400 mm Fmin 20 kN A 4 10
6 9 0.5

Units: Given:

Fmax 170 kN
1

mm cycle

n 3

Nf 10 cycle

Assumption: Solution: 1.

The initial total crack length is lo 5 mm See Mathcad file P0652.

Write equations for the minimum and maximum nominal stresses as a function of the unknown thickness.

min( t)
2.

Fmin W t

max( t)

Fmax W t

Determine the value of the geometry factor from the discussion in Section 5.3 for a plate with a central crack.

1
3. Using equation 5.14b, write an equation for the critical crack length as a function of t for this material at the maximum stress condition.

Kc a c( t) max( t)
1 4. a o 0.5 l o 5.

Calculate the initial crack half-length. a o 2.50 mm

Using equations 6.3, write the stress intensity factor range as a function of crack half-length and sheet thickness

K ( a t) a max( t) min( t)
6. Use equation 6.4 to find the minimum sheet thickness. First, guess a value: t 4 mm Nf A =
ac( t )

Given

K ( a t) 0.5 MPa m

da

t Find ( t)

t 3.2 mm

ao

MACHINE DESIGN - An Integrated Approach, 4th Ed.

6-53-1

PROBLEM 6-53
Statement:

_____

A closed, thin-wall cylinder is made from an aluminum alloy that has a fracture toughness of 38 MPa-m0.5 and has the following dimensions: length = 200 mm, OD = 84 mm, and ID = 70 mm. A 2.8-mm-deep semicircular crack is discovered on the inner diameter away from the ends, oriented along a line parallel to the cylinder axis. If the cylinder is repeatedly pressurized from 0 to 75 MPa, how many pressure cycles can it withstand? The values of the coefficient and exponent in equation 6.4 for this material are A = 5 x 10 -12 (mm/cyc) and n = 4. (Hint: the value of the geometry factor for a semicircular surface flaw is = 2/and the crack grows in the radial direction). cycle 1 Fracture toughness Initial crack depth Cylinder dimensions Internal pressure Coeff. and exponent Geometry factor Kc 38 MPa m L 200 mm p min 0 MPa
12 0.5

Units: Given:

a o 2.8 mm OD 84 mm ID 70 mm p max 75 MPa


1

A 5 10 mm cycle 0.6367

n 4

Solution: 1.

See Mathcad file P0653.

Calculate the nominal hoop stress (tangential direction, normal to cylinder axis) using equation 4.49a based on the pressure levels given in the problem statement. r 0.5 ID r 35.0 mm r t r t t 0.5 ( OD ID) t 7.0 mm

min p min

min 0.0 MPa max 375.0 MPa

max p max
2.

Using equation 5.14b, calculate the critical crack length for this material at the maximum stress condition.

Kc a c max
1 a c t 3.

a c 8.1 mm

However, since this is larger than the wall thickness, failure will occur when the crack reaches the OD so a c 7.0 mm

Using equations 6.3, write the stress intensity factor range as a function of crack depth.

K ( a ) a max min
4. Integrate equation 6.4 to find the number of cycles to failure. 1 Nc A
ac

K ( a ) 0.5 MPa m

da

Nc 1.34 10 cycle

ao

MACHINE DESIGN - An Integrated Approach, 4th Ed.

6-54-1

PROBLEM 6-54
Statement:

_____

A non rotating, hot-rolled, steel beam has a channel section with h = 64 mm and b = 127 mm. It is loaded in repeated bending with the neutral axis through the web. Determine its corrected fatigue strength with 90% reliability if it is used in an environment that has a temperature that is below 450C and has an ultimate tensile strength of 320 MPa. Ultimate tensile strength Reliability Dimensions S ut 320 MPa R 0.90 h 64 mm b 127 mm

Given:

Solution: 1.

See Mathcad file P0654.

Calculate the uncorrected endurance limit using equation 6.5a. S'e 0.5 S ut S'e 160.0 MPa

2.

Determine the loading factor from equation 6.7a. Cload 1

3.

Determine the size factor from equations 6.7b and 6.7d, and Figure 6-25. Area stressed above 95% of max A95 0.05 b h d equiv
0.097

A95 406.4 mm

Equivalent diameter

A95 0.0766

d equiv 72.8 mm

Size factor

d equiv Csize 1.189 mm

Csize 0.784

4.

Determine the surface factor from equation 6.7e and Table 6-3. From Table 6-3 A 57.7 Csurf A b 0.718

Surface factor

Sut MPa

Csurf 0.917

5. 6. 7.

Determine the temperature factor from equation 6.7f. Since T < 450C, Ctemp 1. Determine the reliability factor from Table 6-4, Creliab 0.897. Using equation 6.6, calculate the corrected endurance limit. S e Cload Csize Csurf Ctemp Creliab S'e S e 103.3 MPa

MACHINE DESIGN - An Integrated Approach, 4th Ed.

6-55-1

PROBLEM 6-55
Statement:

_____

A non rotating, machined, steel rod has a round section with d = 50 mm. It is loaded with a fluctuating axial force. Determine its corrected fatigue strength with 99% reliability if it is used in an environment that has a temperature below 450C and has an ultimate tensile strength of 480 MPa. Ultimate tensile strength Reliability Dimensions S ut 480 MPa R 0.99 d 50 mm

Given:

Solution: 1.

See Mathcad file P0655.

Calculate the uncorrected endurance limit using equation 6.5a. S'e 0.5 S ut S'e 240.0 MPa

2.

Determine the loading factor from equation 6.7a. Cload 0.70

3. 4.

The size factor for an axially loaded member is Csize 1 Determine the surface factor from equation 6.7e and Table 6-3. From Table 6-3 A 4.51 b 0.265
b

Surface factor

Csurf

Sut A MPa

Csurf 0.878

5. 6. 7.

Determine the temperature factor from equation 6.7f. Since T < 450C, Ctemp 1. Determine the reliability factor from Table 6-4, Creliab 0.814. Using equation 6.6, calculate the corrected endurance limit. S e Cload Csize Csurf Ctemp Creliab S'e S e 120.1 MPa

MACHINE DESIGN - An Integrated Approach, 4th Ed.

6-56-1

PROBLEM 6-56
Statement:

_____

A non rotating, cold-drawn, steel rod has a round section with d = 76 mm. It is loaded in repeated torsion. Determine its corrected fatigue strength with 99% reliability if it is used in an environment that has a temperature of 500C and has an ultimate tensile strength of 855 MPa. Ultimate tensile strength Reliability Dimensions Temperature S ut 855 MPa R 0.99 d 76 mm T 500 C

Given:

Solution: 1.

See Mathcad file P0656.

Calculate the uncorrected endurance limit using equation 6.5a. S'e 0.5 S ut S'e 427.5 MPa

2.

Determine the loading factor from equation 6.7a. Cload 1

3.

Determine the size factor from equation 6.7b. Size factor Csize 1.189

mm
d

0.097

Csize 0.781

4.

Determine the surface factor from equation 6.7e and Table 6-3. From Table 6-3 A 4.51 b 0.265
b

Surface factor

Csurf

Sut A MPa

Csurf 0.754

5.

Determine the temperature factor from equation 6.7f. Ctemp 1 0.0058

T 450 C C

Ctemp 0.710

6. 7.

Determine the reliability factor from Table 6-4, Creliab 0.814. Using equation 6.6, calculate the corrected endurance limit. S e Cload Csize Csurf Ctemp Creliab S'e S e 145.5 MPa

MACHINE DESIGN - An Integrated Approach, 4th Ed.

6-57-1

PROBLEM 6-57
Statement:

_____

A non rotating, ground, steel rod has a rectangular section with h = 60 mm and b = 40 mm. It is loaded in repeated bending. Determine its corrected fatigue strength with 99.9% reliability if it is used in an environment that has a temperature that is below 450C and has an ultimate tensile strength of 1550 MPa. Ultimate tensile strength Reliability Dimensions S ut 1550 MPa R 0.999 h 60 mm b 40 mm

Given:

Solution: 1.

See Mathcad file P0657.

Calculate the uncorrected endurance limit using equation 6.5a (S ut exceeds 1400 MPa). S'e 700 MPa

2.

Determine the loading factor from equation 6.7a. Cload 1

3.

Determine the size factor from equations 6.7b and 6.7d, and Figure 6-25. Area stressed above 95% of max A95 0.05 b h d equiv A95 0.0766 A95 120.0 mm
2

Equivalent diameter

d equiv 39.6 mm

Size factor

Csize 1.189

d equiv mm

0.097

Csize 0.832

4.

Determine the surface factor from equation 6.7e and Table 6-3. From Table 6-3 A 1.58 b 0.085
b

Surface factor

Csurf

Sut A MPa

Csurf 0.846

5. 6. 7.

Determine the temperature factor from equation 6.7f. Since T < 450C, Ctemp 1. Determine the reliability factor from Table 6-4, Creliab 0.753. Using equation 6.6, calculate the corrected endurance limit. S e Cload Csize Csurf Ctemp Creliab S'e S e 371.2 MPa

MACHINE DESIGN - An Integrated Approach, 4th Ed.

6-58-1

PROBLEM 6-58
Statement:

_____

A steel, grooved shaft similar to that shown in Figure C-5 (Appendix C) is to be loaded in bending Its dimensions are: D = 57 mm, d = 38 mm, r = 3 mm. Determine the fatigue stress- concentration factor if the material S ut = 1130 MPa. Dimensions: D 57 mm Tensile strength S ut 1130 MPa See Figure C-5 and Mathcad file P0658. d 38 mm r 3 mm

Given:

Solution: 1.

The geometric stress-concentration factor is found from the equation in Figure C-5. For D d Kt A 1.500 r
b

A 0.93894 Kt 2.14

b 0.32380

and

2.

The Neuber constant is found by linear interpolation of the values in Table 6-6. S ut 163.9 ksi a 1 0.031 in a S ut S 2 S1 S2
2

S 1 160 ksi

a 2 0.024 in a 0.030 in
0.5

S 2 180 ksi

a 1 a 2 a 2

3.

Calculate the notch sensitivity using equation 6.13. q 1 1 a r q 0.920

4.

The fatigue stress-concentration factor can now be found from equation 6.11b. Kf 1 q Kt 1 Kf 2.05

MACHINE DESIGN - An Integrated Approach, 4th Ed.

6-59-1

PROBLEM 6-59
Statement:

_____

A steel shaft with a transverse hole similar to that shown in Figure C-8 (Appendix C) is to be loaded in torsion. Its dimensions are: D = 32 mm, d = 3 mm. Determine the fatigue stressconcentration factor if the material S ut = 808 MPa. Dimensions: Tensile strength D 32 mm S ut 808 MPa d 3 mm

Given:

Solution: 1.

See Figure C-8 and Mathcad file P0659.

The geometric stress-concentration factor is found from an equation in Figure C-8. Although the maximum torsional stress is on the surface, it will be almost a maximum just below the surface so it will be conservative to use curve B. Kt 3.9702 9.292

D D 4 5 6 d d d 393.19 650.39 15.451 D D D


D

27.159

30.231

Kt 3.34

2.

The Neuber constant is found by linear interpolation of the values in Table 6-6. However, since the loading is torsional, 20 ksi must be added to the value of S ut that is used in the table (see the text in Figure 6-36, Part 1). S utt S ut 20 ksi a 1 0.044 in a S utt S 2 S1 S2
2

S utt 137.2 ksi S 1 130 ksi a 2 0.039 in a 0.040 in


0.5 2

S 2 140 ksi

a 1 a 2 a 2

3.

Calculate the notch sensitivity using equation 6.13. Let r 0.5 d q 1 1 a r q 0.857

4.

The fatigue stress-concentration factor can now be found from equation 6.11b. Kf 1 q Kt 1 Kf 3.00

MACHINE DESIGN - An Integrated Approach, 4th Ed.

6-60-1

PROBLEM 6-60
Statement:

_____

A hardened aluminum filleted flat bar similar to that shown in Figure C-9 (Appendix C) is to be loaded axially. Its dimensions are: D = 1.20 in, d = 1.00 in, r = 0.10 in. Determine the fatigue stress-concentration factor if the material S ut = 76 ksi. Dimensions: Tensile strength D 1.20 in S ut 76 ksi d 1.00 in r 0.100 in

Given:

Solution: 1.

See Figure C-9 and Mathcad file P0660.

The geometric stress-concentration factor is found from the equation in Figure C-9. For D d Kt A 1.200 r
b

A 1.03510 Kt 1.84

b 0.25084

and

2.

The Neuber constant is found by linear interpolation of the values in Table 6-8. a 1 0.144 in a S ut S 2 S1 S2
2

S 1 70 ksi

a 2 0.131 in a 0.136 in
0.5

S 2 80 ksi

a 1 a 2 a 2

3.

Calculate the notch sensitivity using equation 6.13. q 1 1 a r q 0.699

4.

The fatigue stress-concentration factor can now be found from equation 6.11b. Kf 1 q Kt 1 Kf 1.59

MACHINE DESIGN - An Integrated Approach, 4th Ed.

6-61-1

PROBLEM 6-61
Statement: A rotating shaft with a shoulder fillet seated in the inner race of a rolling contact bearing with the shoulder against the edge of the bearing is shown in Figure P6-20. The bearing has a slight eccentricity that induces a fully reversed bending moment in the shaft as it rotates. Measurements indicate that the resulting alternating stress amplitude due to bending is a = 57 MPa. The torque on the shaft fluctuates from a high of 90 N-m to a low of 12 N-m and is in phase with the bending stress. The shaft is ground and its dimensions are: D = 23 mm, d = 19 mm, and r = 1.6 mm. The shaft material is SAE 1040 cold-rolled steel. Determine the infinite-life fatigue safety factor for the shaft for a reliability of 99%. Strength SAE 1040 CR Fluctuating torque Shaft dimensions Solution: 1. S ut 586 MPa Alternating bending stress xa 57 MPa Tmax 90 N m Tmin 12 N m D 23 mm d 19 mm r 1.6 mm

Given:

See Figure P6-20 and Mathcad file P0661.

Determine the mean and alternating components of the fluctuating torsional stress. Distance to outside fiber c J d 2 c 9.5 mm
4

Polar moment of inertia

d
32

J 1.279 10 mm Tmax c J Tmin c J

Torsional stress

xymax xymin xym xya

xymax 66.827 MPa xymin 8.91 MPa xym 37.869 MPa xya 28.958 MPa

xymax xymin
2

xymax xymin
2

2.

Using Appendix C, determine the geometric stress concentration factors for the bending and torsional stresses. Bending (Fig. C-2): For D d 1.211 r d
b

0.084

A 0.97098

b 0.21796

r Kt A d Torsion (Fig. C-3): For D d 1.211

Kt 1.665 r d
b

0.084

A 0.83425

b 0.21649

Kts A 3.

Kts 1.425

Calculate the notch sensitivity of the material for bending and torsion using Table 6-6.

MACHINE DESIGN - An Integrated Approach, 4th Ed.

6-61-2

Bending: Neuber constant (for S ut 586.0 MPa) Notch sensitivity q b 1 1 Torsion: Neuber constant (for S ut 20 MPa 606.0 MPa) Notch sensitivity q s 1 1 4. a r a 0.0585 in q s 0.811
2

a 0.075 in q b 0.77

a r

Calculate the fatigue stress concentration factors for bending and torsion using equation 6.11b. Bending Torsion Kf 1 q b Kt 1 Kfs 1 q s Kts 1 Kf 1.512 Kfs 1.345

5.

Determine what, if any, fatigue stress concentration factor should be applied to the mean torsional stress. Yield strength SAE 1040 CR Evaluate S y 490 MPa which is less than S y so

Kfs 2 xymax 179.8 MPa Kfsm Kfs

6.

Calculate the mean and alternating components of the stresses increased by the appropriate fatigue stress concentration factors. Bending Torsion

m 0 MPa m Kfsm xym a Kfs xya

a Kf xa m 50.934 MPa a 38.949 MPa

a 86.188 MPa

7.

Find the mean and alternating von Mises stresses using equations 6.22b (with y = 0). Mean Alternating

'm 'a

3 m
2

2 2

'm 88.22 MPa 'a 109.451 MPa

a 3 a

8. Calculate the unmodified endurance limit. S'e 0.5 S ut S'e 293 MPa

9. Calculate the endurance limit modification factors for a rotating, round shaft. Load Cload 1 (combined bending and torsion)

MACHINE DESIGN - An Integrated Approach, 4th Ed.


Csize 1.189 A 1.58

6-61-3

Size

mm
d

0.097

Csize 0.894 b 0.085

Surface

(ground)

Csurf Temperature Reliability

Sut A MPa

Csurf 0.919

Ctemp 1 Creliab 0.814 (R = 99%)

10. Calculate the modified endurance limit. S e Cload Csize Csurf Ctemp Creliab S'e S e 196 MPa

11. Assuming a Case 2 load line, determine the factor of safety against fatigue failure. Se

Nf

'a

'm
S ut

Nf 1.5

MACHINE DESIGN - An Integrated Approach, 4th Ed.

6-62-1

PROBLEM 6-62
Statement: A tension member in a machine is filleted as shown in Figure P6-21. The member has a manufacturing defect that causes the fluctuating tension load to be applied eccentrically resulting in a fluctuating bending load as well. Measurements indicate that the maximum bending stress is 16.4 MPa and the minimum is 4.1 MPa. The tensile load fluctuates from a high of 3.6 kN to a low of 0.90 kN and is in phase with the bending stress. The member is machined and its dimensions are: D = 33 mm, d = 25 mm, h = 3 mm and r = 3 mm. The material is SAE 1020 cold-rolled steel. Determine the infinite-life fatigue safety factor for the member for a reliability of 90%. Strength SAE 1020 CR Fluctuating tension Dimensions Solution: 1. S ut 380 MPa xmax 16.4 MPa Fmax 3.6 kN Fmin 0.90 kN h 3 mm r 3 mm d 25 mm

Given:

xmin 4.1 MPa

D 33 mm

See Figure P6-21 and Mathcad file P0662.

Determine the mean and alternating components of the fluctuating stresses. Bending

mb ab

xmax xmin
2

mb 10.250 MPa ab 6.150 MPa xtmax 48.000 MPa

xmax xmin
2 Fmax h d Fmin h d

Tension

xtmax

xtmin mt at

xtmin 12.000 MPa mt 30.000 MPa at 18.000 MPa

xtmax xtmin
2

xtmax xtmin
2

2.

Using Appendix C, determine the geometric stress concentration factors for the bending and tensile stresses. Bending (Fig. C-10): For D d 1.32 r d
b

0.12

A 0.95880

b 0.27269

r Ktb A d Tension (Fig. C-9): For D d 1.32 r

Ktb 1.709 r d 0.12 A 1.05440 b 0.27021

Ktt A

Ktt 1.87

3.

Calculate the notch sensitivity of the material for bending and tension using Table 6-6.

MACHINE DESIGN - An Integrated Approach, 4th Ed.


Bending and tension: Neuber constant (for S ut 55.1 ksi) Notch sensitivity q 1 1 a r a 0.118 in q 0.744
2

6-62-2

4.

Calculate the fatigue stress concentration factors for bending and tension using equation 6.11b. Bending Tension Kfb 1 q Ktb 1 Kft 1 q Ktt 1 Kfb 1.528 Kft 1.648

5.

Determine what, if any, fatigue stress concentration factor should be applied to the mean stresses. Yield strength SAE 1040 CR Evaluate S y 207 MPa which is less than S y so

Kft xmax xtmax 106.1 MPa Kfbm Kfb Kftm Kft

6.

Calculate the mean and alternating components of the stresses increased by the appropriate fatigue stress concentration factors. Bending

mb Kfb mb ab Kfb ab

mb 15.662 MPa ab 9.397 MPa mt 49.427 MPa at 29.656 MPa

Tension

mt Kftm mt at Kft at

7.

Find the mean and alternating von Mises stresses using equations 6.22b (with y = 0). Mean Alternating

'm mb mt 'a ab mb

'm 65.089 MPa 'a 25.06 MPa

8. Calculate the unmodified endurance limit. S'e 0.5 S ut S'e 190 MPa

9. Calculate the endurance limit modification factors for a rotating, round shaft. Load Size Surface Cload 0.70 Csize 1 A 4.51 Csurf A b 0.265 (machined)

Sut MPa

Csurf 0.934

MACHINE DESIGN - An Integrated Approach, 4th Ed.

6-62-3

Temperature Reliability

Ctemp 1 Creliab 0.897 (R = 90%)

10. Calculate the modified endurance limit. S e Cload Csize Csurf Ctemp Creliab S'e S e 111.48 MPa

11. Assuming a Case 3 load line, determine the factor of safety against fatigue failure. S e S ut

Nf

'a S ut 'm S e

Nf 2.5

MACHINE DESIGN - An Integrated Approach, 4th Ed.

6-63a-1

PROBLEM 6-63a
Statement: For a filleted flat bar in tension similar to that shown in Appendix Figure C-9 and the data from row a from Table P6-7, determine the alternating and mean axial stresses as modified by the appropriate stress concentration factors in the bar. Strength SAE 1020 CR Widths Thickness Force Solution: 1. S ut 469 MPa d 20 mm Radius r 4 mm Pmax 32000 N D 40 mm h 10 mm Pmin 8000 N

Given:

See Table P6-7 and Mathcad file P0663a.

Determine the nominal mean and alternating components of the fluctuating stresses.

xmax xmin

Pmax h d Pmin h d

xmax 160.0 MPa xmin 40.0 MPa

xm xa
2.

xmax xmin
2

xm 100.0 MPa xa 60.0 MPa

xmax xmin
2

Using Appendix C-9, determine the geometric stress concentration factor. For D d 2 r r d
b

0.2

A 1.09960

b 0.32077

Kt A 3.

Kt 1.843

Calculate the notch sensitivity of the material using Table 6-6. Neuber constant (for S ut 68 ksi) Notch sensitivity q 1 1 a r a 0.096 in q 0.805
2

4.

Calculate the fatigue stress concentration factor using equation 6.11b. Kf 1 q Kt 1 Kf 1.679

5.

Determine what, if any, fatigue stress concentration factor should be applied to the mean stress. Yield strength SAE 1020 CR Evaluate S y 393 MPa

MACHINE DESIGN - An Integrated Approach, 4th Ed.

6-63a-2

Kfm

S 1 Kf xmax S 2 Kf xmax xmin return Kf if S 1 S y return S y Kf xa if S 1 S y S 2 2 S y

xm

0 otherwise Kfm 1.679

6.

Calculate the mean and alternating components of the stresses increased by the appropriate fatigue stress concentration factors.

m Kfm xm a Kf xa

m 167.9 MPa a 100.7 MPa

MACHINE DESIGN - An Integrated Approach, 4th Ed.

6-64a-1

PROBLEM 6-64a
Statement: For a filleted flat bar in bending similar to that shown in Appendix Figure C-10 and the data from row a from Table P6-7, determine the alternating and mean bending stresses as modified by the appropriate stress concentration factors in the bar. Strength SAE 1020 CR Widths Thickness Moment Solution: 1. S ut 469 MPa D 40 mm d 20 mm Radius r 4 mm h 10 mm Mmin 80 N m Mmax 320 N m

Given:

See Table P6-7 and Mathcad file P0664a.

Determine the nominal mean and alternating components of the fluctuating stresses. c d 2 Mmin c I Mmax c I c 10 mm I h d
3

12

I 6.667 10 mm

xmin xmax

xmin 120 MPa xmax 480 MPa

xm xa
2.

xmax xmin
2

xm 300.0 MPa xa 180.0 MPa

xmax xmin
2

Using Appendix C-10, determine the geometric stress concentration factor. For D d 2 r r d
b

0.2

A 0.93232

b 0.30304

Kt A 3.

Kt 1.518

Calculate the notch sensitivity of the material using Table 6-6. Neuber constant (for S ut 68 ksi) Notch sensitivity q 1 1 a r a 0.096 in q 0.805
2

4.

Calculate the fatigue stress concentration factor using equation 6.11b. Kf 1 q Kt 1 Kf 1.417

5.

Determine what, if any, fatigue stress concentration factor should be applied to the mean stresses. Yield strength SAE 1020 CR S y 393 MPa

MACHINE DESIGN - An Integrated Approach, 4th Ed.

6-64a-2

Evaluate Kfm S 1 Kf xmax S 2 Kf xmax xmin return Kf if S 1 S y return S y Kf xa if S 1 S y S 2 2 S y

xm

0 otherwise Kfm 0.460 6. Calculate the mean and alternating components of the stresses increased by the appropriate fatigue stress concentration factors.

m Kfm xm a Kf xa

m 137.9 MPa a 255.1 MPa

MACHINE DESIGN - An Integrated Approach, 4th Ed.

6-65a-1

PROBLEM 6-65a
Statement: For a shaft, with a shoulder fillet, in tension similar to that shown in Appendix Figure C-1 and the data from row a from Table P6-7, determine the alternating and mean axial stresses as modified by the appropriate stress concentration factors in the shaft. Strength SAE 1020 CR Widths Thickness Force Solution: 1. S ut 469 MPa d 20 mm Radius r 4 mm Pmax 32000 N D 40 mm h 10 mm Pmin 8000 N

Given:

See Table P6-7 and Mathcad file P0663a.

Determine the nominal mean and alternating components of the fluctuating stresses.

xmax

4 Pmax

d xmin

xmax 101.9 MPa

4 Pmin

d xm xa
2.

xmin 25.5 MPa

xmax xmin
2

xm 63.7 MPa xa 38.2 MPa

xmax xmin
2

Using Appendix C-1, determine the geometric stress concentration factor. For D d 2 r d
b

0.2

A 1.01470

b 0.30035

r Kt A d 3.

Kt 1.645

Calculate the notch sensitivity of the material using Table 6-6. Neuber constant (for S ut 68 ksi) Notch sensitivity q 1 1 a r a 0.096 in q 0.805
2

4.

Calculate the fatigue stress concentration factor using equation 6.11b. Kf 1 q Kt 1 Kf 1.52

5.

Determine what, if any, fatigue stress concentration factor should be applied to the mean stress. Yield strength SAE 1020 CR S y 393 MPa

MACHINE DESIGN - An Integrated Approach, 4th Ed.

6-65a-2

Evaluate

Kfm

S 1 Kf xmax S 2 Kf xmax xmin return Kf if S 1 S y return S y Kf xa if S 1 S y S 2 2 S y

xm

0 otherwise Kfm 1.520 6. Calculate the mean and alternating components of the stresses increased by the appropriate fatigue stress concentration factors.

m Kfm xm a Kf xa

m 96.7 MPa a 58.0 MPa

MACHINE DESIGN - An Integrated Approach, 4th Ed.

6-66a-1

PROBLEM 6-66a
Statement: For a shaft, with a shoulder fillet, in bending similar to that shown in Appendix Figure C-2 and the data from row a from Table P6-7, determine the alternating and mean bending stresses as modified by the appropriate stress concentration factors in the shaft. Strength SAE 1020 CR Widths Thickness Moment Solution: 1. S ut 469 MPa D 40 mm d 20 mm Radius r 4 mm h 10 mm Mmin 80 N m Mmax 320 N m

Given:

See Table P6-7 and Mathcad file P0666a.

Determine the nominal mean and alternating components of the fluctuating stresses. c d 2 Mmin c I Mmax c I c 10 mm I

d
64

I 7.854 10 mm

xmin xmax

xmin 101.859 MPa xmax 407.437 MPa

xm xa
2.

xmax xmin
2

xm 254.6 MPa xa 152.8 MPa

xmax xmin
2

Using Appendix C-2, determine the geometric stress concentration factor. For D d 2 r r d
b

0.2

A 0.90879

b 0.28598

Kt A 3.

Kt 1.44

Calculate the notch sensitivity of the material using Table 6-6. Neuber constant (for S ut 68 ksi) Notch sensitivity q 1 1 a r a 0.096 in q 0.805
2

4.

Calculate the fatigue stress concentration factor using equation 6.11b. Kf 1 q Kt 1 Kf 1.354

5.

Determine what, if any, fatigue stress concentration factor should be applied to the mean stress. Yield strength SAE 1020 CR S y 393 MPa

MACHINE DESIGN - An Integrated Approach, 4th Ed.

6-66a-2

Evaluate

Kfm

S 1 Kf xmax S 2 Kf xmax xmin return Kf if S 1 S y return S y Kf xa if S 1 S y S 2 2 S y

xm

0 otherwise Kfm 0.731 6. Calculate the mean and alternating components of the stresses increased by the appropriate fatigue stress concentration factors.

m Kfm xm a Kf xa

m 186.1 MPa a 206.9 MPa

MACHINE DESIGN - An Integrated Approach, 4th Ed.

6-67-1

PROBLEM 6-67
Statement: A machine part is subjected to fluctuating, simple, multiaxial stresses. The fully corrected nonzero stress ranges are: xmin = 50 MPa, xmax = 200 MPa, ymin = 80 MPa, ymax = 320 MPa, xymin = 120 MPa, xymax = 480 MPa. The material properties are: S e = 525 MPa and S ut = 1200 MPa. Using a Case 3 load line, calculate and compare the infinite-life safety factors given by the Sines and von Mises Methods. Material properties: Stresses: S ut 1200 MPa S e 525 MPa

Given:

xmin 50 MPa ymin 80 MPa

xmax 200 MPa ymax 320 MPa xymax 480 MPa

xymin 120 MPa


Solution: 1. See Mathcad file P0667.

Calculate the alternating, and mean components of the given stresses.

xa xm

xmax xmin
2

xa 75 MPa xm 125 MPa

xmax xmin
2

ya ym xya xym
(a) Sines Method 2.

ymax ymin
2

ya 120 MPa ym 200 MPa xya 180 MPa xym 300 MPa

ymax ymin
2

xymax xymin
2

xymax xymin
2

Calculate the equivalent alternating and mean stresses using equations 6.21b.
2 2 2

'a

xa ya xa ya 3 xya

'a 329.0 MPa 'm 325.0 MPa

'm xm ym
3. The factor of safety for the Sines Method, using equation (6.18e) is Nfs S e S ut

'a S ut 'm S e

Nfs 1.11

(b) von Mises Method 4. Calculate the equivalent alternating and mean stresses using equations 6.22b.
2 2 2

'a

xa ya xa ya 3 xya

'a 329.0 MPa

MACHINE DESIGN - An Integrated Approach, 4th Ed.

6-67-2

'm
5.

xm ym xm ym 3 xym

'm 548.3 MPa

The factor of safety for the von Mises Method, using equation (6.18e) is Nfvm S e S ut Nfvm 0.92

'a S ut 'm S e

6.

This example shows that the von Mises method is more conservative when the endurance limit is modified by such factors as surface finish and when there is a high mean shear stress.

MACHINE DESIGN - An Integrated Approach, 4th Ed.

6-68-1

PROBLEM 6-68
Statement: A cylindrical tank with hemispherical ends has been built. It was made from hot rolled steel that has S ut = 380 MPa. The tank outside diameter is 300 mm with 20 mm wall thickness. The pressure may fluctuate from 0 to an unknown maximum. For an infinite-life fatigue safety factor of 4 with 99.99% reliability, what is the maximum pressure to which the tank may be subjected? Ultimate strength Tank dimensions Reliability & FS Solution: 1. See Mathcad file P0668. S ut 380 MPa OD 300 mm R 0.9999 t 20 mm Nf 4

Given:

Determine the maximum principal stresses as functions of the unkown pressure, which occur at the inside wall, for this thick-wall cylinder: Outside radius Inside radius ro 0.5 OD ri ro t ri p
2

ro 150 mm ri 130 mm

Tangential

2 1 ro t( p ) 2 2 2 ri ro ri 2 1 ro 2 2 2 ri ro ri

Radial

r( p )

ri p

Axial

a( p )

ri p ro ri
2 2

These are principal stresses so, using equation 5.7a:

'( p )

t( p ) r( p ) a( p ) t( p ) a( p ) a( p ) r( p ) t( p ) r( p )

'min 0 MPa
2. Determine the alternating, and mean von Mises effective stress using equations 6.1.

'a( p )
3. 4.

'( p ) 'min
2

'm( p )
S'e 0.5 S ut

'( p ) 'min
2 S'e 190 MPa

Calculate the unmodified endurance limit.

Calculate the endurance limit modification factors for axial loading. Load Size Surface Cload 0.7 Csize 1 A 57.7 (axial loading) (axial loading) b 0.718
b

(hot rolled) Csurf 0.811

Csurf Temperature

Sut A MPa

Ctemp 1

MACHINE DESIGN - An Integrated Approach, 4th Ed.


Reliability 5. Creliab 0.702 (R = 99.99%)

6-68-2

Calculate the modified endurance limit. S e Cload Csize Csurf Ctemp Creliab S'e S e 75.7 MPa

6.

Assuming a Case 3 load line, solve equation (6.18e) for p max Guess Given S e S ut Nf 4.00 p 1 MPa

Nf =

'a( p ) S ut 'm( p ) S e

p max Find ( p ) 7. The principal stresses at p max are, respectively:

p max 4.54 MPa

1 t p max 1 31.9 MPa

2 a p max 2 13.7 MPa

3 r p max 3 4.5 MPa

MACHINE DESIGN - An Integrated Approach, 4th Ed.

6-69-1

PROBLEM 6-69
Statement: A rotating shaft has been designed and fabricated from SAE 1040 HR steel. It is made from tubing that has an outside diameter of 60 mm and a wall thickness of 5 mm. Strain gage measurements indicate that there is a fully reversed axial stress of 68 MPa and a torsional stress that fluctuates from 12 MPa to 52 MPa in phase with the axial stress at the critical point on the shaft. Determine the infinite-life fatigue safety factor for the shaft for a reliability of 99%. Strength SAE 1040 HR Torsional stress Shaft dimensions S ut 524 MPa Alternating bending stress xa 68 MPa

Given:

xymax 52 MPa
OD 60 mm

xymin 12 MPa
t 5 mm

See Mathcad file P0669. Solution: 1. Calculate the alternating, and mean components of the given stresses.

xm 0 MPa xymax xymin xya xym


2.

xa 68 MPa xya 20 MPa xym 32 MPa

2 xymax xymin 2

Find the mean and alternating von Mises stresses using equations 6.22b (with y = 0). Mean Alternating

'm 'a

3 xym
2

2 2

'm 55.426 MPa 'a 76.315 MPa

xa 3 xya

3. Calculate the unmodified endurance limit. S'e 0.5 S ut Cload 1 Csize 1.189 A 57.7 S'e 262 MPa

4. Calculate the endurance limit modification factors for a rotating, round shaft. Load (combined bending and torsion)

Size Surface

mm

OD

0.097

Csize 0.799 b 0.718 (hot rolled)

Csurf Temperature Reliability

Sut A MPa

Csurf 0.644

Ctemp 1 Creliab 0.814 (R = 99%)

5. Calculate the modified endurance limit. S e Cload Csize Csurf Ctemp Creliab S'e S e 110 MPa

6. Assuming a Case 3 load line, determine the infinite-life fatigue safety factor. Nf S e S ut

'a S ut 'm S e

Nf 1.25

MACHINE DESIGN - An Integrated Approach, 4th Ed.

6-70a-1

PROBLEM 6-70a
Statement: Given: For the data in row a in Table P6-8, find the safety factor for each of the four loading cases based on the Modified-Goodman diagram if S e = 100, S y = 150, and S ut = 200. S e 100 S y 150 S ut 200

'm 50
Solution: 1.

'a 30

See Table 6-8 and Mathcad file P0670a.

Calculate the coordinates of point D in Figure 6-46 using equations (6.16). Dm S ut S y S e S ut S e Dm 100 Da 50

Da S y Dm 1.

Use equations (6.18) and (6.16) to calculate the required quantities. Case 1: Nf1

if 'a Da S ut 'a 1 otherwise 'm Se


'm
Sy Se

Sy

'a

Nf1 2.40

Case 2: Nf2 1

'm

'a S ut S y 'm 'a

if 'm Dm

Nf2 2.50

otherwise

Case 3: S e S ut Se

Nf3

'a S ut 'm S e
S y 'a

if

'a 'm

Se S ut

Dm

Nf3 1.82

'm
Case 4:
2

otherwise

'mSut

S ut S e S e 'a S ut 'm S e S ut
2 2

'aSut S e
OZ
2

Se S ut

'mSut
2

'a 'm

MACHINE DESIGN - An Integrated Approach, 4th Ed.

6-70a-2

ZSut

'm 'mSut 2 'a 'aSut2 'm 'mSut2 'a 'aSut 2


2

if 'm

Se

'a 'm

Se S ut

otherwise

'mSy

S y 'a 'm

'aSy 'mSy 'a 'm


ZSy

'm 'mSy2 'a 'aSy 2 'm 'mSy 2 'a 'aSy2


if 'mSut Dm otherwise

if 'm

Sy 1

'a 'm

otherwise

Nf4

OZ ZSut OZ OZ ZSy OZ

Nf4 1.69

MACHINE DESIGN - An Integrated Approach, 4th Ed.

6-70h-1

PROBLEM 6-70h
Statement: Given: For the data in row h in Table P6-8, find the safety factor for each of the four loading cases based on the Modified-Goodman diagram if S e = 100, S y = 150, and S ut = 200. S e 100 S y 150 S ut 200

'm 80
Solution: 1.

'a 80

See Table 6-8 and Mathcad file P0670h.

Calculate the coordinates of point D in Figure 6-46 using equations (6.16). Dm S ut S y S e S ut S e Dm 100 Da 50

Da S y Dm 1.

Use equations (6.18) and (6.16) to calculate the required quantities. Case 1: Nf1

if 'a Da S ut 'a 1 otherwise 'm Se


'm
Sy Se

Sy

'a

Nf1 0.50

Case 2: Nf2 1

'm

'a S ut S y 'm 'a

if 'm Dm

Nf2 0.75

otherwise

Case 3: S e S ut Se

Nf3

'a S ut 'm S e
S y 'a

if

'a 'm

Se S ut

Dm

Nf3 0.83

'm
Case 4:
2

otherwise

'mSut

S ut S e S e 'a S ut 'm S e S ut
2 2

'aSut S e
OZ
2

Se S ut

'mSut
2

'a 'm

MACHINE DESIGN - An Integrated Approach, 4th Ed.

6-70h-2

ZSut

'm 'mSut 2 'a 'aSut2 'm 'mSut2 'a 'aSut 2


2

if 'm

Se

'a 'm

Se S ut

otherwise

'mSy

S y 'a 'm

'aSy 'mSy 'a 'm


ZSy

'm 'mSy2 'a 'aSy 2 'm 'mSy 2 'a 'aSy2


if 'mSut Dm otherwise

if 'm

Sy 1

'a 'm

otherwise

Nf4

OZ ZSut OZ OZ ZSy OZ

Nf4 0.84

MACHINE DESIGN - An Integrated Approach, 4th Ed.

6-71-1

PROBLEM 6-71
Statement: A rotating shaft with a shoulder fillet, in torsion similar to that shown in Appendix Figure C-3 is made from SAE 1020 CR steel and has dimensions D = 40 mm, d = 20 mm, and r = 4 mm. The shaft is ground and is subjected to a fully reversed torque of +/- 80 N-m. Determine the infinite life safety factor for the shaft for a reliability of 99.9%. Strength SAE 1020 CR Dimensions Torque Solution: 1. S ut 469 MPa d 20 mm Tm 0 N m r 4 mm D 40 mm Ta 80 N m

Given:

See Mathcad file P0671.

Determine the nominal mean and alternating components of the stresses.

xym 0 MPa xya


2. 16 Ta

xya 50.9 MPa

Using Appendix C-3, determine the geometric stress concentration factor. For D d Kt A 2 r
b

r d

0.2

A 0.86331

b 0.23865

Kt 1.268

3.

Calculate the notch sensitivity of the material using Table 6-6 adding 20 ksi to S ut because of the torsional load Neuber constant (for S ut 20 ksi 88 ksi) Notch sensitivity q 1 1 a r a 0.072 in q 0.846
2

4.

Calculate the fatigue stress concentration factor using equation 6.11b. Kf 1 q Kt 1 Kf 1.226

5.

Calculate the mean and alternating components of the stresses increased by the appropriate fatigue stress concentration factors.

m xym a Kf xya
6. Calculate the von Mises normal stress. von Mises stress 7.

m 0 MPa a 62.46 MPa

'a

3 a

'a 108.19 MPa

Calculate the endurance limit modification factors for a rotating, solid, round steel shaft. Load Cload 1 (pure torsion)

MACHINE DESIGN - An Integrated Approach, 4th Ed.

6-71-2

Size

Csize 1.189 A 1.58 Csurf A

mm
d

0.097

Csize 0.889

Surface

b 0.085

(ground)

Sut MPa

Csurf 0.937

Temperature Reliability

Ctemp 1 Creliab 0.753 (R = 99.9%) S'e 0.5 S ut S'e 234.5 MPa

Uncorrected endurance strength

8.

Calculate the endurance limit. S e Cload Csize Csurf Ctemp Creliab S'e S e 147.1 MPa

9.

Using equation 6.14, calculate the infinite-life factor of safety. Se

Nf

'a

Nf 1.4

MACHINE DESIGN - An Integrated Approach, 4th Ed.

7-1-1

PROBLEM 7-1
Statement: Given: Two 3 x 5 cm blocks of steel with machined finish Ra = 0.6 mm are rubbed together with a normal force of 400 N. Estimate the true area of contact between them if their S y = 400 MPa. Length of block Width of block L 5 cm w 3 cm Normal force Yield strength F 400 N S y 400 MPa

Assumptions: The compressive yield strength is the same as the tensile yield strength. Then, S yc S y. Solution: 1. See Mathcad file P0701.

Using equation 7.1, the true area of contact is estimated as Ar F 3 S yc Aa w L Ar 3.33333 10 Aa 15 cm


2 3

cm

2.

The apparent area of contact is

3.

The ratio of apparent area to true area is

Aa Ar

4500

MACHINE DESIGN - An Integrated Approach, 4th Ed.

7-2-1

PROBLEM 7-2
Statement: Estimate the dry coefficient of friction between the two pieces in Problem 7-1 if their S ut = 600 MPa. Length of block Width of block Normal force L 5 cm w 3 cm F 400 N Yield strength Ultimate strength S y 400 MPa S ut 600 MPa

Given:

Assumptions: The compressive yield strength is the same as the tensile yield strength. Then, S yc S y. Solution: See Mathcad file P0702.

Using equations 7.3 and 7.4, the coefficient of friction is estimated as S us 0.80 S ut S us 480 MPa

S us 3 S yc

0.40

MACHINE DESIGN - An Integrated Approach, 4th Ed.

7-3-1

PROBLEM 7-3
Statement: For the bicycle pedal-arm assembly in Figure P7-1 assume a rider-applied force that ranges from 0 to 400 N at the pedal each cycle. Determine the maximum contact stresses at one sprocket tooth-chain roller interface. Assume that the one tooth takes all the applied torque, that the chain roller is 8-mm dia, the sprocket has a nominal (pitch) dia of 100 mm, and that the sprocket tooth is essentially flat at the point of contact. The roller and sprocket are made of SAE X1340 steel, induction hardened to HRC 45-58. The roller and sprocket contact over a length of 8 mm. Assuming rolling plus 9% sliding, estimate the number of cycles to failure for this particular tooth-roller combination. Roller radius Sprocket radius Sprocket width Pedal force R1 4 mm R2 mm w 8 mm Frider 400 N The parts are steel. Therefore: E 207 GPa

Given:

0.28 Sprocket pitch dia

d p 100 mm

Pedal arm length len 170 mm Assumptions: The coefficient of friction is 0.33 Solution: 1. See Figure P7-1 and Mathcad file P0703.

Determine the maximum contact force. Torque on sprocket Contact force Tmax Frider len Fcmax 2 Tmax dp Tmax 68 N m Fcmax 1.36 kN

2.

Find the material constants from equation 7.9a. Material constants m1 1 E


2

m1 4.452 10

1 MPa

m2 m1 Geometry constant B 1 2 1 1

R1

R2
1 2

B 125 m

Contact patch half-width 3.

2 m1 m2 Fcmax a B w
Fcmax 2 a w 2 Fcmax

a 0.0878 mm

The average and maximum contact pressure can now be found from equations 7.14b and c. Average pressure p avg p max p avg 968.1 MPa p max 1233 MPa fmax 406.8 MPa

Maximum pressure Tangential pressure 4.

a w

fmax p max

With m = 0.33, the principal stresses in the contact zone will be maximal on the surface (z = 0) at x = 0.3a from th centerline as shown in Figures 7-20 and 7-22. The applied stress components are found from equation 7.23a for the normal force and equation 7.23b for the tangential force. For x 0.3 a

MACHINE DESIGN - An Integrated Approach, 4th Ed.


x
2 2

7-3-2

xn p max 1
x a

xn 1176 MPa

xt 2 fmax

xt 244.05 MPa
x
2 2

zn p max 1 zt 0 MPa xzt fmax 1


5.

zn 1176 MPa xzn 0 MPa

2 2

xzt 388.0 MPa

Equations 7.24a and 7.24b can now be solved for the total applied stresses along the x, y, and z axes.

x xn xt z zn zt xz xzn xzt
6.

x 1420 MPa z 1176 MPa xz 388.019 MPa

Assuming the rollers are short, we expect a plane stress condition to exist. The stress in the third dimension is then:

y 0 MPa
7.

also,

xy 0 MPa

yz 0 MPa

Unlike the pure-rolling case, these stresses are not principal because of the applied shear stress. The principal stresses are found from equation 4.4 using a cubic root finding solution. C2

x y z
MPa

C2 2.596 10

xz yz x xy x y MPa MPa MPa MPa MPa MPa C1 xy xz z yz z y MPa MPa MPa MPa MPa MPa

C1 1.519 10

x xy xz MPa MPa MPa xy y yz C0 MPa MPa MPa xz yz z MPa MPa MPa


f ( ) C2 C1 C0
3 2

C0 0

MACHINE DESIGN - An Integrated Approach, 4th Ed.

7-3-3

C0 C1 v C2 1
Principal stresses:

r polyroots ( v) MPa

1704.6 r 891.1 MPa 0.0


1 0 MPa 2 891.1 MPa 3 1704.6 MPa

1 r 2 r 3 r

3 2 1

8.

The maximum normal stress calculated in step 7 is 1705 MPa, compressive. Its K-factor can be calculated from equation 7.25d. K-factor K m1 m2 3
2

K 81.3 MPa

9.

From Table 7-7, Part 1, Line 4 the slope and intercept factors of this cast iron for rolling with 9% sliding are

8.51

41.31

10. These are used in equation 7.26 along with the value of K from above to find the number of cycles that can be expected at this load before pitting begins. log ( K) =

log Nlife
log

Nlife 10

K psi

Nlife 4.6 10

cycles

MACHINE DESIGN - An Integrated Approach, 4th Ed.

7-4-1

PROBLEM 7-4
Statement: For the trailer hitch from Problem 3-4 on p. 169, determine the contact stresses in the ball and ball cup. Assume that the ball is 2-in dia and the ill-fitting cup that surrounds the it is an internal spherical surface 10% larger in diameter than the ball.

F
Given: Ball diameter Cup diameter Pull force Tongue weight Poisson's ratio Modulus of elasticity Solution: d 2 in D 2.2 in Fpull 4.905 kN Wtong 0.981 kN

0.28
E 30.0 10 psi
6

See Figure 7-4 and Mathcad file P0704.

Total force

Fpull Wtong

FIGURE 7-4
Diagram Showing Contact Force for Problem 7-4

F 1125 lbf Ball radius Cup radius Geometry constant R1 0.5 d R2 0.5 D B 1 2 1

R1 1.000 in R2 1.100 in
2

R1
E

R2
1

B 0.045 in

Material constants

m1

m1 3.072 10

8 1

psi

m2 m1
1 3

Contact patch radius Contact area Average pressure

3 m1 m2 a F B 8
A a p avg p max
2

a 0.0829 in A 0.022 in
2

F A 3 2 p avg

p avg 52.1 ksi p max 78.1 ksi

Maximum pressure Stresses Axial

zmax p max xmax


1 2 2 p max

zmax 78.1 ksi xmax 60.9 ksi

In-plane

ymax xmax

MACHINE DESIGN - An Integrated Approach, 4th Ed.


p max 2 1 2 2 2 9 ( 1 ) 2 ( 1 )

7-4-2

Max shear stress

yzmax

yzmax 26.4 ksi


Depth at max shear stress zmax a 2 2 7 2 zmax 0.05228 in

MACHINE DESIGN - An Integrated Approach, 4th Ed.

7-5-1

PROBLEM 7-5
Statement: For the trailer hitch from Problem 3-5 on p. 169, determine the contact stresses in the ball and ball cup. Assume that the ball is 2-in dia and the ill-fitting cup that surrounds the it is an internal spherical surface 10% larger in diameter than the ball. Ball diameter Cup diameter Trailer mass Terminal velocity Time to reach vel. Tongue weight Poisson's ratio Modulus of elasticity Solution: d 2 in D 2.2 in mtrail 2000 kg v 60 m sec t 20 sec Wtong 0.981 kN
1

Given:

0.28
E 30.0 10 psi
6

See Figure 7-5 and Mathcad file P0705. Acceleration Pull force a v t a 3.00 m sec
2

Fpull mtrail a Fpull 6.00 kN

FIGURE 7-5
Diagram Showing Contact Force for Problem 7-5

Total force Ball radius Cup radius Geometry constant

Fpull Wtong

F 1367 lbf R1 1.000 in R2 1.100 in

R1 0.5 d R2 0.5 D B 1 2 1

R1
E

R2
1

B 0.045 in

Material constants

m1

m1 3.072 10

8 1

psi

m2 m1
1 3

Contact patch radius Contact area Average pressure

3 m1 m2 a F B 8
A a p avg p max
2

a 0.0885 in A 0.025 in
2

F A 3 2 p avg

p avg 55.6 ksi p max 83.3 ksi

Maximum pressure Stresses Axial

zmax p max

zmax 83.3 ksi

MACHINE DESIGN - An Integrated Approach, 4th Ed.


In-plane

7-5-2
p max

xmax

1 2 2

xmax 65.0 ksi

ymax xmax
Max shear stress

yzmax

p max 2

1 2 2

2 9

( 1 ) 2 ( 1 )

yzmax 28.1 ksi


Depth at max shear stress zmax a 2 2 7 2 zmax 0.05579 in

MACHINE DESIGN - An Integrated Approach, 4th Ed.

7-6-1

PROBLEM 7-6
Statement: For the trailer hitch from Problem 3-6 on p. 169, determine the contact stresses in the ball and ball cup. Assume that the ball is 2-in dia and the ill-fitting cup that surrounds the it is an internal spherical surface 10% larger in diameter than the ball.

Given:

Ball diameter Cup diameter Trailer mass Tongue weight Poisson's ratio Modulus of elasticity

d 2 in D 2.2 in mtrail 2000 kg Wtong 0.981 kN

0.28
E 30.0 10 psi
6

Solution:

See Figure 7-6 and Mathcad file P0706.

From Problem 3-6, the impact (pull) force is Pull force Total force Fpull 55.1 kN F Fpull Wtong
2 2

FIGURE 7-6
Diagram Showing Contact Force for Problem 7-6

F 12389 lbf Ball radius Cup radius Geometry constant R1 0.5 d R2 0.5 D B 1 2 1 R1 1.000 in R2 1.100 in
2

R1
E

R2
1

B 0.045 in

Material constants

m1

m1 3.072 10

8 1

psi

m2 m1
1 3

Contact patch radius Contact area Average pressure

3 m1 m2 a F B 8
A a p avg p max
2

a 0.1845 in A 0.107 in
2

F A 3 2 p avg

p avg 115.9 ksi p max 173.8 ksi

Maximum pressure Stresses Axial

zmax p max
1 2 2

zmax 173.8 ksi

In-plane

xmax

p max

xmax 135.6 ksi

ymax xmax

MACHINE DESIGN - An Integrated Approach, 4th Ed.

7-6-2

Max shear stress

yzmax

p max 2

1 2 2

2 9

( 1 ) 2 ( 1 )

yzmax 58.7 ksi


Depth at max shear stress zmax a 2 2 7 2 zmax 0.11632 in

MACHINE DESIGN - An Integrated Approach, 4th Ed.

7-7-1

PROBLEM 7-7
Statement: For the 12-mm dia steel wrist pin of Problem 3-7, find the maximum contact stress if the 2500 g acceleration is fully reversed. The aluminum piston has a hole for the wrist pin that is 2% larger than the pin and an engagement length of 20 mm. Wrist pin dia Piston hole multiplier Acceleration Piston mass Length of contact Solution: See Mathcad file P0707. Contact force Pin radius Hole radius F M a R1 0.5 d R2 k R1 B 1 2 1
2

Given:

d 12 mm k 1.02 a 2500 g M 0.5 kg L 2 cm

Material properties: Steel wrist pin 1 0.28 E1 206.8 GPa Aluminum piston 2 0.34 E2 71.7 GPa

F 12.26 kN R1 6 mm R2 6.12 mm 1

Geometry constant

R1
E1

R2

B 1.63399 10

mm

Material constants

m1

1 1

m1 4.456 10

1 MPa 1 MPa

m2

1 2 E2

m2 1.233 10
1

Contact patch half-width Contact area Average pressure Maximum pressure

2 m1 m2 F a B L
A 2 L a p avg p max F A 2 F

a 2.002 mm A 80.10 mm
2

p avg 153.0 MPa p max 194.9 MPa

a L

Wrist pin 1. The maximum normal stresses in the center of the contact patch at the surface of the steel wrist pin are found using equations 7.17a. Axial In-plane 2.

zmax p max y1max 2 1 p max

zmax 194.9 MPa y1max 109.1 MPa

The maximum shear stress and its location under the surface are found from equations 7.17b. Max shear stress Depth at max shear stress

13max 0.304 p max


zmax 0.786 a

13max 59.2 MPa


zmax 1.57 mm

MACHINE DESIGN - An Integrated Approach, 4th Ed.


Piston hole 3.

7-7-2

The maximum normal stresses in the center of the contact patch at the surface of the aluminum piston hole are found using equations 7.17a. Axial In-plane

zmax p max y2max 2 2 p max

zmax 194.9 MPa y2max 132.5 MPa

4.

The maximum shear stress and its location under the surface are found from equations 7.17b. Max shear stress Depth at max shear stress

13max 0.304 p max


zmax 0.786 a

13max 59.2 MPa


zmax 1.57 mm

MACHINE DESIGN - An Integrated Approach, 4th Ed.

7-8-1

PROBLEM 7-8
Statement: A paper mill processes rolls of paper having a density of 984 kg/m3. The paper roll is 1.50 m outside dia (OD) by 0.22 m inside dia (ID) by 3.23 m long and has an effective modulus of elasticity in compression of 14 MPa and n = 0.3. Determine the width of its contact patch when it sits on a flat steel surface, loaded by its own weight. Paper properties

Given:

1 984 kg m
E1 14 MPa

Roll dimensions: Outside diameter OD 1.50 m Inside diameter Lemgth ID 0.22 m L 3.23 m

1 0.3
Steel properties E2 207 GPa

2 0.28
Solution: 1. See Mathcad file P0708.

The weight of the paper roll is equal to its volume times the paper density times g. Wroll

OD ID L 1 g

Wroll 53.895 kN

2.

Determine the radii of the contacting bodies. Roll Plate R1 0.5 OD R2 mm R1 750 mm

3.

Determine the geometry and material constants. Geometry constant B 1 2 1

R1

R2
1

B 0.667 m

Material constants

m1

1 1 E1 1 2 E2

m1 0.065

1 MPa
6

m2

m2 4.452 10

1 MPa

4.

Calculate the contact patch hakf-width and width.


1

Contact patch half-width Contact patch width

2 m1 m2 Wroll B L

a 32.1832 mm width 64.4 mm

width 2 a

MACHINE DESIGN - An Integrated Approach, 4th Ed.

7-9-1

PROBLEM 7-9
Statement: For the ViseGrip plier-wrench for which the forces were analyzed in Problem 3-9, find the force in link 4 needed to create a 0.25-mm-wide flat on each side of a 2-mm dia aluminum pin squeezed in its 5-mm-wide jaws. Pin radius Anvil curvature Contact patch Contact length R1 1 mm R2 mm a 0.125 mm L 5 mm Material properties: Steel anvil E2 206.8 GPa Aluminum pin E1 71.7 GPa Solution: 1. See Mathcad file P0709.

Given:

2 0.28 1 0.34

Determine the geometry and material constants. Geometry constant B 1 2 1 1

R1

R2

B 500 m

Material constants

m1

1 1 E1 1 2 E2

m1 1.233 10

1 MPa 1 MPa

m2

m2 4.456 10

2.

Calculate the contact force

2 m1 m2

a B L

F 3.654 kN

3.

Get the geometry from Problem 3-9 and calculate the pin force.

21.0 deg
F4 sin( 180 deg )

129.2 deg
F cos( 180 deg ) cos( 180 deg) sin( 180 deg)

F4 4.65 kN

MACHINE DESIGN - An Integrated Approach, 4th Ed.

7-10-1

PROBLEM 7-10
Statement: An overhung diving board is shown in Figure P7-4a. A 100-kg person is standing on the free end. The board sits on a fulcrum that has a cylindrical contact surface of 5-mm radius. What is the size of the contact patch between the board and the fulcrum if the board material is fiberglass with E = 10.3 GPa and n = 0.3? Fulcrum radius Board curvature Mass of person Material properties: Aluminum fulcrum Fiberglass board R1 5 mm R2 mm M 100 kg
R1 2000 = L P

Given:

1 0.34
E1 71.7 GPa

2 0.30
E2 10.3 GPa
700 = a

R2

Board dimensions: Width (Prob 4-10) w 305 mm Distance to right support a' 0.7 m Contact length L 2 m Solution: See Figure 7-10 and Mathcad file P0710. Weight of person P M g

FIGURE 7-10
Free Body Diagram for Problem 7-10

P 0.981 kN P L F b = 0 F 2.802 kN 1 1

Summing moments about the support on the left end of the board, Fulcrum reaction F P B 1 2 L a'

Geometry constant

R1

R2

B 0.100 mm

Material constants

m1

1 1 E1 1 2 E2

m1 1.233 10

1 MPa 1 MPa

m2

m2 8.835 10
1

Contact patch half-width Contact patch width

2 m1 m2 F a B w
a2 2 a

a 0.0767 mm a2 0.153 mm

MACHINE DESIGN - An Integrated Approach, 4th Ed.

7-11-1

PROBLEM 7-11
Statement: Repeat Problem 7-10 assuming the 100-kg person in Problem 7-10 jumps up 25 cm and lands back on the board. Assume the board weighs 29 kg and deflects 13.1 cm statically when the person stands on it. What is the size of the contact patch between the board and the 5-mm-radius aluminum fulcrum if the board material is fiberglass with E = 10.3 GPa and n = 0.3? Fulcrum radius Board curvature Mass of person Material properties: Aluminum fulcrum Fiberglass board R1 5 mm R2 mm M 100 kg
R1 2000 = L P

Given:

1 0.34
E1 71.7 GPa

2 0.30
E2 10.3 GPa
700 = a

R2

Board dimensions: Width (Prob 4-10) w 305 mm Distance to right support a' 0.7 m Contact length L 2 m Solution: 1. See Figure 7-11 and Mathcad file P0711.

FIGURE 7-11
Free Body Diagram for Problem 7-11

From Problem 3-11, the dynamic load resulting from the impact of the person with the board is P 3.056 kN Summing moments about the support on the left end of the board, Fulcrum reaction F P B 1 2 L a' 1 P L F b = 0 F 8.731 kN

Geometry constant

R1

R2
1

B 0.100 mm

Material constants

m1

1 1 E1 1 2 E2

m1 1.233 10

1 MPa 1 MPa

m2

m2 8.835 10
1

Contact patch half-width Contact patch width

2 m1 m2 F a B w
a2 2 a

a 0.1355 mm a2 0.271 mm

MACHINE DESIGN - An Integrated Approach, 4th Ed.

7-12-1

PROBLEM 7-12
Statement: Estimate the volume of adhesive wear to expect from an HB270 steel shaft of 40-mm diameter rotating at 250 rpm for 10 years in a plain bronze bushing if the transverse load is 1000 N. (a) For conditions of poor lubrication. (b) For conditions of good lubrication. rpm 2 rad min Journal diameter Journal speed Journal hardness
1

Units: Given:

kilo kg g d 40 mm n 250 rpm kilo HB 270 2 mm

yr 260 day Load Life

rev 2 rad F 1000 N Life 10 yr

Assumptions: The bushing is softer than the journal (shaft) so the wear will take place predominately on the bushing. Solution: 1. See Mathcad file P0712.

From Figure 7-6 in the text we see that iron (the principal ingredient of steel) is metalurgically compatible with copper and tin, the principal ingredients of bronze. From Figure 7-7 we have the following values of adhesive wear coefficient for metalurgically compatible and poor and good lubrication, respectively. Adhesive wear coefficients Ka 10 Kb 10 The length of sliding is L
4 5

poor lubrication good lubrication


8

d
rev

n Life

L 1.176 10 m

2.

Using equation 7.7a we can estimate the volume of wear for each lubrication condition (a) poor lubrication Va Ka F L HB F L HB Va 4.4 10 mm
6 3

(b) good lubrication

Vb Kb

Vb 4.4 10 mm

3.

These numbers are entirely too large. The bushing will fail long before the ten years have gone by.

MACHINE DESIGN - An Integrated Approach, 4th Ed.

7-13-1

PROBLEM 7-13
Statement: Estimate how long it will take to file 1 mm off a 2-cm cube of HB150 steel if the machinist applies 100 N over a 10-cm stroke at 60 strokes per minute. (a) If done dry. (b) If done lubricated. kilo kg g Cube dimension Depth of wear Force on file a 2 cm d 1 mm F 100 N Stroke rate Stroke Steel hardness n 60 min s 10 cm kilo HB 150 2 mm
1

Units: Given:

Assumptions: Only one face will be filed. Solution: 1. See Mathcad file P0713.

This is a two-body abrasion problem. From Table 7-2, the wear coefficients for dry and lubricated abrasion using a file are Wear coefficients Ka 5 10 Kb 1 10
2 1

dry lubricated

2. 3. 4.

The length of sliding is L = s n t where t is the time required. The area of a face is Aa = a , and the depth is d = K
2

F L HB Aa

Combining these three equations and solving for the time, t d HB a


2

(a) dry

ta

Ka F s n

ta 20 min strokesa 1177

strokesa ta n d HB a
2

(b) lubricated

tb

Kb F s n

tb 10 min strokesb 588

strokesb tb n

MACHINE DESIGN - An Integrated Approach, 4th Ed.

7-14-1

PROBLEM 7-14
Statement: Figure P7-5 shows a child's toy called a pogo stick. The child stands on the pads, applying half her weight on each side. She jumps off the ground, holding the pads up against her feet, and bounces along with the spring cushioning the impact and storing energy to help each rebound. Estimate the abrasivbe wear rate for the tip, which impacts the ground assuming a condition of dry, loose abrasive grains (sand). Express the wear rate in number of jumps to remove 0.02 in from the 1-in-dia aluminum tip if its S ut = 50 ksi. kilo kg g Tip diameter Wear depth Tensile strength d 1.00 in a 0.02 in S ut 50 ksi

Units: Given:

Assumptions: The tip slides a distance of one tip diameter per jump. Solution: See Figure 7-14 and Mathcad file P0714.

1. From Problem 3-14, the impact force for each jump is P 224 lbf 2. Use equation (2.10) to estimate the Brinell hardness of the steel tip. HB

Fi /2

Fi /2

Sut kilo 530 psi 2 mm


1
3

HB 94

kilo mm
2

3. This is a two-body abrasion problem. From Table 7-2, the wear coefficient for dry abrasion using loose abrasive grains is K 1 10

4. The length of sliding (see assumption) is

L = d Njumps

where d is the tip diameter and Njumps is the number of jumps. Aa

P
FIGURE 7-14
Free Body Diagram for Problem 7-14

5. The area of the tip is

d
4

Aa 0.785 in

and the wear depth is

a = K

P L HB Aa Njumps a HB Aa K d P Njumps 9410

6. Substituting for L and solving for the number of jumps,

MACHINE DESIGN - An Integrated Approach, 4th Ed.

7-15-1

PROBLEM 7-15
Statement: Create a table of acceptable materials to run against a steel shaft based on their metallurical compatibility. Rank them as to suitability. See Figure 7-6 in text.

Solution:

MACHINE DESIGN - An Integrated Approach, 4th Ed.

7-16-1

PROBLEM 7-16
Statement: Determine the size of the contact patch and the maximum contact stresses for a 20-mm-dia steel ball rolled against a flat aluminum plate with 1 kN. Ball radius Plate curvature Load R1 10 mm R2 mm F 1 kN Aluminum plate Material properties Steel ball

Given:

1 0.28
E1 206.8 GPa

2 0.34
E2 71.7 GPa

Solution:

See Mathcad file P0716.

1. Calculate geometry and material constants, contact patch dimension, and pressures. Geometry constant B 1 2 1

R1

R2
1

B 0.05 mm

Material constants

m1

1 1 E1 1 2 E2

m1 4.456 10

1 MPa 1 MPa

m2

m2 1.233 10
1

Contact patch radius

3 m1 m2 a F B 8
A a p avg p max
2

a 0.501 mm A 0.789 mm
2

Contact area Average pressure

F A 3 2 p avg

p avg 1267 MPa p max 1900 MPa

Maximum pressure

2. Determine the stresses in the ball at the surface Axial

zmax p max xmax1


1 2 1 2 p max

zmax 1900 MPa xmax1 1482 MPa

In-plane

ymax1 xmax1
3. Determine the stresses in the ball below the surface Max shear stress

yzmax1

p max

1 2 1 2 1 1 2 1 1 9 2 2

yzmax1 641.5 MPa

MACHINE DESIGN - An Integrated Approach, 4th Ed.


2 2 1 7 2 1

7-16-2

Depth at max shear stress

zmax1 a

zmax1 0.316 mm

4. Determine the stresses in the plate at the surface Axial

zmax p max xmax2


1 2 2 2 p max

zmax 1900 MPa xmax2 1596 MPa

In-plane

ymax2 xmax2
5. Determine the stresses in the plate below the surface p max

Max shear stress

yzmax2

1 2 2 2 1 2 2 1 2 9 2 2

yzmax2 615.2 MPa


Depth at max shear stress 2 2 2 7 2 2

zmax2 a

zmax2 0.326 mm

MACHINE DESIGN - An Integrated Approach, 4th Ed.

7-17-1

PROBLEM 7-17
Statement: Determine the size of the contact patch and the maximum contact stresses for a 20-mm-dia steel ball rolled against a 30-mm diameter aluminum ball with 800 N. Steel ball radius Aluminum ball radius Load R1 10 mm R2 15 mm F 0.8 kN Aluminum ball Material properties: Steel ball

Given:

1 0.28
E1 206.8 GPa

2 0.34
E2 71.7 GPa

Solution: 1.

See Mathcad file P0717.

Calculate geometry and material constants, contact patch dimension, and pressures. Geometry constant B 1 2 1

R1

R2

B 0.083 mm

Material constants

m1

1 1 E1 1 2 E2

m1 4.456 10

1 MPa 1 MPa

m2

m2 1.233 10
1

Contact patch radius

3 m1 m2 a F B 8
A a p avg p max
2

a 0.392 mm A 0.484 mm
2

Contact area Average pressure

F A 3 2 p avg

p avg 1653 MPa p max 2480 MPa

Maximum pressure

2.

Determine the stresses in the steel ball at the surface Axial

zmax p max xmax1


1 2 1 2 p max

zmax 2480 MPa xmax1 1934 MPa

In-plane

ymax1 xmax1
3. Determine the stresses in the steel ball below the surface p max

Max shear stress

yzmax1

1 2 1 2 1 1 2 1 1 9 2 2

yzmax1 837.1 MPa

MACHINE DESIGN - An Integrated Approach, 4th Ed.

7-17-2

Depth at max shear stress

zmax1 a

2 2 1 7 2 1

zmax1 0.247 mm

4.

Determine the stresses in the aluminum ball at the surface Axial

zmax p max xmax2


1 2 2 2 p max

zmax 2480 MPa xmax2 2083 MPa

In-plane

ymax2 xmax2
5. Determine the stresses in the aluminum ball below the surface p max

Max shear stress

yzmax2

1 2 2 2 1 2 2 1 2 9 2 2

yzmax2 802.9 MPa


Depth at max shear stress 2 2 2 7 2 2

zmax2 a

zmax2 0.256 mm

MACHINE DESIGN - An Integrated Approach, 4th Ed.

7-18-1

PROBLEM 7-18
Statement: Given: Determine the size of the contact patch and the maximum contact stresses for a 40-mm-dia steel cylinder, 25-cm long, rolled against a flat aluminum plate with 4 kN. Cylinder radius Plate curvature Load Contact length R1 20 mm R2 mm F 4 kN L 250 mm Material properties: Steel cylinder

1 0.28
E1 206.8 GPa

Aluminum plate 2 0.34 E2 71.7 GPa

Solution: 1.

See Mathcad file P0718.

Calculate geometry and material constants, contact patch dimension, and pressures. Geometry constant B 1 2 1

R1

R2
1

B 0.025 mm

Material constants

m1

1 1 E1 1 2 E2

m1 4.456 10

1 MPa 1 MPa

m2

m2 1.233 10
1

Contact patch half-width

2 m1 m2 F a B L
A 2 L a p avg p max F A 2 F

a 0.0827 mm A 41.36 mm
2

Contact area Average pressure

p avg 96.7 MPa p max 123.1 MPa

Maximum pressure 2.

a L

Determine the stresses in the cylinder at the surface. The maximum normal stresses in the center of the contact patch at the surface of the steel cylinder are found using equations 7.17a. Axial In-plane

zmax p max y1max 2 1 p max

zmax 123 MPa y1max 69.0 MPa

3. Determine the stresses in the cylinder below the surface. The maximum shear stress and its location under the surface are found from equations 7.17b. Max shear stress Depth at max shear stress 4.

13max 0.304 p max


zmax 0.786 a

13max 37.4 MPa


zmax 0.0650 mm

Determine the stresses in the plate at the surface. The maximum normal stresses in the center of the contact patch at the surface of the aluminum plate are found using equations 7.17a. Axial

zmax p max

zmax 123.1 MPa

MACHINE DESIGN - An Integrated Approach, 4th Ed.

7-18-2

In-plane 5.

y2max 2 2 p max

y2max 83.7 MPa

Determine the stresses in the plate below the surface. The maximum shear stress and its location under the surface are found from equations 7.17b. Max shear stress Depth at max shear stress

13max 0.304 p max


zmax 0.786 a

13max 37.4 MPa


zmax 0.0650 mm

MACHINE DESIGN - An Integrated Approach, 4th Ed.

7-19-1

PROBLEM 7-19
Statement: Given: Determine the size of the contact patch and the maximum contact stresses for a 40-mm-dia steel cylinder, 25-cm long, rolled against a parallel 50-mm-dia steel cylinder with 10 kN. Cylinder radius Cylinder radius Load Contact length R1 20 mm R2 25 mm F 10 kN L 250 mm Steel cylinder Material properties: Steel cylinder

1 0.28
E1 206.8 GPa

2 0.28
E2 206.8 GPa

Solution: 1.

See Mathcad file P0719.

Calculate geometry and material constants, contact patch dimension, and pressures. Geometry constant B 1 2 1

R1

R2
1

B 0.045 mm

Material constants

m1

1 1 E1 1 2 E2

m1 4.456 10

1 MPa 1 MPa

m2

m2 4.456 10
1

Contact patch half-width

2 m1 m2 F a B L
A 2 L a p avg p max F A 2 F

a 0.0710 mm A 35.51 mm
2

Contact area Average pressure

p avg 281.6 MPa p max 358.6 MPa

Maximum pressure 2.

a L

Determine the stresses in either cylinder at the surface. The maximum normal stresses in the center of the contact patch at the surface of the steel cylinder are found using equations 7.17a. Axial In-plane

zmax p max y1max 2 1 p max

zmax 358.6 MPa y1max 200.8 MPa

3.

Determine the stresses in either cylinder below the surface. The maximum shear stress and its location under the surface are found from equations 7.17b. Max shear stress Depth at max shear stress

13max 0.304 p max


zmax 0.786 a

13max 109.0 MPa


zmax 0.0558 mm

MACHINE DESIGN - An Integrated Approach, 4th Ed.

7-20-1

PROBLEM 7-20
Statement: Given: Determine the size of the contact patch and the maximum contact stresses for a 20-mm-dia steel ball rolled against a 40-mm-dia steel cylinder, 25-mm long with 10 kN force. Ball radii Cylinder radii R1 10.00 mm R'1 10.00 mm R2 20.00 mm (radial) R'2 mm (axial) The parts are steel. Therefore: E 206.8 GPa 0.28

(normal to contact plane) Radial load F 10 kN Angle between planes of R1 and R2 0 deg Assumptions: The relative motion is rolling with < 1% sliding. Solution: 1. See Mathcad file P0720. 1 E
2

Find the material constants from equation 7.9b. Material constants m1 m1 4.456 10
6

1 MPa

m2 m1 2. Two geometry constants are needed from equations 7.19a. Geometry constants A 1 2 1 1 R'1 1 R2 1

R1

R'2
1

A 0.1250 mm

2 2 1 1 1 1 B R R'1 2 R1 2 R'2 1 1 1 1 2 R R' R R' cos( 2 ) 1 2 2 1

B 0.0250 mm

Angle Factors from equations 7.19e

B 180 acos A ka 50.192


0.86215

78.46
ka 1.167
2

kb 0.0045333 0.043581 0.0017292 3.7374 10 1.4207 10 3.


5 9

3.7418 10
5

kb 0.875

Determine the contact patch dimensions using the material and geometry constants in equations 7.19d.
1

Major axis half-width

3 3 m1 m2 a ka F A 4 1

a 0.947 mm

Minor axis half-width

3 3 m1 m2 b kb F A 4

b 0.710 mm

MACHINE DESIGN - An Integrated Approach, 4th Ed.


Contact area 4. A a b A 2.11354 mm
2

7-20-2

The average and maximum contact pressure can now be found from equations 7.18b and c. Average pressure Maximum pressure p avg p max F A 3 2 p avg p avg 4731 MPa p max 7097 MPa

5.

The maximum normal stresses in the center of the contact patch at the surface are then found using equations 7.21a. In-plane

x 2 ( 1 2 )

a b a a b

p max p max

x 5312 MPa y 5759 MPa z 7097 MPa

y 2 ( 1 2 )
Axial

z p max

These stresses are principal:

1 x

2 y

3 z

The maximum shear stress associated with them at the surface is

13
6.

1 3
2

13 892 MPa

The maximum shear stress under the surface on the z-axis is approximately Max shear stress

13max 0.34 p max

13max 2413 MPa

7.

All of the stresses found so far exist on the centerline of the patch. At the edge of the patch, at the surface, there will also be a shear stress. Two constants are found from equation 7.21b for this calculation. k3 b a 1 a a b
2 2

k3 0.75

k4 8.

k4 0.662

These constants are used in equations 7.21c and d to find the shear stresses on the surface at the ends of the major and minor axes. k3 k4 Minor axis
2

Major axis

xy ( 1 2 )

k4

atanh k4 1 p max

xy 1084 MPa

xy ( 1 2 )

k3 k4
2

k3 k4

atan

k4 pmax k3

xy 966 MPa

MACHINE DESIGN - An Integrated Approach, 4th Ed.

7-21-1

PROBLEM 7-21
Statement: A cam-follower system has a dynamic load of 0 to 2 kN. The cam is cylindrical with a minimum radius of curvature of 20 mm. The roller follower is crowned with radii of 15 mm in one direction and 150 mm in the other. Find the contact stresses if the follower is steel and the cam is nodular iron. Roller radius Crown radius Cam curvature Cam curvature R1 15.00 mm R'1 150.00 mm(90 deg to roller rad) R2 20.00 mm (radial) R'2 mm (axial)

Given:

(normal to contact plane) Radial load F 2 kN Angle between planes of R1 and R2 0 deg Material properties E1 206.8 GPa 1 0.28 E2 172.4 GPa 2 0.30 Assumptions: The relative motion is rolling with < 1% sliding. Solution: 1. See Mathcad file P0721. steel cast iron

Find the material constants from equation 7.9b. Material constants m1 1 1 E1 1 2 E2


2 2

m1 4.456 10

1 GPa 1 GPa

m2 2.

m2 5.278 10

Two geometry constants are needed from equations 7.19a. Geometry constants A 1 2 1 1 R'1 1 R2

R1

R'2
1
1

A 0.0617 mm

2 2 1 1 1 1 B R R'1 2 R1 2 R'2 1 1 1 1 2 R R' R R' cos( 2 ) 1 2 2 1

B 0.0550 mm

Angle

B 180 acos A ka 50.192


0.86215

26.89

Factors from equations 7.19e

ka 2.939
2

kb 0.0045333 0.043581 0.0017292 3.7374 10 1.4207 10


5 9

3.7418 10
5

kb 0.477

MACHINE DESIGN - An Integrated Approach, 4th Ed.


3. Determine the contact patch dimensions using the material and geometry constants in equations 7.19d.
1

7-21-2

Major axis half-width

3 3 m1 m2 a ka F A 4 1 3

a 1.818 mm

Minor axis half-width

3 m1 m2 b kb F A 4
A a b

b 0.295 mm

Contact area 4.

A 1.68582 mm

The average and maximum contact pressure can now be found from equations 7.18b and c. Average pressure p avg F A 3 2 p avg p avg 1186 MPa

Maximum pressure

p max

p max 1780 MPa

5.

The maximum normal stresses in the steel follower at the center of the contact patch at the surface are then found using equations 7.21a. In-plane

x 2 1 1 2 1

a b a a b

p max p max

x 1106 MPa y 1670 MPa z 1780 MPa

y 2 1 1 2 1
Axial

z p max

These stresses are principal:

1 x

2 y

3 z

The maximum shear stress associated with them at the surface is

13
6.

1 3
2

13 337 MPa

The maximum shear stress under the surface on the z-axis is approximately Max shear stress

13max 0.34 p max

13max 605 MPa

7.

All of the stresses found so far exist on the centerline of the patch. At the edge of the patch, at the surface, there will also be a shear stress. Two constants are found from equation 7.21b for this calculation. k3 k4 b a 1 a a b
2 2

k3 0.162 k4 0.987

MACHINE DESIGN - An Integrated Approach, 4th Ed.

7-21-3

8.

These constants are used in equations 7.21c and d to find the shear stresses on the surface at the ends of the major and minor axes.

Major axis

xy 1 2 1

k3 k4
2

k4

atanh k4 1 p max

xy 201 MPa

Minor axis

xy 1 2 1

k3 k4
2

k3 k4

atan

k4 p max k3

xy 100 MPa

9.

The maximum normal stresses in the nodular iron cam at the center of the contact patch at the surface are then found using equations 7.21a. In-plane

x 2 2 1 2 2

a b a a b

p max p max

x 1167 MPa y 1680 MPa z 1780 MPa

y 2 2 1 2 2
Axial

z p max

These stresses are principal:

1 x

2 y

3 z

The maximum shear stress associated with them at the surface is

13

1 3
2

13 306 MPa

10. The maximum shear stress under the surface on the z-axis is approximately Max shear stress

13max 0.34 p max

13max 605 MPa

11. All of the stresses found so far exist on the centerline of the patch. At the edge of the patch, at the surface, there will also be a shear stress. Two constants are found from equation 7.21b for this calculation. k3 k4 b a 1 a a b
2 2

k3 0.162 k4 0.987

12. These constants are used in equations 7.21c and d to find the shear stresses on the surface at the ends of the major and minor axes. Major axis

xy 1 2 2

k3 k4
2

k4

atanh k4 1 p max

xy 183 MPa

Minor axis

xy 1 2 2

k3 k4
2

k3 k4

atan

k4 p max k3

xy 91 MPa

MACHINE DESIGN - An Integrated Approach, 4th Ed.

7-22-1

PROBLEM 7-22
Statement: An "inline" skate is shown in Figure P7-10. The polyurethane wheels are 72 mm dia. by 12-mm thick with a 6-mm crown radius and are spaced on 104- mm centers. The skate-boot-foot combination weighs 2 kg. The effective "spring rate" of the person-skate subsystem is 6000 N/m. Find the contact stresses in the wheels when a 100-kg person lands a 0.5-m jump on one foot on concrete. Assume the urethane wheels and concrete have the properties below. (a) Assume that all 4 wheels land simultaneously. (b) Assume that one wheel absorbs all the landing force. Force per wheel (from Problem 3-22): Case (a) Fa 897 N Material properties: Urethane E1 600 MPa Concrete E2 21 GPa Wheel dimensions Concrete dimensions Contact angle Fb 3.59 kN

Given:

Case (b)

1 0.4 2 0.2
R'1 6 mm R'2 mm

R1 36 mm R2 mm

0 deg

Assumptions: The relative motion is rolling with < 1% sliding. Solution: 1. See Mathcad file P0722.

Find the material constants from equation 7.9b. Material constants m1 1 1 E1 1 2 E2


2 2

m1 1.400

1 GPa 1 GPa

m2 2.

m2 0.0457

Two geometry constants are needed from equations 7.19a. Geometry constants A 1 2 1 1 R'1 1 R2

R1

R'2
1
1

A 0.0972 mm

2 2 1 1 1 1 B R R'1 2 R1 2 R'2 1 1 1 1 2 R R' R R' cos( 2 ) 1 2 2 1

B 0.0694 mm

Angle

B 180 acos A ka 50.192


0.86215

44.42

Factors from equations 7.19e

ka 1.906
2

kb 0.0045333 0.043581 0.0017292 3.7374 10 1.4207 10


5 9

3.7418 10
5

kb 0.593

MACHINE DESIGN - An Integrated Approach, 4th Ed.

7-22-2

Part (a) 3. Determine the contact patch dimensions using the material and geometry constants in equations 7.19d.
1

Major axis half-width

3 m1 m2 a ka Fa A 4

a 4.108 mm
1

Minor axis half-width

3 3 m1 m2 b kb Fa A 4

b 1.278 mm

Contact area 4.

Ac a b

Ac 16.49024 mm

The average and maximum contact pressure can now be found from equations 7.18b and c. Average pressure p avg Fa Ac 3 2 p avg p avg 54.4 MPa

Maximum pressure

p max

p max 81.6 MPa

5.

The maximum normal stresses in the center of the contact patch at the surface are then found using equations 7.21a. In-plane

x 2 1 1 2 1

a b a a b

p max p max

x 69.1 MPa y 77.7 MPa z 81.6 MPa

y 2 1 1 2 1
Axial

z p max

These stresses are principal:

1 x

2 y

3 z

The maximum shear stress associated with them at the surface is

13
6.

1 3
2

13 6.22 MPa

The maximum shear stress under the surface on the z-axis is approximately Max shear stress

13max 0.34 p max

13max 27.7 MPa

7.

All of the stresses found so far exist on the centerline of the patch. At the edge of the patch, at the surface, there will also be a shear stress. Two constants are found from equation 7.21b for this calculation. k3 b a k3 0.311

MACHINE DESIGN - An Integrated Approach, 4th Ed.

7-22-3

k4 8.

1 a

a b

k4 0.95

These constants are used in equations 7.21c and d to find the shear stresses on the surface at the ends of the major and minor axes.

Major axis

xy 1 2 1

k3 k4
2

k4

atanh k4 1 p max

xy 5.24 MPa

Minor axis

xy 1 2 1

k3 k4
2

k3 k4

atan

k4 p max k3

xy 3.31 MPa

Part (b) 9. Determine the contact patch dimensions using the material and geometry constants in equations 7.19d.
1

Major axis half-width

3 3 m1 m2 a ka Fb A 4 1 3 3 m1 m2 b kb Fb A 4

a 6.522 mm

Minor axis half-width

b 2.029 mm

Contact area

Ac a b

Ac 41.56824 mm

10. The average and maximum contact pressure can now be found from equations 7.18b and c. Average pressure p avg Fb Ac 3 2 p avg p avg 86.4 MPa

Maximum pressure

p max

p max 129.5 MPa

11. The maximum normal stresses in the center of the contact patch at the surface are then found using equations 7.21a. In-plane

x 2 1 1 2 1

a b a a b

p max p max

x 109.8 MPa y 123.4 MPa z 129.5 MPa

y 2 1 1 2 1
Axial

z p max

These stresses are principal:

1 x

2 y

3 z

The maximum shear stress associated with them at the surface is

MACHINE DESIGN - An Integrated Approach, 4th Ed.

7-22-4

13

1 3
2

13 9.88 MPa

12. The maximum shear stress under the surface on the z-axis is approximately Max shear stress

13max 0.34 p max

13max 44.0 MPa

13. All of the stresses found so far exist on the centerline of the patch. At the edge of the patch, at the surface, there will also be a shear stress. Two constants are found from equation 7.21b for this calculation. k3 k4 b a 1 a a b
2 2

k3 0.311 k4 0.95

14. These constants are used in equations 7.21c and d to find the shear stresses on the surface at the ends of the major and minor axes. Major axis

xy 1 2 1

k3 k4
2

k4

atanh k4 1 p max

xy 8.31 MPa

Minor axis

xy 1 2 1

k3 k4
2

k3 k4

atan

k4 p max k3

xy 5.26 MPa

MACHINE DESIGN - An Integrated Approach, 4th Ed.

7-23-1

PROBLEM 7-23
Statement: Given: A pair of 12-in dia cylindrical steel rolls run together with 9% slip. Find their contact stresses for a radial contact forces of 1000 lb/in of length. Roller radii R1 6.00 in R2 6.00 in Radial load/in F' 1000 lbf in
1

The parts are hardened steel. Therefore: E 30 10 psi


6

0.28

Assumptions: The coefficient of friction is 0.33. Solution: 1. See Mathcad file P0723.

Find the material constants from equation 7.9a. Material constants m1 1 E


2

m1 3.072 10

8 1

psi

m2 m1 Geometry constant B 1 2 1

R1

R2
1
1 2

B 0.167 in

Contact patch half-width 2.

2 m1 m2 a F' B
F' 2 a 2 F'

a 0.0153 in

The average and maximum contact pressure can now be found from equations 7.14b and c. Average pressure p avg p max p avg 32.6 ksi p max 41.6 ksi fmax 13.7 ksi

Maximum pressure Tangential pressure 3.

fmax p max

With m = 0.33, the principal stresses in the contact zone will be maximal on the surface (z = 0) at x = 0.3a from the centerline as shown in Figures 7-20 and 7-22. The applied stress components are found from equation 7.23a for the normal force and equation 7.23b for the tangential force. For x 0.3 a

xn p max 1
x a

2 2

xn 39.6 ksi

xt 2 fmax

xt 8.23 ksi
x
2 2

zn p max 1 zt 0 ksi

zn 39.6 ksi xzn 0 ksi

MACHINE DESIGN - An Integrated Approach, 4th Ed.


x
2 2

7-23-2

xzt fmax 1
4.

xzt 13.1 ksi

Equations 7.24a and 7.24b can now be solved for the total applied stresses along the x, y, and z axes.

x xn xt z zn zt xz xzn xzt
5.

x 47.871 ksi z 39.642 ksi xz 13.082 ksi

Assuming the rollers are long, we expect a plane strain condition to exist. The stress in the third dimension is found from equation 7.23b:

y x z
6.

y 24.504 ksi

Unlike the pure-rolling case, these stresses are not principal because of the applied shear stress. The principal stresses are found from equation 4.4 using a cubic root finding solution.

x
ksi

y
ksi

z xz

z
ksi

xy 0
C2 x y z C1

yz 0

xz
ksi

C2 112.017 C1 3.871 10
3

x xy x xz y yz xy y xz z yz z

x xy xz C0 xy y yz xz yz z
f ( ) C2 C1 C0
3 2

C0 4.231 10

C0 C1 v C2 1
Principal stresses:

s polyroots ( v) ksi

57.5 s 30.0 ksi 24.5

1 s 2 s 3 s

3 2 1

1 24.5 ksi 2 30.0 ksi 3 57.5 ksi

Maximum shear stress

13
7.

1 3
2

13 16.5 ksi

The principal stresses are maximum at the surface as seen in Figures 7-20 and 7-22 in the text.

MACHINE DESIGN - An Integrated Approach, 4th Ed.

7-24-1

PROBLEM 7-24
Statement: Given: Estimate the cycle life of the rolls in Problem 7-23 if they are made of class 30 gray cast iron, austempered to HB270. Roller radii R1 6.00 in R2 6.00 in Radial load/in F' 1000 lbf in
1

The parts are gray cast iron. Therefore: E 15 10 psi


6

0.28

Assumptions: The coefficient of friction is 0.33. Solution: 1. See Mathcad file P0724.

Find the material constants from equation 7.9a. Material constants m1 1 E


2

m1 6.144 10

8 1

psi

m2 m1 Geometry constant B 1 2 1 1

R1

R2
1 2

B 0.167 in

Contact patch half-width 2.

2 m1 m2 F' B
F' 2 a 2 F'

a 0.0217 in

The average and maximum contact pressure can now be found from equations 7.14b and c. Average pressure p avg p max p avg 23.1 ksi p max 29.38 ksi fmax 9.70 ksi

Maximum pressure Tangential pressure 3.

fmax p max

With m = 0.33, the principal stresses in the contact zone will be maximal on the surface (z = 0) at x = 0.3a from the centerline as shown in Figures 7-20 and 7-22. The applied stress components are found from equation 7.23a for the normal force and equation 7.23b for the tangential force. For x 0.3 a

xn p max 1
x a

2 2

xn 28.0 ksi

xt 2 fmax

xt 5.82 ksi
x
2 2

zn p max 1 zt 0 ksi

zn 28.0 ksi xzn 0 ksi

MACHINE DESIGN - An Integrated Approach, 4th Ed.

7-24-2

xzt fmax 1
4.

2 2

xzt 9.25 ksi

Equations 7.24a and 7.24b can now be solved for the total applied stresses along the x, y, and z axes.

x xn xt z zn zt xz xzn xzt
5.

x 33.850 ksi z 28.031 ksi xz 9.250 ksi

Assuming the rollers are long, we expect a plane strain condition to exist. The stress in the third dimension is found from equation 7.23b:

y x z
6.

y 17.327 ksi

Unlike the pure-rolling case, these stresses are not principal because of the applied shear stress. The principal stresses are found from equation 4.4 using a cubic root finding solution.

x
ksi

y
ksi

z xz

z
ksi

xy 0
C2 x y z C1

yz 0

xz
ksi

C2 79.208 C1 1.935 10
3

x xy x xz y yz xy y xz z yz z

x xy xz C0 xy y yz xz yz z
f ( ) C2 C1 C0
3 2

C0 1.496 10

C0 C1 v C2 1
Principal stresses:

s polyroots ( v) ksi

40.6 s 21.2 ksi 17.3

1 s 2 s 3 s
7.

3 2 1

1 17.3 ksi 2 21.2 ksi 3 40.6 ksi

The maximum normal stress calculated in step 6 is 40.9 ksi, compressive. Its K-factor can be calculated from equation 7.25d. K-factor K m1 m2 3
2

K 637.5 psi

MACHINE DESIGN - An Integrated Approach, 4th Ed.

7-24-3

8.

From Table 7-7, Part 2, Line 33 the slope and intercept factors of this cast iron for rolling with 9% sliding are

7.87
9.

35.90

These are used in equation 7.26 along with the value of K from above to find the number of cycles that can be expected at this load before pitting begins.

log ( K) =

log Nlife
log

Nlife 10

K psi

Nlife 6.7 10

13

cycles

MACHINE DESIGN - An Integrated Approach, 4th Ed.

7-25-1

PROBLEM 7-25
Statement: A 12-mm-dia, class 30 gray cast iron pad is supported by a steel bar made from SAE 4340 steel, quenched and tempered at 800F. The force on the pad is 3.8 kN. Estimate the real area of contact and the ratio of the real area to the apparent area of contact. Pad diameter d 12 mm Compressive yield strengths: Contact force F 3.8 kN

Given:

Steel Cast iron

S ycs 1469 MPa S ycci 752 MPa

Assumptions: The compressive yield strength of the cast iron is approximately the same as the compressive strength. Solution: 1. See Mathcad file P0725.

Calculate the apparent area of contact. Aa

d
4

Aa 113.097 mm

2.

Use equation 7.1 and the cast iron strength, which is the weaker of the two, to estimate the real area of contact. Ar F 3 S ycci Ar 1.6844 mm
2

3.

The ratio of real to apparent area of contact is: ratio Ar Aa ratio 1.5 %

MACHINE DESIGN - An Integrated Approach 4th Ed.

7-26-1

PROBLEM 7-26
Statement: Estimate the dry coefficient of friction between the two materials in Problem 7-35 if the shear strength of the cast iron is S us = 310 MPa. How does this compare to the value given in Table 7-1? Pad diameter Shear strength: d 12 mm Cast iron S us 310 MPa S yc 752 MPa

Given:

Compressive yield strength

Assumptions: The compressive yield strength of the cast iron is approximately the same as the compressive strength. Solution: 1. See Table 7-1 and Mathcad file P0726.

Using equation 7.3, the coefficient of friction is approximately:

S us 3 S yc

0.14

From Table 7-1, the coefficient of friction for mild steel on cast iron (lubricated) is 0.183.

MACHINE DESIGN - An Integrated Approach, 4th Ed.

7-27-1

PROBLEM 7-27
Statement:

_____

Two 0.5-in x 1-in 1040 hot-rolled steel pads are in contact with a force of 900 lb. Estimate the real area of contact and the ratio of the real area to the apparent area of contact. Pad dimensions Compressive yield strength: Contact force L 1.00 in S yc 42 ksi F 900 lbf w 0.5 in

Given:

Assumptions: The compressive yield strength of the steel is approximately the same as the tensile yield strength. Solution: 1. See Mathcad file P0727.

Calculate the apparent area of contact. Aa L w Aa 0.500 in


2

2.

Use equation 7.1 and the compressive yield strength to estimate the real area of contact. Ar F 3 S yc Ar 0.0071 in
2

3.

The ratio of real to apparent area of contact is: Ar Aa

ratio

ratio 1.43 %

MACHINE DESIGN - An Integrated Approach, 4th Ed.

7-28-1

PROBLEM 7-28
Statement:

_____

Estimate the dry coefficient of friction between the two materials in Problem 7-27. How does this compare to the value given in Table 7-1? Pad dimensions Ultimate tensile strength: Compressive yield strength: L 1.0 in S ut 76 ksi S yc 42 ksi w 0.50 in

Given:

Assumptions: The compressive yield strength of the steel is approximately the same as the tensile yield strength. Solution: 1. See Mathcad file P0728.

Estimate the ultimate shear strength using equation 7.4 for steel. S us 0.8 S ut S us 60.8 ksi

2.

Using equation 7.3, the coefficient of friction is approximately:

S us 3 S yc

0.48

MACHINE DESIGN - An Integrated Approach, 4th Ed.

7-29-1

PROBLEM 7-29
Statement:

_____

Two materials have been tested to determine the amount of adhesive wear that takes place when they are run together. The average depth of wear results are given in Table P7-1 along with the test parameters for a total of 350 tests. What is the average wear coefficient for the materials tested if the Brinell hardness of the softer of the two is HB 277. Hardness HB 277 kgf mm
2

Given:

Data from Table P7-1: F


i

i 1 4 l
i i

A 10 mm
i

NT 100 75 75 100

d
i

100 N 200 N 200 N 400 N Solution: 1.

5000 m 5000 m 7500 m 10000 m

0.180 mm 0.372 mm 0.550 mm 1.47 mm

See Table P7-1 and Mathcad file P0729.

The total number of tests is: Total

NT i
i

Total 350

2.

The average force, apparent contact area, length of sliding, and depth of wear are: Favg Aavg lavg d avg 1 Total 1 Total 1 Total 1 Total

F iNT i
i

Favg 228.571 N Aavg 10.000 mm


2

A iNT i
i i

liNT i

lavg 6.964 10 m d avg 0.669 mm

diNT i
i

3.

Calculate the average adhesive wear coefficient using equation 7.7b and the values calculated above. HB Aavg d avg Favg lavg

Kavg

Kavg 1.14 10

MACHINE DESIGN - An Integrated Approach, 4th Ed.

7-30-1

PROBLEM 7-30
Statement:

_____

A piece of mild steel with HB = 280 has its thickness reduced in an abrasive grinder. Both the grinding wheel and the steel part have the same width, which is 20 mm. On each pass through the grinder 0.1 mm is removed. If the abrasive wear coefficient for this operation is 5E-1, what is the approximate normal force on the grinding wheel? Hardness Depth of wear per stroke Wear coefficient H 280 kgf mm Width of wear d 0.1 mm K 5 10
1 2

Given:

w 20 mm

Solution: 1.

See Mathcad file P0730.

From equation 7.7a, the volume of wear per stroke is V K F l H

2.

Solving for the normal force F, F V H K l where V d w l

3.

Substituting for V and canceling the l-terms, F d w H K F 11.0 kN

MACHINE DESIGN - An Integrated Approach, 4th Ed.

7-31-1

PROBLEM 7-31
Statement:

_____

Two steel gears with involute tooth profiles are in mesh. At the line of contact between the gears they can be modeled as two cylinders in contact. When the contact is away from the pitch point there is a combination of rolling and sliding. Determine and plot the dynamic contact principal stresses on the surface of the teeth for the following gears if the contact force is 500 lb and the coefficient of friction is 0.15: R1 = 2.000 in, R2 = 6.000 in. The thickness (face width) of both gears is 0.5 in. Also find the value of x/a for which the principal stresses have an extreme value. The parts are hardened steel. Therefore: E 30 10 psi 0.28 Radii of curvature R1 2.00 in R2 6.00 in Contact force Gear tooth thickness Coefficient of friction W 500 lbf F 0.5 in (face width) 0.15
6

Given:

Solution: 1.

See Mathcad file P0731.

Calculate the force per unit of length along the line of contact. F' W F F' 1000 lbf in
1

2.

Find the material constants from equation 7.9a. Material constants m1 1 E


2

m1 3.072 10

8 1

psi

m2 m1 B 1 2 1 1

Geometry constant

R1

R2
1 2

B 0.333 in

Contact patch half-width

2 m1 m2 F' B

a 0.0108 in

3.

The average and maximum contact pressure can now be found from equations 7.14b and c. Average pressure p avg p max F' 2 a 2 F' p avg 46.2 ksi p max 58.8 ksi fmax 8.8 ksi

Maximum pressure Tangential pressure 4.

fmax p max

The applied stress components are found from equation 7.23a for the normal force and equation 7.23b for the tangential force.

xn( x) if x a p max 1

2 2

0 psi

zn( x) xn( x)

MACHINE DESIGN - An Integrated Approach, 4th Ed.

7-31-2

xzn 0 ksi
2 x x 1 if x a xt( x) return 2 fmax a 2 a 2 x x return 2 fmax 1 if x a a 2 a

zt 0 ksi

2 fmax

x a

otherwise
2 2

xzt( x) if x a fmax 1

0 ksi

5.

Equations 7.24a can now be written for the total applied stresses along the x and z axes.

x( x) xn( x) xt( x) z( x) zn( x) zt xz( x) xzn xzt( x)


6. The face width of the gears is short with respect to the other dimensions, therefore we expect a plane stress condition to exist. The stress in the third dimension is

y 0 ksi
7. Unlike the pure-rolling case, these stresses are not principal because of the applied shear stress. The principal stresses are found from equation 4.6a for the plane stress case.

1( x)

x( x) z( x)
2

2 x( x) z( x) 2 xz( x) 2

2( x)

x( x) z( x)
2

2 x( x) z( x) 2 xz( x) 2

3 0 ksi

8.

Plot the two nonzero principal stresses over the range x 3 a 2.999 a 3 a

MACHINE DESIGN - An Integrated Approach, 4th Ed.

7-31-3

PRINCIPAL CONTACT STRESSES


0.5 0.18

0 Normalized stress

1( x)
pmax 0.5

2( x)
pmax

1.5 2

0 x a Normalized width, x/a

9.

Note that for = 0.15, the extreme values of the principal stresses occur at approximately x/a = 0.18.

MACHINE DESIGN - An Integrated Approach, 4th Ed.

7-32-1

PROBLEM 7-32
Statement:

_____

Two steel gears with involute tooth profiles are in mesh. At the line of contact between the gears they can be modeled as two cylinders in contact. When the contact is away from the pitch point there is a combination of rolling and sliding. Determine the dynamic contact stresses on the surface of the teeth for the following gears if the contact force is 1500 lb and the coefficient of friction is 0.33: R1 = 2.500 in, R2 = 5.000 in. The thickness (face width) of both gears is 0.625 in. The parts are hardened steel. Therefore E 30 10 psi 0.28 Radii of curvature R1 2.500 in R2 5.000 in Contact force Gear tooth thickness Coefficient of friction W 1500 lbf F 0.625 in (face width) 0.33
6

Given:

Solution: 1.

See Mathcad file P0732.

Calculate the force per unit of length along the line of contact. F' W F F' 2400 lbf in
1

2.

Find the material constants from equation 7.9a. Material constants m1 1 E


2

m1 3.072 10

8 1

psi

m2 m1 B 1 2 1 1

Geometry constant

R1

R2
1 2

B 0.300 in

Contact patch half-width

2 m1 m2 F' B

a 0.0177 in

3.

The average and maximum contact pressure can now be found from equations 7.14b and c. Average pressure p avg p max F' 2 a 2 F' p avg 67.8 ksi p max 86.4 ksi fmax 28.5 ksi

Maximum pressure Tangential pressure 4.

fmax p max

With = 0.33, the principal stresses in the contact zone will be maximal on the surface (z = 0) at x = 0.3a from th centerline as shown in Figures 7-20 and 7-22. The applied stress components are found from equation 7.23a for the normal force and equation 7.23b for the tangential force. For x 0.3 a

xn p max 1

2 2

xn 82.4 ksi

MACHINE DESIGN - An Integrated Approach, 4th Ed.

7-32-2

xt 2 fmax

x a x
2 2

xt 17.10 ksi

zn p max 1 zt 0 ksi xzt fmax 1


5.

zn 82.4 ksi xzn 0 ksi

2 2

xzt 27.2 ksi

Equations 7.24a can now be written for the total applied stresses along the x and z axes.

x xn xt z zn zt xz xzn xzt
6.

x 99.497 ksi z 82.395 ksi xz 27.190 ksi

The face width of the gears is short with respect to the other dimensions, therefore we expect a plane stress condition to exist. The stress in the third dimension is

y 0 ksi
7. Unlike the pure-rolling case, these stresses are not principal because of the applied shear stress. The principal stresses are found from equation 4.6a for the plane stress case.

1 0 ksi x z
2
2 x z 2 xz 2 2 x z 2 xz 2

2 62.443 ksi

x z
2

3 119.449 ksi

Maximum shear stress

13

1 3
2

13 59.7 ksi

MACHINE DESIGN - An Integrated Approach, 4th Ed.

7-33-1

PROBLEM 7-33
Statement:

_____

Two contacting rollers are needed for a machine application. They run together with a combination of rolling and 9% sliding. Both are to be made from 1144 CD steel. The radial contact force is 1200 N and the coefficient of friction is 0.33. The rollers are to have the same radii and are both 10 mm long. If the design life is 8E08 cycles, determine a suitable radius for the rollers. The parts are hardened steel. Therefore Contact force W 1200 N Roller length w 10 mm Coefficient of friction E 30 10 psi
6

Given:

0.28

0.33 4.10

Design life

Nd 8 10

Material data from Table 7-7 Solution: 1. See Mathcad file P0733.

21.79

Calculate the force per unit of length along the line of contact. F' W w F' 120.00 N mm
1

2.

Find the material constants from equation 7.9a. Material constants m1 1 E


2

m1 4.456 10

1 MPa

m2 m1 B( R) 1 2 1 1

Geometry constant

R
1 2

Contact patch half-width

a ( R)

2 m1 m2 F' B( R)

3.

The average and maximum contact pressure can now be found from equations 7.14b and c. Average pressure p avg ( R) p max ( R) F' 2 a ( R) 2 F'

Maximum pressure Tangential pressure 4.

a ( R)

fmax ( R) p max ( R)

With = 0.33, the principal stresses in the contact zone will be maximal on the surface (z = 0) at x = 0.3a from th centerline as shown in Figures 7-20 and 7-22. The applied stress components are found from equation 7.23a for the normal force and equation 7.23b for the tangential force. For x( R) 0.3 a ( R)

xn( R) p max ( R) 1 0.3 xzn 0 MPa

zn( R) p max ( R) 1 0.3

MACHINE DESIGN - An Integrated Approach, 4th Ed.

7-33-2

xt( R) 2 fmax ( R) 0.3 xzt( R) fmax ( R) 1 0.3


5.
2

zt 0 MPa

Equations 7.24a can now be written for the total applied stresses along the x and z axes.

x( R) xn( R) xt( R) z( R) zn( R) zt xz( R) xzn xzt( R)


6. The face width of the rollers is short with respect to the other dimensions, therefore we expect a plane stress condition to exist. The stress in the third dimension is

y 0 MPa
7. Unlike the pure-rolling case, these stresses are not principal because of the applied shear stress. The principal stresses are found from equation 4.6a for the plane stress case.

1 0 MPa x( R) z( R)
2
2 x( R) z( R) 2 xz( R) 2

2( R)

3( R)

x( R) z( R)
2

2 x( R) z( R) 2 xz( R) 2

8.

Calculate the K-factor for this material and the number of cycles before the onset of pitting for this design using equation 7.26.
log Nd

K 10 9.

psi

K 1390 psi

Use equation 7.25e to solve for the roller radius. First, guess R 40 mm Given K = m1 m2 3( R) R Find ( R)
2

R 47.9 mm

MACHINE DESIGN - An Integrated Approach, 4th Ed.

7-34-1

PROBLEM 7-34
Statement:

_____

Two contacting rollers are needed for a machine application. They run together with a combination of rolling and 9% sliding. Both are to be made from Meehanite. The rollers have the same radii (30 mm) and are both 45 mm long. If the design life is 1E08 cycles, determine the allowable load that may be applied to these rollers. Material data from Table 7-7 Roller radii R1 30 mm Roller length L 45 mm K 1450 psi R2 30 mm

Given:

Solution: 1.

See Mathcad file P0734.

Find the geometry constant from equation 7.9a. Geometry constant B 1 2 1

R1

R2
1

B 0.033

1 mm

2.

Calculate the allowable force on the rollers using equation 7.25d. K L 2 B

Allowable force

F 6.75 kN

MACHINE DESIGN - An Integrated Approach, 4th Ed.

7-35-1

PROBLEM 7-35
Statement: A 25-mm-dia, Class 20 gray cast iron pad is supported by a steel bar made from SAE 4130 steel, quenched and tempered at 800F. The force on the pad is 2800 N. Estimate the real area of contact and the ratio of the real area to the apparent area of contact. Pad diameter d 25 mm Compressive yield strengths: Contact force F 2800 N Pad Bar S ycp 572 MPa S ycb 1193 MPa

Given:

Assumptions: The compressive yield strength of the cast iron is approximately the same as its compressive strength. Solution: 1. See Mathcad file P0735.

Calculate the apparent area of contact. Aa

d
4

Aa 490.9 mm

2.

Use equation 7.1 and the pad strength, which is the weaker of the two, to estimate the real area of contact. Ar F 3 S ycp Ar 1.632 mm
2

3.

The ratio of real to apparent area of contact is: ratio Ar Aa ratio 0.3 %

MACHINE DESIGN - An Integrated Approach, 4th Ed.

7-36-1

PROBLEM 7-36
Statement: Estimate the dry coefficient of friction between the two materials in Problem 7-35 if the shear strength of the cast iron is S us = 310 Mpa. How does this compare to the value given in Table 7-1? Pad diameter Shear strength: d 25 mm Cast iron S us 310 MPa S yc 572 MPa

Given:

Compressive yield strength

Assumptions: The compressive yield strength of the cast iron is approximately the same as the compressive strength. Solution: 1. See Mathcad file P0736.

Using equation 7.3, the coefficient of friction is approximately:

S us 3 S yc

0.18

From Table 7-1, the coefficient of friction for mild steel on cast iron (lubricated) is 0.183.

MACHINE DESIGN - An Integrated Approach, 4th Ed.

7-37-1

PROBLEM 7-37
Statement: Given: Two 25-mm x 40-mm SAE 1020 hot-rolled steel pads are in contact with a force of 9 kN. Estimate the real area of contact and the ratio of the real area to the apparent area of contact. Pad dimensions w 25 mm Compressive yield strength: Contact force F 9000 N L 40 mm S yc 379 MPa

Assumptions: The compressive yield strength of the steel is approximately the same as its tensile yield strength. Solution: 1. See Mathcad file P0737.

Calculate the apparent area of contact. Aa L w Aa 1000.0 mm


2

2.

Use equation 7.1 and the pad strength, which is the weaker of the two, to estimate the real area of contact. Ar F 3 S yc Ar 7.916 mm
2

3.

The ratio of real to apparent area of contact is: ratio Ar Aa ratio 0.79 %

MACHINE DESIGN - An Integrated Approach, 4th Ed.

7-38-1

PROBLEM 7-38
Statement: Given: Estimate the dry coefficient of friction between the two materials in Problem 7-37. How does this compare to the value given in Table 7-1? Pad dimensions w 25 mm L 40 mm Ultimate tensile strength: S ut 379 MPa Compressive yield strength: S yc 379 MPa

Assumptions: The compressive yield strength of the steel is approximately the same as the tensile yield strength. Solution: 1. See Table 7-1 and Mathcad file P0738.

Estimate the ultimate shear strength equation 7.4 for steel. S us 0.8 S ut S us 303.2 MPa

2.

Using equation 7.3, the coefficient of friction is approximately:

S us 3 S yc

0.27

From Table 7-1, the coefficient of friction for mild steel on mild steel is 0.74.

MACHINE DESIGN - An Integrated Approach, 4th Ed.

7-39-1

PROBLEM 7-39
Statement: A 25-mm-diameter steel shaft of hardness HB420 rotates at 700 rpm in a 40-mm-long plain bronze bushing with an average radial load of 500 N. Estimate the time it would take to remove 0.05 mm of bushing material by adhesive wear if the lubrication were suddenly lost assuming a uniform wear rate around the bushing. kilo kg g rev 2 rad rpm rev min
1

Units: Given:

Journal diameter d 25 mm Journal speed n 700 rpm Journal hardness HB 420 kilo mm
2

Load Bushing length Depth of wear

F 500 N lbush 40 mm d 0.05 mm

Solution: 1.

See Mathcad file P0739.

From Figure 7-6 in the text we see that iron (the principal ingredient of steel) is metalurgically compatible with copper and tin, the principal ingredients of bronze. From Figure 7-7 we have the following value of the adhesive wear coefficient for metalurgically compatible materials with no lubrication. Wear coefficient K 8 10
4

2.

Using equation 7.7b, estimate the time for the bushing to wear d 0.05 mm Apparent area of contact Length of contact in time t Aa d lbush lcont( t)

d
rev K F

n t

Solving equation 7.7b for l

lcont = t

d HB Aa

Substituting and solving for t

d HB lbush rev n K F

t 29.4 min

In a time of t 29.4 min the shaft will turn

n t

2 10 rev

MACHINE DESIGN - An Integrated Approach, 4th Ed.

7-40-1

PROBLEM 7-40
Statement: A machine has a tripod base that utilizes 15-mm-dia Nylon 11 balls as support pads at its feet. The tripod rests on a flat steel plate. The 360N weight of the machine is distributed equally to the three legs of the tripod. Determine the size of the contact patch and the contact stresses in the nylon balls. Assume that Poisson's ratio for Nylon is 0.25. Ball radius Plate curvature Load R1 7.5 mm R2 mm F 120 N Steel plate Material properties Nylon ball

Given:

1 0.25
E1 1.3 GPa

2 0.28
E2 207 GPa

Solution:

See Mathcad file P0740.

1. Calculate geometry and material constants, contact patch dimension, and pressures. Geometry constant B 1 2 1

R1
E1

R2
1

B 0.067 mm

Material constants

m1

1 1

m1 7.212 10

1 MPa 1 MPa

m2

1 2 E2

m2 4.452 10
1

Contact patch radius Contact area Average pressure

3 m1 m2 a F B 8
A a p avg p max
2

a 0.788 mm A 1.952 mm
2

F A 3 2 p avg

p avg 61 MPa p max 92 MPa

Maximum pressure

2. Determine the stresses in the ball at the surface Axial In-plane

zmax p max xmax1


1 2 1 2 p max

zmax 92 MPa xmax1 69 MPa

ymax1 xmax1
3. Determine the stresses in the ball below the surface Max shear stress

yzmax1

p max

1 2 1 2 1 1 2 1 1 9 2 2

yzmax1 31.8 MPa


Depth at max shear stress zmax1 a 2 2 1 7 2 1 zmax1 0.489 mm

MACHINE DESIGN - An Integrated Approach, 4th Ed.

7-41-1

PROBLEM 7-41
Statement: A ball bearing consists of a number of balls (separated by a ball cage) and two rings with raceways as shown in Figure P7-7. The raceways have compound curvature. In a plane containing the axis of the bearing the curvature is concave and conforms closely to the ball radius. In a plane perpendicular to the axis the curvature is convex for the inner raceway and is related to the bore size of the bearing. Determine the size of the contact patch and the maximum contact stresses between a ball and the inner raceway with a radial load of 5200 N in a steel bearing with the following dimensions: ball dia = 8 mm, raceway radius for concave surface = 4.05 mm, raceway radius for convex surface = 13 mm. Ball radii Raceway radii R1 4.00 mm R'1 4.00 mm R2 13.0 mm (radial) (normal to contact plane) 0 deg R'2 4.05 mm (axial) Radial load F 5200 N Angle between planes of R1 & R2 Solution: 1. See Mathcad file P0741. The parts are steel. Therefore E 206.8 GPa

Given:

0.28

Find the material constants from equation 7.9b. Material constants m1 1 E


2

m1 4.456 10

1 MPa

m2 m1 2. Two geometry constants are needed from equations 7.19a. Geometry constants A 1 2 1 1 R'1 1 R2 1

R1

R'2
1

A 0.1650 mm

2 2 1 1 1 1 B R R'1 2 R1 2 R'2 1 1 1 1 2 R R' R R' cos( 2 ) 1 2 2 1

B 0.1619 mm

Angle

B 180 acos A ka 50.192


0.86215

11.10

Factors from equations 7.19e

ka 6.301
2

kb 0.0045333 0.043581 0.0017292 3.7374 10 1.4207 10


5 9

3.7418 10
5

kb 0.321

MACHINE DESIGN - An Integrated Approach, 4th Ed.


3. Determine the contact patch dimensions using the material and geometry constants in equations 7.19d.
1

7-41-2

Major axis half-width

3 m1 m2 a ka F A 4

a 3.749 mm
1

Minor axis half-width

3 3 m1 m2 b kb F A 4

b 0.191 mm A 2.24904 mm
2

Contact area 4.

A a b

The average and maximum contact pressure can now be found from equations 7.18b and c. Average pressure p avg F A 3 2 p avg p avg 2312 MPa

Maximum pressure 5.

p max

p max 3468 MPa

The maximum normal stresses in the center of the contact patch at the surface are then found using equations 7.21a. In-plane

x 2 ( 1 2 )

a b a a b

p max p max

x 2016 MPa y 3394 MPa z 3468 MPa

y 2 ( 1 2 )
Axial

z p max

These stresses are principal:

1 x

2 y

3 z

The maximum shear stress associated with them at the surface is

13
6.

1 3
2

13 726 MPa

The maximum shear stress under the surface on the z-axis is approximately Max shear stress

13max 0.34 p max

13max 1179 MPa

7.

All of the stresses found so far exist on the centerline of the patch. At the edge of the patch, at the surface, there will also be a shear stress. Two constants are found from equation 7.21b for this calculation. k3 k4 b a 1 a a b
2 2

k3 0.051 k4 0.999

8.

These constants are used in equations 7.21c and d to find the shear stresses on the surface at the ends of the major and minor axes.

MACHINE DESIGN - An Integrated Approach, 4th Ed.

7-41-3

Major axis

xy ( 1 2 )

k3 k4
2

k4

atanh k4 1 p max

xy 208 MPa

Minor axis

xy ( 1 2 )

k3 k4
2

k3 k4

atan

k4 pmax k3

xy 72 MPa

MACHINE DESIGN - An Integrated Approach, 4th Ed.

7-42-1

PROBLEM 7-42
Statement: A pair of steel rollers used in a manufacturing process roll together with a combination of rolling and sliding. One roller has a diameter of 75 mm and the other has a diameter of 50 mm. They are both 200-mm long. The contact force, which is normal to the contact plane, is 18500 N. Assuming that the coefficient of friction between the rollers is 0.33, determine the maximum tensile, compressive, and shear stresses in the rollers. Roller radii Roller length Radial load Solution: 1. R1 25 mm R2 37.5 mm L 200 mm F 18500 N The parts are hardened steel. Therefore E 206.8 GPa Coefficient of friction

Given:

0.28 0.33

See Mathcad file P0742.

Find the material constants from equation 7.9a. Material constants m1 1 E


2

m1 4.456 10

1 MPa

m2 m1 Geometry constant B 1 2 1

R1

R2
1
1 2

B 0.033 mm

Contact patch half-width 2.

2 m1 m2 F a B L
F 2 a L 2 F

a 0.1255 mm

The average and maximum contact pressure can now be found from equations 7.14b and c. Average pressure p avg p max p avg 368.6 MPa p max 469.3 MPa fmax 154.9 MPa

Maximum pressure Tangential unit force 3.

a L

fmax p max

With = 0.33, the principal stresses in the contact zone will be maximal on the surface (z = 0) at x = 0.3a from th centerline as shown in Figures 7-20 and 7-22. The applied stress components are found from equation 7.23a for the normal force and equation 7.23b for the tangential force. For x 0.3 a

xn p max 1
x a

2 2

xn 447.7 MPa

xt 2 fmax

xt 92.92 MPa
x
2 2

zn p max 1

zn 447.7 MPa

MACHINE DESIGN - An Integrated Approach, 4th Ed.

7-42-2

zt 0 MPa xzt fmax 1


x
2 2

xzn 0 MPa xzt 147.7 MPa

a 4.

Equations 7.24a and 7.24b can now be solved for the total applied stresses along the x, y, and z axes.

x xn xt z zn zt xz xzn xzt
5.

x 540.591 MPa z 447.672 MPa xz 147.732 MPa

Since the rollers are long, we expect a plane strain condition to exist. The stress in the third dimension is found from equation 7.23b:

y x z
6.

y 276.714 MPa

Unlike the pure-rolling case, these stresses are not principal because of the applied shear stress. The principal stresses are found from equation 4.4 using a cubic root finding solution.

x
MPa

y
MPa

z xz

z
MPa

xy 0
C2 x y z C1

yz 0

xz
MPa
3

C2 1.265 10 C1 4.936 10
5

x xy x xz y yz xy y xz z yz z

x xy xz C0 xy y yz xz yz z
f ( ) C2 C1 C0
3 2

C0 6.093 10

C0 C1 v C2 1
Principal stresses:

s polyroots ( v) MPa

649.0 s 339.3 MPa 276.7

1 s 2 s 3 s

3 2 1

1 276.7 MPa 2 339.3 MPa 3 649.0 MPa

MACHINE DESIGN - An Integrated Approach, 4th Ed.

7-42-3

Maximum shear stress

13
7.

1 3
2

13 186.1 MPa

The principal stresses are maximum at the surface as seen in Figures 7-20 and 7-22 in the text.

MACHINE DESIGN - An Integrated Approach, 4th Ed.

7-43-1

PROBLEM 7-43
Statement: Given: Repeat Problem 7-41 for contact beween a ball and outer raceway. The outer raceway radius for concave surface = 4.05 mm and the outer raceway radius for concave surface = 17.02 mm. Ball radii Raceway radii R1 4.00 mm R'1 4.00 mm R2 17.02 mm (radial) R'2 4.05 mm (axial) Radial load F 5200 N Angle between planes of R1 & R2 Solution: 1. (normal to contact plane) 0 deg The parts are steel. Therefore E 206.8 GPa

0.28

See Figure P7-7 and Mathcad file P0743.

Find the material constants from equation 7.9b. Material constants m1 1 E


2

m1 4.456 10

1 MPa

m2 m1 2. Two geometry constants are needed from equations 7.19a. Geometry constants A 1 2 1 1 R'1 1 R2 1

R1

R'2
1

A 0.0972 mm

2 2 1 1 1 1 B R R'1 2 R1 2 R'2 1 1 1 1 2 R R' R R' cos( 2 ) 1 2 2 1

B 0.0941 mm

Angle

B 180 acos A ka 50.192


0.86215

14.48

Factors from equations 7.19e

ka 5.011
2

kb 0.0045333 0.043581 0.0017292 3.7374 10 1.4207 10 3.


5 9

3.7418 10
5

kb 0.371

Determine the contact patch dimensions using the material and geometry constants in equations 7.19d.
1

Major axis half-width

3 3 m1 m2 a ka F A 4 1

a 3.557 mm

Minor axis half-width

3 3 m1 m2 b kb F A 4

b 0.263 mm

MACHINE DESIGN - An Integrated Approach, 4th Ed.


Contact area 4. A a b A 2.94254 mm
2

7-43-2

The average and maximum contact pressure can now be found from equations 7.18b and c. Average pressure p avg F A 3 2 p avg p avg 1767 MPa

Maximum pressure 5.

p max

p max 2651 MPa

The maximum normal stresses in the center of the contact patch at the surface are then found using equations 7.21a. In-plane

x 2 ( 1 2 )

a b a a b

p max p max

x 1565 MPa y 2570 MPa z 2651 MPa

y 2 ( 1 2 )
Axial

z p max

These stresses are principal:

1 x 1 3
2

2 y

3 z

The maximum shear stress associated with them at the surface is

13
6.

13 543 MPa

The maximum shear stress under the surface on the z-axis is approximately Max shear stress

13max 0.34 p max

13max 901 MPa

7.

All of the stresses found so far exist on the centerline of the patch. At the edge of the patch, at the surface, there will also be a shear stress. Two constants are found from equation 7.21b for this calculation. k3 k4 b a 1 a a b
2 2

k3 0.074 k4 0.997

8.

These constants are used in equations 7.21c and d to find the shear stresses on the surface at the ends of the major and minor axes. Major axis

xy ( 1 2 )

k3 k4
2

k4

atanh k4 1 p max

xy 200 MPa

Minor axis

xy ( 1 2 )

k3 k4
2

k3 k4

atan

k4 pmax k3

xy 77 MPa

MACHINE DESIGN - An Integrated Approach, 4th Ed.

7-44-1

PROBLEM 7-44
Statement: A machine has two crowned, cylindrical rollers rolling against each other with a dynamic load of 0 to 3.5 kN. The first roller has a major radius of 14 mm with a crown radius of 80 mm. The second roller has a major radius of 75 mm and a crown radius of 100 mm. The two axes of rotation have a 30 degree angle between them. Find the contact stresses if both rollers are steel. Roller 1 radius Crown radius Roller 2 radius Crown radius R1 14.00 mm R'1 80.0 mm (90 deg to roller rad) R2 75.00 mm (radial) R'2 100.0 mm (axial)

Given:

(normal to contact plane) Radial load F 3.5 kN Angle between planes of R1 and R2 30 deg Material properties E1 206.8 GPa 1 0.28 steel

Assumptions: The relative motion is rolling with < 1% sliding. Solution: 1. See Mathcad file P0744.

Find the material constants from equation 7.9b. Material constants m1 1 1 E1


2

m1 4.456 10

1 GPa

m2 m1 2. Two geometry constants are needed from equations 7.19a. Geometry constants A 1 2 1 1 R'1 1 R2 1

R1

R'2
1

A 0.0536 mm

2 2 1 1 1 1 B R R'1 2 R1 2 R'2 1 1 1 1 2 R R' R R' cos( 2 ) 1 2 2 1

B 0.0303 mm

Angle Factors from equations 7.19e

B 180 acos A ka 50.192


0.86215

55.56
ka 1.572
2

kb 0.0045333 0.043581 0.0017292 3.7374 10 1.4207 10


5 9

3.7418 10
5

kb 0.685

MACHINE DESIGN - An Integrated Approach, 4th Ed.


3. Determine the contact patch dimensions using the material and geometry constants in equations 7.19d.
1

7-44-2

Major axis half-width

3 m1 m2 a ka F A 4

a 1.192 mm
1

Minor axis half-width

3 3 m1 m2 b kb F A 4

b 0.519 mm

Contact area 4.

A a b

A 1.94438 mm

The average and maximum contact pressure can now be found from equations 7.18b and c. Average pressure p avg F A 3 2 p avg p avg 1800 MPa

Maximum pressure

p max

p max 2700 MPa

5.

The maximum normal stresses in the steel follower at the center of the contact patch at the surface are then found using equations 7.21a. In-plane

x 2 1 1 2 1

a b a a b

p max p max

x 1872 MPa y 2340 MPa z 2700 MPa

y 2 1 1 2 1
Axial

z p max

These stresses are principal:

1 x

2 y

3 z

The maximum shear stress associated with them at the surface is

13
6.

1 3
2

13 414 MPa

The maximum shear stress under the surface on the z-axis is approximately Max shear stress

13max 0.34 p max

13max 918 MPa

7.

All of the stresses found so far exist on the centerline of the patch. At the edge of the patch, at the surface, there will also be a shear stress. Two constants are found from equation 7.21b for this calculation. k3 k4 b a 1 a a b
2 2

k3 0.436 k4 0.9

MACHINE DESIGN - An Integrated Approach, 4th Ed.


8.

7-44-3

These constants are used in equations 7.21c and d to find the shear stresses on the surface at the ends of the major and minor axes.

Major axis

xy 1 2 1

k3 k4
2

k4

atanh k4 1 p max

xy 406 MPa

Minor axis

xy 1 2 1

k3 k4
2

k3 k4

atan

k4 p max k3

xy 292 MPa

MACHINE DESIGN - An Integrated Approach, 4th Ed.

7-45-1

PROBLEM 7-45
Statement: A cam-follower system has a motion with a combination of rolling and sliding. The cam is cylindrical with a minimum radius of curvature of 80 mm. The roller follower is also cylindrical with a radius of 14 mm. They are both 18-mm long. The maximum contact force, which is normal to the contact plane, is 3200 N. Both the cam and roller are made from hardened steel. Assuming that the coefficient of friction between the cam and roller follower is 0.33, determine the maximum tensile, compressive, and shear stresses in the cam. Roller radius Cam radius Roller length Radial load Solution: 1. R1 14 mm R2 80 mm L 18 mm F 3200 N The parts are hardened steel. Therefore E 206.8 GPa Coefficient of friction

Given:

0.28 0.33

See Mathcad file P0742.

Find the material constants from equation 7.9a. Material constants m1 1 E


2

m1 4.456 10

1 MPa

m2 m1 Geometry constant B 1 2 1

R1

R2
1
1 2

B 0.042 mm

Contact patch half-width 2.

2 m1 m2 F a B L
F 2 a L 2 F

a 0.1550 mm

The average and maximum contact pressure can now be found from equations 7.14b and c. Average pressure p avg p max p avg 573.3 MPa p max 730.0 MPa fmax 240.9 MPa

Maximum pressure Tangential unit force 3.

a L

fmax p max

With = 0.33, the principal stresses in the contact zone will be maximal on the surface (z = 0) at x = 0.3a from th centerline as shown in Figures 7-20 and 7-22. The applied stress components are found from equation 7.23a for the normal force and equation 7.23b for the tangential force. For x 0.3 a

xn p max 1
x a

2 2

xn 696.4 MPa

xt 2 fmax

xt 144.53 MPa
x
2 2

zn p max 1

zn 696.4 MPa

MACHINE DESIGN - An Integrated Approach, 4th Ed.

7-45-2

zt 0 MPa xzt fmax 1


x
2 2

xzn 0 MPa xzt 229.8 MPa

a 4.

Equations 7.24a and 7.24b can now be solved for the total applied stresses along the x, y, and z axes.

x xn xt z zn zt xz xzn xzt
5.

x 840.886 MPa z 696.351 MPa xz 229.796 MPa

Since the contact patch is short, we expect a plane stress condition to exist. The stress in the third dimension is:

y 0 MPa
6.

y 0.000 MPa

Unlike the pure-rolling case, these stresses are not principal because of the applied shear stress. The principal stresses are found from equation 4.4 using a cubic root finding solution.

x
MPa

y
MPa

z xz

z
MPa

xy 0
C2 x y z C1

yz 0

xz
MPa
3

C2 1.537 10 C1 5.327 10
5

x xy x xz y yz xy y xz z yz z

x xy xz C0 xy y yz xz yz z
f ( ) C2 C1 C0
3 2

C0 0

C0 C1 v C2 1
Principal stresses:

s polyroots ( v) MPa

1009.5 s 527.7 MPa 0.0

1 s 2 s 3 s

3 2 1

1 0 MPa 2 527.7 MPa 3 1009.5 MPa

MACHINE DESIGN - An Integrated Approach, 4th Ed.


Maximum shear stress

7-45-3

13
7.

1 3
2

13 504.8 MPa

The principal stresses are maximum at the surface as seen in Figures 7-20 and 7-22 in the text.

MACHINE DESIGN - An Integrated Approach, 4th Ed.

7-46-1

PROBLEM 7-46
Statement: Estimate how long it will take to remove 2 m of material from the 5000 mm2 surface of a block of HB110 steel if a coarse polishing machine applies 80 N over a 400-mm stroke at 120 strokes per minute. (a) If done dry. (b) If done lubricated. kilo kg g Block surface Depth of wear Force on file Aa 5000 mm Stroke rate d 2 m F 80 N Stroke Steel hardness
2

Units: Given:

n 120 min

s 400 mm kilo HB 110 2 mm

Assumptions: Only one face will be polished. Solution: 1. See Mathcad file P0746.

This is a two-body abrasion problem. From Table 7-2, the wear coefficients for dry and lubricated abrasion in a coarse polishing operation are Wear coefficients Ka 1 10 Kb 2 10
4 4

dry lubricated

2. 3. 4.

The length of sliding is L = s n t where t is the time required. The depth of material removed is d = K F L HB Aa

Combining these two equations and solving for the time, t d HB Aa Ka F s n

(a) dry

ta

ta 28 min strokesa 3371

strokesa ta n d HB Aa Kb F s n

(b) lubricated

tb

tb 14 min strokesb 1686

strokesb tb n

MACHINE DESIGN - An Integrated Approach, 4th Ed.

7-47-1

PROBLEM 7-47
Statement:

_____

Loose abrasive grains are introduced in error into the lubricating system of a flat bronze thrust bearing that has a hardness of 60HB and a surface area of 500 mm2. If a hardened steel part exerts a force of 50 N on the bearing while oscillating across it at 200 strokes/min with a stroke of 30 mm, what depth of wear will occur in an 8-hour shift? stroke 1 Hardness Shift time Wear coefficient Stroke rate Contact force H 60 kgf mm t 8 hr K 2 10
3 2

Units: Given:

Area of wear Stroke


1

Aa 500 mm s 30 mm

n 200 stroke min F 50 N

Solution: 1.

See Mathcad file P0747.

From equation 7.7b, the depth of wear is d = K F l H Aa

2.

The length of wear is, l = s n t where t is the elapsed time

3.

Substituting for l , F s n t H Aa

d K

d 1.0 mm

MACHINE DESIGN - An Integrated Approach, 4th Ed.

7-48-1

PROBLEM 7-48
Statement: Two rollers are in contact with a 9% sliding combined with rolling and the resulting maximum compressive principal stress in the contact zone is 15500 psi. Both rollers are made from 6061-T6 hard anodized aluminum. The design life of the rollers is 4 years of 2-shift operation at 260 days/year and they each turn at 200 rpm. What is the expected safety factor against pitting for the roller pair? Roller material properties E 10.4 10 psi Roller speed Max stress Design life Solution: 1. n 200 rpm life d 4 yr shift 2 day
1 6

Given:

0.34
hrspershift 8 hr daysperyr 260 day yr
1

max 15500 psi

See Mathcad file P0748.

Calculate the required cycle life. cycles n shift hrspershift daysperyr life d cycles 1.255 10
9

2.

Calculate the material constants. m1 1 E


2

m1 8.504 10

8 1

psi

m2 m1

3.

The maximum normal stress is max 15.5 ksi, compressive. Its K-factor can be calculated from equation 7.25d. K-factor K m1 m2 max
2

K 128.4 psi

4.

From Table 7-7, Part 2, Line 36 the slope and intercept factors of this aluminum for rolling with 9% sliding are

5.02
5.

20.12

These are used in equation 7.26 along with the value of K from above to find the number of cycles that can be expected at this load before pitting begins. log ( K) =

log Nlife

log psi Nlife 10

Nlife 3.4 10
9

cycles

6.

Calculate the factor of safety against pitting. Nf Nlife cycles Nf 2.7

MACHINE DESIGN - An Integrated Approach, 4th Ed.

7-49-1

PROBLEM 7-49
Statement:

_____

Two contacting rollers run together in pure rolling. Both are made from Class 20 Gray iron, HB 130-180. One roller has a diameter of 2.75 in and the other has a diameter of 3.25 in. Both are 10 in long. The applied load is 5500 lbf. If the design life is 1E08 cycles, determine the factor of safety against pitting failure. Material data from Table 7-7 Roller radii R1 2.75 in Roller length Applied force L 10.0 in F 5500lbf K 960 psi R2 3.25 in

Given:

Solution: 1.

See Mathcad file P0749.

Find the geometry constant from equation 7.9a. Geometry constant B 1 2 1

R1

R2
1

B 0.336

1 in

2.

Calculate the allowable force on the rollers using equation 7.25d. K L 2 B

Allowable force

Fallow

Fallow 14300 lbf

3.

Calculate the factor of safety using equation f in Example 7-5. Nf Fallow F Nf 2.6

Safety factor

MACHINE DESIGN - An Integrated Approach, 4th Ed.

10-1a-1

PROBLEM 10-1a
Statement: A simply supported shaft is shown in Figure P10-1. A constant magnitude transverse load P is applied as the shaft rotates subject to a time-varying torque that varies from Tmin to Tmax. For the data in row a of Table P10-1, find the diameter of shaft required to obtain a safety factor of 2 in fatigue loading if the shaft is steel of S ut = 108 ksi and S y = 62 ksi. The dimensions are in inches, the force in pounds, and the torque is in lb-in. Assume no stress concentrations are present. Distance between bearings a 16 in Applied load P 1000 lbf Minimum torque Maximum torque Tmin 0 lbf in Tmax 2000 lbf in Distance to P Tensile strength Yield strength Design safety factor b 18 in S ut 108 ksi S y 62 ksi Nd 2

Given:

Assumptions: The finish is machined, reliability is 99%, and the shaft is at room temperature. Solution: 1. See Figure P10-1 and Mathcad file P1001a.

The maximum moment in the shaft occurs at the right bearing as seen in the moment diagram in Figure B-3(a) in Appendix B (note that in the figure a is the distance to the load and b is the distance between bearings). Using the equation given in the figure, calculate the alternate bending moment (the mean is zero). Ma P ( a b ) Ma 2000 in lbf

2.

Calculate the mean and alternating components of torque. Tm Ta Tmax Tmin 2 Tmax Tmin 2 Tm 1000 in lbf Ta 1000 in lbf S'e 0.5 S ut S'e 54 ksi

3. 4.

Calculate the unmodified endurance limit.

Determine the endurance limit modification factors for a rotating round shaft. Load Size Cload 1 d Csize( d ) 0.869 in A 2.70
0.097

Surface

b 0.265
b

(machined)

Csurf Temperature Reliability 5.

S ut A ksi

Csurf 0.781

Ctemp 1 Creliab 0.814 (R = 99%)

Determine the modified endurance limit as a function of the unknown diameter, d. S e( d ) Cload Csize( d ) Csurf Ctemp Creliab S'e

6.

Use equation (10.8) with unity for all stress concentration factors as a design equation to find d.

MACHINE DESIGN - An Integrated Approach, 4th Ed.

10-1a-2

Guess Given

d 1 in
1

3 2 2 32 Nd Ma 4 Ta 3 Tm d= 4 S ut S e( d ) d Find ( d )

d 1.188 in

Using this value for d, the size modification factor and endurance limit are: Size modification factor Endurance limit Csize( d ) 0.855 S e( d ) 29.3 ksi

MACHINE DESIGN - An Integrated Approach, 4th Ed.

10-2a-1

PROBLEM 10-2a
Statement: A simply supported shaft is shown in Figure P10-2. A constant magnitude distributed load p is applied as the shaft rotates subject to a time-varying torque that varies from Tmin to Tmax. For the data in row a of Table P10-1, find the diameter of shaft required to obtain a safety factor of 2 in fatigue loading if the shaft is steel of S ut = 745 MPa and Sy = 427 MPa. The dimensions are in cm, the distributed force in N/cm, and the torque is in N-m. Assume no stress concentrations are present. Dist. between bearings Distance to end of p Applied distributed load Minimum torque Maximum torque L 20 cm b 18 cm p 1000 N cm Tmin 0 N m
1

Given:

Distance to p a 16 cm Tensile strength S ut 745 MPa Yield strength S y 427 MPa Design safety factor Nfd 2

Tmax 2000 N m

Assumptions: The finish is machined, reliability is 99%, and the shaft is at room temperature. Solution: 1. See Figure P10-2 and Mathcad file P01002a.

The maximum moment in the shaft occurs between a and b. See the appendix to this problem below for the determination of Ma. Ma 48.45 N m

2.

Calculate the mean and alternating components of torque. Tm Ta Tmax Tmin 2 Tmax Tmin 2 Tm 1000 N m Ta 1000 N m S'e 0.5 S ut S'e 372.5 MPa

3. 4.

Calculate the unmodified endurance limit.

Determine the endurance limit modification factors for a rotating round shaft. Load Size Cload 1 d Csize( d ) 1.189 mm A 4.51
0.097

Surface

b' 0.265
b'

(machined)

Csurf Temperature Reliability 5.

Sut A MPa

Csurf 0.782

Ctemp 1 Creliab 0.814 (R = 99%)

Determine the modified endurance limit as a function of the unknown diameter, d. S e( d ) Cload Csize( d ) Csurf Ctemp Creliab S'e

6.

Use equation (10.8) with unity for all stress concentration factors as a design equation to find d.

MACHINE DESIGN - An Integrated Approach, 4th Ed.


Guess Given
1

10-2a-2

d 1 in

3 2 2 32 Nfd Ma 4 Ta 3 Tm d= 4 S ut S e( d ) d Find ( d )

d 48.6 mm

Using this value for d, the size modification factor and endurance limit are: Size modification factor Endurance limit APPENDIX - Maximum bending moment 1. From inspection of Figure P10-2, write the load function equation q(x) = R1<x - 0>-1 - p<x - a>0 + p<x - b>0 + R2<x - L>-1 2. Integrate this equation from - to x to obtain shear, V(x) V(x) = R1<x - 0>0 - p<x - a>1 + p<x - b>1 + R2<x - L>0 3. Integrate this equation from - to x to obtain moment, M(x) M(x) = R1<x - 0>1 - p<x - a>2/2 + p<x - b>2/2 + R2<x - L>1 4. Solve for the reactions by evaluating the shear and moment equations at a point just to the right of x = L, where both are zero. At x = L+, V = M = 0 V = R1 p ( L a ) p ( L b ) R2 = 0 M = R 1 L R1 p 2 L p 2 ( L a)
2 2

Csize( d ) 0.816 S e( d ) 193.4 MPa

p 2

( L b) = 0
2

( L a ) ( L b )

R1 300 N R2 1700 N

R2 p ( b a ) R1 5. 6. Define the range for x x 0 m 0.005 L L

For a Mathcad solution, define a step function S. This function will have a value of zero when x is less than z, and a value of one when it is greater than or equal to z. S ( x z) if ( x z 1 0 )

7.

Write the shear and moment equations in Mathcad form, using the function S as a multiplying factor to get the effect of the singularity functions.

MACHINE DESIGN - An Integrated Approach, 4th Ed.


V ( x) R1 S ( x 0 m) p S ( x a ) ( x a ) p S ( x b ) ( x b ) R2 S ( x L) M ( x) R1 S ( x 0 m) x 8. Plot the shear and moment diagrams. Shear Diagram
500 0 V ( x) 500 N 1000 1500 2000

10-2a-3

p 2

S ( x a ) ( x a )

p 2

S ( x b ) ( x b )

10 x cm

15

20

Moment Diagram
50

35 M ( x) Nm 20

10

10 x cm

15

20

FIGURE 10-2a
Shear and Moment Diagrams for Problem 10-2a

9.

Determine the maximum maximum moment from inspection of the diagrams. Maximum moment occurs where zero, which is x = c. From the shear diagram, ca R1 = bc R2 c a R2 b R1 R1 R2 c 16.300 cm Mmax 48.45 N m

Mmax M ( c)

MACHINE DESIGN - An Integrated Approach, 4th Ed.

10-3-1

PROBLEM 10-3
Statement: For the bicycle pedal-arm assembly in Figure P6-1 assume a rider-applied force that ranges from 0 to 1500 N at the pedal each cycle. Design a suitable shaft to connect the two pedal arms. Use a fatigue safety factor of 2 and a material with S ut = 500 MPa. The shaft has a square extension on each end where it inserts into the pedal arms. Material tensile strength Applied load by rider S ut 500 MPa Fridermax 1500 N Design safety factor Fridermin 0 N Nfd 2

Given:

Assumptions: 1. The configuration is similar to that shown in Figure 10-3 with the dimensions a, b, and c below. 2. The moments in the shaft are greater when the right pedal is loaded. 3. The torque is transmitted from the sprocket to the shaft through the square shaft extension. 4. The shaft is machined, reliability is 50%, and the bicycle is not operated in extreme temperatures. Shaft dimensions: a 10 mm b 22 mm c 80 mm See Figure 10-3 and Mathcad file P1003. Solution: 1. From problems 3-3 and 7-3, the moment, torque, and chain force Fchain are: Mmax 90 N m Mmin 90 N m Tmax 255 N m Tmin 0 N m d sprocket 100 mm Fchainmax 2 Tmax d sprocket

F chain R 1z

Sprocket

Frame Bearing (2 Places) Pedal Arm (Both Ends)

T T

z y c a F rider

R 2z Horizontal Section (Top View)

R 2y

Fchainmax 5100 N

2.

Note that in Problem 3-3 the pedal-arm torque becomes a concentrated bending moment on this shaft and the pedal-arm bending moment becomes the sprocket torque, which is carried along the shaft from the y-axis to the centerplane of the sprocket, the distance a (see Figure 10-3A).

R 1y b Vertical Section (Front View)


FIGURE 10-3A
Free Body Diagram for Problem 10-3

3.

Find the moment distribution along the shaft from x = 0 to x = c. This will be done by finding the moment paral to the z-axis by looking at the forces and moments in the x-y plane, then finding the moment parrallel to the y-ax by looking at the forces in the x-z plane. These two moment distributions will be added vectorially to get the total moment distribution. Determine the bearing reactions R1y and R2y in the x-y plane.

4.

Fy:

Frider R1y R2y = 0

MACHINE DESIGN - An Integrated Approach, 4th Ed.

10-3-2

Mz:

M b Frider ( c b ) R2y = 0 R2ymax Mmax b Fridermax cb R2ymax 2121 N R1ymax 3621 N

R1ymax Fridermax R2ymax 5.

Use singularity functions to determine the moment distribution of the forces in the x-y plane. q xy = -M<x>-2 - Frider<x>-1 + R1y<x - b>-1 - R2y<x - c>-1 Vxy = -M<x>-1 - Frider<x>0 + R1y<x - b>0 - R2y<x - c>0 Mxy = -M<x>0 - Frider<x>1 + R1y<x - b>1 - R2y<x - c>1

6. 7.

Define the range for x

x 0 mm 0.005 c c

For a Mathcad solution, define a step function S. This function will have a value of zero when x is less than z, and a value of one when it is greater than or equal to z. S ( x u ) if ( x u 1 0 )

8.

Write the moment equation in Mathcad form, using the function S as a multiplying factor to get the effect of the singularity functions. Mxy( x) Mmax Fridermax S ( x 0 mm) x R1ymax S ( x b ) ( x b ) R2ymax S ( x c) ( x c)

9.

Plot the moment diagram (see Figure 10-3B).


0

10. Determine the bearing reactions R1z and R2z in the x-z plane.

Fz : Mz:
R2zmax

Fchain R1z R2z = 0 b Fchain ( c b ) R2z = 0 b Fchainmax cb

50 Mxy ( x) Nm 100

R2zmax 1934 N R1zmax Fchainmax R2zmax R1zmax 7034 N

150

20

40 x mm

60

80

FIGURE 10-3B
Moment Diagram for x-y Plane for Problem 10-3

11. Use singularity functions to determine the moment distribution of the forces in the x-z plane. q xz = - Fchain<x>-1 + R1z<x - b>-1 - R2z<x - c>-1 Vxz = - Fchain<x>0 + R1z<x - b>0 - R2z<x - c>0 Mxz = - Fchain<x>1 + R1z<x - b>1 - R2z<x - c>1

MACHINE DESIGN - An Integrated Approach, 4th Ed.

10-3-3

12. Write the moment equation in Mathcad form, using the function S as a multiplying factor to get the effect of the singularity functions. Mxz( x) Fchainmax S ( x 0 mm) x R1zmax S ( x b ) ( x b ) R2zmax S ( x c) ( x c) 13. Plot the moment diagram (see Figure 10-3C). 14. Determine the total moment distribution. Mtot ( x)
0

Mxy( x) Mxz( x)
200

This is plotted in Figure 10-3D.

150 50 Mxz( x) Nm 100 50 Mtot ( x) Nm 100

150

20

40 x mm

60

80

20

40 x mm

60

80

FIGURE 10-3C

FIGURE 10-3D

Moment Diagram for x-z Plane for Problem 10-3

Total Moment Diagram for Problem 10-3

15. From Figure 10-3D, we see that the moment is a maximum at x = b. However, the torque is carried only from x = to x = a so we will investigate the section just to the right of x = a where there is a stress concentration due to the shoulder. Moment at x = a Moment at x = b Mamax Mtot ( a ) Mbmax Mtot ( b ) Mamax 116.73 N m Mbmax 166.487 N m

16. The bending moment is fully reversed while the torque is repeated from zero to Tmax. Therefore, we have the following alternating and mean components of moment and torque at x = a. Alternating moment Mean moment Alternating torque Mean torque Ma Mamax Mm 0 N m Ta 0.5 Tmax Tm 0.5 Tmax Ma 116.73 N m Mm 0 N m Ta 127.5 N m Tm 127.5 N m

16. There will be a stress concentration due to the step just to the right of the sprocket and also because of the transition from square to round at the shoulder. Estimate this to be Kf 2. Also, let Kfm Kf Kfs Kf Kfsm Kfs S'e 0.5 S ut S'e 250 MPa

17. Calculate the unmodified endurance limit.

MACHINE DESIGN - An Integrated Approach, 4th Ed.


18. Calculate the endurance limit modification factors for a rotating round beam. Load Size Cload 1 Csize( d ) 1.189 A 4.51 Csurf A Temperature Reliability Ctemp 1 Creliab 1.0 (R = 50%)

10-3-4

mm
d

0.097

Surface

b 0.265

(machined)

Sut MPa

Csurf 0.869

19. Calculate the modified endurance limit. S e( d ) Cload Csize( d ) Csurf Ctemp Creliab S'e 20. Use equation (10.8) to solve for the diameter at x = a. Guess d 12 mm Given
1 3

3 2 2 32 Nfd Kf Ma 4 Kfs Ta d= S e( d ) d Find ( d ) d 35.526 mm

Kfm Mm

S ut

3 4

Kfsm Tm

This is too big for a practical design. A stronger material should be chosen to bring this down to about 16 mm.

MACHINE DESIGN - An Integrated Approach, 4th Ed.

10-4a-1

PROBLEM 10-4a
Statement: Given: Determine the maximum deflections in torsion and in bending of the shaft shown in Figure P10-1 for the data in row a in Table P10-1 if the steel shaft diameter is 1.75 in. Shaft length Distance to P Applied load L 20 in b 18 in P 1000 lbf Distance between bearings a 16 in Shaft diameter d 1.750 in Young's modulus Modulus of rigidity E 30 10 psi G 11.5 10 psi
6 6

Maximum torque Tmax 2000 lbf in Solution: 1. See Figure P10-1 and Mathcad file P1004a.

Calculate the area moment of inertia and the polar moment of inertia. Area moment of inertia I J

d
64

I 0.460 in
4

Polar moment of inertia

d
32

J 0.921 in

2.

Use the equation in Figure B-3(a) in Appendix B for this overhung beam with concentrated load to determine the maximum deflection in bending. Note that in the figure a is the distance to the load and b is the distance between bearings. This deflection occurs at the right end of the shaft, under the load P. ymax P 6 E I ab L
3

b a

( L a ) ( L b ) a ( b a ) L
3 3

ymax 0.00357 in

3.

Calculate the torsional deflection using equation (10.9a).

max

Tmax L G J

max 0.216 deg

MACHINE DESIGN - An Integrated Approach, 4th Ed.

10-5a-1

PROBLEM 10-5a
Statement: Determine the maximum deflections in torsion and in bending of the shaft shown in Figure P10-2 for the data in row a in Table P10-1 if the steel shaft diameter is 40 mm. Distance between bearings Distance to end of p Applied distributed load Minimum torque Maximum torque Solution: 1. L 20 cm b 18 cm p 1000 N cm Tmin 0 N m
1

Given:

Distance to p Shaft diameter

a 16 cm d 40 mm

Young's modulus E 206.8 GPa Mod of rigidity G 80.8 GPa

Tmax 2000 N m

See Figure P10-2 and Mathcad file P1005a.

Calculate the area moment of inertia and the polar moment of inertia. Area moment of inertia I J

d
64

I 125664 mm
4

Polar moment of inertia 2.

d
32

J 251327 mm

Calculate the torsional deflection using equation (10.9a).

max
3.

Tmax L G J

max 1.129 deg

From inspection of Figure P10-2, write the load function equation and integrate it twice to get the shear and moment equations. q(x) = R1<x - 0>-1 - p<x - a>0 + p<x - b>0 + R2<x - L>-1 V(x) = R1<x - 0>0 - p<x - a>1 + p<x - b>1 + R2<x - L>0 M(x) = R1<x - 0>1 - p<x - a>2/2 + p<x - b>2/2 + R2<x - L>1

4.

Solve for the reactions by evaluating the shear and moment equations at a point just to the right of x = L, where both are zero. At x = L+, V = M = 0 V = R1 p ( L a ) p ( L b ) R2 = 0 M = R 1 L R1 p 2 L p 2 ( L a)
2 2

p 2

( L b) = 0
2

( L a ) ( L b )

R1 300 N R2 1700 N

R2 p ( b a ) R1 5. Integrate twice more to get the slope and deflection equations.

(x) = [R1<x - 0>2/2 - p<x - a>3/6 + p<x - b>3/6 + R2<x - L>2/2 + C3]/EI
y(x) = [R1<x - 0>3/6 - p<x - a>4/24 + p<x - b>4/24 + R2<x - L>3/6 + C3x + C4]/EI 6. Solve for the constants of integration with the boundary conditions y = 0 at x = 0 and x = L. At x = 0 C4 = 0

MACHINE DESIGN - An Integrated Approach, 4th Ed.


At x = L 0= R1 6 L
3

10-5a-2
4

p 24

( L a)

p 24

( L b ) C 3 L

C3

R1 3 p p 4 4 ( L a) ( L b) L L 24 24 6 1

C3 1.950 N m

7. 8.

Define the range for x

x 0 m 0.005 L L

For a Mathcad solution, define a step function S. This function will have a value of zero when x is less than z, and a value of one when it is greater than or equal to z. S ( x z) if ( x z 1 0 )

9.

Write the slope and deflection equations in Mathcad form, using the function S as a multiplying factor to get the effect of the singularity functions.

( x)

R1 p 2 p 3 3 S ( x 0 mm) x S ( x a ) ( x a ) S ( x b ) ( x b ) C3 E I 2 6 6
1

y ( x)

R1 p p 3 4 4 S ( x 0 mm) x S ( x a ) ( x a ) S ( x b ) ( x b ) C3 x E I 6 24 24
1

10. Plot the slope and deflection diagrams (see Figure 10-5). 11. The maximum deflection occurs at the value of x for which is zero. Let this be x = c, then guess c 10 cm Given

( c) = 0

c Find ( c) ymax y ( c)

c 11.402 cm ymax 0.00570 mm DEFLECTION


0

12. The maximum deflection occurs at x = c and is SLOPE


0.15

0.1 2 10
3

( x)
10
3

0.05

y ( x) mm
3

0 0.05 0.1

4 10

10 x cm

15

20

6 10

10 x cm

15

20

FIGURE 10-5

Slope and Deflection Diagrams for Problem 10-5a

MACHINE DESIGN - An Integrated Approach, 4th Ed.

10-6a-1

PROBLEM 10-6a
Statement: Determine the size of key necessary to give a safety factor of at least 2 against both shear and bearing failure for the design shown in Figure P10-3 using the data from row a in Table P10-1 and the data given below. Assume a shaft diameter of 1.75 in. Shaft diameter d 1.750 in Shaft properties: S ut1 108 ksi S y1 62 ksi Key properties: S ut2 88 ksi S y2 52 ksi Maximum torque Tmax 2000 lbf in Minimum torque Tmin 0 lbf in Design safety factor Nd 2

Given:

Assumptions: 1. The finish is machined, reliability is 90%, and the shaft is at room temperature. 2. Note that the given magnitudes of the radial forces shown acting on gear(s) in this problem are not necessarily consistent with a load associated with the given torque for any real gear of reasonable pressure angle. Since gears are taken up in a later chapter, these shaft design problems ignore the real gear loadings and use an arbitrary value to provide a shaft design exercise. Solution: 1. See Figure P10-3 and Mathcad file P1006a.

As recommended in Table 10-2, for a shaft diameter of d 1.75 in, use a square key of width Key width w 0.375 in

2.

Determine the alternating and mean key shear force components. Mean and alternating force components Fa Fm 1 Tmax Tmin 2 0.5 d 1 Tmax Tmin 2 0.5 d Fa 1143 lbf Fm 1143 lbf

3.

Write the equations for the mean and alternating components of the shear stress and the effective von Mises stress. Key shear area as Ashear = w L function of length Mean and alternating shear stresses Fa w L Fa w L Fm w L Fm w L

a =

m =

Mean and alternating von Mises stresses 4.

'a = 3 a = 3

'm = 3 m = 3

Using the modified Goodman failure criterion, the design equation is S e S ut S ut 'a S e 'm S e S ut

Nd =

3 S F S F ut a e m w L
3 Nd S ut Fa S e Fm w S ut S e

Solving for L 5.

L=

Determine the key material endurance limit.

MACHINE DESIGN - An Integrated Approach, 4th Ed.

10-6a-2

Uncorrected endurance strength Correction factors: Load Size

S'e 0.5 S ut2

S'e 44 ksi

Cload 1 A95 = w L Csize = 0.869 d eq Cs 0.869


0.097 0.5 0.097

d eq =

w L 0.0766

0.0766 in
w
0.0485

Cs 0.805

Csize = 0.805 L Surface (machined) Temperature Reliability Csurf

Sut2 2.7 ksi

0.265

Csurf 0.824

Ctemp 1 Creliab 0.897


0.0485

(R = 90%)
0.0485

S e = Cload Csize Csurf Ctemp Creliab S'e = 1 Cs L where 6. Substitute S e into the design equation. 3 Nd S ut Fa Ce L w S ut Ce L Solving by iteration, let 3 Nd S ut2 Fa Ce RHS( L)
0.0485

Csurf 1 Crelaib S'e = Ce L

Ce Cs Csurf Creliab S'e

Ce 26.175 ksi

L=

Fm

0.0485

in
0.0485

0.0485

Fm

w S ut2 Ce Guess L 0.5 in L RHS( L) Tentatively, let 7.

in
RHS( L) 0.510 in RHS( L) 0.510 in

L 0.625 in

(5/8 in)

Check the realised factor of safety against fatigue failure. Endurance limit S e Ce

in

0.0485

S e 26.778 ksi

MACHINE DESIGN - An Integrated Approach, 4th Ed.

10-6a-3

Realised factor of safety

Nf

S e S ut2

3 S F S F ut2 a e m w L
1 2

Nf 2.4

8.

Check worst-case static bearing stress. Bearing area Maximum force Abear w L Abear 0.117 in Fmax 2286 lbf
2

Fmax Fa Fm Fmax Abear

Maximum bearing stress on key Factor of safety against static bearing yield

max

max 19.50 ksi

Ns

S y2

max

Ns 2.7

9.

Design Summary Key width Key length w 0.375 in L 0.625 in (3/8 x 3/8) (5/8 in)

MACHINE DESIGN - An Integrated Approach, 4th Ed.

10-7a-1

PROBLEM 10-7a
Statement: Determine the size of key necessary to give a safety factor of at least 2 against both shear and bearing failure for the design shown in Figure P10-4 using the data from row a in Table P10-1 and the data given below. Assume a shaft diameter of 40 mm. Shaft diameter Maximum torque Minimum torque Design safety factor d 40 mm Tmax 2000 N m Tmin 0 N m Nd 2 Key properties: Shaft properties: S ut1 745 MPa S y1 427 MPa S ut2 600 MPa S y2 360 MPa

Given:

Assumptions: 1. The finish is machined, reliability is 90%, and the shaft is at room temperature. 2. Note that the given magnitudes of the radial forces shown acting on gear(s) in this problem are not necessarily consistent with a load associated with the given torque for any real gear of reasonable pressure angle. Since gears are taken up in a later chapter, these shaft design problems ignore the real gear loadings and use an arbitrary value to provide a shaft design exercise. Solution: 1. See Figure P10-4 and Mathcad file P1007a.

As recommended in Table 10-2, for a shaft diameter of d 40 mm, use a square key of width Key width w 10 mm

2.

Determine the alternating and mean key shear force components. Mean and alternating force components Fa Fm 1 Tmax Tmin 2 0.5 d 1 Tmax Tmin 2 0.5 d Fa 50.0 kN Fm 50.0 kN

3.

Write the equations for the mean and alternating components of the shear stress and the effective von Mises stress. Key shear area as Ashear = w L function of length Mean and alternating shear stresses Fa w L Fa w L Fm w L Fm w L

a =

m =

Mean and alternating von Mises stresses 4.

'a = 3 a = 3

'm = 3 m = 3

Using the modified Goodman failure criterion, the design equation is S e S ut S ut 'a S e 'm S e S ut

Nd =

3 S F S F ut a e m w L
L= 3 Nd S ut Fa S e Fm w S ut S e

Solving for L 5.

Determine the key material endurance limit. Uncorrected endurance strength S'e 0.5 S ut2 S'e 300 MPa

MACHINE DESIGN - An Integrated Approach, 4th Ed.

10-7a-2

Correction factors: Load Size Cload 1 A95 = w L Csize = 1.189 d eq Cs 1.189 Csize = Cs L Surface (machined) Temperature Reliability Csurf
0.097 0.5 0.097

d eq =

w L 0.0766

0.0766 mm
w

Cs 0.939

0.0485 0.265

S ut2 4.51 MPa

Csurf 0.828

Ctemp 1 Creliab 0.897


0.0485

(R = 90%)
0.0485

S e = Cload Csize Csurf Ctemp Creliab S'e = 1 Cs L where 6. Substitute S e into the design equation. 3 Nd S ut Fa Ce L w S ut Ce L Solving by iteration, let 3 Nd S ut2 Fa Ce RHS( L)
0.0485

Csurf 1 Creliab S'e = Ce L

Ce Cs Csurf Creliab S'e

Ce 209.145 MPa

L=

Fm

0.0485

mm
L

0.0485

Fm

w S ut2 Ce Guess L 10 mm L RHS( L) L RHS( L) Tentatively, let 7.

mm
RHS( L) 121.468 mm RHS( L) 133.39 mm RHS( L) 133.866 mm

0.0485

L 134 mm

(both keys, divide by 2 for one)

Check the realised factor of safety against fatigue failure. L S e Ce mm


0.0485

Endurance limit

S e 164.923 MPa

MACHINE DESIGN - An Integrated Approach, 4th Ed.

10-7a-3

Realised factor of safety

Nf

S e S ut2

3 S F S F ut2 a e m w L
1 2

Nf 2.0

8.

Check worst-case static bearing stress. Bearing area Maximum force Abear w L Abear 670 mm Fmax 100 kN
2

Fmax Fa Fm Fmax Abear

Maximum bearing stress on key Factor of safety against static bearing yield

max

max 149.25 MPa

Ns

S y2

max

Ns 2.4

9.

Design Summary Key width Key length w 10 mm L1 0.5 L L1 67 mm

MACHINE DESIGN - An Integrated Approach, 4th Ed.

10-8-1

PROBLEM 10-8
Statement: A paper mill processes rolls of paper having a density of 984 kg/m3. The paper roll is 1.50-m outside diameter (OD) x 0.22-m inside diameter (ID) x 3.23-m long and is on a simple supported, hollow, steel shaft with S ut = 400 MPa. Find the shaft ID needed to obtain a dynamic safety factor of 2 for a 10-yr life if the shaft OD is 22 cm and the roll turns at 50 rpm with 1.2 hp absorbed. Paper roll: Density Outside dia. Inside dia. Length Shaft: Strength Outside dia. Factor of safety Power Speed kg
3
y w x R V R L/2 0 L x -R M L R

Given:

984

m OD 1500 mm ID 220 mm L 3230 mm S ut 400 MPa od 220 mm Nf 2 HP 1.2 hp 50 rpm

Assumptions: 1. The shaft is stiffer than the paper roll so the weight of the roll on the shaft can be modelled as a uniformly distributed load. 2. The bearings that support the shaft are close to the ends of the paper roll and are thin with respect to the length of the roll so we can consider the distance between the shaft supports to be the same as the length of the roll. 3. The shaft is machined, reliability is 99.9%, and it is at room temperature. 4. The paper machine operates 3 shifts/day. Solution: 1. See Figure 10-8 and Mathcad file P1008.

wL /8

0 L/2 L

FIGURE 10-8
Load, Shear, and Moment Diagrams for Problem 10-8

The weight of the paper roll is, Volume Weight V

OD ID L

V 5.585 m

W g V

W 53.895 kN

2.

From Figure 10-8, we see that the bending moment in the shaft is a maximum at the center of the span. First, determine the magnitude of the distributed load, then find the maximum bending moment using Figure B-2(b) in Appendix B with a = 0 and x = L/2. Distributed load Maximum moment w W L w L 8
2

w 16.686

newton mm
7

Mmax

Mmax 2.176 10 newton mm

3. Using equation 4.11b, find the maximum bending stress as a function of the unkown shaft inside diameter, id. This is the only alternating stress element present at this point on the shaft and there is no alternating shear stress at this point so max = 1 and 2 = 3 = 0. Furthermore, since 2 and 3 are zero, max = 'a. Bending stress at midspan

max =

M c I

4 4 od id

32 Mmax od

= 'a

MACHINE DESIGN - An Integrated Approach, 4th Ed.

10-8-2

4.

Calculate the steady shaft torque using equation (10.1a) and the mean von Mises stress using equation (5.7d). Mean torque Tm HP

16 Tm od

Tm 171 N m

Mean torsional stress Mean von Mises stress 5.

m =

od id 'm = 3 m

2080 hr yr

Determine the number of stress cycles. Let Nshifts 3 Stress cycles

shift_hours

Lcycle 10 yr

Ncycles shift_hours Nshifts Lcycle Ncycles 1.176 10


9

Since this is greater than 10 6, design for infinite life. 6. 7. Calculate the unmodified endurance limit. S'e 0.5 S ut S'e 200 MPa

Determine the endurance limit modification factors for a rotating round shaft. Load Size Cload 1 Csize 1.189 A 4.51

mm
od

0.097

Csize 0.705

Surface

b 0.265
b

(machined)

Csurf Temperature Reliability 8.

Sut A MPa

Csurf 0.922

Ctemp 1 Creliab 0.753 (R = 99.9%)

Determine the modified endurance limit as a function of the unknown diameter, d. S e Cload Csize Csurf Ctemp Creliab S'e S e 97.819 MPa

9.

Use equation (10.7b) with unity for all stress concentration factors as a design equation to find id. Nf = S e S ut

'a S ut 'm S e

Substituting the expressions for 'a and 'm found above,

MACHINE DESIGN - An Integrated Approach, 4th Ed.

10-8-3

Nf =

S e S ut 32 Mmax od

od id

S ut

16 3 Tm od

od id

Se

Solving for the unknown inside diameter,


1

4 16 Nf od 2 Mmax Sut 3 Tm S e id od S e S ut
Round this down (for a thicker wall) to The wall thickness will be id 190 mm t 0.5 ( od id)

id 191.467 mm

t 15 mm

MACHINE DESIGN - An Integrated Approach, 4th Ed.

10-9a-1

PROBLEM 10-9a
Statement: Given: Repeat Problem 10-1 taking the stress concentration at the keyway shown in Figure P10-3 into account. Distance between bearings Applied load Minimum torque a 16 in P 1000 lbf Tmin 0 lbf in Distance to P b 18 in Tensile strength S ut 108 ksi Yield strength S y 62 ksi

Design safety factor Nd 2 Maximum torque Tmax 2000 lbf in Assumptions: 1. The finish is machined, reliability is 99%, and the shaft is at room temperature. 2. The notch radius in the keyway is r 0.015 in. 3. Note that the given magnitudes of the radial forces shown acting on gear(s) in this problem are not necessarily consistent with a load associated with the given torque for any real gear of reasonable pressure angle. Since gears are taken up in a later chapter, these shaft design problems ignore the real gear loadings and use an arbitrary value to provide a shaft design exercise. Solution: 1. See Figures P10-1 and P10-3, and Mathcad file P1009a.

There is no stress concentration at the point on the shaft where the bending moment is a maximum. On the other hand, at the gear where there is a stress concentration, the bending moment is zero. The question is: which point requires the larger diameter in order to meet the safety factor requirement? To answer this question, find the diameter required at each point and choose the larger. Start with the point where the bending moment is maximum. The maximum moment in the shaft occurs at the right bearing as seen in the moment diagram in Figure B-3(a) in Appendix B (note that in the figure a is the distance to the load and b is the distance between bearings). Using the equation given in the figure, calculate the alternating bending moment (the mean is zero). Ma P ( a b ) Ma 2000 in lbf

2.

3.

Calculate the mean and alternating components of torque. Tm Ta Tmax Tmin 2 Tmax Tmin 2 Tm 1000 in lbf Ta 1000 in lbf S'e 0.5 S ut S'e 54 ksi

4. 5.

Calculate the unmodified endurance limit.

Determine the endurance limit modification factors for a rotating round shaft. Load Size Cload 1 Csize( d ) 0.869 A 2.70

in
d

0.097

Surface

b 0.265
b

(machined)

Csurf Temperature Reliability

S ut A ksi

Csurf 0.781

Ctemp 1 Creliab 0.814 (R = 99%)

MACHINE DESIGN - An Integrated Approach, 4th Ed.

10-9a-2

6.

Determine the modified endurance limit as a function of the unknown diameter, d. S e( d ) Cload Csize( d ) Csurf Ctemp Creliab S'e

7.

Use equation (10.8) with unity for all stress concentration factors as a design equation to find d. Guess Given
1

d 1 in

3 2 2 32 Nd Ma 4 Ta Tm 3 d= 4 S ut S e( d )

Diameter required at point of maximum moment: d 1 Find ( d ) Using this value for d, the size modification factor and endurance limit are: Size modification factor Endurance limit 8. Csize d 1 0.855 S e d 1 29.3 ksi

d 1 1.188 in

At the gear the bending moment is zero but the alternating and mean torques are the same as above. Also, there is a stress concentration due to the keyway that must be applied to the alternating and mean shear stresses present at the gear. First, set the moment equal to zero: Ma 0 lbf in, then determine the values of the alternating and mean fatigue stress concentration factors.

9.

Determine the geometric stress concentration factor from the upper curve in Figure 10-16 using a curve-fit equation. Kts( d ) 1.251

0.230

10. Determine the notch sensitivity of the material. Note from Figure 6-36, part 1, that a value of 20 ksi should be added to S ut to obtain a 1/2 from Table 6-6. Using a curve-fit to Table 6-6, a ( S ) 8.69657 10

2 3 3.10030 10 7 S in ksi ksi ksi 4 5 6 9 S 12 S 15 S 1.38322 10 3.28018 10 3.21209 10 ksi ksi ksi 2

2.75956 10

3 S

3.94116 10

Neuber constant Notch radius Notch sensitivity

a a S ut 20 ksi r 0.015 in q 1 1 a r

a 0.045 in

0.5

q 0.732

MACHINE DESIGN - An Integrated Approach, 4th Ed.

10-9a-3

11. Determine the fatigue stress concentration factor from equation (6.11b). Alternating factor Mean factor Kfs( d ) 1 q Kts( d ) 1 Kfsm( d ) Kfs( d )

12. Repeat step 7 using these stress concentration factors with the torsional stresses. Given
1

3 2 2 32 Nd Ma 4 Kfs( d ) Ta Tm 3 d= Kfsm( d ) 4 S e( d ) S ut d 2 Find ( d ) trial_dia

Diameter required at point of gear attachment:

d 2 1.129 in

Putting the two calculated diameters into a vector

d1 d2

13. The larger of the two must be used as the minimum shaft diameter. Thus, d max( trial_dia ). d 1.188 in 14. The factor of safety at the point of maximum bending moment is Nd 2 . The factor of safety at the gear is equal to the ratio of the two diameters cubed times Nd. Thus,

Nfgear

d1 d Nd 2

Nfgear 2.3

MACHINE DESIGN - An Integrated Approach, 4th Ed.

10-10a-1

PROBLEM 10-10a
Statement: Given: Repeat Problem 10-2 taking the stress concentration at the keyway shown in Figure P10-4 into account. Distance between bearings Distance to end of p Applied distributed load Minimum torque Maximum torque L 20 cm b 18 cm p 1000 N cm Tmin 0 N m
1

Distance to p Tensile strength Yield strength Design safety factor

a 16 cm S ut 745 MPa S y 427 MPa Nfd 2

Tmax 2000 N m

Assumptions: 1. The finish is machined, reliability is 99%, and the shaft is at room temperature. 2. The ratio of notch radius to shaft diameter in the keyway is roverd 0.021. 3. The stress concentration factors for the mean stresses are Kfm 1 and Kfsm 1. Solution: 1. See Figures P10-2 and P10-4, and Mathcad file P1010a.

There is no stress concentration at the point on the shaft where the bending moment is a maximum, which is at x = 16.3 cm (see the appendix to this problem, below). The keys are at a 16 cm and b 18 cm. On the other hand, the key at a 16 cm looks as if it extends into the section where the moment is a maximum so, use the maximum moment as the alternating bending moment. The maximum moment in the shaft occurs between a and b. See the appendix to this problem below for the determination of Ma. Ma 48.45 N m

2.

3.

Calculate the mean and alternating components of torque. Tm Ta Tmax Tmin 2 Tmax Tmin 2 Tm 1000 N m Ta 1000 N m

4.

Determine the geometric stress concentration factors from Figure 10-16 using curve-fit equations. Bending Torsion Kt 0.4521 ( roverd) Kts 1.251 ( roverd)
0.4115

Kt 2.2 Kts 3.0

0.230

5. Determine the notch sensitivity of the material for bending and torsion. Note from Figure 6-36, part 1, that a value of 20 ksi should be added to S ut to obtain a 1/2 from Table 6-6 for the torsional Neuber constant. Bending Lookup value of S ut Neuber constant Notch sensitivity S ut 108 ksi a N 0.056 in q ( d ) 1 1 aN roverd d S'ut 128 ksi a N 0.045 in
0.5 2

a N 0.056 in

0.5

Torsion

Lookup value of S ut Neuber constant

S'ut S ut 20 ksi a N 0.045 in


2

MACHINE DESIGN - An Integrated Approach, 4th Ed.

10-10a-2

Notch sensitivity

q s( d ) 1

1 aN roverd d

6.

Determine the fatigue stress concentration factors from equation (6.11b). Bending factor Torsion factor Kf ( d ) 1 q ( d ) Kt 1 Kfs( d ) 1 q s( d ) Kts 1 S'e 0.5 S ut S'e 372.5 MPa

7. 8.

Calculate the unmodified endurance limit.

Determine the endurance limit modification factors for a rotating round shaft. Load Size Cload 1 Csize( d ) 1.189 A 4.51

mm
d

0.097

Surface

b' 0.265
b'

(machined)

Csurf Temperature Reliability 9.

Sut A MPa

Csurf 0.782

Ctemp 1 Creliab 0.814 (R = 99%)

Determine the modified endurance limit as a function of the unknown diameter, d. S e( d ) Cload Csize( d ) Csurf Ctemp Creliab S'e

10. Use equation (10.8) as a design equation to find d. Guess Given


1

d 25 mm

3 2 2 32 Nfd Kf ( d) Ma 4 Kfs( d) Ta 3 Kfsm Tm d= 4 S ut S e( d ) d Find ( d ) d 65.3 mm

Using this value for d, the size modification factor, stress concentration factors, and endurance limit are: Size modification factor Endurance limit Stress concentration factors Csize( d ) 0.793 S e( d ) 187.9 MPa Kf ( d ) 2.0 Kfs( d ) 2.7

MACHINE DESIGN - An Integrated Approach, 4th Ed.


APPENDIX - Maximum bending moment 1. From inspection of Figure P10-2, write the load function equation q(x) = R1<x - 0>-1 - p<x - a>0 + p<x - b>0 + R2<x - L>-1 2. Integrate this equation from - to x to obtain shear, V(x) V(x) = R1<x - 0>0 - p<x - a>1 + p<x - b>1 + R2<x - L>0 3. Integrate this equation from - to x to obtain moment, M(x) M(x) = R1<x - 0>1 - p<x - a>2/2 + p<x - b>2/2 + R2<x - L>1

10-10a-3

4. Solve for the reactions by evaluating the shear and moment equations at a point just to the right of x = L, where both are zero. At x = L+, V = M = 0 V = R1 p ( L a ) p ( L b ) R2 = 0 M = R 1 L R1 p 2 L p 2 ( L a)
2 2

p 2

( L b) = 0
2

( L a ) ( L b )

R1 300 N R2 1700 N

R2 p ( b a ) R1 5. Define the range for x x 0 m 0.005 L L

6. For a Mathcad solution, define a step function S. This function will have a value of zero when x is less than z, and a value of one when it is greater than or equal to z. S ( x z) if ( x z 1 0 ) 7. Write the shear and moment equations in Mathcad form, using the function S as a multiplying factor to get the effect of the singularity functions. V ( x) R1 S ( x 0 m) p S ( x a ) ( x a ) p S ( x b ) ( x b ) R2 S ( x L) M ( x) R1 S ( x 0 m) x 8. Plot the shear and moment diagrams. Shear Diagram
500 0 V ( x) 500 N 1000 1500 2000

p 2

S ( x a ) ( x a )

p 2

S ( x b ) ( x b )

10 x cm

15

20

MACHINE DESIGN - An Integrated Approach, 4th Ed.

10-10a-4

Moment Diagram

50 40 30

M ( x) Nm 20 10 0 10

10 x cm

15

20

FIGURE 10-10a
Shear and Moment Diagrams for Problem 10-10a

9. Determine the maximum maximum moment from inspection of the diagrams. Maximum moment occurs where zero, which is x = c. From the shear diagram, ca R1 = bc R2 c a R2 b R1 R1 R2 c 16.300 cm Mmax 48.45 N m

Mmax M ( c) Moment at x = b M ( b ) 34 N m

MACHINE DESIGN - An Integrated Approach, 4th Ed.

10-11a-1

PROBLEM 10-11a
Statement: Determine the amount of diametral interference needed to provide a suitable interference fit for the 6-in diameter by 1-in thick gear of Figure P10-3 using a shaft diameter of 1.75 in, such that the stresses in the hub and shaft will be safe and the torque from row a in Table P10-1 can be transmitted through the interference fit. Assume that both parts are steel with the properties given below. Gear hub diameter Gear hub length Peak shaft torque d hub 6.00 in L 1.00 in Tp 2000 lbf in Shaft diameter: d shaft 1.75 in
6

Given:

Young's modulus E 30 10 psi Poisson's ratio 0.28

Assumptions: 1. Material properties are: S ut 108 ksi, and S y 62 ksi. 2. Note that the given magnitudes of the radial forces shown acting on gear(s) in this problem are not necessarily consistent with a load associated with the given torque for any real gear of reasonable pressure angle. Since gears are taken up in a later chapter, these shaft design problems ignore the real gear loadings and use an arbitrary value to provide a shaft design exercise. 3. The coefficient of friction between the hub and shaft is 0.15. Solution: 1. 2. See Figure P10-3 and Mathcad file P1011a. Nd 2

As a design choice, let the design factor of safety on torque capacity and hub failure be Calculate hub and shaft radii. Hub radius Nominal interface radius Shaft inside diameter ro 0.5 d hub r 0.5 d shaft ri 0 in

ro 3 in r 0.875 in

3.

The minimum interference is determine by the desired torque capacity. Use equation 10.14c to solve for the minimum diametral interference.

Torque capacity

T=

L r min E ro r
2 2

Nd
2

2 ro 2 ro Nd Tp

Solving for min

min

L r E ro r
2 2

min 0.00071 in

Let the minimum diametral interference be

min 0.0007 in

Note that it is not practical to specify a dimension in inches that has more than 4 decimal places. 4. Find the von Mises stress in the hub as a function of the unknown maximum diametral interference .

Interference pressure Stress in shaft Tangential Radial

p ( )

E ro r
2 2

4 r ro

ti ( ) p ( ) ri( ) p ( )

MACHINE DESIGN - An Integrated Approach, 4th Ed.


Stress in hub ro r ro r Radial
2 2 2 2

10-11a-2

Tangential

to( ) p ( )

ro( ) p ( )

These are principal stresses. The tangential stress is 1 and the radial is 3. Use equation (5.7c) to find the von Mises stress. von Mises

'o( )

E
4 r ro
2

3 ro r
4

0.5

5. 6.

There is no bending stress in the shaft at the gear, therefore

Kt 1

The safety factors against failure (yielding in the shaft and hub) during press fit can now be used to find the maximum diametral interference: Guess Shaft

min
Given Nd = S y Kt ti ( )

smax Find ( )
Hub Given Nd = Sy Kt 'o( )

smax 0.0040 in

hmax Find( )
Using the smaller of the two, 7.

hmax 0.0021 in max hmax max 0.0021 in

The torque capacity of the joint (with an assumed coefficient of friction of 0.15 ) is about 2 times the peak transmitted torque and the safety factors against failure of the shaft and hub exceed 2 therefore, the minimum and maximum diametral interference below is acceptable. Minimum diametral interference Minimum diametral interference

min 0.0007 in max 0.0021 in

8.

If we divide the tolerance on the shaft and hub equally and use the basic hole system, the shaft and hub bore specifications are: Tolerance on shaft or hub Hub bore diameter: t 0.5 max min Dmin d shaft Dmax Dmin t Shaft diameter: d max d shaft max d min d max t Maximum interference t 0.0007 in Dmin 1.7500 in Dmax 1.7507 in d max 1.7521 in d min 1.7514 in

max d max Dmin

max 0.0021 in

MACHINE DESIGN - An Integrated Approach, 4th Ed.

10-11a-3

Minimum interference 9.

min d min Dmax

min 0.0007 in

Check the safety factors against torque capacity and static failure of the hub.

Torque

Ntorque

L r min E ro r
2 2

Tp Sy

2 ro

Ntorque 2.0

Hub strength

Nhub

Kt 'o max

Nhub 2.0

MACHINE DESIGN - An Integrated Approach, 4th Ed.

10-12a-1

PROBLEM 10-12a
Statement: Assume that the device shown keyed to the shaft of Figure P10-3 is a Class 50, cast iron flywheel of 20-in outside diameter and 1-in thickness. The hub is 4-in dia and 3-in thick. Determine the maximum speed at which it can safely be run using a safety factor of 2. Use dimensions and other appropriate data from Problem 10-6 and row a in Table P10-1. Consider the transverse force P to be zero in this case. Flywheel : odfw 20 in thickfw 1 in Hub: Shaft diameter odhub 4 in thickhub 3 in d shaft 1.75 in Safety factor Tensile strength Specific weight Poisson's ratio Nos 2 S ut 52.5 ksi

Given:

0.26 lbf in 0.26

Assumptions: The hub is thicker (in the axial direction) than the flywheel disk. This has the effect of strengthening the flywheel from the shaft out to the od of the hub. This effect will be ignored and the flywheel analyzed as if the hub had the same thickness as the disk portion. Solution: 1. See Mathcad file P1012a.

The stress is maximum at the inside radius of a flywheel, which in this case is at the shaft diameter. At this radiu the radial stress is zero so that the tangential stress is the only nonzero stress and is 1. With the other principal stresses equal to zero, the load line is along the s1 axis on the 1-3 diagram. For that case, the factor of safety equation is the same for all brittle material failure theories and is Ns = S ut

2.

Use equation (10.23a) with t replaced by S ut (Ns = 1) to determine the speed at which the flywheel will fail. Inside radius Outside radius Critical radius ri 0.5 d shaft ro 0.5 odfw r ri ri 0.875 in ro 10 in r 0.875 in
1

Failure speed

fail 3 8
fail 9332 rpm

g S ut

ri ro

ri ro r
2

1 3 2 r 3

3.

The factor of safety on the operating speed (see Example 10-7) is defined as

Nos =

fail os

Maximum safe operating speed

os

fail
Nos

os 4666 rpm

Using this definition results in the static strength factor of safety, Ns, being equal to the square of Nos.

MACHINE DESIGN - An Integrated Approach, 4th Ed.

10-13a-1

PROBLEM 10-13a
Statement: Determine the critical frequency of shaft whirl for the assembly shown in Figure P10-3 using dimensions from row a of Table P10-1 and a steel shaft diameter of 2 in. Use the flywheel dimensions of Problem 10-12. blob lbf sec in odfw 20 in thickfw 1 in Hub: odhub 4 in thickhub 3 in Specific weight Solution: 1. Shaft: Distance between bearings Distance to flywheel Shaft diameter
3 2

Units: Given:

Flywheel :

a 16 in b 18 in d shaft 2.00 in E 30 10 psi


6

0.26 lbf in

Young's modulus

See Mathcad file P1013a.

Determine the shaft stiffness at the flywheel using Figure B-3(a) in Appendix B. Note that in Figure B-3(a) the dimensions a and b are the dimensions b and a, respectively in Figure P10-3. The deflection under the load (x=a in Figure B-3(a)) is y = F 6 E I ba

a b

( a b ) b ( a b ) a
3

To use the symbols of Figure P10-3, change a to b and b to a. y = F 6 E I ab b


3

b a

( b a ) a ( b a ) b
3

. Solve the above equation for F/y = k

The stiffness of the beam at the point of the applied load is k = while substituting d 4/64 for I. k 3 d shaft E 32 ab
4

F y

b a

( b a ) a ( b a ) b
3

k 9.817 10

5 lbf

in

2.

Calculate the mass of the flywheel and hub. Volume of hub Vhub Vfw mtot

2 2 odhub d shaft thickhub


4

Vhub 28.274 in

Volume of flywheel Total mass

2 2 odfw odhub thickfw


4

Vfw 301.593 in

Vhub Vfw

mtot 0.222 blob

3.

Calculate the critical frequency.

k mtot

n 2102

rad sec

n 20075 rpm

MACHINE DESIGN - An Integrated Approach, 4th Ed.


fn

10-13a-2
fn 335 Hz

n
2 mtot g k

4.

Deflection at the flywheel due to its own weight.

8.736 10

in

MACHINE DESIGN - An Integrated Approach, 4th Ed.

10-14a-1

PROBLEM 10-14a
Statement: Determine the critical frequency of shaft whirl for the assembly shown in Figure P10-4 using dimensions from row a of Table P10-1 and a steel shaft diameter of 40 mm. The cast-iron roller diameter is 3 times the shaft diameter. Distance between bearings L 20 cm Distance to start of roller a 16 cm Distance to end of roller b 18 cm Shaft diameter Solution: 1. d shaft 40 mm Shaft Young's modulus Roller specific weight Roller diameter E 206.8 GPa
3

Given:

70580 N m
d roller 3 d shaft

d roller 120 mm

See Figure P10-4 and Mathcad file P1014a.

Calculate the area moment of inertia of the shaft. Area moment of inertia I

d shaft
64

I 1.257 10 mm

2.

Calculate the weight of the roller. Volume of roller Vroller

2 2 d roller d shaft ( b a )
4
5 3

Vroller 2.011 10 mm Weight of roller Mass of roller Wroller Vroller mroller Wroller g p

Wroller 14.191 N mroller 1.447 kg Wroller ba p 709.548 N m

Distributed load on shaft due to weight of roller 3.

From inspection of Figure P10-4, write the load function equation and integrate it twice to get the shear and moment equations. q(x) = R1<x - 0>-1 - p<x - a>0 + p<x - b>0 + R2<x - L>-1 V(x) = R1<x - 0>0 - p<x - a>1 + p<x - b>1 + R2<x - L>0 M(x) = R1<x - 0>1 - p<x - a>2/2 + p<x - b>2/2 + R2<x - L>1

4.

Solve for the reactions by evaluating the shear and moment equations at a point just to the right of x = L, where both are zero. At x = L+, V = M = 0 V = R1 p ( L a ) p ( L b ) R2 = 0 M = R 1 L R1 p 2 L p 2 ( L a)
2 2

p 2

( L b) = 0
2

( L a ) ( L b )

R1 2.129 N R2 12.06 N

R2 p ( b a ) R1 5. Integrate twice more to get the slope and deflection equations.

MACHINE DESIGN - An Integrated Approach, 4th Ed.

10-14a-2

(x) = [R1<x - 0>2/2 - p<x - a>3/6 + p<x - b>3/6 + R2<x - L>2/2 + C3]/EI
y(x) = [R1<x - 0>3/6 - p<x - a>4/24 + p<x - b>4/24 + R2<x - L>3/6 + C3x + C4]/EI 6. Solve for the constants of integration with the boundary conditions y = 0 at x = 0 and x = L. At x = 0 At x = L C4 = 0 0= R1 6 L
3

p 24

( L a)

p 24

( L b ) C 3 L

C3

R1 3 p p 4 4 ( L a) ( L b) L L 24 24 6 1

C3 0.014 N m

7. For a Mathcad solution, define a step function S. This function will have a value of zero when x is less than z, and a value of one when it is greater than or equal to z. S ( x z) if ( x z 1 0 ) 8. Write the deflection equation in Mathcad form, using the function S as a multiplying factor to get the effect of the singularity functions. y ( x) 9.

R1 p p 3 4 4 S ( x 0 mm) x S ( x a ) ( x a ) S ( x b ) ( x b ) C3 x E I 6 24 24
1

Determine the approximate stiffness of the shaft by dividing the weight of the roller by the deflection of the sha due to the roller weight at the mid-point of the roller. Roller midpoint Deflection at x = c c ab 2 c 170 mm

c y ( c)
k Wroller

c 2.345 10

mm

Stiffness

k 6.051 10

8 N

10. Calculate the critical frequency.

k mroller

n 20449

rad sec

n 195275 rpm
fn

n
2

fn 3255 Hz

MACHINE DESIGN - An Integrated Approach, 4th Ed.

10-15a-1

PROBLEM 10-15a
Statement: What are the maximum, minimum, and average power values for the shaft shown in Figure P10-1 for the data in row a of Table P10-1 if the shaft speed is 750 rpm? Shaft speed Minimum torque Maximum torque Solution: 1. See Mathcad file P1015a.

Given:

750 rpm
Tmin 0 in lbf Tmax 2000 in lbf

78.54

rad sec

Use equations (10.1) to calculate the power values. Tmax Tmin 2

Average torque Maximum power Minimum power Average power 2.

Tavg

Tavg 1000 in lbf Pmax 23.8 hp Pmin 0 hp Pavg 11.9 hp

Pmax Tmax Pmin Tmin Pavg Tavg

Note, if doing these calculations by hand, that suitable unit conversion factors must be used.

MACHINE DESIGN - An Integrated Approach, 4th Ed.

10-16a-1

PROBLEM 10-16a
Statement: What are the maximum, minimum, and average power values for the shaft shown in Figure P10-2 for the data in row a of Table P10-1 if the shaft speed is 50 rpm? Shaft speed Minimum torque Maximum torque Solution: 1. See Mathcad file P1016a.

Given:

50 rpm
Tmin 0 N m Tmax 2000 N m

5.236

rad sec

Use equations (10.1) to calculate the power values. Average torque Maximum power Minimum power Average power Tavg Tmax Tmin 2 Tavg 1000 N m Pmax 10.5 kW Pmin 0 kW Pavg 5.24 kW

Pmax Tmax Pmin Tmin Pavg Tavg

2.

Note, if doing these calculations by hand, that suitable unit conversion factors must be used.

MACHINE DESIGN - An Integrated Approach, 4th Ed.

10-17a-1

PROBLEM 10-17a
Statement: Figure P10-5 shows a roller assembly driven by a gear. The roller extends over 80% of the length a and is centered in that dimension. The roller occupies 95% of the exposed shaft length between the bearing faces. The shaft is steel of S ut = 745 MPa and S y = 427 MPa. For the data in row a of Table P10-1, find: (a) The safety factor against fatigue failure for a shaft diameter of 40 mm. (b) The maximum torsional deflection between gear and roller. (c) The torsional natural frequency of the shaft. Distance to end of shaft Distance to gear Applied distributed load Minimum torque Maximum torque L 20 cm b 18 cm p 1000 N cm Tmin 0 N m Distance between bearings Tensile strength
1

Given:

a 16 cm S ut 745 MPa S y 427 MPa d 40 mm P 1000 N

Yield strength Shaft diameter Concentrated load

Tmax 2000 N m

Assumptions: 1. The finish is machined, reliability is 90%, and the shaft is at room temperature. 2. Use SI units with Table P10-1 consistent with Problem 10-2. 3. The ratio of notch radius to shaft diameter in the keyway is roverd 0.021. 4. The roller stiffens the shaft torsionally so that the torsional deflection will occur largely between the right end of the roller (at the second or middle key) and the gear (right key). 5. Note that the given magnitudes of the radial forces shown acting on gear(s) in this problem are not necessarily consistent with a load associated with the given torque for any real gear of reasonable pressure angle. Since gears are taken up in a later chapter, these shaft design problems ignore the real gear loadings and use an arbitrary value to provide a shaft design exercise. Solution: Part (a) 1. The maximum moment in the shaft occurs at approximately the middle of the roller. See the appendix to this problem below for the determination of Mmax. The maximum moment is also the alternating moment. Thus, Mmax 297.3 N m 2. Ma Mmax Ma 297.3 N m See Figure P10-5 and Mathcad file P1017a.

Calculate the mean and alternating components of torque, assuming that one half is taken out at the right-hand key and the other half at the left-hand key. At the middle of the roller only one half of the torque is present. Tm

Tmax Tmin 2 2
1

Tm 500 N m

Ta

Tmax Tmin 2 2
1

Ta 500 N m S'e 0.5 S ut S'e 372.5 MPa

3. 4.

Calculate the unmodified endurance limit.

Determine the endurance limit modification factors for a rotating round shaft. Load Size Surface Cload 1 Csize 1.189 A 4.51

mm
d

0.097

Csize 0.831 (machined)

b' 0.265

MACHINE DESIGN - An Integrated Approach, 4th Ed.


b'

10-17a-2

Csurf Temperature Reliability 5.

Sut A MPa

Csurf 0.782

Ctemp 1 Creliab 0.897 (R = 90%)

Determine the modified endurance limit as a function of the unknown diameter, d. S e Cload Csize Csurf Ctemp Creliab S'e S e 217.148 MPa

6.

Use equation (10.8) with unity for all stress concentration factors to find the factor of safety at the point in the shaft where the bending moment is maximum.

M 2 3 T 2 3 a a 4 d Nfc 32 Se
7.

3 4

Tm

S ut

Nfc 2.1

Although the bending moment is lower at the ends of the roller where the keys are, there is a stress concentration there that is not present at the point where the bending moment is a maximum. Of the two key locations, the one on the right is critical (even though the moment is lower there) because the full torque is present in the shaft at this point. The bending moment and torque at the right keyway (x = f) are: Mf 84.4 N m Tm Ma Mf Tm 1000 N m Ma 84.4 N m

Tmax Tmin 2 Tmax Tmin 2

Ta 8.

Ta 1000 N m

Determine the geometric stress concentration factors from Figure 10-16 using the assumed r/d. For an r/d ratio of roverd 0.021 Kt 2.2 Kts 3.0

9.

Determine the notch sensitivity of the material for bending and torsion. Note from Figure 6-36, part 1, that a value of 20 ksi should be added to S ut to obtain a 1/2 from Table 6-6 for the torsional Neuber constant. Notch radius Bending Lookup value of S ut Neuber constant Notch sensitivity r roverd d S ut 108 ksi a N 0.056 in q 1 1 aN r S'ut 128 ksi a N 0.045 in
0.5 2

r 0.84 mm

a N 0.056 in q 0.765

0.5

Torsion

Lookup value of S ut Neuber constant

S'ut S ut 20 ksi a N 0.045 in


2

MACHINE DESIGN - An Integrated Approach, 4th Ed.

10-17a-3

Notch sensitivity

q s 1

1 aN r

q s 0.802

10. Determine the fatigue stress concentration factors from equation (6.11b). Bending factor Torsion factor Kf 1 q Kt 1 Kfs 1 q s Kts 1 Kf 1.917 Kfs 2.603

Maximum and minimum nominal stress

anom

16 Ta

anom 79.6 MPa

mnom

16 Tm

mnom 79.6 MPa

maxnom mnom anom minnom mnom anom

maxnom 159.155 MPa minnom 0 MPa

Using equations 6.17, the mean-stress fatigue-concentration factors are Kfm max min K S return K if K max S return S K anom if K max S

mnom

return 0 if K max min 2 S Kfsm Kfm maxnom minnom Kfs 0.577 S y

Kfsm 0.49

11. Use equation (10.8) with the stress concentration factors above to find the factor of safety at the point in the shaft where the right-hand roller key is located. 3 2 2 3 Kf Ma K fs Ta 4 d Nff 32 Se

Kfsm Tm 4 S ut
3

Nff 0.57

So, in this case, the critical section on the shaft is not at the point of maximum bending moment. This is for two reasons, 1) there is no stress concentration at this point but there is elsewhere, and 2) the torque is maximum at the point of a stress concentration. Part (b) 12. For the assumptions made, the length of the shaft that will deflect torsionally is Lbf b f J Lbf 36 mm

13. Calculate the polar moment of inertia of the shaft.

d
32

4 14. Using 105 mm(10.9a), calculate the angular deflection between the gear and the right-hand key for the maximu J 2.513 equation applied torque.

MACHINE DESIGN - An Integrated Approach, 4th Ed.


G 80.8 GPa

10-17a-4

Modulus of rigidity Angular deflection Part (c)

Tmax Lbf J G

0.203 deg

15. Use equations (10.27) to determine the shaft spring constant, mass moment of inertia, and natural frequency. Density of steel

76500

kg m
3

Mass of shaft

mshaft

d
4

L d
2

mshaft 19.227 kg
3 2

Mass moment of inertia

Im kt

mshaft 2 G J Lbf kt Im

Im 3.845 10 kt 564090

m kg

Spring constant

N m rad rad sec

Natural frequency

n 12112

fn APPENDIX - Maximum bending moment

n
2

fn 1928 Hz

1. Determine the distance from the origin to the left and right ends of the roller. Distance to left end Distance to right end e 0.1 a f 0.9 a e 16 mm f 144 mm

2. From inspection of Figure P10-5, write the load function equation q(x) = R1<x - 0>-1 - p<x - e>0 + p<x - f>0 + R2<x - a>-1 - P<x - b>-1 2. Integrate this equation from - to x to obtain shear, V(x) V(x) = R1<x - 0>0 - p<x - e>1 + p<x - f>1 + R2<x - a>0 - P<x - b>0 3. Integrate this equation from - to x to obtain moment, M(x) M(x) = R1<x - 0>1 - p<x - e>2/2 + p<x - f>2/2 + R2<x - a>1 - P<x - b>1 4. Solve for the reactions by evaluating the shear and moment equations at a point just to the right of x = L, where both are zero. At x = b +, V = M = 0 V = R1 p ( b e ) p ( b f ) R 2 P = 0 M = R 1 b
2

p 2

( b e)
2

p 2

( b f ) R 2 ( b a ) = 0

R1

e f 2 a

f e p

b a P a

R1 6275 N

MACHINE DESIGN - An Integrated Approach, 4th Ed.

10-17a-5

R2 p ( f e) R1 P 5. Define the range for x x 0 m 0.005 b b

R2 7525 N

6. For a Mathcad solution, define a step function S. This function will have a value of zero when x is less than z, and a value of one when it is greater than or equal to z. S ( x z) if ( x z 1 0 ) 7. Write the shear and moment equations in Mathcad form, using the function S as a multiplying factor to get the effect of the singularity functions. V ( x) R1 S ( x 0 m) p S ( x e) ( x e) p S ( x f ) ( x f ) R2 S ( x a ) P S ( x b ) M ( x) R1 S ( x 0 m) x 8. Plot the shear and moment diagrams. p 2 S ( x e) ( x e)
2

p 2

S ( x f ) ( x f ) R2 S ( x a ) ( x a )

SHEAR
10 300

MOMENT

5 V ( x) kN 0 M ( x) Nm

200

100

10

10 x cm

15

20

100

10 x cm

15

20

FIGURE 10-17a

Shear and Moment Diagrams for Problem 10-17a

9. Determine the maximum maximum moment from inspection of the diagrams. Maximum moment occurs where V zero, which is x = c. From the shear diagram, ce R1 = f c R2 P c f R1 e R2 e P R1 R2 P c 7.875 cm Mmax 297.3 N m Me M ( e) Mf M ( f ) Ma M ( a ) Me 100.4 N m Mf 84.4 N m Ma 20 N m

Mmax M ( c)

The moment at the left edge of the roller (at key) is The moment at the right edge of the roller (at key) is The moment at the right-hand bearing is

MACHINE DESIGN - An Integrated Approach, 4th Ed.

10-18a-1

PROBLEM 10-18a
Statement: Figure P10-5 shows a roller assembly driven by a gear. The roller extends over 80% of the length a and is centered in that dimension. The roller occupies 95% of the exposed shaft length between the bearing faces. For the data in row a of Table P10-1, find the maximum bending deflection of th 40-mm-diameter shaft. Distance to end of shaft Distance to gear Applied distributed load Maximum torque Minimum torque L 20 cm b 18 cm Distance between bearings Shaft diameter
1

Given:

a 16 cm d 40 mm P 1000 N E 206.8 GPa

p 1000 N cm Tmax 2000 N m Tmin 0 N m

Concentrated load Young's modulus

Assumptions: 1. The roller does not stiffen the shaft in bending but it does transmit the distributed load directly to the shaft. 2. Note that the given magnitudes of the radial forces shown acting on gear(s) in this problem are not necessarily consistent with a load associated with the given torque for any real gear of reasonable pressure angle. Since gears are taken up in a later chapter, these shaft design problems ignore the real gear loadings and use an arbitrary value to provide a shaft design exercise. Solution: 1. See Figure P10-5 and Mathcad file P1018a.

Determine the distance from the origin to the left and right ends of the roller and the area moment of inertia. Distance to left end Distance to right end Moment of inertia e 0.1 a f 0.9 a I e 16 mm f 144 mm I 1.257 10 mm
5 4

d
64

2.

From inspection of Figure P10-5, write the load function equation and integrate it twice to get the shear and moment equations. q(x) = R1<x - 0>-1 - p<x - e>0 + p<x - f>0 + R2<x - a>-1 - P<x - b>-1 V(x) = R1<x - 0>0 - p<x - e>1 + p<x - f>1 + R2<x - a>0 - P<x - b>0 M(x) = R1<x - 0>1 - p<x - e>2/2 + p<x - f>2/2 + R2<x - a>1 - P<x - b>1

3.

Solve for the reactions by evaluating the shear and moment equations at a point just to the right of x = L, where both are zero. At x = b +, V = M = 0 V = R1 p ( b e ) p ( b f ) R 2 P = 0 M = R 1 b R1 p 2 ( b e)
2

p 2

( b f ) R 2 ( b a ) = 0 R1 6275 N R2 7525 N

e2 f 2 b a f e p a P 2 a

R2 p ( f e) R1 P 4. Integrate twice more to get the slope and deflection equations.

(x) = [R1<x - 0>2/2 - p<x - e>3/6 + p<x - f>3/6 + R2<x - a>2/2 - P<x - b>2/2 + C3]/EI
y(x) = [R1<x - 0>3/6 - p<x - e>4/24 + p<x - f>4/24 + R2<x - a>3/6 - P<x - b>3/6 + C3x + C4]/EI

MACHINE DESIGN - An Integrated Approach, 4th Ed.


5. Solve for the constants of integration with the boundary conditions y = 0 at x = 0 and x = a. At x = 0 At x = L C4 = 0 0= R1 6 a
3

10-18a-2

p 24

( a e)

p 24

( a f ) C 3 a

C3

R1 3 p p 4 4 ( a e) (a f ) a a 24 24 6 1

C3 15.578 N m

6. 7.

Define the range for x

x 0 m 0.005 L L

For a Mathcad solution, define a step function S. This function will have a value of zero when x is less than z, and a value of one when it is greater than or equal to z. S ( x z) if ( x z 1 0 )

8.

Write the slope and deflection equations in Mathcad form, using the function S as a multiplying factor to get the effect of the singularity functions.

( x)

1 E I

R1
2

S ( x 0 mm) x

p 6

S ( x e) ( x e)

p 6

R 2 S ( x a ) ( x a) 2 P S( x b) ( x b ) 2 C3 2 2
S ( x 0 mm) x
3

S ( x f ) ( x f )

y ( x)

1 E I

R1
6

p 24

S ( x e) ( x e)

p 24

R 2 S( x a) ( x a ) 3 P S ( x b ) ( x b) 3 C3 x 6 6

S ( x f ) ( x f )

9.

Plot the slope and deflection diagrams (see Figure 10-18).

SLOPE
1 0.04

DEFLECTION

0.5

0.02

( x)
10
3

y ( x) 0 mm 0

0.5

0.02

10 x cm

15

20

0.04

10 x cm

15

20

FIGURE 10-18

Slope and Deflection Diagrams for Problem 10-18a

MACHINE DESIGN - An Integrated Approach, 4th Ed.

10-18a-3

10. The maximum deflection occurs at the value of x for which is zero. Let this be x = c, then guess c 8 cm Given

( c) = 0

c Find ( c)

c 7.955 cm

11. The maximum deflection occurs at x = c and is ymax y ( c) 12. The deflection at the end of the shaft is yL y ( L) yL 0.0229 mm ymax 0.0300 mm

MACHINE DESIGN - An Integrated Approach, 4th Ed.

10-19a-1

PROBLEM 10-19a
Statement: Figure P10-6 shows two gears on a common shaft. Assume that the constant radial force P1 is 40% of P2. For the data in row a from Table P10-1, find the diameter of the shaft required to obtain a safety factor of 2 in fatigue loading if the shaft is steel with properties given below. Maximum torque Radial load Shaft dimensions: S ut 108 ksi b 18 in Yield strength S y 62 ksi Design safety factor L 20 in Nd 2 Assumptions: There are no applied axial loads. Steel will be used for infinite life. The shaft is machined, reliability is 90%, and the shaft is at room temperature. The ratio of the fillet radius in the keyway to shaft diameter is roverd 0.02. Solution: 1. See Mathcad file P1019a. Tmax 2000 lbf in P2 1000 lbf a 16 in Minimum torque Load factor Tensile strength Tmin 0 lbf in n 0.4

Given:

Calculate the radial load on gear 1: P1 n P2 P1 400 lbf

2.

Using Figures D-2(a) and D-3(a), the reaction and moment equations are R1 P1 1 a

P2 1 b

R1 67 lbf R2 1467 lbf

R2 P1 P2 R1 M(z) = R1<z>1 - P1<z - a>1 + R2<z-b>1 - P2<z - L>1 3. 4. Define the range for z z 0 in 0.005 L L

For a Mathcad solution, define a step function S. This function will have a value of zero when z is less than u, and a value of one when it is greater than or equal to u. S ( z u ) if ( z u 1 0 )

5.

Write the shear and moment in Mathcad form, using the function S as a multiplying factor to get the effect of the singularity functions. V ( z) R1 S ( z 0 in) P1 S ( z a ) R2 S ( z b ) P2 S ( z L) M ( z) R1 S ( z 0 mm) ( z 0 mm) P1 S ( z a ) ( z a ) R2 S ( z b ) ( z b ) P2 S ( z L) ( z L)

6.

Plot the shear and moment diagrams (see Figure 10-19a). The maximum moment occurs at the right-hand bearing and is MB M ( b ) . The moment at the left-hand gear is MA M ( a ) . The moment at the right-hand gear is zero so the two points of interest are A and B. At A the bending moment is lower than at B but there is a stress concentration due to the key seat. At B the moment is a maximum but there is no stress concentration. We will investigate both points. MA 1067 in lbf MB 2000 in lbf

MACHINE DESIGN - An Integrated Approach, 4th Ed.

10-19a-2

Shear Diagram
1000 0

Moment Diagram

500 500 V ( z) lbf 0 1500 1000 lbf in M ( z)

500

10 z in

15

20

2000

10 z in

15

20

FIGURE 10-19a

Shear and Moment Diagrams for Problem 10-19a

7.

Calculate the mean and alternating components of torque. Tm Ta Tmax Tmin 2 Tmax Tmin 2 Tm 1000 in lbf Ta 1000 in lbf

Right-hand bearing (point B) 8. 9. Calculate the unmodified endurance limit. S'e 0.5 S ut S'e 54 ksi

Determine the endurance limit modification factors for a rotating round shaft. Load Size Surface Cload 1 d Csize( d ) 0.869 in A 2.70
0.097

b 0.265
b

(machined)

Csurf Temperature Reliability

S ut A ksi

Csurf 0.781

Ctemp 1 Creliab 0.897 (R = 90%)

10. Determine the modified endurance limit as a function of the unknown diameter, d. S e( d ) Cload Csize( d ) Csurf Ctemp Creliab S'e 11. Use equation (10.8) with unity for all stress concentration factors as a design equation to find d. Guess d 1 in

MACHINE DESIGN - An Integrated Approach, 4th Ed.

10-19a-3

Given
1 3

3 2 2 32 Nd MB 4 Ta Tm 3 d= 4 S ut S e( d ) d B Find ( d )

d B 1.153 in Csize d B 0.857 S e d B 32.4 ksi

Using this value for d, the size modification factor and endurance limit are: Size modification factor Endurance limit Left-hand gear (point A) 12. Determine the geometric stress concentration factors from Figure 10-16 using a curve-fit equation. Bending Torsion Kt 0.452 ( roverd)
0.4115 0.230

Kt 2.3 Kts 3.1

Kts 1.251 ( roverd)

13. Determine the notch sensitivity of the material in bending and torsion. Note from Figure 6-36, part 1, that a value of 20 ksi should be added to S ut to obtain a 1/2 from Table 6-6 when the loading is torsional. Bending: Lookup value of S ut Neuber constant Notch radius Notch sensitivity S'ut S ut a 0.057 in r( d ) roverd d q ( d ) 1 Torsion: Lookup value of S ut Neuber constant Notch radius Notch sensitivity S'ut S ut 20 ksi a s 0.045 in r( d ) roverd d q s( d ) 1 1 as r( d )
2 2

S'ut 108 ksi a 0.057 in


0.5

1 a r( d )

S'ut 128 ksi a s 0.045 in


0.5

14. Determine the fatigue stress concentration factors from equation 6.11b and 6.17. Bending Torsion Kf ( d ) 1 q ( d ) Kt 1 Kfs( d ) 1 q s( d ) Kts 1

MACHINE DESIGN - An Integrated Approach, 4th Ed.

10-19a-4

Mean factor

Kfsm( d ) Kfs( d )

15. Repeat step 11 using these stress concentration factors with the mean and alternating stresses. Given 3 2 2 32 Nd Kf ( d ) MA 4 Kfs( d) Ta Tm 3 d= Kfsm( d ) 4 S e( d ) S ut Diameter required at point A: 16. Check the assumption that Kfsm = Kfs using equation 6.17. d A Find ( d )
1 3

d A 1.337 in

maxnom

16

Tm Ta

maxnom 10.2 ksi


0.577 S y 35.774 ksi

Kfs( d ) maxnom 26.2 ksi

Since Kfsmax is less than S sy, the assumption was valid. 17. Since d A is larger than d B, it must be used as the minimum shaft diameter. Thus, d d A. 18. The factor of safety at A is Nd 2 . The safety factor at B is d 1.337 in

M 2 3 T 2 3 B a 4 d 3 Tm NB 32 4 S ut S e( d )

NB 3.1

MACHINE DESIGN - An Integrated Approach, 4th Ed.

10-20-1

PROBLEM 10-20
Statement: A 300-mm-long, solid, straight shaft is supported in self-aligning bearings at each end. A gear is attached at the middle of the shaft with a 10-mm square, steel key in a slot. The geometric stress-concentration factor in the keyslot is 2.5 and its corner radius is 0.5 mm. The gear drives a fluctuating load that creates a bending moment that varies from +10 N-m to +100 N-m and a torque that varies from -35 N-m to +170 N-m each cycle. The material chosen is cold-drawn 4140 steel, hardened and tempered to RC 45 (S ut = 1250 MPa). Design the shaft for infinite life and determine the diameter needed for a safety factor of 1.5. Minimum moment Maximum moment Minimum torque Maximum torque Mmin 10 N m Mmax 100 N m Tmin 35 N m Tmax 170 N m Stress conc. factor Notch radius Tensile strength Kt 2.5 r 0.5 mm S ut 1250 MPa

Given:

Design safety factor Nfd 1.5

Assumptions: 1. The finish is machined, reliability is 90%, and the shaft is at room temperature. 2. The geometric stress concentration factors are the same for bending and torsion. 3. Note that the given magnitudes of the radial forces shown acting on gear(s) in this problem are not necessarily consistent with a load associated with the given torque for any real gear of reasonable pressure angle. Since gears are taken up in a later chapter, these shaft design problems ignore the real gear loadings and use an arbitrary value to provide a shaft design exercise. Solution: 1. See Mathcad file P1020.

Calculate the mean and alternating components of bending moment and torque. Mm Ma Tm Ta Mmax Mmin 2 Mmax Mmin 2 Tmax Tmin 2 Tmax Tmin 2 Mm 55 N m Ma 45 N m Tm 68 N m Ta 103 N m

2. Determine the notch sensitivity of the material for bending and torsion. Note from Figure 6-36, part 1, that a value of 20 ksi should be added to S ut to obtain a 1/2 from Table 6-6 for the torsional Neuber constant. Bending Lookup value of S ut Neuber constant Notch sensitivity S ut 181 ksi a N 0.024 in q 1 Torsion Lookup value of S ut Neuber constant Notch sensitivity 1 aN r S'ut 201 ksi a N 0.018 in q s 0.886
0.5 2

a N 0.024 in q 0.854

0.5

S'ut S ut 20 ksi a N 0.018 in q s 1 1 aN r


2

MACHINE DESIGN - An Integrated Approach, 4th Ed.


3. Determine the fatigue stress concentration factors from equation (6.11b). Bending factor Torsion factor Assume Kf 1 q Kt 1 Kfs 1 q s Kt 1 Kfm Kf S'e 0.5 S ut Kfsm Kfs S'e 625 MPa Kf 2.281 Kfs 2.329

10-20-2

4. 5.

Calculate the unmodified endurance limit.

Determine the endurance limit modification factors for a rotating round shaft. Load Size Cload 1 Csize( d ) 1.189 A 4.51 Csurf A Temperature Reliability Ctemp 1 Creliab 0.897 (R = 90%)

mm
d

0.097

Surface

b' 0.265

(machined)

Sut MPa

b'

Csurf 0.682

6.

Determine the modified endurance limit as a function of the unknown diameter, d. S e( d ) Cload Csize( d ) Csurf Ctemp Creliab S'e

7.

Use equation (10.8) as a design equation to find d. Guess Given


1 3

d 20 mm

2 3 2 32 Nfd Kf Ma 4 Kfs Ta d= S e( d )
d Find ( d ) d 23.395 mm

Kfm Mm

S ut

3 4

Kfsm Tm

Round this to

d 22 mm

Using this value for d, the size modification factor and endurance limit are: Size modification factor Endurance limit Csize( d ) 0.881 S e( d ) 337 MPa

MACHINE DESIGN - An Integrated Approach, 4th Ed.

10-21-1

PROBLEM 10-21
Statement:

_____

Figure P10-7 shows a dial assembly. The dial is 500-mm-dia by 16-mm-thick, solid steel and is bolted to the top flange of the shaft. The shaft is 70 mm dia over length L1, 40 mm dia over length L2, and is carried on tapered roller bearings. All shaft fillet radii are 0.5 mm. A 5 kW motor drives the shaft through a 20:1 reduction gearbox and a commercial bellows coupling. The maximum rated torque from the motor is 17.75 N-m and its stall torque is 3x its rated torque. Find the maximum stress and angular deflection in the shaft under conditions of dial locked and motor at stall. Rated motor torque Gear reduction ratio Shaft dimensions TMR 17.75 N m mG 20 d 1 70 mm Stall torque factor Modulus of rigidity L1 144 mm d 2 40 mm fstall 3 G 80.8 GPa L2 24 mm

Given:

Solution: 1.

See Mathcad file P1021.

For a locked dial the motor will be stalled. The torque on the shaft at the gearbox output will be Tss TMR fstall mG Tss 1065 N m

2.

Calculate the polar moment of inertia and outside radius of each section of the shaft. J1

d 1
32

J1 2.357 10 mm
4

r1 0.5 d 1

r1 35 mm

J2 3.

d 2
32

J2 2.513 10 mm

r2 0.5 d 2

r2 20 mm

Calculate the stress in each section of the shaft when the dial is locked.

Tss r1 J1

1 15.8 MPa

Tss r2 J2

2 84.8 MPa

The maximum stress for a locked dial is in the smaller diameter shaft and is

max 2
4.

max 84.8 MPa

Calculate the deflection of the shaft under the stall torque.

stall

Tss L1 J 1 G

Tss L2 J2 G

stall 0.118 deg

MACHINE DESIGN - An Integrated Approach, 4th Ed.

10-22-1

PROBLEM 10-22
Statement:

_____

For the dial assembly of Problem 10-21 and Figure P10-7, find the first torsional natural frequency the shaft-dial assembly as felt at the motor shaft. The gearbox has a torsional stiffness of 1.56E5 N-m/rad and the coupling has a torsional stiffness of 5.1E5 N-m/rad. Assume the dial is empty of tooling and that ground support is infinitely stiff. Dial dimensions: Gear reduction ratio Shaft dimensions D 500 mm mG 20 d 1 70 mm
5

Given:

t 16 mm Density of steel L1 144 mm

7800 kg m
d 2 40 mm

L2 24 mm
5

Gearbox stiffness kG 1.56 10 N m Solution: 1. See Mathcad file P1022.

Coupling stiffness

kC 5.1 10 N m

Calculate the mass of the dial (the other masses will be neglected). mD

D2 t 4
2

mD 24.504 kg

2.

Calculate the mass moment of inertia of the dial. ID 1 2 mD

ID 0.766 m

2.000

kg

3.

Calculate the stiffness of the shaft using equations 10.27. For steel, G 80.8 GPa J1

d 1
32 J 1 G L1

J1 2.357 10 mm

J2

d 2
32 J 2 G L2

J2 2.513 10 mm

k1

k1 1.323 10
1

6 N m

rad

k2
5 N m

k2 8.461 10

5 N m

rad

kshaft

1 1 k 1 k2

kshaft 5.160 10

rad

4.

Calculate the stiffness of the shaft-coupling-gearbox assembly. ktotal

1 1 1 k shaft kC kG

ktotal 9.700 10

4 N m

rad

5.

Using equation 10.27a, calculate the torsional natural frequency. ktotal ID rad sec

n 356

fn

n
2

fn 56.6 Hz

MACHINE DESIGN - An Integrated Approach, 4th Ed.

10-23-1

PROBLEM 10-23
Statement:

_____

For the dial assembly of Problem 10-21 and Figure P10-7, find the first torsional natural frequency the shaft-dial-tool assembly as felt at the motor shaft. The gearbox has a torsional stiffness of 1.56E5 N-m/rad and the coupling has a torsional stiffness of 5.1E5 N-m/rad. The dial is fitted with 20 sets of tooling bolted to its top surface, equispaced on a 416-mm bolt circle. Each tooling assembly weighs 75.62 N. Assume all ground support is infinitely stiff. Dial dimensions: Gear reduction ratio Shaft dimensions Gearbox stiffness Tooling: D 500 mm mG 20 d 1 70 mm
5

Given:

t 16 mm Density of steel L1 144 mm

7800 kg m
d 2 40 mm

L2 24 mm
5

kG 1.56 10 N m Bolt circle dia

Coupling stiffness Weight

kC 5.1 10 N m Wt 75.62 N Nt 20

Dt 416 mm

Solution: 1.

See Mathcad file P1023.

Calculate the mass of the dial and the tools. mD

D2 t 4
2

mD 24.504 kg

mt Nt

Wt g

mt 154.222 kg

2.

Calculate the mass moment of inertia of the dial and the tools. D ID mD 2 2 1 Itotal ID It ID 0.766 m
2.000

kg kg

Dt It mt 2

It 6.672 m

2.000

kg

Itotal 7.438 m

2.000

3.

Calculate the stiffness of the shaft using equations 10.27. For steel, G 80.8 GPa J1

d 1
32 J 1 G L1

J1 2.357 10 mm

J2

d 2
32 J 2 G L2

J2 2.513 10 mm

k1

k1 1.323 10
1

6 N m

rad

k2
5 N m

k2 8.461 10

5 N m

rad

kshaft

1 1 k 1 k2

kshaft 5.160 10

rad

4.

Calculate the stiffness of the shaft-coupling-gearbox assembly. ktotal

1 1 1 k shaft kC kG

ktotal 9.700 10

4 N m

rad

5.

Using equation 10.27a, calculate the torsional natural frequency. ktotal Itotal rad sec

n 114

fn

n
2

fn 18.2 Hz

MACHINE DESIGN - An Integrated Approach, 4th Ed.

10-24-1

PROBLEM 10-24
Statement:

_____

For the dial assembly of Problem 10-21 and Figure P10-7, find the first torsional natural frequency the shaft-dial assembly as felt at the motor shaft. The gearbox has a torsional stiffness of 1.56E5 N-m/rad, the coupling has a torsional stiffness of 5.1E5 N-m/rad, and the motor mount has a torsional stiffness of 2.44E5 N-m. Assume the dial is empty of tooling. Dial dimensions: Gear reduction ratio Shaft dimensions D 500 mm mG 20 d 1 70 mm
5

Given:

t 16 mm Density of steel L1 144 mm

7800 kg m
d 2 40 mm

L2 24 mm
5

Gearbox stiffness kG 1.56 10 N m Motor mount stiffness Solution: 1. See Mathcad file P1024.
5

Coupling stiffness

kC 5.1 10 N m

km 2.44 10 N m

Calculate the mass of the dial (the other masses will be neglected). mD

D2 t 4
2

mD 24.504 kg

2.

Calculate the mass moment of inertia of the dial. ID 1 2 mD D

ID 0.766 m

2.000

kg

3.

Calculate the stiffness of the shaft using equations 10.27. For steel, G 80.8 GPa J1

d 1
32 J 1 G L1

J1 2.357 10 mm

J2

d 2
32 J 2 G L2

J2 2.513 10 mm

k1

k1 1.323 10
1

6 N m

rad

k2
5 N m

k2 8.461 10

5 N m

rad

kshaft

1 1 k 1 k2

kshaft 5.160 10

rad

4.

Calculate the stiffness of the shaft-coupling-gearbox-motor-mount assembly. ktotal

1 1 1 1 k shaft kC kG km

ktotal 6.941 10

4 N m

rad

5.

Using equation 10.27a, calculate the torsional natural frequency. ktotal ID rad sec

n 301

fn

n
2

fn 47.9 Hz

MACHINE DESIGN - An Integrated Approach, 4th Ed.

10-25-1

PROBLEM 10-25
Statement:

_____

The tooling on the dial assembly of Problem 10-21 and Figure P10-7 imparts a time varying torque to the dial that ranges from a peak of 30% of the motor's rated torque (at the motor) to zero, 20 time per dial revolution. If the motor speed is 600 rpm, find the worst-case stress-time and deflection-time functions for the shaft. Choose a shaft material to give a safety factor of at least 3 against failure. Disregard the response of the system to the forced vibration. Rated motor torque Gear reduction ratio Shaft dimensions Motor speed Design safety factor TMR 17.75 N m mG 20 d 1 70 mm n m 600 rpm Nd 3 Stall torque factor Modulus of rigidity L1 144 mm d 2 40 mm Tool torque factor Number of tools fstall 3 G 80.8 GPa L2 24 mm ftool 0.3 Nt 20

Given:

Solution: 1.

See Mathcad file P1025.

Calculate the mean and alternating torque components on the dial shaft. Maximum torque Minimum torque Mean torque Tmax ftool TMR mG Tmin 0 N m Tm Ta Tmax Tmin 2 Tmax Tmin 2 Tm 53.25 N m Ta 53.25 N m Tmax 106.5 N m

Alternating torque 2.

Calculate the polar moment of inertia and outside radius of each section of the shaft. J1 J2

d 1
32

J1 2.357 10 mm
4

r1 0.5 d 1 r2 0.5 d 2

r1 35 mm r2 20 mm

d 2
32

J2 2.513 10 mm

3.

Calculate the mean and alternating stress components in the smaller diameter shaft.

2m 'm
4.

Tm r2 J2 3 2m

2m 4.24 MPa 'm 7.34 MPa

2a 'a

Ta r2 J2 3 2a

2a 4.24 MPa 'a 7.34 MPa

Calculate the mean and alternating deflection of the shaft.

Tm L1 J 1 G Ta L1 J1 G

Tm L2 J 2 G Ta L2 J2 G

m 5.913 10

deg

a
5.

a 5.913 10

deg

Calculate the endurance limit correction factors using equations 6.7 and the modified endurance limit, as a function of S ut, using equations 6.5a and 6.6.

MACHINE DESIGN - An Integrated Approach, 4th Ed.

10-25-2
0.097

Cload 1 Csurf S ut 4.51

d2 Csize 1.189 mm S ut MPa


0.265

Csize 0.831

Ctemp 1

Creliab 0.897

(R = 0.90)

S e S ut Cload Csize Csurf S ut Ctemp Creliab 0.5 S ut 6. Calculate the fatigue stress-concentration factors at the step where shafts 1 and 2 join for a fillet radius of r 0.5 mm. Use Figure C-3 in appendix C to calculate the geometric stress-concentration factor and equations 6.11 and 6.17 to calculate the fatigue factors. Doverd A b d1 d2 Doverd 1.750 ( 0.84897 0.86331 ) 0.86331 A 0.8580 b 0.2360

2 Doverd 2 1.33 2 Doverd 2 1.33

( 0.23161 0.23865 ) 0.23865 Kts 2.41

Kts A

d2
r

Neuber constant function: a ( S ) 8.69657 10

2 3 3.10030 10 7 S in ksi ksi ksi 4 5 6 9 S 12 S 15 S 1.38322 10 3.28018 10 3.21209 10 ksi ksi ksi 2

2.75956 10

3 S

3.94116 10

Notch sensitivity

q S ut 1

1 a S ut 20 ksi r

Alternating factor Mean factor 7.

Kfs S ut 1 q S ut Kts 1 Kfsm S ut Kfs S ut

Using equation 10.8, solve for the minimum required ultimate tensile strength. Guess S ut 200 MPa
1

Given

3 32 Nd 4 Kfs Sut Ta d2 = S e S ut
S ut Find S ut

Kfsm S ut Tm 4 S ut
3

S ut 112 MPa

MACHINE DESIGN - An Integrated Approach, 4th Ed.

10-26-1

PROBLEM 10-26
Statement:

_____

Combine the data from Problems 10-23 and 10-25 and (a) Find the ratio between the torsional forcing frequency and the first torsional natural frequency of the dial assembly. (b) Using the frequency ratio calculated in part (a), repeat Problem 10-25 taking into account the forced vibration response of the system if the damping ratio = 0.20. Dial dimensions: Gear reduction ratio Shaft dimensions D 500 mm mG 20 d 1 70 mm
5

Given:

t 16 mm Density of steel L1 144 mm

7800 kg m
d 2 40 mm
5

L2 24 mm Nt 20 Nd 3

Gearbox stiffness kG 1.56 10 N m Tooling: Motor speed Rated motor torque Solution: 1. See Mathcad file P1026. Bolt circle dia n m 600 rpm

Coupling stiffness Weight Tool torque factor

kC 5.1 10 N m Wt 75.62 N ftool 0.3

Dt 416 mm

TMR 17.75 N m

Design safety factor

Calculate the mass of the dial and the tools. mD

D2 t 4
2

mD 24.504 kg

mt Nt

Wt g

mt 154.222 kg

2.

Calculate the mass moment of inertia of the dial and the tools. ID 1 2 mD D

ID 0.766 m

2.000

kg kg

It mt

Dt 2

It 6.672 m

2.000

kg

Itotal ID It 3.

Itotal 7.438 m

2.000

Calculate the stiffness of the shaft using equations 10.27. For steel, G 80.8 GPa J1

d 1
32 J 1 G L1

J1 2.357 10 mm

J2

d 2
32 J 2 G L2

J2 2.513 10 mm

k1

k1 1.323 10
1

6 N m

rad

k2
5 N m

k2 8.461 10

5 N m

rad

kshaft

1 1 k 1 k2

kshaft 5.160 10

rad

4.

Calculate the stiffness of the shaft-coupling-gearbox assembly. ktotal

1 1 1 k shaft kC kG

ktotal 9.700 10

4 N m

rad

5.

Using equation 10.27a, calculate the torsional natural frequency. ktotal Itotal rad sec

n
6.

n 114

fn

n
2

fn 18.2 Hz

Determine the forcing frequency.

MACHINE DESIGN - An Integrated Approach, 4th Ed.

10-26-2

Dial speed

n D

nm mG

n D 30.0 rpm rad sec ff

f Nt n D
7.

f 62.8

f
2

ff 10.0 Hz

(a) Calculate the ratio of forcing frequency to natural frequency. Fratio

f n

Fratio 0.55

8.

Determine the amplitude ratio of the dial-tooling-shaft-coupling-gearbox system, when subjected to the externally forced vibration, using the graph in Figure 10-26(a). For the frequency ratio Fratio 0.55 , and damping ratio = 0.20, the amplitude (and torque) magnification is Y 1.46 . Calculate the mean and alternating torque response components on the dial shaft. Maximum torque Minimum torque Mean torque Tmax Y ftool TMR mG Tmin 0 N m Tm Ta Tmax Tmin 2 Tmax Tmin 2 Tm 77.75 N m Ta 77.75 N m Tmax 155.5 N m

9.

Alternating torque

10. Calculate the polar moment of inertia and outside radius of each section of the shaft. J1 J2

d 1
32

J1 2.357 10 mm
4

r1 0.5 d 1 r2 0.5 d 2

r1 35 mm r2 20 mm

d 2
32

J2 2.513 10 mm

11. Calculate the mean and alternating stress components in the smaller diameter shaft.

2m 'm

Tm r2 J2 3 2m

2m 6.19 MPa 'm 10.72 MPa

2a 'a

Ta r2 J2 3 2a

2a 6.19 MPa 'a 10.72 MPa

12. Calculate the mean and alternating deflection of the shaft.

m a

Tm L1 J 1 G Ta L1 J1 G

Tm L2 J 2 G Ta L2 J2 G

m 8.632 10 a 8.632 10

deg

deg

13. Calculate the endurance limit correction factors using equations 6.7 and the modified endurance limit, as a function of S ut, using equations 6.5a and 6.6. Cload 1 Csize 1.189

d2 mm

0.097

Csize 0.831

MACHINE DESIGN - An Integrated Approach, 4th Ed.

10-26-3

S ut Csurf S ut 4.51 MPa

0.265

Ctemp 1

Creliab 0.897

(R = 0.90)

S e S ut Cload Csize Csurf S ut Ctemp Creliab 0.5 S ut 14. Calculate the fatigue stress-concentration factors at the step where shafts 1 and 2 join for a fillet radius of r 0.5 mm. Use Figure C-3 in appendix C to calculate the geometric stress-concentration factor and equations 6.11 and 6.17 to calculate the fatigue factors. Doverd A b d1 d2 Doverd 1.750 ( 0.84897 0.86331 ) 0.86331 A 0.8580 b 0.2360

2 Doverd 2 1.33 2 Doverd 2 1.33

( 0.23161 0.23865 ) 0.23865 Kts 2.41

Kts A

d2
r

Neuber constant function: a ( S ) 8.69657 10

2 3 3.10030 10 7 S in ksi ksi ksi 4 5 6 9 S 12 S 15 S 1.38322 10 3.28018 10 3.21209 10 ksi ksi ksi 2

2.75956 10

3 S

3.94116 10

Notch sensitivity

q S ut 1

1 a S ut 20 ksi r

Alternating factor Mean factor

Kfs S ut 1 q S ut Kts 1 Kfsm S ut Kfs S ut

15. (b) Using equation 10.8, solve for the minimum required ultimate tensile strength. Guess S ut 200 MPa
1

Given

3 32 Nd 4 Kfs Sut Ta d2 = S e S ut
S ut Find S ut

3 4

Kfsm S ut Tm S ut

S ut 187 MPa

MACHINE DESIGN - An Integrated Approach, 4th Ed.

10-27-1

PROBLEM 10-27
Statement:

_____

An impulsive impact load of 500 N is applied tangentially to the dial rim 20 times per dial revolution. If the motor speed is 30 rpm, find the worst-case stress-time and deflection-time functions for the shaft. Choose a shaft material to give a safety factor of at least 1.5 against failure. Dial diameter Gear reduction ratio Shaft dimensions Motor speed Design safety factor D 500 mm mG 20 d 1 70 mm n m 30 rpm Nd 1.5 G 80.8 GPa L2 24 mm F 500 N Nt 20

Given:

Modulus of rigidity L1 144 mm Impact load Number of tools

d 2 40 mm

Solution: 1.

See Mathcad file P1027.

Calculate the mean and alternating torque components on the dial shaft. Assume that the impact load causes a reaction in the dial shaft that is 3 times the applied load. (See discussion of force impact loads in Section 3.8). Maximum torque Minimum torque Mean torque Tmax 3 F D 2 Tmax 375.0 N m

Tmin 0 N m Tm Ta Tmax Tmin 2 Tmax Tmin 2 Tm 187.50 N m Ta 187.50 N m

Alternating torque 2.

Calculate the polar moment of inertia and outside radius of each section of the shaft. J1 J2

d 1
32

J1 2.357 10 mm
4

r1 0.5 d 1 r2 0.5 d 2

r1 35 mm r2 20 mm

d 2
32

J2 2.513 10 mm

3.

Calculate the mean and alternating stress components in the smaller diameter shaft.

2m 'm
4.

Tm r2 J2 3 2m

2m 14.92 MPa 'm 25.84 MPa

2a 'a

Ta r2 J2 3 2a

2a 14.92 MPa 'a 25.84 MPa

Calculate the mean and alternating deflection of the shaft.

Tm L1 J 1 G Ta L1 J1 G

Tm L2 J 2 G Ta L2 J2 G

m 0.021 deg

a
5.

a 0.021 deg

Calculate the endurance limit correction factors using equations 6.7 and the modified endurance limit, as a function of S ut, using equations 6.5a and 6.6.

MACHINE DESIGN - An Integrated Approach, 4th Ed.

10-27-2
0.097

Cload 1 Csurf S ut 4.51

d2 Csize 1.189 mm S ut MPa


0.265

Csize 0.831

Ctemp 1

Creliab 0.897

(R = 0.90)

S e S ut Cload Csize Csurf S ut Ctemp Creliab 0.5 S ut 6. Calculate the fatigue stress-concentration factors at the step where shafts 1 and 2 join for a fillet radius of r 0.5 mm. Use Figure C-3 in appendix C to calculate the geometric stress-concentration factor and equations 6.11 and 6.17 to calculate the fatigue factors. Doverd A b d1 d2 Doverd 1.750 ( 0.84897 0.86331 ) 0.86331 A 0.8580 b 0.2360

2 Doverd 2 1.33 2 Doverd 2 1.33

( 0.23161 0.23865 ) 0.23865 Kts 2.41

Kts A

d2
r

Neuber constant function: a ( S ) 8.69657 10

2 3 3.10030 10 7 S in ksi ksi ksi 4 5 6 9 S 12 S 15 S 1.38322 10 3.28018 10 3.21209 10 ksi ksi ksi 2

2.75956 10

3 S

3.94116 10

Notch sensitivity

q S ut 1

1 a S ut 20 ksi r

Alternating factor Mean factor 7.

Kfs S ut 1 q S ut Kts 1 Kfsm S ut Kfs S ut

Using equation 10.8, solve for the minimum required ultimate tensile strength. Guess S ut 200 MPa
1

Given

3 32 Nd 4 Kfs Sut Ta d2 = S e S ut
S ut Find S ut

3 4

Kfsm S ut Tm S ut

S ut 244 MPa

MACHINE DESIGN - An Integrated Approach, 4th Ed.

10-28-1

PROBLEM 10-28
Statement: Given:

_____

For the dial assembly of Problem 10-21 and Figure P10-7, size a square key to couple the 40-mm-d dial shaft to the coupling. Shaft stall torque Tss 1065 N m Shaft diameter d 40 mm

Design choices: Shaft material: SAE 1050, normalized steel S y1 427 MPa Key material: SAE 1020, cold-rolled steel S y2 393 MPa S ut1 745 MPa S ut2 469 MPa

Design safety factor Nd 1, let the key shear at stall torque to protect the motor. Key is machined, reliability is 90%, and the dial operates at room temperature. Maximum torque Minimum torque Solution: 1. See Mathcad file P1028. Tmax Tss Tmin 0 N m

As recommended in Table 10-2, for a shaft diameter of d 40.0 mm, use a square key of width Key width w 12 mm

2.

Determine the alternating and mean key shear force components. Mean and alternating force components Fa Fm 1 Tmax Tmin 2 0.5 d 1 Tmax Tmin 2 0.5 d Fa 26625 N Fm 26625 N

3.

Write the equations for the mean and alternating components of the shear stress and the effective von Mises stress. Key shear area as function of length Mean and alternating shear stresses Mean and alternating von Mises stresses Ashear = w L Fa w L Fa w L Fm w L Fm w L

a =

m =

'a = 3 a = 3

'm = 3 m = 3

4.

Using the modified Goodman failure criterion, the design equation is S e S ut S ut 'a S e 'm S e S ut

Nd =

3 S F S F ut a e m w L
3 Nd S ut Fa S e Fm w S ut S e

Solving for L 5.

L=

Determine the key material endurance limit.

MACHINE DESIGN - An Integrated Approach, 4th Ed.


Uncorrected endurance strength Correction factors: Load Size Cload 1 A95 = w L Csize = 1.189 d eq Cs 1.189
0.097 0.5 0.097

10-28-2

S'e 0.5 S ut2

S'e 235 MPa

d eq =

w L 0.0766

0.0766 mm
w
0.0485

Cs 0.931

Csize = 0.931 L Surface (machined) Temperature Reliability Csurf

S ut2 4.51 MPa

0.265

Csurf 0.884

Ctemp 1 Creliab 0.897


0.0485

(R = 90%)
0.0485

S e = Cload Csize Csurf Ctemp Creliab S'e = 1 Cs L where 6. Substitute S e into the design equation. 3 Nd S ut Fa Ce L w S ut Ce L Solving by iteration, let 3 Nd S ut2 Fa Ce RHS( L)
0.0485

Csurf 1 Crelaib S'e = Ce L

Ce Cs Csurf Creliab S'e

Ce 172.973 MPa

L=

Fm

0.0485

mm
L

0.0485

Fm

w S ut2 Ce Guess L 20 mm L RHS( L) L RHS( L) Tentatively, let 7.

mm
RHS( L) 33.886 mm RHS( L) 34.551 mm RHS( L) 34.576 mm

0.0485

L 35 mm
0.0485

Check the realized factor of safety against fatigue failure. Endurance limit L S e Ce mm S e 145.6 MPa

MACHINE DESIGN - An Integrated Approach, 4th Ed.

10-28-3

Realized factor of safety

Nf

S e S ut2

3 S F S F ut2 a e m w L

Nf 1.0

8.

Check worst-case static bearing stress. Bearing area Maximum force Abear 1 2 w L Abear 210 mm Fmax 53.3 kN
2

Fmax Fa Fm Fmax Abear

Maximum bearing stress on key Factor of safety against static bearing yield

max

max 254 MPa

Ns

S y2

max

Ns 1.5

9.

Design Summary Key width Key length w 12 mm L 35 mm (12 x 12 mm)

MACHINE DESIGN - An Integrated Approach, 4th Ed.

10-29-1

PROBLEM 10-29
Statement: Given:

_____

For the dial assembly of Problem 10-25 and Figure P10-7, size a square key to couple the 40-mm-d dial shaft to the coupling. Shaft torque Tdial 106.5 N m Shaft diameter d 40 mm

Design choices: Shaft material: SAE 1050, normalized steel S y1 427 MPa Key material: SAE 1020, cold-rolled steel Design safety factor Nd 3 Key is machined, reliability is 90%, and the dial operates at room temperature. Maximum torque Minimum torque Solution: 1. See Mathcad file P1029. Tmax Tdial Tmin 0 N m S y2 393 MPa S ut1 745 MPa S ut2 469 MPa

As recommended in Table 10-2, for a shaft diameter of d 40.0 mm, use a square key of width Key width w 12 mm

2.

Determine the alternating and mean key shear force components. Mean and alternating force components Fa Fm 1 Tmax Tmin 2 0.5 d 1 Tmax Tmin 2 0.5 d Fa 2663 N Fm 2663 N

3.

Write the equations for the mean and alternating components of the shear stress and the effective von Mises stress. Key shear area as function of length Mean and alternating shear stresses Mean and alternating von Mises stresses Ashear = w L Fa w L Fa w L Fm w L Fm w L

a =

m =

'a = 3 a = 3

'm = 3 m = 3

4.

Using the modified Goodman failure criterion, the design equation is S e S ut S ut 'a S e 'm S e S ut

Nd =

3 S F S F ut a e m w L
L= 3 Nd S ut Fa S e Fm w S ut S e

Solving for L 5.

Determine the key material endurance limit. Uncorrected endurance strength S'e 0.5 S ut2 S'e 235 MPa

MACHINE DESIGN - An Integrated Approach, 4th Ed.

10-29-2

Correction factors: Load Size Cload 1 A95 = w L Csize = 1.189 d eq Cs 1.189


0.097 0.5 0.097

d eq =

w L 0.0766

0.0766 mm
w
0.0485

Cs 0.931

Csize = 0.931 L Surface (machined) Temperature Reliability Csurf

S ut2 4.51 MPa

0.265

Csurf 0.884

Ctemp 1 Creliab 0.897


0.0485

(R = 90%)
0.0485

S e = Cload Csize Csurf Ctemp Creliab S'e = 1 Cs L where 6. Substitute S e into the design equation. 3 Nd S ut Fa Ce L w S ut Ce L Solving by iteration, let 3 Nd S ut2 Fa Ce RHS( L)
0.0485

Csurf 1 Crelaib S'e = Ce L

Ce Cs Csurf Creliab S'e

Ce 172.973 MPa

L=

Fm

0.0485

mm
L

0.0485

Fm

w S ut2 Ce Guess L 20 mm L RHS( L) L RHS( L) Tentatively, let 7.

mm
RHS( L) 10.166 mm RHS( L) 9.917 mm RHS( L) 9.908 mm

0.0485

L 10 mm

Check the realized factor of safety against fatigue failure. L S e Ce mm


0.0485

Endurance limit

S e 154.7 MPa

MACHINE DESIGN - An Integrated Approach, 4th Ed.

10-29-3

Realized factor of safety

Nf

S e S ut2

3 S F S F ut2 a e m w L

Nf 3.0

8.

Check worst-case static bearing stress. Bearing area Maximum force Abear 1 2 w L Abear 60 mm Fmax 5.3 kN
2

Fmax Fa Fm Fmax Abear

Maximum bearing stress on key Factor of safety against static bearing yield

max

max 89 MPa

Ns

S y2

max

Ns 4.4

9.

Design Summary Key width Key length w 12 mm L 10 mm (12 x 12 mm)

MACHINE DESIGN - An Integrated Approach, 4th Ed.

10-30-1

PROBLEM 10-30
Statement: Given:

_____

For the dial assembly of Problem 10-27 and Figure P10-7, size a square key to couple the 40-mm-d dial shaft to the coupling. Shaft torque Tdial 375 N m Shaft diameter d 40 mm

Design choices: Shaft material: SAE 1050, normalized steel S y1 427 MPa Key material: SAE 1020, cold-rolled steel Design safety factor Nd 3 Key is machined, reliability is 90%, and the dial operates at room temperature. Maximum torque Minimum torque Solution: 1. See Mathcad file P1030. Tmax Tdial Tmin 0 N m S y2 393 MPa S ut1 745 MPa S ut2 469 MPa

As recommended in Table 10-2, for a shaft diameter of d 40.0 mm, use a square key of width Key width w 12 mm

2.

Determine the alternating and mean key shear force components. Mean and alternating force components Fa Fm 1 Tmax Tmin 2 0.5 d 1 Tmax Tmin 2 0.5 d Fa 9375 N Fm 9375 N

3.

Write the equations for the mean and alternating components of the shear stress and the effective von Mises stress. Key shear area as function of length Mean and alternating shear stresses Mean and alternating von Mises stresses Ashear = w L Fa w L Fa w L Fm w L Fm w L

a =

m =

'a = 3 a = 3

'm = 3 m = 3

4.

Using the modified Goodman failure criterion, the design equation is S e S ut S ut 'a S e 'm S e S ut

Nd =

3 S F S F ut a e m w L
L= 3 Nd S ut Fa S e Fm w S ut S e

Solving for L 5.

Determine the key material endurance limit. Uncorrected endurance strength S'e 0.5 S ut2 S'e 235 MPa

MACHINE DESIGN - An Integrated Approach, 4th Ed.


Correction factors: Load Size Cload 1 A95 = w L Csize = 1.189 d eq
0.097 0.5 0.097

10-30-2

d eq =

w L 0.0766

w Cs 1.189 0.0766 mm Csize = 0.931 L Surface (machined) Temperature Reliability Csurf


0.0485

Cs 0.931

S ut2 4.51 MPa

0.265

Csurf 0.884

Ctemp 1 Creliab 0.897


0.0485

(R = 90%)
0.0485

S e = Cload Csize Csurf Ctemp Creliab S'e = 1 Cs L where 6. Substitute S e into the design equation. 3 Nd S ut Fa Ce L w S ut Ce L Solving by iteration, let 3 Nd S ut2 Fa Ce RHS( L)
0.0485

Csurf 1 Crelaib S'e = Ce L

Ce Cs Csurf Creliab S'e

Ce 172.973 MPa

L=

Fm

0.0485

mm
L

0.0485

Fm

w S ut2 Ce Guess L 20 mm L RHS( L) L RHS( L) Tentatively, let 7.

mm
L

0.0485

RHS( L) 35.795 mm RHS( L) 36.572 mm RHS( L) 36.601 mm L 38 mm

Check the realized factor of safety against fatigue failure. S e Ce

Endurance limit

mm
L

0.0485

S e 145.0 MPa

MACHINE DESIGN - An Integrated Approach, 4th Ed.

10-30-3

Realized factor of safety

Nf

S e S ut2

3 S F S F ut2 a e m w L

Nf 3.1

8.

Check worst-case static bearing stress. Bearing area Maximum force Abear 1 2 w L Abear 228 mm Fmax 18.8 kN
2

Fmax Fa Fm Fmax Abear

Maximum bearing stress on key Factor of safety against static bearing yield

max

max 82 MPa

Ns

S y2

max

Ns 4.8

9.

Design Summary Key width Key length w 12 mm L 38 mm (12 x 12 mm)

MACHINE DESIGN - An Integrated Approach, 4th Ed.

10-31a-1

PROBLEM 10-31a
Statement:

_____

A simply supported shaft with overhanging load is shown in Figure P10-1. A constant magnitude transverse force P is applied as the shaft rotates. The shaft is also subject to a steady torque of Tmax. For the data in the row(s) assigned from Table P10-1 (ignoring Tmin), find the diameter of shaft required to obtain a safety factor of 2.5 in fatigue loading if the shaft is steel of S ut = 118 kpsi and S y = 102 kpsi. The dimensions are in inches, the force in pounds, and the torque is in lb-in. Assume no stress concentrations are present, the shaft is machined, the required reliability is 90%, and the shaft operates at room temperature. Distance between bearings a 16 in Applied load P 1000 lbf Maximum torque Design safety factor Tmax 2000 lbf in Nd 2.5 Distance to P Tensile strength Yield strength b 18 in S ut 118 ksi S y 102 ksi

Given:

Solution: 1.

See Figure P10-1, Table P10-1, and Mathcad file P1031a.

The maximum moment in the shaft occurs at the right bearing as seen in the moment diagram in Figure B-3(a) in Appendix B (note that in the figure a is the distance to the load and b is the distance between bearings). Using the equation given in the figure, calculate the alternating bending moment (the mean is zero). Ma P ( a b ) Ma 2000 in lbf

2.

Calculate the unmodified endurance limit. S'e 0.5 S ut S'e 59.0 ksi

3.

Determine the endurance limit modification factors for a rotating round shaft. Load Size Surface Cload 1 Csize( d ) 0.869 A 2.70 Csurf A Temperature Reliability Ctemp 1 Creliab 0.897 (R = 90%)

in
d

0.097

b 0.265

(machined)

S ut ksi

Csurf 0.763

4.

Determine the modified endurance limit as a function of the unknown diameter, d. S e( d ) Cload Csize( d ) Csurf Ctemp Creliab S'e

5.

Use equation 10.6a with unity for all stress concentration factors as a design equation to find d. Guess d 1.00 in
1 2 32 Nd Ma 2 3 Tmax 2 d= 4 S S e( d ) y 1 3

Given

d Find ( d )

d 1.153 in

MACHINE DESIGN - An Integrated Approach, 4th Ed.

10-32a-1

PROBLEM 10-32a
Statement: Given:

_____

Repeat Problem 10-31 taking the stress concentration at the keyway shown in Figure P10-3 into account. Distance to P Distance between bearings a 16 in b 18 in Tensile strength Applied load P 1000 lbf S ut 118 ksi Maximum torque Tmax 2000 lbf in Yield strength S y 102 ksi

Design safety factor Nd 2.5 1. The finish is machined, reliability is 90%, and the shaft is at room temperature. Assumptions: 2. The notch radius in the keyway is r 0.015 in 3. Note that the given magnitudes of the radial forces shown acting on gear(s) in this problem are not necessarily consistent with a load associated with the given torque for any real gear of reasonable pressure angle. Since gears are taken up in a later chapter, these shaft design problems ignore the real gear loadings and use an arbitrary value to provide a shaft design exercise. Solution: 1. See Figure P10-3, Table P10-1, and Mathcad file P1032a. There is no stress concentration at the point on the shaft where the bending moment is a maximum. On the other hand, at the gear where there is a stress concentration, the bending moment is zero. The question is: which point requires the larger diameter in order to meet the safety factor requirement? To answer this question, find the diameter required at each point and choose the larger. Start with the point where the bending moment is maximum. The maximum moment in the shaft occurs at the right bearing as seen in the moment diagram in Figure B-3(a) in Appendix B (note that in the figure a is the distance to the load and b is the distance between bearings). Using the equation given in the figure, calculate the alternating bending moment (the mean is zero). Ma P ( a b ) 3. Calculate the unmodified endurance limit. S'e 0.5 S ut 4. S'e 59.0 ksi Ma 2000 in lbf

2.

Determine the endurance limit modification factors for a rotating round shaft. Load Size Cload 1 Csize( d ) 0.869 A 2.70 Csurf A Temperature Ctemp 1 Creliab 0.897

in
d

0.097

Surface

b 0.265

(machined)

S ut ksi

Csurf 0.763

Reliability 5.

(R = 90%)

Determine the modified endurance limit as a function of the unknown diameter, d. S e( d ) Cload Csize( d ) Csurf Ctemp Creliab S'e

6.

Use equation 10.6a with unity for all stress concentration factors as a design equation to find d. Guess d 1.00 in

MACHINE DESIGN - An Integrated Approach, 4th Ed.

10-32a-2

1 1 2 32 Nd Ma 2 3 Tmax 2 d= S e( d ) 4 Sy 3

Given

d 1 Find ( d ) 7.

d 1 1.153 in

At the gear the bending moment is zero but the steady torque is the same as above. Also, there is a stress concentration due to the keyway that must be applied to the mean shear stress present at the gear. First, set the moment equal to zero: Ma 0 lbf in, then determine the values of the alternating and mean fatigue stress concentration factors.

8.

Determine the geometric stress concentration factor from the upper curve in Figure 10-16 using a curve-fit equation. Kts( d ) 1.251

0.230

9.

Determine the notch sensitivity of the material. Note from Figure 6-36, part 1, that a value of 20 ksi should be added to S ut to obtain a 1/2 from Table 6-6. Using a curve-fit to Table 6-6, a ( S ) 8.69657 10

2 3 3.10030 10 7 S in ksi ksi ksi 4 5 6 9 S 12 S 15 S 1.38322 10 3.28018 10 3.21209 10 ksi ksi ksi 2

2.75956 10

3 S

3.94116 10

Neuber constant Notch radius Notch sensitivity

a a S ut 20 ksi r 0.015 in q 1 1 a r

a 0.040 in

0.5

q 0.754

10. Determine the fatigue stress concentration factor from equation (6.11b). Alternating factor Mean factor Kfs( d ) 1 q Kts( d ) 1 Kfsm( d ) Kfs( d )

11. Repeat step 6 using these stress concentration factors with the torsional stresses.
1

Given
1 2 2 32 Nd Ma 2 3 Tmax d= S e( d ) 4 Kfsm( d) Sy

MACHINE DESIGN - An Integrated Approach, 4th Ed.

10-32a-3

Diameter required at point of gear attachment:

d 2 Find ( d ) trial_dia

d 2 1.061 in

Putting the two calculated diameters into a vector

d1 d2

12. The larger of the two must be used as the minimum shaft diameter. Thus, d max( trial_dia ). d 1.153 in 13. The factor of safety at the point of maximum bending moment is Nd 2.500 . The factor of safety at the gear is equal to the ratio of the two diameters cubed times Nd. Thus,

d1 Nfgear Nd d2

Nfgear 3.2

MACHINE DESIGN - An Integrated Approach, 4th Ed.

10-33a-1

PROBLEM 10-33a
Statement:

_____

A simply supported shaft is shown in Figure P10-2. A constant magnitude distributed unit load p is applied as the shaft rotates. The shaft is also subject to a steady torque of Tmax. For the data in the row(s) assigned from Table P10-1 (ignoring Tmin), find the diameter of shaft required to obtain a safety factor of 2.5 in fatigue loading if the shaft is steel of S ut = 814 MPa and S y = 703 MPa. The dimensions are in cm, the distributed force in N/cm, and the torque is in N-m. Assume no stress concentrations are present, the shaft is machined, the required reliability is 90%, and the shaft operates at room temperature. Distance between bearings L 20 cm Distance to end of p b 18 cm Applied distributed load Maximum torque p 1000 N cm Tmax 2000 N m
1

Given:

Distance to start of p Tensile strength Yield strength Design safety factor

a 16 cm S ut 814 MPa S y 703 MPa Nfd 2.5

See Figure P10-2, Table P10-1, and Mathcad file P1033a. Solution: 1. The maximum moment in the shaft occurs between a and b. See the appendix to this problem below for the determination of Ma. Ma 48.45 N m 2. Calculate the unmodified endurance limit. S'e 0.5 S ut 3. Cload 1 Csize( d ) 1.189 A 4.51 Csurf A Temperature Reliability 4. Ctemp 1 Creliab 0.897 (R = 90%) S'e 407.0 MPa

Determine the endurance limit modification factors for a rotating round shaft. Load Size Surface

mm
d

0.097

b' 0.265

(machined)

Sut MPa

b'

Csurf 0.764

Determine the modified endurance limit as a function of the unknown diameter, d. S e( d ) Cload Csize( d ) Csurf Ctemp Creliab S'e

5.

Use equation 10.6a with unity for all stress concentration factors as a design equation to find d. Guess d 20 mm
1 1 2 32 Nfd Ma 2 3 Tmax 2 d= Se( d) 4 S y 3

Given

d Find ( d )

d 39.8 mm

APPENDIX - Maximum bending moment 1. From inspection of Figure P10-2, write the load function equation q(x) = R1<x - 0>-1 - p<x - a>0 + p<x - b>0 + R2<x - L>-1

MACHINE DESIGN - An Integrated Approach, 4th Ed.


2. Integrate this equation from - to x to obtain shear, V(x) V(x) = R1<x - 0>0 - p<x - a>1 + p<x - b>1 + R2<x - L>0 3. Integrate this equation from - to x to obtain moment, M(x) M(x) = R1<x - 0>1 - p<x - a>2/2 + p<x - b>2/2 + R2<x - L>1 4.

10-33a-2

Solve for the reactions by evaluating the shear and moment equations at a point just to the right of x = L, where both are zero. At x = L+, V = M = 0 V = R1 p ( L a ) p ( L b ) R2 = 0 M = R 1 L R1 p 2 L p 2 ( L a)
2 2

p 2

( L b) = 0
2

( L a ) ( L b )

R1 300.000 N R2 1700 N

R2 p ( b a ) R1 5. 6. Define the range for x x 0 m 0.005 L L

For a Mathcad solution, define a step function S. This function will have a value of zero when x is less than z, and a value of one when it is greater than or equal to z. S ( x z) if ( x z 1 0 )

7.

Write the shear and moment equations in Mathcad form, using the function S as a multiplying factor to get the effect of the singularity functions. V ( x) R1 S ( x 0 m) p S ( x a ) ( x a ) p S ( x b ) ( x b ) R2 S ( x L) M ( x) R1 S ( x 0 m) x p 2 S ( x a ) ( x a )
2

p 2

S ( x b ) ( x b )

8.

Plot the shear and moment diagrams. Shear Diagram


500 0

Moment Diagram
50

35 V ( x) 500 N 1000 1500 2000 M ( x) Nm 20

5 10

10 x cm

15

20

10 x cm

15

20

FIGURE 10-33a

Shear and Moment Diagrams for Problem 10-33a

MACHINE DESIGN - An Integrated Approach, 4th Ed.

10-33a-3

9.

Determine the maximum maximum moment from inspection of the diagrams. Maximum moment occurs where V is zero, which is x = c. From the shear diagram, ca R1 = bc R2 c a R2 b R1 R1 R2 c 16.300 cm Mmax 48.450 N m

Mmax M ( c)

MACHINE DESIGN - An Integrated Approach, 4th Ed.

10-34a-1

PROBLEM 10-34a
Statement: Given:

_____

Repeat Problem 10-33 taking the stress concentration at the keyways shown in Figure P10-4 into account. Distance to start of p Distance between bearings L 20 cm a 16 cm Tensile strength Distance to end of p b 18 cm S ut 814 MPa Applied distributed load Maximum torque p 1000 N cm Tmax 2000 N m
1

Yield strength Design safety factor

S y 703 MPa Nfd 2.5

Assumptions: 1. The finish is machined, reliability is 90%, and the shaft is at room temperature. 2. The ratio of notch radius to shaft diameter in the keyway is roverd 0.021 3. Note that the given magnitudes of the radial forces shown acting on gear(s) in this problem are not necessarily consistent with a load associated with the given torque for any real gear of reasonable pressure angle. Since gears are taken up in a later chapter, these shaft design problems ignore the real gear loadings and use an arbitrary value to provide a shaft design exercise. Solution: 1. See Figure P10-4, Table P10-1, and Mathcad file P1034a. There is no stress concentration at the point on the shaft where the bending moment is a maximum, which is at x = 16.3 cm (see the appendix to this problem, below). The keys are at a 16 cm and b 18 cm. On the other hand, the key at a 16 cm looks as if it extends into the section where the moment is a maximum so, use the maximum moment as the alternating bending moment. The maximum moment in the shaft occurs between a and b. See the appendix to this problem below for the determination of Ma. Ma 48.45 N m 3. Calculate the unmodified endurance limit. S'e 0.5 S ut 4. S'e 407.0 MPa

2.

Determine the endurance limit modification factors for a rotating round shaft. Load Cload 1 Csize( d ) 1.189 A 4.51

Size Surface

mm
d

0.097

b' 0.265
b'

(machined)

Csurf Temperature Reliability 5.

Sut A MPa

Csurf 0.764

Ctemp 1 Creliab 0.897 (R = 90%)

Determine the modified endurance limit as a function of the unknown diameter, d. S e( d ) Cload Csize( d ) Csurf Ctemp Creliab S'e

6.

Determine the geometric stress concentration factors from Figure 10-16 using curve-fit equations. Bending Torsion Kt 0.4521 ( roverd) Kts 1.251 ( roverd)
0.4115

Kt 2.2 Kts 3.0

0.230

MACHINE DESIGN - An Integrated Approach, 4th Ed.

10-34a-2

7.

Determine the notch sensitivity of the material for bending and torsion. Note from Figure 6-36, part 1, that a value of 20 ksi should be added to S ut to obtain a 1/2 from Table 6-6 for the torsional Neuber constant. Bending aN ( S ) 8.69657 10 Lookup value of S ut
2

S ut 118 ksi 3.94116 10


5

2 3 3.10030 10 7 S in ksi ksi ksi 4 5 6 9 S 12 S 15 S 1.38322 10 3.28018 10 3.21209 10 ksi ksi ksi

2.75956 10

3 S

Neuber constant

a N aN S ut q ( d ) 1 1 aN roverd d S'ut S ut 20 ksi

a N 0.050 in

0.5

Notch sensitivity

Torsion

Lookup value of S ut

S'ut 138 ksi a N 0.040 in


0.5

Neuber constant Notch sensitivity

a N aN S ut 20 ksi q s( d ) 1 1 aN roverd d

8.

Determine the fatigue stress concentration factors from equation 6.11b. Kf ( d ) 1 q ( d ) Kt 1 Kfs( d ) 1 q ( d ) Kts 1 Kfm( d ) Kf ( d ) Kfsm( d ) Kfs( d )

9.

Use equation 10.6a as a design equation to find d. Guess d 1.00 in


1 1 2 32 Nfd Kf ( d ) Ma 2 3 Kfsm( d) Tmax 2 d= S e( d ) 4 Sy 3

Given

d 1 Find ( d )

d 1 55.0 mm

APPENDIX - Maximum bending moment 1. From inspection of Figure P10-2, write the load function equation q(x) = R1<x - 0>-1 - p<x - a>0 + p<x - b>0 + R2<x - L>-1

MACHINE DESIGN - An Integrated Approach, 4th Ed.

10-34a-3

2.

Integrate this equation from - to x to obtain shear, V(x) V(x) = R1<x - 0>0 - p<x - a>1 + p<x - b>1 + R2<x - L>0

3.

Integrate this equation from - to x to obtain moment, M(x) M(x) = R1<x - 0>1 - p<x - a>2/2 + p<x - b>2/2 + R2<x - L>1

4.

Solve for the reactions by evaluating the shear and moment equations at a point just to the right of x = L, where both are zero. At x = L+, V = M = 0 V = R1 p ( L a ) p ( L b ) R2 = 0 M = R 1 L R1 p 2 L p 2 ( L a)
2 2

p 2

( L b) = 0
2

( L a ) ( L b )

R1 300.000 N R2 1700 N

R2 p ( b a ) R1 5. 6. Define the range for x x 0 m 0.005 L L

For a Mathcad solution, define a step function S. This function will have a value of zero when x is less than z, and a value of one when it is greater than or equal to z. S ( x z) if ( x z 1 0 )

7.

Write the shear and moment equations in Mathcad form, using the function S as a multiplying factor to get the effect of the singularity functions. V ( x) R1 S ( x 0 m) p S ( x a ) ( x a ) p S ( x b ) ( x b ) R2 S ( x L) M ( x) R1 S ( x 0 m) x p 2 S ( x a ) ( x a )
2

p 2

S ( x b ) ( x b )

8.

Plot the shear and moment diagrams. Shear Diagram


500 0 V ( x) 500 N 1000 1500 0 2000 0 5 10 x cm 15 20 10 0 5 10 x cm 15 20 50 40 30 M ( x) Nm 20 10

Moment Diagram

FIGURE 10-34a
Shear and Moment Diagrams for Problem 10-34a

MACHINE DESIGN - An Integrated Approach, 4th Ed.

10-34a-4

9.

Determine the maximum maximum moment from inspection of the diagrams. Maximum moment occurs where zero, which is x = c. From the shear diagram, ca R1 = bc R2 c a R2 b R1 R1 R2 c 16.300 cm Mmax 48.450 N m

Mmax M ( c)

MACHINE DESIGN - An Integrated Approach, 4th Ed.

10-35-1

PROBLEM 10-35
Statement: Figure P10-8 shows the last stage in a gearbox with dual output. The gear is manufactured integrally with the shaft. The shaft is supported by two self-aligning ball bearings. Crank arms are connected to each end of the shaft. The load on the cranks produces equal fluctuating transverse forces on the shaft ends as well as equal fluctuating torques. The torque is transmitted through end-milled keyways in the crank and shaft and a parallel key that fits snugly in each keyway. The crank is located axially by a shoulder that is 50-mm from the plane in which the transverse load acts. The fillet radius to shaft diameter is r/d = 0.05 and the shoulder to shaft diameter ratio is D/d = 1.2. The shaft/gear material is SAE 4130 steel Q&T @ 1200F. The transverse force fluctuates from 8 kN to 16.5 kN and the torque fluctuates from 1.1 kN-m to 2.2 kN-m. For a factor of safety of 2.5 against an infinite-life fatigue failure, determine a suitable shaft diameter, d. Tensile strength S ut 814 MPa Ratios: Doverd 1.2 Loads: Pmin 8 kN Pmax 16.5 kN Distance from center of crank to shoulder Solution: 1. See Figure P10-8 and Mathcad file P1035. l 50 mm roverd 0.05 Tmax 2.2 kN m Nf 2.5 Safety factor Tmin 1.1 kN m

Given:

Calculate the mean and alternating components of the loads. Mean Pm Tm Pa Ta Pmin Pmax 2 Tmin Tmax 2 Pmax Pmin 2 Tmax Tmin 2 Pm 12.3 kN Tm 1.65 kN m Pa 4.3 kN Ta 0.55 kN m

Alternating

2.

Calculate the mean and alternating bending moment at the shoulder. Mean Alternating Mm Pm l Ma Pa l Mm 0.61 kN m Ma 0.21 kN m

3.

Using Appendix C, determine the geometric stress concentration factors for the bending and torsional stresses at the shoulder. Bending (Fig. C-2): For Doverd 1.2 roverd 0.05
b

A 0.97098

b 0.21796

Kt A ( roverd) Torsion (Fig. C-3): A 0.83425

Kt 1.865 b 0.21649

Kts A ( roverd) 4.

Kts 1.596

Calculate the notch sensitivity of the material for bending and torsion using Table 6-6. Assume a value for the notch radius and iterate the solution if necessary. Assume d 50 mm r roverd d r 2.500 mm

MACHINE DESIGN - An Integrated Approach, 4th Ed.


Bending: Neuber constant (for S ut 814 MPa) Notch sensitivity q b 1 1 Torsion: Neuber constant (for S ut 20 MPa 834 MPa) Notch sensitivity q s 1 1 5. a r a 0.040 in q s 0.887
2

10-35-2

a 0.050 in q b 0.863

a r

Calculate the fatigue stress concentration factors for bending and torsion using equation 6.11b. Bending Torsion Kf 1 q b Kt 1 Kfs 1 q s Kts 1 Kf 1.746 Kfs 1.528

6.

Assume that the stresses are such that the conditions of equation 6.17 will result in Kfm Kf and Kfsm Kfs S'e 0.5 S ut S'e 407 MPa

7. 8.

Calculate the unmodified endurance limit (equation 6.5a).

Calculate the endurance limit modification factors for a rotating round beam. Load Cload 1 Combined bending and torsion

Size

Csize( d ) 1.189 A 4.51

mm
d

0.097

Surface

b 0.265
b

(machined)

Csurf Temperature Reliability 9.

Sut A MPa

Csurf 0.764

Ctemp 1 Creliab 1.0 (R = 50%)

Calculate the modified endurance limit. S e( d ) Cload Csize( d ) Csurf Ctemp Creliab S'e

10. Use equation (10.8) to solve for the diameter at the shoulder. Given
1 3

3 2 2 32 Nf Kf Ma 4 Kfs Ta d= S e( d )

Kfm Mm

S ut

3 4

Kfsm Tm

MACHINE DESIGN - An Integrated Approach, 4th Ed.


d Find ( d ) d 54.177 mm d 56 mm

10-35-3

Say

10. Calculate the minimum, maximum, mean, and alternating bending stresses to check the assumption regarding th mean stress concentration factors.

xmin

32 Pmin l

d xmax

xmin 23.2 MPa

32 Pmax l

d xm
32 Pm l

xmax 47.851 MPa

d xa

xm 35.526 MPa

32 Pa l

xa 12.325 MPa
S y 703 MPa

Yield strength SAE 4130 Q&T @ 1200F Evaluate Kfm S 1 Kf xmax S 2 Kf xmax xmin return Kf if S 1 S y return S y Kf xa

xm

if S 1 S y S 2 2 S y

0 otherwise Kfm 1.746 Kf 1.746

Which checks with the assumption made in step 6 above. 11. DESIGN SUMMARY Shaft diameter Shoulder dia Fillet radius d 56 mm D Doverd d r roverd d D 67.2 mm r 2.8 mm

MACHINE DESIGN - An Integrated Approach, 4th Ed.

10-36-1

PROBLEM 10-36
Statement: Determine the size of key necessary to give a safety factor of at least 2 against both shear and bearing failure for the crank/shaft connection of Problem 10-35. Assume a shaft diameter of 58 mm and a key made from SAE 1040 CR steel. Shaft diameter d 58 mm Shaft properties: S ut1 814 MPa S y1 703 MPa Key properties: S ut2 586 MPa S y2 490 MPa Maximum torque Tmax 2.2 kN m Minimum torque Tmin 1.1 kN m Design safety factor Nd 2

Given:

Assumptions: The finish is machined, reliability is 50%, and the shaft is at room temperature. Solution: 1. See Figure P10-8 and Mathcad file P1036.

As recommended in Table 10-2, for a shaft diameter of d 58 mm Key width w 16 mm

2.

Determine the alternating and mean key shear force components. Mean and alternating force components Fa Fm 1 Tmax Tmin 2 0.5 d 1 Tmax Tmin 2 0.5 d Fa 18.97 kN Fm 56.90 kN

3.

Write the equations for the mean and alternating components of the shear stress and the effective von Mises stress. Key shear area as Ashear = w L function of length Mean and alternating shear stresses Fa w L Fa w L Fm w L Fm w L

a =

m =

Mean and alternating von Mises stresses 4.

'a = 3 a = 3

'm = 3 m = 3

Using the modified Goodman failure criterion, the design equation is S e S ut S ut 'a S e 'm S e S ut

Nd =

3 S F S F ut a e m w L
3 Nd S ut Fa S e Fm w S ut S e

Solving for L

L=

5.

Determine the key material endurance limit. Uncorrected endurance strength S'e 0.5 S ut2 S'e 293 MPa

MACHINE DESIGN - An Integrated Approach, 4th Ed.

10-36-2

Correction factors: Load Size Cload 1 A95 = w L Csize = 1.189 d eq


0.097 0.5 0.097

d eq =

w L 0.0766

w Cs 1.189 0.0766 mm Csize = 0.805 L Surface (machined) Temperature Reliability Csurf


0.0485

Cs 0.918

S ut2 4.51 MPa

0.265

Csurf 0.833

Ctemp 1 Creliab 1.000


0.0485

(R = 50%)
0.0485

S e = Cload Csize Csurf Ctemp Creliab S'e = 1 Cs L where

Csurf 1 Crelaib S'e = Ce L

Ce Cs Csurf Creliab S'e

Ce 224 MPa

6.

Substitute S e into the design equation. 3 Nd S ut Fa Ce L w S ut Ce L Solving by iteration, let 3 Nd S ut2 Fa Ce RHS( L)
0.0485

L=

Fm

0.0485

in
0.0485

0.0485

Fm

w S ut2 Ce Guess L 25 mm L RHS( L) Tentatively, let 7.

in
RHS( L) 39.339 mm RHS( L) 39.747 mm

L 40 mm

Check the realised factor of safety against fatigue failure. S e Ce L


0.0485

Endurance limit

in

S e 219.1 MPa

MACHINE DESIGN - An Integrated Approach, 4th Ed.

10-36-3

Realised factor of safety

Nf

S e S ut2

3 S F S F ut2 a e m w L
1 2

Nf 2.0

8.

Check worst-case static bearing stress. Bearing area Maximum force Abear w L Abear 320 mm Fmax 75.9 kN
2

Fmax Fa Fm Fmax Abear

Maximum bearing stress on key Factor of safety against static bearing yield

max

max 237.07 MPa

Ns

S y2

max

Ns 2.1

9.

Design Summary Key width Key length w 16 mm L 40 mm (16 x 10)

MACHINE DESIGN - An Integrated Approach, 4th Ed.

10-37-1

PROBLEM 10-37
Statement: As an alternative to the keyed connection in Problem 10-35 determine the amount of diametral interference needed to provide a suitable interference fit for the crank of Figure P10-8 using a shaft diameter of 58 mm, such that the stresses in the hub and shaft will be safe and the maximum torque can be transmitted through the interference fit. The crank material is the same as the shaft and its length along the shaft is 64 mm. The effective outside diameter of the crank is 150 mm. Gear hub diameter Gear hub length Peak shaft torque Material properties are: d hub 150 mm L 64 mm Tp 2.2 kN m Shaft diameter: d shaft 58 mm Young's modulus E 209 GPa Poisson's ratio 0.28

Given:

S ut 814 MPa, and S y 703 MPa.

Assumptions: The coefficient of friction between the hub and shaft is 0.15. Solution: 1. 2. See Figure P10-8 and Mathcad file P1037. Nd 2 ro 75 mm r 29 mm

As a design choice, let the design factor of safety on torque capacity and hub failure be Calculate hub and shaft radii. Hub radius Nominal interface radius Shaft inside diameter ro 0.5 d hub r 0.5 d shaft ri 0 mm

3.

The minimum interference is determine by the desired torque capacity. Use equation 10.14c to solve for the minimum diametral interference.

Torque capacity

T=

L r min E ro r
2 2

Nd
2

2 ro 2 ro Nd Tp
2

Solving for min

min

L r E ro r
2

min 0.057 mm

Let the minimum diametral interference be 4.

min 0.06 mm

Find the von Mises stress in the hub as a function of the unknown maximum diametral interference .

Interference pressure Stress in shaft Tangential Radial Stress in hub Tangential

p ( )

E ro r
2 2

4 r ro

ti ( ) p ( ) ri( ) p ( )
ro r ro r
2 2 2 2

to( ) p ( )

Radial

ro( ) p ( )

These are principal stresses. The tangential stress is 1 and the radial is 3. Use equation (5.7c) to find the von Mises stress.

MACHINE DESIGN - An Integrated Approach, 4th Ed.

10-37-2

von Mises

'o( )

E
4 r ro
2

3 ro r
4

0.5

5. 6.

There is no bending stress in the shaft at the crank, therefore

Kt 1

The safety factors against failure (yielding in the shaft and hub) during press fit can now be used to find the maximum diametral interference: Guess Shaft

min
Given Nd = S y Kt ti ( )

smax Find ( )
Hub Given Nd = Sy Kt 'o( )

smax 0.23 mm

hmax Find( )
Using the smaller of the two, 7.

hmax 0.11 mm max hmax max 0.11 mm

The torque capacity of the joint (with an assumed coefficient of friction of 0.15 ) is about 2 times the peak transmitted torque and the safety factors against failure of the shaft and hub exceed 2 therefore, the minimum and maximum diametral interference below is acceptable. Minimum diametral interference Minimum diametral interference

min 0.06 mm max 0.12 mm

8.

If we divide the tolerance on the shaft and hub equally and use the basic hole system, the shaft and hub bore specifications are: Tolerance on shaft or hub Hub bore diameter: t 0.5 max min Dmin d shaft Dmax Dmin t Shaft diameter: d max d shaft max d min d max t Maximum interference Minimum interference t 0.03 mm Dmin 58.00 mm Dmax 58.03 mm d max 58.12 mm d min 58.09 mm

max d max Dmin min d min Dmax

max 0.12 mm min 0.06 mm

9.

Check the safety factors against torque capacity and static failure of the hub.

Torque

Ntorque Nhub

L r min E ro r
2 2

Tp Sy

2 ro

Ntorque 2.1 Nhub 1.9

Hub strength

Kt 'o max

MACHINE DESIGN - An Integrated Approach, 4th Ed.

10-38-1

PROBLEM 10-38
Statement: An electric motor-driven shaft that has a fluctuating load torque turns at an average speed of 1600 rpm. The load torque varies sinusoidally once per revolution of the shaft with a peak torque of 29500 in-lb. The shaft diameter is 2.0 in. Design a suitable flywheel for this system that will provide a coefficient of fluctuation of 0.05 and an overspeed safety factor of at least 4. The flywheel is to be a hollow circular disc of constant thickness made from SAE 1020 CR steel. Yield strength of SAE 1020 CR steel S y 57 ksi Average shaft speed avg 1600 rpm Shaft diameter Solution: 1. See Mathcad file P1038. d i 2.000 in

Given:

0.28
Coeff. of fluct. Peak torque

0.28 lbf in
Cf 0.05

Tpeak 29500 in lbf

Plot the load torque over one revolution of the shaft. Tl( ) Tpeak sin( )

0 0.1 2

4 10 Load Torque 2 10 T l( ) in lbf

4 4

0 2 10
4 4

4 10

60

120

180

240

300

360

deg Shaft Position

The average load torque is The minimum shaft speed occurs at The maximum shaft speed occurs at 2.

Tavg 0 in lbf

min 180 deg max 360 deg

Calculate the required change in kinetic energy by integrating the left-hand side of equation 10.18d.
max Ek Tl( ) d min

Energy variation 3.

Ek 59000 in lbf

Calculate the required system moment of inertia using equation 10.22. System moment of inertia Let Is Im Is Ek Cf avg
2

Is 42.032 lbf in sec

4.

As a design choice, let the disc thickness be t 0.4 d i , t 0.800 in (this value of t was repeatedly changed to arrive at a value of 4.0 for Nos). Calculate the outside radius required for this thickness using equation 10.17d.

MACHINE DESIGN - An Integrated Approach, 4th Ed.

10-38-2

Outside radius

2 Im g d i 4 ro t 2

0.25

ro 14.655 in

5.

Let the maximum tangential stress (at r = ri) be equal to the yield stress and solve equation 10.23a for the yield speed.

Yield speed

yield

2 2 3 di di 2 1 3 2 ro 3 2 8 2
S y g

0.5

yield 6376 rpm


Factor of safety against overspeeding Nos

yield avg

Nos 4.0

6.

DESIGN SUMMARY Inside radius Outside radius Tickness ri 0.5 d i ro 14.655 in t 0.800 in ri 1 in

MACHINE DESIGN - An Integrated Approach, 4th Ed.

10-39-1

PROBLEM 10-39
Statement: Repeat Problem 10-38 for a peak load torque of 40500 in-lb and a torque vs shaft position function of Tl = Tpeak (sin + sin 2). Yield strength of SAE 1020 CR steel S y 57 ksi Average shaft speed avg 1600 rpm Shaft diameter Solution: 1. See Mathcad file P1039. d i 2.000 in

Given:

0.28
Coeff. of fluct. Peak torque

0.28 lbf in
Cf 0.05

Tpeak 40500 in lbf

Plot the load torque over one revolution of the shaft. Tl( ) Tpeak ( sin( ) sin( 2 ) )
1 10 Load Torque 5 10 T l( ) in lbf
5 4

0 0.1 2

0 5 10
4 5

1 10

60

120

180

240

300

360

deg Shaft Position

The average load torque is The minimum shaft speed occurs at The maximum shaft speed occurs at 2.

Tavg 0 in lbf

min 120 deg max 360 deg

(and again at 240 deg)

Calculate the required change in kinetic energy by integrating the left-hand side of equation 10.18d.
max Ek Tl( ) d min

Energy variation

Ek 91125 in lbf

3.

Calculate the required system moment of inertia using equation 10.22. System moment of inertia Let Is Im Is Ek Cf avg
2

Is 64.919 lbf in sec

4.

As a design choice, let the disc thickness be t 0.6 d i , t 1.200 in (this value of t was repeatedly changed to arrive at a value of 4.0 for Nos). Calculate the outside radius required for this thickness using equation 10.17d.
0.25

Outside radius

2 Im g d i 4 ro t 2

ro 14.762 in

MACHINE DESIGN - An Integrated Approach, 4th Ed.


5.

10-39-2

Let the maximum tangential stress (at r = ri) be equal to the yield stress and solve equation 10.23a for the yield speed.

Yield speed

yield 3 8

2 di 2 di 2 1 3 2 2 ro 3 2
S y g

0.5

yield 6330 rpm


Factor of safety against overspeeding Nos

yield avg

Nos 4.0

6.

DESIGN SUMMARY Inside radius Outside radius Tickness ri 0.5 d i ro 14.762 in t 1.200 in ri 1.000 in

MACHINE DESIGN - An Integrated Approach, 4th Ed.

10-40a-1

PROBLEM 10-40a
Statement: As an alternative to the keyed connection in Problem 10-6 determine the amount of diametral interference needed to provide a suitable interference fit for the gear of Figure P10-3 using the data of Problem 10-6 and row a in Table P10-1. The gear hub diameter is 3.50 in, its length along the shaft is 2.00 in, and it has the same properties as the shaft material. Shaft diameter: d shaft 1.75 in Gear hub diameter d hub 3.50 in Gear hub length Peak shaft torque Material properties are: L 2.00 in Tp 2000 lbf in Young's modulus E 30 10 psi Poisson's ratio 0.28
6

Given:

S ut 108 ksi, and S y 62 ksi.

Assumptions: The coefficient of friction between the hub and shaft is 0.15. Solution: 1. 2. See Figure P10-3 and Mathcad file P1040a. Nd 2 ro 1.75 in r 0.875 in

As a design choice, let the design factor of safety on torque capacity and hub failure be Calculate hub and shaft radii. Hub radius Nominal interface radius Shaft inside diameter ro 0.5 d hub r 0.5 d shaft ri 0 in

3.

The minimum interference is determine by the desired torque capacity. Use equation 10.14c to solve for the minimum diametral interference.

Torque capacity

T=

L r min E ro r
2 2

Nd
2

2 ro 2 ro Nd Tp
2

Solving for min

min

L r E ro r
2

min 0.00043 in

Let the minimum diametral interference be

min 0.0005 in

Note that it is not practical to specify a dimension in inches that has more than 4 decimal places. 4. Find the von Mises stress in the hub as a function of the unknown maximum diametral interference .

Interference pressure Stress in shaft Tangential Radial Stress in hub Tangential

p ( )

E ro r
2 2

4 r ro

ti ( ) p ( ) ri( ) p ( )

to( ) p ( )

ro r ro r
2

2 2

Radial

ro( ) p ( )

These are principal stresses. The tangential stress is 1 and the radial is 3. Use equation (5.7c) to find the von Mises stress.

MACHINE DESIGN - An Integrated Approach, 4th Ed.


0.5

10-40a-2

von Mises

'o( )

E
4 r ro
2

3 ro r
4

5. 6.

There is no bending stress in the shaft at the gear, therefore

Kt 1

The safety factors against failure (yielding in the shaft and hub) during press fit can now be used to find the maximum diametral interference: Guess Shaft

min
Given Nd = S y Kt ti ( )

smax Find ( )
Hub Given Nd = Sy Kt 'o( )

smax 0.0048 in

hmax Find( )
Using the smaller of the two, 7.

hmax 0.0021 in max hmax max 0.0021 in

The torque capacity of the joint (with an assumed coefficient of friction of 0.15 ) is about 2 times the peak transmitted torque and the safety factors against failure of the shaft and hub exceed 2 therefore, the minimum and maximum diametral interference below is acceptable. Minimum diametral interference Minimum diametral interference

min 0.0005 in max 0.0021 in

8.

If we divide the tolerance on the shaft and hub equally and use the basic hole system, the shaft and hub bore specifications are: Tolerance on shaft or hub Hub bore diameter: t 0.5 max min Dmin d shaft Dmax Dmin t Shaft diameter: d max d shaft max d min d max t Maximum interference Minimum interference t 0.0008 in Dmin 1.7500 in Dmax 1.7508 in d max 1.7521 in d min 1.7513 in

max d max Dmin min d min Dmax

max 0.0021 in min 0.0005 in

9.

Check the safety factors against torque capacity and static failure of the hub. Torque Ntorque Nhub

L r min E ro r
2 2

Tp Sy

2 ro

Ntorque 2.3 Nhub 2.0

Hub strength

Kt 'o max

MACHINE DESIGN - An Integrated Approach, 4th Ed.

10-41-1

PROBLEM 10-41
Statement: Figure P10-6 shows a shaft, running at 400 rpm, with two attached gears. The right-hand gear (2) furnishes the load torque and the left-hand gear (1) furnishes an equal but opposite direction input torque. The torque varies between 2.2 kN-m and 6.2 kN-m. Determine the minimum, maximum, and average power transmitted by the shaft. Shaft speed Minimum torque Maximum torque Solution: 1.

Given:

400 rpm
Tmin 2.2 kN m Tmax 6.2 kN m

41.888

rad sec

See Figure P10-6 and Mathcad file P1041.

Use equations (10.1) to calculate the power values. Tmax Tmin 2

Average torque Maximum power Minimum power Average power 2.

Tavg

Tavg 4 kN m Pmax 259.7 kW Pmin 92 kW Pavg 175.9 kW

Pmax Tmax Pmin Tmin Pavg Tavg

Note, if doing these calculations by hand, that suitable unit conversion factors must be used.

MACHINE DESIGN - An Integrated Approach, 4th Ed.

10-42-1

PROBLEM 10-42
Statement: Figure P10-6 shows a shaft, running at 750 rpm, with two attached gears. The right-hand gear (2) furnishes the load torque and the left-hand gear (1) furnishes an equal but opposite direction input torque. The forces on a gear act at the pitch diameter and have a radial (P in the figure) and a tangential component (not shown). If the pitch diameter of gear 1 is 250 mm and the tangential force varies between 15 kN and 60 kN, determine the minimum, maximum, and average power transmitted by the shaft.

Given:

Shaft speed Pitch diameter Minimum tangential force Maximum tangential force

750 rpm
d 250 mm Fmin 15 kN

78.54

rad sec

Fmax 60 kN

Solution: 1.

See Figure P10-6 and Mathcad file P1042.

Determine the minimum and maximum torque. Minimum torque Maximum torque Tmin 0.5 d Fmin Tmax 0.5 d Fmax Tmin 1.875 kN m Tmax 7.5 kN m

1.

Use equations (10.1) to calculate the power values. Tmax Tmin 2

Average torque Maximum power Minimum power Average power 2.

Tavg

Tavg 5 kN m Pmax 589.0 kW Pmin 147 kW Pavg 368.2 kW

Pmax Tmax Pmin Tmin Pavg Tavg

Note, if doing these calculations by hand, that suitable unit conversion factors must be used.

MACHINE DESIGN - An Integrated Approach, 4th Ed.

10-43-1

PROBLEM 10-43
Statement: Determine the amount of diametral interference needed to provide a suitable interference fit (instead of the keyed attachment shown) for the 5-in diameter by 1.5-in thick gear (1) of Figure P10-6 using a shaft diameter of 1.5 in, such that the stresses in the hub and shaft will be safe and the input torque of 1500 lbf-in can be transmitted through the interference fit. Both parts are SAE 4130 steel normalized @ 1650F. Gear hub diameter Gear hub length Peak shaft torque Material properties are: d hub 5.00 in L 1.50 in Tp 1500 lbf in S ut 97 ksi Shaft diameter: d shaft 1.50 in
6

Given:

Young's modulus E 30 10 psi Poisson's ratio 0.28 S y 63 ksi

Assumptions: The coefficient of friction between the hub and shaft is 0.15. Solution: 1. 2. See Figure P10-6 and Mathcad file P1043. Nd 2 ro 2.5 in r 0.75 in

As a design choice, let the design factor of safety on torque capacity and hub failure be Calculate hub and shaft radii. Hub radius Nominal interface radius Shaft inside diameter ro 0.5 d hub r 0.5 d shaft ri 0 in

3.

The minimum interference is determine by the desired torque capacity. Use equation 10.14c to solve for the minimum diametral interference.

Torque capacity

T=

L r min E ro r
2 2

Nd
2

2 ro 2 ro Nd Tp

Solving for min

min

2 2 L r E ro r

min 0.00041 in

Let the minimum diametral interference be

min 0.0004 in

Note that it is not practical to specify a dimension in inches that has more than 4 decimal places. 4. Find the von Mises stress in the hub as a function of the unknown maximum diametral interference .

Interference pressure Stress in shaft Tangential Radial Stress in hub Tangential Radial

p ( )

E ro r
2 2

4 r ro

ti ( ) p ( ) ri( ) p ( )
ro r ro r
2 2 2 2

to( ) p ( )

ro( ) p ( )

MACHINE DESIGN - An Integrated Approach, 4th Ed.

10-43-2

These are principal stresses. The tangential stress is 1 and the radial is 3. Use equation (5.7c) to find the von Mises stress. von Mises

'o( )

E
4 r ro
2

3 ro r
4

0.5

5. 6.

There is very little bending stress in the shaft at the gear, therefore Kt 1 The safety factors against failure (yielding in the shaft and hub) during press fit can now be used to find the maximum diametral interference: Guess Shaft

min
Given Nd = S y Kt ti ( )

smax Find ( )
Hub Given Nd = Sy Kt 'o( )

smax 0.0035 in

hmax Find( )
Using the smaller of the two, 7.

hmax 0.0018 in max hmax max 0.0018 in

The torque capacity of the joint (with an assumed coefficient of friction of 0.15 ) is about 2 times the peak transmitted torque and the safety factors against failure of the shaft and hub exceed 2 therefore, the minimum and maximum diametral interference below is acceptable. Minimum diametral interference Maximum diametral interference

min 0.0004 in max 0.0018 in

8.

If we divide the tolerance on the shaft and hub equally and use the basic hole system, the shaft and hub bore specifications are: Tolerance on shaft or hub Hub bore diameter: t 0.5 max min Dmin d shaft Dmax Dmin t Shaft diameter: d max d shaft max d min d max t Maximum interference Minimum interference t 0.0007 in Dmin 1.5000 in Dmax 1.5007 in d max 1.5018 in d min 1.5011 in

max d max Dmin min d min Dmax

max 0.0018 in min 0.0004 in

9.

Check the safety factors against torque capacity and static failure of the hub. Torque Ntorque Nhub

L r min E ro r
2 2

Tp Sy

2 ro

Ntorque 1.9 Nhub 2.0

Hub strength

Kt 'o max

MACHINE DESIGN - An Integrated Approach, 4th Ed.

10-44-1

PROBLEM 10-44
Statement: Determine the amount of diametral interference needed to provide a suitable interference fit (instead of the keyed attachment shown) for the 125-mm diameter by 75-mm thick gear (1) of Figure P10-6 using a shaft diameter of 80 mm, such that the stresses in the hub and shaft will be safe and the input torque of 170 N-m can be transmitted through the interference fit. Both parts are SAE 4140 steel normalized @ 1650F. Gear hub diameter Gear hub length Peak shaft torque Material properties are: d hub 125 mm L 75 mm Tp 170 N m S ut 1020 MPa Shaft diameter: d shaft 80 mm

Given:

Young's modulus E 209 GPa Poisson's ratio 0.28 S y 655 MPa

Assumptions: The coefficient of friction between the hub and shaft is 0.15. Solution: 1. 2. See Figure P10-6 and Mathcad file P1044. Nd 2 ro 62.5 mm r 40 mm

As a design choice, let the design factor of safety on torque capacity and hub failure be Calculate hub and shaft radii. Hub radius Nominal interface radius Shaft inside diameter ro 0.5 d hub r 0.5 d shaft ri 0 mm

3.

The minimum interference is determine by the desired torque capacity. Use equation 10.14c to solve for the minimum diametral interference.

Torque capacity

T=

L r min E ro r
2 2

Nd
2

2 ro 2 ro Nd Tp
2

Solving for min

min

L r E ro r
2

min 0.0039 mm

Let the minimum diametral interference be

min 0.004 mm

Note that it is not practical to specify a dimension in milimeters that has more than 3 decimal places. 4. Find the von Mises stress in the hub as a function of the unknown maximum diametral interference .

Interference pressure Stress in shaft Tangential Radial Stress in hub Tangential Radial

p ( )

E ro r
2 2

4 r ro

ti ( ) p ( ) ri( ) p ( )
ro r ro r
2 2 2 2

to( ) p ( )

ro( ) p ( )

MACHINE DESIGN - An Integrated Approach, 4th Ed.

10-44-2

These are principal stresses. The tangential stress is 1 and the radial is 3. Use equation (5.7c) to find the von Mises stress. von Mises

'o( )

E
4 r ro
2

3 ro r
4

0.5

5. 6.

There is very little bending stress in the shaft at the gear, therefore Kt 1 The safety factors against failure (yielding in the shaft and hub) during press fit can now be used to find the maximum diametral interference: Guess Shaft

min
Given Nd = S y Kt ti ( )

smax Find ( )
Hub Given Nd = Sy Kt 'o( )

smax 0.425 mm

hmax Find( )
Using the smaller of the two, 7.

hmax 0.141 mm max hmax max 0.140 mm

The torque capacity of the joint (with an assumed coefficient of friction of 0.15 ) is about 2 times the peak transmitted torque and the safety factors against failure of the shaft and hub exceed 2 therefore, the minimum and maximum diametral interference below is acceptable. Minimum diametral interference Maximum diametral interference

min 0.004 mm max 0.140 mm

8.

If we divide the tolerance on the shaft and hub equally and use the basic hole system, the shaft and hub bore specifications are: Tolerance on shaft or hub Hub bore diameter: t 0.5 max min Dmin d shaft Dmax Dmin t Shaft diameter: d max d shaft max d min d max t Maximum interference Minimum interference t 0.0680 mm Dmin 80.000 mm Dmax 80.068 mm d max 80.140 mm d min 80.072 mm

max d max Dmin min d min Dmax

max 0.140 mm min 0.004 mm

9.

Check the safety factors against torque capacity and static failure of the hub. Torque Ntorque Nhub

L r min E ro r
2 2

Tp Sy

2 ro

Ntorque 2.1 Nhub 2.0

Hub strength

Kt 'o max

MACHINE DESIGN - An Integrated Approach, 4th Ed.

10-45-1

PROBLEM 10-45
Statement: Table P10-2 shows the energy pulses delivered to (positive) and from (negative) a rotating system along with the shaft angles at which the torque function crosses the average torque line in a torque-time function. Using this data, determine the shaft angles at which minimum and maximum shaft speed occur and the total change in energy from the shaft position at which maximum speed occurs to the position at which minimum speed occurs. Range E

Given:

0 0 deg to 1 75 deg 1 75 deg to 2 195 deg 2 195 deg to 3 330 deg 3 330 deg to 4 360 deg
Solution: 1.

E1 1040 N m E2 2260 N m E3 2950 N m E4 1740 N m

See Table P10-2 and Mathcad file P1045.

The minimum shaft speed occurs at the shaft angle at the end of the largest positive energy pulse. In this case, it is at 2 195 deg. The maximum shaft speed occurs at the shaft angle at the end of the largest negative energy pulse. In this case, it is at 3 330 deg. The total energy change is the algebraic difference between the accumulated energy between the maximum and minimum shaft speeds. E1 E1 E2 E1 E2 E3 E2 E3 E4 E3 E4 E1 1040 N m E2 1220 N m E3 1730 N m E4 10 N m

2.

3.

Energy E3 E2

Energy 2950 N m

MACHINE DESIGN - An Integrated Approach, 4th Ed.

10-46-1

PROBLEM 10-46
Statement: An electric motor-driven shaft that has a fluctuating load torque turns at an average speed of 1950 rpm. The energy pulses to (positive) and from (negative) the driven system are given in Table P10-2. The shaft diameter is 50 mm. Design a suitable flywheel for this system that will provide a coefficient of fluctuation of 0.05 and an overspeed safety factor of at least 5. The flywheel is to be a hollow circular disc of constant thickness made from SAE 1040 CR steel. Yield strength of SAE 1040 CR steel S y 490 MPa Coeff. of fluct. Average shaft speed avg 1950 rpm Shaft diameter Solution: 1. d i 50 mm

Given:

7800 kg m
Cf 0.05

0.28

See Table P10-2 and Mathcad file P1046.

Calculate the change in kinetic energy from the data in Table P10-2. The minimum shaft speed occurs at The maximum shaft speed occurs at

min 195 deg max 330 deg

The change in kinetic energy between the maximum and minimum speeds is Ek 2950 N m 2. Calculate the required system moment of inertia using equation 10.22. System moment of inertia Let 3. Is Im Is Ek Cf avg
2

Is 1.415 kg m

As a design choice, let the disc thickness be t 0.4 d i , t 20.0 mm (this value of t was repeatedly changed to arrive at a value of 5.0 for Nos). Calculate the outside radius required for this thickness using equation 10.17d.

Outside radius

2 Im d i 4 ro t 2

0.25

ro 276 mm

4.

Let the maximum tangential stress (at r = ri) be equal to the yield stress and solve equation 10.23a for the yield speed.
0.5 Sy yield 2 3 di 2 di 2 1 3 2 2 ro 3 2 8

Yield speed

yield 9580 rpm


Factor of safety against overspeeding 5. DESIGN SUMMARY Inside radius Outside radius ri 0.5 d i ro 276 mm ri 25 mm Tickness t 20 mm Nos

yield avg

Nos 5

MACHINE DESIGN - An Integrated Approach, 4th Ed.

12-1-1

PROBLEM 12-1
Statement: A 20-deg pressure angle, 27-tooth spur gear has a diametral pitch of 5. Find the pitch diameter, addendum, dedendum, outside diameter, and circular pitch. Diametral pitch Pressure angle Solution: See Mathcad file P1201. Pitch diameter d N pd 1.0 pd 1.25 pd d 5.400 in p d 5 in
1

Given:

Number of teeth N 27

20 deg

Addendum

a b

a 0.200 in b 0.250 in Do 5.800 in p c 0.628 in

Dedendum

Outside diameter

Do d 2 a p c

Circular pitch

pd

MACHINE DESIGN - An Integrated Approach, 4th Ed.

12-2-1

PROBLEM 12-2
Statement: A 25-deg pressure angle, 43-tooth spur gear has a diametral pitch of 8. Find the pitch diameter, addendum, dedendum, outside diameter, and circular pitch. Diametral pitch Pressure angle Solution: See Mathcad file P1202. Pitch diameter d N pd 1.0 pd 1.25 pd d 5.375 in p d 8 in
1

Given:

Number of teeth N 43

25 deg

Addendum

a b

a 0.125 in b 0.156 in Do 5.625 in p c 0.393 in

Dedendum

Outside diameter

Do d 2 a p c

Circular pitch

pd

MACHINE DESIGN - An Integrated Approach, 4th Ed.

12-3-1

PROBLEM 12-3
Statement: A 57-tooth spur gear is in mesh with a 23-tooth pinion. For the diametral pitch and pressure angle given below, find the contact ratio. Tooth numbers: pinion gear Solution: Np 23 Ng 57 Diametral pitch Pressure angle p d 6 in
1

Given:

25 deg

See Mathcad file P1203. Circular pitch p c

pd

p c 0.524 in p b 0.475 in

Base pitch Pinion: Pitch dia Pitch rad Gear: Pitch dia Pitch rad

p b p c cos( )

d p

Np pd

d p 3.833 in rp 1.917 in

rp 0.5 d p

d g

Ng pd

d g 9.500 in rg 4.750 in a 0.167 in

rg 0.5 d g a 1.0 pd Np Ng 2 pd

Addendum

Center distance

C 6.667 in

Length of action

rp a 2 rp cos( ) 2 rg a 2 rg cos( ) 2 C sin( )

Z 0.708 in Contact ratio mp Z pb mp 1.491

MACHINE DESIGN - An Integrated Approach, 4th Ed.

12-4-1

PROBLEM 12-4
Statement: A 78-tooth spur gear is in mesh with a 27-tooth pinion. For the diametral pitch and pressure angle given below, find the contact ratio. Tooth numbers: pinion gear Solution: Np 27 Ng 78 Diametral pitch Pressure angle p d 6 in
1

Given:

20 deg

See Mathcad file P1203. Circular pitch p c

pd

p c 0.524 in p b 0.492 in

Base pitch Pinion: Pitch dia Pitch rad Gear: Pitch dia Pitch rad

p b p c cos( )

d p

Np pd

d p 4.500 in rp 2.250 in

rp 0.5 d p

d g

Ng pd

d g 13.000 in rg 6.500 in a 0.167 in

rg 0.5 d g a 1.0 pd Np Ng 2 pd

Addendum

Center distance

C 8.750 in

Length of action

rp a 2 rp cos( ) 2 rg a 2 rg cos( ) 2 C sin( )

Z 0.849 in Contact ratio mp Z pb mp 1.726

MACHINE DESIGN - An

12-5-1

PROBLEM 12-5
Statement: Given: What will the pressure angle be if the center distance of the spur gearset in Problem 12-3 is increased by 5%? Tooth numbers: pinion Np 23 gear Diametral pitch Pressure angle Solution: 1. Ng 57 p d 6 in
1

new

Base Circle Old Pitch Circle New Pitch Circle rbp C + C

25 deg

fc rp

See Figure 12-5 and Mathcad file P1205.

fc r g

rbg

Define the factor by which the center distance changes. Let C C C = fc

Dividing numerator and denominator of the left side by C, fc = 1 2. 3.

C
C fc 1.05

FIGURE 12-5
Diagram Showing Center Change for Problem 12-5

In this case, fc 1 0.05

Determine the pitch diameter and pitch radius of the pinion. Pitch diameter Pitch radius d p Np pd d p 3.833 in rp 1.917 in

rp 0.5 d p

4.

When the centers are moved apart the base circle diameters don't change but new pitch circle diameters are defined by the intersection of the new line of action, which is always tangent to the two base circles, with the line of centers. The new pitch radii are proportional to fc. Thus, from Figure 12-5, the new pressure angle is cos new = rbp fc rp

Substituting rp cos( ) for rbp and solving for new,

new acos

rp cos( ) fc rp

new 30.33 deg

MACHINE DESIGN - An

12-6-1

PROBLEM 12-6
Statement: Given: What will the pressure angle be if the center distance of the spur gearset in Problem 12-4 is increased by 7%? Tooth numbers: pinion Np 27 gear Diametral pitch Pressure angle Solution: 1. Ng 78 p d 6 in
1

new

Base Circle Old Pitch Circle New Pitch Circle rbp C + C

20 deg

fc rp

See Figure 12-6 and Mathcad file P1206.

fc r g

rbg

Define the factor by which the center distance changes. Let C C C = fc

Dividing numerator and denominator of the left side by C, fc = 1 2. 3.

C
C fc 1.07

FIGURE 12-6
Diagram Showing Center Change for Problem 12-6

In this case, fc 1 0.07

Determine the pitch diameter and pitch radius of the pinion. Pitch diameter Pitch radius d p Np pd d p 4.500 in rp 2.250 in

rp 0.5 d p

4.

When the centers are moved apart the base circle diameters don't change but new pitch circle diameters are defined by the intersection of the new line of action, which is always tangent to the two base circles, with the line of centers. The new pitch radii are proportional to fc. Thus, from Figure 12-5, the new pressure angle is cos new = rbp fc rp

Substituting rp cos( ) for rbp and solving for new,

new acos

rp cos( ) fc rp

new 28.57 deg

MACHINE DESIGN - An Integrated Approach, 4th Ed.

12-7-1

PROBLEM 12-7
Statement: Given: If the spur gearsets in Problems 12-3 and 12-4 are compounded as shown in Figure 12-14, what wil the overall train ratio be? Tooth numbers N2 57 N4 78 Solution: See Mathcad file P1207. N3 23 N5 27

1. Using equation 12.9b and assuming that gears 2 and 4 are the driver gears, the train (velocity) ratio is mv N2 N4 N3 N5 mv 7.159

2. On the other hand, if gears 3 and 5 are the driver gears then the train ratio is mv N3 N5 N2 N4 mv 0.1397

MACHINE DESIGN - An Integrated Approach, 4th Ed.

12-8-1

PROBLEM 12-8
Statement: A paper mill processes rolls of paper having a density of 984 kg/m3. The paper roll is 1.50-m outside diameter (OD) x 0.22-m inside diameter (ID) x 3.23-m long and is on a simple supported, hollow, steel shaft with S ut = 400 MPa. Design a 2.5:1 reduction spur gearset to drive this roll shaft to obtain a minimum dynamic safety factor of 2 for 10-year life if the roll turns at 50 rpm with 1.2 hp absorbed. yr 2080 hr Bending factor of safety Surface factor of safety Power to be transmitted Nfb 2 Nfs 2 H 1.2 hp Life (years) Gear speed Gear ratio Life 10 yr n g 50 rpm mg 2.5

Units: Given:

Assumptions: 1. If both pinion and gear are the same material, it will only be necessary to determine the pinion size as it will be governing for the set. 2. If the gears are not surface hardened, it will only be necessary to design to the surface requirement as it will be governing for both bending and surface stresses. Solution: See Mathcad file P1208.

Design Choices: Pressure angle Number of pinion teeth

25 deg
Np 26

AGMA Quality level Reliability

Qv 7 R 0.99

Material: AGMA Grade 2 steel for both pinion and gear, through hardened to HB 350 S ac ( 27000 364 HB) psi S ac 154400 psi 1. Determine the number of gear teeth, the pinion speed, and the cycle life. Number of gear teeth Rotational speed of pinion (rpm) Cycle life: 2. Determine the surface geometry factor, I. I 3. Ng Np mg n p n g mg N Life np 2 Ng 65 n p 125 rpm N 1.56 10
8

sin( ) cos( ) Ng N N 2 p g

I 0.1368

Write the equations for pitch diameter, pitch-line velocity, and transmitted load in terms of the unknown diametral pitch, pd. Note that, in Mathcad, unit conversion factors are not included. Pitch diameter of pinion (in) d p p d Vt p d Wt p d Ca 1 B 0.25 12 Qv
0.6667

Np pd d p p d n p 2 Vt p d H

d g p d

Ng pd

Pitch-line velocity (fpm) Transmitted load (lbf) 4. 5. Set the application factor, Ca Write the equation for the dynamic load factor, C

B 0.731 A 65.062

A 50 56 ( 1 B)

MACHINE DESIGN - An Integrated Approach, 4th Ed.

12-8-2

Cv p d

A min A Vt p d ft Cm 1.6

6. 7.

Tentatively choose the mounting factor, Cm (Assume 0 < F < 2 in Choose an elastic coefficient from Table 12-18 (steel on steel).

Cp 2300 psi

0.5

8. 9.

The surface stress equation for the pinion is Determine the endurance strength of the pinion. Life factor Reliability Temperature factor Material surface strength (psi) Grade 2, 350 HB Endurance strength

cp p d F Cp

Cv p d F d p p d I

C a W t p d C m

CL 2.466 N

0.056

CL 0.857 CR 1 CT 1 S ac 154400 psi

CR 0.7 0.15 log( 1 R)

S fcp

S ac CL CT CR

S fcp 132381 psi

10. Write the design equations using the range of face-width to diametral-pitch ratio given in the text, and the bending stress equation, solved for the unknown face width. Limits and safety factor FU p d 16 pd C a W t p d C m FL p d
2

8 pd

Sfc Nfc = c

Face width

Cp F p d Nfs Cv p d d p p d I S fcp
p d 6 6.5 16 in in in

11. Plot F(P) vs p over the range


d

3 F pd in FL pd in FU pd 1 in

12. From the graph, choose a standard value of p d from Table 12-2. p d 10 in
1

13. The calculated value of F is F p d 1.393 in 14. Round this up to the decimal equivalent of a common fractional value. F 1.375 in 15. Then, the parameters that depend on p d and F are: d p p d 2.600 in d g p d 6.500 in

10

12 pd in

14

16

FIGURE 12-8
Graph of Face Width and Limits for the Pinion (surface) in Problem 12-8

MACHINE DESIGN - An Integrated Approach, 4th Ed.

12-8-3

Vt p d 85

ft min

Wt p d 465 lbf

cp p d F 94207 psi

16. There is a wide range of choice for the number of teeth on the pinion. The total weight of the gears goes down very slightly with increasing pinion tooth number. The choice of Np 26 was somewhat arbitrary but was arrived at after trying values from 18 to 26.

MACHINE DESIGN - An Integrated Approach, 4th Ed.

12-9-1

PROBLEM 12-9
Statement: Given: Design a two-stage compound spur gear train for an overall ratio of approximately 47:1. Specify tooth numbers for each gear in the train. Approximate ratio mG 47 Number of stages n 2

Assumptions: The gears will be cut with a hob and the pressure angle will be 25 deg Solution: See Mathcad file P1209.
1

1. 2.

For equal stages, the stage ratio is

mGs mG

mGs 6.856

The minimum number of teeth that we can have without interference on a 25-deg gear cut with a hob is 14. Try pinions with 14, 15, 16, etc. teeth to see if the mating gear will have close to an integral number of teeth. 14 mGs 95.979 15 mGs 102.835 16 mGs 109.69

3. 4.

The first trial is very close to an integer so try 96 teeth:

mG

96 14

mG 47.02

We can get slightly closer by using two 14-tooth pinions with 95 and 97-tooth gears mG 95 97 14 14 mG 47.015

5.

However, it may be harder to find the 97-tooth gear and, it may be less expensive to have two gears with the same number of teeth than to have different numbers of teeth.

MACHINE DESIGN - An Integrated Approach, 4th Ed.

12-10-1

PROBLEM 12-10
Statement: Given: Solution: Design a three-stage compound spur gear train for an overall ratio of approximately 656:1. Specify tooth numbers for each gear in the train. Gear ratio mG 656 Number of stages s 3

See Mathcad file P1210.


1

1. 2.

The average stage ratio is

ravg mG

ravg 8.689 Nmin 18 NG1 156 NG2 157

If the minimum number of teeth is (20 deg pa) NG1 floor ravg Nmin or NG2 ceil ravg Nmin

then the number of teeth on the driven gear for each stage is

3. But, the prime factors of 656 are 2 4 and 41. This suggests that one of the gears have a number of teeth that is a multiple of 41, say 164 164 ravg 18.875 Let N2 18 N2 N3 N4 18 N3 164

Then, the remaing 2-stage ratio is N5 N7 N4 N6

mG2 mG

mG2 72

Thus,

= 72
5 6

if

and

N6 18

then Try

N5 N7 = 72 18 18 = 2 3 N5 2 3
4 2

N7 2 3

1 4

then

N5 N7 23328

Summarizing,

N2 18 N4 18 N6 18

N3 164 N5 144 N7 162

4. 5.

Since the driven gears all have less than 180 teeth, no stage ratio is greater than 10. Checking the overall gear ratio mG N 3 N 5 N 7 N 2 N 4 N 6 mG 656.00

MACHINE DESIGN - An Integrated Approach, 4th Ed.

12-11-1

PROBLEM 12-11
Statement: An epicycle spur gear train as shown in Figure 12-16 has a sun gear of 33 teeth and a planet gear of 21 teeth. Find the required number of teeth in the ring gear and determine the ratio between the arm and the sun gear if the ring gear is held stationary. Sun tooth number Planet tooth number Solution: 1. See Mathcad file P1211. Ns 33 Np 21 Arm speed Ring speed

Given:

a 1 rpm r 0 rpm

From Figure 12-16 we see that the diameter of the ring gear is equal to the sum of the diameter of the sun gear and two times the diameter of the planet gear. Since sun, planets, and ring gear must have the same diametral pitch, Number of teeth on ring: Nr Ns 2 Np Nr 75

2.

Choose the sun gear as the first gear and the ring gear as the last. Then, write equation (12.11c).

ra sa

r a s a

Ns Nr Nr a Ns a Nr r Ns

Solving for the sun speed,

s 3.273 rpm

3.

The ratio between the arm and sun gear is

a s

0.306

MACHINE DESIGN - An Integrated Approach, 4th Ed.

12-12-1

PROBLEM 12-12
Statement: An epicycle spur gear train as shown in Figure 12-16 has a sun gear of 23 teeth and a planet gear of 31 teeth. Find the required number of teeth in the ring gear and determine the ratio between the arm and the ring gear if the sun gear is held stationary. Sun tooth number Planet tooth number Solution: 1. See Mathcad file P1212. Ns 23 Np 31 Arm speed Sun speed

Given:

a 1 rpm s 0 rpm

From Figure 12-16 we see that the diameter of the ring gear is equal to the sum of the diameter of the sun gear and two times the diameter of the planet gear. Since sun, planets, and ring gear must have the same diametral pitch, Number of teeth on ring: Nr Ns 2 Np Nr 85

2.

Choose the sun gear as the first gear and the ring gear as the last. Then, write equation (12.11c).

ra sa

r a s a

Ns Nr Nr a Ns a Ns s Nr

Solving for the ring speed,

r 1.271 rpm

3.

The ratio between the arm and ring gear is

a r

0.787

MACHINE DESIGN - An Integrated Approach, 4th Ed.

12-13-1

PROBLEM 12-13
Statement: An epicycle spur gear train as shown in Figure 12-16 has a sun gear of 23 teeth and a planet gear of 31 teeth. Find the required number of teeth in the ring gear and determine the ratio between the sun and the ring gear if the arm is held stationary. Sun tooth number Planet tooth number Solution: 1. See Mathcad file P1213. Ns 23 Np 31 Arm speed Sun speed

Given:

a 0 rpm s 1 rpm

From Figure 12-16 we see that the diameter of the ring gear is equal to the sum of the diameter of the sun gear and two times the diameter of the planet gear. Since sun, planets, and ring gear must have the same diametral pitch, Number of teeth on ring: Nr Ns 2 Np Nr 85

2.

Choose the sun gear as the first gear and the ring gear as the last. Then, write equation (12.11c).

ra sa

r a s a

Ns Nr Nr a Ns a Ns s Nr

Solving for the ring speed,

r 0.271 rpm

3.

The ratio between the sun and ring gear is

s r

3.696

MACHINE DESIGN - An Integrated Approach, 4th Ed.

12-14-1

PROBLEM 12-14
Statement: Given: If the gearset in Problem 12-3 transmits 125 HP at 1000 pinion rpm, find the torque on each shaft. Tooth numbers: pinion Np 23 gear Ng 57

Pinion speed Transmitted power

p 1000 rpm
P 125 hp

Assumptions: There is no loss of power in the gear mesh (100% efficiency). Solution: 1. See Mathcad file P1214. Tp P Tp 7878 in lbf Tp 656.5 ft lbf 2. The gear shaft will rotate at a lower speed, which is determined by the gear ratio. (The speed will be decreased in proportion to the ratio and the torque will be increased by the reciprocal of the ratio). For the gear shaft

For the pinion shaft

3.

g
Tg

Np Ng P

g 403.509 rpm
Tg 19524 in lbf Tg 1627 ft lbf

4.

We could have calculated the torque on the gear shaft directly without finding the gear shaft speed, Ng Tg Tp Tg 19524 in lbf Np Tg 1627 ft lbf

MACHINE DESIGN - An Integrated Approach, 4th Ed.

12-15-1

PROBLEM 12-15
Statement: Given: If the gearset in Problem 12-4 transmits 33 kW at 1600 pinion rpm, find the torque on each shaft. Tooth numbers: pinion Np 27 gear Ng 78

Pinion speed Transmitted power

p 1600 rpm
P 33 kW

Assumptions: There is no loss of power in the gear mesh (100% efficiency). Solution: 1. 2. See Mathcad file P1215. Tp P Tp 197 N m

For the pinion shaft

The gear shaft will rotate at a lower speed, which is determined by the gear ratio. (The speed will be decreased i proportion to the ratio and the torque will be increased by the reciprocal of the ratio). For the gear shaft

3.

g
Tg

Np Ng P

g 553.846 rpm
Tg 569 N m

4.

We could have calculated the torque on the gear shaft directly without finding the gear shaft speed, Ng Tg Tp Tg 569 N m Np

MACHINE DESIGN - An Integrated Approach, 4th Ed.

12-16-1

PROBLEM 12-16
Statement: Size the spur gears in problem 12-14 for a bending factor of safety of at least 2 assuming a steady torque, 25 deg pressure angle, full depth teeth, quality index of 9, an AISI 4140 steel pinion, and a class 40 cast iron gear. Factor of safety Power to be transmitted (hp) Rotational speed of pinion Number of pinion teeth Solution: Pinion 1. 2. Determine the bending geometry factor, J (Table 12-13) Jp 0.42 Write the equations for pitch diameter, pitch-line velocity, and transmitted load in terms of the unknown diametral pitch, pd. Note that, in Mathcad, unit conversion factors are not included. Pitch diameter of pinion (in) d p d Vt p d Wt p d Ka 1
v

Given:

Nfb 2

Number of gear teeth

Ng 57

Reliability H 125 hp R 0.99 AGMA Quality level n 1000 rpm Qv 9 Np 23 Life (cycles) N 10
7

See Mathcad file P1216.

Np pd dpd n 2 Vt p d H

Pitch-line velocity (fpm) Transmitted load (lbf)

3. 4.

Set the application factor, Ka Write the equation for the dynamic load factor, K

B 0.25 12 Qv

0.6667

B 0.52 A 76.878
B

A 50 56 ( 1 B) Kv p d A min A Vt p d ft

5. 6. 7.

Tentatively choose the mounting factor, Km (Assume 2 < F < 6 in) Km 1.7 The bending stress equation for the pinion is Determine the endurance strength of the pinion. Life factor Reliability Temperature factor Material bending strength (psi) AISI 4140 Nitrided steel Endurance strength S fbp S atp KL KT KR KL 1.6831 N
0.0323

bp p d F

Ka Wt p d p d Km Kv p d F Jp

KL 1 KR 1 KT 1 S atp 40000 psi S fbp 40001 psi

KR 0.7 0.15 log( 1 R)

MACHINE DESIGN - An Integrated Approach, 4th Ed.


8.

12-16-2

Write the design equations using the range of face-width to diametral-pitch ratio given in the text, and the bending stress equation, solved for the unknown face width. Upper limit FU p d FL p d S fb 16 pd 8 pd

Lower limit

Safety factor

Nfb =

b
Ka Wt p d p d Km Nfb Kv p d Jp S fbp

Face width

F p d 1 1.5 6 in in in

9.

Plot F(P) vs. p over the range


d

p d

4 F pd in FL pd in FU pd in

10. From the graph, choose a standard value of p d from Table 12-2. p d 4 in
1

11. The calculated value of F is F p d 2.743 in 12. Round this up to the decimal equivalent of a common fractional value. F 2.750 in 13. Then, the parameters that depend on p d and F are: d p d 5.750 in Vt p d 1505 ft min Kv p d 0.809 Wt p d 2740 lbf

2.5

3.5

4 pd in

4.5

5.5

FIGURE 12-16A
Graph of Face Width and Limits for the Pinion in Problem 12-16

bp p d F 19952 psi

The assumption made in step 5 is correct so no further iteration is required. Gear 14. Determine the bending geometry factor, J (Table 12-13) Jg 0.49 15. The bending stress equation for the gear is 16. Determine the endurance strength of the pinion. Material bending strength (psi) Class 40 cast iron Endurance strength S fbg S atg KL KT KR S atg 13000 psi S fbg 13000 psi Ka Wt p d p d Km Kv p d F Jg

bg p d F

MACHINE DESIGN - An Integrated Approach, 4th Ed.

12-16-3

17. Write the design equations using the range of face-width to diametral-pitch ratio given in the text, and the bending stress equation, solved for the unknown face width. Upper limit FU p d FL p d S fb 16 pd 8 pd

Lower limit

Safety factor

Nfb =

b
Ka Wt p d p d Km Nfb Kv p d Jg S fbg

Face width

F p d 1 1.5 6 in in in

18. Plot F(P) vs. p over the range p d


d

19. From the graph, choose a standard value of p d from Table 12-2. p d 3 in
1 F pd in FL pd in FU pd in

20. The calculated value of F is F p d 4.178 in 21. Round this up to the decimal equivalent of a common fractional value. F 4.250 in 22. Then, the parameters that depend on P and F are: d p d 7.667 in Vt p d 2007 ft min Kv p d 0.788 Wt p d 2055 lbf

4.5


1.5

2.5

3.5

4 pd in

4.5

5.5

FIGURE 12-16B
Graph of Face Width and Limits for the Gear in Problem 12-16

bg p d F 6391 psi

23. The gear dimensions are larger (smaller diametral pitch means bigger teeth) than for the pinion. This means that we will accept the gear requirements for the pinion, thus, for the set Diametral pitch Face width p d 3 in
1

F 4.250 in

24. Determine the realized factor of safety for the gear using the above values for F and p d. Gear factor of safety 25. Check the factor of safety on the pinion: Pinion factor of safety Nfbp Nfbg

bg p d F

S fbg

Nfbg 2.0

bp p d F

S fbp

Nfbp 5.4

MACHINE DESIGN - An Integrated Approach, 4th Ed.

12-17-1

PROBLEM 12-17
Statement: Size the spur gears in problem 12-15 for a bending factor of safety of at least 2.5 assuming a steady torque, 20 deg pressure angle, full depth teeth, quality index of 11, an AISI 4340 steel pinion, and an A-7-d nodular iron gear. Factor of safety Power to be transmitted (hp) Rotational speed of pinion Number of pinion teeth Solution: Pinion 1. 2. See Mathcad file P1217. Nfb 2.5 H 33 kW n 1600 rpm Np 27 Number of gear teeth Reliability AGMA Quality level Life (cycles) Ng 78 R 0.99 Qv 11 N 10
7

Given:

Determine the bending geometry factor, J (Table 12-13) Jp 0.37 Write the equations for pitch diameter, pitch-line velocity, and transmitted load in terms of the unknown diametral pitch, p d. Note that, in Mathcad, unit conversion factors are not included. Pitch diameter of pinion (in) d p d Vt p d Wt p d Ka 1
v

Np pd dpd n 2 Vt p d H

Pitch-line velocity (fpm) Transmitted load (lbf) 3. 4. Set the application factor, Ka Write the equation for the dynamic load factor, K

B 0.25 12 Qv

0.6667

B 0.25 A 92
B

A 50 56 ( 1 B) Kv p d A min A Vt p d ft

5. 6. 7.

Tentatively choose the mounting factor, Km (Assume 0 < F < 2 in) Km 1.6 The bending stress equation for the pinion is Determine the endurance strength of the pinion. Life factor Reliability Temperature factor Material bending strength (psi) AISI 4340 Nitrided steel Endurance strength S fbp S atp KL KT KR KL 1.6831 N
0.0323

bp p d F

Ka Wt p d p d Km Kv p d F Jp

KL 1 KR 1 KT 1 S atp 42000 psi S fbp 42001 psi

KR 0.7 0.15 log( 1 R)

8.

Write the design equations using the range of face-width to diametral-pitch ratio given in the text, and the bending stress equation, solved for the unknown face width.

MACHINE DESIGN - An Integrated Approach, 4th Ed.

12-17-2

Upper limit

FU p d FL p d S fb

16 pd 8 pd

Lower limit

Safety factor

Nfb =

b
Ka Wt p d p d Km Nfb Kv p d Jp S fbp

Face width

F p d 1 1.5 10 in in in

9.

Plot F(P) vs. p over the range


d

p d

4 F pd in FL pd in FU pd in

10. From the graph, choose a standard value of p d from Table 12-2. p d 6 in
1

11. The calculated value of F is F p d 1.318 in 12. Round this up to the decimal equivalent of a common fractional value. F 1.375 in 13. Then, the parameters that depend on p d and F are: d p d 4.500 in Vt p d 1885 ft min Kv p d 0.908 Wt p d 775 lbf

7 pd in

10

FIGURE 12-17A
Graph of Face Width and Limits for the Pinion in Problem 12-17

bp p d F 16103 psi

The assumption made in step 5 is correct so no further iteration is required. Gear 14. Determine the bending geometry factor, J (Table 12-13) Jg 0.42 15. The bending stress equation for the gear is 16. Determine the endurance strength of the pinion. Material bending strength (psi) A-7-d nodular iron Endurance strength S fbg S atg KL KT KR S atg 34000 psi S fbg 34001 psi Ka Wt p d p d Km Kv p d F Jg

bg p d F

17. Write the design equations using the range of face-width to diametral-pitch ratio given in the text, and the bending stress equation, solved for the unknown face width.

MACHINE DESIGN - An Integrated Approach, 4th Ed.

12-17-3

Upper limit

FU p d FL p d S fb

16 pd 8 pd

Lower limit

Safety factor

Nfb =

b
Ka Wt p d p d Km Nfb Kv p d Jg S fbg

Face width

F p d 1 1.5 10 in in in

18. Plot F(P) vs. p over the range p d


d

19. From the graph, choose a standard value of p d from Table 12-2. p d 6 in
1 F pd in FL pd in FU pd in

20. The calculated value of F is F p d 1.434 in 21. Round this up to the decimal equivalent of a common fractional value. F 1.500 in 22. Then, the parameters that depend on P and F are: d p d 4.500 in Vt p d 1885 ft min Kv p d 0.908 Wt p d 775 lbf

4.5


1.5

7 pd in

10

FIGURE 12-17B
Graph of Face Width and Limits for the Gear in Problem 12-17

bg p d F 13004 psi

23. The gear dimensions are larger than those for the pinion. This means that we will accept the gear requirements for the pinion, thus, for the set Diametral pitch Face width p d 6 in
1

F 1.500 in

24. Determine the realized factor of safety for the gear using the above values for F and p d. Gear factor of safety Nfbg

bg p d F

S fbg

Nfbg 2.6

25. Check the factor of safety on the pinion: Pinion factor of safety Nfbp

bp p d F

S fbp

Nfbp 2.8

MACHINE DESIGN - An Integrated Approach, 4th Ed.

12-18-1

PROBLEM 12-18
Statement: Size the spur gears in problem 12-14 for a surface factor of safety of at least 2 assuming a steady torque, 25 deg pressure angle, full depth teeth, quality index of 9, an AISI 4140 steel pinion, and a class 40 cast iron gear. Factor of safety Power to be transmitted (hp) Rotational speed of pinion Number of pinion teeth Pressure angle Solution: Pinion 1. Determine the surface geometry factor, I. I 2. See Mathcad file P1218. Nfs 2 Number of gear teeth Ng 57 Reliability H 125 hp R 0.99 AGMA Quality level n 1000 rpm Qv 9 Np 23 25 deg Life (cycles) N 10
7

Given:

sin( ) cos( ) Ng N N 2 p g

I 0.1365

Write the equations for pitch diameter, pitch-line velocity, and transmitted load in terms of the unknown diametral pitch, p d. Note that, in Mathcad, unit conversion factors are not included. Pitch diameter of pinion (in) d p p d Vt p d Wt p d Ca 1
v

Np pd d p p d n 2 H

Pitch-line velocity (fpm)

Transmitted load (lbf)

Vt p d

3. 4.

Set the application factor, Ca Write the equation for the dynamic load factor, C

B 0.25 12 Qv

0.6667

B 0.52 A 76.878
B

A 50 56 ( 1 B) Cv p d A min A Vt p d ft

5. 6. 6. 7.

Tentatively choose the mounting factor, Cm (Assume 2 < F < 6 in Cm 1.7 Choose an elastic coefficient from Table 12-18. (Steel on cast-iron) The surface stress equation for the pinion is Determine the endurance strength of the pinion. Life factor CL 2.466 N
0.056

Cp 2100 psi

0.5

cp p d F Cp

Cv p d F d p p d I

C a W t p d C m

CL 1

MACHINE DESIGN - An Integrated Approach, 4th Ed.


Reliability Temperature factor Material surface strength (psi) AISI 4140 Nitrided steel Endurance strength 8. S fcp S acp CL CT CR CR 0.7 0.15 log( 1 R) CR 1 CT 1

12-18-2

S acp 165000 psi S fcp 164997 psi

Write the design equations using the range of face-width to diametral-pitch ratio given in the text, and the surface stress equation, solved for the unknown face width. Upper limit FU p d FL p d 16 pd 8 pd
2

Lower limit

Safety factor

Sfc Nfc = c

Face width Plot F(P) vs. p over the range p d


d

Cp F p d Nfs Cv p d d p p d I S fcp
1 1.5 6 in in in
4 F pd in FL pd in FU pd in

C a W t p d C m

9.

10. From the graph, choose a standard value of p d from Table 12-2. p d 4 in
1

11. The calculated value of F is F p d 2.379 in 12. Round this up to the decimal equivalent of a common fractional value. F 2.375 in 13. Then, the parameters that depend on p d and F are: d p p d 5.750 in Vt p d 1505 ft min Cv p d 0.809 Wt p d 2740 lbf

2.5

3.5

4 pd in

4.5

5.5

FIGURE 12-18A
Graph of Face Width and Limits for the Pinion in Problem 12-18

cp p d F 116767 psi

The assumption made in step 5 is correct so no further iteration is required. Gear 14. Write the equation for the gear pitch diameter d g p d Ng pd

MACHINE DESIGN - An Integrated Approach, 4th Ed.

12-18-3

15. The surface stress equation for the gear is 16. Determine the endurance strength of the pinion. Material bending strength (psi) Class 40 cast iron Hardness factor From equation 12.26, Gear ratio Hardness ratio factor Endurance strength Hp 230 A 0 mg

cg p d F Cp

Cv p d F d g p d I

C a W t p d C m

S acg 80000 psi Hg 200 Hp Hg 1.2

Ng Np

mg 2.478 CH 1 S fcg 79999 psi

CH 1 A mg 1 S fcg S acg CL CH CT CR

17. Write the design equations using the range of face-width to diametral-pitch ratio given in the text, and the surface stress equation, solved for the unknown face width. Upper limit Lower limit FU p d FL p d 16 pd 8 pd
2

Safety factor

Sfc Nfc = c
F p d 1 1.5 6 in in in

Face width 18. Plot F(P) vs. p over the range p d


d

2 Cp Nfs Cv p d d g p d I S fcg

C a W t p d C m

19. From the graph, choose a standard value of p d from Table 12-2. p d 4 in
1

F pd in


in

4.5

20. The calculated value of F is F p d 4.083 in 21. Round this up to the decimal equivalent of a common fractional value. F 4.125 in 22. Then, the parameters that depend on p d and F are: d g p d 14.250 in Vt p d 1505 ft min Cv p d 0.809 Wt p d 2740 lbf

FL pd

FU pd in


1.5

2.5

3.5

4 pd in

4.5

5.5

FIGURE 12-18B
Graph of Face Width and Limits for the Gear in Problem 12-18

MACHINE DESIGN - An Integrated Approach, 4th Ed.

12-18-4

cg p d F 56282 psi
23. The gear face width is larger than for the pinion. This means that we will accept the gear requirements for the pinion thus, for the set Diametral pitch Face width p d 4 in
1

F 4.125 in

24. Determine the realized factor of safety for the gear using the above values for F and p d.

Gear factor of safety

Sfcg Nfsg cg pd F

Nfsg 2.0

25. Check the factor of safety on the pinion: Pinion factor of safety Nfsp

Sfcp p F cp d

Nfsp 3.5

MACHINE DESIGN - An Integrated Approach, 4th Ed.

12-19-1

PROBLEM 12-19
Statement: Size the spur gears in problem 12-15 for a surface factor of safety of at least 1.2 assuming a steady torque, 20 deg pressure angle, full depth teeth, quality index of 11, an AISI 4340 steel pinion, and an A-7-d nodular iron gear. Factor of safety Power to be transmitted (hp) Rotational speed of pinion Number of pinion teeth Pressure angle Solution: Pinion 1. Determine the surface geometry factor, I. I 2. See Mathcad file P1219. Nfs 1.2 Number of gear teeth Ng 78 Reliability H 33 kW R 0.99 AGMA Quality level n 1600 rpm Qv 11 Np 27 20 deg Life (cycles) N 10
7

Given:

sin( ) cos( ) Ng N N 2 p g

I 0.1194

Write the equations for pitch diameter, pitch-line velocity, and transmitted load in terms of the unknown diametral pitch, p d. Note that, in Mathcad, unit conversion factors are not included. Pitch diameter of pinion (in) d p p d Vt p d Wt p d Ca 1
v

Np pd d p p d n 2 H

Pitch-line velocity (fpm)

Transmitted load (lbf)

Vt p d

3. 4.

Set the application factor, Ca Write the equation for the dynamic load factor, C

B 0.25 12 Qv

0.6667

B 0.25 A 92
B

A 50 56 ( 1 B) Cv p d A min A Vt p d ft

5. 6. 6. 7.

Tentatively choose the mounting factor, Cm (Assume 0 < F < 2 in Cm 1.6 Choose an elastic coefficient from Table 12-18. (Steel on nodular-iron) The surface stress equation for the pinion is Determine the endurance strength of the pinion. Life factor Reliability CL 2.466 N
0.056

Cp 2160 psi

0.5

cp p d F Cp

Cv p d F d p p d I

C a W t p d C m

CL 1 CR 1

CR 0.7 0.15 log( 1 R)

MACHINE DESIGN - An Integrated Approach, 4th Ed.


Temperature factor Material surface strength (psi) AISI 4340 Nitrided steel Endurance strength 8. S fcp S acp CL CT CR CT 1

12-19-2

S acp 162000 psi S fcp 161997 psi

Write the design equations using the range of face-width to diametral-pitch ratio given in the text, and the surface stress equation, solved for the unknown face width. Upper limit FU p d FL p d 16 pd 8 pd
2

Lower limit

Safety factor

Sfc Nfc = c

Face width Plot F(P) vs. p over the range p d


d

Cp F p d Nfs Cv p d d p p d I S fcp
6 6.5 16 in in in
2 F pd in FL pd in FU pd in

C a W t p d C m

9.

10. From the graph, choose a standard value of p d from Table 12-2. p d 10 in
1

1.5

11. The calculated value of F is F p d 1.478 in 12. Round this up to the decimal equivalent of a common fractional value. F 1.500 in 13. Then, the parameters that depend on p d and F are: d p p d 2.700 in Vt p d 1131 ft min Cv p d 0.925 Wt p d 1291 lbf


0.5

10

12 pd in

14

16

FIGURE 12-19A
Graph of Face Width and Limits for the Pinion in Problem 12-19

cp p d F 146808 psi

The assumption made in step 5 is correct so no further iteration is required. Gear 14. Write the equation for the gear pitch diameter d g p d Ng pd Cv p d F d g p d I C a W t p d C m

15. The surface stress equation for the gear is

cg p d F Cp

MACHINE DESIGN - An Integrated Approach, 4th Ed.


16. Determine the endurance strength of the pinion. Material bending strength (psi) A-7-d nodular iron Hardness factor From equation 12.26, Gear ratio Hardness ratio factor Hp 235 A 0 mg Ng Np mg 2.889 CH 1 S fcg 96998 psi Hg 230 S acg 97000 psi Hp Hg 1.0

12-19-3

CH 1 A mg 1 S fcg S acg CL CH CT CR

Endurance strength

17. Write the design equations using the range of face-width to diametral-pitch ratio given in the text, and the surface stress equation, solved for the unknown face width. Upper limit Lower limit FU p d FL p d 16 pd 8 pd
2

Safety factor

Nfc =

Sfc c

Face width 18. Plot F(P) vs. p over the range p d


d

Cp F p d Nfs Cv p d d g p d I S fcg
6 6.5 16 in in in
3

C a W t p d C m

19. From the graph, choose a standard value of p d from Table 12-2. p d 10 in
1

F pd in


in 2

20. The calculated value of F is F p d 1.427 in 21. Round this up to the decimal equivalent of a common fractional value. F 1.500 in 22. Then, the parameters that depend on p d and F are: d g p d 7.800 in Vt p d 1131 ft min Cv p d 0.925 Wt p d 1291 lbf

FL pd

FU pd 1 in

10

12 pd in

14

16

FIGURE 12-19B
Graph of Face Width and Limits for the Gear in Problem 12-19

cg p d F 86374 psi

MACHINE DESIGN - An Integrated Approach, 4th Ed.

12-19-4

23. The gear face width is the same as that for the pinion. This means that we will accept the gear requirements for the set Diametral pitch Face width p d 10 in
1

F 1.500 in

24. Determine the realized factor of safety for the gear using the above values for F and p d.

Gear factor of safety

Sfcg Nfsg cg pd F

Nfsg 1.3

25. Check the factor of safety on the pinion: Pinion factor of safety Nfsp

Sfcp p F cp d

Nfsp 1.2

MACHINE DESIGN - An Integrated Approach, 4th Ed.

12-20-1

PROBLEM 12-20
Statement: Given: If the gearset in Problem 12-11 transmits 83 kW at 1200 arm rpm, find the torque on each shaft. Sun tooth number Planet tooth number Ring gear speed Solution: 1. 2. See Mathcad file P1220. Nr Ns 2 Np Nr 75 Ns 33 Np 21 Transmitted power Arm speed P 83 kW

a 1200 rpm

r 0 rpm

Determine the number of teeth on the ring:

Choose the sun gear as the first gear and the ring gear as the last. Then, write equation (12.11c).

ra sa

r a s a

Ns Nr Nr a Ns a Nr r Ns

Solving for the planet speed,

s 3927 rpm

3.

Calculate the torque on the sun and arm shafts. Arm shaft Ta P

a
P

Ta 660 N m

Sun shaft

Ts

Ts 202 N m

MACHINE DESIGN - An Integrated Approach, 4th Ed.

12-21-1

PROBLEM 12-21
Statement: Given: If the gearset in Problem 12-12 transmits 39 HP at 2600 arm rpm, find the torque on each shaft. Tooth numbers: sun Arm speed Transmitted power Ns 23 P 39 hp planet Sun speed Np 31

a 2600 rpm

s 0 rpm

Assumptions: There is no loss of power in the gear mesh (100% efficiency). Solution: 1. 2. See Mathcad file P1221. Ta P Ta 945 in lbf

For the arm shaft

The ring gear will rotate at a higher speed, which is determined by the gear ratio.

3.

For the ring gear (see Problem 12-12) Number of teeth on ring: Nr a Ns a Ns s Nr P Tr 744 in lbf Nr Ns 2 Np Nr 85

r 3303.529 rpm

Tr

MACHINE DESIGN - An Integrated Approach, 4th Ed.

12-22-1

PROBLEM 12-22
Statement: Given: If the gearset in Problem 12-13 transmits 23 kW at 4800 sun rpm, find the torque on each shaft. Tooth numbers: sun planet Transmitted power Ns 23 Np 31 P 23 kW

Arm speed Sun speed

a 0 rpm s 4800 rpm

Assumptions: There is no loss of power in the gear mesh (100% efficiency). Solution: 1. 2. 3. See Mathcad file P1222. Ts P

For the sun shaft

Ts 45.8 N m

The ring gear will rotate at a lower speed, which is determined by the gear ratio. For the ring gear (see Problem 12-13) Number of teeth on ring: Nr a Ns a Ns s Nr P Tr 169 N m Nr Ns 2 Np Nr 85

r 1298.8 rpm

Tr

MACHINE DESIGN - An Integrated Approach, 4th Ed.

12-23-1

PROBLEM 12-23
Statement: Size the spur gears in problem 12-20 for a bending factor of safety of at least 2.8 assuming a steady torque, 25 deg pressure angle, full depth teeth, quality index of 9, an AISI 4140 steel pinion, and a class 40 cast iron gear. Factor of safety Power to be transmitted Rotational speed of pinion Number of pinion teeth Solution: Pinion 1. 2. Determine the bending geometry factor, J (Table 12-13) Jp 0.40 Write the equations for pitch diameter, pitch-line velocity, and transmitted load in terms of the unknown diametral pitch, p d. Note that, in Mathcad, unit conversion factors are not included. Pitch diameter of pinion and gear (in) d p p d Vt p d Wt p d Ka 1 B 0.25 12 Qv
0.6667

Given:

Nfb 2.8

Number of gear teeth

Ng 33

Reliability H 83 kW R 0.99 AGMA Quality level n 1200 rpm Qv 9 Np 21 Life (cycles) N 10


7

See Mathcad file P1223.

Np pd d g p d n 2 Vt p d H

d g p d

Ng pd

Pitch-line velocity (fpm) Transmitted load (lbf) 3. 4. Set the application factor, Ka Write the equation for the dynamic load factor, K

B 0.52 A 76.878
B

A 50 56 ( 1 B) Kv p d A min A Vt p d ft

5. 6. 7.

Tentatively choose the mounting factor, Km (Assume 2 < F < 6 in) Km 1.7 The bending stress equation for the pinion is Determine the endurance strength of the pinion. Life factor Reliability Temperature factor Material bending strength (psi) AISI 4140 Nitrided steel Endurance strength S fbp S atp KL KT KR KL 1.6831 N
0.0323

bp p d F

Ka Wt p d p d Km Kv p d F Jp

KL 1 KR 1 KT 1 S atp 40000 psi S fbp 40001 psi

KR 0.7 0.15 log( 1 R)

MACHINE DESIGN - An Integrated Approach, 4th Ed.


8.

12-23-2

Write the design equations using the range of face-width to diametral-pitch ratio given in the text, and the bending stress equation, solved for the unknown face width. Upper limit FU p d FL p d S fb 16 pd 8 pd

Lower limit

Safety factor

Nfb =

b
Ka Wt p d p d Km Nfb Kv p d Jp S fbp

Face width

F p d 1 1.5 6 in in in

9.

Plot F(P) vs. p over the range


d

p d

6 F pd in FL pd in

10. From the graph, choose a standard value of p d from Table 12-2. p d 4 in
1

5 4 3

11. The calculated value of F is F p d 2.196 in 12. Round this up to the decimal equivalent of a common fractional value. F 2.375 in 13. Then, the parameters that depend on p d and F are: d p p d 5.250 in Vt p d 2592 ft min Kv p d 0.768 Wt p d 1417 lbf

FU pd 2 in 1 0

4 pd in

FIGURE 12-23A
Graph of Face Width and Limits for the Pinion in Problem 12-23

bp p d F 13212 psi

The assumption made in step 5 is correct so no further iteration is required. Gear 14. Determine the bending geometry factor, J (Table 12-13) Jg 0.45 15. The bending stress equation for the gear is 16. Determine the endurance strength of the pinion. Material bending strength (psi) Class 40 cast iron Endurance strength S fbg S atg KL KT KR S atg 13000 psi S fbg 13000 psi Ka Wt p d p d Km Kv p d F Jg

bg p d F

MACHINE DESIGN - An Integrated Approach, 4th Ed.

12-23-3

17. Write the design equations using the range of face-width to diametral-pitch ratio given in the text, and the bending stress equation, solved for the unknown face width. Upper limit FU p d FL p d S fb 16 pd 8 pd

Lower limit

Safety factor

Nfb =

b
Ka Wt p d p d Km Nfb Kv p d Jg S fbg

Face width

F p d 1 1.25 6 in in in

18. Plot F(P) vs. p over the range p d


d

19. From the graph, choose a standard value of p d from Table 12-2. p d 3 in
1 F pd in FL pd in FU pd in

20. The calculated value of F is F p d 3.486 in 21. Round this up to the decimal equivalent of a common fractional value. F 3.500 in 22. Then, the parameters that depend on P and F are: d g p d 11.000 in Vt p d 3456 ft min Kv p d 0.744 Wt p d 1063 lbf

3 pd in

FIGURE 12-23B
Graph of Face Width and Limits for the Gear in Problem 12-23

bg p d F 4624 psi

23. The gear dimensions are larger (smaller diametral pitch means bigger teeth) than for the pinion. This means that we will accept the gear requirements for the pinion, thus, for the set Diametral pitch Face width p d 3 in
1

F 3.500 in

24. Determine the realized factor of safety for the gear using the above values for F and p d. Gear factor of safety 25. Check the factor of safety on the pinion: Pinion factor of safety Nfbp Nfbg

bg p d F
S fbp

S fbg

Nfbg 2.8

bp p d F

Nfbp 7.7

MACHINE DESIGN - An Integrated Approach, 4th Ed.

12-24-1

PROBLEM 12-24
Statement: Size the spur gears in Problem 12-21 for a bending factor of safety of at least 2.4 assuming a steady torque, 20 deg pressure angle, full depth teeth, quality index of 11, an AISI 4340 steel pinion, and an A-7-d nodular iron gear. Factor of safety Power to be transmitted (hp) Number of pinion (sun) teeth Number of gear (planet) teeth Solution: See Mathcad file P1224. Nfb 2.4 H 39 hp Np 23 Ng 31 Reliability AGMA Quality level Life (cycles) R 0.99 Qv 11 N 10
7

Given:

Take the sun as the pinion and the planet as the gear, which is also an idler. With a stationary sun, the planet gear will have one point (where it meshes with the sun) that has an instantaneous zero velocity. All other points on the planet will have a linear velocity that is proportional to its distance from this instant center. The point that meshes with the ring gear has maximum velocity. The pitchline velocity at the ring gear is a function of the tooth numbers and the diametral pitch of the set. But the absolute planet velocity depends only on the tooth numbers and the arm velocity. For the data of Problem 12-21, the absolute planet (gear) rotational velocity is For

a 2600 rpm g
Np 2 Ng Ng r

r 3303.53 rpm g 9058.1 rpm

The sun (pinion) is stationary and, therefore, has a zero pitchline velocity. However, the planet (gear) mesh point rotates about the pitch circle of the sun at the rotational velocity of the arm. Therefore, take the pinion pitchline velocity to be Vtp = rp a = Np 2 pd a

The maximum pitchline velocity of the planet (gear) is at the point where it meshes with the ring gear, which is also the pitchline velocity of the ring gear. Thus, for the gear, the pitchline velocity is Vtg = rr r = Np 2 Ng 2 pd r

The torques on the arm and ring were determined in Problem 12-21 to be Ta 945.38 in lbf and Tr 744.05 in lbf . Load analysis of the gearset reveals that the transmitted load on the sun, planet, and ring gear teeth is Wt = 2 p d Tr Np 2 Ng

Pinion 1. 2. Determine the bending geometry factor, J (Table 12-13) Jp 0.34 Write the equations for pitch diameter, pitch-line velocity, and transmitted load in terms of the unknown diametral pitch, pd. Note that, in Mathcad, unit conversion factors are not included. Pitch diameter of pinion (in) d p d Np pd

MACHINE DESIGN - An Integrated Approach, 4th Ed.


Vtp p d Wt p d Ka 1 B 0.25 12 Qv
0.6667

12-24-2
Np 2 pd

Pitch-line velocity (fpm)

Transmitted load (lbf) 3. 4. Set the application factor, Ka Write the equation for the dynamic load factor, K

2 p d Tr Np 2 Ng

B 0.25 A 92
B

A 50 56 ( 1 B) Kvp p d A min A Vtp pd ft

5. 6. 7.

Tentatively choose the mounting factor, Km (Assume 0 < F < 2 in) Km 1.6 The bending stress equation for the pinion is Determine the endurance strength of the pinion. Life factor Reliability Temperature factor Material bending strength (psi) AISI 4340 Nitrided steel Endurance strength S fbp S atp KL KT KR KL 1.6831 N
0.0323

bp p d F

Ka Wt p d p d Km Kvp p d F Jp

KL 1 KR 1 KT 1 S atp 42000 psi S fbp 42001 psi

KR 0.7 0.15 log( 1 R)

8.

Write the design equations using the range of face-width to diametral-pitch ratio given in the text, and the bending stress equation, solved for the unknown face width. Upper limit FU p d FL p d S fb 16 pd 8 pd

Lower limit

Safety factor

Nfb =

b
Ka Wt p d p d Km Nfb Kvp p d Jp S fbp

Face width

F p d 10 11.5 20 in in in

9.

Plot F(P) vs. p over the range


d

p d

10. From the graph, choose a standard value of p d from Table 12-2.

MACHINE DESIGN - An Integrated Approach, 4th Ed.


1 2 F pd in FL pd in FU pd in

12-24-3

p d 14 in

11. The calculated value of F is F p d 0.997 in 12. Round this up to the decimal equivalent of a common fractional value. F 1.000 in 13. Then, the parameters that depend on p d and F are: d p d 1.643 in Vtp p d 1118 ft min Kvp p d 0.925 Wt p d 245 lbf

1.5


0.5

0 10

12

14

16 pd in

18

20

bp p d F 17449 psi
FIGURE 12-24A The assumption made in step 5 is correct so no further iteration is required. Gear 14. Determine the bending geometry factor, J (Table 12-9) Jg 0.37

Graph of Face Width and Limits for the Pinion in Problem 12-24

15. Write the equations for pitch diameter and pitchline velocity for the gear in terms of the unknown diametral pitc p d. Note that, in Mathcad, unit conversion factors are not included. Pitch diameter of pinion (in) dg p d Vtg p d Ng pd Np Ng 2 pd r

Pitchline velocity (fpm)

16. Write the equation for the dynamic load factor, K


v

A Kvg p d min A Vtg pd ft KI 1.42

17. Account for the fact that the gear is an idler by setting the idler factor 17. The bending stress equation for the gear is 18. Determine the endurance strength of the pinion. Material bending strength (psi) A-7-d nodular iron Endurance strength S fbg S atg KL KT KR

bg p d F

Ka Wt p d p d Km KI Kvg p d F Jg

S atg 34000 psi S fbg 34001 psi

19. Write the design equations using the range of face-width to diametral-pitch ratio given in the text, and the bending stress equation, solved for the unknown face width.

MACHINE DESIGN - An Integrated Approach, 4th Ed.

12-24-4

Upper limit

FU p d FL p d S fb

16 pd 8 pd

Lower limit

Safety factor

Nfb =

b
Ka Wt p d p d Km KI Nfb Kvg p d Jg S fbg

Face width

F p d

17. Plot F(P) vs. p d over the range

p d

6 6.5 16 in in in
2

18. From the graph, choose a standard value of p d from Table 12-2. p d 12 in
1

F pd in


in

1.5

19. The calculated value of F is F p d 1.244 in 20. Round this up to the decimal equivalent of a common fractional value. F 1.250 in 21. Then, the parameters that depend on P and F are: dg p d 2.583 in Vtg p d 3892 ft min Kvg p d 0.879 Wt p d 210 lbf

FL pd

FU pd in


0.5

10

12 pd in

14

16

FIGURE 12-24B

Graph of Face Width and Limits for the Gear in Problem 12-24

bg p d F 14095 psi
22. The gear dimensions are larger (smaller diametral pitch means bigger teeth) than for the pinion. This means that we will accept the gear requirements for the pinion, thus, for the set Diametral pitch Face width p d 12 in
1

F 1.250 in

23. Determine the realized factor of safety for the gear using the above values for F and p d. Gear factor of safety 24. Check the factor of safety on the pinion: Pinion factor of safety Nfbp Nfbg

bg p d F

S fbg

Nfbg 2.4

bp p d F

S fbp

Nfbp 4.1

MACHINE DESIGN - An Integrated Approach, 4th Ed.

12-25-1

PROBLEM 12-25
Statement: Size the spur gears in problem 12-20 for a surface factor of safety of at least 1.7 assuming a steady torque, 25 deg pressure angle, full depth teeth, quality index of 9, an AISI 4140 steel pinion, and a class 40 cast iron gear. Factor of safety Power to be transmitted Rotational speed of pinion Number of pinion teeth Pressure angle Solution: Pinion 1. Determine the surface geometry factor, I. I 2. See Mathcad file P1225. Nfs 1.7 Number of gear teeth Ng 33 Reliability H 83 kW R 0.99 AGMA Quality level n g 1200 rpm Qv 9 Np 21 25 deg Life (cycles) N 10
7

Given:

sin( ) cos( ) Ng N N 2 p g

I 0.1170

Write the equations for pitch diameter, pitchline velocity, and transmitted load in terms of the unknown diametral pitch, p d. Note that, in Mathcad, unit conversion factors are not included. Pitch diameters of pinion and gear (in) d p p d Vt p d Wt p d Ca 1
v

Np pd d g p d n g 2 Vt p d H

d g p d

Ng pd

pitchline velocity (fpm)

Transmitted load (lbf)

3. 4.

Set the application factor, Ca Write the equation for the dynamic load factor, C

B 0.25 12 Qv

0.6667

B 0.52 A 76.878
B

A 50 56 ( 1 B) Cv p d A min A Vt p d ft

5. 6.

Tentatively choose the mounting factor, Cm (Assume 2 < F < 6 in Cm 1.7 Choose an elastic coefficient from Table 12-18. (Steel on cast iron). Cp 2100 psi
0.5

7. 8.

The surface stress equation for the pinion is Determine the endurance strength of the pinion. Life factor Reliability

cp p d F Cp

Cv p d F d p p d I

C a W t p d C m

CL 2.466 N

0.056

CL 1 CR 1

CR 0.7 0.15 log( 1 R)

MACHINE DESIGN - An Integrated Approach, 4th Ed.


Temperature factor Material surface strength (psi) AISI 4140 Nitrided steel Endurance strength 9. S fcp S acp CL CT CR CT 1

12-25-2

S acp 165000 psi S fcp 164997 psi

Write the design equations using the range of face-width to diametral-pitch ratio given in the text, and the bending stress equation, solved for the unknown face width. Upper limit FU p d FL p d 16 pd 8 pd
2

Lower limit

Safety factor

Sfc Nfc = c

Face width

Cp F p d Nfs Cv p d d p p d I S fcp
p d 1 1.5 6 in in in
4 F pd in FL pd in FU pd in

C a W t p d C m

10. Plot F(P) vs. p d over the range

11. From the graph, choose a standard value of p d from Table 12-2. p d 5 in
1

12. The calculated value of F is F p d 2.149 in 13. Round this up to the decimal equivalent of a common fractional value. F 2.250 in 14. Then, the parameters that depend on p d and F are: d p p d 4.200 in Vt p d 2073 ft min Cv p d 0.785 Wt p d 1771 lbf

4 pd in

FIGURE 12-25A

Graph of Face Width and Limits for the Pinion in Problem 12-25

cp p d F 123671 psi
The assumption made in step 5 is correct so no further iteration is required. Gear 15. Write the equation for the gear pitch diameter d g p d Ng pd Cv p d F d g p d I C a W t p d C m

16. The surface stress equation for the gear is

cg p d F Cp

MACHINE DESIGN - An Integrated Approach, 4th Ed.


17. Determine the endurance strength of the pinion. Material bending strength (psi) Class 40 cast iron Hardness factor From equation 12.26, Gear ratio Hardness ratio factor Hp 230 A 0 mg Ng Np mg 1.571 CH 1 S fcg 79999 psi Hg 200 S acg 80000 psi Hp Hg 1.2

12-25-3

CH 1 A mg 1 S fcg S acg CL CH CT CR

Endurance strength

18. Write the design equations using the range of face-width to diametral-pitch ratio given in the text, and the bending stress equation, solved for the unknown face width. Upper limit FU p d FL p d 16 pd 8 pd
2

Lower limit

Safety factor

Nfc =

Sfc c

Face width

Cp F p d Nfs Cv p d d g p d I S fcg
p d 1 1.5 6 in in in
6 F pd in FL pd in FU pd in

C a W t p d C m

19. Plot F(P) vs. p d over the range

20. From the graph, choose a standard value of p d from Table 12-2. p d 4 in
1

4.5

21. The calculated value of F is F p d 3.807 in 22. Round this up to the decimal equivalent of a common fractional value. F 4.000 in 23. Then, the parameters that depend on P and F are: d g p d 8.250 in Cv p d 0.768


1.5

2.5

3.5

4 pd in

4.5

5.5

FIGURE 12-25B

Graph of Face Width and Limits for the Gear in Problem 12-25

MACHINE DESIGN - An Integrated Approach, 4th Ed.


Vt p d 2592 ft min Wt p d 1417 lbf

12-25-4

cg p d F 59858 psi

24. The gear dimensions are larger (smaller diametral pitch means bigger teeth) than for the pinion. This means that we will accept the gear requirements for the pinion, thus for the set Diametral pitch Face width p d 4 in
1

F 4.000 in

25. Determine the realized factor of safety for the gear using the above values for F and p d. Gear factor of safety

Sfcg Nfsg cg pd F

Nfsg 1.8

26. Check the factor of safety on the pinion: Pinion factor of safety Nfsp

Sfcp p F cp d

Nfsp 4.8

MACHINE DESIGN - An Integrated Approach, 4th Ed.

12-26-1

PROBLEM 12-26
Statement: Size the spur gears in Problem 12-21 for a surface factor of safety of at least 1.3 assuming a steady torque, 20 deg pressure angle, full depth teeth, quality index of 11, an AISI 4340 steel pinion, and an A-7-d nodular iron gear. Factor of safety Power to be transmitted (hp) Number of pinion (sun) teeth Number of gear (planet) teeth Solution: See Mathcad file P1226. Nfs 1.3 H 39 hp Np 23 Ng 31 Pressure angle Reliability AGMA Quality level Life (cycles)

Given:

20 deg
R 0.99 Qv 11 N 10
7

Take the sun as the pinion and the planet as the gear, which is also an idler. With a stationary sun, the planet gear will have one point (where it meshes with the sun) that has an instantaneous zero velocity. All other points on the planet will have a linear velocity that is proportional to its distance from this instant center. The point that meshes with the ring gear has maximum velocity. The pitchline velocity at the ring gear is a function of the tooth numbers and the diametral pitch of the set. But the absolute planet velocity depends only on the tooth numbers and the arm velocity. For the data of Problem 12-21, the absolute planet (gear) rotational velocity is For

a 2600 rpm g
Np 2 Ng Ng r

r 3303.53 rpm g 9058.1 rpm

The sun (pinion) is stationary and, therefore, has a zero pitchline velocity. However, the planet (gear) mesh point rotates about the pitch circle of the sun at the rotational velocity of the arm. Therefore, take the pinion pitchline velocity to be Vtp = rp a = Np 2 pd a

The maximum pitchline velocity of the planet (gear) is at the point where it meshes with the ring gear, which is also the pitchline velocity of the ring gear. Thus, for the gear, the pitchline velocity is Vtg = rr r = Np 2 Ng 2 pd r

The torques on the arm and ring were determined in Problem 12-21 to be Ta 945.38 in lbf and Tr 744.05 in lbf . Load analysis of the gearset reveals that the transmitted load on the sun, planet, and ring gear teeth is Wt = Pinion 1. Determine the surface geometry factor, I. I 2. 2 p d Tr Np 2 Ng

sin( ) cos( ) Ng N N 2 p g

I 0.0923

Write the equations for pitch diameter, pitchline velocity, and transmitted load in terms of the unknown diametr pitch, p d. Note that, in Mathcad, unit conversion factors are not included. Pitch diameter of pinion (in) d p p d Np pd

MACHINE DESIGN - An Integrated Approach, 4th Ed.

12-26-2

pitchline velocity (fpm)

Vtp p d Wt p d Ca 1
v

Np 2 pd

Transmitted load (lbf) 3. 4. Set the application factor, Ca Write the equation for the dynamic load factor, C

2 p d Tr Np 2 Ng

B 0.25 12 Qv

0.6667

B 0.25 A 92
B

A 50 56 ( 1 B) A Cvp p d min A Vtp pd ft 5. 6. Tentatively choose the mounting factor, Cm (Assume 0 < F < 2 in) Choose an elastic coefficient from Table 12-18. Cm 1.6 Cp 2100 psi
0.5

7. 8.

The surface stress equation for the pinion is Determine the endurance strength of the pinion. Life factor Reliability Temperature factor Material surface strength (psi) AISI 4340 Nitrided steel Endurance strength

cp p d F Cp

Cvp p d F d p p d I

Ca Wt p d Cm

CL 2.466 N

0.056

CL 1 CR 1 CT 1 S acp 162000 psi

CR 0.7 0.15 log( 1 R)

S fcp

S acp CL CT CR

S fcp 161997 psi

9.

Write the design equations using the range of face-width to diametral-pitch ratio given in the text, and the bending stress equation, solved for the unknown face width. Upper limit FU p d FL p d 16 pd 8 pd
2

Lower limit

Safety factor

Sfc Nfc = c

Face width

Cp F p d Nfs Cvp p d d p p d I S fcp

C a W t p d C m

MACHINE DESIGN - An Integrated Approach, 4th Ed.

12-26-3

10. Plot F(P) vs. p over the range


d

p d

6 6.5 16 in in in
2 F pd in

11. From the graph, choose a standard value of p d from Table 12-2. p d 14 in
1


in

1.5

12. The calculated value of F is F p d 0.611 in 13. Round this up to the decimal equivalent of a common fractional value. F 0.625 in 14. Then, the parameters that depend on p d and F are: d p p d 1.643 in Vtp p d 1118 ft min Cvp p d 0.925 Wt p d 245 lbf

FL pd

FU pd in


0.5

10

12 pd in

14

16

FIGURE 12-26A

Graph of Face Width and Limits for the Pinion in Problem 12-26

cp p d F 140460 psi
The assumption made in step 5 is correct so no further iteration is required. Gear 15. Write the equations for pitch diameter and pitchline velocity for the gear in terms of the unknown diametral pitc p d. Note that, in Mathcad, unit conversion factors are not included. Pitch diameter of pinion (in) d g p d Vtg p d Ng pd Np Ng 2 pd r

pitchline velocity (fpm)

16. Write the equation for the dynamic load factor, C


v

Cvg p d

A min A Vtg pd ft

17. The surface stress equation for the gear is 18. Determine the endurance strength of the pinion. Material bending strength (psi) A-7-d nodular iron Hardness factor From equation 12.26, Hp 235 A 0

cg p d F Cp

Cvg p d F d g p d I

Ca Wt p d Cm

S acg 97000 psi Hg 230 Hp Hg 1.0

MACHINE DESIGN - An Integrated Approach, 4th Ed.

12-26-4

Gear ratio Hardness ratio factor

mg

Ng Np

mg 1.348 CH 1

CH 1 A mg 1 S acg CL CH CT CR

Endurance strength

S fcg

S fcg 96998 psi

19. Write the design equations using the range of face-width to diametral-pitch ratio given in the text, and the bending stress equation, solved for the unknown face width. Upper limit Lower limit FU p d FL p d 16 pd 8 pd
2

Safety factor

Sfc Nfc = c
F p d p d

Face width

2 Cp Nfs Cvg p d d g p d I S fcg

C a W t p d C m

20. Plot F(P) vs. p d over the range

6 6.5 16 in in in
3 F pd in FL pd in FU pd in 0.5 0

21. From the graph, choose a standard value of p d from Table 12-2. p d 12 in
1

2.5 2 1.5 1

22. The calculated value of F is F p d 0.978 in 23. Round this up to the decimal equivalent of a common fractional value. F 1.000 in 24. Then, the parameters that depend on P and F are: d g p d 2.583 in Vtg p d 3892 ft min Cvg p d 0.879 Wt p d 210 lbf

10

12 pd in

14

16

FIGURE 12-26B
Graph of Face Width and Limits for the Gear in Problem 12-26

cg p d F 84140 psi
Diametral pitch Face width p d 12 in
1

25. The gear dimensions are larger than that for the pinion. Thus, for the set

F 1.000 in

26. Determine the realized factor of safety for the gear using the above values for F and p d.

MACHINE DESIGN - An Integrated Approach, 4th Ed.

12-26-5

Gear factor of safety 27. Check the factor of safety on the pinion: Pinion factor of safety

Sfcg Nfsg cg pd F Sfcp Nfsp cp pd F

Nfsg 1.3

Nfsp 2.9

MACHINE DESIGN - An Integrated Approach, 4th Ed.

12-27-1

PROBLEM 12-27
Statement: Given: If the gearset in Problem 12-10 transmits 190 kW at 1800 input pinion rpm, find the torque on each of the four shafts. Transmitted power P 190 kW Input speed

p1 1800 rpm

Assumptions: Tooth numbers: Stage 1 Stage 2 Stage 3 Solution: 1. Np1 18 Np2 18 Np3 18 Ng1 164 Ng2 144 Ng3 162

See Mathcad file P1227.

Confirm that the gear ratio is correct. mG Ng1 Ng2 Ng3 Np1 Np2 Np3 mG 656.000

2.

Calculate the gear speeds. Stage 1

p1 1800 rpm

g1

Np1 Ng1 Np2 Ng2 Np3 Ng3

p1

g1 197.561 rpm

Stage 2

p2 g1

g2

p2

g2 24.695 rpm

Stage 3

p3 g2

g3

p3

g3 2.744 rpm

3.

Calculate the shaft torque. Shaft 1 T1 P

p1
P

T1 1008 N m

Input

Shaft 2

T2

g1
P

T2 9184 N m

Shaft 3

T3

g2
P

T3 73471 N m

Shaft 4

T4

g3

T4 661236 N m

Output

MACHINE DESIGN - An Integrated Approach, 4th Ed.

12-28-1

PROBLEM 12-28
Statement: Size the first-stage spur gears in problem 12-27 for a bending factor of safety of at least 3.2 and a surface factor of safety of at least 1.7 assuming a steady torque, 25 deg pressure angle, full depth teeth, Qv = 8, and AISI 4140 steel for all gears. Pressure angle Bending factor of safety Surface factor of safety Power to be transmitted (hp) Number of pinion teeth

Given:

25 deg
Nfb 3.2 Nfs 1.7 H 190 kW Np 18 Reliability AGMA Quality level Life (cycles) Number of gear teeth R 0.99 Qv 8 N 10
7

Ng 164

Rotational speed of pinion (rpm) n 1800 rpm Assumptions: Since both pinion and gear from each stage are the same material, it will only be necessary to determine the pinion size as it will be governing for the set. Solution: See Mathcad file P1228.

Stage 1 Pinion - Bending 1. 2. Determine the bending geometry factor, J (Table 12-13) Jp 0.38

Write the equations for pitch diameter, pitchline velocity, and transmitted load in terms of the unknown diametral pitch, pd. Note that, in Mathcad, unit conversion factors are not included. Pitch diameter of pinion and gear (in) d p p d Vt p d Wt p d Ka 1 Np pd d p p d n 2 Vt p d H d g p d Ng pd

pitchline velocity (fpm) Transmitted load (lbf) 3. 4. Set the application factor, Ka

Write the equation for the dynamic load factor, Kv B 0.25 12 Qv B 0.63
0.6667

A 50 56 ( 1 B) Kv p d A min A Vt p d ft Km 1.7
B

A 70.721

5. 6.

Tentatively choose the mounting factor, Km (Assume 2 < F < 6 in) The bending stress equation for the pinion is

bp p d F
7. Determine the endurance strength of the pinion. Life factor KL 1.6831 N

Ka Wt p d p d Km Kv p d F Jp

0.0323

KL 1

MACHINE DESIGN - An Integrated Approach, 4th Ed.


Reliability Temperature factor Material bending strength (psi) AISI 4140 Nitrided steel Endurance strength 8. S fbp S atp KL KT KR KR 0.7 0.15 log( 1 R) KR 1 KT 1

12-28-2

S atp 40000 psi S fbp 40001 psi

Write the design equations using the range of face-width to diametral-pitch ratio given in the text, and the bending stress equation, solved for the unknown face width. Upper limit FU p d FL p d S fb 16 pd 8 pd

Lower limit

Safety factor

Nfb =

b
Ka Wt p d p d Km Nfb Kv p d Jp S fbp

Face width

F p d

9.

Plot F(P) vs. p over the range


d

p d

1 1.1 5 in in in
8 F pd in

10. From the graph, choose a standard value of p d from Table 12-2. p d 3 in
1


in

11. The calculated value of F is F p d 4.546 in 12. Round this up to the decimal equivalent of a common fractional value. F 4.500 in 13. Then, the parameters that depend on p d and F are: d p p d 6.000 in Vt p d 2827 ft min d g p d 54.667 in

FL pd

FU pd in

2.5

3 pd in

3.5

FIGURE 12-28A Wt p d 2974 lbf Kv p d 0.702


Graph of Face Width and Limits for the Stage 1 Pinion (bending) in Problem 12-28

bp p d F 12627 psi

The assumption made in step 5 is correct so no further iteration is required. Stage 1 Pinion - Surface Fatigue 1. Determine the surface geometry factor, I.

MACHINE DESIGN - An Integrated Approach, 4th Ed.

12-28-3

I 2.

sin( ) cos( ) Ng N N 2 p g

I 0.1726

Write the equations for pitch diameter, pitchline velocity, and transmitted load in terms of the unknown diametral pitch, pd. Note that, in Mathcad, unit conversion factors are not included. Pitch diameter of pinion (in) d p p d Vt p d Wt p d Ca 1 Np pd d p p d n 2 H

pitchline velocity (fpm)

Transmitted load (lbf)

Vt p d

3. 4.

Set the application factor, Ca

Write the equation for the dynamic load factor, Cv B 0.25 12 Qv B 0.63
0.6667

A 50 56 ( 1 B) Cv p d A min A Vt p d ft Cm 1.7 Cp 2300 psi


0.5 B

A 70.721

5. 6. 7.

Tentatively choose the mounting factor, Cm (Assume 2 < F < 6 in) Choose an elastic coefficient from Table 12-18. (Steel on steel). The surface stress equation for the pinion is

cp p d F Cp
8. Determine the endurance strength of the pinion. Life factor Reliability Temperature factor Material surface strength (psi) AISI 4140 Nitrided steel Endurance strength 9. S fcp S acp CL CT CR CL 2.466 N

Cv p d F d p p d I

C a W t p d C m

0.056

CL 1 CR 1 CT 1 S acp 165000 psi S fcp 164997 psi

CR 0.7 0.15 log( 1 R)

Write the design equations using the range of face-width to diametral-pitch ratio given in the text, and the bending stress equation, solved for the unknown face width. Upper limit FU p d 16 pd

MACHINE DESIGN - An Integrated Approach, 4th Ed.


FL p d 8 pd
2

12-28-4

Lower limit

Safety factor

Sfc Nfc = c
F p d

Face width

Cp Nfs Cv p d d p p d I S fcp

C a W t p d C m

10. Plot F(P) vs. p over the range


d

p d

1 1.1 6 in in in
6

11. From the graph, choose a standard value of p d from Table 12-2. p d 4 in
1

F pd in


in 4

12. The calculated value of F is F p d 3.933 in 13. Round this up to the decimal equivalent of a common fractional value. F 4.000 in

FL pd

FU pd 2 in

14. The bending requirement is governing in this case. The diametral pitch and face width for the stage 2 gearset are: p d 3 in
1

4 pd in

FIGURE 12-28B F 4.500 in


Graph of Face Width and Limits for the Stage 1 Pinion (surface) in Problem 12-28

MACHINE DESIGN - An Integrated Approach, 4th Ed.

12-29-1

PROBLEM 12-29
Statement: Size the second-stage spur gears in problem 12-27 for a bending factor of safety of at least 3.2 and a surface factor of safety of at least 1.7 assuming a steady torque, 25 deg pressure angle, full depth teeth, quality index of 8, and AISI 4140 steel for all gears. Pressure angle Bending factor of safety Surface factor of safety Power to be transmitted Number of pinion teeth Rotational speed of pinion (rpm)

Given:

25 deg
Nfb 3.2 Nfs 1.7 H 190 kW Np 18 n 197.561 rpm Reliability AGMA Quality level Life (cycles) Number of gear teeth R 0.99 Qv 8 N 10 Ng 144
7

Assumptions: Since both pinion and gear from each stage are the same material, it will only be necessary to determine the pinion size as it will be governing for the set. See Mathcad file P1229. Solution: Stage 2 Pinion - Bending 1. 2. Determine the bending geometry factor, J (Table 12-13) Jp 0.38

Write the equations for pitch diameter, pitchline velocity, and transmitted load in terms of the unknown diametral pitch, p d. Note that, in Mathcad, unit conversion factors are not included. d p p d Vt p d Wt p d Ka 1 Np pd d p p d n 2 Vt p d H d g p d Ng pd

Pitch diameter of pinion and gear (in)

pitchline velocity (fpm) Transmitted load (lbf) 3. 4. Set the application factor, Ka

Write the equation for the dynamic load factor, Kv B 0.25 12 Qv B 0.63
0.6667

A 50 56 ( 1 B) Kv p d A min A Vt p d ft Km 1.8
B

A 70.721

5. 6.

Tentatively choose the mounting factor, Km (Assume 6 < F < 9 in) The bending stress equation for the pinion is

bp p d F
7. Determine the endurance strength of the pinion.

Ka Wt p d p d Km Kv p d F Jp

Life factor

KL 1.6831 N

0.0323

KL 1

MACHINE DESIGN - An Integrated Approach, 4th Ed.


Reliability Temperature factor Material bending strength (psi) AISI 4140 Nitrided steel Endurance strength 8. S fbp S atp KL KT KR KR 0.7 0.15 log( 1 R) KR 1 KT 1

12-29-2

S atp 40000 psi S fbp 40001 psi

Write the design equations using the range of face-width to diametral-pitch ratio given in the text, and the bending stress equation, solved for the unknown face width. Upper limit FU p d FL p d S fb 16 pd 8 pd

Lower limit

Safety factor

Nfb =

b
Ka Wt p d p d Km Nfb Kv p d Jp S fbp

Face width

F p d

9.

Plot F(P) vs. p over the range


d

p d

1 1.1 3 in in in

10. From the graph, choose a standard value of p d from Table 12-2. p d 1.5 in
1 F pd in FL pd in FU pd in

20

11. The calculated value of F is F p d 9.313 in 12. Round this up to the decimal equivalent of a common fractional value. F 9.375 in 13. Then, the parameters that depend on p d and F are: d p p d 12.000 in Vt p d 621 ft min d g p d 96.000 in Wt p d 13547 lbf Kv p d 0.827

15

10

1.5 pd in

2.5

FIGURE 12-29A
Graph of Face Width and Limits for the Stage 2 Pinion (bending) in Problem 12-29

bp p d F 12417 psi

The assumption made in step 5 is correct so no further iteration is required. Stage 2 Pinion - Surface Fatigue 1. Determine the surface geometry factor, I.

MACHINE DESIGN - An Integrated Approach, 4th Ed.


I 2.

12-29-3
I 0.1702

sin( ) cos( ) Ng N N 2 p g

Write the equations for pitch diameter, pitchline velocity, and transmitted load in terms of the unknown diametral pitch, p d. Note that, in Mathcad, unit conversion factors are not included. Pitch diameter of pinion (in) d p p d Vt p d Wt p d Ca 1 Np pd d p p d n 2 H

pitchline velocity (fpm)

Transmitted load (lbf)

Vt p d

3. 4.

Set the application factor, Ca

Write the equation for the dynamic load factor, Cv B 0.25 12 Qv B 0.63
0.6667

A 50 56 ( 1 B) Cv p d A min A Vt p d ft Cm 1.8 Cp 2300 psi


0.5 B

A 70.721

5. 6. 7.

Tentatively choose the mounting factor, Cm (Assume 6 < F < 9 in) Choose an elastic coefficient from Table 12-18. (Steel on steel). The surface stress equation for the pinion is

cp p d F Cp
8. Determine the endurance strength of the pinion. Life factor Reliability Temperature factor Material surface strength (psi) AISI 4140 Nitrided steel Endurance strength 9. S fcp S acp CL CT CR CL 2.466 N

Cv p d F d p p d I

C a W t p d C m

0.056

CL 1 CR 1 CT 1 S acp 165000 psi S fcp 164997 psi

CR 0.7 0.15 log( 1 R)

Write the design equations using the range of face-width to diametral-pitch ratio given on page 740 of the text, and the bending stress equation, solved for the unknown face width. Upper limit FU p d 16 pd

MACHINE DESIGN - An Integrated Approach, 4th Ed.

12-29-4

Lower limit

FL p d

8 pd
2

Safety factor

Nfc =

Sfc c

Face width

Cp F p d Nfs Cv p d d p p d I S fcp
p d 1 1.1 3 in in in
20

C a W t p d C m

10. Plot F(P) vs. p over the range


d

11. From the graph, choose a standard value of p d from Table 12-2. p d 2 in
1

F pd in


in

15

12. The calculated value of F is F p d 8.290 in 13. Round this up to the decimal equivalent of a common fractional value. F 8.250 in

FL pd

10

FU pd in

14. The bending requirement is governing in this case. The diametral pitch and face width for the stage 2 gearset are: p d 1.5 in
1

1.5

2 pd in

2.5

FIGURE 12-29B
Graph of Face Width and Limits for the Stage 2 Pinion (surface) in Problem 12-29

F 9.375 in

MACHINE DESIGN - An Integrated Approach, 4th Ed.

12-30-1

PROBLEM 12-30
Statement: Size the third-stage spur gears in problem 12-27 for a bending factor of safety of at least 3.2 and a surface factor of safety of at least 1.7 assuming a steady torque, 25 deg pressure angle, full depth teeth, quality index of 8, and AISI 4140 steel for all gears. Pressure angle Bending factor of safety Surface factor of safety Power to be transmitted Number of pinion teeth Rotational speed of pinion

Given:

25 deg
Nfb 3.2 Nfs 1.7 H 190 kW Np 18 Reliability AGMA Quality level Life (cycles) Number of gear teeth R 0.99 Qv 8 N 10 Ng 162
7

n 24.695 rpm

Assumptions: Since both pinion and gear from each stage are the same material, it will only be necessary to determine the pinion size as it will be governing for the set. Solution: See Mathcad file P1230.

Stage 3 Pinion - Bending 1. 2. Determine the bending geometry factor, J (Table 12-13) Jp 0.38

Write the equations for pitch diameter, pitchline velocity, and transmitted load in terms of the unknown diametral pitch, p d. Note that, in Mathcad, unit conversion factors are not included. Pitch diameter of pinion and gear (in) d p p d Vt p d Wt p d Ka 1 Np pd d p p d n 2 Vt p d H d g p d Ng pd

pitchline velocity (fpm) Transmitted load (lbf) 3. 4. Set the application factor, Ka

Write the equation for the dynamic load factor, Kv B 0.25 12 Qv B 0.63
0.6667

A 50 56 ( 1 B) Kv p d A min A Vt p d ft Km 2.0
B

A 70.721

5. 6.

Tentatively choose the mounting factor, Km (Assume 9 < F < 20 in) The bending stress equation for the pinion is

bp p d F
7. Determine the endurance strength of the pinion. Life factor KL 1.6831 N

Ka Wt p d p d Km Kv p d F Jp

0.0323

KL 1

MACHINE DESIGN - An Integrated Approach, 4th Ed.


Reliability Temperature factor Material bending strength (psi) AISI 4140 Nitrided steel Endurance strength 8. S fbp S atp KL KT KR KR 0.7 0.15 log( 1 R) KR 1 KT 1

12-30-2

S atp 40000 psi S fbp 40001 psi

Write the design equations using the range of face-width to diametral-pitch ratio given in the text, and the bending stress equation, solved for the unknown face width. Upper limit FU p d FL p d S fb 16 pd 8 pd

Lower limit

Safety factor

Nfb =

b
Ka Wt p d p d Km Nfb Kv p d Jp S fbp

Face width

F p d

9.

Plot F(P) vs. p over the range


d

p d

0.5 0.6 2 in in in
40 F pd in

10. From the graph, choose a standard value of p d from Table 12-2. p d 0.75 in
1


in

30

11. The calculated value of F is F p d 18.954 in 12. Round this up to the decimal equivalent of a common fractional value. F 19.000 in 13. Then, the parameters that depend on p d and F are: d p p d 24.000 in Vt p d 155 ft min d g p d 216.000 in

FL pd

20

FU pd in


10

0 0.5

0.75

1.25 pd in

1.5

1.75

FIGURE 12-30A Wt p d 54189 lbf Kv p d 0.903


Graph of Face Width and Limits for the Stage 3 Pinion (bending) in Problem 12-30

bp p d F 12470 psi

The assumption made in step 5 is correct so no further iteration is required. Stage 3 Pinion - Surface Fatigue 1. Determine the surface geometry factor, I.

MACHINE DESIGN - An Integrated Approach, 4th Ed.


I 2.

12-30-3
I 0.1724

sin( ) cos( ) Ng N N 2 p g

Write the equations for pitch diameter, pitchline velocity, and transmitted load in terms of the unknown diametral pitch, p d. Note that, in Mathcad, unit conversion factors are not included. d p p d Vt p d Wt p d Ca 1 Np pd d p p d n 2 H

Pitch diameter of pinion (in)

pitchline velocity (fpm)

Transmitted load (lbf)

Vt p d

3. 4.

Set the application factor, Ca

Write the equation for the dynamic load factor, Cv B 0.25 12 Qv B 0.63
0.6667

A 50 56 ( 1 B) Cv p d A min A Vt p d ft Cm 2.0 Cp 2300 psi


0.5 B

A 70.721

5. 6. 7.

Tentatively choose the mounting factor, Cm (Assume 9 < F < 20 in) Choose an elastic coefficient from Table 12-18. (Steel on steel). The surface stress equation for the pinion is

cp p d F Cp
8. Determine the endurance strength of the pinion. Life factor Reliability Temperature factor Material surface strength (psi) AISI 4140 Nitrided steel Endurance strength S fcp S acp CL CT CR CL 2.466 N

Cv p d F d p p d I

C a W t p d C m

0.056

CL 1 CR 1 CT 1 S acp 165000 psi S fcp 164997 psi

CR 0.7 0.15 log( 1 R)

9.

Write the design equations using the range of face-width to diametral-pitch ratio given in the text, and the bending stress equation, solved for the unknown face width. Upper limit FU p d 16 pd

MACHINE DESIGN - An Integrated Approach, 4th Ed.


FL p d 8 pd
2

12-30-4

Lower limit

Safety factor

Sfc Nfc = c
F p d

Face width

Cp Nfs Cv p d d p p d I S fcp

C a W t p d C m

10. Plot F(P) vs. p over the range


d

p d

0.5 0.6 2 in in in
40

11. From the graph, choose a standard value of p d from Table 12-2. p d 1 in
1

F pd in


in

30

12. The calculated value of F is F p d 16.825 in 13. Round this up to the decimal equivalent of a common fractional value. F 16.875 in 14. The bending requirement is governing in this case. The diametral pitch and face width for the stage 2 gearset are: p d 0.75 in
1

FL pd

20

FU pd in


10

0 0.5

0.75

1.25 pd in

1.5

1.75

FIGURE 12-30B
Graph of Face Width and Limits for the Stage 3 Pinion (surface) in Problem 12-30

F 19.000 in

MACHINE DESIGN - An Integrated Approach, 4th Ed.

12-31-1

PROBLEM 12-31
Statement: Given: Design a two-stage compound spur gear train for an overall ratio of approximately 78:1. Specify tooth numbers for each gear in the train. Approximate ratio mG 78 Number of stages n 2

Assumptions: The gears will be cut with a hob and the pressure angle will be 25 deg Solution: 1. See Mathcad file P1231.

For equal stages, the stage ratio is


1 n

mGs mG

mGs 8.832

2.

The minimum number of teeth that we can have without interference on a 25-deg gear cut with a hob is 12. Try pinions with 12, 13, 14, etc. teeth to see if the mating gear will have close to an integral number of teeth. 12 mGs 105.981 13 mGs 114.813 14 mGs 123.645 15 mGs 132.476 16 mGs 141.308 17 mGs 150.14

3.

The first trial is very close to an integer so try 106 teeth: mG

106 12

mG 78.028

4.

We can get the exact ratio by using 12-tooth and 16-tooth pinions with 104- and 144-tooth gears Let Stage 1 Stage 2 Np1 12 Np2 16 Ng1 104 Ng2 144

Then, the gear ratio is

mG

Ng1 Ng2 Np1 Np2

mG 78.000

MACHINE DESIGN - An Integrated Approach, 4th Ed.

12-32-1

PROBLEM 12-32
Statement: Figure P12-1 shows the same paper machine that was analyzed in Problem 6-46 and in other problems in previous chapters. Using the data of Problem 4-46, design a suitable spur-gear set for this application for a 10-yr life against surface failure. State all assumptions. yr 2080 hr Gear ratio Torque on gear mg 2 Tg 74240 in lbf Life (years) Life 10 yr

Units: Given:

Assumptions: 1. If both pinion and gear are the same material, it will only be necessary to determine the pinion size as it will be governing for the set. 2. If the gears are not surface hardened, it will only be necessary to design to the surface requirement as it will be governing for both bending and surface stresses. 3. Pinion speed is n p 3 rpm (5 seconds for a 90-deg rotation). Design Choices: Pressure angle Number of pinion teeth

25 deg
Np 26

AGMA Quality level Reliability

Qv 8 R 0.99

Material: AGMA Grade 2 steel for both pinion and gear, through hardened to HB 300 S ac ( 27000 364 HB) psi S ac 136200 psi Surface factor of safety Solution: 1. See Mathcad file P1232. Nfs 2 Number of shifts shifts 3

Determine the number of gear teeth and the cycle life. Number of gear teeth Cycle life: Ng Np mg N Life shifts np 2 Ng 52 N 1.123 10
7

2.

Determine the surface geometry factor, I. I

sin( ) cos( ) Ng N N 2 p g

I 0.1277

3. Write the equations for pitch diameter, pitchline velocity, and transmitted load in terms of the unknown diametral pitch, p d. Note that, in Mathcad, unit conversion factors are not included. Pitch diameter of pinion (in) d p p d Vt p d Wt p d Ca 1
v

Np pd d p p d n p 2 d g p d 2 Tg

d g p d

Ng pd

pitchline velocity (fpm)

Transmitted load (lbf) 4. 5. Set the application factor, Ca Write the equation for the dynamic load factor, C

B 0.25 12 Qv

0.6667

B 0.63 A 70.721

A 50 56 ( 1 B)

MACHINE DESIGN - An Integrated Approach, 4th Ed.


Cv p d A min A Vt p d ft Cm 1.7 Cp 2300 psi
0.5 B

12-32-2

6. 7. 8. 9.

Tentatively choose the mounting factor, Cm (Assume 2 < F < 6 in) Choose an elastic coefficient from Table 12-18 (steel on steel). The surface stress equation for the pinion is Determine the endurance strength of the pinion. Life factor Reliability Temperature factor Material surface strength (psi) Grade 2, 300 HB Endurance strength S fcp S ac CL CT CR CL 2.466 N

cp p d F Cp

Cv p d F d p p d I

C a W t p d C m

0.056

CL 0.993 CR 1 CT 1 S ac 136200 psi S fcp 135315 psi

CR 0.7 0.15 log( 1 R)

10. Write the design equations using the range of face-width to diametral-pitch ratio given in the text, and the bending stress equation, solved for the unknown face width. Limits and safety factor FU p d F p d 16 pd C a W t p d C m FL p d 8 pd

Sfc Nfc = c

Face width

2 Cp Nfs Cv p d d p p d I S fcp

9. Plot F(P) vs. p d over the range

p d

1 1.5 10 in in in

8 F pd in FL pd in FU pd in

10. From the graph, choose a standard value of p d from Table 12-2. p d 2.5 in
1

11. The calculated value of F is F p d 5.414 in 12. Round this up to the decimal equivalent of a common fractional value. F 5.500 in 13. Then, the parameters that depend on p d and F are: d p p d 10.400 in d g p d 20.800 in

1.5

2.5 pd in

3.5

FIGURE 12-32
Graph of Face Width and Limits for the Pinion (surface) in Problem 12-32

MACHINE DESIGN - An Integrated Approach, 4th Ed.


Vt p d 8 ft min Wt p d 7138 lbf

12-32-3

cp p d F 94935 psi

14. There is a wide range of choice for the number of teeth on the pinion. The total weight of the gears goes down very slightly with increasing pinion tooth number. The choice of Np 26 was somewhat arbitrary but was arrived at after trying values from 18 to 26.

MACHINE DESIGN - An Integrated Approach, 4th Ed.

12-33-1

PROBLEM 12-33
Statement: Design a non reverted compound transmission based on the arrangement shown in Figure 12-14a for an overall train ratio of approximately 90:1. It should be capable of transmitting 50 hp at 1000 rpm input shaft speed. State all assumptions. Power to be transmitted Input speed (rpm) H 50 hp n in 1000 rpm Approximate gear ratio Number of stages mv 90 n 2

Given:

Assumptions: 1. If both pinion and gear from each stage are the same material, it will only be necessary to determine the pinion size as it will be governing for the set. 2. If the gears are not surface hardened, it will only be necessary to design to the surface requirement as it will be governing for both bending and surface stresses. Design Choices: Pressure angle Surface factor of safety

25 deg
Nfs 1.7

AGMA Quality level Reliability

Qv 8 R 0.99
7

Life (cycles) N 10 Material: AGMA Grade 2 steel for both pinion and gear, both stages, through hardened to HB 300 S ac ( 27000 364 HB) psi S ac 136200 psi Solution: 1. See Mathcad file P1233. For equal stage ratios, the stage ratio is
1 n

mvs mv 2.

mvs 9.487

The minimum number of teeth that we can have without interference on a 25-deg gear cut with a hob is 14. Try pinions with 14, 15, 16, etc. teeth to see if the mating gear will have close to an integral number of teeth. 14 mvs 132.816 15 mvs 142.302 16 mvs 151.789 17 mvs 161.276 18 mvs 170.763 19 mvs 180.25 20 mvs 189.737 21 mvs 199.223 22 mvs 208.71 23 mvs 218.197 24 mvs 227.684 25 mvs 237.171 26 mvs 246.658 27 mvs 256.144 28 mvs 265.631 29 mvs 275.118 30 mvs 284.605 31 mvs 294.092 32 mvs 303.579 33 mvs 313.065 34 mvs 322.552 35 mvs 332.039 36 mvs 341.526 37 mvs 351.013

3.

The last trial is very close to an integer so try 351 teeth: mv

351 37

mv 89.993 Ng 351 n 1 n in n 1 1000 rpm

4.

For each stage let

Np 37

Stage 1 Pinion Given: 1. Rotational speed of pinion (rpm)

Determine the surface geometry factor, I. I

sin( ) cos( ) Ng N N 2 p g

I 0.1732

MACHINE DESIGN - An Integrated Approach, 4th Ed.


2.

12-33-2

Write the equations for pitch diameter, pitch-line velocity, and transmitted load in terms of the unknown diametral pitch, p d. Note that, in Mathcad, unit conversion factors are not included. Pitch diameter of pinion (in) d p p d Vt p d Wt p d Ca 1 Np pd d p p d n 1 2 Vt p d H d g p d Ng pd

Pitch-line velocity (fpm)

Transmitted load (lbf)

3. 4.

Set the application factor, Ca

Write the equation for the dynamic load factor, Cv B 0.25 12 Qv B 0.63
0.6667

A 50 56 ( 1 B) A Cv p d min A Vt p d ft Cm 1.6 Cp 2300 psi


B

A 70.721

5. 6. 7.

Tentatively choose the mounting factor, Cm (Assume 0 < F < 2 in) Choose an elastic coefficient from Table 12-18. The surface stress equation for the pinion is

0.5

cp p d F Cp
8.

Cv p d F d p p d I

C a W t p d C m

Determine the endurance strength of the pinion. Life factor Reliability Temperature factor Material surface strength (psi) AISI 4140 Nitrided steel Endurance strength S fcp S ac CL CT CR CL 2.466 N
0.056

CL 1 CR 1 CT 1 S ac 136200 psi S fcp 136198 psi

CR 0.7 0.15 log( 1 R)

9.

Write the design equations using the range of face-width to diametral-pitch ratio given in the text, and the bending stress equation, solved for the unknown face width. Upper limit FU p d FL p d 16 pd 8 pd

Lower limit

MACHINE DESIGN - An Integrated Approach, 4th Ed.

12-33-3

Safety factor

Sfc Nfc = c
F p d

Face width

2 Cp Nfs Cv p d d p p d I S fcp

C a W t p d C m

10.

Plot F(P) vs. p over the range


d

p d

6 6.5 16 in in in

11.

From the graph, choose a standard value of p d from Table 12-2. p d 8 in


1 F pd in FL pd in

12.

The calculated value of F is F p d 1.697 in

13.

Round this up to the decimal equivalent of a common fractional value. F 1.750 in

FU pd 1 in

14.

Then, the parameters that depend on p d and F are: d p p d 4.625 in Vt p d 1211 ft min d g p d 43.875 in Wt p d 1363 lbf
0 6 8 10 12 pd in 14 16

FIGURE 12-33A
Graph of Face Width and Limits for the Stage 1 Pinion (surface) in Problem 12-33

cp p d F 102877 psi
Stage 2 Pinion Given: 1. Rotational speed of pinion (rpm) n 2

Np Ng

n in

n 2 105.413 rpm

Write the equations for pitch diameter, pitch-line velocity, and transmitted load in terms of the unknown diametral pitch, p d. Note that, in Mathcad, unit conversion factors are not included. Pitch diameter of pinion (in) d p p d Vt p d Wt p d Np pd d p p d n 2 2 Vt p d H

Pitch-line velocity (fpm) Transmitted load (lbf) 2. Write the equation for the dynamic load factor, Cv Cv p d A min A Vt p d ft
B

MACHINE DESIGN - An Integrated Approach, 4th Ed.


3. 4. Tentatively choose the mounting factor, Cm (Assume 2 < F < 6 in) The surface stress equation for the pinion is Cm 1.7

12-33-4

cp p d F Cp
5.

Cv p d F d p p d I

C a W t p d C m

Write the design equations using the range of face-width to diametral-pitch ratio given in the text, and the bending stress equation, solved for the unknown face width. Upper limit FU p d FL p d 16 pd 8 pd
2

Lower limit

Safety factor

Sfc Nfc = c
F p d

Face width

2 Cp Nfs Cv p d d p p d I S fcp

C a W t p d C m

6.

Plot F(P) vs. p over the range


d

p d

2 2.5 12 in in in
8

7.

From the graph, choose a standard value of p d from Table 12-2. p d 4 in


1

F pd in


in 5.333

8.

The calculated value of F is F p d 3.779 in

FL pd

FU pd 2.667 in

9.

Round this up to the decimal equivalent of a common fractional value. F 3.750 in

10. Then, the parameters that depend on p d and F are: d p p d 9.250 in Vt p d 255 ft min d g p d 87.750 in Wt p d 6464 lbf

5 pd in

FIGURE 12-33B
Graph of Face Width and Limits for the Stage 2 Pinion (surface) in Problem 12-33

cp p d F 104866 psi

MACHINE DESIGN - An Integrated Approach, 4th Ed.

12-34-1

PROBLEM 12-34
Statement: Design a reverted compound transmission based on the arrangement shown in Figure 12-14b for an overall train ratio of approximately 80:1. It should be capable of transmitting 30 hp at 1500 rpm input shaft speed. State all assumptions. Power to be transmitted Input speed (rpm) H 30 hp n in 1500 rpm Approximate gear ratio Number of stages mv 80 n 2

Given:

Assumptions: 1. If both pinion and gear from each stage are the same material, it will only be necessary to determine the pinion size as it will be governing for the set. 2. If the gears are not surface hardened, it will only be necessary to design to the surface requirement as it will be governing for both bending and surface stresses. 3. If both stages have the same tooth numbers, then both must have the same diametral pitch in order that the center distances be the same. In this case, the dimensions of the second stage will govern the design. Design Choices: Pressure angle Surface factor of safety

25 deg
Nfs 1.7

AGMA Quality level Reliability

Qv 8 R 0.99
7

Life (cycles) N 10 Material: AGMA Grade 2 steel for both pinion and gear, both stages, through hardened to HB 300 S ac ( 27000 364 HB) psi S ac 136200 psi Solution: 1. See Mathcad file P1234.

For equal stage ratios, the stage ratio is


1 n

mvs mv 2.

mvs 8.944

The minimum number of teeth that we can have without interference on a 25-deg gear cut with a hob is 14. Try pinions with 14, 15, 16, etc. teeth to see if the mating gear will have close to an integral number of teeth. 14 mvs 125.22 15 mvs 134.164 16 mvs 143.108 17 mvs 152.053 18 mvs 160.997 19 mvs 169.941 20 mvs 178.885 21 mvs 187.83 22 mvs 196.774 23 mvs 205.718 24 mvs 214.663 25 mvs 223.607 26 mvs 232.551 27 mvs 241.495 28 mvs 250.44 29 mvs 259.384 30 mvs 268.328 31 mvs 277.272 32 mvs 286.217 33 mvs 295.161 34 mvs 304.105 35 mvs 313.05 36 mvs 321.994 37 mvs 330.938

3.

The next to last trial is very close to an integer so try 322 teeth: mv

322 36

mv 80.003 Ng 322 Np Ng

4.

For each stage let

Np 36

Stage 2 Pinion Given: Rotational speed of pinion (rpm) n 2 n in n 2 167.702 rpm

MACHINE DESIGN - An Integrated Approach, 4th Ed.


1. Determine the surface geometry factor, I. I 2.

12-34-2

sin( ) cos( ) Ng N N 2 p g

I 0.1723

Write the equations for pitch diameter, pitch-line velocity, and transmitted load in terms of the unknown diametral pitch, p d. Note that, in Mathcad, unit conversion factors are not included. Pitch diameter of pinion (in) d p p d Vt p d Wt p d Ca 1 Np pd d p p d n 2 2 Vt p d H d g p d Ng pd

Pitch-line velocity (fpm)

Transmitted load (lbf)

3. 4.

Set the application factor, Ca

Write the equation for the dynamic load factor, Cv B 0.25 12 Qv B 0.63
0.6667

A 50 56 ( 1 B) Cv p d A min A Vt p d ft Cm 1.7 Cp 2300 psi


B

A 70.721

5. 6. 7.

Tentatively choose the mounting factor, Cm (Assume 2 < F < 6 in) Choose an elastic coefficient from Table 12-18. The surface stress equation for the pinion is

0.5

cp p d F Cp
8.

Cv p d F d p p d I

C a W t p d C m

Determine the endurance strength of the pinion. Life factor Reliability Temperature factor Material surface strength (psi) AISI 4140 Nitrided steel Endurance strength S fcp S ac CL CT CR CL 2.466 N
0.056

CL 1 CR 1 CT 1 S ac 136200 psi S fcp 136198 psi

CR 0.7 0.15 log( 1 R)

9.

Write the design equations using the range of face-width to diametral-pitch ratio given in the text, and the bending stress equation, solved for the unknown face width. Upper limit FU p d 16 pd

MACHINE DESIGN - An Integrated Approach, 4th Ed.


FL p d

12-34-3

Lower limit

8 pd
2

Safety factor

Nfc =

Sfc c

Face width

Cp F p d Nfs Cv p d d p p d I S fcp
p d 2 2.1 12 in in in
4 F pd in FL pd in FU pd in

C a W t p d C m

10. Plot F(P) vs. p over the range


d

11. From the graph, choose a standard value of p d from Table 12-2. p d 5 in
1

12. The calculated value of F is F p d 2.397 in 13. Round this up to the decimal equivalent of a common fractional value. F 2.375 in 14. Then, the parameters that depend on p d and F are: d p p d 7.200 in Vt p d 316 ft min d g p d 64.400 in

7 pd in

9 10 11 12

FIGURE 12-34A Wt p d 3132 lbf


Graph of Face Width and Limits for the Stage 2 Pinion (surface) in Problem 12-34

cp p d F 104942 psi
15. This diametral pitch and face width should be used for both stages in the gearset.

MACHINE DESIGN - An Integrated Approach, 4th Ed.

12-35-1

PROBLEM 12-35
Statement: Given: Solution: 1.

_____

If the 23-tooth pinion in the gearset in Problem 12-3 is on the input shaft, find the velocity ratio, torque ratio, and gear ratio for the gearset. Input tooth number See Mathcad file P1235. Nin 23 Output tooth number Nout 57

Calculate the velocity ratio using equation 12.1a. Note that, for a given diametral pitch, the pitch radius of a gear is directly proportional to the number of teeth on the gear. mV Nin Nout mV 0.404

2.

Calculate the torque ratio using equation 12.1b. mA 1 mV mA 2.478

3.

Calculate the gear ratio using equation 12.1c. mG mA mG 2.478

MACHINE DESIGN - An Integrated Approach, 4th Ed.

12-36-1

PROBLEM 12-36
Statement: Given: Solution: 1.

_____

If the 78-tooth gear in the gearset in Problem 12-4 is on the input shaft, find the velocity ratio, torque ratio, and gear ratio for the gearset. Input tooth number See Mathcad file P1236. Nin 78 Output tooth number Nout 27

Calculate the velocity ratio using equation 12.1a. Note that, for a given diametral pitch, the pitch radius of a gear is directly proportional to the number of teeth on the gear. mV Nin Nout mV 2.889

2.

Calculate the torque ratio using equation 12.1b. mA 1 mV mA 0.346

3.

Calculate the gear ratio using equation 12.1c. mG mV mG 2.889

MACHINE DESIGN - An Integrated Approach, 4th Ed.

12-37-1

PROBLEM 12-37
Statement:

_____

Figure P12-2 shows an involute of a circle that starts at point A(0, rb) and continues to point P(x,y) The angle is known as the roll angle and is the involute pressure angle. Derive expressions for the x,y coordinates of P in terms of the base circle radius rb and the involute pressure angle only. Plot y vs. x over the range 0 < < 40 deg for rb = 2 in. Base circle radius rb 2.00 in See Figure P12-2 and Mathcad file P1237.

Given: Solution: 1.

From the definition of the involute, the distance from P to Q is the arc-length along the base circle subtended by the angle . Thus, PQ rb

2.

The distance OP = rp is then rp rb rb


2 2 2

rp rb 1

3.

Looking at triangle OPQ, we see that rb rb

tan ( )

tan( )

4.

Writing the x and y coordinates of P in terms of the above relationships, we have


2

x( ) rb 1 tan ( ) sin( tan ( ) )

y ( ) rb 1 tan( ) cos( tan( ) ) 5. Plot y vs. x over the range 0 deg 1 deg 40 deg
2.6

2.4 y ( ) in 2.2

0.1

0.2 x( ) in

0.3

0.4

MACHINE DESIGN - An Integrated Approach, 4th Ed.

12-38-1

PROBLEM 12-38
Statement: Solution: 1. Derive equation 12.2 using Figure 12-5. See Figure 12-5 and Mathcad file P1238.

_____

Label points on Figure 12-5 as follows: Center of pinion, Op Center of gear, Og Beginning of contact, E Pitch point, P Leaving contact, F Extend line of action to tangency with pinion base circle, A (not shown on figure) Extend line of action to tangency with gear base circle, B (not shown on figure).

2. 3. 4. 5.

The length of action, Z, is equal to the distance EF = EP + PF. The distance PF = AF - AP and EP = BE - BP. Thus, Z = AF + BE - (AP + BP). The distance OpF = rp + a p, and OgE = rg + a g, where a p is the pinion addendum and a g is the gear addendum . Since the line of action is tangent to both base circles, angle AOpP = angle BOgP = , the gear pressure angle. Also, the distance OpA = rbp = rpcos() and OgB = rbg = rgcos().

6.

From the relationships above,

AF

rp ap2 rp cos( ) 2

BE

rg ag2 rg cos( ) 2

AP rp sin( ) 7.

BP rg sin( )

Substituting the equations in step 6 into the equation for Z in step 3,

rp ap2 rp cos( ) 2 rg ag 2 rg cos( ) 2 rp rg sin( )

But, rp + rg = C, the center distance. So,

rp ap2 rp cos( ) 2 rg ag 2 rg cos( ) 2 C sin( )

which is equations 12-2.

MACHINE DESIGN - An Integrated Approach, 4th Ed.

12-39-1

PROBLEM 12-39
Statement:

_____

A 39-tooth spur gear is in mesh with an 18-tooth pinion. The p d = 8 and = 25 deg. Find the contact ratio. Tooth numbers: Pinion Diametral pitch Np 18
1

Given:

Gear Pressure angle

Ng 39

p d 8 in

25 deg

Solution:

See Mathcad file P1239. Circular pitch p c

pd

p c 0.393 in p b 0.356 in

Base pitch Pinion: Pitch dia Pitch rad Gear: Pitch dia Pitch rad

p b p c cos( )

d p

Np pd

d p 2.250 in rp 1.125 in

rp 0.5 d p

d g

Ng pd

d g 4.875 in rg 2.438 in a 0.125 in

rg 0.5 d g a 1.0 pd Np Ng 2 pd

Addendum

Center distance

C 3.563 in

Length of action

rp a 2 rp cos( ) 2 rg a 2 rg cos( ) 2 C sin( )

Z 0.516 in Contact ratio mp Z pb mp 1.450

MACHINE DESIGN - An Integrated Approach, 4th Ed.

12-40-1

PROBLEM 12-40
Statement:

_____

A 79-tooth spur gear is in mesh with a 20-tooth pinion. The p d = 8 and = 20 deg. Find the contact ratio. Tooth numbers: Pinion Diametral pitch Np 20 p d 8 in
1

Given:

Gear Pressure angle

Ng 79

20 deg

Solution:

See Mathcad file P1240. Circular pitch p c

pd

p c 0.393 in p b 0.369 in

Base pitch Pinion: Pitch dia Pitch rad Gear: Pitch dia Pitch rad

p b p c cos( )

d p

Np pd

d p 2.500 in rp 1.250 in

rp 0.5 d p

d g

Ng pd

d g 9.875 in rg 4.938 in a 0.125 in

rg 0.5 d g a 1.0 pd Np Ng 2 pd

Addendum

Center distance

C 6.188 in

Length of action

rp a 2 rp cos( ) 2 rg a 2 rg cos( ) 2 C sin( )

Z 0.624 in Contact ratio mp Z pb mp 1.690

MACHINE DESIGN - An Integrated Approach, 4th Ed.

12-41-1

PROBLEM 12-41
Statement:

_____

What will the pressure angle be if the center distance of the spur gearset in Problem 12-39 is increased by 6%? Tooth numbers: Pinion Diametral pitch Np 18 p d 8 in
1

Given:

Gear Pressure angle

Ng 39

25 deg

Solution: 1.

See Figure 12-41 and Mathcad file P1241.

Define the factor by which the center distance changes. Let C C C

new
= fc

Base Circle Old Pitch Circle New Pitch Circle rbp C + C

Dividing numerator and denominator of the left side by C, C fc = 1 C 2. 3. In this case, fc 1 0.06 fc 1.060

fc rp

fc r g

rbg

Determine the pitch diameter and pitch radius of the pinion. Pitch diameter d p Np pd FIGURE 12-41 Pitch radius rp 0.5 d p rp 1.125 in
Diagram Showing Center Change for Problem 12-41

d p 2.250 in

4.

When the centers are moved apart the base circle diameters don't change but new pitch circle diameters are defined by the intersection of the new line of action, which is always tangent to the two base circles, with the line of centers. The new pitch radii are proportional to f c. Thus, from Figure 12-41, the new pressure angle is cos new = rbp fc rp

Substituting rp cos( ) for rbp and solving for new,

new acos

rp cos( ) fc rp

new 31.24 deg

MACHINE DESIGN - An Integrated Approach, 4th Ed.

12-42-1

PROBLEM 12-42
Statement:

_____

What will the pressure angle be if the center distance of the spur gearset in Problem 12-40 is increased by 5%? Tooth numbers: Pinion Diametral pitch Np 20 p d 8 in
1

Given:

Gear Pressure angle

Ng 79

20 deg

Solution: 1.

See Figure 12-42 and Mathcad file P1242.

Define the factor by which the center distance changes. Let C C C = fc

new

Base Circle Old Pitch Circle New Pitch Circle rbp C + C

Dividing numerator and denominator of the left side by C, C fc = 1 C 2. 3. In this case, fc 1 0.05 fc 1.050

fc rp

fc r g

rbg

Determine the pitch diameter and pitch radius of the pinion. Pitch diameter d p Np pd FIGURE 12-42
Diagram Showing Center Change for Problem 12-42

d p 2.500 in Pitch radius rp 0.5 d p rp 1.250 in 4.

When the centers are moved apart the base circle diameters don't change but new pitch circle diameters are defined by the intersection of the new line of action, which is always tangent to the two base circles, with the line of centers. The new pitch radii are proportional to f c. Thus, from Figure 12-41, the new pressure angle is cos new = rbp fc rp

Substituting rp cos( ) for rbp and solving for new,

new acos

rp cos( ) fc rp

new 26.50 deg

MACHINE DESIGN - An Integrated Approach, 4th Ed.

12-43-1

PROBLEM 12-43
Statement: Given: If the spur gearsets in Problems 12-39 and 12-40 are compounded as shown in Figure 12-14, what will the overall train ratio be? Tooth numbers N2 18 N4 20 Solution: 1. See Mathcad file P1243. N3 39 N5 79

Using equation 12.9b and assuming that gears 2 and 4 are the driver gears, the train (velocity) ratio is mv N2 N4 N3 N5 mv 0.117

2.

On the other hand, if gears 3 and 5 are the driver gears then the train ratio is mv N3 N5 N2 N4 mv 8.5583

MACHINE DESIGN - An Integrated Approach, 4th Ed.

12-44-1

PROBLEM 12-44
Statement: Given:

_____

A 20-deg pressure angle, 23-tooth spur gear has a diametral pitch of 6. Find the pitch diameter, addendum, dedendum, outside diameter, and circular pitch. Number of teeth Diametral pitch N 23 p d 6 in
1

Pressure angle

20 deg

Solution:

See Mathcad file P1244. Pitch diameter d N pd 1.0 pd 1.25 pd d 3.833 in

Addendum

a b

a 0.167 in b 0.208 in Do 4.167 in p c 0.524 in

Dedendum

Outside diameter

Do d 2 a p c

Circular pitch

pd

MACHINE DESIGN - An Integrated Approach, 4th Ed.

12-45-1

PROBLEM 12-45
Statement: Given:

_____

A 25-deg pressure angle, 32-tooth spur gear has a diametral pitch of 4. Find the pitch diameter, addendum, dedendum, outside diameter, and circular pitch. Number of teeth Diametral pitch N 32 p d 4 in
1

Pressure angle

20 deg

Solution:

See Mathcad file P1245. Pitch diameter d N pd 1.0 pd 1.25 pd d 8.000 in

Addendum

a b

a 0.250 in b 0.313 in Do 8.500 in p c 0.785 in

Dedendum

Outside diameter

Do d 2 a p c

Circular pitch

pd

MACHINE DESIGN - An Integrated Approach, 4th Ed.

12-46-1

PROBLEM 12-46
Statement:

_____

Design a two-stage compound spur gear train for an overall ratio of approximately 53:1. Specify tooth numbers for each gear in the train. Approximate ratio mG 53 Number of stages n 2

Given:

Assumptions: The gears will be cut with a hob and the pressure angle will be 25 deg Solution: 1. See Mathcad file P1246.

For equal stage ratios, the stage ratio is approximately


1 n

mGs mG 2.

mGs 7.280

The minimum number of teeth that we can have without interference on a 25-deg gear cut with a hob is 14. Try pinions with 14, 15, 16, etc. teeth to see if the mating gear will have close to an integral number of teeth. 14 mGs 101.922 15 mGs 109.202 16 mGs 116.482

3.

The first trial is very close to an integer so try 102 teeth: mG

102 14

mG 53.082

4.

We can get slightly closer by using two 14-tooth pinions with 101 and 103-tooth gears mG 101 103 14 14 mG 53.077

5.

However, it may be harder to find the 101 and 103-tooth gears and, it may be less expensive to have two gears with the same number of teeth than to have different numbers of teeth.

MACHINE DESIGN - An Integrated Approach, 4th Ed.

12-47-1

PROBLEM 12-47
Statement:

_____

Design a three-stage compound spur gear train for an overall ratio of approximately 592:1. Specify tooth numbers for each gear in the train. Approximate ratio See Mathcad file P1247.
1

Given: Solution:

mG 592

Number of stages

s 3

1. 2.

The average stage ratio is

ravg mG

ravg 8.397

If the minimum number of teeth is (for a 20 deg pressure angle) Nmin 18 then the number of teeth on the driven gear for each stage is NG1 floor ravg Nmin or NG2 ceil ravg Nmin NG1 151 NG2 152

3.

But, the prime factors of 592 are 2 4 and 37. This suggests that one of the gears have a number of teeth that is a multiple of 37, say 148. 148 ravg 17.626 Let N2 18 N3 148

Thus,

N5 N7 N4 N6

= 72
5 6

if

N4 18

and

N6 18

then Try

N5 N7 = 72 18 18 = 2 3 N5 2 3
4 2

N7 2 3 N2 18 N4 18 N6 18

1 4

then

N5 N7 23328

Summarizing,

N3 148 N5 144 N7 162

4. 5.

Since the driven gears all have less than 180 teeth, no stage ratio is greater than 10. Checking the overall gear ratio mG N 3 N 5 N 7 N 2 N 4 N 6 mG 592.00

MACHINE DESIGN - An Integrated Approach, 4th Ed.

12-48-1

PROBLEM 12-48
Statement:

_____

Design a planetary gear train similar to that shown in Figure 12-16 for an overall velocity ratio of exactly 0.2 if the sun gear is the input, the arm is the output, and the ring gear is stationary. Specify tooth numbers for each gear in the train. Exact ratio mv 0.2

Given:

Assumptions: The gears will be cut with a hob and the pressure angle will be 20 deg Solution: 1. See Figure 12-16 and Mathcad file P1248.

Let the first gear be the sun gear (gear 2) and the last be the ring gear (gear 4). Then, equation 12.11c can be written as

4 arm 2 arm
2.

N2 N4

Taking 4 = 0 and solving for out/in, we have

out in
3.

arm 2

N2 N2 N4

= mv

Let the sun gear have tooth number 1 mv mv

N2 20 , then the number of teeth required for the ring gear is N4 80

N4

N2

4.

In order to mesh properly, the number of teeth on the ring gear must be equal to the sum of the number of teeth on the sun gear plus two times the number of teeth on the planet gear(s). Thus, N3 N4 N2 2 N3 30

5.

Check the overall velocity ratio and summarize the tooth numbers.

Overall velocity ratio

mv

N2 N2 N4

mv 0.200

Sun gear Planet gear(s) Ring gear

N2 20 N3 30 N4 80

MACHINE DESIGN - An Integrated Approach, 4th Ed.

12-49-1

PROBLEM 12-49
Statement:

_____

Design a planetary gear train similar to that shown in Figure 12-16 for an overall velocity ratio of exactly 4/3 if the sun gear is stationary, the arm is the input, and the ring gear is the output. Specify tooth numbers for each gear in the train. Exact ratio mv 4 3

Given:

Assumptions: The gears will be cut with a hob and the pressure angle will be 20 deg Solution: 1. See Figure 12-16 and Mathcad file P1249.

Let the first gear be the sun gear (gear 2) and the last be the ring gear (gear 4). Then, equation 12.11c can be written as

4 arm 2 arm
2.

N2 N4

Taking 2 = 0 and solving for out/in, we have

out in
3.

4 arm

N4 N2 N4

= mv

Let the sun gear have tooth number N2 mv 1

N2 20 , then the number of teeth required for the ring gear is N4 60

N4

4.

In order to mesh properly, the number of teeth on the ring gear must be equal to the sum of the number of teeth on the sun gear plus two times the number of teeth on the planet gear(s). Thus, N3 N4 N2 2 N3 20

5.

Check the overall velocity ratio and summarize the tooth numbers. N4 N2 N4

Overall velocity ratio

mv

mv 1.333

Sun gear Planet gear(s) Ring gear

N2 20 N3 20 N4 60

MACHINE DESIGN - An Integrated Approach, 4th Ed.

12-50-1

PROBLEM 12-50
Statement:

_____

A 21-tooth pinion rotating at 1800 rpm meshes with a 33-tooth gear in a spur gear reducer. Both pinion and gear are manufactured to a quality level of 9. A reliability of 0.9 has been specified, and the transmitted tangential load is 2800 lb. Conditions are such that Km = 1.7. It is proposed that standard 25-degree, full-depth teeth be used, with both pinion and gear hobbed from an AISI 4140 nitrided steel. The diametral pitch is 6, and the face width 2.000 in. Estimate the number of cycles of bending stress (using the AGMA equations) that the gearset can withstand. Tooth numbers Pinion speed Diametral pitch Face width Tangential load Np 21 n 1800 rpm p d 6 in
1

Given:

Ng 33 Mounting factor Quality index Pressure angle Reliability Km 1.7 Qv 9

F 2.000 in Wt 2800 lbf

25 deg
R 0.90

Assumptions: The life of the pinion will be less than that of the gear. Solution: 1. See Mathcad file P1250. d n 2

Calculate the pitch diameter of the pinion using equation 12.4a and the pitch-line velocity of the gearset. d Np pd d 3.500 in V V 1649 ft min

2.

Calculate the dynamic factor Kv using equations 12.16 and 12.17. B 0.25 12 Qv Kv
0.6667 B

B 0.520

A 50 56 ( 1 B) Kv 0.802

A 76.878

A V min A ft

3. 4. 5.

Determine the bending geometry factor, J (Table 12-13) Assume an application factor, Ka Ka 1

J 0.40

Calculate the bending stress using equation 12.15.

b
6. 7. 8. 9.

Ka Wt p d Km Kv F J

b 44.51 ksi
S'fb 39.5 ksi

Determine the material bending fatigue strength from Table 12-20 using the average value. Determine the reliability factor for R 0.90 Assume a temperature factor, KT KT 1 from Table 12-19. KR 0.85

Set the bending stress equal to the corrected bending-fatigue strength in equation 12.24 and solve for the life factor, KL . KL KT KR b S'fb KL 0.958

10. Assume that the fatigue life is over 10 6 cycles and use the lower curve in Figure 12-24 to solve for the cycle life
1 0.0323

KL = 1.6831 N

1.6831 K L

0.0323

N 3.80 10

MACHINE DESIGN - An Integrated Approach, 4th Ed.

12-51-1

PROBLEM 12-51
Statement:

_____

A 21-tooth pinion rotating at 1800 rpm meshes with a 33-tooth gear in a spur gear reducer. Both pinion and gear are manufactured to a quality level of 9. A reliability of 0.9 has been specified, and the transmitted tangential load is 2800 lb. Conditions are such that Km = 1.7. It is proposed that standard 25-degree, full-depth teeth be used, with both pinion and gear hobbed from an AISI 4140 nitrided steel. The diametral pitch is 6, and the face width 2.000 in. Estimate the number of cycles of contact (surface) stress (using the AGMA equations) that the gearset can withstand. Tooth numbers Pinion speed Diametral pitch Face width Tangential load Np 21 n 1800 rpm p d 6 in
1

Given:

Ng 33 Mounting factor Quality index Pressure angle Reliability Cm 1.7 Qv 9

F 2.000 in Wt 2800 lbf

25 deg
R 0.90

Assumptions: The life of the pinion will be less than that of the gear. Solution: 1. See Mathcad file P1251. d n 2

Calculate the pitch diameter of the pinion using equation 12.4a and the pitch-line velocity of the gearset. d Np pd d 3.500 in V V 1649 ft min A 76.878

2.

Calculate the dynamic factor Kv using equations 12.16 and 12.17. B 0.25 12 Qv Cv
0.6667 B

B 0.520

A 50 56 ( 1 B) Cv 0.802

A V min A ft

3.

Calculate the geometry factor, I I

sin( ) cos( ) Ng N N 2 p g

I 0.117 Cp 2300 psi


0.5

4. 5. 6.

Determine the elastic coefficient from Table 12-18. Assume an application factor, Ca Ca 1

Calculate the contact stress using equation 12.21.


1 2 Ca Wt 1 Cm Cv F d I

c Cp
7. 8. 9.

c 195.76 ksi
S'fc 167.5 ksi

Determine the material surface-fatigue strength from Table 12-21 using the average value. Determine the reliability factor for R 0.90 Assume a temperature factor, CT CT 1 from Table 12-19. CR 0.85

10. Set the contact stress equal to the corrected surface-fatigue strength in equation 12.25 and solve for the life factor, CL .

MACHINE DESIGN - An Integrated Approach, 4th Ed.

12-51-2

CL

CT CR c S'fc

CL 0.993

11. Use the lower curve in Figure 12-26 to solve for the cycle life.
1

CL = 2.466 N

0.056

2.466 C L

0.056

N 1.13 10

MACHINE DESIGN - An Integrated Approach, 4th Ed.

12-52-1

PROBLEM 12-52
Statement: Given: If the gearset in Problem 12-46 transmits 7.5 kW at 1750 input pinion rpm, find the torque on each of the three shafts. Transmitted power P 7.5 kW Input speed

p1 1750 rpm

Assumptions: Tooth numbers: Stage 1 Stage 2 Solution: 1. Np1 14 Np2 14 Ng1 101 Ng2 103

See Mathcad file P1252.

Confirm that the gear ratio is correct. mG Ng1 Ng2 Np1 Np2 mG 53.077

2.

Calculate the gear speeds. Stage 1

p1 1750 rpm

g1

Np1 Ng1 Np2 Ng2

p1

g1 242.574 rpm

Stage 2 3. Calculate the shaft torque Shaft 1

p2 g1

g2

p2

g2 32.971 rpm

T1

p1
P

T1 40.9 N m

Input

Shaft 2

T2

g1
P

T2 295 N m

Shaft 3

T3

g2

T3 2172 N m

Output

MACHINE DESIGN - An Integrated Approach, 4th Ed.

12-53-1

PROBLEM 12-53
Statement: Size the first-stage spur gears in problem 12-52 for a bending factor of safety of at least 2.8 and a surface factor of safety of at least 1.8 assuming a steady torque, 25-deg pressure angle, full depth teeth, Qv = 9, and AISI 4340 steel for all gears. Pressure angle Bending factor of safety Surface factor of safety Power to be transmitted (hp) Number of pinion teeth

Given:

25 deg
Nfb 2.8 Nfs 1.8 H 10 hp Np 14 Reliability AGMA Quality level Life (cycles) Number of gear teeth R 0.99 Qv 9 N 10
7

Ng 101

Rotational speed of pinion (rpm) n 1750 rpm Assumptions: Since both pinion and gear from each stage are the same material, it will only be necessary to determine the pinion size as it will be governing for the set. Solution: See Mathcad file P1253. Jp 0.35

Stage 1 Pinion - Bending 1. 2. Determine the bending geometry factor, J (Table 12-13)

Write the equations for pitch diameter, pitchline velocity, and transmitted load in terms of the unknown diametral pitch, pd. Note that, in Mathcad, unit conversion factors are not included. Pitch diameter of pinion and gear (in) d p p d Vt p d Wt p d Ka 1 Np pd d p p d n 2 Vt p d H d g p d Ng pd

pitchline velocity (fpm) Transmitted load (lbf) 3. 4. Set the application factor, Ka

Write the equation for the dynamic load factor, Kv B 0.25 12 Qv B 0.52
0.6667

A 50 56 ( 1 B) Kv p d A min A Vt p d ft Km 1.6 Ka Wt p d p d Km Kv p d F Jp
B

A 76.878

5. 6.

Tentatively choose the mounting factor, Km (Assume 0 < F < 2 in) The bending stress equation for the pinion is

bp p d F
7. Determine the endurance strength of the pinion. Life factor Reliability KL 1.6831 N

0.0323

KL 1 KR 1

KR 0.7 0.15 log( 1 R)

MACHINE DESIGN - An Integrated Approach, 4th Ed.


Temperature factor Material bending strength (psi) AISI 4340 Nitrided steel Endurance strength 8. S fbp S atp KL KT KR KT 1

12-53-2

S atp 41500 psi S fbp 41501 psi

Write the design equations using the range of face-width to diametral-pitch ratio given in the text, and the bending stress equation, solved for the unknown face width. Upper limit FU p d FL p d S fb 16 pd 8 pd

Lower limit

Safety factor

Nfb =

b
Ka Wt p d p d Km Nfb Kv p d Jp S fbp

Face width

F p d

9.

Plot F(P) vs. p over the range


d

p d

5 5.1 10 in in in
4 F pd in

10. From the graph, choose a standard value of p d from Table 12-2. p d 8 in
1


in

11. The calculated value of F is F p d 1.195 in 12. Round this up to the decimal equivalent of a common fractional value. F 1.250 in 13. Then, the parameters that depend on p d and F are: d p p d 1.750 in Vt p d 802 ft min d g p d 12.625 in Wt p d 412 lbf Kv p d 0.850

FL pd

FU pd in

7 pd in

10

FIGURE 12-53A Graph of Face Width and Limits for the Stage 1 Pinion (bending) in Problem 12-53

bp p d F 14175 psi

The assumption made in step 5 is correct so no further iteration is required. Stage 1 Pinion - Surface Fatigue 1. Determine the surface geometry factor, I. I

sin( ) cos( ) Ng N N 2 p g

I 0.1682

MACHINE DESIGN - An Integrated Approach, 4th Ed.

12-53-3

2.

Write the equations for pitch diameter, pitchline velocity, and transmitted load in terms of the unknown diametral pitch, pd. Note that, in Mathcad, unit conversion factors are not included. Pitch diameter of pinion (in) d p p d Vt p d Wt p d Ca 1 Np pd d p p d n 2 H

pitchline velocity (fpm)

Transmitted load (lbf)

Vt p d

3. 4.

Set the application factor, Ca

Write the equation for the dynamic load factor, Cv B 0.25 12 Qv B 0.52
0.6667

A 50 56 ( 1 B) Cv p d A min A Vt p d ft Cm 1.6 Cp 2300 psi


0.5 B

A 76.878

5. 6. 7.

Tentatively choose the mounting factor, Cm (Assume 0 < F < 2 in) Choose an elastic coefficient from Table 12-18. (Steel on steel). The surface stress equation for the pinion is

cp p d F Cp
8. Determine the endurance strength of the pinion. Life factor Reliability Temperature factor Material surface strength (psi) AISI 4340 Nitrided steel Endurance strength 9. S fcp S acp CL CT CR CL 2.466 N

Cv p d F d p p d I

C a W t p d C m

0.056

CL 1 CR 1 CT 1 S acp 162500 psi S fcp 162497 psi

CR 0.7 0.15 log( 1 R)

Write the design equations using the range of face-width to diametral-pitch ratio given on page 740 of the text, and the bending stress equation, solved for the unknown face width. Upper limit FU p d FL p d 16 pd 8 pd

Lower limit

MACHINE DESIGN - An Integrated Approach, 4th Ed.

12-53-4

Safety factor

Sfc Nfc = c
F p d

Face width

2 Cp Nfs Cv p d d p p d I S fcp

C a W t p d C m

10. Plot F(P) vs. p over the range


d

p d

5 5.1 10 in in in
4 F pd in FL pd in FU pd in

11. From the graph, choose a standard value of p d from Table 12-2. p d 8 in
1

12. The calculated value of F is F p d 0.950 in 13. Round this up to the decimal equivalent of a common fractional value. F 1.000 in 14. The bending requirement is governing in this case. The diametral pitch and face width for the stage 1 gearset are: p d 8 in
1

7 pd in

10

FIGURE 12-53B F 1.250 in Graph of Face Width and Limits for the Stage 1 Pinion (surface) in Problem 12-53

MACHINE DESIGN - An Integrated Approach, 4th Ed.

12-54-1

PROBLEM 12-54
Statement: Size the second-stage spur gears in problem 12-52 for a bending factor of safety of at least 2.8 and a surface factor of safety of at least 1.8 assuming a steady torque, 25-deg pressure angle, full depth teeth, Qv = 9, and AISI 4340 steel for all gears. Pressure angle Bending factor of safety Surface factor of safety Power to be transmitted (hp) Number of pinion teeth

Given:

25 deg
Nfb 2.8 Nfs 1.8 H 10 hp Np 14 Reliability AGMA Quality level Life (cycles) Number of gear teeth R 0.99 Qv 9 N 10
7

Ng 103

Rotational speed of pinion (rpm) n 242.574 rpm Assumptions: Since both pinion and gear from each stage are the same material, it will only be necessary to determine the pinion size as it will be governing for the set. Solution: See Mathcad file P1254.

Stage 1 Pinion - Bending 1. 2. Determine the bending geometry factor, J (Table 12-13) Jp 0.35

Write the equations for pitch diameter, pitchline velocity, and transmitted load in terms of the unknown diametral pitch, pd. Note that, in Mathcad, unit conversion factors are not included. Pitch diameter of pinion and gear (in) d p p d Vt p d Wt p d Ka 1 Np pd d p p d n 2 Vt p d H d g p d Ng pd

pitchline velocity (fpm) Transmitted load (lbf) 3. 4. Set the application factor, Ka

Write the equation for the dynamic load factor, Kv B 0.25 12 Qv B 0.52
0.6667

A 50 56 ( 1 B) Kv p d A min A Vt p d ft Km 1.7
B

A 76.878

5. 6.

Tentatively choose the mounting factor, Km (Assume 1.99 < F < 6 in) The bending stress equation for the pinion is

bp p d F
7. Determine the endurance strength of the pinion. Life factor KL 1.6831 N

Ka Wt p d p d Km Kv p d F Jp

0.0323

KL 1

MACHINE DESIGN - An Integrated Approach, 4th Ed.


Reliability Temperature factor Material bending strength (psi) AISI 4340 Nitrided steel Endurance strength 8. S fbp S atp KL KT KR KR 0.7 0.15 log( 1 R) KR 1 KT 1

12-54-2

S atp 41500 psi S fbp 41501 psi

Write the design equations using the range of face-width to diametral-pitch ratio given in the text, and the bending stress equation, solved for the unknown face width. Upper limit FU p d FL p d S fb 16 pd 8 pd

Lower limit

Safety factor

Nfb =

b
Ka Wt p d p d Km Nfb Kv p d Jp S fbp

Face width

F p d

9.

Plot F(P) vs. p over the range


d

p d

2 2.1 6 in in in
6 F pd in

10. From the graph, choose a standard value of p d from Table 12-2. p d 4 in
1


in 4

11. The calculated value of F is F p d 2.134 in 12. Round this up to the decimal equivalent of a common fractional value. F 2.250 in 13. Then, the parameters that depend on p d and F are: d p p d 3.500 in Vt p d 222 ft min d g p d 25.750 in Wt p d 1485 lbf Kv p d 0.912

FL pd

FU pd 2 in

3 pd in

FIGURE 12-54A Graph of Face Width and Limits for the Stage 2 Pinion (bending) in Problem 12-54

bp p d F 14058 psi

The assumption made in step 5 is correct so no further iteration is required. Stage 1 Pinion - Surface Fatigue 1. Determine the surface geometry factor, I.

MACHINE DESIGN - An Integrated Approach, 4th Ed.

12-54-3

I 2.

sin( ) cos( ) Ng N N 2 p g

I 0.1686

Write the equations for pitch diameter, pitchline velocity, and transmitted load in terms of the unknown diametral pitch, pd. Note that, in Mathcad, unit conversion factors are not included. Pitch diameter of pinion (in) d p p d Vt p d Wt p d Ca 1 Np pd d p p d n 2 H

pitchline velocity (fpm)

Transmitted load (lbf)

Vt p d

3. 4.

Set the application factor, Ca

Write the equation for the dynamic load factor, Cv B 0.25 12 Qv B 0.52
0.6667

A 50 56 ( 1 B) Cv p d A min A Vt p d ft Cm 1.7 Cp 2300 psi


0.5 B

A 76.878

5. 6. 7.

Tentatively choose the mounting factor, Cm (Assume 1.99 < F < 6 in) Choose an elastic coefficient from Table 12-18. (Steel on steel). The surface stress equation for the pinion is

cp p d F Cp
8. Determine the endurance strength of the pinion. Life factor Reliability Temperature factor Material surface strength (psi) AISI 4340 Nitrided steel Endurance strength 9. S fcp S acp CL CT CR CL 2.466 N

Cv p d F d p p d I

C a W t p d C m

0.056

CL 1 CR 1 CT 1 S acp 162500 psi S fcp 162497 psi

CR 0.7 0.15 log( 1 R)

Write the design equations using the range of face-width to diametral-pitch ratio given in of the text, and the bending stress equation, solved for the unknown face width. Upper limit FU p d 16 pd

MACHINE DESIGN - An Integrated Approach, 4th Ed.

12-54-4

Lower limit

FL p d

8 pd
2

Safety factor

Sfc Nfc = c
F p d

Face width

2 Cp Nfs Cv p d d p p d I S fcp

C a W t p d C m

10. Plot F(P) vs. p over the range


d

p d

2 2.1 6 in in in
4 F pd in FL pd in FU pd in

11. From the graph, choose a standard value of p d from Table 12-2. p d 4 in
1

12. The calculated value of F is F p d 1.691 in 13. Round this up to the decimal equivalent of a common fractional value. F 1.750 in 14. The bending requirement is governing in this case. The diametral pitch and face width for the stage 2 gearset are: p d 4 in
1

3 pd in

FIGURE 12-54B F 2.250 in Graph of Face Width and Limits for the Stage 2 Pinion (surface) in Problem 12-54

MACHINE DESIGN - An Integrated Approach, 4th Ed.

12-55-1

PROBLEM 12-55
Statement: Given: If the gearset in Problem 12-47 transmits 18.5 kW at 1184 input pinion rpm, find the torque on each of the four shafts. Transmitted power P 18.5 kW Input speed

p1 1184 rpm

Assumptions: Tooth numbers: Stage 1 Stage 2 Stage 3 Solution: 1. Np1 18 Np2 18 Np3 18 Ng1 148 Ng2 144 Ng3 162

See Mathcad file P1255.

Confirm that the gear ratio is correct. mG Ng1 Ng2 Ng3 Np1 Np2 Np3 mG 592.000

2.

Calculate the gear speeds. Stage 1

p1 1184 rpm

g1

Np1 Ng1 Np2 Ng2 Np3 Ng3

p1

g1 144 rpm

Stage 2

p2 g1 p3 g2

g2 g3

p2 p3

g2 18 rpm g3 2 rpm

Stage 3

3.

Calculate the shaft torque. Shaft 1 T1 P

p1
P

T1 149.2 N m

Input

Shaft 2

T2

g1
P

T2 1227 N m

Shaft 3

T3

g2
P

T3 9815 N m

Shaft 4

T4

g3

T4 88331 N m

Output

MACHINE DESIGN - An Integrated Approach, 4th Ed.

12-56-1

PROBLEM 12-56
Statement: Size the first-stage spur gears in problem 12-55 for a bending factor of safety of at least 2.4 and a surface factor of safety of at least 2.0 assuming a steady torque, 25-deg pressure angle, full depth teeth, Qv = 10, and AISI 4140 steel for all gears. Pressure angle Bending factor of safety Surface factor of safety Power to be transmitted (hp) Number of pinion teeth

Given:

25 deg
Nfb 2.4 Nfs 2.0 H 25 hp Np 18 Reliability AGMA Quality level Life (cycles) Number of gear teeth R 0.99 Qv 10 N 10
7

Ng 148

Rotational speed of pinion (rpm) n 1184 rpm Assumptions: Since both pinion and gear from each stage are the same material, it will only be necessary to determine the pinion size as it will be governing for the set. Solution: See Mathcad file P1256.

Stage 1 Pinion - Bending 1. 2. Determine the bending geometry factor, J (Table 12-13) Jp 0.39

Write the equations for pitch diameter, pitchline velocity, and transmitted load in terms of the unknown diametral pitch, pd. Note that, in Mathcad, unit conversion factors are not included. Pitch diameter of pinion and gear (in) d p p d Vt p d Wt p d Ka 1 Np pd d p p d n 2 Vt p d H d g p d Ng pd

pitchline velocity (fpm) Transmitted load (lbf) 3. 4. Set the application factor, Ka

Write the equation for the dynamic load factor, Kv B 0.25 12 Qv B 0.397
0.6667

A 50 56 ( 1 B) Kv p d A min A Vt p d ft Km 1.7
B

A 83.776

5. 6.

Tentatively choose the mounting factor, Km (Assume 1.99 < F < 6 in) The bending stress equation for the pinion is

bp p d F
7. Determine the endurance strength of the pinion. Life factor KL 1.6831 N

Ka Wt p d p d Km Kv p d F Jp

0.0323

KL 1

MACHINE DESIGN - An Integrated Approach, 4th Ed.


Reliability Temperature factor Material bending strength (psi) AISI 4140 Nitrided steel Endurance strength 8. S fbp S atp KL KT KR KR 0.7 0.15 log( 1 R) KR 1 KT 1

12-56-2

S atp 40000 psi S fbp 40001 psi

Write the design equations using the range of face-width to diametral-pitch ratio given in the text, and the bending stress equation, solved for the unknown face width. Upper limit FU p d FL p d S fb 16 pd 8 pd

Lower limit

Safety factor

Nfb =

b
Ka Wt p d p d Km Nfb Kv p d Jp S fbp

Face width

F p d

9.

Plot F(P) vs. p over the range


d

p d

5 5.1 10 in in in
4 F pd in

10. From the graph, choose a standard value of p d from Table 12-2. p d 6 in
1


in

11. The calculated value of F is F p d 1.575 in 12. Round this up to the decimal equivalent of a common fractional value. F 1.750 in 13. Then, the parameters that depend on p d and F are: d p p d 3.000 in Vt p d 930 ft min d g p d 24.667 in Wt p d 887 lbf Kv p d 0.884

FL pd

FU pd in

7 pd in

10

FIGURE 12-56A Graph of Face Width and Limits for the Stage 1 Pinion (bending) in Problem 12-56

bp p d F 14997 psi

The assumption made in step 5 is correct so no further iteration is required. Stage 1 Pinion - Surface Fatigue 1. Determine the surface geometry factor, I.

MACHINE DESIGN - An Integrated Approach, 4th Ed.

12-56-3

I 2.

sin( ) cos( ) Ng N N 2 p g

I 0.1707

Write the equations for pitch diameter, pitchline velocity, and transmitted load in terms of the unknown diametral pitch, pd. Note that, in Mathcad, unit conversion factors are not included. Pitch diameter of pinion (in) d p p d Vt p d Wt p d Ca 1 Np pd d p p d n 2 H

pitchline velocity (fpm)

Transmitted load (lbf)

Vt p d

3. 4.

Set the application factor, Ca

Write the equation for the dynamic load factor, Cv B 0.25 12 Qv B 0.397
0.6667

A 50 56 ( 1 B) Cv p d A min A Vt p d ft Cm 1.7 Cp 2300 psi


0.5 B

A 83.776

5. 6. 7.

Tentatively choose the mounting factor, Cm (Assume 1.99 < F < 6 in) Choose an elastic coefficient from Table 12-18. (Steel on steel). The surface stress equation for the pinion is

cp p d F Cp
8. Determine the endurance strength of the pinion. Life factor Reliability Temperature factor Material surface strength (psi) AISI 4140 Nitrided steel Endurance strength 9. S fcp S acp CL CT CR CL 2.466 N

Cv p d F d p p d I

C a W t p d C m

0.056

CL 1 CR 1 CT 1 S acp 165000 psi S fcp 164997 psi

CR 0.7 0.15 log( 1 R)

Write the design equations using the range of face-width to diametral-pitch ratio given in the text, and the bending stress equation, solved for the unknown face width. Upper limit FU p d 16 pd

MACHINE DESIGN - An Integrated Approach, 4th Ed.

12-56-4

Lower limit

FL p d

8 pd
2

Safety factor

Sfc Nfc = c
F p d

Face width

2 Cp Nfs Cv p d d p p d I S fcp

C a W t p d C m

10. Plot F(P) vs. p over the range


d

p d

5 5.1 10 in in in
4 F pd in FL pd in FU pd in

11. From the graph, choose a standard value of p d from Table 12-2. p d 6 in
1

12. The calculated value of F is F p d 1.294 in 13. Round this up to the decimal equivalent of a common fractional value. F 1.375 in 14. The bending requirement is governing in this case. The diametral pitch and face width for the stage 1 gearset are: p d 6 in
1

7 pd in

10

FIGURE 12-56B F 1.750 in Graph of Face Width and Limits for the Stage 1 Pinion (surface) in Problem 12-56

MACHINE DESIGN - An Integrated Approach, 4th Ed.

12-57-1

PROBLEM 12-57
Statement: Size the second-stage spur gears in problem 12-55 for a bending factor of safety of at least 2.4 and a surface factor of safety of at least 2.0 assuming a steady torque, 25-deg pressure angle, full depth teeth, Qv = 10, and AISI 4140 steel for all gears. Pressure angle Bending factor of safety Surface factor of safety Power to be transmitted (hp) Number of pinion teeth

Given:

25 deg
Nfb 2.4 Nfs 2.0 H 25 hp Np 18 Reliability AGMA Quality level Life (cycles) Number of gear teeth R 0.99 Qv 10 N 10
7

Ng 144

Rotational speed of pinion (rpm) n 144 rpm Assumptions: Since both pinion and gear from each stage are the same material, it will only be necessary to determine the pinion size as it will be governing for the set. Solution: See Mathcad file P1257. Jp 0.39

Stage 1 Pinion - Bending 1. 2. Determine the bending geometry factor, J (Table 12-13)

Write the equations for pitch diameter, pitchline velocity, and transmitted load in terms of the unknown diametral pitch, pd. Note that, in Mathcad, unit conversion factors are not included. Pitch diameter of pinion and gear (in) d p p d Vt p d Wt p d Ka 1 Np pd d p p d n 2 Vt p d H d g p d Ng pd

pitchline velocity (fpm) Transmitted load (lbf) 3. 4. Set the application factor, Ka

Write the equation for the dynamic load factor, Kv B 0.25 12 Qv B 0.397
0.6667

A 50 56 ( 1 B) Kv p d A min A Vt p d ft Km 1.7
B

A 83.776

5. 6.

Tentatively choose the mounting factor, Km (Assume 1.99 < F < 6 in) The bending stress equation for the pinion is

bp p d F
7. Determine the endurance strength of the pinion. Life factor KL 1.6831 N

Ka Wt p d p d Km Kv p d F Jp

0.0323

KL 1

MACHINE DESIGN - An Integrated Approach, 4th Ed.


Reliability Temperature factor Material bending strength (psi) AISI 4140 Nitrided steel Endurance strength 8. S fbp S atp KL KT KR KR 0.7 0.15 log( 1 R) KR 1 KT 1

12-57-2

S atp 40000 psi S fbp 40001 psi

Write the design equations using the range of face-width to diametral-pitch ratio given in the text, and the bending stress equation, solved for the unknown face width. Upper limit FU p d FL p d S fb 16 pd 8 pd

Lower limit

Safety factor

Nfb =

b
Ka Wt p d p d Km Nfb Kv p d Jp S fbp

Face width

F p d

9.

Plot F(P) vs. p over the range


d

p d

1 1.1 6 in in in
6 F pd in

10. From the graph, choose a standard value of p d from Table 12-2. p d 3 in
1


in 4

11. The calculated value of F is F p d 3.055 in 12. Round this up to the decimal equivalent of a common fractional value. F 3.250 in 13. Then, the parameters that depend on p d and F are: d p p d 6.000 in Vt p d 226 ft min d g p d 48.000 in Wt p d 3647 lbf Kv p d 0.937

FL pd

FU pd 2 in

3 pd in

FIGURE 12-57A Graph of Face Width and Limits for the Stage 2 Pinion (bending) in Problem 12-57

bp p d F 15669 psi

The assumption made in step 5 is correct so no further iteration is required. Stage 1 Pinion - Surface Fatigue 1. Determine the surface geometry factor, I.

MACHINE DESIGN - An Integrated Approach, 4th Ed.


I

12-57-3
I 0.1702

sin( ) cos( ) Ng N N 2 p g

2.

Write the equations for pitch diameter, pitchline velocity, and transmitted load in terms of the unknown diametral pitch, pd. Note that, in Mathcad, unit conversion factors are not included. Pitch diameter of pinion (in) d p p d Vt p d Wt p d Ca 1 Np pd d p p d n 2 H

pitchline velocity (fpm)

Transmitted load (lbf)

Vt p d

3. 4.

Set the application factor, Ca

Write the equation for the dynamic load factor, Cv B 0.25 12 Qv B 0.397
0.6667

A 50 56 ( 1 B) Cv p d A min A Vt p d ft Cm 1.7 Cp 2300 psi


0.5 B

A 83.776

5. 6. 7.

Tentatively choose the mounting factor, Cm (Assume 1.99 < F < 6 in) Choose an elastic coefficient from Table 12-18. (Steel on steel). The surface stress equation for the pinion is

cp p d F Cp
8. Determine the endurance strength of the pinion. Life factor Reliability Temperature factor Material surface strength (psi) AISI 4140 Nitrided steel Endurance strength 9. S fcp S acp CL CT CR CL 2.466 N

Cv p d F d p p d I

C a W t p d C m

0.056

CL 1 CR 1 CT 1 S acp 165000 psi S fcp 164997 psi

CR 0.7 0.15 log( 1 R)

Write the design equations using the range of face-width to diametral-pitch ratio given on page 740 of the text, and the bending stress equation, solved for the unknown face width. Upper limit FU p d 16 pd

MACHINE DESIGN - An Integrated Approach, 4th Ed.


Lower limit FL p d 8 pd

12-57-4

Safety factor

Nfc =

Sfc c

Face width

Cp F p d Nfs Cv p d d p p d I S fcp
p d 1 1.1 6 in in in
6 F pd in FL pd in FU pd 2 in

C a W t p d C m

10. Plot F(P) vs. p over the range


d

11. From the graph, choose a standard value of p d from Table 12-2. p d 3 in
1

12. The calculated value of F is F p d 2.519 in 13. Round this up to the decimal equivalent of a common fractional value. F 2.750 in 14. The bending requirement is governing in this case. The diametral pitch and face width for the stage 2 gearset are: p d 3 in
1

3 pd in

FIGURE 12-57B F 3.250 in Graph of Face Width and Limits for the Stage 2 Pinion (surface) in Problem 12-57

MACHINE DESIGN - An Integrated Approach, 4th Ed.

12-58-1

PROBLEM 12-58
Statement: Size the third-stage spur gears in problem 12-55 for a bending factor of safety of at least 2.4 and a surface factor of safety of at least 2.0 assuming a steady torque, 25-deg pressure angle, full depth teeth, Qv = 10, and AISI 4140 steel for all gears. Pressure angle Bending factor of safety Surface factor of safety Power to be transmitted (hp) Number of pinion teeth

Given:

25 deg
Nfb 2.4 Nfs 2.0 H 25 hp Np 18 Reliability AGMA Quality level Life (cycles) Number of gear teeth R 0.99 Qv 10 N 10
7

Ng 162

Rotational speed of pinion (rpm) n 18 rpm Assumptions: Since both pinion and gear from each stage are the same material, it will only be necessary to determine the pinion size as it will be governing for the set. Solution: See Mathcad file P1258.

Stage 1 Pinion - Bending 1. 2. Determine the bending geometry factor, J (Table 12-13) Jp 0.39

Write the equations for pitch diameter, pitchline velocity, and transmitted load in terms of the unknown diametral pitch, pd. Note that, in Mathcad, unit conversion factors are not included. Pitch diameter of pinion and gear (in) d p p d Vt p d Wt p d Ka 1 Np pd d p p d n 2 Vt p d H d g p d Ng pd

pitchline velocity (fpm) Transmitted load (lbf) 3. 4. Set the application factor, Ka

Write the equation for the dynamic load factor, Kv B 0.25 12 Qv B 0.397
0.6667

A 50 56 ( 1 B) Kv p d A min A Vt p d ft Km 1.8
B

A 83.776

5. 6.

Tentatively choose the mounting factor, Km (Assume 5.99 < F < 6 in) The bending stress equation for the pinion is

bp p d F
7. Determine the endurance strength of the pinion. Life factor KL 1.6831 N

Ka Wt p d p d Km Kv p d F Jp

0.0323

KL 1

MACHINE DESIGN - An Integrated Approach, 4th Ed.


Reliability Temperature factor Material bending strength (psi) AISI 4140 Nitrided steel Endurance strength 8. S fbp S atp KL KT KR KR 0.7 0.15 log( 1 R) KR 1 KT 1

12-58-2

S atp 40000 psi S fbp 40001 psi

Write the design equations using the range of face-width to diametral-pitch ratio given in the text, and the bending stress equation, solved for the unknown face width. Upper limit FU p d FL p d S fb 16 pd 8 pd

Lower limit

Safety factor

Nfb =

b
Ka Wt p d p d Km Nfb Kv p d Jp S fbp

Face width

F p d

9.

Plot F(P) vs. p over the range


d

p d

1 1.1 6 in in in
10 F pd in

10. From the graph, choose a standard value of p d from Table 12-2. p d 1.5 in
1


in

11. The calculated value of F is F p d 6.27 in 12. Round this up to the decimal equivalent of a common fractional value. F 6.500 in 13. Then, the parameters that depend on p d and F are: d p p d 12.000 in Vt p d 57 ft min d g p d 108.000 in Wt p d 14589 lbf Kv p d 0.966

FL pd

FU pd in

1 1.25 1.5 1.75 2 2.25 2.5 2.75 3 pd in

FIGURE 12-58A Graph of Face Width and Limits for the Stage 3 Pinion (bending) in Problem 12-58

bp p d F 16078 psi

The assumption made in step 5 is correct so no further iteration is required. Stage 1 Pinion - Surface Fatigue 1. Determine the surface geometry factor, I.

MACHINE DESIGN - An Integrated Approach, 4th Ed.

12-58-3

I 2.

sin( ) cos( ) Ng N N 2 p g

I 0.1724

Write the equations for pitch diameter, pitchline velocity, and transmitted load in terms of the unknown diametral pitch, pd. Note that, in Mathcad, unit conversion factors are not included. Pitch diameter of pinion (in) d p p d Vt p d Wt p d Ca 1 Np pd d p p d n 2 H

pitchline velocity (fpm)

Transmitted load (lbf)

Vt p d

3. 4.

Set the application factor, Ca

Write the equation for the dynamic load factor, Cv B 0.25 12 Qv B 0.397
0.6667

A 50 56 ( 1 B) Cv p d A min A Vt p d ft Cm 1.8 Cp 2300 psi


0.5 B

A 83.776

5. 6. 7.

Tentatively choose the mounting factor, Cm (Assume 5.99 < F < 9 in) Choose an elastic coefficient from Table 12-18. (Steel on steel). The surface stress equation for the pinion is

cp p d F Cp
8. Determine the endurance strength of the pinion. Life factor Reliability Temperature factor Material surface strength (psi) AISI 4140 Nitrided steel Endurance strength 9. S fcp S acp CL CT CR CL 2.466 N

Cv p d F d p p d I

C a W t p d C m

0.056

CL 1 CR 1 CT 1 S acp 165000 psi S fcp 164997 psi

CR 0.7 0.15 log( 1 R)

Write the design equations using the range of face-width to diametral-pitch ratio given in the text, and the bending stress equation, solved for the unknown face width. Upper limit FU p d 16 pd

MACHINE DESIGN - An Integrated Approach, 4th Ed.

12-58-4

Lower limit

FL p d

8 pd
2

Safety factor

Nfc =

Sfc c

Face width

Cp F p d Nfs Cv p d d p p d I S fcp
p d 1 1.1 6 in in in
10 F pd in FL pd in FU pd in

C a W t p d C m

10. Plot F(P) vs. p over the range


d

11. From the graph, choose a standard value of p d from Table 12-2. p d 1.5 in
1

12. The calculated value of F is F p d 5.105 in 13. Round this up to the decimal equivalent of a common fractional value. F 5.250 in 14. The bending requirement is governing in this case. The diametral pitch and face width for the stage 3 gearset are: p d 1.5 in
1

1 1.25 1.5 1.75 2 2.25 2.5 2.75 3 pd in

FIGURE 12-58B F 6.500 in Graph of Face Width and Limits for the Stage 3 Pinion (surface) in Problem 12-58

MACHINE DESIGN - An Integrated Approach, 4th Ed.

12-59-1

PROBLEM 12-59
Statement: The pinion of an external gearset has pitch radius rp = 40 mm and the gear has pitch radius rg = 160 mm. If the pinion is the input member of the set, determine the velocity ratio, the torque ratio, and the gear ratio of the set. Pitch radii rp 40 mm rg 160 mm

Given: Solution: 1.

See Mathcad file P1259.

Equate the pinion pitch radius to rin and the gear pitch radius to rout (in this case). rin rp rin 40 mm rout rg rout 160 mm

2.

Using equation 12.1a, calculate the velocity ratio using the minus sign because this is an external set. Velocity ratio mV

rin rout

mV 0.25

3.

Using equation 12.1b, calculate the torque ratio using the minus sign because this is an external set. Torque ratio mA 1 mV mA 4

4.

Using equation 12.1c, calculate the gear ratio using the torque ratio since it is greater than 1. Gear ratio mG mA mG 4

This is usually stated as being 4:1.

MACHINE DESIGN - An Integrated Approach, 4th Ed.

12-60-1

PROBLEM 12-60
Statement: A pinion having 20 teeth and a diametral pitch of 8 (in -1) is in mesh with a rack. If the pinion rotates one revolution, how far will the rack move? Number of teeth See Mathcad file P1260. Np 20 Diametral pitch p d 8 in
1

Given: Solution: 1.

Calculate the pitch diameter using equation 12-4a. Pitch diameter d Np pd d 2.500 in

2.

The rack and pinion can be modeled as a cylinder, with diameter equal to the pitch diameter of the gear, in contact with a plane along the pitch line of the rack. When the gear is rotated one revolution, the rack will move a distance equal to the circumference of the cylinder. Distance moved s d s 7.854 in

MACHINE DESIGN - An Integrated Approach, 4th Ed.

12-61-1

PROBLEM 12-61
Statement: A gearset with full-depth teeth is designed to have a pinion with 24 teeth, a gear with 54 teeth, and a diametral pitch of 6. Compare the contact ratio for this set for pressure angles of 14.5, 20, and 25 degrees. Number of teeth Np 24 Diametral pitch Solution: 1. p d 6 in
1

Given:

Ng 54

See Mathcad file P1261.

Calculate the pitch radus and addendum for each gear, and the nominal center distamce. Pitch radius: rp rg Addendum: a p 1 Np 2 pd 1 Ng 2 pd 1 pd rp 2.000 in rg 4.500 in a p 0.167 in a g 0.167 in C 6.500 in

a g a p Center distance 2. C rp rg

Write the equation for the length of action as a function of pressure angle using equation 12.2. Z ( )

rp ap 2 rp cos( ) 2 rg ag2 rg cos( ) 2 C sin( )


p d Z ( )

3.

Write the equation for the contact ratio as a function of pressure angle using equation 12.7b. mp( )

cos( )

4.

Evaluate the contact ratio for the three pressure angles. 14.5 degrees 20 degrees 25 degrees mp( 14.5 deg) 2.007 mp( 20 deg) 1.685 mp( 25 deg) 1.492

MACHINE DESIGN - An Integrated Approach, 4th Ed.

12-62-1

PROBLEM 12-62
Statement:

_____

Design a planetary gear train similar to that shown in Figure 12-16 for an overall velocity ratio of exactly 5 if the ring gear is stationary, the arm is the input, and the sun gear is the output. Specify tooth numbers for each gear in the train. Exact ratio mV 5

Given:

Assumptions: The gears will be cut with a hob and the pressure angle will be 20 deg Solution: 1. See Figure 12-16 and Mathcad file P1262.

Let the first gear be the sun gear (gear 2) and the last be the ring gear (gear 4). Then, equation 12.11c can be written as

4 arm 2 arm
2.

N2 N4

Taking 4 = 0 and solving for out/in, we have

out in
3.

2 a

N4 N2

1 = mv

Let the sun gear have tooth number N4 N2 mV 1

N2 20 , then the number of teeth required for the ring gear is N4 80

4.

In order to mesh properly, the number of teeth on the ring gear must be equal to the sum of the number of teeth on the sun gear plus two times the number of teeth on the planet gear(s). Thus, N3 N4 N2 2 N3 30

5.

Check the overall velocity ratio and summarize the tooth numbers.

Overall velocity ratio

mV

N4 N2

mV 5.000

Sun gear Planet gear(s) Ring gear

N2 20 N3 30 N4 80

MACHINE DESIGN - An Integrated Approach, 4th Ed.

12-63-1

PROBLEM 12-63
Statement:

_____

A 22-tooth pinion rotating at 1650 rpm meshes with a 66-tooth gear in a spur gear reducer. Both pinion and gear are manufactured to a quality level of 10. A reliability of 0.9 has been specified, and the transmitted tangential load is 5000 lb. Conditions are such that Km = 1.7. It is proposed that standard 25-degree, full-depth teeth be used, with both pinion and gear hobbed from an AISI 4340 nitrided steel. The diametral pitch is 5, and the face width 2.500 in. Estimate the number of cycles of bending stress (using the AGMA equations) that the gearset can withstand. Tooth numbers Pinion speed Diametral pitch Face width Tangential load Np 22 n 1650 rpm p d 5 in
1

Given:

Ng 66 Mounting factor Quality index Pressure angle Reliability Km 1.7 Qv 10

F 2.500 in Wt 5000 lbf

25 deg
R 0.90

Assumptions: The life of the pinion will be less than that of the gear. Solution: 1. See Mathcad file P1263. d n 2

Calculate the pitch diameter of the pinion using equation 12.4a and the pitch-line velocity of the gearset. d Np pd d 4.400 in V V 1901 ft min

2.

Calculate the dynamic factor Kv using equations 12.16 and 12.17. B 0.25 12 Qv Kv
0.6667 B

B 0.397

A 50 56 ( 1 B) Kv 0.847

A 83.776

A V min A ft

3. 4. 5.

Determine the bending geometry factor, J (Table 12-13) Assume an application factor, Ka Ka 1

J 0.42

Calculate the bending stress using equation 12.15.

b
6. 7. 8. 9.

Ka Wt p d Km Kv F J

b 47.80 ksi
S'fb 41.5 ksi

Determine the material bending fatigue strength from Table 12-20 using the average value. Determine the reliability factor for R 0.90 Assume a temperature factor, KT KT 1 from Table 12-19. KR 0.85

Set the bending stress equal to the corrected bending-fatigue strength in equation 12.24 and solve for the life factor, KL . KL KT KR b S'fb KL 0.979

10. Assume that the fatigue life is over 10 6 cycles and use the lower curve in Figure 12-24 to solve for the cycle life
1 0.0323

KL = 1.6831 N

1.6831 K L

0.0323

N 1.93 10

MACHINE DESIGN - An Integrated Approach, 4th Ed.

12-64-1

PROBLEM 12-64
Statement:

_____

A 22-tooth pinion rotating at 1650 rpm meshes with a 66-tooth gear in a spur gear reducer. Both pinion and gear are manufactured to a quality level of 10. A reliability of 0.9 has been specified, and the transmitted tangential load is 5000 lb. Conditions are such that Km = 1.7. It is proposed that standard 25-degree, full-depth teeth be used, with both pinion and gear hobbed from an AISI 4340 nitrided steel. The diametral pitch is 5, and the face width 2.500 in. Estimate the number of cycles of contact (surface) stress (using the AGMA equations) that the gearset can withstand. Tooth numbers Pinion speed Diametral pitch Face width Tangential load Np 22 n 1650 rpm p d 5 in
1

Given:

Ng 66 Mounting factor Quality index Pressure angle Reliability Cm 1.7 Qv 10

F 2.500 in Wt 5000 lbf

25 deg
R 0.90

Assumptions: The life of the pinion will be less than that of the gear. Solution: 1. See Mathcad file P1264. d n 2

Calculate the pitch diameter of the pinion using equation 12.4a and the pitch-line velocity of the gearset. d Np pd d 4.400 in V V 1901 ft min A 83.776

2.

Calculate the dynamic factor Kv using equations 12.16 and 12.17. B 0.25 12 Qv Cv
0.6667 B

B 0.397

A 50 56 ( 1 B) Cv 0.847

A V min A ft

3.

Calculate the geometry factor, I I

sin( ) cos( ) Ng N N 2 p g

I 0.144 Cp 2300 psi


0.5

4. 5. 6.

Determine the elastic coefficient from Table 12-18. Assume an application factor, Ca Ca 1

Calculate the contact stress using equation 12.21.


1 2 Ca Wt 1 Cm Cv F d I

c Cp
7. 8. 9.

c 183.32 ksi
S'fc 162.5 ksi

Determine the material surface-fatigue strength from Table 12-21 using the average value. Determine the reliability factor for R 0.90 Assume a temperature factor, CT CT 1 from Table 12-19. CR 0.85

10. Set the contact stress equal to the corrected surface-fatigue strength in equation 12.25 and solve for the life factor, CL .

MACHINE DESIGN - An Integrated Approach, 4th Ed.

12-64-2

CL

CT CR c S'fc

CL 0.959

11. Use the lower curve in Figure 12-26 to solve for the cycle life.
1

CL = 2.466 N

0.056

2.466 C L

0.056

N 2.11 10

MACHINE DESIGN - An Integrated Approach, 4th Ed.

12-65-1

PROBLEM 12-65
Statement: The pinion of an internal gearset has pitch radius rp = 30 mm and the gear has pitch radius rg = 150 mm. If the pinion is the input member of the set, determine the velocity ratio, the torque ratio, and the gear ratio of the set. Pitch radii rp 30 mm rg 150 mm

Given: Solution: 1.

See Mathcad file P1265.

Equate the pinion pitch radius to rin and the gear pitch radius to rout (in this case). rin rp rin 30 mm rout rg rout 150 mm

2.

Using equation 12.1a, calculate the velocity ratio using the plus sign because this is an internal set. Velocity ratio mV

rin r out

mV 0.2

3.

Using equation 12.1b, calculate the torque ratio using the minus sign because this is an external set. Torque ratio mA 1 mV mA 5

4.

Using equation 12.1c, calculate the gear ratio using the torque ratio since it is greater than 1. Gear ratio mG mA mG 5

This is usually stated as being 5:1.

MACHINE DESIGN - An Integrated Approach, 4th Ed.

12-66-1

PROBLEM 12-66
Statement: A pinion having 18 teeth and a diametral pitch of 10 (in -1) is in mesh with a rack. If the rack moves 1 in, how many degrees will the pinion rotate? Number of teeth See Mathcad file P1266. Np 18 Diametral pitch p d 10 in
1

Given: Solution: 1.

Calculate the pitch diameter using equation 12-4a. Pitch diameter d Np pd d 1.800 in

2.

The rack and pinion can be modeled as a cylinder, with diameter equal to the pitch diameter of the gear, in contact with a plane along the pitch line of the rack. When the rack is moved 1 in, the pinion will rotate through an angle = d/2s, where d is the diameter of the cylinder and s is the distance that the rack moves. Distance moved Pinion rotation s 1 in

d 2 s

51.566 deg

MACHINE DESIGN - An Integrated Approach, 4th Ed.

12-67-1

PROBLEM 12-67
Statement: A gearset with 25 deg, full-depth teeth is designed to have a pinion with 24 teeth, a gear with 54 teeth, and a diametral pitch of 6. Compare the contact ratio for this set for a range of center distances from 0.90C to 1.10C. Number of teeth Np 24 Diametral pitch Solution: 1. p d 6 in
1

Given:

Ng 54 Pressure angle

25 deg

See Mathcad file P1267.

Calculate the pitch radus, base radius, and addendum for each gear, and the nominal center distamce. Pitch radius: rp rg Base circle radius: Addendum: 1 Np 2 pd 1 Ng 2 pd rp 2.000 in rg 4.500 in rbp 1.813 in a p 0.167 in a g 0.167 in C 6.500 in

rbp rp cos( ) a p 1 pd

a g a p Center distance 2. C rp rg

Write the equation for the pressure angle as a function of variation in center distance. Let f be a factor which, when multiplied by C gives the actual center distance C'. Then, the actual pitch radius of the pinion will be rp' frp and the actual pressure angle will be = acos(rbp/frp).

'( f ) acos
3.

rbp f rp

Note that '( 1 ) 25 deg

Write the equation for the length of action as a function of actual pressure angle and center distance using equation 12.2. Z ( f )

rp ap2 rp cos( '( f ) ) 2 rg ag2 rg cos( '( f ) ) 2 f C sin( '( f ) )


pd Z ( f )

4.

Write the equation for the contact ratio as a function of pressure angle using equation 12.7b. mp( f )

cos( '( f ) )

5.

Evaluate the contact ratio for the range of center distance. Let f 0.90 0.905 1.10 See the graph on the next page.

MACHINE DESIGN - An Integrated Approach, 4th Ed.

12-67-2

CONTACT RATIO vs CHANGE IN CENTER DISANCE


4

Contact Ratio

m p( f ) 2

0 0.9

0.95

1 f

1.05

1.1

Change in Center Distance Factor

MACHINE DESIGN - An Integrated Approach, 4th Ed.

12-68-1

PROBLEM 12-68
Statement:

_____

Design a planetary gear train similar to that shown in Figure 12-16 for an overall velocity ratio of exactly 1.25 if the sun gear is stationary, the arm is the input, and the ring gear is the output. Specify tooth numbers for each gear in the train. Exact ratio mV 1.25

Given:

Assumptions: The gears will be cut with a hob and the pressure angle will be 20 deg Solution: 1. See Figure 12-16 and Mathcad file P1268.

Let the first gear be the sun gear (gear 2) and the last be the ring gear (gear 4). Then, equation 12.11c can be written as

4 arm 2 arm
2.

N2 N4

Taking 2 = 0 and solving for out/in, we have

out in
3.

4 a

N2 N4

1 = mv

Let the sun gear have tooth number N2

N2 20 , then the number of teeth required for the ring gear is N4 80

N4

mV 1

4.

In order to mesh properly, the number of teeth on the ring gear must be equal to the sum of the number of teeth on the sun gear plus two times the number of teeth on the planet gear(s). Thus, N3 N4 N2 2 N3 30

5.

Check the overall velocity ratio and summarize the tooth numbers.

Overall velocity ratio

mV

N2 N4

mV 1.250

Sun gear Planet gear(s) Ring gear

N2 20 N3 30 N4 80

MACHINE DESIGN - An Integrated Approach, 4th Ed.

13-1-1

PROBLEM 13-1
Statement: A 20-deg pressure angle, 30-deg helix angle, 27-tooth helical gear has a diametral pitch p d = 5. Find the pitch diameter, addendum, dedendum, outside diameter, normal, transverse, and axial pitch. Pressure angle Number of teeth Solution: 1. See Mathcad file P1301.

Given:

20 deg
N 27

Helix angle Diametral pitch

30 deg
p d 5 in
1

Use equations 13.1 to find the pitch diameter, transverse pitch, normal pitch, and axial pitch. Pitch diameter d p t N pd d 5.400 in p t 0.628 in p n 0.544 in p x 1.088 in

Transverse pitch

pd

Normal pitch

p n p t cos( ) p x pn sin( )

Axial pitch

2.

Use the equations in Table 12-1 to calculate the addendum, dedendum, and outside diameter. Addendum a b 1 pd 1.25 pd a 0.200 in b 0.250 in D 5.800 in

Dedendum Outside diameter

D d 2 a

MACHINE DESIGN - An Integrated Approach, 4th Ed.

13-2-1

PROBLEM 13-2
Statement: A 25-deg pressure angle, 20-deg helix angle, 43-tooth helical gear has a diametral pitch p d = 8. Find the pitch diameter, addendum, dedendum, outside diameter, normal, transverse, and axial pitch. Pressure angle Number of teeth Solution: 1. See Mathcad file P1302.

Given:

25 deg
N 43

Helix angle Diametral pitch

20 deg
p d 8 in
1

Use equations 13.1 to find the pitch diameter, transverse pitch, normal pitch, and axial pitch. Pitch diameter d p t N pd d 5.375 in p t 0.393 in p n 0.369 in p x 1.079 in

Transverse pitch

pd

Normal pitch

p n p t cos( ) p x pn sin( )

Axial pitch

2.

Use the equations in Table 12-1 to calculate the addendum, dedendum, and outside diameter. Addendum a b 1 pd 1.25 pd a 0.125 in b 0.156 in D 5.625 in

Dedendum Outside diameter

D d 2 a

MACHINE DESIGN - An Integrated Approach, 4th Ed.

13-3-1

PROBLEM 13-3
Statement: Given: A 57-tooth, 10-deg helix angle, helical gear is in mesh with a 23-tooth pinion. The p d = 6 and = 25 deg. Find the transverse and axial contact ratios. Pressure angle Number of pinion teeth Number of gear teeth

25 deg
Np 23 Ng 57

Helix angle Diametral pitch

10 deg
p d 6 in
1

Assumptions: The face width factor is fwf 10. Solution: 1. See Mathcad file P1303.

Use equation 12.7 to find the transverse contact ratio. Circular pitch Base pitch Pinion: Pitch dia Pitch rad Gear: Pitch dia Pitch rad d g Ng pd d g 9.500 in rg 4.750 in a 0.167 in d p Np pd d p 3.833 in rp 1.917 in p c

pd

p c 0.524 in p b 0.475 in

p b p c cos( )

rp 0.5 d p

rg 0.5 d g a 1.0 pd Np Ng 2 pd

Addendum

Center distance Length of action

C 6.667 in

rp a 2 rp cos( ) 2 rg a 2 rg cos( ) 2 C sin( )


Z pb

Z 0.708 in Transverse contact ratio 2. mp mp 1.491

Use equation 13.5 to find the axial contact ratio. Face width F fwf pd F p d tan ( ) F 1.667 in

Axial contact ratio

mF

mF 0.561

MACHINE DESIGN - An Integrated Approach, 4th Ed.

13-4-1

PROBLEM 13-4
Statement: Given: A 78-tooth, 30-deg helix angle, helical gear is in mesh with a 27-tooth pinion. The p d = 6 and = 20 deg. Find the transverse and axial contact ratios. Pressure angle Number of pinion teeth Number of gear teeth

20 deg
Np 27 Ng 78

Helix angle Diametral pitch

30 deg
p d 6 in
1

Assumptions: The face width factor is fwf 12. Solution: 1. See Mathcad file P1304.

Use equation 12.7 to find the transverse contact ratio. Circular pitch Base pitch Pinion: Pitch dia Pitch rad Gear: Pitch dia Pitch rad d g Ng pd d g 13.000 in rg 6.500 in a 0.167 in d p Np pd d p 4.500 in rp 2.250 in p c

pd

p c 0.524 in p b 0.492 in

p b p c cos( )

rp 0.5 d p

rg 0.5 d g a 1.0 pd Np Ng 2 pd

Addendum

Center distance Length of action

C 8.750 in

rp a 2 rp cos( ) 2 rg a 2 rg cos( ) 2 C sin( )


Z pb

Z 0.849 in Transverse contact ratio 2. mp mp 1.726

Use equation 13.5 to find the axial contact ratio. Face width F fwf pd F p d tan ( ) F 2.000 in

Axial contact ratio

mF

mF 2.205

MACHINE DESIGN - An Integrated Approach, 4th Ed.

13-5-1

PROBLEM 13-5
Statement: A 90-deg straight bevel gearset is needed to give a 9:1 reduction. Determine the pitch cone angles, pitch diameters, and gear forces if the 25-deg pressure angle pinion has 14 teeth of p d = 6, and the transmitted power is 746 W at 1000 pinion rpm. Power transmitted Pinion speed Teeth on pinion Solution: 1. 2. See Mathcad file P1305. Ng mG Np Ng 126 H 746 W Gear ratio Diametral pitch Pressure angle mG 9 p d 6 in
1

Given:

p 1000 rpm
Np 14

25 deg

Use equation 13.7b to calculate number of teeth on the gear. Use equation 12.4a to calculate the pitch diameters. Pinion d p Np pd Ng pd

d p 2.333 in

Gear

d g

d g 21.000 in

3.

Use equation 13.7b to calculate the pitch cone angles. Pinion

p atan

mG

p 6.340 deg

Gear Note that 4.

g atan mG

g 83.660 deg

p g 90 deg

Determine the torque on the pinion shaft and the transmitted force. Pinion Tp H

p
2 Tp dp

Tp 63.051 in lbf

Transmitted force

Wt

Wt 54.043 lbf

5.

Use equations 13.8a to calculate the pinion and gear forces. Pinion Wap Wt tan ( ) sin p Wrp Wt tan( ) cos p Gear Wag Wt tan ( ) sin g Wrg Wt tan( ) cos g Total force W Wt ( cos( ) )
1

Wap 2.78 lbf Wrp 25.05 lbf Wag 25.05 lbf Wrg 2.78 lbf W 59.63 lbf

MACHINE DESIGN - An Integrated Approach, 4th Ed.

13-6-1

PROBLEM 13-6
Statement: A 90-deg straight bevel gearset is needed to give a 4.5:1 reduction. Determine the pitch cone angles, pitch diameters, and gear forces if the 20-deg pressure angle pinion has 18 teeth of p d = 5, and the transmitted power is 7460 W at 800 pinion rpm. Power transmitted Pinion speed Teeth on pinion Solution: 1. 2. See Mathcad file P1306. Ng mG Np Ng 81 H 7460 W Gear ratio Diametral pitch Pressure angle mG 4.5 p d 5 in
1

Given:

p 800 rpm
Np 18

20 deg

Use equation 13.7b to calculate number of teeth on the gear. Use equation 12.4a to calculate the pitch diameters. Pinion d p Np pd Ng pd

d p 3.600 in

Gear

d g

d g 16.200 in

3.

Use equation 13.7b to calculate the pitch cone angles. Pinion

p atan

mG

p 12.529 deg

Gear Note that 4.

g atan mG

g 77.471 deg

p g 90 deg

Determine the torque on the pinion shaft and the transmitted force. Pinion Tp H

p
2 Tp dp

Tp 788.134 in lbf

Transmitted force

Wt

Wt 437.852 lbf

5.

Use equations 13.8a to calculate the pinion and gear forces. Pinion Wap Wt tan ( ) sin p Wrp Wt tan( ) cos p Gear Wag Wt tan ( ) sin g Wrg Wt tan( ) cos g Total force W Wt ( cos( ) )
1

Wap 34.57 lbf Wrp 155.57 lbf Wag 155.57 lbf Wrg 34.57 lbf W 465.953 lbf

MACHINE DESIGN - An Integrated Approach, 4th Ed.

13-7-1

PROBLEM 13-7
Statement: A 90-deg straight bevel gearset is needed to give a 5:1 reduction. Determine the pitch cone angles, pitch diameters, and gear forces if the 20-deg pressure angle pinion has 16 teeth of p d = 7, and the transmitted power is 3 hp at 600 pinion rpm. Power transmitted Pinion speed Teeth on pinion H 3 hp Gear ratio Diametral pitch Pressure angle mG 5 p d 7 in
1

Given:

p 600 rpm
Np 16

20 deg

Assumptions: The spiral angle is 35 deg Solution: 1. 2. See Mathcad file P1307. Ng mG Np Ng 80

Use equation 13.7b to calculate number of teeth on the gear. Use equation 12.4a to calculate the pitch diameters. Pinion d p Np pd Ng pd

d p 2.286 in

Gear

d g

d g 11.429 in

3.

Use equation 13.7b to calculate the pitch cone angles. Pinion

p atan

mG

p 11.310 deg

Gear Note that 4.

g atan mG

g 78.690 deg

p g 90 deg

Determine the torque on the pinion shaft and the transmitted force. Pinion Tp H

p
2 Tp dp

Tp 315.127 in lbf

Transmitted force

Wt

Wt 275.736 lbf

5.

Use equations 13.2 to calculate the pressure angle in the normal plane.

n atan( tan( ) cos( ) )


6. Use equations 13.8b to calculate the pinion and gear forces. Pinion Wap Wrp Wt cos( ) Wt cos( )

n 16.602 deg

tan n sin p sin( ) cos p tan n cos p sin( ) sin p

Wap 169.64 lbf Wrp 136.28 lbf

MACHINE DESIGN - An Integrated Approach, 4th Ed.


Gear Wag Wrg Wt cos( ) Wt cos( ) tan n sin g sin( ) cos g tan n cos g sin( ) sin g
1

13-7-2

Wag 60.55 lbf Wrg 209.01 lbf

Total force

W Wt ( cos( ) )

W 293.432 lbf

MACHINE DESIGN - An Integrated Approach, 4th Ed.

13-8-1

PROBLEM 13-8
Statement: A paper mill processes rolls of paper having a density of 984 kg/m3. The paper roll is 1.50-m outside diameter (OD) x 0.22-m inside diameter (ID) x 3.23-m long and is on a simple supported, hollow, steel shaft with S ut = 400 MPa. Design a 2.5:1 reduction helical gearset to drive this roll shaft to obtain a minimum dynamic safety factor of 2 for 10-year life if the roll turns at 50 rpm with 1.2 hp absorbed. Surface factor of safety Power to be transmitted Gear ratio Nfc 2 H 1.2 hp mg 2.5 Life (years) Gear speed Life 10 yr n g 50 rpm

Given:

Assumptions: 1. If both pinion and gear are the same material, it will only be necessary to determine the pinion size as it will be governing for the set. 2. If the gears are not surface hardened, it will only be necessary to design to the surface requirement as it will be governing for both bending and surface stresses. Design Choices: Pressure angle Number of pinion teeth

25 deg
Np 14

AGMA Quality level Relibility

Qv 11 R 0.99

Helix angle 20 deg Material: AGMA Grade 1 steel for both pinion and gear, through hardened to HB 230 S ac ( 26000 327 HB) psi S ac 101210 psi Solution: 1. See Mathcad file P1308.

Determine the number of gear teeth, the pinion speed, and the cycle life. Number of gear teeth Rotational speed of pinion (rpm) Cycle life: Ng Np mg n p n g mg N Life np 2 Ng 35 n p 125 rpm N 6.574 10
8

2.

Determine the surface geometry factor, I. Addendum a p d rp p d 1 pd Np 2 pd rg p d Ng 2 pd

Pitch radii

Center distance Radii of curvature

C p d rp p d rg p d

p p d

2 2 0.5 rp p d C pd rg pd rp p d cos( )

g p d C p d sin( ) p p d
Normal plane pressure angle

n atan( tan( ) cos( ) ) b acos cos( )

n 23.662 deg

Base helix angle

cos n cos( )

b 18.256 deg

MACHINE DESIGN - An Integrated Approach, 4th Ed.


Length of action Z p d

13-8-2

r p a p 2 r p cos( ) 2 d p d p d r p a p 2 r p cos( ) 2 C p sin( ) d g d d g d


mp p d Z pd

Transverse contact ratio

pd

cos( ) F p d tan( )

Axial contact ratio

mF p d F

Fractional parts of mp and mF

n r p d mp p d floor mp p d n a p d F mF p d F floor mF p d F

Axial pitch

p t p d p x p d

pd pn pd sin( )

p n p d p t p d cos( )

Minimum length of lines of contact Lmin p d F

return

return

mp pd F napd F nrpd pxpd if n a p d F 1 n r p d cos b mp pd F 1 na p d F 1 nr pd px p d if n p F 1 n p a d r d cos b


mN p d F I p d F Lmin p d F cos( ) F

Load sharing ratio

Geometry factor

1 1 2 r p m p F p p p d N d g d p d

3.

Write the equations for pitch diameter, pitchline velocity, and transmitted load in terms of the unknown diametr pitch, p d. Note that, in Mathcad, unit conversion factors are not included. Pitch diameter of pinion (in) d p p d Vt p d Wt p d Np pd d p p d n p 2 Vt p d H d g p d Ng pd

pitchline velocity (fpm) Transmitted load (lbf)

MACHINE DESIGN - An Integrated Approach, 4th Ed.

13-8-3

4. 5.

Set the application factor, Ca Write the equation for the dynamic load factor, C
v

Ca 1 B 0.25 12 Qv
0.6667

B 0.250 A 92.000
B

A 50 56 ( 1 B) Cv p d A min A Vt p d ft Cm 1.7 Cp 2300 psi


0.5

6. 7. 8. 9.

Tentatively choose the mounting factor, Cm (Assume 2 < F < 6 in Choose an elastic coefficient from Table 12-18 (steel on steel). The surface stress equation for the pinion is Determine the endurance strength of the pinion. Life factor Reliability Temperature factor Material surface strength (psi) Grade 1, 230 HB Endurance strength S fcp S ac CL CT CR CL 2.466 N

cp p d F Cp

Cv p d F d p p d I p d F

C a W t p d C m

0.056

CL 0.791 CR 1.000 CT 1 S ac 101210 psi S fcp 80060 psi

CR 0.7 0.15 log( 1 R)

10. Write the design equations using the range of face-width to diametral-pitch ratio given in the text, and the bending stress equation, solved for the unknown face width. Upper limit FU p d FL p d 16 pd 8 pd
2

Lower limit

Safety factor

Sfc Nfc = c
F' p d F

Face width

2 Cp Nfc Cv p d d p p d I p d F S fcp

C a W t p d C m

Guess value for F

F 2.35 in

Found by iteration with value in step 11.

MACHINE DESIGN - An Integrated Approach, 4th Ed.

13-8-4

9. Plot F(P) vs. p d over the range

p d

2 2.5 12 in in in
F' pd F in FL pd in FU pd in

10. From the graph, choose a standard value of p d from Table 12-2. p d 5 in
1

11. The calculated value of F is F' p d F 2.355 in 12. Round this up to the decimal equivalent of a common fractional value. F 2.375 in 13. Then, the parameters that depend on p d and F are: d p p d 2.800 in Vt p d 92 ft min d g p d 7.000 in Wt p d 432 lbf mp p d 1.420 mN p d F 0.728

7 pd in

9 10 11 12

FIGURE 13-8
Graph of Face Width and Limits for the Pinion (surface) in Problem 13-8

cp p d F 56448 psi
n a p d F 0.376

mF p d F 1.376 Lmin p d F 3.264 in

n r p d 0.420 I p d F 0.188

14. The realized factor of safety against surface failure in the pinion is

Sfcp Nf p d F cp pd F

Nf p d F 2.0

MACHINE DESIGN - An Integrated Approach, 4th Ed.

13-9-1

PROBLEM 13-9
Statement: A 2-start wormset has the dimensions given below. Find the lead, lead angle, worm gear diameter, and center distance. Will it self-lock? The input speed is 2200 rpm. Worm pitch diameter Axial pitch Threads on worm Solution: 1. 2. 3. 4. See Mathcad file P1309. L p x Nw L 20 mm d 50 mm p x 10 mm Nw 2 Gear ratio Input (worm) speed mG 22 n w 2200 rpm

Given:

Use equation 13.13 to calculate the lead. Use equation 13.12 to calculate the lead angle. Calculate the number of teeth on the gear Use equation 13.13 to calculate gear diameter.

atan

d
L

7.256 deg
Ng 44

Ng mG Nw p c p x d g C p c Ng

d dg 2

d g 140.06 mm C 95.03 mm

5. 6.

Use equation 13.17 to calculate the center distance.

If the lead angle per worm tooth is less than 6 deg, the set is self-locking. The lead angle per tooth is

Nw

t 3.628 deg

Self-locking

MACHINE DESIGN - An Integrated Approach, 4th Ed.

13-10-1

PROBLEM 13-10
Statement: A 3-start wormset has the dimensions given below. Find the lead, lead angle, worm gear diameter, and center distance. Will it self-lock? The input speed is 1400 rpm. Worm pitch diameter Axial pitch Threads on worm Solution: 1. 2. 3. 4. See Mathcad file P1310. L p x Nw L 0.600 in d 1.750 in p x 0.200 in Nw 3 Gear ratio Input (worm) speed mG 17 n w 1400 rpm

Given:

Use equation 13.13 to calculate the lead. Use equation 13.12 to calculate the lead angle. Calculate the number of teeth on the gear Use equation 13.13 to calculate gear diameter.

atan

d
L

6.228 deg
Ng 51

Ng mG Nw p c p x d g C p c Ng

d dg 2

d g 3.247 in C 2.498 in

5. 6.

Use equation 13.17 to calculate the center distance.

If the lead angle per worm tooth is less than 6 deg, the set is self-locking. The lead angle per tooth is

Nw

t 2.076 deg

Self-locking

MACHINE DESIGN - An Integrated Approach, 4th Ed.

13-11-1

PROBLEM 13-11
Statement: Given: A 1-start wormset has the dimensions given below. Find the lead, lead angle, worm gear diameter, and center distance. Will it self-lock? The input speed is 4500 rpm. Worm pitch diameter Axial pitch Threads on worm Solution: 1. 2. 3. 4. See Mathcad file P1311. L p x Nw L 5 mm d 40 mm p x 5 mm Nw 1 Gear ratio Input (worm) speed mG 82 n w 4500 rpm

Use equation 13.13 to calculate the lead. Use equation 13.12 to calculate the lead angle. Calculate the number of teeth on the gear Use equation 13.13 to calculate gear diameter.

atan

d
L

2.279 deg
Ng 82

Ng mG Nw p c p x d g C p c Ng

d dg 2

d g 130.51 mm C 85.25 mm

5. 6.

Use equation 13.17 to calculate the center distance.

If the lead angle per worm tooth is less than 6 deg, the set is self-locking. The lead angle per tooth is

Nw

t 2.279 deg

Self-locking

MACHINE DESIGN - An Integrated Approach, 4th Ed.

13-12-1

PROBLEM 13-12
Statement: Determine the power transmitted and the torques and forces in the mesh for the wormset in Problem 13-9 if it turns at 1000 worm rpm. fpm ft min
1

Units: Given:

Worm pitch diameter Axial pitch Threads on worm

d w 50 mm p x 10 mm Nw 2

Gear ratio Input (worm) speed Pressure angle

mG 22 n w 1000 rpm

20 deg

Solution: 1. 2. 3. 4. 5.

See Mathcad file P1312. L p x Nw L 20 mm

Use equation 13.13 to calculate the lead. Use equation 13.12 to calculate the lead angle. Calculate the number of teeth on the gear Use equation 13.13 to calculate gear diameter. Use equation 13.17 to calculate the center distance.

atan

Ng mG Nw p c p x d g C p c Ng

d w

7.256 deg
Ng 44

dw dg 2

d g 140.06 mm C 95.03 mm

6.

Find the maximum recommended face width from equation 13.19. Fmax 0.67 d w Fmax 33.5 mm Find the materials factor Cs from equation 13.24. Since C < 203 mm, Cs 1000. Find the ratio correction factor Cm from equations 13.25. Based on mG 22 , the second of the expressions in that equation set will be used. Cm 0.0107 mG 56 mG 5145
2

7. 8.

Cm 0.821

9.

Find the tangential velocity Vt from equation 13.27. Vt nw dw 2 cos( ) Vt 519.5 fpm

10. Use this velocity to find the velocity factor Cv from equations 13.26. For this value of Vt, the first of these equations is appropriate.
0.0011 Vt fpm

Cv 0.659 e

Cv 0.372

11. Find the tangential load Wt from equation 13.23.

dg Wtg Cs Cm Cv mm

0.8

Fmax

N 75.948 mm

Wtg 7028 N

12. Find the coefficient of friction from the third expression in equation 13.29.

Vt 0.110 fpm 0.103 e

0.450

0.012

0.028

MACHINE DESIGN - An Integrated Approach, 4th Ed.


13. Find the friction force Wf from equation 13.28. Wf

13-12-2

Wtg
cos( ) cos( )

Wf 214.5 N

14. Find the rated output power from equation 13.21.

nw dg Wtg mG 2

o 2.34 kW

15. Find the power lost in the mesh from equation 13.22.

l Vt Wf
16. Find the rated input power from equation 13.20.

l 0.57 kW 2.91 kW

o l
17. The efficiency of the gearset is e

e 80.5 %

18. Find the rated output torque from equation 13.31. Tg Wtg dg 2 Tg 492 N m

MACHINE DESIGN - An Integrated Approach, 4th Ed.

13-13-1

PROBLEM 13-13
Statement: Determine the power transmitted and the torques and forces in the mesh for the wormset in Problem 13-10 if it turns at 500 worm rpm. fpm ft min
1

Units: Given:

Worm pitch diameter Axial pitch Threads on worm

d w 1.75 in p x 0.200 in Nw 3

Gear ratio Input (worm) speed Pressure angle

mG 17 n w 500 rpm

20 deg

Solution: 1. 2. 3. 4. 5.

See Mathcad file P1313. L p x Nw L 0.600 in

Use equation 13.13 to calculate the lead. Use equation 13.12 to calculate the lead angle. Calculate the number of teeth on the gear Use equation 13.13 to calculate gear diameter. Use equation 13.17 to calculate the center distance.

atan

Ng mG Nw p c p x d g C p c Ng

d w
L

6.228 deg
Ng 51

dw dg 2

d g 3.247 in C 2.498 in

6.

Find the maximum recommended face width from equation 13.19. Fmax 0.67 d w Fmax 1.173 in Find the materials factor Cs from equation 13.24. Since C < 8 in, Cs 1000. Find the ratio correction factor Cm from equations 13.25. Based on mG 17 , the first of the expressions in that equation set will be used. Cm 0.0200 mG 40 mG 76 0.46
2

7. 8.

Cm 0.815

9.

Find the tangential velocity Vt from equation 13.27. Vt nw dw 2 cos( ) Vt 230.4 fpm

10. Use this velocity to find the velocity factor Cv from equations 13.26. For this value of Vt, the first of these equations is appropriate.
0.0011 Vt fpm

Cv 0.659 e

Cv 0.511

11. Find the tangential load Wt from equation 13.23.

dg Wtg Cs Cm Cv in

0.8

Fmax

lbf in

Wtg 1254 lbf

12. Find the coefficient of friction from the third expression in equation 13.29.
0.110

0.103 e

fpm
Vt

0.450

0.012

0.041

MACHINE DESIGN - An Integrated Approach, 4th Ed.


13. Find the friction force Wf from equation 13.28. Wf

13-13-2

Wtg
cos( ) cos( )

Wf 54.8 lbf

14. Find the rated output power from equation 13.21.

nw dg Wtg mG 2

o 0.95 hp

15. Find the power lost in the mesh from equation 13.22.

l Vt Wf
16. Find the rated input power from equation 13.20.

l 0.38 hp 1.33 hp

o l
17. The efficiency of the gearset is e

e 71.3 %

18. Find the rated output torque from equation 13.31. Tg Wtg dg 2 Tg 2035 in lbf

MACHINE DESIGN - An Integrated Approach, 4th Ed.

13-14-1

PROBLEM 13-14
Statement: Given: If the gearset in Problem 13-3 transmits 125 HP at 1000 pinion rpm, find the torque on each shaft. Tooth numbers: Number of pinion teeth Number of gear teeth Np 23 Ng 57

Pinion speed Transmitted power

p 1000 rpm
P 125 hp

Assumptions: There is no loss of power in the gear mesh (100% efficiency). Solution: See Mathcad file P1314.

See Mathcad file P1214. P Solution: 1. For the pinion shaft Tp 2.

Tp 7878 in lbf

Tp 656.5 ft lbf

The gear shaft will rotate at a lower speed, which is determined by the gear ratio. (The speed will be decreased in proportion to the ratio and the torque will be increased by the reciprocal of the ratio). For the gear shaft

3.

g
Tg

Np Ng P

g 403.509 rpm
Tg 19524 in lbf Tg 1627 ft lbf

4.

We could have calculated the torque on the gear shaft directly without finding the gear shaft speed, Tg Ng Np Tp Tg 19524 in lbf Tg 1627 ft lbf

MACHINE DESIGN - An Integrated Approach, 4th Ed.

13-15-1

PROBLEM 13-15
Statement: Given: If the gearset in Problem 13-4 transmits 33 kW at 1600 pinion rpm, find the torque on each shaft. Tooth numbers: Number of pinion teeth Number of gear teeth Np 27 Ng 78

Pinion speed Transmitted power

p 1600 rpm
P 33 kW

Assumptions: There is no loss of power in the gear mesh (100% efficiency). Solution: 1. 2. See Mathcad file P1315. Tp P

For the pinion shaft

Tp 197 N m

The gear shaft will rotate at a lower speed, which is determined by the gear ratio. (The speed will be decreased in proportion to the ratio and the torque will be increased by the reciprocal of the ratio). For the gear shaft

3.

g
Tg

Np Ng P

g 553.846 rpm
Tg 569 N m

4.

We could have calculated the torque on the gear shaft directly without finding the gear shaft speed, Tg Ng Np Tp Tg 569 N m

MACHINE DESIGN - An Integrated Approach, 4th Ed.

13-16-1

PROBLEM 13-16
Statement: Size the helical gears in problem 13-14 for a bending factor of safety of at least 2 assuming a steady torque, 25 deg pressure angle, full depth teeth, a face width factor of 10, quality index of 9, an AISI 4140 steel pinion, and a class 40 cast iron gear. Factor of safety Power to be transmitted Rotational speed of pinion Number of pinion teeth Pressure angle Solution: Pinion 1. 2. Determine the bending geometry factor, J (Table 13-4) Jp 0.57 See Mathcad file P1316. Nfb 2 H 125 hp n 1000 rpm Np 23 25 deg Number of gear teeth Reliability AGMA Quality level Life (cycles) Helix angle Ng 57 R 0.99 Qv 9 N 10 10 deg
7

Given:

Write the equations for pitch diameter, pitchline velocity, and transmitted load in terms of the unknown diametral pitch, p d. Note that, in Mathcad, unit conversion factors are not included. Pitch diameter of pinion (in) d p d Vt p d Wt p d Ka 1
v

Np pd dpd n 2 H

pitchline velocity (fpm)

Transmitted load (lbf)

Vt p d

3. 4.

Set the application factor, Ka Write the equation for the dynamic load factor, K

B 0.25 12 Qv

0.6667

B 0.52 A 76.878
B

A 50 56 ( 1 B) Kv p d A min A Vt p d ft Km 1.7 Ka Wt p d p d Km Kv p d F Jp

5. 6.

Tentatively choose the mounting factor, Km (Assume 2 < F < 6 in) The bending stress equation for the pinion is

bp p d F

7.

Determine the endurance strength of the pinion. Life factor Reliability Temperature factor Material bending strength (psi) AISI 4140 Nitrided steel KL 1.6831 N
0.0323

KL 1 KR 1 KT 1 S atp 40000 psi

KR 0.7 0.15 log( 1 R)

MACHINE DESIGN - An Integrated Approach, 4th Ed.


Endurance strength 8. S fbp S atp KL KT KR

13-16-2
S fbp 40001 psi

Write the design equations using the range of face-width to diametral-pitch ratio given in the text, and the bending stress equation, solved for the unknown face width. Upper limit FU p d FL p d S fb 16 pd 8 pd

Lower limit

Safety factor

Nfb =

b
Ka Wt p d p d Km Nfb Kv p d Jp S fbp

Face width

F p d

9. Plot F(P) vs. p d over the range

p d

1 1.5 6 in in in
4

10. From the graph, choose a standard value of p d from Table 11-2. p d 5 in
1

F pd in


in

11. The calculated value of F is F p d 3.100 in 12. Round this up to the decimal equivalent of a common fractional value. F 3.125 in

FL pd

FU pd in

13. Then, the parameters that depend on p d and F are: d p d 4.600 in Vt p d 1204 ft min Kv p d 0.824 Wt p d 3425 lbf

2.5

3.5

4 pd in

4.5

5.5

FIGURE 13-16A
Graph of Face Width and Limits for the Pinion in Problem 13-16

bp p d F 19839 psi
The assumption made in step 5 is correct so no further iteration is required. Gear 14. Determine the bending geometry factor, J (Table 13-4) 15. The bending stress equation for the gear is 16. Determine the endurance strength of the pinion. Jg 0.63 Ka Wt p d p d Km Kv p d F Jg

bg p d F

MACHINE DESIGN - An Integrated Approach, 4th Ed.


Material bending strength (psi) Class 40 cast iron Endurance strength S fbg S atg KL KT KR

13-16-3

S atg 13000 psi S fbg 13000 psi

17. Write the design equations using the range of face-width to diametral-pitch ratio given in the text, and the bending stress equation, solved for the unknown face width. Upper limit FU p d FL p d S fb 16 pd 8 pd

Lower limit

Safety factor

Nfb =

b
Ka Wt p d p d Km Nfb Kv p d Jg S fbg

Face width

F p d p d

18. Plot F(P) vs. p d over the range

1 1.5 6 in in in
6 F pd in

19. From the graph, choose a standard value of p d from Table 11-2. p d 3 in
1


in

4.5

20. The calculated value of F is F p d 3.250 in 21. Round this up to the decimal equivalent of a common fractional value. F 3.250 in 22. Then, the parameters that depend on P and F are: d p d 7.667 in Vt p d 2007 ft min Kv p d 0.788 Wt p d 2055 lbf

FL pd

FU pd in


1.5

2.5

3.5

4 pd in

4.5

5.5

FIGURE 13-16B
Graph of Face Width and Limits for the Gear in Problem 13-16

bg p d F 6500 psi
23. The gear dimensions are larger (smaller diametral pitch means bigger teeth) than for the pinion. This means that we will accept the gear requirements for the pinion, thus, for the set Diametral pitch Face width p d 3 in
1

F 3.250 in

24. Determine the realized factor of safety for the gear using the above values for F and p d.

MACHINE DESIGN - An Integrated Approach, 4th Ed.

13-16-4

Gear factor of safety

Nfbg

bg p d F

S fbg

Nfbg 2.0

25. Check the factor of safety on the pinion: Pinion factor of safety Nfbp

bp p d F

S fbp

Nfbp 5.6

MACHINE DESIGN - An Integrated Approach, 4th Ed.

13-17-1

PROBLEM 13-17
Statement: Size the helical gears in problem 13-15 for a bending factor of safety of at least 2.5 assuming a steady torque, 20 deg pressure angle, full depth teeth, a face width factor of 12, quality index of 11, an AISI 4340 steel pinion, and an A-7-d nodular 40 iron gear. Factor of safety Power to be transmitted Rotational speed of pinion Number of pinion teeth Pressure angle Solution: Pinion 1. 2. Determine the bending geometry factor, J (Table 13-4) Jp 0.48 See Mathcad file P1317. Nfb 2.5 H 33 kW n 1600 rpm Np 27 20 deg Number of gear teeth Reliability AGMA Quality level Life (cycles) Helix angle Ng 78 R 0.99 Qv 11 N 10 30 deg
7

Given:

Write the equations for pitch diameter, pitchline velocity, and transmitted load in terms of the unknown diametral pitch, p d. Note that, in Mathcad, unit conversion factors are not included. Pitch diameter of pinion (in) d p d Vt p d Wt p d Ka 1
v

Np pd dpd n 2 H

pitchline velocity (fpm)

Transmitted load (lbf)

Vt p d

3. 4.

Set the application factor, Ka Write the equation for the dynamic load factor, K

B 0.25 12 Qv

0.6667

B 0.25 A 92
B

A 50 56 ( 1 B) Kv p d A min A Vt p d ft Km 1.7 Ka Wt p d p d Km Kv p d F Jp

5. 6.

Tentatively choose the mounting factor, Km (Assume 2 < F < 6 in) The bending stress equation for the pinion is

bp p d F

7.

Determine the endurance strength of the pinion. Life factor Reliability Temperature factor Material bending strength (psi) AISI 4340 Nitrided steel KL 1.6831 N
0.0323

KL 1 KR 1 KT 1 S atp 42000 psi

KR 0.7 0.15 log( 1 R)

MACHINE DESIGN - An Integrated Approach, 4th Ed.


S atp KL KT KR

13-17-2

Endurance strength 8.

S fbp

S fbp 42001 psi

Write the design equations using the range of face-width to diametral-pitch ratio given in the text, and the bending stress equation, solved for the unknown face width. Upper limit FU p d FL p d S fb 16 pd 8 pd

Lower limit

Safety factor

Nfb =

b
Ka Wt p d p d Km Nfb Kv p d Jp S fbp

Face width

F p d

9. Plot F(P) vs. p d over the range

p d

1 1.5 10 in in in
4

10. From the graph, choose a standard value of p d from Table 12-2. p d 8 in
1

F pd in


in

11. The calculated value of F is F p d 1.898 in 12. Round this up to the decimal equivalent of a common fractional value. F 2.000 in

FL pd

FU pd in

13. Then, the parameters that depend on p d and F are: d p d 3.375 in Vt p d 1414 ft min Kv p d 0.918 Wt p d 1033 lbf

7 pd in

10

FIGURE 13-17A
Graph of Face Width and Limits for the Pinion in Problem 13-17

bp p d F 15943 psi
The assumption made in step 5 is correct so no further iteration is required. Gear 14. Determine the bending geometry factor, J (Table 13-4) 15. The bending stress equation for the gear is 16. Determine the endurance strength of the pinion. Jg 0.51 Ka Wt p d p d Km Kv p d F Jg

bg p d F

MACHINE DESIGN - An Integrated Approach, 4th Ed.

13-17-3

Material bending strength (psi) A-7-d nodular iron Endurance strength S fbg S atg KL KT KR

S atg 34000 psi S fbg 34001 psi

17. Write the design equations using the range of face-width to diametral-pitch ratio given in the text, and the bending stress equation, solved for the unknown face width. Upper limit FU p d FL p d S fb 16 pd 8 pd

Lower limit

Safety factor

Nfb =

b
Ka Wt p d p d Km Nfb Kv p d Jg S fbg

Face width

F p d p d

18. Plot F(P) vs. p d over the range

1 1.5 10 in in in
6 F pd in FL pd in FU pd in

19. From the graph, choose a standard value of p d from Table 12-2. p d 8 in
1

4.5

20. The calculated value of F is F p d 2.207 in 21. Round this up to the decimal equivalent of a common fractional value. F 2.250 in 22. Then, the parameters that depend on P and F are: d p d 3.375 in Vt p d 1414 ft min Kv p d 0.918


1.5

7 pd in

10

FIGURE 13-17B Wt p d 1033 lbf


Graph of Face Width and Limits for the Gear in Problem 13-17

bg p d F 13338 psi
23. The gear dimensions are larger (smaller diametral pitch means bigger teeth) than for the pinion. This means that we will accept the gear requirements for the pinion, thus, for the set Diametral pitch Face width p d 8 in
1

F 2.250 in

24. Determine the realized factor of safety for the gear using the above values for F and p d.

MACHINE DESIGN - An Integrated Approach, 4th Ed.

13-17-4

Gear factor of safety

Nfbg

bg p d F

S fbg

Nfbg 2.5

25. Check the factor of safety on the pinion: Pinion factor of safety Nfbp

bp p d F

S fbp

Nfbp 3.0

MACHINE DESIGN - An Integrated Approach, 4th Ed.

13-18-1

PROBLEM 13-18
Statement: Size the helical gears in problem 13-14 for a surface factor of safety of at least 1.6 assuming a steady torque, 25 deg pressure angle, full depth teeth, a face width factor of 10, quality index of 9, an AISI 4140 steel pinion, and a class 40 cast iron gear. Factor of safety Power to be transmitted Rotational speed of pinion Number of pinion teeth Pressure angle Solution: Pinion 1. Determine the surface geometry factor, I. Addendum a p d rp p d 1 pd Np 2 pd rg p d Ng 2 pd See Mathcad file P1318. Nfc 1.6 H 125 hp n 1000 rpm Np 23 25 deg Number of gear teeth Reliability AGMA Quality level Life (cycles) Helix angle Ng 57 R 0.99 Qv 9 N 10 10 deg
7

Given:

Pitch radii

Center distance Radii of curvature

C p d rp p d rg p d

p p d

2 2 0.5 rp p d C pd rg pd rp p d cos( )

g p d C p d sin( ) p p d
Normal plane pressure angle

n atan( tan( ) cos( ) ) b acos cos( )

n 24.666 deg

Base helix angle Length of action Z p d

cos n cos( )

b 9.079 deg

r p a p 2 r p cos( ) 2 d p d p d r p a p 2 r p cos( ) 2 C p sin( ) d g d d g d


mp p d Z pd

Transverse contact ratio

pd

cos( ) F p d tan( )

Axial contact ratio Fractional parts of mp and mF

mF p d F

n r p d mp p d floor mp p d

MACHINE DESIGN - An Integrated Approach, 4th Ed.


n a p d F mF p d F floor mF p d F Axial pitch p t p d p x p d Minimum length of lines of contact Lmin p d F

13-18-2

pd pn pd sin( )

p n p d p t p d cos( )

return

return

mp pd F napd F nrpd pxpd if n a p d F 1 n r p d cos b mp pd F 1 na p d F 1 nr pd px p d if n p F 1 n p a d r d cos b


mN p d F Lmin p d F cos( ) F

Load sharing ratio

Geometry factor

I p d F

1 1 2 r p m p F p p p d N d g d p d

2.

Write the equations for pitch diameter, pitchline velocity, and transmitted load in terms of the unknown diametral pitch, p d. Note that, in Mathcad, unit conversion factors are not included. Pitch diameter of pinion (in) d p p d Vt p d Wt p d Ca 1
v

Np pd d p p d n 2 Vt p d H

d g p d

Ng pd

pitchline velocity (fpm) Transmitted load (lbf) 3. 4. Set the application factor, Ca Write the equation for the dynamic load factor, C

B 0.25 12 Qv

0.6667

B 0.52 A 76.878
B

A 50 56 ( 1 B) Cv p d A min A Vt p d ft Cm 1.7 Cp 2100 psi


0.5

5. 6. 7.

Tentatively choose the mounting factor, Cm (Assume 2 < F < 6 in Choose an elastic coefficient from Table 12-18 (steel on CI). The surface stress equation for the pinion is

cp p d F Cp

Cv p d F d p p d I p d F

C a W t p d C m

MACHINE DESIGN - An Integrated Approach, 4th Ed.


8. Determine the endurance strength of the pinion. Life factor Reliability Temperature factor Material surface strength (psi) AISI 4140 steel Endurance strength 9. S fcp S ac CL CT CR CL 2.466 N
0.056

13-18-3

CL 1 CR 1 CT 1 S ac 165000 psi S fcp 164997 psi

CR 0.7 0.15 log( 1 R)

Write the design equations using the range of face-width to diametral-pitch ratio given in the text, and the bending stress equation, solved for the unknown face width. Limits, safety factor FU p d 16 pd FL p d 8 pd
2

Nfc =

Sfc c

Face width

Cp F' p d F Nfc Cv p d d p p d I p d F S fcp


F 2.347 in p d 2 2.5 12 in in in
4 F' pd F in FL pd in FU pd in

C a W t p d C m

Guess value for F 10. Plot F(P) vs. p d over the range

Found by iteration with value in step 12.

11. From the graph, choose a standard value of p d from Table 12-2. p d 5 in
1

12. The calculated value of F is F' p d F 2.347 in 13. Round this up to the decimal equivalent of a common fractional value. F 2.375 in 14. Then, the parameters that depend on p d and F are: d p p d 4.600 in Vt p d 1204 ft min d g p d 11.400 in Wt p d 3425 lbf mp p d 1.491 mN p d F 0.798

7 pd in

9 10 11 12

FIGURE 13-18A
Graph of Face Width and Limits for the Pinion (surface) in Problem 13-18

cp p d F 129197 psi
n a p d F 0.667

mF p d F 0.667 Lmin p d F 2.975 in

n r p d 0.491 I p d F 0.171

MACHINE DESIGN - An Integrated Approach, 4th Ed.


Gear 15. The surface stress equation for the gear is 16. Determine the endurance strength of the gear. Life factor Reliability Temperature factor Material surface strength (psi) Class 40 CI Endurance strength S fcg S ac CL CT CR CL 2.466 N
0.056

13-18-4

cg p d F Cp

Cv p d F d g p d I p d F

C a W t p d C m

CL 1 CR 1 CT 1 S ac 80000 psi S fcg 79999 psi

CR 0.7 0.15 log( 1 R)

17. Write the design equations using the range of face-width to diametral-pitch ratio given in the text, and the bending stress equation, solved for the unknown face width. Limits, safety factor FU p d 16 pd FL p d 8 pd

Sfc Nfc = c

Face width

F' p d F

2 Cp Nfc Cv p d d g p d I p d F S fcg

C a W t p d C m

Guess value for F 18. Plot F(P) vs. p d over the range p d

F 2.745 in 2 2.5 12 in in in

Found by iteration with value in step 20.

19. From the graph, choose a standard value of p d from Table 12-2. p d 4 in
1

F' pd F in FL pd in FU pd in

20. The calculated value of F is F' p d F 2.746 in 21. Round this up to the decimal equivalent of a common fractional value. F 2.750 in 22. Then, the parameters that depend on p d and F are: d p p d 5.750 in Vt p d 1505 ft min d g p d 14.250 in Wt p d 2740 lbf mp p d 1.491

2 2.5 3 3.5 4 4.5 5 5.5 6 6.5 7 pd in

FIGURE 13-18B
Graph of Face Width and Limits for the Gear (surface) in Problem 13-18

cp p d F 99427 psi

mF p d F 0.617

n r p d 0.491

MACHINE DESIGN - An Integrated Approach, 4th Ed.


n a p d F 0.617 mN p d F 0.84 Lmin p d F 3.276 in

13-18-5
I p d F 0.163

23. The gear teeth are larger than for the pinion. This means that we will accept the gear requirements for the pinion thus, for the set Diametral pitch Face width p d 4 in
1

F 2.750 in

24. Determine the realized factor of safety for the gear using the above values for F and p d.

Gear factor of safety

Sfcg Nfsg cg pd F

Nfsg 1.6

25. Check the factor of safety on the pinion: Pinion factor of safety

Sfcp Nfsp cp pd F

Nfsp 2.8

MACHINE DESIGN - An Integrated Approach, 4th Ed.

13-19-1

PROBLEM 13-19
Statement: Size the helical gears in problem 13-15 for a surface factor of safety of at least 1.2 assuming a steady torque, 20 deg pressure angle, full depth teeth, a face width factor of 12, quality index of 11, an AISI 4340 steel pinion, and an A-7-d nodular iron gear. Factor of safety Power to be transmitted Rotational speed of pinion Number of pinion teeth Pressure angle Solution: Pinion 1. Determine the surface geometry factor, I. Addendum a p d rp p d 1 pd Np 2 pd rg p d Ng 2 pd See Mathcad file P1319. Nfc 1.2 H 33 kW n 1600 rpm Np 27 20 deg Number of gear teeth Reliability AGMA Quality level Life (cycles) Helix angle Ng 78 R 0.99 Qv 11 N 10 30 deg
7

Given:

Pitch radii

Center distance Radii of curvature

C p d rp p d rg p d

p p d

2 2 0.5 rp p d C pd rg pd rp p d cos( )

g p d C p d sin( ) p p d
Normal plane pressure angle

n atan( tan( ) cos( ) ) b acos cos( )

n 17.495 deg

Base helix angle Length of action Z p d

cos n cos( )

b 28.481 deg

r p a p 2 r p cos( ) 2 d p d p d r p a p 2 r p cos( ) 2 C p sin( ) d g d d g d


mp p d Z pd

Transverse contact ratio

pd

cos( ) F p d tan( )

Axial contact ratio Fractional parts of mp and mF

mF p d F

n r p d mp p d floor mp p d

MACHINE DESIGN - An Integrated Approach, 4th Ed.


n a p d F mF p d F floor mF p d F Axial pitch p t p d p x p d Minimum length of lines of contact Lmin p d F

13-19-2

pd pn pd sin( )

p n p d p t p d cos( )

return

return

mp pd F napd F nrpd pxpd if n a p d F 1 n r p d cos b mp pd F 1 na p d F 1 nr pd px p d if n p F 1 n p a d r d cos b


mN p d F I p d F Lmin p d F cos( ) F

Load sharing ratio

Geometry factor

1 1 2 r p m p F p p p d N d g d p d

2.

Write the equations for pitch diameter, pitchline velocity, and transmitted load in terms of the unknown diametral pitch, p d. Note that, in Mathcad, unit conversion factors are not included. Pitch diameter of pinion (in) d p p d Vt p d Wt p d Ca 1
v

Np pd d p p d n 2 Vt p d H

d g p d

Ng pd

pitchline velocity (fpm) Transmitted load (lbf) 3. 4. Set the application factor, Ca Write the equation for the dynamic load factor, C

B 0.25 12 Qv

0.6667

B 0.25 A 92
B

A 50 56 ( 1 B) Cv p d A min A Vt p d ft Cm 1.6 Cp 2160 psi


0.5

5. 6.

Tentatively choose the mounting factor, Cm (Assume 0 < F < 2 in Choose an elastic coefficient from Table 11-18 (steel on NI).

7.

The surface stress equation for the pinion is

cp p d F Cp

Cv p d F d p p d I p d F

C a W t p d C m

MACHINE DESIGN - An Integrated Approach, 4th Ed.


8. Determine the endurance strength of the pinion. Life factor Reliability Temperature factor Material surface strength (psi) AISI 4340 steel Endurance strength 9. S fcp S ac CL CT CR CL 2.466 N
0.056

13-19-3

CL 1 CR 1 CT 1 S ac 160000 psi S fcp 159997 psi

CR 0.7 0.15 log( 1 R)

Write the design equations using the range of face-width to diametral-pitch ratio given in the text, and the bending stress equation, solved for the unknown face width. Limits, safety factor FU p d 16 pd FL p d 8 pd
2

Nfc =

Sfc c

Face width

Cp F' p d F Nfc Cv p d d p p d I p d F S fcp


F 1.140 in p d 6 6.1 16 in in in
2 F' pd F in FL pd in FU pd in

C a W t p d C m

Guess value for F 10. Plot F(P) vs. p d over the range

Found by iteration with value in step 12.

11. From the graph, choose a standard value of p d from Table 12-2. p d 12 in
1

1.5

12. The calculated value of F is F' p d F 1.140 in 13. Round this up to the decimal equivalent of a common fractional value. F 1.250 in 14. Then, the parameters that depend on p d and F are: d p p d 2.250 in Vt p d 942 ft min d g p d 6.500 in Wt p d 1550 lbf mp p d 1.726 mN p d F 0.516


0.5

9 10 11 12 13 14 15 16 pd in

FIGURE 13-19A
Graph of Face Width and Limits for the Pinion (surface) in Problem 13-19

cp p d F 138282 psi
n a p d F 0.757

mF p d F 2.757 Lmin p d F 2.42 in

n r p d 0.726 I p d F 0.231

MACHINE DESIGN - An Integrated Approach, 4th Ed.


Gear 15. The surface stress equation for the gear is 16. Determine the endurance strength of the gear. Life factor Reliability Temperature factor Material surface strength (psi) A-7-d nodular iron Endurance strength S fcg S ac CL CT CR CL 2.466 N
0.056

13-19-4

cg p d F Cp

Cv p d F d g p d I p d F

C a W t p d C m

CL 1 CR 1 CT 1 S ac 100000 psi S fcg 99998 psi

CR 0.7 0.15 log( 1 R)

17. Write the design equations using the range of face-width to diametral-pitch ratio given in the text, and the bending stress equation, solved for the unknown face width. Limits, safety factor FU p d 16 pd FL p d 8 pd

Sfc Nfc = c

Face width

F' p d F

2 Cp Nfc Cv p d d g p d I p d F S fcg

C a W t p d C m

Guess value for F 18. Plot F(P) vs. p d over the range p d

F 1.028 in 6 6.1 16 in in in

Found by iteration with value in step 20.

19. From the graph, choose a standard value of p d from Table 12-2. p d 12 in
1

F' pd F in FL pd in FU pd in

1.5

20. The calculated value of F is F' p d F 1.030 in 21. Round this up to the decimal equivalent of a common fractional value. F 1.125 in 22. Then, the parameters that depend on p d and F are: d p p d 2.250 in Vt p d 942 ft min d g p d 6.500 in Wt p d 1550 lbf mp p d 1.726


0.5

9 10 11 12 13 14 15 16 pd in

FIGURE 13-19B
Graph of Face Width and Limits for the Gear (surface) in Problem 13-19

cp p d F 147201 psi

mF p d F 2.481

n r p d 0.726

MACHINE DESIGN - An Integrated Approach, 4th Ed.


n a p d F 0.481 mN p d F 0.527 Lmin p d F 2.136 in

13-19-5
I p d F 0.227

23. The gear teeth are the same size as the pinion. This means that we will accept the gear requirements for the pinion thus, for the set Diametral pitch Face width p d 12 in
1

F 1.125 in

24. Determine the realized factor of safety for the gear using the above values for F and p d.

Gear factor of safety

Sfcg Nfsg cg pd F

Nfsg 1.3

25. Check the factor of safety on the pinion: Pinion factor of safety

Sfcp Nfsp cp pd F

Nfsp 1.2

MACHINE DESIGN - An Integrated Approach, 4th Ed.

13-20-1

PROBLEM 13-20
Statement: Given: Size the bevel gears in problem 13-5 for a bending factor of safety of at least 2 assuming a 5-year, 1-shift life, steady torque, quality index of 9, an AISI 4140 steel pinion and gear. Factor of safety Power to be transmitted (hp) Rotational speed of pinion (rpm) Number of pinion teeth Solution: Pinion 1. 2. Determine the bending geometry factor, J (Figure 13-5) Jp 0.285 See Mathcad file P1320. Nfb 2 Number of gear teeth Ng 126 R 0.99 Qv 9 Life 5 yr Reliability H 746 W AGMA Quality level n p 1000 rpm Np 14 Life (years)

Write the equations for pitch diameter, pitchline velocity, and transmitted torque in terms of the unknown diametral pitch, p d. Note that, in Mathcad, unit conversion factors are not included. Pitch diameter of pinion (in) d p p d Vt p d Tp H np Ks 1
0.6667

Np pd d p p d n p 2

d g p d

Ng pd

pitchline velocity (fpm)

Transmitted torque (in-lbf)

Tp 63.051 in lbf Kx 1 B 0.52 A 76.878


B

3. Set the application, size, and type factors. 4. Write the equation for the dynamic load factor, Kv

Ka 1

B 0.25 12 Qv

A 50 56 ( 1 B) Kv p d A min A Vt p d ft Km 1.6

5. Tentatively choose the mounting factor, Km (Assume 0 < F < 2 in) 6. The bending stress equation for the pinion is

bp p d F

d p p d F Jp Kv p d Kx

2 Tp

pd

Ka Km Ks

7. Determine the endurance strength of the pinion. Cycle life Life factor Reliability Temperature factor Material bending strength (psi) AISI 4140 Nitrided steel Endurance strength S fbp S atp KL KT KR N Life np 2
0.0323

N 2.63 10 KL 0.835 KR 1 KT 1

KL 1.6831 N

KR 0.7 0.15 log( 1 R)

S atp 40000 psi S fbp 33412 psi

MACHINE DESIGN - An Integrated Approach, 4th Ed.

13-20-2

8. Write the design equations using the face-width to pitch-cone length ratio given in the text, and the bending stress equation, solved for the unknown face width. Upper limit, F = L/3 FLover3 p d Np 6 p d sin atan

Np Ng

Safety factor

Nfb =

S fb

b
d p p d Jp Kv p d Kx S fbp 2 Tp p d Ka Km Ks Nfb

Face width

F p d

9. Plot F(P) vs. p d over the range

p d

16 16.5 26 in in in

10. From the graph, choose a standard value of p d from Table 12-2. p d 18 in
1

F pd in

1.5

11. The calculated value of F is F p d 1.071 in 12. Round this to the decimal equivalent of a common fractional value. F 1.125 in 13. Then, the parameters that depend on p d and F are: d p p d 0.778 in Vt p d 204 ft min Kv p d 0.915

FLover3 pd in

0.5

0 16

18

20

22 pd in

24

26

FIGURE 13-20A
Graph of Face Width and Limits for the Pinion in Problem 13-20

bp p d F 15912 psi
The assumption made in step 5 is correct so no further iteration is required. GEAR 14. Determine the bending geometry factor, J (Figure 13-5) 15. The bending stress equation for the gear is Jg 0.220

bg p d F

d g p d F Jg Kv p d Kx

2 Tp

pd

Ka Km Ks

16. Write the design equations using the face-width to pitch-cone length ratio given in the text, and the bending stress equation, solved for the unknown face width. F = L/3 FLover3 p d Np 6 p d sin atan

Np Ng

MACHINE DESIGN - An Integrated Approach, 4th Ed.


S fb

13-20-3

Safety factor

Nfb =

b
d g p d Jg Kv p d Kx S fbp 2 Tp p d Ka Km Ks Nfb

Face width

F p d

17. Plot F(P) vs. p d over the range p d

30 30.5 40 in in in

18. From the graph, choose a standard value of p d from Table 12-2. p d 36 in
1

F pd in

1.5

19. The calculated value of F is F p d 0.602 in 20. Round this to the decimal equivalent of a common fractional value. F 0.625 in 21. Then, the parameters that depend on p d and F are: d g p d 3.500 in Vt p d 102 ft min Kv p d 0.938

FLover3 pd in

0.5

0 30

32

34

36 pd in

38

40

FIGURE 13-20B
Graph of Face Width and Limits for the Gear in Problem 13-20

bg p d F 16093 psi

22. The gear dimensions are smaller (larger diametral pitch means smaller teeth) than for the pinion. This means that we will accept the pinion requirements for the gear, thus, for the set Diametral pitch Face width Pitch cone angle p d 18 in
1

F 1.125 in

p atan

Np Ng

p 6.34 deg
L 3

Pitch-cone length

2 sin p

d p p d

L 3.522 in

1.174 in

23. Determine the realized factor of safety for the pinion using the above values for F and p d. Pinion factor of safety Nfbp

bp p d F

S fbp

Nfbp 2.1

25. Check the factor of safety on the gear: Gear factor of safety Nfbg

bg p d F

S fbp

Nfbg 14.6

MACHINE DESIGN - An Integrated Approach, 4th Ed.

13-21-1

PROBLEM 13-21
Statement: Units: Given: Size the bevel gears in problem 13-6 for a bending factor of safety of at least 2.5 assuming a 15-year, 3-shift life, steady torque, quality index of 11, an AISI 4340 steel pinion and gear. yr 2080 hr Factor of safety Power to be transmitted (hp) Rotational speed of pinion (rpm) Number of pinion teeth Solution: Pinion 1. 2. Determine the bending geometry factor, J (Figure 13-5) Jp 0.267 See Mathcad file P1321. Nfb 2.5 H 7460 W n p 800 rpm Np 18 Number of gear teeth Reliability AGMA Quality level Life (years) Ng 81 R 0.99 Qv 11 Life 5 yr

Write the equations for pitch diameter, pitchline velocity, and transmitted torque in terms of the unknown diametral pitch, p d. Note that, in Mathcad, unit conversion factors are not included. d p p d Vt p d Tp H np Ks 1
0.6667

Pitch diameter of pinion (in)

Np pd d p p d n p 2

d g p d

Ng pd

pitchline velocity (fpm)

Transmitted torque (in-lbf)

Tp 788.134 in lbf Kx 1 B 0.25 A 92


B

3. Set the application, size, and type factors. 4. Write the equation for the dynamic load factor, Kv

Ka 1

B 0.25 12 Qv

A 50 56 ( 1 B) Kv p d A min A Vt p d ft Km 1.6 2 Tp pd

5. Tentatively choose the mounting factor, Km (Assume 0 < F < 2 in)

6. The bending stress equation for the pinion is

bp p d F

d p p d F Jp Kv p d Kx

Ka Km Ks

7. Determine the endurance strength of the pinion. Cycle life Life factor Reliability Temperature factor Material bending strength (psi) AISI 4340 Nitrided steel shifts 3 N Life shifts KL 1.6831 N np 2
0.0323

N 1.498 10 KL 0.851 KR 1 KT 1

KR 0.7 0.15 log( 1 R)

S atp 42000 psi

MACHINE DESIGN - An Integrated Approach, 4th Ed.

13-21-2

Endurance strength

S fbp

S atp KL KT KR

S fbp 35727 psi

8. Write the design equations using the face-width to pitch-cone length ratio given in the text, and the bending stress equation, solved for the unknown face width. Upper limit, F = L/3 FLover3 p d Np 6 p d sin atan

Np Ng

Safety factor

Nfb =

S fb

b
d p p d Jp Kv p d Kx S fbp 2 Tp p d Ka Km Ks Nfb

Face width

F p d

9. Plot F(P) vs. p d over the range

p d

2 2.5 12 in in in
4

10. From the graph, choose a standard value of p d from Table 12-2. p d 6 in
1 F pd in FLover3 pd in

11. The calculated value of F is F p d 1.404 in 12. Round this to the decimal equivalent of a common fractional value. F 1.500 in 13. Then, the parameters that depend on p d and F are: d p p d 3.000 in Vt p d 628 ft min Kv p d 0.942

2
1

6 pd in

10

12

FIGURE 13-21A
Graph of Face Width and Limits for the Pinion in Problem 13-21

bp p d F 13376 psi
The assumption made in step 5 is correct so no further iteration is required. GEAR 14. Determine the bending geometry factor, J (Figure 13-5) 15. The bending stress equation for the gear is Jg 0.221

bg p d F

d g p d F Jg Kv p d Kx

2 Tp

pd

Ka Km Ks

16. Write the design equations using the face-width to pitch-cone length ratio given in the text, and the bending stress equation, solved for the unknown face width.

MACHINE DESIGN - An Integrated Approach, 4th Ed.


FLover3 p d Np 6 p d sin atan

13-21-3

F = L/3

Np Ng

Safety factor

Nfb =

S fb

b
d g p d Jg Kv p d Kx S fbp 2 Tp p d Ka Km Ks Nfb

Face width

F p d

17. Plot F(P) vs. p d over the range p d

6 6.5 16 in in in
2

18. From the graph, choose a standard value of p d from Table 12-2. p d 10 in
1 F pd in FLover3 pd in

1.5

19. The calculated value of F is F p d 1.034 in 20. Round this to the decimal equivalent of a common fractional value. F 1.125 in 21. Then, the parameters that depend on p d and F are: d g p d 8.100 in Vt p d 377 ft min Kv p d 0.953

0.5

10

12 pd in

14

16

FIGURE 13-21B
Graph of Face Width and Limits for the Gear in Problem 13-21

bg p d F 13137 psi

22. The gear dimensions are smaller (larger diametral pitch means smaller teeth) than for the pinion. This means th we will accept the pinion requirements for the gear, thus, for the set Diametral pitch Face width Pitch cone angle p d 6 in
1

F 1.500 in

p atan

Np Ng

p 12.529 deg
L 3

Pitch-cone length

2 sin p

d p p d

L 6.915 in

2.305 in

23. Determine the realized factor of safety for the pinion using the above values for F and p d. Pinion factor of safety 25. Check the factor of safety on the gear: Gear factor of safety Nfbg Nfbp

bp p d F

S fbp

Nfbp 2.7

bg p d F

S fbp

Nfbg 9.9

MACHINE DESIGN - An Integrated Approach, 4th Ed.

13-22-1

PROBLEM 13-22
Statement: Units: Given: Size the spiral gears in problem 13-7 for a bending factor of safety of at least 2.2 assuming a 10-year, 3-shift life, steady torque, quality index of 8, an AISI 4340 steel pinion and gear. yr 2080 hr Factor of safety Power to be transmitted Rotational speed of pinion Number of pinion teeth Solution: Pinion 1. 2. Determine the bending geometry factor, J (Figure 13-8) Jp 0.305 See Mathcad file P1322. Nfb 2.2 Number of gear teeth Ng 80 Reliability H 3 hp R 0.99 n p 600 rpm AGMA Quality level Qv 8 Np 16 Life (years) Life 10 yr

Write the equations for pitch diameter, pitchline velocity, and transmitted torque in terms of the unknown diametral pitch, p d. Note that, in Mathcad, unit conversion factors are not included. Pitch diameter of pinion (in) d p p d Vt p d Tp H np Ks 1
0.6667

Np pd d p p d n p 2

d g p d

Ng pd

pitchline velocity (fpm)

Transmitted torque (in-lbf)

Tp 315.127 in lbf Kx 1.15 B 0.63 A 70.721


B

3. 4.

Set the application, size, and type factors. Write the equation for the dynamic load factor, K
v

Ka 1

B 0.25 12 Qv

A 50 56 ( 1 B) Kv p d A min A Vt p d ft Km 1.6

5. 6. 7.

Tentatively choose the mounting factor, Km (Assume 0 < F < 2 in) The bending stress equation for the pinion is Determine the endurance strength of the pinion. Cycle life Life factor Reliability Temperature factor Material bending strength (psi) AISI 4340 Nitrided steel shifts 3 N Life shifts KL 1.6831 N np

bp p d F

d p p d F Jp Kv p d Kx N 2.246 10 KL 0.84 KR 1 KT 1 S atp 42000 psi


9

2 Tp

pd

Ka Km Ks

2
0.0323

KR 0.7 0.15 log( 1 R)

MACHINE DESIGN - An Integrated Approach, 4th Ed.


S atp KL KT KR

13-22-2

Endurance strength 8.

S fbp

S fbp 35262 psi

Write the design equations using the face-width to pitch-cone length ratio given in the text, and the bending stress equation, solved for the unknown face width. Upper limit, F = L/3 FLover3 p d Np 6 p d sin atan

Np Ng

Safety factor

Nfb =

S fb

b
d p p d Jp Kv p d Kx S fbp 2 Tp p d Ka Km Ks Nfb

Face width

F p d p d

9. Plot F(P) vs. p d over the range

2 2.5 12 in in in
4

10. From the graph, choose a standard value of p d from Table 12-2. p d 10 in
1

F pd in

11. The calculated value of F is F p d 1.273 in 12. Round this to the decimal equivalent of a common fractional value. F 1.375 in

FLover3 pd in

2
1

13. Then, the parameters that depend on p d and F are: d p p d 1.600 in Vt p d 251 ft min Kv p d 0.880

6 pd in

10

12

FIGURE 13-21A
Graph of Face Width and Limits for the Pinion in Problem 13-21

bp p d F 14844 psi
The assumption made in step 5 is correct so no further iteration is required. Gear 14. Determine the bending geometry factor, J (Figure 13-8) Jg 0.302

15. The bending stress equation for the gear is

bg p d F

d g p d F Jg Kv p d Kx

2 Tp

pd

Ka Km Ks

16. Write the design equations using the face-width to pitch-cone length ratio given in the text, and the bending stress equation, solved for the unknown face width.

MACHINE DESIGN - An Integrated Approach, 4th Ed.

13-22-3

F = L/3

FLover3 p d

Np 6 p d sin atan

Np Ng

Safety factor

Nfb =

S fb

b
d g p d Jg Kv p d Kx S fbp 2 Tp p d Ka Km Ks Nfb

Face width

F p d

17. Plot F(P) vs. p d over the range p d

10 11.5 20 in in in
2

18. From the graph, choose a standard value of p d from Table 12-2. p d 16 in
1 F pd in FLover3 pd in

1.5

19. The calculated value of F is F p d 0.642 in 20. Round this to the decimal equivalent of a common fractional value. F 0.750 in 21. Then, the parameters that depend on p d and F are: d g p d 5.000 in Vt p d 157 ft min Kv p d 0.902

0.5

0 10

12

14

16 pd in

18

20

FIGURE 13-21B
Graph of Face Width and Limits for the Gear in Problem 13-21

bg p d F 13730 psi

22. The gear dimensions are smaller (larger diametral pitch means smaller teeth) than for the pinion. This means tha we will accept the pinion requirements for the gear, thus, for the set Diametral pitch Face width Pitch cone angle p d 10 in
1

F 1.375 in

p atan

Np Ng

p 11.31 deg
L 3

Pitch-cone length

2 sin p

d p p d

L 4.079 in

1.360 in

23. Determine the realized factor of safety for the pinion using the above values for F and p d. Pinion factor of safety 25. Check the factor of safety on the gear: Gear factor of safety Nfbg Nfbp

bp p d F

S fbp

Nfbp 2.4

bg p d F

S fbp

Nfbg 11.8

MACHINE DESIGN - An Integrated Approach, 4th Ed.

13-23-1

PROBLEM 13-23
Statement: Size the bevel gears in problem 13-5 for a minimum safety factor of 1.4 for any mode of failure of pinion or gear assuming a 5-year, 1-shift life, steady torque, quality index of 9, an AISI 4140 steel pinion and gear. yr 2080 hr Factor of safety Power to be transmitted (hp) Rotational speed of pinion (rpm) Number of pinion teeth Nfc 1.4 Number of gear teeth Ng 126 R 0.99 Qv 9 Life 5 yr

Units: Given:

Reliability H 746 W AGMA Quality level n p 1000 rpm Np 14 Life (years)

Assumptions: 1. If both pinion and gear are the same material, it will only be necessary to determine the pinion size as it will be governing for the set. 2. If the gears are not surface hardened, it will only be necessary to design to the surface requirement as it will be governing for both bending and surface stresses. Solution: 1. 2. See Mathcad file P1323. I 0.092

Determine the surface geometry factor, I, from Figure 13-6.

Write the equations for pitch diameter, pitchline velocity, and transmitted load in terms of the unknown diametral pitch, p d. Note that, in Mathcad, unit conversion factors are not included. Pitch diameter of pinion (in) d p p d Vtp p d Tp H np Cs 1
0.6667

Np pd Np 2 pd np

d g p d

Ng pd

pitchline velocity (fpm)

Transmitted load (lbf) 3. 4. Set the application factor, Ca Write the equation for the dynamic load factor, C
v

Tp 63.051 in lbf Cf 1 B 0.52 A 76.878


B

Ca 1

B 0.25 12 Qv

A 50 56 ( 1 B) Cvp p d A min A Vtp pd ft

5. 6. 7.

Tentatively choose the mounting factor, Cmd Choose an elastic coefficient from Table 11-18 (steel on steel) and a stress adjustment constant. Determine the endurance strength of the pinion. Cycle life Life factor Reliability shifts 1

Cmd 3.0 Cp 2100 psi np 2


0.056 0.5

Cxc 1 Cb 0.634

N Life shifts CL 2.466 N

N 6.24 10 CL 0.793 CR 1

CR 0.7 0.15 log( 1 R)

MACHINE DESIGN - An Integrated Approach, 4th Ed.


Hardness Temperature factor Material surface strength (psi) AISI 4140 Nitrided steel Endurance strength 8. Write the equation for the design pinion torque. S fcp S acp CL CT CR
2

13-23-2
CH 1 CT 1 S acp 167500 psi S fcp 132885 psi

Sacp d p pd 0.774 CH TD p d F 2 Cs Cmd Cf Ca Cxc Cp Cb CT CR


F 9. Define the exponent z. z p d F

I Cvp p d

return 0.667 if Tp TD p d F 1 otherwise

10. The surface stress equation for the pinion is

2 TD pd F Tp cp p d F Cp Cb F d p pd 2 I TD p d F

z pd F

Cvp p d

Ca Cmd

Cs Cf Cxc

11. Write the design equations using the face-width to pitch-cone length ratio given in the text, and the bending stress equation, solved for the unknown face width. Upper limit, F = L/3 FLover3 p d Np 6 p d sin atan Face width

Np Ng

Sfc Nfc = c

Cp Cb F' p d F S fcp
12. Plot F(P) vs. p d over the range

2 TD p d F Tp d p pd 2 I TD pd F
12 12.5 22 in in in

z pd F

Cvp p d

Ca Cmd

Cs Cf Cxc Nfc

p d

Guess value for F


2

F 0.955 in

13. From the graph, choose a standard value of p d from Table 12-2. p d 16 in
1 F' pd F in

1.5

14. The calculated value of F is F' p d F 0.956 in 15. Round this to the decimal equivalent of a common fractional value. F 1.000 in 16. Then, the parameters that depend on p d and F are: d p p d 0.875 in d g p d 7.875 in
in

FLover3 pd

0.5

0 12

14

16

18 pd in

20

22

FIGURE 13-23
Graph of Face Width and Limits for for Problem 13-23

MACHINE DESIGN - An Integrated Approach, 4th Ed.


Vtp p d 229 Cvp p d 0.911

13-23-3
TD p d F 101 in lbf

ft min

cp p d F 1 10 psi
5

17. When both pinion and gear are of the same material, the stress in the gear will always be lower than in the pinion Therefore, the face width and diametral pitch found for the pinion should also be used for the gear. Diametral pitch Face width Pitch cone angle p d 16 in
1

F 1.000 in

p atan

Np Ng

p 6.34 deg
L 3

Pitch-cone length

2 sin p

d p p d

L 3.962 in

1.321 in

18. Determine the realized factor of safety for the pinion using the above values for F and p d. Nfcp

Pinion factor of safety

Sfcp p F cp d

Nfcp 1.4

MACHINE DESIGN - An Integrated Approach, 4th Ed.

13-24-1

PROBLEM 13-24
Statement: Units: Given: Size the bevel gears in problem 13-6 for a surface factor of safety of at least 1.3 assuming a 15-year, 3-shift life, steady torque, quality index of 11 an AISI 4340 steel pinion and gear. yr 2080 hr Factor of safety Power to be transmitted (hp) Rotational speed of pinion (rpm) Number of pinion teeth Solution: 1. 2. See Mathcad file P1324. I 0.091 Nfc 1.3 H 7460 W n p 800 rpm Np 18 Number of gear teeth Reliability AGMA Quality level Life (years) Ng 81 R 0.99 Qv 11 Life 15 yr

Determine the surface geometry factor, I, from Figure 13-6.

Write the equations for pitch diameter, pitchline velocity, and transmitted load in terms of the unknown diametral pitch, p d. Note that, in Mathcad, unit conversion factors are not included. Pitch diameter of pinion (in) d p p d Vtp p d Tp H np Cs 1
0.6667

Np pd Np 2 pd np

d g p d

Ng pd

pitchline velocity (fpm)

Transmitted load (lbf)

Tp 788.134 in lbf Cf 1 B 0.25 A 92


B

3. 4.

Set the application factor, Ca Write the equation for the dynamic load factor, C
v

Ca 1

B 0.25 12 Qv

A 50 56 ( 1 B) Cvp p d A min A Vtp pd ft

5. 6. 7.

Tentatively choose the mounting factor, Cmd Choose an elastic coefficient from Table 12-18 (steel on steel) and a stress adjustment constant. Determine the endurance strength of the pinion. Cycle life Life factor Reliability Hardness Temperature factor Material surface strength (psi) AISI 4340 Nitrided steel shifts 3

Cmd 3.0 Cp 2100 psi np 2


0.056 0.5

Cxc 1 Cb 0.634

N Life shifts CL 2.466 N

N 4.493 10 CL 0.71 CR 1 CH 1 CT 1

CR 0.7 0.15 log( 1 R)

S acp 162000 psi

MACHINE DESIGN - An Integrated Approach, 4th Ed.


S acp CL CT CR
2

13-24-2

Endurance strength 8. Write the equation for the design pinion torque.

S fcp

S fcp 115071 psi

Sacp d p pd 0.774 CH TD p d F 2 Cs Cmd Cf Ca Cxc Cp Cb CT CR


F 9. Define the exponent z. z p d F

I Cvp p d

return 0.667 if Tp TD p d F 1 otherwise

10. The surface stress equation for the pinion is

2 TD pd F Tp cp p d F Cp Cb F d p pd 2 I TD p d F

z pd F

Cvp p d

Ca Cmd

Cs Cf Cxc

11. Write the design equations using the face-width to pitch-cone length ratio given in the text, and the bending stress equation, solved for the unknown face width. Upper limit, F = L/3 FLover3 p d Np 6 p d sin atan Face width

Np Ng

Sfc Nfc = c

Cp Cb F' p d F S fcp
12. Plot F(P) vs. p d over the range

2 TD p d F Tp d p pd 2 I TD pd F
2 2.5 12 in in in

z pd F

Cvp p d

Ca Cmd

Cs Cf Cxc Nfc

p d

Guess value for F


4

F 1.325 in

13. From the graph, choose a standard value of p d from Table 12-2. p d 6 in
1 F' pd F in

14. The calculated value of F is F' p d F 1.325 in 15. Round this to the decimal equivalent of a common fractional value. F 1.375 in 16. Then, the parameters that depend on p d and F are: d p p d 3.000 in Vtp p d 628 d g p d 13.500 in Cvp p d 0.942
in

FLover3 pd

2
1

6 pd in

10

12

FIGURE 13-24
Graph of Face Width and Limits for for Problem 13-24

ft min

cp p d F 99697 psi

TD p d F 1567 in lbf

MACHINE DESIGN - An Integrated Approach, 4th Ed.

13-24-3

17. When both pinion and gear are of the same material, the stress in the gear will always be lower than in the pinion Therefore, the face width and diametral pitch found for the pinion should also be used for the gear. Diametral pitch Face width Pitch cone angle p d 6 in
1

F 1.375 in

p atan

Np Ng

p 12.529 deg
L 3

Pitch-cone length

2 sin p

d p p d

L 6.915 in

2.305 in

18. Determine the realized factor of safety for the pinion using the above values for F and p d. Nfcp

Pinion factor of safety

Sfcp p F cp d

Nfcp 1.3

MACHINE DESIGN - An Integrated Approach, 4th Ed.

13-25-1

PROBLEM 13-25
Statement: Units: Given: Size the spiral gears in problem 13-7 for a surface factor of safety of at least 1.4 assuming a 10-year, 3-shift life, steady torque, quality index of 8, an AISI 4340 steel pinion and gear. yr 2080 hr Factor of safety Power to be transmitted Rotational speed of pinion Number of pinion teeth Solution: Pinion 1. 2. Determine the surface geometry factor, I, from Figure 13-7. I 0.158 See Mathcad file P1325. Nfc 1.4 Number of gear teeth Ng 80 Reliability H 3 hp R 0.99 n p 600 rpm AGMA Quality level Qv 8 Np 16 Life (years) Life 10 yr

Write the equations for pitch diameter, pitchline velocity, and transmitted load in terms of the unknown diametral pitch, p d. Note that, in Mathcad, unit conversion factors are not included. Pitch diameter of pinion (in) d p p d Vtp p d Tp H np Cs 1
0.6667

Np pd Np 2 pd np

d g p d

Ng pd

pitchline velocity (fpm)

Transmitted load (lbf) 3. 4. Set the application factor, Ca Write the equation for the dynamic load factor, C
v

Tp 315.127 in lbf Cf 1 B 0.63 A 70.721


B

Ca 1

B 0.25 12 Qv

A 50 56 ( 1 B) Cvp p d A min A Vtp pd ft

5. 6.

Tentatively choose the mounting factor, Cmd Choose an elastic coefficient from Table 11-18 (steel on steel) and a stress adjustment constant. Determine the endurance strength of the pinion. Cycle life Life factor Reliability Hardness Temperature factor Material surface strength (psi) AISI 4340 Nitrided steel shifts 3

Cmd 3.0 Cp 2100 psi


0.5

Cxc 1 Cb 0.634

7.

N Life shifts CL 2.466 N

np 2
0.056

N 2.246 10 CL 0.738 CR 1 CH 1 CT 1

CR 0.7 0.15 log( 1 R)

S acp 162000 psi

MACHINE DESIGN - An Integrated Approach, 4th Ed.

13-25-2

Endurance strength 8. Write the equation for the design pinion torque.

S fcp

S acp CL CT CR

S fcp 119626 psi

Sacp d p pd 0.774 CH TD p d F 2 Cs Cmd Cf Ca Cxc Cp Cb CT CR


F 9. Define the exponent z. z p d F

I Cvp p d

return 0.667 if Tp TD p d F 1 otherwise

10. The surface stress equation for the pinion is

2 TD pd F Tp cp p d F Cp Cb F d p pd 2 I TD p d F

z pd F

Cvp p d

Ca Cmd

Cs Cf Cxc

11. Write the design equations using the face-width to pitch-cone length ratio given in the text, and the bending stress equation, solved for the unknown face width. Upper limit, F = L/3 FLover3 p d Np 6 p d sin atan Face width

Np Ng

Sfc Nfc = c

Cp Cb F' p d F S fcp
Guess value for F 12. Plot F(P) vs. p d over the range

2 TD p d F Tp d p pd 2 I TD pd F
F 1.141 in

z pd F

Cvp p d

Ca Cmd

Cs Cf Cxc Nfc

Found by iteration with value in step 14.

p d

2 2.5 12 in in in

13. From the graph, choose a standard value of p d from Table 12-2. p d 10 in
1

F' pd F in

14. The calculated value of F is F' p d F 1.141 in 15. Round this to the decimal equivalent of a common fractional value. F 1.250 in 16. Then, the parameters that depend on p d and F are: d p p d 1.600 in d g p d 8.000 in

FLover3 pd in

2
1

7 pd in

9 10 11 12

FIGURE 13-25
Graph of Face Width and Limits for the Pinion in Problem 13-25

MACHINE DESIGN - An Integrated Approach, 4th Ed.


Vtp p d 251 ft min Cvp p d 0.880

13-25-3
TD p d F 658 in lbf

cp p d F 98091 psi

17. When both pinion and gear are of the same material, the stress in the gear will always be lower than in the pinion Therefore, the face width and diametral pitch found for the pinion should also be used for the gear. Diametral pitch Face width Pitch cone angle p d 10 in
1

F 1.250 in

p atan

Np Ng

p 11.31 deg
L 3

Pitch-cone length

2 sin p

d p p d

L 4.079 in

1.360 in

18. Determine the realized factor of safety for the pinion using the above values for F and p d.

Pinion factor of safety

Sfcp Nfcp cp pd F

Nfcp 1.5

MACHINE DESIGN - An Integrated Approach, 4th Ed.

13-26-1

PROBLEM 13-26
Statement: Units: Given: Find the rated power and rated output torque of the wormset in Problem 13-9 with an input speed o 2200 rpm. fpm ft min
1

Worm pitch diameter Axial pitch Threads on worm

d w 50 mm p x 10 mm Nw 2

Gear ratio Input (worm) speed Pressure angle

mG 22 n w 2200 rpm

20 deg

Solution: 1. 2. 3. 4.

See Mathcad file P1326. L p x Nw L 20 mm

Use equation 13.13 to calculate the lead. Use equation 13.12 to calculate the lead angle. Calculate the number of teeth on the gear Use equation 13.13 to calculate gear diameter.

atan

Ng mG Nw p c p x d g C p c Ng

d w

7.256 deg
Ng 44

dw dg 2

d g 140.056 mm

5.

Use equation 13.17 to calculate the center distance.

6.

Find the maximum recommended face width from equation 13.19. Fmax 0.67 d w Fmax 33.5 mm

7. 8.

Find the materials factor Cs from equation 13.24. Since C < 203 mm, Cs 1000. Find the ratio correction factor Cm from equations 13.25. Based on mG 22 , the second of the expressions in that equation set will be used. Cm 0.0107 mG 56 mG 5145
2

Cm 0.821

9.

Find the tangential velocity Vt from equation 13.27. Vt nw dw 2 cos( ) Vt 1143 fpm

10. Use this velocity to find the velocity factor Cv from equations 13.26. For this value of Vt, the second of these equations is appropriate.

Vt Cv 13.31 fpm

0.571

Cv 0.239

11. Find the tangential load Wt from equation 13.23.

dg Wtg Cs Cm Cv mm

0.8

Fmax

N 75.948 mm

Wtg 4510 N

12. Find the coefficient of friction from the third expression in equation 13.29.
0.110

0.103 e

fpm
Vt

0.450

0.012

0.020

MACHINE DESIGN - An Integrated Approach, 4th Ed.


13. Find the friction force Wf from equation 13.28. Wf

13-26-2

Wtg
cos( ) cos( )

Wf 94.5 N

14. Find the rated output power from equation 13.21.

nw dg Wtg mG 2

o 3.31 kW

15. Find the power lost in the mesh from equation 13.22.

l Vt Wf
16. Find the rated input power from equation 13.20.

l 0.55 kW 3.86 kW

o l
17. The efficiency of the gearset is e

e 85.8 %

18. Find the rated output torque from equation 13.31. Tg Wtg dg 2 Tg 316 N m

MACHINE DESIGN - An Integrated Approach, 4th Ed.

13-27-1

PROBLEM 13-27
Statement: Units: Given: Find the rated power and rated output torque of the wormset in Problem 13-10 with an input speed of 1400 rpm. fpm ft min
1

Worm pitch diameter Axial pitch Threads on worm

d w 1.75 in p x 0.200 in Nw 3

Gear ratio Input (worm) speed Pressure angle L p x Nw

mG 17 n w 1400 rpm

20 deg
L 0.600 in

Solution: 1. 2. 3. 4.

See Mathcad file P1327.

Use equation 13.13 to calculate the lead. Use equation 13.12 to calculate the lead angle. Calculate the number of teeth on the gear Use equation 13.13 to calculate gear diameter.

atan

Ng mG Nw p c p x d g C p c Ng

d w

6.228 deg
Ng 51

dw dg 2

d g 3.247 in C 2.498 in

5.

Use equation 13.17 to calculate the center distance.

6.

Find the maximum recommended face width from equation 13.19. Fmax 0.67 d w Fmax 1.173 in

7. 8.

Find the materials factor Cs from equation 13.24. Since C < 203 mm, Cs 1000. Find the ratio correction factor Cm from equations 13.25. Based on mG 17 , the first of the expressions in that equation set will be used. Cm 0.0200 mG 40 mG 76 0.46
2

Cm 0.815

9.

Find the tangential velocity Vt from equation 13.27. Vt nw dw 2 cos( ) Vt 645 fpm

10. Use this velocity to find the velocity factor Cv from equations 13.26. For this value of Vt, the first of these equations is appropriate.
0.0011 Vt fpm

Cv 0.659 e

Cv 0.324

11. Find the tangential load Wt from equation 13.23.

dg Wtg Cs Cm Cv mm

0.8

Fmax

N 75.948 mm

Wtg 794 lbf

12. Find the coefficient of friction from the third expression in equation 13.29.

Vt 0.110 fpm 0.103 e

0.450

0.012

0.026

MACHINE DESIGN - An Integrated Approach, 4th Ed.


13. Find the friction force Wf from equation 13.28. Wf

13-27-2

Wtg
cos( ) cos( )

Wf 21.8 lbf

14. Find the rated output power from equation 13.21.

nw dg Wtg mG 2

o 1.69 hp

15. Find the power lost in the mesh from equation 13.22.

l Vt Wf
16. Find the rated input power from equation 13.20.

l 0.43 hp 2.11 hp

o l
17. The efficiency of the gearset is e

e 79.8 %

18. Find the rated output torque from equation 13.31. Tg Wtg dg 2 Tg 1290 in lbf

MACHINE DESIGN - An Integrated Approach, 4th Ed.

13-28-1

PROBLEM 13-28
Statement: Units: Given: Find the rated power and rated output torque of the wormset in Problem 13-11 with an input speed of 4500 rpm. fpm ft min
1

Worm pitch diameter Axial pitch Threads on worm

d w 40 mm p x 5 mm Nw 1

Gear ratio Input (worm) speed Pressure angle L p x Nw

mG 82 n w 4500 rpm

20 deg
L 5 mm

Solution: 1. 2. 3. 4.

See Mathcad file P1328.

Use equation 13.13 to calculate the lead. Use equation 13.12 to calculate the lead angle. Calculate the number of teeth on the gear Use equation 13.13 to calculate gear diameter.

atan

Ng mG Nw p c p x d g C p c Ng

d w
L

2.279 deg
Ng 82

dw dg 2

d g 130.51 mm C 85.25 mm

5.

Use equation 13.17 to calculate the center distance.

6.

Find the maximum recommended face width from equation 13.19. Fmax 0.67 d w Fmax 26.8 mm

7. 8.

Find the materials factor Cs from equation 13.24. Since C < 203 mm, Cs 1000. Find the ratio correction factor Cm from equations 13.25. Based on mG 82 , the third of the expressions in that equation set will be used. Cm 1.1483 0.00658 mG Cm 0.609

9.

Find the tangential velocity Vt from equation 13.27. Vt nw dw 2 cos( ) Vt 1857 fpm

10. Use this velocity to find the velocity factor Cv from equations 13.26. For this value of Vt, the second of these equations is appropriate.

Vt Cv 13.31 fpm

0.571

Cv 0.181

11. Find the tangential load Wt from equation 13.23.

dg Wtg Cs Cm Cv mm

0.8

Fmax

N 75.948 mm

Wtg 1915 N

12. Find the coefficient of friction from the third expression in equation 13.29.
0.110

0.103 e

fpm
Vt

0.450

0.012

0.016

MACHINE DESIGN - An Integrated Approach, 4th Ed.


13. Find the friction force Wf from equation 13.28. Wf

13-28-2

Wtg
cos( ) cos( )

Wf 32.6 N

14. Find the rated output power from equation 13.21.

nw dg Wtg mG 2

o 718 W

15. Find the power lost in the mesh from equation 13.22.

l Vt Wf
16. Find the rated input power from equation 13.20.

l 307 W 1026 W

o l
17. The efficiency of the gearset is e

e 70.0 %

18. Find the rated output torque from equation 13.31. Tg Wtg dg 2 Tg 125 N m

MACHINE DESIGN - An Integrated Approach, 4th Ed.

13-29-1

PROBLEM 13-29
Statement:

_____

A 23-tooth helical gear is cut with a 20-degree-pressure-angle hob at a helix angle of 25 deg. The hob has a standard diametral pitch of 5. The resulting teeth have standard spur-gear dimensions in the normal plane. Find the pitch diameter, addendum, dedendum, outside diameter, normal, transverse, and axial pitch, and the transverse pressure angle. Normal plane data: Pressure angle
1

Given:

n 20 deg

Diametral pitch Helix angle

p nd 5 in

Other data: Number of teeth N 23 Solution: 1. See Mathcad file P1329.

25 deg

Use equation 13.1d to calculate the transverse diametral pitch. p d p nd cos( ) p d 4.532 in
1

2.

Use equations 13.1 to find the pitch diameter, transverse pitch, normal pitch, and axial pitch. Pitch diameter d p t N pd d 5.076 in p t 0.693 in p n 0.628 in p x 1.487 in

Transverse pitch

pd

Normal pitch

p n p t cos( ) p x pn sin( )

Axial pitch

3.

Use the equations in Table 12-1 with the normal diametral pitch to calculate the addendum, dedendum, and outside diameter. Addendum a 1 p nd 1.25 p nd a 0.200 in

Dedendum

b 0.250 in D 5.476 in

Outside diameter 4.

D d 2 a

Use equation 13.2 to find the pressure angle in the transverse plane.

t atan

tan n cos( )

t 21.880 deg

MACHINE DESIGN - An Integrated Approach, 4th Ed.

13-30-1

PROBLEM 13-30
Statement:

_____

A 38-tooth helical gear is cut with a 25-degree-pressure-angle hob at a helix angle of 30 deg. The hob has a standard diametral pitch of 4. The resulting teeth have standard spur-gear dimensions in the normal plane. Find the pitch diameter, addendum, dedendum, outside diameter, normal, transverse, and axial pitch, and the transverse pressure angle. Normal plane data: Pressure angle
1

Given:

n 25 deg

Diametral pitch Helix angle

p nd 4 in

Other data: Number of teeth N 38 Solution: 1. See Mathcad file P1330.

30 deg

Use equation 13.1d to calculate the transverse diametral pitch. p d p nd cos( ) p d 3.464 in
1

2.

Use equations 13.1 to find the pitch diameter, transverse pitch, normal pitch, and axial pitch. Pitch diameter d p t N pd d 10.970 in p t 0.907 in p n 0.785 in p x 1.571 in

Transverse pitch

pd

Normal pitch

p n p t cos( ) p x pn sin( )

Axial pitch

3.

Use the equations in Table 12-1 with the normal diametral pitch to calculate the addendum, dedendum, and outside diameter. Addendum a 1 p nd 1.25 p nd a 0.250 in

Dedendum

b 0.313 in D 11.470 in

Outside diameter 4.

D d 2 a

Use equation 13.2 to find the pressure angle in the transverse plane.

t atan

tan n cos( )

t 28.300 deg

MACHINE DESIGN - An Integrated Approach, 4th Ed.

13-31-1

PROBLEM 13-31
Statement: = 25 deg. Find the transverse and axial contact ratios. Given: Tooth numbers: Ng 39 Pressure angle Diametral pitch Np 18 Helix angle

_____

A 39-tooth, 20-deg helix angle, helical gear is in mesh with an 18-tooth pinion. The p d = 8 and

25 deg
p d 8 in
1

20 deg

Assumptions: The face width factor is fwf 12 Solution: 1. See Mathcad file P1331.

Use equation 12.7 to find the transverse contact ratio. Circular pitch Base pitch Pinion: Pitch dia Pitch rad Gear: Pitch dia Pitch rad d g Ng pd d g 4.875 in rg 2.438 in a 0.125 in d p Np pd d p 2.250 in rp 1.125 in p c

pd

p c 0.393 in p b 0.356 in

p b p c cos( )

rp 0.5 d p

rg 0.5 d g a 1.0 pd Np Ng 2 pd

Addendum

Center distance Length of action Z

C 3.563 in

rp a 2 rp cos( ) 2 rg a 2 rg cos( ) 2 C sin( )


Z pb

Z 0.516 in Transverse contact ratio 2. mp mp 1.450

Use equation 13.5 to find the axial contact ratio. Face width F fwf pd F p d tan ( ) F 1.500 in

Axial contact ratio

mF

mF 1.390

MACHINE DESIGN - An Integrated Approach, 4th Ed.

13-32-1

PROBLEM 13-32
Statement: = 20 deg. Find the transverse and axial contact ratios. Given: Tooth numbers: Ng 79 Pressure angle Diametral pitch Np 20 Helix angle

_____

A 79-tooth, 30-deg helix angle, helical gear is in mesh with an 20-tooth pinion. The p d = 6 and

20 deg
p d 6 in
1

30 deg

Assumptions: The face width factor is fwf 12 Solution: 1. See Mathcad file P1332.

Use equation 12.7 to find the transverse contact ratio. Circular pitch Base pitch Pinion: Pitch dia Pitch rad Gear: Pitch dia Pitch rad d g Ng pd d g 13.167 in rg 6.583 in a 0.167 in d p Np pd d p 3.333 in rp 1.667 in p c

pd

p c 0.524 in p b 0.492 in

p b p c cos( )

rp 0.5 d p

rg 0.5 d g a 1.0 pd Np Ng 2 pd

Addendum

Center distance Length of action Z

C 8.250 in

rp a 2 rp cos( ) 2 rg a 2 rg cos( ) 2 C sin( )


Z pb

Z 0.832 in Transverse contact ratio 2. mp mp 1.690

Use equation 13.5 to find the axial contact ratio. Face width F fwf pd F p d tan ( ) F 2.000 in

Axial contact ratio

mF

mF 2.205

MACHINE DESIGN - An Integrated Approach, 4th Ed.

13-33-1

PROBLEM 13-33
Statement: Given:

_____

If the gearset in Problem 13-31 transmits 135 HP at 1200 pinion rpm, find the torque on each shaft Tooth numbers: Ng 39 Pinion speed Np 18 Transmitted power P 135 hp

p 1200 rpm

Assumptions: There is no loss of power in the gear mesh (100% efficiency). Solution: 1. See Mathcad file P1333.

For the pinion shaft Tp P

Tp 7090 in lbf

Tp 590.9 ft lbf

2.

The gear shaft will rotate at a lower speed, which is determined by the gear ratio. (The speed will be decreased in proportion to the ratio and the torque will be increased by the reciprocal of the ratio). For the gear shaft

3.

g
Tg

Np Ng P

g 553.846 rpm
Tg 15362 in lbf Tg 1280 ft lbf

4.

We could have calculated the torque on the gear shaft directly without finding the gear shaft speed, Tg Ng Np Tp Tg 15362 in lbf Tg 1280 ft lbf

MACHINE DESIGN - An Integrated Approach, 4th Ed.

13-34-1

PROBLEM 13-34
Statement: Given: If the gearset in Problem 13-32 transmits 30 kW at 1200 pinion rpm, find the torque on each shaft. Tooth numbers: Number of pinion teeth Number of gear teeth Np 20

Pinion speed

p 1200 rpm
P 30 kW

Ng 79Transmitted power

Assumptions: There is no loss of power in the gear mesh (100% efficiency). Solution: 1. 2. See Mathcad file P1334. Tp P

For the pinion shaft

Tp 239 N m

The gear shaft will rotate at a lower speed, which is determined by the gear ratio. (The speed will be decreased in proportion to the ratio and the torque will be increased by the reciprocal of the ratio). For the gear shaft

3.

g
Tg

Np Ng P

g 303.797 rpm
Tg 943 N m

4.

We could have calculated the torque on the gear shaft directly without finding the gear shaft speed, Tg Ng Np Tp Tg 943 N m

MACHINE DESIGN - An Integrated Approach, 4th Ed.

13-35-1

PROBLEM 13-35
Statement: Size the helical gears in problem 13-33 for a bending factor of safety of at least 2 assuming a steady torque, 25 deg pressure angle, full depth teeth, quality index of 9, an AISI 4140 steel pinion, and a class 40 cast iron gear. Number of gear teeth Factor of safety Nfb 2 Ng 39 Power to be transmitted Rotational speed of pinion Number of pinion teeth Pressure angle Solution: Pinion 1. 2. Determine the bending geometry factor, J (Table 13-4) Jp 0.57 See Mathcad file P1335. H 135 hp n 1200 rpm Np 18 25 deg Reliability AGMA Quality level Life (cycles) Helix angle R 0.99 Qv 9 N 10 10 deg
7

Given:

Write the equations for pitch diameter, pitchline velocity, and transmitted load in terms of the unknown diametral pitch, p d. Note that, in Mathcad, unit conversion factors are not included. Pitch diameter of pinion (in) d p d Vt p d Wt p d Ka 1
v

Np pd dpd n 2 H

pitchline velocity (fpm)

Transmitted load (lbf)

Vt p d

3. 4.

Set the application factor, Ka Write the equation for the dynamic load factor, K

B 0.25 12 Qv

0.6667

B 0.52 A 76.878
B

A 50 56 ( 1 B) Kv p d A min A Vt p d ft Km 1.7 Ka Wt p d p d Km Kv p d F Jp

5. 6.

Tentatively choose the mounting factor, Km (Assume 2 < F < 6 in) The bending stress equation for the pinion is

bp p d F

7.

Determine the endurance strength of the pinion. Life factor Reliability Temperature factor Material bending strength (psi) AISI 4140 Nitrided steel KL 1.6831 N
0.0323

KL 1 KR 1 KT 1 S atp 40000 psi

KR 0.7 0.15 log( 1 R)

MACHINE DESIGN - An Integrated Approach, 4th Ed.


Endurance strength 8. S fbp S atp KL KT KR

13-35-2
S fbp 40001 psi

Write the design equations using the range of face-width to diametral-pitch ratio given in the text, and the bending stress equation, solved for the unknown face width. Upper limit FU p d FL p d S fb 16 pd 8 pd

Lower limit

Safety factor

Nfb =

b
Ka Wt p d p d Km Nfb Kv p d Jp S fbp

Face width

F p d

9. Plot F(P) vs. p d over the range

p d

1 1.5 6 in in in
4

10. From the graph, choose a standard value of p d from Table 12-2. p d 3 in
1

F pd in


in

11. The calculated value of F is F p d 1.335 in 12. Round this up to the decimal equivalent of a common fractional value. F 1.375 in

FL pd

FU pd in

13. Then, the parameters that depend on p d and F are: d p d 6.000 in Vt p d 1885 ft min Kv p d 0.792 Wt p d 2363 lbf

2.5

3.5

4 pd in

4.5

5.5

FIGURE 13-35A
Graph of Face Width and Limits for the Pinion in Problem 13-35

bp p d F 19411 psi
The assumption made in step 5 is correct so no further iteration is required. Gear 14. Determine the bending geometry factor, J (Table 13-4) 15. The bending stress equation for the gear is 16. Determine the endurance strength of the pinion. Material bending strength (psi) Class 40 cast iron Jg 0.64 Ka Wt p d p d Km Kv p d F Jg

bg p d F

S atg 13000 psi

MACHINE DESIGN - An Integrated Approach, 4th Ed.


S atg KL KT KR

13-35-3

Endurance strength

S fbg

S fbg 13000 psi

17. Write the design equations using the range of face-width to diametral-pitch ratio given in the text, and the bending stress equation, solved for the unknown face width. Upper limit FU p d FL p d S fb 16 pd 8 pd

Lower limit

Safety factor

Nfb =

b
Ka Wt p d p d Km Nfb Kv p d Jg S fbg

Face width

F p d p d

18. Plot F(P) vs. p d over the range

1 1.5 6 in in in
6 F pd in

19. From the graph, choose a standard value of p d from Table 12-2. p d 3 in
1


in

4.5

20. The calculated value of F is F p d 3.657 in 21. Round this up to the decimal equivalent of a common fractional value. F 3.750 in 22. Then, the parameters that depend on P and F are: d p d 6.000 in Vt p d 1885 ft min Kv p d 0.792 Wt p d 2363 lbf

FL pd

FU pd in


1.5

2.5

3.5

4 pd in

4.5

5.5

FIGURE 13-35B
Graph of Face Width and Limits for the Gear in Problem 13-35

bg p d F 6339 psi
23. The gear dimensions are larger (smaller diametral pitch means bigger teeth) than for the pinion. This means that we will accept the gear requirements for the pinion, thus, for the set Diametral pitch Face width p d 3 in
1

F 3.750 in

24. Determine the realized factor of safety for the gear using the above values for F and p d. Gear factor of safety Nfbg

bg p d F

S fbg

Nfbg 2.1

MACHINE DESIGN - An Integrated Approach, 4th Ed.

13-35-4

25. Check the factor of safety on the pinion: Pinion factor of safety Nfbp

bp p d F

S fbp

Nfbp 5.6

MACHINE DESIGN - An Integrated Approach, 4th Ed.

13-36-1

PROBLEM 13-36
Statement: Size the helical gears in problem 13-34 for a bending factor of safety of at least 2.5 assuming a steady torque, 20 deg pressure angle, full depth teeth, quality index of 11, an AISI 4340 steel pinio and an A-7-d nodular 40 iron gear. Factor of safety Power to be transmitted Rotational speed of pinion Number of pinion teeth Pressure angle Solution: Pinion 1. 2. Determine the bending geometry factor, J (Table 13-4) Jp 0.46 See Mathcad file P1336. Nfb 2.5 H 30 kW n 1200 rpm Np 20 20 deg Number of gear teeth Reliability AGMA Quality level Life (cycles) Helix angle Ng 79 R 0.99 Qv 11 N 10 30 deg
7

Given:

Write the equations for pitch diameter, pitchline velocity, and transmitted load in terms of the unknown diametral pitch, p d. Note that, in Mathcad, unit conversion factors are not included. Pitch diameter of pinion (in) d p d Vt p d Wt p d Ka 1
v

Np pd dpd n 2 H

pitchline velocity (fpm)

Transmitted load (lbf)

Vt p d

3. 4.

Set the application factor, Ka Write the equation for the dynamic load factor, K

B 0.25 12 Qv

0.6667

B 0.25 A 92
B

A 50 56 ( 1 B) Kv p d A min A Vt p d ft Km 1.7 Ka Wt p d p d Km Kv p d F Jp

5. 6.

Tentatively choose the mounting factor, Km (Assume 2 < F < 6 in) The bending stress equation for the pinion is

bp p d F

7.

Determine the endurance strength of the pinion. Life factor Reliability Temperature factor Material bending strength (psi) AISI 4340 Nitrided steel KL 1.6831 N
0.0323

KL 1 KR 1 KT 1 S atp 42000 psi

KR 0.7 0.15 log( 1 R)

MACHINE DESIGN - An Integrated Approach, 4th Ed.


Endurance strength 8. S fbp S atp KL KT KR

13-36-2
S fbp 42001 psi

Write the design equations using the range of face-width to diametral-pitch ratio given in the text, and the bending stress equation, solved for the unknown face width. Upper limit FU p d FL p d S fb 16 pd 8 pd

Lower limit

Safety factor

Nfb =

b
Ka Wt p d p d Km Nfb Kv p d Jp S fbp

Face width

F p d

9. Plot F(P) vs. p d over the range

p d

1 1.5 10 in in in
4

10. From the graph, choose a standard value of p d from Table 12-2. p d 6 in
1

F pd in


in

11. The calculated value of F is F p d 1.804 in 12. Round this up to the decimal equivalent of a common fractional value. F 1.875 in

FL pd

FU pd in

13. Then, the parameters that depend on p d and F are: d p d 3.333 in Vt p d 1047 ft min Kv p d 0.927 Wt p d 1268 lbf

7 pd in

10

FIGURE 13-36A
Graph of Face Width and Limits for the Pinion in Problem 13-36

bp p d F 16166 psi
The assumption made in step 5 is correct so no further iteration is required. Gear 14. Determine the bending geometry factor, J (Table 13-4) 15. The bending stress equation for the gear is 16. Determine the endurance strength of the pinion. Material bending strength (psi) A-7-d nodular iron S atg 34000 psi Jg 0.50 Ka Wt p d p d Km Kv p d F Jg

bg p d F

MACHINE DESIGN - An Integrated Approach, 4th Ed.


S fbg S atg KL KT KR S fbg 34001 psi

13-36-3

Endurance strength

17. Write the design equations using the range of face-width to diametral-pitch ratio given in the text, and the bending stress equation, solved for the unknown face width. Upper limit FU p d FL p d S fb 16 pd 8 pd

Lower limit

Safety factor

Nfb =

b
Ka Wt p d p d Km Nfb Kv p d Jg S fbg

Face width

F p d p d

18. Plot F(P) vs. p d over the range

1 1.5 10 in in in
6 F pd in FL pd in FU pd in

19. From the graph, choose a standard value of p d from Table 12-2. p d 6 in
1

4.5

20. The calculated value of F is F p d 2.050 in 21. Round this up to the decimal equivalent of a common fractional value. F 2.125 in 22. Then, the parameters that depend on P and F are: d p d 3.333 in Vt p d 1047 ft min Kv p d 0.927


1.5

7 pd in

10

FIGURE 13-36B Wt p d 1268 lbf


Graph of Face Width and Limits for the Gear in Problem 13-36

bg p d F 13123 psi
23. The gear dimensions are larger (smaller diametral pitch means bigger teeth) than for the pinion. This means that we will accept the gear requirements for the pinion, thus, for the set Diametral pitch Face width p d 6 in
1

F 2.125 in

24. Determine the realized factor of safety for the gear using the above values for F and p d. Gear factor of safety Nfbg

bg p d F

S fbg

Nfbg 2.6

MACHINE DESIGN - An Integrated Approach, 4th Ed.

13-36-4

25. Check the factor of safety on the pinion: Pinion factor of safety Nfbp

bp p d F

S fbp

Nfbp 2.9

MACHINE DESIGN - An Integrated Approach, 4th Ed.

13-37-1

PROBLEM 13-37
Statement: Size the helical gears in problem 13-33 for a surface factor of safety of at least 1.6 assuming a steady torque, 25 deg pressure angle, full depth teeth, quality index of 9, an AISI 4140 steel pinion, and a class 40 cast iron gear. Factor of safety Power to be transmitted Rotational speed of pinion Number of pinion teeth Pressure angle Solution: Pinion 1. Determine the surface geometry factor, I. Addendum a p d rp p d 1 pd Np 2 pd rg p d Ng 2 pd See Mathcad file P1337. Nfc 1.6 H 135 hp n 1200 rpm Np 18 25 deg Number of gear teeth Reliability AGMA Quality level Life (cycles) Helix angle Ng 39 R 0.99 Qv 9 N 10 20 deg
7

Given:

Pitch radii

Center distance Radii of curvature

C p d rp p d rg p d

p p d

2 2 0.5 rp p d C pd rg pd rp p d cos( )

g p d C p d sin( ) p p d
Normal plane pressure angle

n atan( tan( ) cos( ) ) b acos cos( )

n 23.662 deg

Base helix angle Length of action Z p d

cos n cos( )

b 18.256 deg

r p a p 2 r p cos( ) 2 d p d p d r p a p 2 r p cos( ) 2 C p sin( ) d g d d g d


mp p d Z pd

Transverse contact ratio

pd

cos( ) F p d tan( )

Axial contact ratio Fractional parts of mp and mF

mF p d F

n r p d mp p d floor mp p d

MACHINE DESIGN - An Integrated Approach, 4th Ed.


n a p d F mF p d F floor mF p d F Axial pitch p t p d p x p d Minimum length of lines of contact Lmin p d F

13-37-2

pd pn pd sin( )

p n p d p t p d cos( )

return

return

mp pd F napd F nrpd pxpd if n a p d F 1 n r p d cos b mp pd F 1 na p d F 1 nr pd px p d if n p F 1 n p a d r d cos b


mN p d F I p d F Lmin p d F cos( ) F

Load sharing ratio

Geometry factor

1 1 2 r p m p F p p p d N d g d p d

2.

Write the equations for pitch diameter, pitchline velocity, and transmitted load in terms of the unknown diametral pitch, p d. Note that, in Mathcad, unit conversion factors are not included. Pitch diameter of pinion (in) d p p d Vt p d Wt p d Ca 1
v

Np pd d p p d n 2 Vt p d H

d g p d

Ng pd

pitchline velocity (fpm) Transmitted load (lbf)

3. 4.

Set the application factor, Ca Write the equation for the dynamic load factor, C

B 0.25 12 Qv

0.6667

B 0.52 A 76.878
B

A 50 56 ( 1 B) Cv p d A min A Vt p d ft Cm 1.7 Cp 2100 psi


0.5

5. 6.

Tentatively choose the mounting factor, Cm (Assume 2 < F < 6 in Choose an elastic coefficient from Table 12-18 (steel on CI).

7.

The surface stress equation for the pinion is

cp p d F Cp

Cv p d F d p p d I p d F

C a W t p d C m

MACHINE DESIGN - An Integrated Approach, 4th Ed.


8. Determine the endurance strength of the pinion. Life factor Reliability Temperature factor Material surface strength (psi) AISI 4140 steel Endurance strength 9. S fcp S ac CL CT CR CL 2.466 N
0.056

13-37-3

CL 1 CR 1 CT 1 S ac 165000 psi S fcp 164997 psi

CR 0.7 0.15 log( 1 R)

Write the design equations using the range of face-width to diametral-pitch ratio given in the text, and the bending stress equation, solved for the unknown face width. Limits, safety factor FU p d 16 pd FL p d 8 pd
2

Nfc =

Sfc c

Face width

Cp F' p d F Nfc Cv p d d p p d I p d F S fcp


F 3.059 in p d 2 2.5 12 in in in
4 F' pd F in FL pd in FU pd in

C a W t p d C m

Guess value for F 10. Plot F(P) vs. p d over the range

Found by iteration with value in step 12.

11. From the graph, choose a standard value of p d from Table 12-2. p d 5 in
1

12. The calculated value of F is F' p d F 3.059 in 13. Round this up to the decimal equivalent of a common fractional value. F 3.125 in 14. Then, the parameters that depend on p d and F are: d p p d 3.600 in Vt p d 1131 ft min d g p d 7.800 in Wt p d 3939 lbf mp p d 1.45 mN p d F 0.682

7 pd in

9 10 11 12

FIGURE 13-37A
Graph of Face Width and Limits for the Pinion (surface) in Problem 13-37

cp p d F 128455 psi
n a p d F 0.81

mF p d F 1.81 Lmin p d F 4.582 in

n r p d 0.45 I p d F 0.192

MACHINE DESIGN - An Integrated Approach, 4th Ed.


Gear 15. The surface stress equation for the gear is 16. Determine the endurance strength of the gear. Life factor Reliability Temperature factor Material surface strength (psi) Class 40 CI Endurance strength S fcg S ac CL CT CR CL 2.466 N
0.056

13-37-4

cg p d F Cp

Cv p d F d g p d I p d F

C a W t p d C m

CL 1 CR 1 CT 1 S ac 80000 psi S fcg 79999 psi

CR 0.7 0.15 log( 1 R)

17. Write the design equations using the range of face-width to diametral-pitch ratio given in the text, and the bending stress equation, solved for the unknown face width. Limits, safety factor FU p d 16 pd FL p d 8 pd

Sfc Nfc = c

Face width

F' p d F

2 Cp Nfc Cv p d d g p d I p d F S fcg

C a W t p d C m

Guess value for F 18. Plot F(P) vs. p d over the range p d

F 3.887 in 2 2.5 12 in in in

Found by iteration with value in step 20.

19. From the graph, choose a standard value of p d from Table 12-2. p d 4 in
1

F' pd F in FL pd in FU pd in

4.5

20. The calculated value of F is F' p d F 3.887 in 21. Round this up to the decimal equivalent of a common fractional value. F 3.875 in 22. Then, the parameters that depend on p d and F are: d p p d 4.500 in Vt p d 1414 ft min d g p d 9.750 in Wt p d 3151 lbf mp p d 1.45


1.5

2.5

3.5

4 pd in

4.5

5.5

FIGURE 13-37B
Graph of Face Width and Limits for the Gear (surface) in Problem 13-37

cp p d F 93300 psi

mF p d F 1.796

n r p d 0.45

MACHINE DESIGN - An Integrated Approach, 4th Ed.


n a p d F 0.796 mN p d F 0.684 Lmin p d F 5.662 in

13-37-5
I p d F 0.191

23. The gear teeth are larger than for the pinion. This means that we will accept the gear requirements for the pinion thus, for the set Diametral pitch Face width p d 4 in
1

F 3.875 in

24. Determine the realized factor of safety for the gear using the above values for F and p d.

Gear factor of safety

Sfcg Nfsg cg pd F

Nfsg 1.6

25. Check the factor of safety on the pinion: Pinion factor of safety

Sfcp Nfsp cp pd F

Nfsp 3.1

MACHINE DESIGN - An Integrated Approach, 4th Ed.

13-38-1

PROBLEM 13-38
Statement: Size the helical gears in problem 13-34 for a surface factor of safety of at least 1.2 assuming a stead torque, 20 deg pressure angle, full depth teeth, quality index of 11, an AISI 4340 steel pinion, and A-7-d nodular iron gear. Factor of safety Power to be transmitted Rotational speed of pinion Number of pinion teeth Pressure angle Solution: Pinion 1. Determine the surface geometry factor, I. Addendum a p d rp p d 1 pd Np 2 pd rg p d Ng 2 pd See Mathcad file P1338. Nfc 1.2 H 30 kW n 1200 rpm Np 20 20 deg Number of gear teeth Reliability AGMA Quality level Life (cycles) Helix angle Ng 79 R 0.99 Qv 11 N 10 30 deg
7

Given:

Pitch radii

Center distance Radii of curvature

C p d rp p d rg p d

p p d

2 2 0.5 rp p d C pd rg pd rp p d cos( )

g p d C p d sin( ) p p d
Normal plane pressure angle

n atan( tan( ) cos( ) ) b acos cos( )

n 17.495 deg

Base helix angle Length of action Z p d

cos n cos( )

b 28.481 deg

r p a p 2 r p cos( ) 2 d p d p d r p a p 2 r p cos( ) 2 C p sin( ) d g d d g d


mp p d Z pd

Transverse contact ratio

pd

cos( ) F p d tan( )

Axial contact ratio Fractional parts of mp and mF

mF p d F

n r p d mp p d floor mp p d

MACHINE DESIGN - An Integrated Approach, 4th Ed.


n a p d F mF p d F floor mF p d F Axial pitch p t p d p x p d Minimum length of lines of contact Lmin p d F

13-38-2

pd pn pd sin( )

p n p d p t p d cos( )

return

return

mp pd F napd F nrpd pxpd if n a p d F 1 n r p d cos b mp pd F 1 na p d F 1 nr pd px p d if n p F 1 n p a d r d cos b


mN p d F I p d F Lmin p d F cos( ) F

Load sharing ratio

Geometry factor

1 1 2 r p m p F p p p d N d g d p d

2.

Write the equations for pitch diameter, pitchline velocity, and transmitted load in terms of the unknown diametral pitch, p d. Note that, in Mathcad, unit conversion factors are not included. Pitch diameter of pinion (in) d p p d Vt p d Wt p d Ca 1
v

Np pd d p p d n 2 Vt p d H

d g p d

Ng pd

pitchline velocity (fpm) Transmitted load (lbf)

3. 4.

Set the application factor, Ca Write the equation for the dynamic load factor, C

B 0.25 12 Qv

0.6667

B 0.25 A 92
B

A 50 56 ( 1 B) Cv p d A min A Vt p d ft Cm 1.6 Cp 2160 psi


0.5

5. 6.

Tentatively choose the mounting factor, Cm (Assume 0 < F < 2 in Choose an elastic coefficient from Table 12-18 (steel on NI).

7.

The surface stress equation for the pinion is

cp p d F Cp

Cv p d F d p p d I p d F

C a W t p d C m

MACHINE DESIGN - An Integrated Approach, 4th Ed.


8. Determine the endurance strength of the pinion. Life factor Reliability Temperature factor Material surface strength (psi) AISI 4340 steel Endurance strength 9. S fcp S ac CL CT CR CL 2.466 N
0.056

13-38-3

CL 1 CR 1 CT 1 S ac 160000 psi S fcp 159997 psi

CR 0.7 0.15 log( 1 R)

Write the design equations using the range of face-width to diametral-pitch ratio given in the text, and the bending stress equation, solved for the unknown face width. Limits, safety factor FU p d 16 pd FL p d 8 pd
2

Nfc =

Sfc c

Face width

Cp F' p d F Nfc Cv p d d p p d I p d F S fcp


F 1.599 in p d 6 6.5 16 in in in
2 F' pd F in FL pd in FU pd in

C a W t p d C m

Guess value for F 10. Plot F(P) vs. p d over the range

Found by iteration with value in step 12.

11. From the graph, choose a standard value of p d from Table 12-2. p d 10 in
1

1.5

12. The calculated value of F is F' p d F 1.598 in 13. Round this up to the decimal equivalent of a common fractional value. F 1.625 in 14. Then, the parameters that depend on p d and F are: d p p d 2.000 in Vt p d 628 ft min d g p d 7.900 in Wt p d 2113 lbf mp p d 1.69 mN p d F 0.52


0.5

9 10 11 12 13 14 15 16 pd in

FIGURE 13-38A
Graph of Face Width and Limits for the Pinion (surface) in Problem 13-38

cp p d F 144632 psi
n a p d F 0.986

mF p d F 2.986 Lmin p d F 3.123 in

n r p d 0.69 I p d F 0.246

MACHINE DESIGN - An Integrated Approach, 4th Ed.


Gear 15. The surface stress equation for the gear is 16. Determine the endurance strength of the gear. Life factor Reliability Temperature factor Material surface strength (psi) A-7-d nodular iron Endurance strength S fcg S ac CL CT CR CL 2.466 N
0.056

13-38-4

cg p d F Cp

Cv p d F d g p d I p d F

C a W t p d C m

CL 1 CR 1 CT 1 S ac 100000 psi S fcg 99998 psi

CR 0.7 0.15 log( 1 R)

17. Write the design equations using the range of face-width to diametral-pitch ratio given in the text, and the bending stress equation, solved for the unknown face width. Limits, safety factor FU p d 16 pd FL p d 8 pd

Sfc Nfc = c

Face width

F' p d F

2 Cp Nfc Cv p d d g p d I p d F S fcg

C a W t p d C m

Guess value for F 18. Plot F(P) vs. p d over the range p d

F 1.040 in 6 6.5 16 in in in

Found by iteration with value in step 20.

19. From the graph, choose a standard value of p d from Table 12-2. p d 10 in
1

F' pd F in FL pd in FU pd in

1.5

20. The calculated value of F is F' p d F 1.041 in 21. Round this up to the decimal equivalent of a common fractional value. F 1.125 in 22. Then, the parameters that depend on p d and F are: d p p d 2.000 in Vt p d 628 ft min d g p d 7.900 in Wt p d 2113 lbf mp p d 1.69


0.5

9 10 11 12 13 14 15 16 pd in

FIGURE 13-38B
Graph of Face Width and Limits for the Gear (surface) in Problem 13-38

cg p d F 88013 psi

mF p d F 2.067

n r p d 0.69

MACHINE DESIGN - An Integrated Approach, 4th Ed.


n a p d F 0.067 mN p d F 0.527 Lmin p d F 2.135 in

13-38-5
I p d F 0.243

23. The gear teeth are smaller than for the pinion. This means that we will accept the pinnion requirements for the gear thus, for the set Diametral pitch Face width p d 10 in
1

F 1.625 in

24. Determine the realized factor of safety for the gear using the above values for F and p d.

Gear factor of safety

Sfcg Nfsg cg pd F

Nfsg 1.9

25. Check the factor of safety on the pinion: Pinion factor of safety

Sfcp Nfsp cp pd F

Nfsp 1.2

MACHINE DESIGN - An Integrated Approach, 4th Ed.

13-39-1

PROBLEM 13-39
Statement:

_____

A 90-deg straight bevel gearset is needed to give a 3:1 reduction. Determine the pitch cone angles, pitch diameters, and gear forces if the 25-deg pressure angle pinion has 15 teeth of p d = 4, and the transmitted power is 8 hp at 550 pinion rpm. Power transmitted Pinion speed Teeth on pinion H 8 hp Gear ratio Diametral pitch Pressure angle Ng mG Np mG 3 p d 4 in
1

Given:

p 550 rpm
Np 15

25 deg

Solution: 1. 2.

See Mathcad file P1339. Ng 45

Use equation 13.7b to calculate number of teeth on the gear. Use equation 12.4a to calculate the pitch diameters. Pinion d p Np pd Ng pd

d p 3.750 in

Gear

d g

d g 11.250 in

3.

Use equation 13.7b to calculate the pitch cone angles. Pinion

p atan

mG

p 18.435 deg g 71.565 deg

Gear Note that 4.

g atan mG

p g 90.000 deg

Determine the torque on the pinion shaft and the transmitted force. Pinion Tp H

p
2 Tp dp

Tp 916.7 in lbf

Transmitted force 5.

Wt

Wt 488.9 lbf

Use equations 13.8a to calculate the pinion and gear forces. Pinion Wap Wt tan ( ) sin p Wrp Wt tan( ) cos p Gear Wag Wt tan ( ) sin g Wrg Wt tan( ) cos g Total force W Wt ( cos( ) )
1

Wap 72.10 lbf Wrp 216.29 lbf Wag 216.29 lbf Wrg 72.10 lbf W 539.468 lbf

MACHINE DESIGN - An Integrated Approach, 4th Ed.

13-40-1

PROBLEM 13-40
Statement:

_____

A 90-deg straight bevel gearset is needed to give a 6:1 reduction. Determine the pitch cone angles, pitch diameters, and gear forces if the 20-deg pressure angle pinion has 20 teeth of p d = 8, and the transmitted power is 3 kW at 900 pinion rpm. Power transmitted Pinion speed Teeth on pinion H 3 kW Gear ratio Diametral pitch Pressure angle Ng mG Np mG 6 p d 8 in
1

Given:

p 900 rpm
Np 20

20 deg

Solution: 1. 2.

See Mathcad file P1340. Ng 120

Use equation 13.7b to calculate number of teeth on the gear. Use equation 12.4a to calculate the pitch diameters. Pinion d p Np pd Ng pd

d p 2.500 in

Gear

d g

d g 15.000 in

3.

Use equation 13.7b to calculate the pitch cone angles. Pinion

p atan

mG

p 9.462 deg g 80.538 deg

Gear Note that 4.

g atan mG

p g 90.000 deg

Determine the torque on the pinion shaft and the transmitted force. Pinion Tp H

p
2 Tp dp

Tp 281.7 in lbf

Transmitted force 5.

Wt

Wt 225.4 lbf

Use equations 13.8a to calculate the pinion and gear forces. Pinion Wap Wt tan ( ) sin p Wrp Wt tan( ) cos p Gear Wag Wt tan ( ) sin g Wrg Wt tan( ) cos g Total force W Wt ( cos( ) )
1

Wap 13.49 lbf Wrp 80.92 lbf Wag 80.92 lbf Wrg 13.49 lbf W 239.847 lbf

MACHINE DESIGN - An Integrated Approach, 4th Ed.

13-41-1

PROBLEM 13-41
Statement:

_____

A 90-deg spiral bevel gearset is needed to give an 8:1 reduction. Determine the pitch cone angles, pitch diameters, and gear forces if the 20-deg pressure angle pinion has 21 teeth of p d = 10, and the transmitted power is 2.5 kW at 1100 pinion rpm. Power transmitted Pinion speed Teeth on pinion H 2.5 kW Gear ratio Diametral pitch Pressure angle mG 8 p d 10 in
1

Given:

p 1100 rpm
Np 21

20 deg

Assumptions: The spiral angle is 35 deg. Solution: 1. 2. See Mathcad file P1341. Ng mG Np Ng 168

Use equation 13.7b to calculate number of teeth on the gear. Use equation 12.4a to calculate the pitch diameters. Pinion d p Np pd Ng pd

d p 2.100 in

Gear

d g

d g 16.800 in

3.

Use equation 13.7b to calculate the pitch cone angles. Pinion

p atan

mG

p 7.125 deg g 82.875 deg

Gear Note that 4.

g atan mG

p g 90.000 deg

Determine the torque on the pinion shaft and the transmitted force. Pinion Tp H

p
2 Tp dp

Tp 192.1 in lbf

Transmitted force

Wt

Wt 182.9 lbf

5.

Use equations 13.2 to calculate the pressure angle in the normal plane.

n atan( tan( ) cos( ) )


6. Use equations 13.8b to calculate the pinion and gear forces. Pinion Wap Wt cos( )

n 16.602 deg

tan n sin p sin( ) cos p

Wap 118.85 lbf

MACHINE DESIGN - An Integrated Approach, 4th Ed.

13-41-2

Wrp Gear Wag Wrg

Wt cos( )

tan n cos p sin( ) sin p

Wrp 81.96 lbf

Wt cos( ) Wt cos( )

tan n sin g sin( ) cos g tan n cos g sin( ) sin g


1

Wag 50.18 lbf Wrg 135.37 lbf

Total force

W Wt ( cos( ) )

W 194.681 lbf

MACHINE DESIGN - An Integrated Approach, 4th Ed.

13-42-1

PROBLEM 13-42
Statement:

_____

A 1-start wormset has d = 2.00 in, p x = 0.25 in, mG = 40. Find the lead, lead angle, worm gear diameter, and center distance. Will it self-lock? The input speed is 1100 rpm. Threads on worm Worm pitch diameter Axial pitch Nw 1 d 2.00 in p x 0.25 in Gear ratio Input (worm) speed mG 40 n w 1100 rpm

Given:

Solution: 1.

See Mathcad file P1342.

Use equation 13.13 to calculate the lead. L p x Nw L 0.250 in

2.

Use equation 13.12 to calculate the lead angle.

atan
3.

d
L

2.279 deg

Calculate the number of teeth on the gear Ng mG Nw Ng 40

4.

Use equation 13.13 to calculate gear diameter. p c p x d g p c Ng d g 3.183 in

5.

Use equation 13.17 to calculate the center distance. C d dg 2 C 2.592 in

6.

If the lead angle per worm tooth is less than 6 deg, the set is self-locking. The lead angle per tooth is

Nw

t 2.279 deg

Self-locking

MACHINE DESIGN - An Integrated Approach, 4th Ed.

13-43-1

PROBLEM 13-43
Statement:

_____

A 2-start wormset has d = 2.50 in, p x = 0.30 in, mG = 50. Find the lead, lead angle, worm gear diameter, and center distance. Will it self-lock? The input speed is 1800 rpm. Threads on worm Worm pitch diameter Axial pitch Nw 2 d 2.50 in p x 0.30 in Gear ratio Input (worm) speed mG 50 n w 1800 rpm

Given:

Solution: 1.

See Mathcad file P1343.

Use equation 13.13 to calculate the lead. L p x Nw L 0.600 in

2.

Use equation 13.12 to calculate the lead angle.

atan
3.

d
L

4.369 deg

Calculate the number of teeth on the gear Ng mG Nw Ng 100

4.

Use equation 13.13 to calculate gear diameter. p c p x d g p c Ng d g 9.549 in

5.

Use equation 13.17 to calculate the center distance. C d dg 2 C 6.025 in

6.

If the lead angle per worm tooth is less than 6 deg, the set is self-locking. The lead angle per tooth is

Nw

t 2.184 deg

Self-locking

MACHINE DESIGN - An Integrated Approach, 4th Ed.

13-44-1

PROBLEM 13-44
Statement:

_____

A 3-start wormset has d = 60 mm, p x = 12 mm, mG = 60. Find the lead, lead angle, worm gear diameter, and center distance. Will it self-lock? The input speed is 2500 rpm. Threads on worm Worm pitch diameter Axial pitch Nw 3 d 60 mm p x 12 mm Gear ratio Input (worm) speed mG 60 n w 1800 rpm

Given:

Solution: 1.

See Mathcad file P1344.

Use equation 13.13 to calculate the lead. L p x Nw L 36.000 mm

2.

Use equation 13.12 to calculate the lead angle.

atan
3.

d
L

10.812 deg

Calculate the number of teeth on the gear Ng mG Nw Ng 180

4.

Use equation 13.13 to calculate gear diameter. p c p x d g p c Ng d g 687.55 mm

5.

Use equation 13.17 to calculate the center distance. C d dg 2 C 373.77 mm

6.

If the lead angle per worm tooth is less than 6 deg, the set is self-locking. The lead angle per tooth is

Nw

t 3.604 deg

Self-locking

MACHINE DESIGN - An Integrated Approach, 4th Ed.

13-45-1

PROBLEM 13-45
Statement:

_____

Determine the power transmitted and the torques and forces in the mesh for the wormset in Problem 13-42 if it runs at 800 worm rpm. fpm ft min
1

Units: Given:

Threads on worm Worm pitch diameter Axial pitch

Nw 1 d w 2.00 in p x 0.25 in

Pressure angle Gear ratio Input (worm) speed

20 deg
mG 40 n w 800 rpm

Solution: 1.

See Mathcad file P1345.

Use equation 13.13 to calculate the lead. L p x Nw L 0.250 in

2.

Use equation 13.12 to calculate the lead angle.

atan
3.

d w

2.279 deg

Calculate the number of teeth on the gear Ng mG Nw Ng 40

4.

Use equation 13.13 to calculate gear diameter. p c p x d g p c Ng

d g 3.183 in

5.

Use equation 13.17 to calculate the center distance. C dw dg 2 C 2.592 in

6.

Find the maximum recommended face width from equation 13.19. Fmax 0.67 d w Fmax 1.340 in

7. 8.

Find the materials factor Cs from equation 13.24. Since C < 8 in, Cs 1000. Find the ratio correction factor Cm from equations 13.25. Based on mG 40 , the second of the expressions in that equation set will be used. Cm 0.0107 mG 56 mG 5145
2

Cm 0.814

9.

Find the tangential velocity Vt from equation 13.27. Vt nw dw 2 cos( ) Vt 419.2 fpm

10. Use this velocity to find the velocity factor Cv from equations 13.26. For this value of Vt, the first of these equations is appropriate.
0.0011 Vt fpm

Cv 0.659 e

Cv 0.416

MACHINE DESIGN - An Integrated Approach, 4th Ed.


11. Find the tangential load Wt from equation 13.23.

13-45-2

dg Wtg Cs Cm Cv in

0.8

Fmax

lbf in

Wtg 1144 lbf

12. Find the coefficient of friction from the third expression in equation 13.29.

Vt 0.110 fpm 0.103 e

0.450

0.012

0.031

13. Find the friction force Wf from equation 13.28. Wf

Wtg
cos( ) cos( )

Wf 38.4 lbf

14. Find the rated output power from equation 13.21.

nw dg Wtg mG 2

o 0.58 hp

15. Find the power lost in the mesh from equation 13.22.

l Vt Wf
16. Find the rated input power from equation 13.20.

l 0.49 hp

o l
17. The efficiency of the gearset is e

1.07 hp

e 54.2 %

18. Find the rated output torque from equation 13.31. Tg Wtg dg 2 Tg 1821 in lbf

MACHINE DESIGN - An Integrated Approach, 4th Ed.

13-46-1

PROBLEM 13-46
Statement: Units: Given:

_____

Determine the power transmitted and the torques and forces in the mesh for the wormset in Problem 13-43 if it runs at 1200 worm rpm. fpm ft min
1

Threads on worm Worm pitch diameter Axial pitch

Nw 2 d w 2.50 in p x 0.30 in

Pressure angle Gear ratio Input (worm) speed

20 deg
mG 50 n w 1200 rpm

Solution: 1.

See Mathcad file P1346.

Use equation 13.13 to calculate the lead. L p x Nw L 0.600 in

2.

Use equation 13.12 to calculate the lead angle.

atan
3.

d w

4.369 deg

Calculate the number of teeth on the gear Ng mG Nw Ng 100

4.

Use equation 13.13 to calculate gear diameter. p c p x d g p c Ng d g 9.549 in

5.

Use equation 13.17 to calculate the center distance. C dw dg 2 C 6.025 in

6.

Find the maximum recommended face width from equation 13.19. Fmax 0.67 d w Fmax 1.675 in Find the materials factor Cs from equation 13.24. Since C < 8 in, Cs 1000. Find the ratio correction factor Cm from equations 13.25. Based on mG 50 , the second of the expressions in that equation set will be used. Cm 0.0107 mG 56 mG 5145
2

7. 8.

Cm 0.790

9.

Find the tangential velocity Vt from equation 13.27. Vt nw dw 2 cos( ) Vt 787.7 fpm

10. Use this velocity to find the velocity factor Cv from equations 13.26. For this value of Vt, the second of these equations is appropriate.

Cv 13.31

Vt fpm

0.571

Cv 0.295

11. Find the tangential load Wt from equation 13.23.

MACHINE DESIGN - An Integrated Approach, 4th Ed.


0.8

13-46-2

dg Wtg Cs Cm Cv in

Fmax

lbf in

Wtg 2375 lbf

12. Find the coefficient of friction from the third expression in equation 13.29.

Vt 0.110 fpm 0.103 e

0.450

0.012

0.023

13. Find the friction force Wf from equation 13.28. Wf

Wtg
cos( ) cos( )

Wf 59.0 lbf

14. Find the rated output power from equation 13.21.

nw dg Wtg mG 2

o 4.32 hp

15. Find the power lost in the mesh from equation 13.22.

l Vt Wf
16. Find the rated input power from equation 13.20.

l 1.41 hp

o l
17. The efficiency of the gearset is e

5.73 hp

e 75.4 %

18. Find the rated output torque from equation 13.31. Tg Wtg dg 2 Tg 11342 in lbf

MACHINE DESIGN - An Integrated Approach, 4th Ed.

13-47-1

PROBLEM 13-47
Statement:

_____

A 2-start wormset has L = 2.00 in, C = 9.00 in, mG = 20, and the angle between the shafts is 90 deg. Find the pitch diameters of the worm and worm gear, the lead angle, and the axial pitch. Threads on worm Worm lead Nw 2 L 2.00 in Center distance Gear ratio C 9.00 in mG 20

Given:

Solution: 1.

See Mathcad file P1347.

Determine the pitch diameter of the worm gear using equation 13.13. L Nw

d g
Ng

L = d g

Nw Ng

d g
mG

Solving for d g, d g mG L d g 12.732 in

2.

Determine the pitch diameter of the worm using equation 13.17 solved for d w. d w 2 C d g d w 5.268 in

3.

Calculate the lead angle using equation 13.12.

atan

d w

6.891 deg

4.

Calculate the axial pitch using equation 13.13. p x L Nw p x 1.000 in

MACHINE DESIGN - An Integrated Approach, 4th Ed.

13-48-1

PROBLEM 13-48
Statement:

_____

A 5-start wormset has = 20 deg, C = 2.75 in, Ng = 33, and the angle between the shafts is 90 deg. Find the pitch diameters of the worm and worm gear, the lead, and the axial pitch. Threads on worm Worm lead angle Nw 5 Center distance Teeth on worm gear C 2.75 in Ng 33

Given:

20 deg

Solution: 1.

See Mathcad file P1348.

Determine the pitch diameter of the worm gear using equations 13.12, 13.13, and 13.17. tan ( ) = L L = d w tan ( ) Nw Ng

d w

L Nw

d g
Ng

L = d g

Eliminating L and solving for d w, dw = Nw Ng tan ( ) dg

Substitute into equation 13.17 and solve for d g, dg = 2 C Nw Ng tan ( ) dg d g 1 2 C Nw Ng tan ( ) d g 3.883 in

2.

Determine the pitch diameter of the worm using equation 13.17 solved for d w. d w 2 C d g d w 1.617 in

3.

Calculate the lead using equation 13.12. L d w tan( ) L 1.848 in

4.

Calculate the axial pitch using equation 13.13. p x L Nw p x 0.370 in

MACHINE DESIGN - An Integrated Approach, 4th Ed.

13-49-1

PROBLEM 13-49
Statement: Units: Given: Size the bevel gears in problem 13-40 for a bending factor of safety of at least 2.5 assuming a 5-year, 2-shift life, steady torque, quality index of 8, and an AISI 4140 steel pinion and gear. yr 2080 hr Factor of safety Power to be transmitted (hp) Rotational speed of pinion (rpm) Number of pinion teeth Solution: Pinion 1. 2. Determine the bending geometry factor, J (Figure 13-5) Jp 0.300 See Mathcad file P1349. Nfb 2.5 H 3 kW n p 900 rpm Np 20 Number of gear teeth Reliability AGMA Quality level Life (years) Ng 120 R 0.99 Qv 8 Life 5 yr

Write the equations for pitch diameter, pitchline velocity, and transmitted torque in terms of the unknown diametral pitch, p d. Note that, in Mathcad, unit conversion factors are not included. Pitch diameter of pinion (in) d p p d Vt p d Tp H np Ks 1
0.6667

Np pd d p p d n p 2

d g p d

Ng pd

pitchline velocity (fpm)

Transmitted torque (in-lbf)

Tp 281.728 in lbf Kx 1 B 0.63 A 70.721


B

3. Set the application, size, and type factors. 4. Write the equation for the dynamic load factor, Kv

Ka 1

B 0.25 12 Qv

A 50 56 ( 1 B) Kv p d A min A Vt p d ft Km 1.6 2 Tp pd

5. Tentatively choose the mounting factor, Km (Assume 0 < F < 2 in)

6. The bending stress equation for the pinion is

bp p d F

d p p d F Jp Kv p d Kx

Ka Km Ks

7. Determine the endurance strength of the pinion. Cycle life Life factor Reliability Temperature factor Material bending strength (psi) AISI 4140 Nitrided steel shifts 2 N Life shifts KL 1.6831 N np 2
0.0323

N 1.123 10 KL 0.859 KR 1 KT 1

KR 0.7 0.15 log( 1 R)

S atp 40000 psi

MACHINE DESIGN - An Integrated Approach, 4th Ed.


S atp KL KT KR

13-49-2

Endurance strength

S fbp

S fbp 34343 psi

8. Write the design equations using the face-width to pitch-cone length ratio given in the text, and the bending stress equation, solved for the unknown face width. Upper limit, F = L/3 FLover3 p d Np 6 p d sin atan

Np Ng

Safety factor

Nfb =

S fb

b
d p p d Jp Kv p d Kx S fbp 2 Tp p d Ka Km Ks Nfb

Face width

F p d

9. Plot F(P) vs. p d over the range

p d

8 8.5 18 in in in

10. From the graph, choose a standard value of p d from Table 12-2. p d 10 in
1 F pd in FLover3 pd in

11. The calculated value of F is F p d 1.295 in 12. Round this to the decimal equivalent of a common fractional value. F 1.375 in 13. Then, the parameters that depend on p d and F are: d p p d 2.000 in Vt p d 471 ft min Kv p d 0.845

2
1

10

12

14 pd in

16

18

FIGURE 13-49A
Graph of Face Width and Limits for the Pinion in Problem 13-49

bp p d F 12935 psi

The assumption made in step 5 is correct so no further iteration is required.

GEAR 14. Determine the bending geometry factor, J (Figure 13-5) 15. The bending stress equation for the gear is Jg 0.245

bg p d F

d g p d F Jg Kv p d Kx

2 Tp

pd

Ka Km Ks

16. Write the design equations using the face-width to pitch-cone length ratio given in the text, and the bending stress equation, solved for the unknown face width.

MACHINE DESIGN - An Integrated Approach, 4th Ed.


FLover3 p d Np 6 p d sin atan

13-49-3

F = L/3

Np Ng

Safety factor

Nfb =

S fb

b
d g p d Jg Kv p d Kx S fbp 2 Tp p d Ka Km Ks Nfb

Face width

F p d

17. Plot F(P) vs. p d over the range p d

14 14.5 24 in in in
2

18. From the graph, choose a standard value of p d from Table 12-2. p d 20 in
1

F pd in

1.5

19. The calculated value of F is F p d 1.01 in 20. Round this to the decimal equivalent of a common fractional value. F 1.000 in 21. Then, the parameters that depend on p d and F are: d g p d 6.000 in Vt p d 236 ft min Kv p d 0.884

FLover3 pd in

0.5

0 14

16

18

20 pd in

22

24

FIGURE 13-49B

bg p d F 13881 psi

Graph of Face Width and Limits for the Gear in Problem 13-49

22. The gear dimensions are smaller (larger diametral pitch means smaller teeth) than for the pinion. This means th we will accept the pinion requirements for the gear, thus, for the set Diametral pitch Face width Pitch cone angle p d 10 in
1

F 1.375 in

p atan

Np Ng

p 9.462 deg
L 3

Pitch-cone length

2 sin p

d p p d

L 6.083 in

2.028 in

23. Determine the realized factor of safety for the pinion using the above values for F and p d. Pinion factor of safety 24. Check the factor of safety on the gear: Gear factor of safety Nfbg Nfbp

bp p d F

S fbp

Nfbp 2.7

bg p d F

S fbp

Nfbg 13.0

MACHINE DESIGN - An Integrated Approach, 4th Ed.

13-50-1

PROBLEM 13-50
Statement: Size the bevel gears in problem 13-40 for a minimum safety factor of 1.8 for any mode of failure of pinion or gear assuming a 5-year, 2-shift life, steady torque, quality index of 8, and an AISI 4140 steel pinion and gear. yr 2080 hr Factor of safety Power to be transmitted (hp) Rotational speed of pinion (rpm) Number of pinion teeth Nfc 1.8 H 3 kW n p 900 rpm Np 20 Number of gear teeth Reliability AGMA Quality level Life (years) Ng 120 R 0.99 Qv 8 Life 5 yr

Units: Given:

Assumptions: 1. If both pinion and gear are the same material, it will only be necessary to determine the pinion size as it will be governing for the set. 2. If the gears are not surface hardened, it will only be necessary to design to the surface requirement as it will be governing for both bending and surface stresses. Solution: See Mathcad file P1350. I 0.107

1. Determine the surface geometry factor, I, from Figure 13-6.

2. Write the equations for pitch diameter, pitchline velocity, and transmitted load in terms of the unknown diametral pitch, p d. Note that, in Mathcad, unit conversion factors are not included. Pitch diameter of pinion (in) d p p d Vtp p d Tp H np Cs 1
0.6667

Np pd Np 2 pd np

d g p d

Ng pd

pitchline velocity (fpm)

Transmitted load (lbf)

Tp 281.728 in lbf Cf 1 B 0.63 A 70.721


B

3. Set the application factor, Ca 4. Write the equation for the dynamic load factor, Cv

Ca 1

B 0.25 12 Qv

A 50 56 ( 1 B) Cvp p d A min A Vtp pd ft

5. Tentatively choose the mounting factor, Cmd 6. Choose an elastic coefficient from Table 12-18 (steel on steel) and a stress adjustment constant. 7. Determine the endurance strength of the pinion. Cycle life Life factor Reliability shifts 2

Cmd 3.0 Cp 2100 psi


0.5

Cxc 1 Cb 0.634

N Life shifts CL 2.466 N

np 2
0.056

N 1.123 10 CL 0.768 CR 1

CR 0.7 0.15 log( 1 R)

MACHINE DESIGN - An Integrated Approach, 4th Ed.


Hardness Temperature factor Material surface strength (psi) AISI 4140 Nitrided steel Endurance strength 8. Write the equation for the design pinion torque. S fcp S acp CL CT CR
2

13-50-2
CH 1 CT 1 S acp 167500 psi S fcp 128583 psi

Sacp d p pd 0.774 CH TD p d F 2 Cs Cmd Cf Ca Cxc Cp Cb CT CR


F 9. Define the exponent z. z p d F return 0.667 if Tp TD p d F 1 otherwise 10. The surface stress equation for the pinion is

I Cvp p d

2 TD pd F Tp cp p d F Cp Cb F d p pd 2 I TD p d F

z pd F

Cvp p d

Ca Cmd

Cs Cf Cxc

11. Write the design equations using the face-width to pitch-cone length ratio given in the text, and the bending stress equation, solved for the unknown face width. Upper limit, F = L/3 FLover3 p d Np 6 p d sin atan Face width

Np Ng

Sfc Nfc = c

Cp Cb F' p d F S fcp
12. Plot F(P) vs. p d over the range

2 TD p d F Tp d p pd 2 I TD pd F
8 8.5 18 in in in

z pd F

Cvp p d

Ca Cmd

Cs Cf Cxc Nfc

p d

Guess value for F


4

F 1.21 in

13. From the graph, choose a standard value of p d from Table 12-2. p d 10 in
1 F' pd F in

14. The calculated value of F is F' p d F 1.217 in 15. Round this to the decimal equivalent of a common fractional value. F 1.250 in 16. Then, the parameters that depend on p d and F are:
in

FLover3 pd

2
1

10

12

14 pd in

16

18

MACHINE DESIGN - An Integrated Approach, 4th Ed.


d p p d 2.000 in d g p d 12.000 in FIGURE 13-50

13-50-3

Graph of Face Width and Limits for for Problem 13-50

Vtp p d 471

ft min

Cvp p d 0.845

cp p d F 95063 psi

TD p d F 714 in lbf

17. When both pinion and gear are of the same material, the stress in the gear will always be lower than in the pinion. Therefore, the face width and diametral pitch found for the pinion should also be used for the gear. Diametral pitch Face width Pitch cone angle p d 10 in
1

F 1.250 in

p atan

Np Ng

p 9.462 deg
L 3

Pitch-cone length

2 sin p

d p p d

L 6.083 in

2.028 in

18. Determine the realized factor of safety for the pinion using the above values for F and p d.

Pinion factor of safety

Sfcp Nfcp cp pd F

Nfcp 1.8

MACHINE DESIGN - An Integrated Approach, 4th Ed.

13-51-1

PROBLEM 13-51
Statement: Size the spiral gears in problem 13-41 for a bending factor of safety of at least 2.0 assuming a 7-year, 3-shift life, steady torque, quality index of 8, an AISI 4340 steel pinion and gear. yr 2080 hr Factor of safety Power to be transmitted (kW) Rotational speed of pinion (rpm) Number of pinion teeth Solution: Pinion 1. Determine the bending geometry factor, J (Figure 13-8) Jp 0.360 (assumed) See Mathcad file P1351. Nfb 2.0 Number of gear teeth Ng 168 R 0.99 Qv 8 Life 7 yr Reliability H 3 kW n p 1100 rpm AGMA Quality level Np 21 Life (years)

Units: Given:

2. Write the equations for pitch diameter, pitchline velocity, and transmitted torque in terms of the unknown diametral pitch, p d. Note that, in Mathcad, unit conversion factors are not included. Pitch diameter of pinion (in) d p p d Vt p d Tp H np Ks 1
0.6667

Np pd d p p d n p 2

d g p d

Ng pd

pitchline velocity (fpm)

Transmitted torque (in-lbf)

Tp 230.505 in lbf Kx 1.15 B 0.63 A 70.721


B

3. Set the application, size, and type factors. 4. Write the equation for the dynamic load factor, Kv

Ka 1

B 0.25 12 Qv

A 50 56 ( 1 B) Kv p d A min A Vt p d ft Km 1.6

5. Tentatively choose the mounting factor, Km (Assume 0 < F < 2 in) 6. The bending stress equation for the pinion is

bp p d F

d p p d F Jp Kv p d Kx

2 Tp

pd

Ka Km Ks

7. Determine the endurance strength of the pinion. Cycle life Life factor Reliability Temperature factor Material bending strength (psi) AISI 4340 Nitrided steel shifts 3 N Life shifts KL 1.6831 N np 2
0.0323

N 2.883 10 KL 0.833 KR 1 KT 1

KR 0.7 0.15 log( 1 R)

S atp 42000 psi

MACHINE DESIGN - An Integrated Approach, 4th Ed.


S atp KL KT KR

13-51-2

Endurance strength

S fbp

S fbp 34979 psi

8. Write the design equations using the face-width to pitch-cone length ratio given in the text, and the bending stress equation, solved for the unknown face width. Upper limit, F = L/3 FLover3 p d Np 6 p d sin atan

Np Ng

Safety factor

Nfb =

S fb

b
d p p d Jp Kv p d Kx S fbp 2 Tp p d Ka Km Ks Nfb

Face width

F p d p d 2 2.5 20 in in in

9. Plot F(P) vs. p d over the range

10. From the graph, choose a standard value of p d from Table 12-2. p d 16 in
1

F pd in

11. The calculated value of F is F p d 1.447 in 12. Round this to the decimal equivalent of a common fractional value. F 1.500 in 13. Then, the parameters that depend on p d and F are: d p p d 1.313 in Vt p d 378 ft min Kv p d 0.858

FLover3 pd in

2
1

10 12 14 16 18 20 pd in

FIGURE 13-51A
Graph of Face Width and Limits for the Pinion in Problem 13-51

bp p d F 16874 psi
The assumption made in step 5 is correct so no further iteration is required. GEAR 14. Determine the bending geometry factor, J (Figure 13-8) 15. The bending stress equation for the gear is Jg 0.360

bg p d F

d g p d F Jg Kv p d Kx

2 Tp

pd

Ka Km Ks

16. Write the design equations using the face-width to pitch-cone length ratio given in the text, and the bending stress equation, solved for the unknown face width.

MACHINE DESIGN - An Integrated Approach, 4th Ed.


FLover3 p d Np 6 p d sin atan

13-51-3

F = L/3

Np Ng

Safety factor

Nfb =

S fb

b
d g p d Jg Kv p d Kx S fbp 2 Tp p d Ka Km Ks Nfb

Face width

F p d

17. Plot F(P) vs. p d over the range p d

10 11.5 50 in in in

18. From the graph, choose a standard value of p d from Table 12-2. p d 32 in
1

F pd in

1.5

19. The calculated value of F is F p d 0.694 in 20. Round this to the decimal equivalent of a common fractional value. F 0.750 in 21. Then, the parameters that depend on p d and F are: d g p d 5.250 in Vt p d 189 ft min Kv p d 0.894

FLover3 pd in

0.5

0 10 15

20 25

30 pd in

35 40

45 50

FIGURE 13-51B
Graph of Face Width and Limits for the Gear in Problem 13-51

bg p d F 16194 psi

22. The gear dimensions are smaller (larger diametral pitch means smaller teeth) than for the pinion. This means th we will accept the pinion requirements for the gear, thus, for the set Diametral pitch Face width Pitch cone angle p d 16 in
1

F 1.500 in

p atan

Np Ng

p 7.125 deg
L 3

Pitch-cone length

2 sin p

d p p d

L 5.291 in

1.764 in

23. Determine the realized factor of safety for the pinion using the above values for F and p d. Pinion factor of safety Nfbp

bp p d F

S fbp

Nfbp 2.1

25. Check the factor of safety on the gear: Gear factor of safety Nfbg

bg p d F

S fbp

Nfbg 16.6

MACHINE DESIGN - An Integrated Approach, 4th Ed.

14-1-1

PROBLEM 14-1
Statement: A linear spring is to give 200 N at its maximum deflection of 150 mm and 40 N at its minimum deflection of 50 mm. What is the spring rate? Working force Working deflection Solution: 1. See Mathcad file P1401. Fwork 200 N ywork 150 mm Initial force Initial deflection Finit 40 N yinit 50 mm

Given:

Spring rate is the slope of the load vs. deflection function. Thus, for a linear spring Spring rate k Fwork Finit ywork yinit k 1.60 N mm

MACHINE DESIGN - An Integrated Approach, 4th Ed.

14-2-1

PROBLEM 14-2
Statement: Find the ultimate tensile strength, the ultimate shear strength, and the torsional yield strength of a 1.8-mm-dia, A229 oil-tempered steel wire. Wire diameter d 1.8 mm

Given: Solution: 1.

See Mathcad file P1402.

From Table 14-4, for A229 A 1831.2 MPa b 0.1833

2.

Using equation 14.3, Tensile strength S ut A

mm
d

S ut 1644 MPa

3.

From Table 14-6, with a factor of 0.50, Maximum torsional yield strength S ys 0.50 S ut S ys 822 MPa

4.

Using equation 14.4, Ultimate shear strength S us 0.67 S ut S us 1102 MPa

MACHINE DESIGN - An Integrated Approach, 4th Ed.

14-3-1

PROBLEM 14-3
Statement: Find the torsional yield and ultimate shear strength of an 0.105-in-dia, unset A230 wire to be used in a helical compression spring. Wire diameter d 0.105 in d 2.667 mm

Given: Solution: 1.

See Mathcad file P1403.

ASTM A230 is not listed in Table 14-4, but it is shown in Figure 14-3. At the wire diameter given, it has a tensile strength that is approximately the same as that for A229. From Table 14-4, for A229 A 146.78 ksi b 0.1833

2.

Using equation 14.3, Tensile strength d S ut A in


b

S ut 222 ksi

3.

From Table 14-6, with a factor of 0.50, Maximum torsional yield strength S ys 0.50 S ut S ys 111 ksi

4.

Using equation 14.4, Ultimate shear strength S us 0.67 S ut S us 149 ksi

MACHINE DESIGN - An Integrated Approach, 4th Ed.

14-4-1

PROBLEM 14-4
Statement: Given: What is the torsional fatigue strength of the wire in Problem 14-3 at N = 5E6 cycles? Wire diameter Cycle life Solution: 1. d 0.105 in N 5 10
6

d 2.667 mm

See Mathcad file P1404.

ASTM A230 is not listed in Table 14-4, but it is shown in Figure 14-3. At the wire diameter given, it has a tensile strength that is approximately the same as that for A229. From Table 14-4, for A229 A 146.78 ksi b 0.1833

2. Using equation 14.3, Tensile strength 3. d S ut A in


b

S ut 222 ksi

The required life falls between the values given in Table 14-7 for 1E6 and 1E7 cycles. We will use an exponential fit to the data for interpolation. Using equation 6.10 At 1E6 cycles At 1E7 cycles 0.40 S ut = a 10 0.38 S ut

6 b 7 = a 10
b

4.

Solving both equations for a, equating the results and solving for b 0.40 S ut = 0.38 S ut

106

b b

107

107 = 0.38 0.40 106


b log

b log( 10) = log 1

0.40

0.38

0.40 log( 10)

0.38

b 0.022

0.40 S ut

106
b

a 120.726 ksi

5.

Interpolating, Fatigue strength at 5E6 cycles S fs a N S fs 85.6 ksi

MACHINE DESIGN - An Integrated Approach, 4th Ed.

14-5-1

PROBLEM 14-5
Statement: Given: Draw the Modified Goodman diagram for the wire of Problem 14-3. Wire diameter Cycle life Solution: 1. d 0.105 in N 5 10
6

d 2.667 mm

See Mathcad file P1405.

ASTM A230 is not listed in Table 14-4, but it is shown in Figure 14-3. At the wire diameter given, it has a tensile strength that is approximately the same as that for A229. From Table 14-4, for A229 A 146.78 ksi b 0.1833

2.

Using equation 14.3, Tensile strength d S ut A in


b

S ut 221.9 ksi

3.

From Table 14-6, with a factor of 0.50, Maximum torsional yield strength S ys 0.50 S ut S ys 110.9 ksi

4.

Using equation 14.4, Ultimate shear strength S us 0.67 S ut S us 148.6 ksi

5.

Using Table 14-7 and a cycle life of 1E6, for unpeened wire Torsional fatigue strength (R=0) S fw .40 S ut S fw 88.7 ksi

6.

From equation (d) in Example 14-2, S fs 0.5 S fw S us S us 0.5 S fw S fs 63.3 ksi

7.

Now, the equation for the Goodman line is

a m

S fs S us

m S fs

8.

Plotting this over the range

m 0 ksi 10 ksi 150 ksi

100 Alternating shear stress 80 60 40 20 0

25

50

75

100

125

150

Mean shear stress

MACHINE DESIGN - An Integrated Approach, 4th Ed.

14-6-1

PROBLEM 14-6
Statement: Given: What are the spring rate and spring index of a squared and ground compression spring with the data given below? Wire diameter Mean coil dia d 1 mm D 10 mm Total coils Nt 12

Assumptions: The spring wire is steel so that G 80.8 GPa. Solution: 1. 2. See Mathcad file P1406. Na Nt 2 Na 10

From Figure 14-9, the number of active coils is Using equation 14.7, Spring rate k d G 8 D Na
3 4

k 1.01

N mm

3.

Using equation 14.5, Spring Index C D d C 10

MACHINE DESIGN - An Integrated Approach, 4th Ed.

14-7-1

PROBLEM 14-7
Statement: Given: Find the natural frequency of the spring in Problem 14-6. Wire diameter Mean coil dia d 1 mm D 10 mm
3

Total coils

Nt 12 , 76005 N m
3

Assumptions: The spring wire is steel so that G 80.8 GPa and 0.28 lbf in Solution: 1. 2. See Mathcad file P1407. Na Nt 2

From Figure 14-9, the number of active coils is Using equation 14.7, Spring rate

Na 10

d G 8 D Na
3

k 1.01

N mm

3.

Using equation 14.5, Spring Index C D d C 10

4.

Calculate the natural frequency using equation 14.11c. Natural frequency fn 2 d D


2

N a

G g 32

fn 363.4 Hz

MACHINE DESIGN - An Integrated Approach, 4th Ed.

14-8-1

PROBLEM 14-8
Statement: A paper mill processes rolls of paper having a density of 984 kg/m3. The paper roll is 1.50 m outside dia (OD) by 0.22 m inside dia (ID) by 3.23 m long and is on a simply supported, hollow, steel shaft with 22-cm OD x 20-cm ID and as long as the paper roll. Find the spring rate of the shaft and the fundamental natural frequency of the shaft-roll assembly. Paper density Shaft outside dia Shaft inside dia Young's modulus

Given:

984

kg
3

Roll dimensions: Outside diameter Inside diameter Length OD 1.50 m ID 0.22 m L 3.23 m

m od 220 mm id 200 mm E 207 GPa

Assumptions: The shaft (beam) supporting the paper roll is simply-supported at the ends and is the same length as the paper roll. The paper acts as a distributed load over the length of the shaft. Solution: 1. See Mathcad file P1408.

The mass of the paper roll is equal to its volume times the paper density. mroll

OD ID L I

mroll 5495.74 kg

2. 3.

The area moment of inertia of the shaft is

64

od id

I 3.645 10 mm

The spring rate (stiffness) of the shaft can be found from the deflection equation in Figure B-2(b) in Appendix B When the distributed load covers the entire span, the maximum deflection is ymax = 5 w L
4

384 E I k

5 W L

384 E I

k=

W y

384 E I 5 L
3

Solving for k, 4.

384 E I 5 L
3

k 1.720 10

4 N

mm

Use equations 3.4 to find the natural frequency of the system.

k mroll

n 55.9

rad sec

fn

n
2 fn 8.90 Hz

MACHINE DESIGN - An Integrated Approach, 4th Ed.

14-9-1

PROBLEM 14-9
Statement: Determine the minimum allowable bending radius for an 50 HRC strip steel spring of 1-mm thickness. Thickness t 1 mm

Given: Solution: 1.

See Mathcad file P1409.

From Table 14-5, the bend factor for spring steel with a 50 HRC is bf 5

2.

Use the bend factor definition given in the text to calculate the minimum bend radius. rmin bf t 2 rmin 2.5 mm

MACHINE DESIGN - An Integrated Approach, 4th Ed.

14-10-1

PROBLEM 14-10
Statement: An over-hung diving board is shown in Figure P14-1a. A 100-kg person is standing at the free end Assume cross-section dimensions of 305 mm x 32 mm and a material E = 10.3 GPa. What is the spring rate and fundamental natural frequency of the diver-board combination?
2000 = L

Given:

Weight of person Board dimensions: Distance to support Length of board Cross-section Young's modulus

W 100 kgf a 0.7 m L 2 m w 305 mm t 32 mm E 10.3 GPa

R1

R2 700 = a

Assumptions: The weight of the board is negligible compared to the applied load and so can be ignored. Solution: 1. 2. See Figure 14-10 and Mathcad file P1410. I w t
3

FIGURE 14-10
Free Body Diagram for Problem 14-10

The area moment of inertia of the board is

12

I 8.329 10 mm

The spring rate (stiffness) of the board can be found from the deflection equation in Figure B-3(a) in Appendix B. When the load is at the end of the beam, the maximum deflection is ymax = F L 3 E I k ( L a)
2

k=

F y

3 E I L ( L a ) N mm
2

Solving for k,

3 E I L ( L a )
2

k 7.614

3.

Use equations 3.4 to find the natural frequency of the system.

k g W

n 8.73

rad sec

fn

n
2 fn 1.39 Hz

MACHINE DESIGN - An Integrated Approach, 4th Ed.

14-11-1

PROBLEM 14-11
Statement: Design a helical compression spring for a static load of 45 lb at a deflection of 1.25 in with a safety factor of 2.5. Use C = 7.5. Specify all parameters necessary to manufacture the spring. Working force Working deflection Spring index Design choices: Clash allowance ASTM A228 wire Solution: 1. 2. See Mathcad file P1411. k Fwork ywork k 36 lbf in Fwork 45 lbf ywork 1.25 in C 7.5 Shear modulus Safety factor G 11.5 10 psi Ns 2.5
6

Given:

0.15
A 184.649 ksi b 0.1625

Set removed

Km 0.65

Determine the desired spring rate.

Use the design equation from Example 14-3A (Mathcad Supplement) to determine the wire diameter.
1

8 Ns ( C 0.5) Fwork ( 1 ) 2 Km A in
d 0.125 in

2 b

in

Wire diameter 3.

Let

d 0.125 in

Calculate the mean coil diameter and number of active coils. Mean coil dia Number of active coils D C d Na d G 8 D k
3 4

D 0.938 in Na 11.831 Na 11.75

Note that we round it to the nearest 1/4 coil as the manufacturing tolerance cannot achieve better than that accuracy. We must now calculate the actual (corrected) spring rate: Corrected spring rate 4. k d G 8 D Na
3 4

k 36.249

lbf in

Assume squared and ground ends making the total number of coils, from Figure 14-9: Total coils Nt Na 2 Nt 13.75

5.

The shut height can now be determined. Shut height Ls d Nt Ls 1.719 in

6.

The free length (see Figure 14-8) can now be found from Deflection to shut height Free length yshut ywork ywork Lf Ls yshut yshut 1.438 in Lf 3.156 in

7.

To check for buckling, two ratios need to be calculated, Lf/D and y max/Lf.

MACHINE DESIGN - An Integrated Approach, 4th Ed.


Slenderness ratio sr y' Lf D ywork Lf sr 3.367 y' 0.396

14-11-2

Deflection ratio

Take these two values to Figure 14-14 and find that their coordinates are safely within the zones that are stable against buckling for either end-condition case. 8. The inside and outside coil diameters are Inside coil dia Outside coil dia 9. Di D d Do D d holemin Do 0.05 D pin max Di 0.05 D
3

Di 0.813 in Do 1.063 in holemin 1.11 in pin max 0.77 in

The smallest hole and largest pin that should be used with this spring are Smallest hole Largest pin

10. The total weight of the spring is Weight density

0.28 lbf in
2 2

Weight

Wt

d D Nt
4

Wt 0.14 lbf

11. We now have a complete design specification for this A228 wire spring: Wire diameter Outside diameter Total coils Free length d 0.125 in Do 1.063 in Nt 13.75 Lf 3.156 in ends squared and ground

MACHINE DESIGN - An Integrated Approach, 4th Ed.

14-12-1

PROBLEM 14-12
Statement: Repeat Problem 14-10 using the cantilevered diving board design in Figure P14-1b.
2000 1300 = L

Given:

Weight at free end Board dimensions: Length of board Cross-section Young's modulus

P 100 kgf
P

L 1.3 m w 305 mm t 32 mm E 10.3 GPa

M1 700

R1

Assumptions: The weight of the board is negligible compared to the applied load and so can be ignored. Solution: 1. 2. See Figure 14-12 and Mathcad file P1412. I w t
3

FIGURE 14-12
Free Body Diagram for Problem 14-12

The area moment of inertia of the board is

12

I 8.329 10 mm

The spring rate (stiffness) of the board can be found from the deflection equation in Figure B-1(a) in Appendix B. When the load is at the end of the beam, the maximum deflection is ymax = F L
3

3 E I

k=

F y

3 E I L
3

Solving for k,

3 E I L
3

k 11.71

N mm

3.

Use equations 3.4 to find the natural frequency of the system. k g P rad sec

n 10.82

fn

n
2

fn 1.72 Hz

MACHINE DESIGN - An Integrated Approach, 4th Ed.

14-13-1

PROBLEM 14-13
Statement: For the data given below, find Na, D, Lf, Lshut, yinitial, and the minimum hole diameter for the spring. Infinite life is desired with a safety factor of 1.4. Choose an acceptable spring index. Setting will be done. Minimum force Maximum force Working deflection Clash allowance Shear modulus Solution: 1. See Mathcad file P1413. Fmin 50 lbf Fmax 250 lbf

Given:

Chrome-vanadium wire properties: Diameter d 0.312 in Unpeened Set after winding Squared ends

y 0.75 in 0.15
G 11.5 10 psi
6

Find the the mean and alternating forces from equation 14.15a: Alternating force Fa Fm Fmax Fmin 2 Fmax Fmin 2 Fa 100.0 lbf Fm 150.0 lbf

Mean force 2.

Assume a spring index (found by trial and error to get the correct safety factor) and calculate the mean coil diameter D from equation 14.5. Spring index Mean coil diameter C 4.4 D C d D 1.373 in

3.

Find the direct shear factor Ks and use it to calculate the shear stress i at the initial deflection (lowest defined force), and the mean stress m. Direct shear factor Ks 1 0.5 C Ks 1.114

Stress at Fmin

i Ks

8 Fmin D

d
Stress at Fm 4.

i 6.4 ksi

m Ks

8 Fm D

m 19.2 ksi

Find the Wahl factor Kw and use it to calculate the alternating shear stress a in the coil. Wahl factor Kw 4 C 1 4 C 4 0.615 C Kw 1.36

Alternating stress

a Kw

8 F a D

d
5.

a 15.7 ksi

Find the ultimate tensile strength of this wire material from equation 14.3 and Table 14-4 and use it to find the ultimate shear strength from equation 14.4 and the torsional yield strength from Table 14-6, assuming that the set has been removed and using the low end of the recommended range. From Table 14-4, for A228 music wire Ultimate tensile strength A 173.128 ksi S ut A b 0.1453 S ut 205.1 ksi

in
d

MACHINE DESIGN - An Integrated Approach, 4th Ed.


Shear yield strength Ultimate shear strength 6. S ys 0.65 S ut S us 0.67 S ut S ys 133.3 ksi S us 137.4 ksi

14-13-2

Find the wire endurance limit for unpeened springs in repeated loading from equation 14.12 and convert it to fu reversed endurance strength with equation 14.16c. Wire endurance limit Fully reversed endurance limit S ew 45 ksi S es 0.5 S ew S us S us 0.5 S ew S es 26.91 ksi

7.

The safety factor is calculated from equation 14.16b. Fatigue factor of safety Nfs S es m i S us a S es S us i Nfs 1.41

8.

The spring rate is defined from the two specified forces at their relative deflection. Spring rate k Fmax Fmin k 266.7 lbf in

9.

To get the defined spring rate, the number of active coils must satisfy equation 14.7, solving for Na yields: Number of active coils Na d G 8 D k
3 4

Na 19.744

Na 19.75

Note that we round it to the nearest 1/4 coil as the manufacturing tolerance cannot achieve better than that accuracy. Having rounded the number of active coils, we must now calculate the spring rate using equation 14. Corrected spring rate k d G 8 D Na
3 4

k 266.59

lbf in

10. For squared ends the total number of coils, from Figure 14-9: Total coils Nt Na 2 Nt 21.75

11. The shut height can now be determined. Shut height Lshut d Nt Lshut 6.786 in

12. The initial deflection to reach the smaller of the two loads is Initial deflection yinit Fmin k yinit 0.188 in

13. For the given clash allowance of 15% of the working deflection: Clash allowance

yclash y

yclash 0.112 in

14. The free length (see Figure 14-8) can now be found from Lf Lshut yclash y yinit 15. The deflection to the shut height is Lf 7.836 in

MACHINE DESIGN - An Integrated Approach, 4th Ed.


yshut Lf Lshut 16. The force at this shut height is Fshut k yshut 17. The shut-height stress and safety factor are Stress at Fshut Fshut 279.9 lbf yshut 1.05 in

14-13-3

shut Ks
S ys

8 Fshut D

d
Safety factor at working deflection which is acceptable. Ns

shut 35.9 ksi

shut

Ns 3.71

18. To check for buckling, two ratios need to be calculated, Lf/D and ymax /Lf. Slenderness ratio sr y' Lf D yinit y Lf sr 5.708 y' 0.12

Deflection ratio

Take these two values to Figure 14-14 and find that their coordinates are safely within the zones that are stable against buckling for either end-condition case. 19. The inside and outside coil diameters and minimum hole dia are Inside coil dia Outside coil dia Minimum hole dia 20. Summarizing the results: Number of active coils Mean coil diameter Free length Shut length Deflection to Fmin Minimum hole dia Nt 21.75 D 1.373 in Lf 7.836 in Lshut 6.786 in yinit 0.188 in d hole 1.753 in Di D d Do D d d hole Do 0.05 D Di 1.061 in Do 1.685 in d hole 1.753 in

MACHINE DESIGN - An Integrated Approach, 4th Ed.

14-14-1

PROBLEM 14-14
Statement: Figure P14-2 shows a child's toy called a pogo stick. The child stands on the pads, applying half her weight on each side. She jumps off the ground, holding the pads up against her feet, and bounces along with the spring cushioning the impact and storing energy to help each rebound. Assume a 60-lb child and a spring constant of 100 lb/in. Design the helical compression spring to survive jumping 2 in off the ground with a dynamic safety factor of 2 for a finite life of 5E4 cycles Determine the fundamental natural frequency of the system. Weight of child Spring life W 60 lbf Ncycles 5 10
4

Given:

Spring constant Shear modulus

k 100 lbf in
6

G 11.5 10 psi

Design Choices: Clash allowance 0.15 Spring index C 7 Squared and ground ends Design safety factor Nfsd 1.5 Solution: 1. 2. 3. 4. See Mathcad file P1414.

Music wire properties: Strength A 184.65 ksi Set after winding, unpeened

b 0.1625

From Problem 3-14, Let the force of holding the pads against the feet be Then, the working deflection is

Maximum force Minimum force

Fmax 224.2 lbf Fmin 20 lbf Fmax Fmin k

y 2.042 in

Find the the mean and alternating forces from equation 14.15a: Alternating force Mean force Fa Fm Fmax Fmin 2 Fmax Fmin 2 0.5 C 0.615 C Fa 102.1 lbf Fm 122.1 lbf

5.

Calculate the factors necessary to find the wire diameter. Direct shear factor Wahl factor Yield strength factor Fatigue strength factor Ultimate shear strength factor Ks 1 Kw Ks 1.071 Kw 1.213 (Table 14-6, set removed) (Table 14-7, unpeened) (equation 14.4)

4 C 1 4 C 4

Kys 0.60 Kfw 0.37 KU 0.67

6.

Solving for the wire diameter using an equation similar to equation g in Example 14-4A (Mathcad Supplement) where S fw is used in equation 14.16b instead of S es and S fw = Kfw S ut.
1

Nfsd 1 8 C Nfsd d Ks Fm Ks Fmin Nfsd KU A in2 2 KU Kfw K w Fa Kfw

2 b

in

MACHINE DESIGN - An Integrated Approach, 4th Ed.


d 0.281 in 7. Let the wire diameter be d 0.281 in

14-14-2

Calculate the ultimate tensile strength, ultimate shear strength, and the fatigue strength at 5E4 cycles. Ultimate tensile strength Ultimate shear strength Wire fatigue strength Shear yield strength S ut A

in
d

S ut 227 ksi S us 152 ksi S fw 84.0 ksi S ys 136 ksi

S us KU S ut S fw Kfw S ut S ys Kys S ut S fs 0.5 S fw S us S us 0.5 S fw

Fatigue strength

S fs 58.0 ksi

8.

Calculate the mean coil diameter D from equation 14.5. Mean coil diameter D C d D 1.967 in

9.

Calculate the shear stress i at the initial deflection (lowest defined force), and the mean stress m. Stress at Fmin

i Ks

8 Fmin D

d
Stress at Fm

i 4.8 ksi

m Ks

8 Fm D

d
10. Calculate the alternating shear stress a in the coil. Alternating stress

m 29.5 ksi

a Kw

8 F a D

d
11. The safety factor is calculated from equation 14.16b. Fatigue factor of safety Nfs

a 28.0 ksi

S fs m i S us a

S fs S us i

Nfs 1.50

12. To get the defined spring rate, the number of active coils must satisfy equation 14.7, solving for Na yields: Number of active coils Na d G 8 D k
3 4

Na 11.777

Na 11.75

Note that we round it to the nearest 1/4 coil as the manufacturing tolerance cannot achieve better than that accuracy. Having rounded the number of active coils, we must now calculate the spring rate using equation 14.
4

Corrected spring rate

d G 8 D Na
3

k 100.23

lbf in

13. For squared and ground ends the total number of coils, from Figure 14-9: Total coils Nt Na 2 Nt 13.75

MACHINE DESIGN - An Integrated Approach, 4th Ed.


14. The shut height can now be determined. Shut height Lshut d Nt Lshut 3.864 in

14-14-3

15. The initial deflection to reach the smaller of the two loads is Initial deflection 16. For the given clash allowance factor: Clash allowance yinit Fmin k yinit 0.200 in

yclash y

yclash 0.306 in

17. The free length (see Figure 14-8) can now be found from Lf Lshut yclash y yinit 18. The deflection to the shut height is yshut Lf Lshut 19. The force at this shut height is Fshut k yshut 20. The shut-height stress and safety factor are Stress at Fshut Fshut 255.4 lbf yshut 2.548 in Lf 6.412 in

shut Ks
S ys

8 Fshut D

d
Safety factor at shut height which is acceptable. Ns

shut 61.8 ksi

shut

Ns 2.20

21. To check for buckling, two ratios need to be calculated, Lf/D and ymax /Lf. Slenderness ratio sr y' Lf D yinit y Lf sr 3.26 y' 0.35

Deflection ratio

Take these two values to Figure 14-14 and find that their coordinates are safely within the zones that are stable against buckling for either end-condition case. 22. The inside and outside coil diameters and minimum hole dia are Inside coil dia Outside coil dia Maximum pin dia 23. Summarizing the results: Number of active coils Mean coil diameter Free length Nt 13.75 D 1.967 in Lf 6.412 in Deflection to Fmin Maximum pin dia Shut length yinit 0.2 in d pin 1.588 in Lshut 3.864 in Di D d Do D d d pin Di 0.05 D Di 1.686 in Do 2.248 in d pin 1.588 in

MACHINE DESIGN - An Integrated Approach, 4th Ed.

14-15-1

PROBLEM 14-15
Statement: Given: Draw the Modified Goodman diagram for the spring data below and find its safety factor. Fatigue strength (R=0) Ultimate shear strength S fw 40 ksi S us 200 ksi Alternating shear stress Initial shear stress Solution: 1. See Mathcad file P1415.

a 12 ksi i 75 ksi

Mean shear stress m 95 ksi

From equation (d) in Example 14-2, S fs 0.5 S fw S us S us 0.5 S fw S fs 22.2 ksi

2.

Now, the equation for the Goodman line is

'a 'm aL 'm

S fs S us

'm S fs

Equation for load line

a m

'm

3.

Plotting this over the range

'm 0 ksi 10 ksi 200 ksi

25 Alternating shear stress 20 15 10 5 0

m
ksi

a
ksi

25

50

75

100

125

150

175

200

Mean shear stress

4.

Use equation 14.16b to calculate the safety factor. S fs S us i

Nfs

S fs m i S us a

Nfs 0.98

MACHINE DESIGN - An Integrated Approach, 4th Ed.

14-16-1

PROBLEM 14-16
Statement: Problem 6-16 describes a track for bowling balls that are 4.5-in dia and 2.5-lb weight. Design a spring-loaded launcher that will allow quadriplegic bowlers to launch the balls down the bowling alley from the point where the track of Problem 6-16 drops them with only a switch closure that releases the launcher. The launcher's plunger will be cocked by an assistant and the energy stored in the helical compression spring, which you will design, will drive the plunger into the ball and roll it down the bowling alley. Ball weight Wball 2.5 lbf Shear modulus G 11.5 10 psi
6

Given:

Design Choices: Clash allowance Spring index Design safety factor Ball initial velocity Deflection to Fmax Solution: 1. See Mathcad file P1416.

0.10
C 12 Nfsd 2 vi 150 in sec

Music wire properties: Strength A 184.65 ksi b 0.1625 Set after winding, unpeened
1

Squared and ground ends

y 5.000 in

Determine the required spring rate by equating the spring potential energy with the ball's initial velocity when the spring force reaches zero. 1 1 2 2 PE = KE k y = mball vi 2 2 mball Wball g
2

mball 6.475 10

3 lbf sec

in

vi k mball y
2.

k 5.828

lbf in

Determine the maximum spring force and set the minimum force to zero. Fmin 0 lbf Fmax k y Fmax 29.138 lbf

3.

Find the the mean and alternating forces from equation 14.15a: Alternating force Fa Fm Fmax Fmin 2 Fmax Fmin 2 Fa 14.6 lbf Fm 14.6 lbf

Mean force 4.

Calculate the factors necessary to find the wire diameter. Direct shear factor Wahl factor Yield strength factor Ultimate shear strength factor Torsional endurance limit Ks 1 Kw 0.5 C 0.615 C Ks 1.042 Kw 1.119 (Table 14-6, set removed) (equation 14.4) (unpeened)

4 C 1 4 C 4

Kys 0.60 KU 0.67 S ew 45 ksi

MACHINE DESIGN - An Integrated Approach, 4th Ed.


5. Solve for the wire diameter using equation g in Example 14-4A (Mathcad Supplement). Guess Given Nfsd 1 8 C Nfsd Ks Fm Ks Fmin Nfsd 2 0.67 A in b d A 0.67 in 1 K F w a 0.50 S ew d 0.209 in Let the wire diameter be d 0.2 in
1

14-16-2

d=

2 b

in

d Find ( d ) 6.

d 0.207 in

Calculate the ultimate tensile strength, ultimate shear strength, and the shear endurance limit. Ultimate tensile strength Ultimate shear strength Shear yield strength S ut A

in
d

S ut 239 ksi S us 160 ksi S ys 143 ksi

S us KU S ut S ys Kys S ut S es 0.5 S ew S us S us 0.5 S ew

Fatigue strength

S es 26.2 ksi

7.

Calculate the mean coil diameter D from equation 14.5. Mean coil diameter D C d D 2.484 in

8.

Calculate the shear stress i at the initial deflection (lowest defined force), and the mean stress m. Stress at Fmin

i Ks

8 Fmin D

d
Stress at Fm

i 0.0 ksi

m Ks

8 Fm D

d
9. Calculate the alternating shear stress a in the coil. Alternating stress

m 10.8 ksi

a Kw

8 F a D

d
10. The safety factor is calculated from equation 14.16b. Fatigue factor of safety Nfs

a 11.6 ksi

S es m i S us a
4

S es S us i

Nfs 2.0

11. To get the defined spring rate, the number of active coils must satisfy equation 14.7, solving for Na yields: Number of active coils Na d G 8 D k
3

Na 29.549

Na 29.5

MACHINE DESIGN - An Integrated Approach, 4th Ed.

14-16-3

Note that we round it to the nearest 1/4 coil as the manufacturing tolerance cannot achieve better than that accuracy. Having rounded the number of active coils, we must now calculate the spring rate using equation 14. Corrected spring rate k d G 8 D Na
3 4

k 5.84

lbf in

12. For squared and ground ends the total number of coils, from Figure 14-9: Total coils Nt Na 2 Nt 31.50

13. The shut height can now be determined. Shut height Lshut d Nt Lshut 6.52 in

14. The initial deflection to reach the smaller of the two loads is Initial deflection 15. For the given clash allowance factor: Clash allowance yinit Fmin k yinit 0.000 in

yclash y

yclash 0.500 in

16. The free length (see Figure 14-8) can now be found from Lf Lshut yclash y yinit 17. The deflection to the shut height is yshut Lf Lshut 18. The force at this shut height is Fshut k yshut 19. The shut-height stress and safety factor are Stress at Fshut 8 Fshut D Fshut 32.1 lbf yshut 5.500 in Lf 12.021 in

shut Ks
S ys

d
Safety factor at shut height which is acceptable. Ns

shut 23.8 ksi

shut

Ns 6.00

20. To check for buckling, two ratios need to be calculated, Lf/D and ymax /Lf. Slenderness ratio Deflection ratio sr y' Lf D yinit y Lf sr 4.839 y' 0.416

Take these two values to Figure 14-14 and find that their coordinates are safely within the zone that is stable against buckling for parallel ends. 21. The inside and outside coil diameters and minimum hole dia are Inside coil dia Outside coil dia Maximum pin dia Di D d Do D d d pin Di 0.05 D Di 2.277 in Do 2.691 in d pin 2.153 in

MACHINE DESIGN - An Integrated Approach, 4th Ed.


22. Summarizing the results: Number of active coils Mean coil diameter Free length Shut length Deflection to Fmin Maximum pin dia Nt 31.50 D 2.484 in Lf 12.021 in Lshut 6.520 in yinit 0 in d pin 2.153 in

14-16-4

MACHINE DESIGN - An Integrated Approach, 4th Ed.

14-17-1

PROBLEM 14-17
Statement: Design a helical extension spring to handle a dynamic load that varies from 175 N to 225 N over 8.5 mm working deflection. Use music wire and standard hooks. The forcing frequency is 1500 rpm. Infinite life is desired. Minimize the package size. Choose appropriate safety factors against fatigue, yielding, and surging. Minimum force Maximum force Working deflection Design Choices: Fatigue safety factor Wire endurance limit Fmin 175 N Fmax 225 N Life Shear modulus Forcing freq Spring index Wire strength L G 80.8 GPa

Given:

y 8.5 mm
Nfb 1.5 S ew 310 MPa

f 1500 rpm
C 8 A 2153.5 MPa b 0.1625

Solution: 1.

See Mathcad file P1417. Fmax Fmin 2 Fmax Fmin 2

Find the mean and alternating loads. Fa Fm Fa 25.0 N Fm 200.0 N

2.

Calculate the hook bending factor. Kb 4 C C 1 4 C ( C 1)


2

Kb 1.103

3.

Solve for d by iteration using an equation derived from the safety factor equation for bending in the hooks.
1

d=

4 4 K b C 1 Nfb Fm Nfb 1 Fmin A 0.577 0.67 A db 0.5 S ew Nfb Fa 0.5 0.67 S ew

2 b

4 4 Kb C 1 RHS( d ) Nfb Fm Nfb 1 Fmin A mm2 b 0.577 0.67 A d 0.5 S ew mm Nfb Fa 0.5 0.67 S ew
d 1.000 mm d RHS( d ) d RHS( d ) d RHS( d ) 4. Calculate the mean coil diameter. Mean coil diameter D C d D 28 mm RHS( d ) 3.561 mm RHS( d ) 3.329 mm RHS( d ) 3.340 mm RHS( d ) 3.339 mm d 3.5 mm

2 b

mm

MACHINE DESIGN - An Integrated Approach, 4th Ed.


5. Use the assumed value of C to find an appropiate value of initial coil stress i from equations 14.21:
3 2 3 2 2.987 C 139.7 C 3427 C 38404 psi

14-17-2

i1 4.231 C 181.5 C 3387 C 28640 psi i2

i1 75.8 MPa i2 126.9 MPa i 101.3 MPa

i
6.

i1 i2
2 0.5 C

Find the direct shear factor Ks : Direct shear factor Ks 1 Ks 1.063

7.

Use the value of i from (c) in equation 14.8 to find the corresponding initial coil-tension force Fi: Fi

d i
8 K s D

Fi 57.3 N

Check that this force is less than the required minimum applied force Fmin, which in this case, it is. Any applied force smaller than Fi will not deflect the spring. 8. Use the direct shear factor Ks and previously assumed values to find the mean stress m: Stress at Fm

m Ks

8 Fm D

d
9.

m 353.4 MPa

Find the Wahl factor Kw and use it to calculate the alternating shear stress a in the coil. Wahl factor Kw 4 C 1 4 C 4 0.615 C Kw 1.184

Alternating stress

a Kw

8 F a D

a 49.2 MPa

10. Find the ultimate tensile strength of this wire material from equation 14.3 and Table 14-4 and use it to find the ultimate shear strength from equation 14.4 and the torsional yield strength for the coil body from Table 14-10, assuming no set removal. Ultimate tensile strength Shear yield strength Ultimate shear strength S ut A

mm
d

S ut 1756.8 MPa S ys 790.6 MPa S us 1177.1 MPa

S ys 0.45 S ut S us 0.67 S ut

11. Find the wire endurance limit for unpeened springs from equation 14.13 and convert it to fully reversed endurance strength with equation 14.16c. Fully reversed endurance limit S es 0.5 S ew S us S us 0.5 S ew S es 178.51 MPa

12. The fatigue safety factor for the coils in torsion is calculated from equation 14.16b. Minimum stress

min m a
Nfs S es S us min

min 304.2 MPa


Nfs 2.34

Fatigue safety factor

S es m min S us a

MACHINE DESIGN - An Integrated Approach, 4th Ed.

14-17-3

Note that the minimum stress due to force Fmin is used in this calculation, not the coil-winding stress from step 5 13. The stresses in the end hooks also need to be determined. The bending stresses in the hook are found from equation 14.23: C1 = 2 R1 d =
2

2 D 2 d

=C

C1 C

C1 8.00

Kb

4 C1 C1 1 4 C1 C1 1 16 D Fa

Kb 1.103

a Kb

d m Kb

4 Fa

a 94.29 MPa

16 D Fm

4 Fm

d min Kb

m 754.3 MPa

16 D Fmin

4 Fmin

min 660.01 MPa

14. Convert the torsional endurance strength to a tensile endurance strength with the von Mises relationship and use it and the ultimate tensile strength in equation 14.16 to find a fatigue safety factor for the hook in bending: S e S es 0.577 S e S ut min S e 309.37 MPa

Nfb

S e m min S ut a C2 d 2 4 C2 1 4 C2 4 8 Fa D

Nfb 1.74

15. The torsional stresses in the hook are found from equation 14.24 using an assumed value of C2 5. R2 R2 8.75 mm Kw2 1.188

Kw2

Ba Kw2

d Bm Kw2

Ba 49.4 MPa

8 Fm D

d Bmin Kw2

Bm 395.0 MPa

8 Fmin D

Bmin 345.6 MPa

16. The fatigue safety factor for the hook in torsion is calculated from equation 14.16b. Nfs S es Bm Bmin S us Ba S es S us Bmin Nfs 2.22

MACHINE DESIGN - An Integrated Approach, 4th Ed.


17. The spring rate is defined from the two specified forces at their relative deflection. Spring rate k Fmax Fmin k 5882.4 N m

14-17-4

18. To get the defined spring rate, the number of active coils must satisfy equation 14.7, solving for Na yields: Number of active coils Na d G 8 D k Note that we round it to the nearest 1/4 coil as the manufacturing tolerance cannot achieve better than that accuracy. Having rounded the number of active coils, we must now calculate the spring rate using equation 14. Corrected spring rate k d G 8 D Na 19. The total number of coils in the body and the body length are Total coils Body length Nt Na 1 Lb Nt d Nt 12.75 Lb 44.6 mm
3 4 3 4

Na 11.737

Na 11.75

k 5876.0

N m

20. The free length can now be determined. The length of a standard hook is equal to the coil inside diameter: Hook length Free length Lhook D d Lf Lb 2 Lhook Lhook 24.5 mm Lf 93.6 mm

21. The initial coil tension force must be found again in order to obtain the deflection to reach the larger of the two loads.

i1 4.231 C 181.5 C 3387 C 28640 psi i2 i1 i2


2

3 2 3 2 2.987 C 139.7 C 3427 C 38404 psi

i1 75.8 MPa i2 126.9 MPa i 101.3 MPa


Fi 57.3 N

i
Fi

d i
8 K s D Fmax Fi k

ymax

ymax 28.53 mm

22. The inside and outside coil diameters are Inside coil dia Outside coil dia Di D d Do D d
3

Di 24.50 mm Do 31.50 mm

23. The weight of the spring's active coils is found from equation 14.11b and is Weight density Weight

0.28 lbf in
2 2

Wa

d D Na
4

Wa 0.756 N

24. The natural frequency of this spring is found from equation 14.11a and is:

MACHINE DESIGN - An Integrated Approach, 4th Ed.


Natural frequency fn 1 2 k g Wa fn 138.1 Hz

14-17-5

25. The ratio between the natural frequency and the forcing frequency is Forcing frequency ff fn ff

f
2

ff 25 Hz

Frequency ratio

5.5

which could be higher.

26. We now have a complete design specification for this A228-wire spring: Wire diameter Outside diameter d 3.50 mm Do 31.50 mm Total coils Free length Standard hooks Nt 12.75 Lf 93.63 mm

MACHINE DESIGN - An Integrated Approach, 4th Ed.

14-18-1

PROBLEM 14-18
Statement: Design a helical extension spring with standard hooks to handle a dynamic load that varies from 300 lb to 500 lb over 2 in working deflection. Use chrome-vanadium wire. The forcing frequency is 1000 rpm. Infinite life is desired. Minimize the package size. Choose appropriate safety factors against fatigue, yielding, and surging. Minimum force Maximum force Working deflection Design Choices: Fatigue safety factor Wire endurance limit Solution: 1. See Mathcad file P1418. Fmax Fmin 2 Fmax Fmin 2 4 C C 1 4 C ( C 1)
2

Given:

Fmin 300 lbf Fmax 500 lbf y 2.00 in Nfb 1.28 S ew 45 ksi

Life Shear modulus Forcing freq.

L G 11.7 10 psi f 1000 rpm C 5.5 A 173.13 ksi b 0.1453


6

Spring index Wire strength

Find the mean and alternating loads. Fa Fm Fa 100.0 lbf Fm 400.0 lbf

2.

Calculate the hook bending factor. Kb Kb 1.157

3.

Solve for d by iteration using an equation derived from the safety factor equation for bending in the hooks.
1

d=

4 4 K b C 1 Nfb Fm Nfb 1 Fmin A 0.577 0.67 A db 0.5 S ew Nfb Fa 0.5 0.67 S ew

2 b

4 4 Kb C 1 RHS( d ) Nfb Fm Nfb 1 Fmin A in2 b 0.577 0.67 A d 0.5 S ew in Nfb Fa 0.5 0.67 S ew
d 0.1 in d RHS( d ) d RHS( d ) d RHS( d ) 4. Calculate the mean coil diameter. Mean coil diameter D C d RHS( d ) 0.438 in RHS( d ) 0.404 in RHS( d ) 0.406 in RHS( d ) 0.406 in

2 b

in

d 0.406 in D 2.233 in

MACHINE DESIGN - An Integrated Approach, 4th Ed.


5. Use the assumed value of C to find an appropiate value of initial coil stress i from equations 14.21:
3 2 3 2 2.987 C 139.7 C 3427 C 38404 psi

14-18-2

i1 4.231 C 181.5 C 3387 C 28640 psi i2

i1 14.8 ksi i2 23.3 ksi i 19.0 ksi

i
6.

i1 i2
2 0.5 C

Find the direct shear factor Ks : Direct shear factor Ks 1 Ks 1.091

7.

Use the value of i from (c) in equation 14.8 to find the corresponding initial coil-tension force Fi: Fi

d i
8 K s D

Fi 205.4 lbf

Check that this force is less than the required minimum applied force Fmin, which in this case, it is. Any applied force smaller than Fi will not deflect the spring. 8. Use the direct shear factor Ks and previously assumed values to find the mean stress m: Stress at Fm

m Ks

8 Fm D

d
9.

m 37.1 ksi

Find the Wahl factor Kw and use it to calculate the alternating shear stress a in the coil. Wahl factor Kw 4 C 1 4 C 4 0.615 C Kw 1.278

Alternating stress

a Kw

8 F a D

a 10.9 ksi

10. Find the ultimate tensile strength of this wire material from equation 14.3 and Table 14-4 and use it to find the ultimate shear strength from equation 14.4 and the torsional yield strength for the coil body from Table 14-10, assuming no set removal. Ultimate tensile strength Shear yield strength Ultimate shear strength S ut A

in
d

S ut 197.4 ksi S ys 98.7 ksi S us 132.2 ksi

S ys 0.50 S ut S us 0.67 S ut

11. Find the wire endurance limit for unpeened springs from equation 14.13 and convert it to fully reversed endurance strength with equation 14.16c. Fully reversed endurance limit S es 0.5 S ew S us S us 0.5 S ew S es 27.11 ksi

12. The fatigue safety factor for the coils in torsion is calculated from equation 14.16b. Minimum stress

min m a
Nfs S es S us min

min 26.214 ksi


Nfs 1.66

Fatigue safety factor

S es m min S us a

MACHINE DESIGN - An Integrated Approach, 4th Ed.

14-18-3

Note that the minimum stress due to force Fmin is used in this calculation, not the coil-winding stress from step 5. 13. The stresses in the end hooks also need to be determined. The bending stresses in the hook are found from equation 14.23: C1 = 2 R1 d =
2

2 D 2 d

=C

C1 C

C1 5.50

Kb

4 C1 C1 1 4 C1 C1 1 16 D Fa

Kb 1.157

a Kb

d m Kb

4 Fa

a 20.43 ksi

16 D Fm

4 Fm

d min Kb

m 81.7 ksi

16 D Fmin

4 Fmin

min 61.28 ksi

14. Convert the torsional endurance strength to a tensile endurance strength with the von Mises relationship and use it and the ultimate tensile strength in equation 14.16 to find a fatigue safety factor for the hook in bending: S e Nfb S es 0.577 S e m min S ut a C2 d 2 4 C2 1 4 C2 4 8 Fa D S e S ut min S e 46.99 ksi Nfb 1.28

15. The torsional stresses in the hook are found from equation 14.24 using an assumed value of C2 5. R2 R2 1.015 in Kw2 1.188

Kw2

Ba Kw2

d Bm Kw2

Ba 10.1 ksi

8 Fm D

d Bmin Kw2

Bm 40.4 ksi

8 Fmin D

Bmin 30.3 ksi

16. The fatigue safety factor for the hook in torsion is calculated from equation 14.16b. Nfs S es Bm Bmin S us Ba S es S us Bmin Nfs 1.72

MACHINE DESIGN - An Integrated Approach, 4th Ed. 17. The spring rate is defined from the two specified forces at their relative deflection.
Spring rate k Fmax Fmin k 100.0 lbf in

14-18-4

18. To get the defined spring rate, the number of active coils must satisfy equation 14.7, solving for Na yields: Number of active coils Na d G 8 D k Note that we round it to the nearest 1/4 coil as the manufacturing tolerance cannot achieve better than that accuracy. Having rounded the number of active coils, we must now calculate the spring rate using equation 14. Corrected spring rate k d G 8 D Na 19. The total number of coils in the body and the body length are Total coils Body length Nt Na 1 Lb Nt d Nt 36.75 Lb 14.92 in
3 4 3 4

Na 35.689

Na 35.75

k 99.8

lbf in

20. The free length can now be determined. The length of a standard hook is equal to the coil inside diameter: Hook length Free length Lhook D d Lf Lb 2 Lhook Lhook 1.827 in Lf 18.57 in

21. The initial coil tension force must be found again in order to obtain the deflection to reach the larger of the two loads.

i1 4.231 C 181.5 C 3387 C 28640 psi i2 i1 i2


2

3 2 3 2 2.987 C 139.7 C 3427 C 38404 psi

i1 14.8 ksi i2 23.3 ksi i 19.0 ksi


Fi 205.4 lbf ymax 2.95 in

i
Fi

d i
8 K s D Fmax Fi k Di D d Do D d
3

ymax

22. The inside and outside coil diameters are Inside coil dia Outside coil dia Di 1.83 in Do 2.64 in

23. The weight of the spring's active coils is found from equation 14.11b and is Weight density Weight

0.28 lbf in
2 2

Wa

d D Na
4 1 2 k g Wa

Wa 9.091 lbf

24. The natural frequency of this spring is found from equation 14.11a and is: Natural frequency fn fn 32.6 Hz

MACHINE DESIGN - An Integrated Approach, 4th Ed.


25. The ratio between the natural frequency and the forcing frequency is

14-18-5

Forcing frequency Frequency ratio

ff fn ff

f
2

ff 16.667 Hz which could be higher.

2.0

26. We now have a complete design specification for this A228-wire spring: Wire diameter Outside diameter d 0.41 in Do 2.64 in Total coils Free length Standard hooks Nt 36.75 Lf 18.57 in

MACHINE DESIGN - An Integrated Approach, 4th Ed.

14-19-1

PROBLEM 14-19
Statement: Design a helical compression spring to handle a dynamic load that varies from 780 N to 1000 N over a 22-mm working deflection. Use squared and ground, unpeened music wire and a 10% clash allowance. The forcing frequency is 500 rpm. Infinite life is desired. Minimize the package size. Choose appropriate safety factors against fatigue, yielding, and surging. Minimum force Maximum force Fmin 780 N Fmax 1000 N Forcing freq. Shear modulus Clash factor b 0.1625

Given:

f 500 rpm
G 79.6 GPa

Working deflection y 22 mm Music wire properties: Strength A 2153.5 MPa Set after winding, unpeened Design Choices: Design safety factor Nfsd 1.1 Solution: 1. See Mathcad file P1419.

0.10

Spring index

C 9

Find the the mean and alternating forces from equation 14.15a: Alternating force Fa Fm Fmax Fmin 2 Fmax Fmin 2 Fa 110.0 N Fm 890.0 N

Mean force 2.

Calculate the factors necessary to find the wire diameter. Direct shear factor Wahl factor Yield strength factor Ultimate shear strength factor Torsional endurance limit Ks 1 Kw 0.5 C 0.615 C Ks 1.056 Kw 1.162 (Table 14-6, set removed) (equation 14.4) (unpeened)

4 C 1 4 C 4

Kys 0.60 KU 0.67 S ew 310 MPa

3.

Solve for the wire diameter using equation g in Example 14-4A (Mathcad Supplement). Guess Given Nfsd 1 8 C Nfsd d= Ks Fm Ks Fmin Nfsd 2 0.67 A mm b d A 0.67 mm 1 K F w a S ew 0.50 d Find ( d ) d 6.172 mm Let the wire diameter be d 5 mm
1

2 b

mm

d 6.5 mm

4.

Calculate the ultimate tensile strength, ultimate shear strength, and the shear endurance limit.

MACHINE DESIGN - An Integrated Approach, 4th Ed.


Ultimate tensile strength Ultimate shear strength Shear yield strength S ut A

14-19-2
d
b

mm

S ut 1589 MPa S us 1064 MPa S ys 953 MPa

S us KU S ut S ys Kys S ut S es 0.5 S ew S us S us 0.5 S ew

Fatigue strength

S es 181.4 MPa

5.

Calculate the mean coil diameter D from equation 14.5. Mean coil diameter D C d D 58.5 mm

6.

Calculate the shear stress i at the initial deflection (lowest defined force), and the mean stress m. Stress at Fmin

i Ks

8 Fmin D

d
Stress at Fm

i 446.6 MPa

m Ks

8 Fm D

d
7. Calculate the alternating shear stress a in the coil. Alternating stress

m 509.6 MPa

a Kw

8 F a D

d
8. The safety factor is calculated from equation 14.16b. Fatigue factor of safety 9. Nfs

a 69.3 MPa

S es m i S us a

S es S us i

Nfs 1.3

The spring rate is defined in this problem because of the two specified forces at a particular relative deflection. Spring rate k Fmax Fmin k 10.00 N mm

10. To get the defined spring rate, the number of active coils must satisfy equation 14.7, solving for Na yields: Number of active coils Na d G 8 D k
3 4

Na 8.872

Na 8.75

Note that we round it to the nearest 1/4 coil as the manufacturing tolerance cannot achieve better than that accuracy. Having rounded the number of active coils, we must now calculate the spring rate using equation 14. Corrected spring rate k d G 8 D Na
3 4

k 10.14

N mm

11. For squared and ground ends the total number of coils, from Figure 14-9: Total coils Nt Na 2 Nt 10.75

MACHINE DESIGN - An Integrated Approach, 4th Ed.


12. The shut height can now be determined. Shut height Lshut d Nt Lshut 69.9 mm

14-19-3

13. The initial deflection to reach the smaller of the two loads is Initial deflection 14. For the given clash allowance factor: Clash allowance yinit Fmin k yinit 76.9 mm

yclash y

yclash 2.2 mm

15. The free length (see Figure 14-8) can now be found from Lf Lshut yclash y yinit 16. The deflection to the shut height is yshut Lf Lshut 17. The force at this shut height is Fshut k yshut 18. The shut-height stress and safety factor are Stress at Fshut Fshut 1025 N yshut 101.1 mm Lf 171.0 mm

shut Ks
S ys

8 Fshut D

d
Safety factor at shut height which is acceptable. Ns

shut 587.1 MPa

shut

Ns 1.62

20. To check for buckling, two ratios need to be calculated, Lf/D and ymax /Lf. Slenderness ratio sr y' Lf D yinit y Lf sr 2.923 y' 0.579

Deflection ratio

Take these two values to Figure 14-14 and find that their coordinates are safely within the zones that are stable against buckling for both cases. 21. The inside and outside coil diameters are Inside coil dia Outside coil dia Di D d Do D d holemin Do 0.05 D pin max Di 0.05 D
3

Di 52 mm Do 65 mm holemin 67.9 mm pin max 49.1 mm

22. The smallest hole and largest pin that should be used with this spring are Smallest hole Largest pin

23. The weight of the springs active coils is found from equation 14.11b Weight density

0.285 lbf in

MACHINE DESIGN - An Integrated Approach, 4th Ed.


Wa

14-19-4
Wa 4.13 N

Weight

d D Na
4

2 2

24. The natural frequency of this spring is found from equation 14.11a and is: Natural frequency fn 1 2 k g Wa fn 77.6 Hz

25. The ratio between the natural frequency and the forcing frequency is Forcing frequency ff fn ff which is sufficiently high. 26. We now have a complete design specification for this A228 wire spring: Wire diameter Outside diameter Total coils Free length 25. The initial and working forces are Initial force Working force 26. Safety factors: Yielding Nys S ys Nys 1.6 Finit k yinit Fwork k yinit y Finit 780 N Fwork 1003 N d 6.5 mm Do 65.0 mm Nt 10.75 Lf 171 mm ends squared and ground

f
2

ff 8.333 Hz

Frequency ratio

9.3

shut
S es S us i

Fatigue

Nfs

S es m i S us a fn ff

Nfs 1.32

Surging

Nsurge

Nsurge 9.3

MACHINE DESIGN - An Integrated Approach, 4th Ed.

14-20-1

PROBLEM 14-20
Statement: Design a helical compression spring to handle a dynamic load that varies from 135 N to 220 N over a 32-mm working deflection. Use squared, peened chrome-vanadium wire and a 15% clash allowance. The forcing frequency is 250 rpm. Infinite life is desired. Minimize the package size. Choose appropriate safety factors against fatigue, yielding, and surging. Minimum force Maximum force Fmin 135 N Fmax 220 N Forcing freq. Shear modulus Clash factor b 0.1453

Given:

f 250 rpm
G 79.6 GPa

Working deflection y 32 mm Chrome-vanadium wire properties: Strength A 1909.9 MPa Set after winding, unpeened Design Choices: Design safety factor Nfsd 1.1 Solution: 1. See Mathcad file P1420.

0.15

Spring index

C 10.5

Find the the mean and alternating forces from equation 14.15a: Alternating force Fa Fm Fmax Fmin 2 Fmax Fmin 2 Fa 42.5 N Fm 177.5 N

Mean force 2.

Calculate the factors necessary to find the wire diameter. Direct shear factor Wahl factor Yield strength factor Ultimate shear strength factor Torsional endurance limit Ks 1 Kw 0.5 C 0.615 C Ks 1.048 Kw 1.138 (Table 14-6, set removed) (equation 14.4) (peened)

4 C 1 4 C 4

Kys 0.65 KU 0.67 S ew 465 MPa

3.

Solve for the wire diameter using equation g in Example 14-4A (Mathcad Supplement). Guess Given Nfsd 1 8 C Nfsd Ks Fm Ks Fmin Nfsd 2 0.67 A mm b d A mm 0.67 1 Kw Fa S ew 0.50 d 3.081 mm Let the wire diameter be d 5 mm
1

d=

2 b

mm

d Find ( d ) 4.

d 3.5 mm

Calculate the ultimate tensile strength, ultimate shear strength, and the shear endurance limit.

MACHINE DESIGN - An Integrated Approach, 4th Ed.


Ultimate tensile strength Ultimate shear strength Shear yield strength S ut A d

14-20-2

mm

S ut 1592 MPa S us 1067 MPa S ys 1035 MPa

S us KU S ut S ys Kys S ut S es 0.5 S ew S us S us 0.5 S ew

Fatigue strength

S es 297.3 MPa

5.

Calculate the mean coil diameter D from equation 14.5. Mean coil diameter D C d D 36.8 mm

6.

Calculate the shear stress i at the initial deflection (lowest defined force), and the mean stress m. Stress at Fmin

i Ks

8 Fmin D

d
Stress at Fm

i 308.7 MPa

m Ks

8 Fm D

d
7. Calculate the alternating shear stress a in the coil. Alternating stress

m 405.9 MPa

a Kw

8 F a D

d
8. The safety factor is calculated from equation 14.16b. Fatigue factor of safety 9. Nfs

a 105.5 MPa

S es m i S us a

S es S us i

Nfs 1.6

The spring rate is defined in this problem because of the two specified forces at a particular relative deflection. Spring rate k Fmax Fmin k 2.66 N mm

10. To get the defined spring rate, the number of active coils must satisfy equation 14.7, solving for Na yields: Number of active coils Na d G 8 D k
3 4

Na 11.325

Na 9.75

Note that we round it to the nearest 1/4 coil as the manufacturing tolerance cannot achieve better than that accuracy. Having rounded the number of active coils, we must now calculate the spring rate using equation 14. Corrected spring rate k d G 8 D Na
3 4

k 3.09

N mm

11. For squared and ground ends the total number of coils, from Figure 14-9: Total coils Nt Na 2 Lshut d Nt Nt 11.75 Lshut 1.619 in

12. The shut height can now be determined. Shut height

MACHINE DESIGN - An Integrated Approach, 4th Ed.


13. The initial deflection to reach the smaller of the two loads is Initial deflection 14. For the given clash allowance factor: Clash allowance yinit Fmin k yinit 43.8 mm

14-20-3

yclash y

yclash 4.8 mm

15. The free length (see Figure 14-8) can now be found from Lf Lshut yclash y yinit 16. The deflection to the shut height is yshut Lf Lshut 17. The force at this shut height is Fshut k yshut 18. The shut-height stress and safety factor are Stress at Fshut Fshut 248.5 N yshut 80.6 mm Lf 121.7 mm

shut Ks
S ys

8 Fshut D

d
Safety factor at shut height which is acceptable. Ns

shut 568.3 MPa

shut

Ns 1.82

20. To check for buckling, two ratios need to be calculated, Lf/D and ymax /Lf. Slenderness ratio sr y' Lf D yinit y Lf sr 3.311 y' 0.623

Deflection ratio

Take these two values to Figure 14-14 and find that their coordinates are safely within the zones that are stable against buckling for both cases. 21. The inside and outside coil diameters are Inside coil dia Outside coil dia Di D d Do D d holemin Do 0.05 D pin max Di 0.05 D Di 33.25 mm Do 40.25 mm holemin 42.1 mm pin max 31.4 mm

22. The smallest hole and largest pin that should be used with this spring are Smallest hole Largest pin

23. The weight of the springs active coils is found from equation 14.11b Weight density

0.285 lbf in
2 2

Weight

Wa

d D Na
4

Wa 0.188 lbf

24. The natural frequency of this spring is found from equation 14.11a and is:

MACHINE DESIGN - An Integrated Approach, 4th Ed.


1 2 k g Wa

14-20-4

Natural frequency

fn

fn 95.02 Hz

25. The ratio between the natural frequency and the forcing frequency is

Forcing frequency

ff fn ff

f
2

ff 4.167 Hz

Frequency ratio which is sufficiently high.

22.8

26. We now have a complete design specification for this A228 wire spring: Wire diameter Outside diameter Total coils Free length 25. The initial and working forces are Initial force Working force 26. Safety factors: Yielding Nys S ys Nys 1.8 Finit k yinit Fwork k yinit y Finit 135 N Fwork 234 N d 3.5 mm Do 40.3 mm Nt 11.75 Lf 122 mm ends squared

shut
S es S us i

Fatigue

Nfs

S es m i S us a fn ff

Nfs 1.6

Surging

Nsurge

Nsurge 23

MACHINE DESIGN - An Integrated Approach, 4th Ed.

14-21-1

PROBLEM 14-21
Statement: Given: Design a helical compression spring for a static load of 400 N at a deflection of 45 mm with a safety factor of 2.5. Use C = 8. Specify all parameters necessary to manufacture the spring. Working force Working deflection Spring index Design choices: Clash allowance ASTM A228 wire Solution: 1. 2. See Mathcad file P1421. k Fwork ywork k 8.889 N mm Fwork 400 N ywork 45 mm C 8 Shear modulus Safety factor G 80.8 GPa Ns 2.5 Km 0.65

0.15
A 2153.5 MPa b 0.1625

Set removed

Determine the desired spring rate.

Use the design equation from Example 14-3A (Mathcad Supplement) to determine the wire diameter.
1

8 Ns ( C 0.5) Fwork ( 1 ) 2 Km A mm
d 4.789 mm

2 b

mm

Wire diameter 3.

Let

d 5 mm

Calculate the mean coil diameter and number of active coils. Mean coil dia Number of active coils D C d Na d G 8 D k
3 4

D 40 mm Na 11.096 Na 11

Note that we round it to the nearest 1/4 coil as the manufacturing tolerance cannot achieve better than that accuracy. We must now calculate the actual (corrected) spring rate: Corrected spring rate 4. k d G 8 D Na
3 4

k 8.967

N mm

Assume squared and ground ends making the total number of coils, from Figure 14-9: Total coils Nt Na 2 Nt 13.00

5.

The shut height can now be determined. Shut height Ls d Nt Ls 65 mm

6.

The free length (see Figure 14-8) can now be found from Deflection to shut height Free length yshut ywork ywork Lf Ls yshut yshut 51.75 mm Lf 116.75 mm

7.

To check for buckling, two ratios need to be calculated, Lf/D and y max/Lf. Slenderness ratio sr Lf D sr 2.919

MACHINE DESIGN - An Integrated Approach, 4th Ed.


Deflection ratio y' ywork Lf y' 0.385

14-21-2

Take these two values to Figure 14-14 and find that their coordinates are safely within the zones that are stable against buckling for either end-condition case. 8. The inside and outside coil diameters are Inside coil dia Outside coil dia 9. Di D d Do D d holemin Do 0.05 D pin max Di 0.05 D
3

Di 35 mm Do 45 mm holemin 47.00 mm pin max 33.00 mm

The smallest hole and largest pin that should be used with this spring are Smallest hole Largest pin

10. The total weight of the spring is Weight density

0.28 lbf in
2 2

Weight

Wt

d D Nt
4

Wt 2.44 N

11. We now have a complete design specification for this A228 wire spring: Wire diameter Outside diameter Total coils Free length d 5 mm Do 45 mm Nt 13.00 Lf 116.75 mm ends squared and ground

MACHINE DESIGN - An Integrated Approach, 4th Ed.

14-22-1

PROBLEM 14-22
Statement: Design a straight-ended helical torsion spring for a static load 200 N-m at a deflection of 45 deg with a safety factor of 1.8. Specify all parameters necessary to manufacture the spring. State all assumptions. Applied moment M 200 N m Bending modulus E 206.8 GPa Deflection at load 45 deg 0.125 rev

Given:

Design Choices: Use unpeened oil tempered wire with 40-mm-long, straight ends. The coil is loaded to close it. Spring index Design safety factor Nyd 1.8 C 11 Yield strength factor Length of ends Material strength factors Solution: 1. See Mathcad file P1422. Ks 0.85 (from Table 14-13) L1 40 mm L2 40 mm A 1831.2 MPa b 0.1833 (ASTM A229 wire)

Calculate the Wahl bending factors for inside surface. Kbi 4 C C 1 4 C ( C 1 )


2

Kbi 1.073

2.

Solve for d using the static yield criterion.


1

32 Kbi Nyd M d Ks A mm3


3.

3 b

mm

d 16.144 mm

Use a wire diameter from the available sizes in Table 14-2. Calculate the mean coil diameter D from equation 14.5. Wire diameter Spring index Mean coil diameter d 16 mm C 11 D C d D 176 mm

4.

Calculate the maximum compressive stress in the coil at the inner surface.

imax Kbi
5.

32 M

imax 533.5 MPa

Find the ultimate tensile strength of this oil tempered material from equation 14.3 and Table 14-4 and use it to find the bending yield strength from Table 14-13, assuming stress relieving. Ultimate tensile strength Bending yield strength S ut A

mm
d

S ut 1102 MPa S y 936 MPa Nyb Sy Nyb 1.8

S y Ks S ut

6. 7.

The realized static safety factor against yielding is

imax

The spring rate is defined from the two specified moments at their relative deflection. Spring rate k M

k 1600

N m rev

8.

To get the defined spring rate, the number of active coils must satisfy equation 14.28, solving for Na yields:

MACHINE DESIGN - An Integrated Approach, 4th Ed.


Number of active coils Na d E 10.8 D k rev
4

14-22-2
Na 4.46

Note that to force k to be in units of N-m per rev we must multiply k by rev. 9. The ends contribute to the active coils from equation 14.26a as Ne L1 L2 3 D Ne 0.05

and, from equation 14.26b, the number of body coils in the spring are Nb Na Ne 10. Check the angular deflection at the specified load from equation 14.27c.

Nb 4.4

10.8

M D Na d E
4

rev

45.0 deg

MACHINE DESIGN - An Integrated Approach, 4th Ed.

14-23-1

PROBLEM 14-23
Statement: Design a straight-ended helical torsion spring for a static load 300 in-lb at a deflection of 75 deg with a safety factor of 2. Specify all parameters necessary to manufacture the spring. State all assumptions. Applied moment Deflection at load M 300 in lbf 75 deg Bending modulus E 30 10 psi 0.208 rev
6

Given:

Design Choices: Use unpeened music wire with 2-in-long, straight ends. The coil is loaded to close it. Spring index Design safety factor Nyd 2 C 7.5 Yield strength factor Length of ends Material strength factors Solution: 1. See Mathcad file P1423. Ks 1 (from Table 14-13) L1 2 in A 184.65 ksi L2 2 in b 0.1625 (ASTM A228 wire)

Calculate the Wahl bending factors for inside surface. Kbi 4 C C 1 4 C ( C 1 )


2

Kbi 1.110

2.

Solve for d using the static yield criterion.


1

32 Kbi Nyd M d Ks A in3


3.

3 b

in

d 0.312 in

Use a wire diameter from the available sizes in Table 14-2. Calculate the mean coil diameter D from equation 14.5. Wire diameter Spring index Mean coil diameter d 0.312 in C 7.5 D C d D 2.34 in

4.

Calculate the maximum compressive stress in the coil at the inner surface.

imax Kbi
5.

32 M

imax 111.7 ksi

Find the ultimate tensile strength of this oil tempered material from equation 14.3 and Table 14-4 and use it to find the bending yield strength from Table 14-13, assuming stress relieving. Ultimate tensile strength Bending yield strength S ut A

in
d

S ut 223 ksi S y 223 ksi Nyb Sy Nyb 2.0

S y Ks S ut

6. 7.

The realized static safety factor against yielding is

imax

The spring rate is defined from the two specified moments at their relative deflection. Spring rate k M

k 1440

in lbf rev

MACHINE DESIGN - An Integrated Approach, 4th Ed.


8.

14-23-2

To get the defined spring rate, the number of active coils must satisfy equation 14.28, solving for Na yields: Number of active coils Na d E 10.8 D k rev
4

Na 7.81

Note that to force k to be in units of N-m per rev we must multiply k by rev. 9. The ends contribute to the active coils from equation 14.26a as Ne L1 L2 3 D Ne 0.18 Nb 7.6

and, from equation 14.26b, the number of body coils in the spring are Nb Na Ne 10. Check the angular deflection at the specified load from equation 14.27c.

10.8

M D Na d E
4

rev

75.0 deg

MACHINE DESIGN - An Integrated Approach, 4th Ed.

14-24-1

PROBLEM 14-24
Statement: Design a straight-ended helical torsion spring for a dynamic load 50-105 N-m at a deflection of 80 deg with a safety factor of 2. Specify all parameters necessary to manufacture the spring. State all assumptions. Bending modulus Minimum moment Mmin 50 N m E 206.8 GPa Maximum moment Mmax 105 N m Deflection at load

Given:

80 deg

Design Choices: Use unpeened oil tempered wire with 40-mm-long, straight ends. The coil is loaded to close it. Design safety factor Nd 2.5 Yield strength factor Length of ends Material strength factors Wire endurance limit See Mathcad file P1424. Solution: 1. Find the mean and alternating loads. Ma Mm 2. Mmax Mmin 2 Mmax Mmin 2 Ma 27.5 N m Mm 77.5 N m Ks 1 (from Table 14-13) L1 40 mm L2 40 mm C 8.3 A 1831.2 MPa b 0.1833 (ASTM A229 wire) Spring index S ewb 537 MPa

Calculate the Wahl bending factors for inside and outside surfaces. Kbi Kbo 4 C C 1 4 C ( C 1 ) 4 C C 1 4 C ( C 1 )
2 2

Kbi 1.099 Kbo 0.916

3.

Solve for d using the static yield criterion. Use this value of d for starting the iteration in the next step.
1 3 b

32 Kbi Nd Mmax d Ks A mm3


4.

mm

d 13.739 mm

Solve for d by iteration using the fatigue criterion.


1 b d A N M mm M min 32 Kbo fb a 0.5 S ewb d= A 3 b

1 b d A mm 32 Kbo Nd Ma Mmin 0.5 S ewb RHS( d ) 3 A mm 3 b

mm

MACHINE DESIGN - An Integrated Approach, 4th Ed.


d RHS( d ) d RHS( d ) d RHS( d ) d 14.120 mm d 14.099 mm d 14.100 mm

14-24-2

Since this results in a slightly larger wire diameter, the fatigue criterion is governing and the spring must be designed for fatigue. 3. Use a wire diameter from the available sizes in Table 14-2. Calculate the mean coil diameter D from equation 14 Wire diameter Spring index Mean coil diameter 4. d 15 mm C 8.3 D C d

D 124.5 mm

Calculate the maximum compressive stress in the coil at the inner surface.

imax Kbi
5.

32 Mmax

imax 348.1 MPa

Calculate the maximum, minimum, alternating, and mean tensile stresses in the coil at the outer surface.

omin Kbo

32 Mmin

d omax Kbo

omin 138.2 MPa

32 Mmax

d m

omax 290.3 MPa

omax omin
2

m 214.3 MPa

a
6.

omax omin
2

a 76.0 MPa

Find the ultimate tensile strength of this oil tempered material from equation 14.3 and Table 14-4 and use it to find the bending yield strength from Table 14-13, assuming stress relieving. Ultimate tensile strength Bending yield strength S ut A

in
d

S ut 2017 MPa S y 2017 MPa

S y Ks S ut

7.

Convert the wire bending endurance limit for unpeened springs from equation 14.33 to fully reversed endurance strength with equation 14.34b. S ewb S ut Fully reversed S e 0.5 S e 309.74 MPa S ut 0.5 S ewb endurance limit The fatigue safety factor for the coils in bending is calculated from equation 14.34a. Fatigue factor of safety Nfb S e m omin S ut a S e S ut omin Nfb 3.3

8.

9.

The static safety factor against yielding is Static factor of safety Nyb Sy

imax

Nyb 5.8

MACHINE DESIGN - An Integrated Approach, 4th Ed.


These are both acceptable safety factors. 10. The spring rate is defined from the two specified moments at their relative deflection. Spring rate k Mmax Mmin k 247.5 N m rev

14-24-3

11. To get the defined spring rate, the number of active coils must satisfy equation 14.28, solving for Na yields: Number of active coils Na d E 10.8 D k rev
4

Na 31.46

Note that to force k to be in units of N-m per rev we must multiply k by rev. 12. The ends contribute to the active coils from equation 14.26a as Ne L1 L2 3 D Ne 0.07 Nb 31

and, from equation 14.26b, the number of body coils in the spring are Nb Na Ne 13. The angular deflections at the specified loads from equation 14.27c are

min 10.8

Mmin D Na d E
4

rev

min 73 deg

max 10.8

Mmax D Na d E
4

rev

max 153 deg

MACHINE DESIGN - An Integrated Approach, 4th Ed.

14-25-1

PROBLEM 14-25
Statement: Design a straight-ended helical torsion spring for a dynamic load 150-350 in-lb over a deflection of 50 deg with a safety factor of 1.4. Specify all parameters necessary to manufacture the spring. State all assumptions. Minimum moment Maximum moment Mmin 150 in lbf Mmax 350 in lbf Bending modulus Deflection at load E 30 10 psi 50 deg
6

Given:

Design Choices: Use unpeened music wire with 2-in-long, straight ends. The coil is loaded to close it. Design safety factor Nd 1.4 Yield strength factor Length of ends Material strength factors Wire endurance limit Solution: 1. See Mathcad file P1425. Mmax Mmin 2 Mmax Mmin 2 Ks 0.80 (from Table 14-13) L1 2 in L2 2 in A 184.65 ksi b 0.1625 (ASTM A228 wire) Spring index S ewb 77.99 ksi C 4.5

Find the mean and alternating loads. Ma Mm Ma 100.0 in lbf Mm 250.0 in lbf

2.

Calculate the Wahl bending factors for inside and outside surfaces. Kbi Kbo 4 C C 1 4 C ( C 1 ) 4 C C 1 4 C ( C 1 )
2 2

Kbi 1.198 Kbo 0.854

3.

Solve for d using the static yield criterion.


1 3 b

32 Kbi Nd Mmax d Ks A in3


4.

in

d 0.323 in

Solve for d by iteration using the fatigue criterion.


1 b d A N M mm M min 32 Kbo d a 0.5 S ewb d= A 3 b

1 b d A in 32 Kbo Nd Ma Mmin 0.5 S ewb RHS( d ) 3 A in 3 b

in

MACHINE DESIGN - An Integrated Approach, 4th Ed.


d RHS( d ) d RHS( d ) d 0.334 in d 0.334 in

14-25-2

Since this results in a slightly larger wire diameter, the fatigue criterion is governing and the spring must be designed for fatigue. 3. Use a wire diameter from the available sizes in Table 14-2. Calculate the mean coil diameter D from equation 14 Wire diameter Spring index Mean coil diameter 4. d 0.343 in C 4.5 D C d D 1.544 in

Calculate the maximum compressive stress in the coil at the inner surface.

imax Kbi
5.

32 Mmax

imax 105.9 ksi

Calculate the maximum, minimum, alternating, and mean tensile stresses in the coil at the outer surface.

omin Kbo

32 Mmin

d omax Kbo

omin 32.3 ksi

32 Mmax

d m

omax 75.4 ksi

omax omin
2

m 53.9 ksi

a
6.

omax omin
2

a 21.5 ksi

Find the ultimate tensile strength of this oil tempered material from equation 14.3 and Table 14-4 and use it to find the bending yield strength from Table 14-13, assuming stress relieving. Ultimate tensile strength S ut A

in
d

S ut 220 ksi S y 176 ksi

Bending yield strength 7.

S y Ks S ut

Convert the wire bending endurance limit for unpeened springs from equation 14.33 to fully reversed endurance strength with equation 14.34b. Fully reversed endurance limit S e 0.5 S ewb S ut S ut 0.5 S ewb S e 47.41 ksi

8.

The realized fatigue safety factor for the coils in bending is calculated from equation 14.34a. Fatigue factor of safety Nfb S e m omin S ut a S e S ut omin Nfb 1.5

9.

The static safety factor against yielding is

MACHINE DESIGN - An Integrated Approach, 4th Ed.


Static factor of safety Nyb Sy Nyb 1.7

14-25-3

imax

These are both acceptable safety factors. 10. The spring rate is defined from the two specified moments at their relative deflection. Spring rate k Mmax Mmin k 1440 in lbf rev

11. To get the defined spring rate, the number of active coils must satisfy equation 14.28, solving for Na yields: Number of active coils Na d E 10.8 D k rev
4

Na 17.30

Note that to force k to be in units of in-lb per rev we must multiply k by rev. 12. The ends contribute to the active coils from equation 14.26a as Ne L1 L2 3 D Ne 0.27 Nb 17

and, from equation 14.26b, the number of body coils in the spring are Nb Na Ne 13. The angular deflections at the specified loads from equation 14.27c are

min 10.8

Mmin D Na d E
4

rev

min 38 deg

max 10.8

Mmax D Na d E
4

rev

max 88 deg

MACHINE DESIGN - An Integrated Approach, 4th Ed.

14-26-1

PROBLEM 14-26
Statement: Given: Design a Belleville spring to give a constant 400 N 10% force over a 1-mm deflection. Nominal force Fflat 400 N Working deflection ywork 1 mm

Design Choices: The diameter ratio is Rd 2. Use unset carbon spring steel 50HRC. Properties, E 30 10 psi ,
6

0.28 . The outside diameter of the spring is Do 39.55 mm


Solution: 1. See Mathcad file P1426. Since a constant force spring is needed, the h/t ratio (see Figure 14-30) is h/t ratio 2. hovert 1.414

The required force variation of not more than 10% can be met by choosing an appropriate deflection range to operate in from Figure 14-31. If the deflection is kept between about 53% and 146% of the flat deflection, this tolerance will be achieved. The nominal force will then occur at the flat position and the spring must operate on both sides, so must be mounted in similar fashion to that shown in Figure 14-33. Use the above assumptions and the specified nominal force in equation 14.37a to find an appropriate spring thickness t:
1 4 2 Fflat Do t 10 132.4 MPa hovert

3.

t 0.760 mm h 1.075 mm

Let

t 0.760 mm

4. 5.

The height h can now be found:

h hovert t

Based on the choices in step 2, find the minimum and maximum deflections: ymin 0.53 h ymax 1.46 h ymin 0.57 mm ymax 1.57 mm

The difference between these distances is equal to the required deflection range, so the force tolerance can be met over that range. 6. Figure 14-34 shows that the worst stress state will occur at the largest deflection ymax, so solve equations 14.36 for stresses at that deflection:

Rd 1 2 K1 ln Rd 2 Rd
6 K2

K1 0.689

ln Rd ln Rd ln Rd
6

Rd 1 Rd 2

K2 1.22

K3

K3 1.378

K4 K5

Rd ln Rd Rd 1 Rd ln Rd Rd 1 2
2 Rd 1 Rd

K4 1.115 K5 1

MACHINE DESIGN - An Integrated Approach, 4th Ed.


4 E ymax K1 Do 1
2

14-26-2

K2 h

ymax 2

K3 t

c 1831 MPa

ti

4 E ymax K1 Do2 1 2

y K2 h max K3 t 2 y K4 h max K5 t 2

ti 906.1 MPa

to

4 E ymax K1 Do2 1 2

to 1416 MPa

7.

Table 14-5 gives S ut 246 ksi for this material. Table 14-15 indicates that 120% of this value can be used for an unset spring. The safety factor for static loading is then Ns 1.20 S ut Ns 1.1

which is acceptable. 8. A summary of the spring design is Outside diameter Inside diameter Thickness Height Do 39.55 mm Di Do Rd Di 19.77 mm

t 0.76 mm h 1.07 mm

MACHINE DESIGN - An Integrated Approach, 4th Ed.

14-27-1

PROBLEM 14-27
Statement: Given: Design a Belleville spring for bimodal operation between 50 N. Nominal force Fflat 50 N

Design Choices: The diameter ratio is Rd 2. Use unset carbon spring steel 50HRC. Properties: E 30 10 psi, 0.28 . The outside diameter of the spring is Do 24.50 mm Solution: 1. See Mathcad file P1427.
6

Since a bimodal force spring is needed, the h/t ratio (see Figure 14-30) is h/t ratio hovert 2.828

2.

The required force variation of not more than 100% can be met by choosing an appropriate deflection range to operate in from Figure 14-30. If the deflection is kept between about 0% and 200% of the flat deflection, this will be achieved. A zero force will then occur at the flat position and the spring must operate on both sides, so must be mounted in similar fashion to that shown in Figure 14-32. Use the above assumptions and the specified nominal force in equation 14.37a to find an appropriate spring thickness t:
1 2 Do Fflat t 7 hovert 19.2 10 psi 4

3.

t 0.299 mm h hovert t

Let

t 0.30 mm h 0.848 mm

4. 5.

The height h can now be found:

Based on the choices in step 2, find the minimum and maximum deflections: ymin 0 h ymax 2.00 h ymin 0 mm ymax 1.70 mm

The difference between these distances is equal to the required deflection range, so Working deflection 6. ywork ymax ywork 1.70 mm

Figure 14-34 shows that the worst stress state will occur at the largest deflection ymax, so solve equations 14.36 for stresses at that deflection:

R 1 2 d K1 ln Rd 2 Rd
6 K2

K1 0.689

ln Rd ln Rd ln Rd
6

Rd 1 Rd 2

K2 1.22

K3

K3 1.378

K4

Rd ln Rd Rd 1 Rd ln Rd Rd 1 2
2 Rd 1 Rd

K4 1.115

K5

K5 1

MACHINE DESIGN - An Integrated Approach, 4th Ed.


4 E ymax K1 Do 1
2

14-27-2

K2 h

ymax 2

K3 t

c 1523 MPa

ti

4 E ymax K1 Do2 1 2

y K2 h max K3 t 2 y K4 h max K5 t 2

ti 1523 MPa

to

4 E ymax K1 Do2 1 2

to 1105 MPa

7. for

Table 14-5 gives S ut 246 ksi for this material. Table 14-15 indicates that 120% of this value can be used an unset spring. The safety factor for static loading is then 1.20 S ut Ns Ns 1.3

which is acceptable. 8. A summary of the spring design is Outside diameter Inside diameter Thickness Height Do 24.5 mm Di Do Rd Di 12.25 mm

t 0.3 mm h 0.85 mm

MACHINE DESIGN - An Integrated Approach, 4th Ed.

14-28-1

PROBLEM 14-28
Statement: Given: Given the data below for a helical compression spring loaded in fatigue, design the spring for infinite life. State all assumptions and empirical data used. Minimum force Maximum force Fmin 225 N Fmax 450 N Forcing freq. Shear modulus

f 625 rpm
G 79.3 GPa

Working deflection y 20 mm Music wire properties: Strength A 2153.5 MPa b 0.1625 Wire diameter d 8 mm Solution: 1. See Mathcad file P1428.

Clash factor 0.15 Spring index C 8.5 Set after winding, unpeened

Find the the mean and alternating forces from equation 14.15a: Alternating force Fa Fm Fmax Fmin 2 Fmax Fmin 2 Fa 112.5 N Fm 337.5 N

Mean force 2.

Calculate the factors necessary to find the fatigue factor of safety. Direct shear factor Wahl factor Yield strength factor Ultimate shear strength factor Torsional endurance limit Ks 1 Kw 0.5 C 0.615 C Ks 1.059 Kw 1.172 (Table 14-6, set removed) (equation 14.4) (unpeened)

4 C 1 4 C 4

Kys 0.60 KU 0.67 S ew 310 MPa

3.

Calculate the ultimate tensile strength, ultimate shear strength, and the shear endurance limit. Ultimate tensile strength Ultimate shear strength Shear yield strength S ut A

mm
d

S ut 1536 MPa S us 1029 MPa S ys 922 MPa

S us KU S ut S ys Kys S ut S es 0.5 S ew S us S us 0.5 S ew

Fatigue strength

S es 182.5 MPa

4.

Calculate the mean coil diameter D from equation 14.5. Mean coil diameter D C d D 68.0 mm

5.

Calculate the shear stress i at the initial deflection (lowest defined force), and the mean stress m. Stress at Fmin

i Ks

8 Fmin D

i 80.6 MPa

MACHINE DESIGN - An Integrated Approach, 4th Ed.

14-28-2

Stress at Fm

m Ks

8 Fm D

d
6. Calculate the alternating shear stress a in the coil. Alternating stress

m 120.9 MPa

a Kw

8 F a D

d
7. The safety factor is calculated from equation 14.16b. Fatigue factor of safety 8. Nfs

a 44.6 MPa

S es m i S us a

S es S us i

Nfs 3.3

The spring rate is defined in this problem because of the two specified forces at a particular relative deflection. Spring rate k Fmax Fmin k 11.25 N mm

9.

To get the defined spring rate, the number of active coils must satisfy equation 14.7, solving for Na yields: Number of active coils Na d G 8 D k
3 4

Na 11.478

Na 11.5

Note that we round it to the nearest 1/4 coil as the manufacturing tolerance cannot achieve better than that accuracy. Having rounded the number of active coils, we must now calculate the spring rate using equation 14. Corrected spring rate k d G 8 D Na Nt Na 2 Nt 13.50
3 4

k 11.23

N mm

10. For squared ends the total number of coils, from Figure 14-9: Total coils

11. The shut height can now be determined. Shut height Lshut d Nt Lshut 108 mm

12. The initial deflection to reach the smaller of the two loads is Initial deflection 13. For the given clash allowance factor: Clash allowance yinit Fmin k yinit 20.0 mm

yclash y

yclash 3.0 mm

14. The free length (see Figure 14-8) can now be found from Lf Lshut yclash y yinit 15. The deflection to the shut height is yshut Lf Lshut 16. The force at this shut height is Fshut k yshut Fshut 483.3 N yshut 43.0 mm Lf 151 mm

MACHINE DESIGN - An Integrated Approach, 4th Ed.

14-28-3

17. The shut-height stress and safety factor are Stress at Fshut

shut Ks
S ys

8 Fshut D

d
Safety factor at shut height which is acceptable. Ns

shut 173.1 MPa

shut

Ns 5.3

18. To check for buckling, two ratios need to be calculated, Lf/D and ymax /Lf. Slenderness ratio sr y' Lf D yinit y Lf sr 2.221 y' 0.265

Deflection ratio

Take these two values to Figure 14-14 and find that their coordinates are safely within the zones that are stable against buckling for both cases. 19. The inside and outside coil diameters are Inside coil dia Outside coil dia Di D d Do D d holemin Do 0.05 D pin max Di 0.05 D
3

Di 60 mm Do 76 mm holemin 79.4 mm pin max 56.6 mm

20. The smallest hole and largest pin that should be used with this spring are Smallest hole Largest pin

21. The weight of the springs active coils is found from equation 14.11b Weight density

0.285 lbf in
2 2

Weight

Wa

d D Na
4

Wa 9.6 N

22. The natural frequency of this spring is found from equation 14.11a and is: Natural frequency fn 1 2 k g Wa fn 53.68 Hz

23. The ratio between the natural frequency and the forcing frequency is

Forcing frequency Frequency ratio which is a little low.

ff fn ff

f
2

ff 10.42 Hz

5.2

MACHINE DESIGN - An Integrated Approach, 4th Ed.

14-28-4

24. We now have a complete design specification for this A228 wire spring: Wire diameter Outside diameter Total coils Free length 25. The initial and working forces are Initial force Working force 26. Safety factors: Yielding Nys S ys Nys 5.3 Finit k yinit Fwork k yinit y Finit 225 N Fwork 450 N d 8 mm Do 76.0 mm Nt 13.50 Lf 151.038 mm ends squared

shut
S es S us i

Fatigue

Nfs

S es m i S us a fn ff

Nfs 3.3

Surging

Nsurge

Nsurge 5.2

MACHINE DESIGN - An Integrated Approach, 4th Ed.

14-29-1

PROBLEM 14-29
Statement: A helical extension spring, loaded in fatigue, has been designed for infinite life with the data given below. Find the safety factors for failure in the standard hooks. State all assumptions and sources of empirical data. Minimum force Maximum force Working deflection Wire endurance limit Wire diameter Number active coils Solution: 1. See Mathcad file P1429. Fmax Fmin 2 Fmax Fmin 2 Fmin 665 N Fmax 935 N Life Shear modulus Spring index Wire strength L G 79.3 GPa C 9 A 2059.2 MPa

Given:

y 2.00 in
S ew 310 MPa d 8 mm Na 13.75

(Chrome-silicon) b 0.0934

Find the mean and alternating loads. Fa Fm Fa 135.0 N Fm 800.0 N

2.

Calculate the hook bending factor. Kb 4 C C 1 4 C ( C 1)


2

Kb 1.090

3.

Calculate the mean coil diameter. Mean coil diameter D C d D 72 mm

4.

Use the value of C to find an appropiate value of initial coil stress i from equations 14.21:

i1 4.231 C 181.5 C 3387 C 28640 psi i2

3 2 3 2 2.987 C 139.7 C 3427 C 38404 psi

i1 67.4 MPa i2 115.1 MPa i 91.3 MPa

i
5.

i1 i2
2

Find the direct shear factor Ks : Direct shear factor Ks 1 0.5 C Ks 1.056

6.

Use the direct shear factor Ks and previously assumed values to find the mean stress m: Stress at Fm

m Ks

8 Fm D

d
7.

m 302.4 MPa

Find the Wahl factor Kw and use it to calculate the alternating shear stress a in the coil. Wahl factor Kw 4 C 1 4 C 4 0.615 C Kw 1.162

Alternating stress

a Kw

8 F a D

a 56.2 MPa

MACHINE DESIGN - An Integrated Approach, 4th Ed.


8.

14-29-2

Find the ultimate tensile strength of this wire material from equation 14.3 and Table 14-4 and use it to find the ultimate shear strength from equation 14.4 and the torsional yield strength for the coil body from Table 14-10, assuming no set removal. Ultimate tensile strength Shear yield strength Ultimate shear strength S ut A

mm
d

S ut 1695.7 MPa S ys 847.9 MPa S us 1136.1 MPa

S ys 0.50 S ut S us 0.67 S ut

9.

Find the wire endurance limit for unpeened springs from equation 14.13 and convert it to fully reversed endurance strength with equation 14.16c. Fully reversed endurance limit S es 0.5 S ew S us S us 0.5 S ew S es 179.49 MPa

10. The stresses in the end hooks also need to be determined. The bending stresses in the hook are found from equation 14.23: C1 = 2 R1 d =
2

2 D 2 d

=C

C1 C

C1 9.00

Kb

4 C1 C1 1 4 C1 C1 1 16 D Fa

Kb 1.09

a Kb

d m Kb

4 Fa

a 108.1 MPa

16 D Fm

4 Fm

d min Kb

m 640.6 MPa

16 D Fmin

4 Fmin

min 532.5 MPa

11. Convert the torsional endurance strength to a tensile endurance strength with the von Mises relationship and use it and the ultimate tensile strength in equation 14.16 to find a fatigue safety factor for the hook in bending: S e S es 0.577 S e S ut min S e 311.1 MPa

Nfb

S e m min S ut a

Nfb 1.67

12. The torsional stresses in the hook are found from equation 14.24 using an assumed value of C2 5. R2 C2 d 2 4 C2 1 4 C2 4 R2 20 mm Kw2 1.188

Kw2

MACHINE DESIGN - An Integrated Approach, 4th Ed.

14-29-3

Ba Kw2

8 Fa D

d Bm Kw2

Ba 57.4 MPa

8 Fm D

d Bmin Kw2

Bm 340.2 MPa

8 Fmin D

Bmin 282.8 MPa

13. The fatigue safety factor for the hook in torsion is calculated from equation 14.16b. Nfs S es Bm Bmin S us Ba S es S us Bmin Nfs 2.0

MACHINE DESIGN - An Integrated Approach, 4th Ed.

14-30-1

PROBLEM 14-30
Statement: Given the following data for a helical torsion spring, loaded in fatigue, find the spring index, unloaded coil diameter, minimum loaded coil diameter, and safety factor in fatigue. State all assumptions and sources of empirical data used. Minimum deflection Maximum deflection Number active coils

Given:

min 0.25 rev max 0.75 rev


Na 20

Bending modulus

E 30 10 psi
1

Spring rate k 60 in lbf rev Wire dia. d 0.192 in

Design Choices: Use unpeened music wire with 2-in-long, straight ends. The coil is loaded to close it. Yield strength factor Ks 0.80 (from Table 14-13) Length of ends Material strength factors Wire endurance limit Solution: 1. See Mathcad file P1430. L1 2 in L2 2 in A 184.65 ksi b 0.1625 (ASTM A228 wire) S ewb 77.99 ksi

Find the minimum, maximum, mean, and alternating loads. Mmin k min Mmax k max Ma Mm Mmax Mmin 2 Mmax Mmin 2 Mmin 15 in lbf Mmax 45 in lbf Ma 15.0 in lbf Mm 30.0 in lbf k= d E 10.8 C Na
3

2.

Use equation 14.28 to calculate the spring index. d E 10.8 k rev Na


3

C 3.

C 16.38

Calculate the Wahl bending factors for inside and outside surfaces. Kbi Kbo 4 C C 1 4 C ( C 1 ) 4 C C 1 4 C ( C 1 )
2 2

Kbi 1.048 Kbo 0.956

4.

Calculate the mean coil diameter D from equation 14.5. Mean coil diameter D C d D 3.146 in

5.

Calculate the maximum compressive stress in the coil at the inner surface.

imax Kbi
6.

32 Mmax

imax 67.9 ksi

Calculate the maximum, minimum, alternating, and mean tensile stresses in the coil at the outer surface.

omin Kbo

32 Mmin

omin 20.6 ksi

MACHINE DESIGN - An Integrated Approach, 4th Ed.

14-30-2

omax Kbo

32 Mmax

d m

omax 61.9 ksi

omax omin
2

m 41.3 ksi

a
7.

omax omin
2

a 20.6 ksi

Find the ultimate tensile strength of this oil tempered material from equation 14.3 and Table 14-4 and use it to find the bending yield strength from Table 14-13, assuming stress relieving. Ultimate tensile strength S ut A

in
d

S ut 241 ksi S y 193 ksi

Bending yield strength 8.

S y Ks S ut

Convert the wire bending endurance limit for unpeened springs from equation 14.33 to fully reversed endurance strength with equation 14.34b. Fully reversed endurance limit S e 0.5 S ewb S ut S ut 0.5 S ewb S e 46.51 ksi

9.

The fatigue safety factor for the coils in bending is calculated from equation 14.34a. Fatigue factor of safety Nfb S e m omin S ut a Ne L1 L2 3 D S e S ut omin Nfb 1.7

10. The ends contribute to the active coils from equation 14.26a as

Ne 0.13 Nb 20

and, from equation 14.26b, the number of body coils in the spring are Nb Na Ne 11. The unloaded coil diameter and minimum loaded coil diameters are: Unloaded coil diameter Di D d Dimin D Nb Nb d Di 2.954 in Dimin 2.839 in

Loaded coil diameter

max
rev

MACHINE DESIGN - An Integrated Approach, 4th Ed.

14-31-1

PROBLEM 14-31
Statement: A helical compression spring is required to provide a minimum force of 150 lb at installation and have a working deflection of 1 in. The spring rate is 75 lb/in. The coil must fit in a 2.1-in-dia hole with 0.1-in clearance. Use 0.250-in diameter, unpeened, music wire and squared/ground ends. Using a 15% clash allowance, find: (a) The stresses and safety factor for infinte life in fatigue. (b) The shut height. (c) The stress and safety factor at shut height. (d) The total number of coils. (e) The free length. (f) The natural frequency in Hz. (g) Draw a Goodman diagram and show the safety factor from (a) on it. Minimum force Fmin 150 lbf Spring rate k 75 lbf in
1

Given:

Spring OD OD 2.000 in Working deflection y 1.000 in Music wire properties: Strength A 184.65 ksi b 0.1625 Solution: 1. See Mathcad file P1431.

Shear modulus G 11.5 10 psi Clash factor 0.15 Wire diameter d 0.250 in Set after winding, unpeened

From the given information, find the maximum force and the spring index. Maximum force Spring index Fmax Fmin k y C OD d d Fmax 225 lbf C7

2.

Find the the mean and alternating forces from equation 14.15a: Alternating force Fa Fm Fmax Fmin 2 Fmax Fmin 2 Fa 37.5 lbf Fm 187.5 lbf

Mean force 3.

Calculate the factors necessary to find the fatigue factor of safety. Direct shear factor Wahl factor Yield strength factor Ultimate shear strength factor Torsional endurance limit Ks 1 Kw 0.5 C 0.615 C Ks 1.071 Kw 1.213 (Table 14-6, set removed) (equation 14.4) (unpeened)

4 C 1 4 C 4

Kys 0.60 KU 0.67 S ew 45 ksi

4.

Calculate the ultimate tensile strength, ultimate shear strength, and the shear endurance limit. Ultimate tensile strength Ultimate shear strength S ut A

in
d

S ut 231 ksi S us 155 ksi

S us KU S ut

MACHINE DESIGN - An Integrated Approach, 4th Ed.


Shear yield strength S ys Kys S ut S es 0.5 S ew S us S us 0.5 S ew S ys 139 ksi S es 26.3 ksi

14-31-2

Fatigue strength

5.

Calculate the mean coil diameter D from equation 14.5. Mean coil diameter D C d D 1.750 in

6.

Calculate the shear stress i at the initial deflection (lowest defined force), and the mean stress m. Stress at Fmin

i Ks

8 Fmin D

d
Stress at Fm

i 45.8 ksi

m Ks

8 Fm D

d
7. Calculate the alternating shear stress a in the coil. Alternating stress

m 57.3 ksi

a Kw

8 F a D

d
8. The safety factor is calculated from equation 14.16b. Fatigue factor of safety 9. Nfs

a 13.0 ksi

S es m i S us a

S es S us i

Nfs 1.2

To get the defined spring rate, the number of active coils must satisfy equation 14.7, solving for Na yields: Number of active coils Na d G 8 D k
3 4

Na 13.97

Na 14

Note that we round it to the nearest 1/4 coil as the manufacturing tolerance cannot achieve better than that accuracy. Having rounded the number of active coils, we must now calculate the spring rate using equation 14. Corrected spring rate d G 8 D Na
3 4

k 74.84

lbf in

10. For squared ends the total number of coils, from Figure 14-9: Total coils Nt Na 2 Nt 16.00

11. The shut height can now be determined. Shut height Lshut d Nt Fmin k Lshut 4 in

12. The initial deflection to reach the smaller of the two loads is Initial deflection 13. For the given clash allowance factor: Clash allowance yinit yinit 2.004 in

yclash y

yclash 0.150 in

14. The free length (see Figure 14-8) can now be found from

MACHINE DESIGN - An Integrated Approach, 4th Ed.


Lf Lshut yclash y yinit 15. The deflection to the shut height is yshut Lf Lshut 16. The force at this shut height is Fshut k yshut 17. The shut-height stress and safety factor are Stress at Fshut Fshut 236.1 lbf yshut 3.154 in Lf 7.154 in

14-31-3

shut Ks
S ys

8 Fshut D

d
Safety factor at shut height which is acceptable. 18. The inside and outside coil diameters are Inside coil dia Outside coil dia Di D d Do D d Ns

shut 72.1 ksi

shut

Ns 1.92

Di 1.5 in Do 2 in holemin 2.09 in pin max 1.41 in

19. The smallest hole and largest pin that should be used with this spring are Smallest hole Largest pin holemin Do 0.05 D pin max Di 0.05 D
3

20. The weight of the springs active coils is found from equation 14.11b Weight density

0.285 lbf in
2 2

Weight

Wa

d D Na
4

Wa 1.077 lbf

21. The natural frequency of this spring is found from equation 14.11a and is: Natural frequency fn 1 2 k g Wa fn 81.91 Hz

22. We now have results for questions (a) through (f): (a) Fatigue stresses and safety factor (b) Shut height (c) Stress at shut height and safety factor (d) Total coils (e) Free length (f) Natural frequency

i 45.8 ksi
Nfs 1.2 Lshut 4.000 in

a 13 ksi

m 57.3 ksi

shut 72.1 ksi


Ns 1.9 Nt 16.00 Lf 7.154 in fn 81.91 Hz ends squared and ground

MACHINE DESIGN - An Integrated Approach, 4th Ed.


23. (g) Plot the Goodman diagram. The equation for the Goodman line is

14-31-4

'a 'm aL 'm

S es S us

'm S es

Equation for load line

a m

'm

Plotting this over the range

'm 0 ksi 10 ksi 200 ksi

30 Alternating shear stress 25 20 15 10 5 0 0 25

m
ksi

a
ksi

50

75

100

125

150

175

200

Mean shear stress

The Goodman boundary is the solid line with negative slope. The stress state is at the intersection of the vertical and horizontal dashed lines. The safety margin is the distance along the load line from the stress state point to the intersection of the load line with the Goodman boundary.

MACHINE DESIGN - An Integrated Approach, 4th Ed.

14-32-1

PROBLEM 14-32
Statement: A helical compression spring is required to provide a time-varying force that ranges from a mimimum of 100 lb to a maximum of 300 lb over a deflection of 1 in. It needs to work freely over shaft of 1.25-in dia. Use a cold-drawn carbon steel wire having an S ut = 250 ksi. A spring index of 6, a clash allowance of 15%, and squared and ground ends are desired. Minimum force Maximum force Working deflection Wire strength Fmin 100 lbf Fmax 300 lbf y 1.000 in S ut 250 ksi Spring rate Shear modulus Clash factor Spring index Shaft OD k 75 lbf in
1

Given:

G 11.5 10 psi 0.15 C 6 ODshaft 1.250 in

Unset after winding, unpeened Solution: 1. See Mathcad file P1432.

From the given information, find a trial wire diameter. Maximum shaft dia Trial wire diameter Standard wire size ODshaft = IDspring 0.05 D = ( D d ) 0.05 D = C d d 0.05 C d d ODshaft 0.95 C 1 d 0.266 in (Found by iteration, see note in step 8)

d 0.343 in

2.

Find the the mean and alternating forces from equation 14.15a: Alternating force Fa Fm Fmax Fmin 2 Fmax Fmin 2 Fa 100.0 lbf Fm 200.0 lbf

Mean force 3.

Calculate the factors necessary to find the fatigue factor of safety. Direct shear factor Wahl factor Yield strength factor Ultimate shear strength factor Torsional endurance limit Ks 1 Kw 0.5 C 0.615 C Ks 1.083 Kw 1.252 (Table 14-6, set removed) (equation 14.4) (unpeened)

4 C 1 4 C 4

Kys 0.45 KU 0.67 S ew 45 ksi

4.

Calculate the ultimate tensile strength, ultimate shear strength, and the shear endurance limit. Ultimate tensile strength Ultimate shear strength Shear yield strength S us KU S ut S ys Kys S ut S es 0.5 S ew S us S us 0.5 S ew S ut 250 ksi S us 168 ksi S ys 113 ksi S es 26.0 ksi

Fatigue strength

MACHINE DESIGN - An Integrated Approach, 4th Ed.


5. Calculate the mean coil diameter D from equation 14.5. Mean coil diameter 6. D C d D 2.058 in

14-32-2

Calculate the shear stress i at the initial deflection (lowest defined force), and the mean stress m. Stress at Fmin

i Ks

8 Fmin D

d
Stress at Fm

i 14.1 ksi

m Ks

8 Fm D

d
7. Calculate the alternating shear stress a in the coil. Alternating stress

m 28.1 ksi

a Kw

8 F a D

d
8. The safety factor is calculated from equation 14.16b. Fatigue factor of safety Nfs

a 16.3 ksi

S es m i S us a

S es S us i

Nfs 1.3

This was originally too low. The wire diameter in step 1 was repeatedly changed to get a satisfactory safety factor. 9. The spring rate is defined in this problem because of the two specified forces at a particular relative deflection. Spring rate k Fmax Fmin k 200.00 lbf in

10. To get the defined spring rate, the number of active coils must satisfy equation 14.7, solving for Na yields: Number of active coils Na d G 8 D k
3 4

Na 11.413

Na 11.5

Note that we round it to the nearest 1/4 coil as the manufacturing tolerance cannot achieve better than that accuracy. Having rounded the number of active coils, we must now calculate the spring rate using equation 14. Corrected spring rate k d G 8 D Na
3 4

k 198.50

lbf in

11. For squared ends the total number of coils, from Figure 14-9: Total coils Nt Na 2 Nt 13.50

12. The shut height can now be determined. Shut height Lshut d Nt Lshut 4.631 in

13. The initial deflection to reach the smaller of the two loads is Initial deflection 14. For the given clash allowance factor: Clash allowance yinit Fmin k yinit 0.504 in

yclash y

yclash 0.150 in

MACHINE DESIGN - An Integrated Approach, 4th Ed.


15. The free length (see Figure 14-8) can now be found from Lf Lshut yclash y yinit 16. The deflection to the shut height is yshut Lf Lshut 17. The force at this shut height is Fshut k yshut 18. The shut-height stress and safety factor are Stress at Fshut Fshut 328.3 lbf yshut 1.654 in Lf 6.284 in

14-32-3

shut Ks
S ys

8 Fshut D

d
Safety factor at shut height which is acceptable. 19. The inside and outside coil diameters are Inside coil dia Outside coil dia Di D d Do D d Ns

shut 46.2 ksi

shut

Ns 2.4

Di 1.715 in Do 2.401 in holemin 2.50 in pin max 1.61 in

20. The smallest hole and largest pin that should be used with this spring are Smallest hole Largest pin holemin Do 0.05 D pin max Di 0.05 D
3

21. The weight of the springs active coils is found from equation 14.11b Weight density

0.285 lbf in
2 2

Weight

Wa

d D Na
4

Wa 1.958 lbf

22. The natural frequency of this spring is found from equation 14.11a and is: Natural frequency fn 1 2 k g Wa fn 98.92 Hz

23. We now have a complete design specification for this cold-drawn wire spring: Wire diameter Outside diameter Total coils Free length d 0.343 in Do 2.401 in Nt 13.50 Lf 6.284 in ends squared and ground

MACHINE DESIGN - An Integrated Approach, 4th Ed.

14-33-1

PROBLEM 14-33
Statement: Given:

_____

Find the ultimate tensile strength and the ultimate shear strength of ASTM A228 music wire in the following preferred diameters: 0.5 mm, 1.0 mm, 2.0 mm, 4.0 mm, and 6.0 mm. Wire diameters: i 1 2 5 d 0.5 mm d 1.0 mm
1 2

d 2.0 mm
3

d 4.0 mm
4

d 6.0 mm
5

Solution: 1.

See Mathcad file P1433.

Calculate the ultimate tensile strength using equation 14.3 and the coefficient and exponent from Table 14-4. For ASTM A228 music wire, A 2153.5 MPa d
i

b 0.1625

di S ut A i mm

mm

S ut

GPa

0.5 1.0 2.0 4.0 6.0

2.41 2.15 1.92 1.72 1.61

2.

Calculate the ultimate shear strength using equation 14.4. d S us 0.67 S ut


i i

mm

S us

GPa

0.5 1.0 2.0 4.0 6.0

1.61 1.44 1.29 1.15 1.08

MACHINE DESIGN - An Integrated Approach, 4th Ed.

14-34-1

PROBLEM 14-34
Statement: Given:

_____

Find the ultimate tensile strength and the ultimate shear strength of ASTM A228 music wire in the following preferred diameters: 0.020 in, 0.038 in, 0.081 in, 0.162 in, and 0.250 in. Wire diameters: i 1 2 5 d 0.020 in d 0.038 in
1 2

d 0.081 in
3

d 0.162 in
4

d 0.250 in
5

Solution: 1.

See Mathcad file P1434.

Calculate the ultimate tensile strength using equation 14.3 and the coefficient and exponent from Table 14-4. For ASTM A228 music wire, A 184.649 ksi d
i

b 0.1625

di S ut A i in

in

0.020 0.038 0.081 0.162 0.250

S ut

ksi

348.7 314.1 277.8 248.2 231.3

2.

Calculate the ultimate shear strength using equation 14.4. d S us 0.67 S ut


i i

in

0.020 0.038 0.081 0.162 0.250

S us

ksi

234 210 186 166 155

MACHINE DESIGN - An Integrated Approach, 4th Ed.

14-35-1

PROBLEM 14-35
Statement: Given: Solution: 1.

_____

Select the preferred diameter for an ASTM A227 cold-drawn wire that will have an ultimate tensile strength as close to, but not less than, 180 kpsi. Minimum tensile strength: S ut 180 ksi See Mathcad file P1435.

Calculate the maximum wire diameter using equation 14.3 and the coefficient and exponent from Table 14-4. For ASTM A227 cold-drawn wire,
1

A 141.04 ksi

b 0.1822

S ut d trial in A
2.

d trial 0.262 in

Choose the next lower diameter from Table 14-2 as it will have a higher ultimate strength than the next higher diameter. d 0.250 in

3.

Verify that the strength of a wire of this diameter is greater than the minimum required value. S ut A

in
d

S ut 182 ksi

MACHINE DESIGN - An Integrated Approach, 4th Ed.

14-36-1

PROBLEM 14-36
Statement: Given: Solution: 1.

_____

Select the preferred diameter for an ASTM A229 oil-tempered wire that will have an ultimate tensile strength as close to, but not less than, 1430 MPa. Minimum tensile strength: S ut 1430 MPa See Mathcad file P1436.

Calculate the maximum wire diameter using equation 14.3 and the coefficient and exponent from Table 14-4. For ASTM A229 oil-tempered wire, A 1831.2 MPa
1

b 0.1833

S ut d trial mm A
2.

d trial 3.85 mm

Choose the next lower diameter from Table 14-2 as it will have a higher ultimate strength than the next higher diameter. d 3.5 mm

3.

Verify that the strength of a wire of this diameter is greater than the minimum required value. S ut A

mm
d

S ut 1455 MPa

MACHINE DESIGN - An Integrated Approach, 4th Ed.

14-37-1

PROBLEM 14-37
Statement: Design a helical extension spring to handle a dynamic load that varies from 275 N to 325 N over 10 mm working deflection. Use chrome silicon wire and standard hooks. The forcing frequency is 800 rpm. Infinite life is desired. Minimize the package size. Choose appropriate safety factors against fatigue, yielding, and surging. Life Minimum force Fmin 275 N L Maximum force Working deflection Design Choices: Fatigue safety factor Wire endurance limit Fmax 325 N Shear modulus Forcing freq. Spring index Wire strength G 80.8 GPa

Given:

y 10.0 mm
Nfb 1.5 S ew 310 MPa

f 800 rpm
C 9 A 2059.2 MPa b 0.0934

Solution: 1.

See Mathcad file P1437. Fmax Fmin 2 Fmax Fmin 2

Find the mean and alternating loads. Fa Fm Fa 25.0 N Fm 300.0 N

2.

Calculate the hook bending factor. Kb 4 C C 1 4 C ( C 1)


2

Kb 1.090

3.

Solve for d by iteration using an equation derived from the safety factor equation for bending in the hooks.
1

4 4 K b C 1 d= Nfb Fm Nfb 1 Fmin A 0.577 0.67 A db 0.5 S ew Nfb Fa 0.5 0.67 S ew

2 b

RHS( d )

4 4 Kb C 1 Nfb Fm Nfb 1 Fmin A mm2 b 0.577 0.67 A d 0.5 S ew mm Nfb Fa 0.5 0.67 S ew
RHS( d ) 4.011 mm RHS( d ) 3.879 mm RHS( d ) 3.882 mm RHS( d ) 3.882 mm d 4.0 mm

2 b

mm

d 1.000 mm d RHS( d ) d RHS( d ) d RHS( d ) 4. Calculate the mean coil diameter. Mean coil diameter

D C d

D 36 mm

MACHINE DESIGN - An Integrated Approach, 4th Ed.


5. Use the assumed value of C to find an appropiate value of initial coil stress i from equations 14.21:

14-37-2

i1 4.231 C 181.5 C 3387 C 28640 psi i2

3 2 3 2 2.987 C 139.7 C 3427 C 38404 psi

i1 67.4 MPa i2 115.1 MPa i 91.3 MPa

i
6.

i1 i2
2

Find the direct shear factor Ks : Direct shear factor Ks 1 0.5 C Ks 1.056

7.

Use the value of i from (c) in equation 14.8 to find the corresponding initial coil-tension force Fi: Fi

d i
8 K s D

Fi 60.4 N

Check that this force is less than the required minimum applied force Fmin, which in this case, it is. Any applied force smaller than Fi will not deflect the spring. 8. Use the direct shear factor Ks and previously assumed values to find the mean stress m: Stress at Fm

m Ks

8 Fm D

d
9.

m 453.6 MPa

Find the Wahl factor Kw and use it to calculate the alternating shear stress a in the coil. Wahl factor Kw 4 C 1 4 C 4 0.615 C Kw 1.162

Alternating stress

a Kw

8 F a D

a 41.6 MPa

10. Find the ultimate tensile strength of this wire material from equation 14.3 and Table 14-4 and use it to find the ultimate shear strength from equation 14.4 and the torsional yield strength for the coil body from Table 14-10, assuming no set removal. Ultimate tensile strength Shear yield strength Ultimate shear strength S ut A

mm
d

S ut 1809.1 MPa S ys 814.1 MPa S us 1212.1 MPa

S ys 0.45 S ut S us 0.67 S ut

11. Find the wire endurance limit for unpeened springs from equation 14.13 and convert it to fully reversed endurance strength with equation 14.16c. Fully reversed endurance limit S es 0.5 S ew S us S us 0.5 S ew S es 177.73 MPa

12. The fatigue safety factor for the coils in torsion is calculated from equation 14.16b. Minimum stress

min m a

min 412.0 MPa

MACHINE DESIGN - An Integrated Approach, 4th Ed.


Nfs S es S us min

14-37-3
Nfs 2.5

Fatigue safety factor

S es m min S us a

Note that the minimum stress due to force Fmin is used in this calculation, not the coil-winding stress from step 5. 13. The stresses in the end hooks also need to be determined. The bending stresses in the hook are found from equation 14.23: C1 = 2 R1 d =
2

2 D 2 d

=C

C1 C

C1 9.00

Kb

4 C1 C1 1 4 C1 C1 1 16 D Fa

Kb 1.09

a Kb

d m Kb

4 Fa

a 80.07 MPa

16 D Fm

4 Fm

d min Kb

m 960.9 MPa

16 D Fmin

4 Fmin

min 880.82 MPa

14. Convert the torsional endurance strength to a tensile endurance strength with the von Mises relationship and use it and the ultimate tensile strength in equation 14.16 to find a fatigue safety factor for the hook in bending: S e Nfb S es 0.577 S e m min S ut a S e S ut min S e 308.02 MPa Nfb 1.7

15. The torsional stresses in the hook are found from equation 14.24 using an assumed value of C2 5. R2 C2 d 2 4 C2 1 4 C2 4 8 Fa D R2 10 mm Kw2 1.188

Kw2

Ba Kw2

d Bm Kw2

Ba 42.5 MPa

8 Fm D

d Bmin Kw2

Bm 510.3 MPa

8 Fmin D

Bmin 467.8 MPa

MACHINE DESIGN - An Integrated Approach, 4th Ed.


16. The fatigue safety factor for the hook in torsion is calculated from equation 14.16b. Nfs S es Bm Bmin S us Ba S es S us Bmin Nfs 2.2

14-37-4

17. The spring rate is defined from the two specified forces at their relative deflection. Spring rate k Fmax Fmin k 5000.0 N m

18. To get the defined spring rate, the number of active coils must satisfy equation 14.7, solving for Na yields: Number of active coils Na d G 8 D k Note that we round it to the nearest 1/4 coil as the manufacturing tolerance cannot achieve better than that accuracy. Having rounded the number of active coils, we must now calculate the spring rate using equation 14. Corrected spring rate k d G 8 D Na 19. The total number of coils in the body and the body length are Total coils Body length Nt Na 1 Lb Nt d Nt 12.00 Lb 48.0 mm
3 4 3 4

Na 11.084

Na 11.0

k 5038.0

N m

20. The free length can now be determined. The length of a standard hook is equal to the coil inside diameter: Hook length Free length Lhook D d Lf Lb 2 Lhook Lhook 32.0 mm Lf 112.0 mm

21. The initial coil tension force must be found again in order to obtain the deflection to reach the larger of the two loads.

i1 4.231 C 181.5 C 3387 C 28640 psi i2 i1 i2


2

3 2 3 2 2.987 C 139.7 C 3427 C 38404 psi

i1 67.4 MPa i2 115.1 MPa i 91.3 MPa


Fi 60.4 N

i
Fi

d i
8 K s D Fmax Fi k Di D d Do D d
3

ymax

ymax 52.53 mm

22. The inside and outside coil diameters are Inside coil dia Outside coil dia Di 32.00 mm Do 40.00 mm

23. The weight of the spring's active coils is found from equation 14.11b and is Weight density

0.28 lbf in

MACHINE DESIGN - An Integrated Approach, 4th Ed.


Wa

14-37-5
Wa 1.188 N

Weight

d D Na
4

2 2

24. The natural frequency of this spring is found from equation 14.11a and is: Natural frequency fn 1 2 k g Wa fn 102 Hz

25. The ratio between the natural frequency and the forcing frequency is Forcing frequency ff fn ff

f
2

ff 13.333 Hz

Frequency ratio

7.6

which could be higher.

26. We now have a complete design specification for this A228-wire spring: Wire diameter Outside diameter d 4.00 mm Do 40.00 mm Total coils Free length Standard hooks Nt 12.00 Lf 112.00 mm

MACHINE DESIGN - An Integrated Approach, 4th Ed.

14-38-1

PROBLEM 14-38
Statement: Design a helical extension spring with standard hooks to handle a dynamic load that varies from 60 lb to 75 lb over 0.5 in working deflection. Use music wire. The forcing frequency is 1200 rpm. Infinite life is desired. Minimize the package size. Choose appropriate safety factors against fatigue, yielding, and surging. Minimum force Maximum force Working deflection Design Choices: Fatigue safety factor Wire endurance limit Solution: 1. See Mathcad file P1438. Fmax Fmin 2 Fmax Fmin 2 Fmin 60 lbf Fmax 75 lbf y 0.50 in Nfb 1.5 S ew 45 ksi Life Shear modulus Forcing freq. L G 11.7 10 psi f 1200 rpm C 9 A 184.649 ksi b 0.1625 Find the mean and alternating loads. Fa Fm 2. Fa 7.5 lbf Fm 67.5 lbf
6

Given:

Spring index Wire strength

Calculate the hook bending factor. Kb 4 C C 1 4 C ( C 1)


2

Kb 1.090

3.

Solve for d by iteration using an equation derived from the safety factor equation for bending in the hooks.
1

d=

4 4 K b C 1 Nfb Fm Nfb 1 Fmin A 0.577 0.67 A db 0.5 S ew Nfb Fa 0.5 0.67 S ew

2 b

4 4 Kb C 1 RHS( d ) Nfb Fm Nfb 1 Fmin A in2 b 0.577 0.67 A d 0.5 S ew in Nfb Fa 0.5 0.67 S ew
d 1.00 in d RHS( d ) d RHS( d ) d RHS( d ) 4. Calculate the mean coil diameter. Mean coil diameter D C d RHS( d ) 0.153 in RHS( d ) 0.167 in RHS( d ) 0.166 in RHS( d ) 0.166 in

2 b

in

d 0.177 in D 1.593 in

MACHINE DESIGN - An Integrated Approach, 4th Ed.


5. Use the assumed value of C to find an appropiate value of initial coil stress i from equations 14.21:
3 2 3 2 2.987 C 139.7 C 3427 C 38404 psi

14-38-2

i1 4.231 C 181.5 C 3387 C 28640 psi i2

i1 9.77 ksi i2 16.70 ksi i 13.2 ksi

i
6.

i1 i2
2

Find the direct shear factor Ks : Direct shear factor Ks 1 0.5 C Ks 1.056

7.

Use the value of i from (c) in equation 14.8 to find the corresponding initial coil-tension force Fi: Fi

d i
8 K s D

Fi 17.1 lbf

Check that this force is less than the required minimum applied force Fmin, which in this case, it is. Any applied force smaller than Fi will not deflect the spring. 8. Use the direct shear factor Ks and previously assumed values to find the mean stress m: Stress at Fm

m Ks

8 Fm D

d
9.

m 52.1 ksi

Find the Wahl factor Kw and use it to calculate the alternating shear stress a in the coil. Wahl factor Kw 4 C 1 4 C 4 0.615 C Kw 1.162

Alternating stress

a Kw

8 F a D

a 6.4 ksi

10. Find the ultimate tensile strength of this wire material from equation 14.3 and Table 14-4 and use it to find the ultimate shear strength from equation 14.4 and the torsional yield strength for the coil body from Table 14-10, assuming no set removal. Ultimate tensile strength Shear yield strength Ultimate shear strength S ut A

in
d

S ut 244.7 ksi S ys 122.3 ksi S us 163.9 ksi

S ys 0.50 S ut S us 0.67 S ut

11. Find the wire endurance limit for unpeened springs from equation 14.13 and convert it to fully reversed endurance strength with equation 14.16c. S ew S us Fully reversed S es 0.5 S es 26.08 ksi S us 0.5 S ew endurance limit 12. The fatigue safety factor for the coils in torsion is calculated from equation 14.16b. Minimum stress Fatigue safety factor

min m a
Nfs S es m min S us a S es S us min

min 45.75 ksi


Nfs 2.5

MACHINE DESIGN - An Integrated Approach, 4th Ed.

14-38-3

Note that the minimum stress due to force Fmin is used in this calculation, not the coil-winding stress from step 5. 13. The stresses in the end hooks also need to be determined. The bending stresses in the hook are found from equation 14.23: C1 = 2 R1 d =
2

2 D 2 d

=C

C1 C

C1 9.00

Kb

4 C1 C1 1 4 C1 C1 1 16 D Fa

Kb 1.09

a Kb

d m Kb

4 Fa

a 12.27 ksi

16 D Fm

4 Fm

d min Kb

m 110.4 ksi

16 D Fmin

4 Fmin

min 98.15 ksi

14. Convert the torsional endurance strength to a tensile endurance strength with the von Mises relationship and use it and the ultimate tensile strength in equation 14.16 to find a fatigue safety factor for the hook in bending: S e Nfb S es 0.577 S e m min S ut a C2 d 2 4 C2 1 4 C2 4 8 Fa D S e S ut min S e 45.20 ksi Nfb 1.9

15. The torsional stresses in the hook are found from equation 14.24 using an assumed value of C2 5. R2 R2 0.442 in Kw2 1.188

Kw2

Ba Kw2

d Bm Kw2

Ba 6.5 ksi

8 Fm D

d Bmin Kw2

Bm 58.6 ksi

8 Fmin D

Bmin 52.1 ksi

16. The fatigue safety factor for the hook in torsion is calculated from equation 14.16b. Nfs S es Bm Bmin S us Ba S es S us Bmin Nfs 2.4

MACHINE DESIGN - An Integrated Approach, 4th Ed.


17. The spring rate is defined from the two specified forces at their relative deflection. Spring rate k Fmax Fmin k 30.0 lbf in

14-38-4

18. To get the defined spring rate, the number of active coils must satisfy equation 14.7, solving for Na yields: Number of active coils Na d G 8 D k Note that we round it to the nearest 1/4 coil as the manufacturing tolerance cannot achieve better than that accuracy. Having rounded the number of active coils, we must now calculate the spring rate using equation 14. Corrected spring rate k d G 8 D Na 19. The total number of coils in the body and the body length are Total coils Body length Nt Na 1 Lb Nt d Nt 12.75 Lb 2.3 in
3 4 3 4

Na 11.836

Na 11.75

k 30.2

lbf in

20. The free length can now be determined. The length of a standard hook is equal to the coil inside diameter: Hook length Free length Lhook D d Lf Lb 2 Lhook Lhook 1.42 in Lf 5.09 in

21. The initial coil tension force must be found again in order to obtain the deflection to reach the larger of the two loads.

i1 4.231 C 181.5 C 3387 C 28640 psi i2 i1 i2


2

3 2 3 2 2.987 C 139.7 C 3427 C 38404 psi

i1 9.8 ksi i2 16.7 ksi i 13.2 ksi


Fi 17.1 lbf ymax 1.91 in

i
Fi

d i
8 K s D Fmax Fi k Di D d Do D d

ymax

22. The inside and outside coil diameters are Inside coil dia Outside coil dia Di 1.416 in Do 1.770 in
3

23. The weight of the spring's active coils is found from equation 14.11b and is Weight density Weight

0.28 lbf in
2 2

Wa

d D Na
4

Wa 0.405 lbf

24. The natural frequency of this spring is found from equation 14.11a and is: Natural frequency fn 1 2 k g Wa fn 84.9 Hz

MACHINE DESIGN - An Integrated Approach, 4th Ed.


25. The ratio between the natural frequency and the forcing frequency is Forcing frequency ff fn ff

14-38-5

f
2

ff 20 Hz

Frequency ratio

4.2

which could be higher.

26. We now have a complete design specification for this A228-wire spring: Wire diameter Outside diameter d 0.177 in Do 1.770 in Total coils Free length Standard hooks Nt 12.75 Lf 5.089 in

MACHINE DESIGN - An Integrated Approach, 4th Ed.

14-39-1

PROBLEM 14-39
Statement: Design a straight-ended helical torsion spring for a static load 50 N-m at a deflection of 60 deg with a safety factor of 2. Specify all parameters necessary to manufacture the spring. State all assumptions. Bending modulus Applied moment M 50 N m E 206.8 GPa Deflection at load 60 deg 0.167 rev

Given:

Design Choices: Use unpeened oil tempered wire with 40-mm-long, straight ends. The coil is loaded to close it. Design safety factor Nyd 2 Spring index C 10 Yield strength factor Length of ends Material strength factors Solution: 1. See Mathcad file P1439. Ks 0.85 (from Table 14-13) L1 40 mm L2 40 mm A 1831.2 MPa b 0.1833 (ASTM A229 wire)

Calculate the Wahl bending factors for inside surface. Kbi 4 C C 1 4 C ( C 1 )


2

Kbi 1.081

2.

Solve for d using the static yield criterion.


1

32 Kbi Nyd M d Ks A mm3


3.

3 b

mm

d 10.272 mm

Use a wire diameter from the available sizes in Table 14-2. Calculate the mean coil diameter D from equation 14.5. Wire diameter Spring index Mean coil diameter d 11 mm C 10 D C d D 110 mm

4.

Calculate the maximum compressive stress in the coil at the inner surface.

imax Kbi
5.

32 M

imax 413.5 MPa

Find the ultimate tensile strength of this oil tempered material from equation 14.3 and Table 14-4 and use it to find the bending yield strength from Table 14-13, assuming stress relieving. Ultimate tensile strength Bending yield strength S ut A

mm
d

S ut 1180 MPa S y 1003 MPa Nyb Sy Nyb 2.4

S y Ks S ut

6. 7.

The realized static safety factor against yielding is

imax

The spring rate is defined from the two specified moments at their relative deflection. Spring rate k M

k 300

N m rev

8.

To get the defined spring rate, the number of active coils must satisfy equation 14.28, solving for Na yields:

MACHINE DESIGN - An Integrated Approach, 4th Ed.


Number of active coils Na d E 10.8 D k rev
4

14-39-2
Na 8.50

Note that to force k to be in units of N-m per rev we must multiply k by rev. 9. The ends contribute to the active coils from equation 14.26a as Ne L1 L2 3 D Ne 0.08

and, from equation 14.26b, the number of body coils in the spring are Nb Na Ne 10. Check the angular deflection at the specified load from equation 14.27c.

Nb 8.4

10.8

M D Na d E
4

rev

60.0 deg

MACHINE DESIGN - An Integrated Approach, 4th Ed.

14-40-1

PROBLEM 14-40
Statement: Design a straight-ended helical torsion spring for a static load 430 in-lb at a deflection of 55 deg with a safety factor of 2. Specify all parameters necessary to manufacture the spring. State all assumptions. Applied moment Deflection at load M 430 in lbf 55 deg Bending modulus E 30 10 psi 0.153 rev
6

Given:

Design Choices: Use unpeened music wire with 2-in-long, straight ends. The coil is loaded to close it. Spring index Design safety factor Nyd 2 C 9 Yield strength factor Length of ends Material strength factors Solution: 1. See Mathcad file P1440. Ks 1 (from Table 14-13) L1 2 in A 184.65 ksi L2 2 in b 0.1625 (ASTM A228 wire)

Calculate the Wahl bending factors for inside surface. Kbi 4 C C 1 4 C ( C 1 )


2

Kbi 1.090

2.

Solve for d using the static yield criterion.


1

32 Kbi Nyd M d Ks A in3


3.

3 b

in

d 0.352 in

Use a wire diameter from the available sizes in Table 14-2. Calculate the mean coil diameter D from equation 14.5. Wire diameter Spring index Mean coil diameter d 0.362 in C9 D C d D 3.258 in

4.

Calculate the maximum compressive stress in the coil at the inner surface.

imax Kbi
5.

32 M

imax 100.7 ksi

Find the ultimate tensile strength of this oil tempered material from equation 14.3 and Table 14-4 and use it to find the bending yield strength from Table 14-13, assuming stress relieving. Ultimate tensile strength Bending yield strength d S ut A in S y Ks S ut Nyb
b

S ut 218 ksi S y 218 ksi Sy Nyb 2.2

6. 7.

The realized static safety factor against yielding is

imax

The spring rate is defined from the two specified moments at their relative deflection. Spring rate k M

k 2815

in lbf rev

8.

To get the defined spring rate, the number of active coils must satisfy equation 14.28, solving for Na yields:

MACHINE DESIGN - An Integrated Approach, 4th Ed.


Number of active coils d E 10.8 D k rev
4

14-40-2

Na

Na 5.20

Note that to force k to be in units of N-m per rev we must multiply k by rev. 9. The ends contribute to the active coils from equation 14.26a as Ne L1 L2 3 D Ne 0.13 Nb 5.1

and, from equation 14.26b, the number of body coils in the spring are Nb Na Ne 10. Check the angular deflection at the specified load from equation 14.27c.

10.8

M D Na d E
4

rev

55.0 deg

MACHINE DESIGN - An Integrated Approach, 4th Ed.

14-41-1

PROBLEM 14-41
Statement: Given the following data for a helical torsion spring, loaded in fatigue, find the spring index, unloaded coil diameter, minimum loaded coil diameter, and safety factor in fatigue. State all assumptions and sources of empirical data used. Deflection at assembly = 0.15 rev, working deflection = 0.35 rev, k = 10 N-m/rev, Na = 25, 4.50 mm oil-tempered wire, unpeened. Minimum deflection Maximum deflection Number active coils

Given:

min 0.15 rev max 0.50 rev


Na 25

Bending modulus Spring rate Wire diameter

E 206.8 GPa k 10 N m rev d 4.50 mm


1

Design Choices: Use unpeened oil-tempered wire with 40-mm-long, straight ends. The coil is loaded to close it. Yield strength factor Ks 0.85 (from Table 14-13) Length of ends Material strength factors Wire endurance limit Solution: 1. See Mathcad file P1441. L1 40 mm L2 40 mm A 1831.2 MPa b 0.1833 (ASTM A229 wire) S ewb 537 MPa

Find the minimum, maximum, mean, and alternating loads. Mmin k min Mmax k max Ma Mm Mmax Mmin 2 Mmax Mmin 2 Mmin 1.50 N m Mmax 5.00 N m Ma 1.75 N m Mm 3.25 N m k= d E 10.8 C Na
3

2.

Use equation 14.28 to calculate the spring index. d E 10.8 k rev Na


3

C 3.

C 6.98

Calculate the Wahl bending factors for inside and outside surfaces. Kbi Kbo 4 C C 1 4 C ( C 1 ) 4 C C 1 4 C ( C 1 )
2 2

Kbi 1.119 Kbo 0.902

4.

Calculate the mean coil diameter D from equation 14.5. Mean coil diameter D C d D 31.41 mm

5.

Calculate the maximum compressive stress in the coil at the inner surface.

imax Kbi
6.

32 Mmax

imax 625.7 MPa

Calculate the maximum, minimum, alternating, and mean tensile stresses in the coil at the outer surface.

MACHINE DESIGN - An Integrated Approach, 4th Ed.


32 Mmin

14-41-2

omin Kbo

d omax Kbo

omin 151.2 MPa

32 Mmax

d m

omax 503.9 MPa

omax omin
2

m 327.5 MPa

a
7.

omax omin
2

a 176.4 MPa

Find the ultimate tensile strength of this oil tempered material from equation 14.3 and Table 14-4 and use it to find the bending yield strength from Table 14-13, assuming stress relieving. Ultimate tensile strength S ut A

mm
d

S ut 1390 MPa S y 1181 MPa

Bending yield strength 8.

S y Ks S ut

Convert the wire bending endurance limit for unpeened springs from equation 14.33 to fully reversed endurance strength with equation 14.34b. Fully reversed endurance limit S e 0.5 S ewb S ut S ut 0.5 S ewb S e 332.78 MPa

9.

The fatigue safety factor for the coils in bending is calculated from equation 14.34a. Fatigue factor of safety S e S ut omin

Nfb

S e m omin S ut a L1 L2 3 D

Nfb 1.4

10. The ends contribute to the active coils from equation 14.26a as

Ne

Ne 0.27 Nb 25

and, from equation 14.26b, the number of body coils in the spring are Na Ne Nb 11. The unloaded coil diameter and minimum loaded coil diameters are: Unloaded coil diameter Di D d Dimin D Nb Nb d Di 26.9 mm

Loaded coil diameter

max
rev

Dimin 26.3 mm

MACHINE DESIGN - An Integrated Approach, 4th Ed.

14-42-1

PROBLEM 14-42
Statement: Given: Design a Belleville spring to give a static force of approximately 2000 lb at a maximum deflection of 0.05 in with a nearly constant spring rate. Maximum force Fflat 2000 lbf Maximum deflection ymax 0.05 in
6

Assumptions: The diameter ratio is Rd 2. Use unset carbon spring steel 50HRC. Properties, E 30 10 psi ,

0.28 .
Solution: 1. See Mathcad file P1442.

Since a constant rate spring is needed, the h/t ratio (see Figure 14-30) is h/t ratio hovert 0.40

2.

Use the above choice and the specified maximum deflection to find the spring thickness t: h ymax t h hovert t 0.125 in

3.

Use equation 14.37 to find the force at maximum deflection. This must be done by iterating on the outside diameter, Do. Guess: Do 3.0 in 19.2 10 psi hovert t Do
2 7 4

Fflat

Fflat 2083.33 lbf

4.

Figure 14-34 shows that the worst stress state will occur at the largest deflection ymax, so solve equations 14.36 for stresses at that deflection:

Rd 1 2 K1 ln Rd 2 Rd
6 K2

K1 0.689

ln Rd ln Rd ln Rd
6

Rd 1

K2 1.220

K3

Rd 1 2

K3 1.378

K4

Rd ln Rd Rd 1 Rd ln Rd Rd 1 2
2 Rd 1 4 E ymax K1 Do
2

K4 1.115

K5

Rd

K5 1

2 1

K2 h

ymax

K3 t 2

c 213 ksi

ti

y K2 h max K3 t 2 K1 Do2 1 2 4 E ymax

ti 148.8 ksi

MACHINE DESIGN - An Integrated Approach, 4th Ed.

14-42-2

to

4 E ymax K1 Do2 1 2

y K4 h max K5 t 2

to 161 ksi

5.

Table 14-5 gives S ut 246 ksi for this material. Table 14-15 indicates that 120% of this value can be used for an unset spring. The safety factor for static loading is then Ns 1.20 S ut Ns 1.4

which is acceptable. 6. A summary of the spring design is Outside diameter Inside diameter Thickness Height Do 3.000 in Di Do Rd Di 1.500 in

t 0.125 in h 0.050 in

MACHINE DESIGN - An Integrated Approach, 4th Ed.

14-43-1

PROBLEM 14-43
Statement: Given: Design a Belleville spring to give a static force of 400 lb at 50% deflection to flat and 200 lb at fla Force at flat Fflat 200 lbf
6

Assumptions: The diameter ratio is Rd 2. Use unset carbon spring steel 50HRC. Properties, E 30 10 psi ,

0.28 . The outside diameter of the spring is Do 4.00 in


Solution: 1. 2. See Mathcad file P1443.

Since a spring with 200% force to flat at 50% deflection to flat is needed, the h/t ratio (see Figure 14-30) is h/t ratio hovert 2.828 Use the above assumptions and the specified nominal force in equation 14.37a to find an appropriate spring thickness t:
1 2 Do Fflat t 7 hovert 19.2 10 psi 4

t 0.049 in h 0.139 in

3. 4.

The height h can now be found:

h hovert t

Figure 14-34 shows that the worst stress state will occur at the largest deflection ymax, so solve equations 14.36 for stresses at that deflection: ymax h

Rd 1 2 K1 ln Rd 2 Rd
6 K2

K1 0.689

ln Rd ln Rd ln Rd
6

Rd 1 Rd 2

K2 1.220

K3

K3 1.378

K4

Rd ln Rd Rd 1 Rd ln Rd Rd 1 2
2 Rd 1 4 E ymax K1 Do
2 2 1

K4 1.115

K5

Rd

K5 1

K2 h

ymax

K3 t 2

c 252 ksi

ti

4 E ymax K1 Do2 1 2

y K2 h max K3 t 2

ti 28.2 ksi

to

y K4 h max K5 t 2 K1 Do2 1 2 4 E ymax

to 209 ksi

MACHINE DESIGN - An Integrated Approach, 4th Ed.


5.

14-43-2

Table 14-5 gives S ut 246 ksi for this material. Table 14-15 indicates that 120% of this value can be used for an unset spring. The safety factor for static loading is then Ns 1.20 S ut Ns 1.2

which is acceptable. 6. A summary of the spring design is Outside diameter Inside diameter Thickness Height Do 4.000 in Di Do Rd Di 2.000 in

t 0.049 in h 0.139 in

MACHINE DESIGN - An Integrated Approach, 4th Ed.

14-44-1

PROBLEM 14-44
Statement: Design a helical compression spring for a static load of 60 lb at a deflection of 1.50 in with a safety factor of 2.0 to work in a 1.06-in hole. Specify all parameters necessary to manufacture the spring. Working force Working deflection Fwork 60 lbf ywork 1.50 in Shear modulus Safety factor G 11.5 10 psi Ns 2.0
6

Given:

Design choices: Clash allowance ASTM A228 wire Solution: 1. See Mathcad file P1444. k Fwork ywork k 40 lbf in

0.15
A 184.649 ksi b 0.1625

Set removed

Km 0.65

Spring index C 7.0 (found by trial-and-error)

Determine the desired spring rate.

2.

Use the design equation from Example 14-3A (Mathcad Supplement) to determine the wire diameter.
1

8 Ns ( C 0.5) Fwork ( 1 ) 2 Km A in
d 0.125 in

2 b

in

Wire diameter 3.

Let

d 0.125 in

Calculate the mean and outside coil diameters and number of active coils. Mean coil dia Outside coil dia Number of active coils D C d Do D d Na d G 8 D k
3 4

D 0.875 in Do 1.000 in Na 13.097 Na 13.0

Note that we round it to the nearest 1/4 coil as the manufacturing tolerance cannot achieve better than that accuracy. We must now calculate the actual (corrected) spring rate: Corrected spring rate 4. k d G 8 D Na
3 4

k 40.298

lbf in

Assume squared and ground ends making the total number of coils, from Figure 14-9: Total coils Nt Na 2 Nt 15.00

5.

The shut height can now be determined. Shut height Ls d Nt Ls 1.875 in

6.

The free length (see Figure 14-8) can now be found from Deflection to shut height Free length yshut ywork ywork Lf Ls yshut yshut 1.725 in Lf 3.600 in

MACHINE DESIGN - An Integrated Approach, 4th Ed.


7. To check for buckling, two ratios need to be calculated, Lf/D and y max/Lf. Slenderness ratio sr y' Lf D ywork Lf sr 4.114 y' 0.417

14-44-2

Deflection ratio

Take these two values to Figure 14-14 and find that their coordinates are safely within the zones that are stable against buckling for either end-condition case. 8. The inside and outside coil diameters are Inside coil dia Outside coil dia 9. Di D d Do D d holemin Do 0.05 D pin max Di 0.05 D
3

Di 0.750 in Do 1.000 in holemin 1.04 in pin max 0.71 in

The smallest hole and largest pin that should be used with this spring are Smallest hole Largest pin

10. The total weight of the spring is Weight density

0.28 lbf in
2 2

Weight

Wt

d D Nt
4

Wt 0.14 lbf

11. We now have a complete design specification for this A228 wire spring: Wire diameter Outside diameter Total coils Free length d 0.125 in Do 1.000 in Nt 15.00 Lf 3.600 in ends squared and ground

MACHINE DESIGN - An Integrated Approach, 4th Ed.

14-45-1

PROBLEM 14-45
Statement: Design a helical compression spring for a static load of 200 N at a deflection of 40 mm with a safety factor of 1.8 to work in a 25-mm hole. Specify all parameters necessary to manufacture the spring. Working force Fwork 200 N ywork 40 mm Shear modulus Safety factor Set removed G 80.8 GPa Ns 1.8 Km 0.65

Given:

Working deflection Design choices: Clash allowance ASTM A228 wire Solution: 1. 2. See Mathcad file P1445.

0.15
A 2153.5 MPa b 0.1625

Spring index C 7.5 (found by trial-and-error)

Determine the desired spring rate.

Fwork ywork

k 5.000

N mm

Use the design equation from Example 14-3A (Mathcad Supplement) to determine the wire diameter.
1 2 b 8 Ns ( C 0.5) Fwork ( 1 ) d mm 2 Km A mm

Wire diameter 3.

d 2.658 mm

Let

d 2.8 mm

Calculate the mean and outside coil diameters and number of active coils. Mean coil dia Outside coil dia Number of active coils D C d Do D d Na d G 8 D k
3 4

D 21.000 mm Do 23.800 mm Na 13.407 Na 13.5

Note that we round it to the nearest 1/4 coil as the manufacturing tolerance cannot achieve better than that accuracy. We must now calculate the actual (corrected) spring rate: Corrected spring rate 4. k d G 8 D Na
3 4

k 4.965

N mm

Assume squared and ground ends making the total number of coils, from Figure 14-9: Total coils Nt Na 2 Nt 15.50

5.

The shut height can now be determined. Shut height Ls d Nt Ls 43.4 mm

6.

The free length (see Figure 14-8) can now be found from Deflection to shut height Free length yshut ywork ywork Lf Ls yshut yshut 46.0 mm Lf 89.4 mm

7.

To check for buckling, two ratios need to be calculated, Lf/D and y max/Lf.

MACHINE DESIGN - An Integrated Approach, 4th Ed.


Slenderness ratio sr y' Lf D ywork Lf sr 4.257 y' 0.447

14-45-2

Deflection ratio

Take these two values to Figure 14-14 and find that their coordinates are safely within the zones that are stable against buckling for either end-condition case. 8. The inside and outside coil diameters are Inside coil dia Outside coil dia 9. Di D d Do D d holemin Do 0.05 D pin max Di 0.05 D
3

Di 18.2 mm Do 23.8 mm holemin 24.85 mm pin max 17.15 mm

The smallest hole and largest pin that should be used with this spring are Smallest hole Largest pin

10. The total weight of the spring is Weight density

0.28 lbf in
2 2

Weight

Wt

d D Nt
4

Wt 0.48 N

11. We now have a complete design specification for this A228 wire spring: Wire diameter Outside diameter Total coils Free length d 2.8 mm Do 23.8 mm Nt 15.50 Lf 89.4 mm ends squared and ground

MACHINE DESIGN - An Integrated Approach, 4th Ed.

14-46-1

PROBLEM 14-46
Statement: Three springs are arranged in series similar to the configuration shown in Figure 3-1(a). They have spring rates k1 = 50 N/mm, k2 = 150 N/mm, and k3 = 500 N/mm, respectively. Determine the total spring rate, the deflection of each spring, and the overall deflection if a load of F = 600 N is applied. Spring rates: Load: Solution: 1. k1 50 N k2 150 N mm k3 500 N mm

Given:

mm F 600 N

See Figure 14-1 and Mathcad file P1446.

Using equation 14.1, calculate the deflection of each spring. y1 y2 F k1 F k2 F k3 y1 12 mm y2 4 mm

y3 2.

y3 1.2 mm

The sum of the three is the overall deflection. ytotal y1 y2 y3 ytotal 17.2 mm

3.

Alternatively, use equation 14.2b to calculate the total spring rate and use it to determine the total deflection. ktotal ytotal k1 k2 k3 k2 k3 k1 k3 k1 k2 F ktotal ktotal 34.884 N mm

ytotal 17.2 mm

MACHINE DESIGN - An Integrated Approach, 4th Ed.

14-47-1

PROBLEM 14-47
Statement: Three springs are arranged in parallel similar to the configuration shown in Figure 3-1(b). They have spring rates k1 = 50 N/mm, k2 = 150 N/mm, and k3 = 500 N/mm, respectively. Determine the total spring rate, the force carried by each spring, and the overall deflection if a load of F = 600 N is applied. Spring rates: Load: Solution: 1. k1 50 N k2 150 N mm k3 500 N mm

Given:

mm F 600 N

See Figure 14-1 and Mathcad file P1447.

Using equation 14.2a, calculate the total spring rate. ktotal k1 k2 k3 ktotal 700 N mm

2.

Calculate the deflection, which is the same for each spring (assuming that the plate that carries the load does not rotate). y F ktotal y 0.857 mm

3.

Calculate the force carried by each of the three springs using equation 14.1 F1 y k1 F2 y k2 F3 y k3 F1 42.857 N F2 128.571 N F3 428.571 N

4.

The sum of the three is the total load carried by the plate. Ftotal F1 F2 F3 Ftotal 600 N

MACHINE DESIGN - An Integrated Approach, 4th Ed.

14-48-1

PROBLEM 14-48
Statement: A spring made from ASTM A228 wire with ends squared and ground, wire diameter d = 3 mm, outside diameter Do = 27 mm, 14 total coils, and free length Lf = 80 mm has been chosen for an application. Determine the static safety factor if the spring is subjected to a static load of 175 N. Working force Wire diameter Outside diameter Free length Fwork 175 N d 3 mm Do 27 mm Lf 80 mm Shear modulus Total coils G 80 GPa Nt 14 b 0.1625

Given:

ASTM A228 wire A 2153.5 MPa

Ends squared and ground Ne 2 Assumption: Solution: 1. Set removed See Mathcad file P1448.

Determine the mean diameter, spring index, shear factor, number of active coils, and spring rate. Mean diameter Spring index Shear factor Number of active coils Spring rate D Do d C D d 0.5 C D 24 mm C8 Na Ks 1.063 Na 12 k 4.883 N mm

Ks 1

Na Nt Ne k d G 8 D Na
3 4

2.

Calculate the ultimate tensile strength of the wire material from equation 14.3 and Table 14-4 and use it to find the torsional yield strength from Table 14-6. Ultimate tensile strength Shear yield strength d S ut A mm S ys 0.60 S ut
b

S ut 1801 MPa S ys 1081 MPa

3.

Calculate the shear stress in the coil at the working load and the factor of safety against yielding under the working load. Working shear stress

work Ks
Nswork

8 Fwork D

d
Factor of safety 4. S ys

work 420.9 MPa


Nswork 2.6

work

Calculate the shear stress in the coil at shut height and the factor of safety against yielding at shut height. Shut height Ls d Nt Fs k yshut Ls 42 mm yshut 38 mm Fs 185.547 N

Deflection to shut height yshut Lf Ls Force at shut height Shut height stress

s Ks
Nss

8 Fs D

d
Factor of safety S ys

s 446.2 MPa
Nss 2.4

MACHINE DESIGN - An Integrated Approach, 4th Ed.

14-49-1

PROBLEM 14-49
Statement: A spring with ends squared and ground, wire diameter d = 4 mm, outside diameter Do = 40 mm, 18 total coils, and free length Lf = 140 mm has been chosen for an application where the initial deflection is 15 mm and the working deflection is 50 mm. Determine minimum working length, shut height, clash allowance, spring index, and spring rate for this spring. Working deflection Wire diameter Outside diameter Free length ywork 50 mm d 4 mm Do 40 mm Lf 140 mm Initial deflection yinit 15 mm Shear modulus Total coils G 80 GPa Nt 18

Given:

Ends squared and ground Ne 2 Solution: 1. See Mathcad file P1449.

Determine the mean diameter, spring index, number of active coils, and spring rate. Mean diameter Spring index Number of active coils Spring rate D Do d C D d D 36 mm C9 Na 16 k 3.429 N mm

Na Nt Ne k d G 8 D Na
3 4

2.

Calculate the minimum working length, shut height, and clash allowance. Minimum work length Shut height Lmin Lf yinit ywork Lmin 75 mm Ls d Nt Ls 72 mm

Clash allowance

Lmin Ls ywork

6 %

MACHINE DESIGN - An Integrated Approach, 4th Ed.

14-50-1

PROBLEM 14-50
Statement: An extension spring has wire diameter d = 3 mm and outside diameter Do = 27 mm. Determine the preferred preload for this spring. Wire diameter See Mathcad file P1450. d 3 mm Outside diameter Do 27 mm

Given: Solution: 1.

Determine the mean diameter, spring index, and shear factor. Mean diameter Spring index Shear factor D Do d C D d 0.5 C D 24 mm C8 Na Ks 1.063

Ks 1

2.

Use the value of C to find an appropiate value of initial coil stress i from equations 14.21:

i1 4.231 C 181.5 C 3387 C 28640 psi i2

3 2 3 2 2.987 C 139.7 C 3427 C 38404 psi

i1 75.8 MPa i2 126.9 MPa i 101.3 MPa

i
3.

i1 i2
2

Use the value of i from step 2 in equation 14.8b to find the corresponding initial coil-tension force Fi:

Fi

d i
8 K s D

Fi 42.1 N

MACHINE DESIGN - An Integrated Approach, 4th Ed.

13-51-1

PROBLEM 14-51
Statement: A series-stack of Belleville springs is required for more deflection in a design. The stack will be guided by an internal pin as shown in Figure 14-35(b). The minimum inside diameter of the individual springs in the stack is Di = 25 mm. Determine the recommended surface conditions for the pin and the maximum pin diamter. (Hint: go to Spirol.com on the internet and look up disc springs, stacking). Inside diameter Di 25 mm

Given: Solution:

See Figure 14-35(b) and Mathcad file P1451.

1. The Spirol recommended surface conditions are "case hardened to a depth not less than 0.6 mm and a hardness of 58 HRC. A surface finish of less than or equal to 4 microns is also recommended." 2. From the table of recommended clearances c 0.4 mm. Maximum pin dia. d pin Di c d pin 24.6 mm

MACHINE DESIGN - An Integrated Approach, 4th Ed.

14-52-1

PROBLEM 14-52
Statement: Equation 14.9a defines a combined direct shear and stress concentration factor, Kw, to be used with helical springs that are made with round wire. Equation 14.11 defines a similar factor, Krw, that is to be used to calculate shear stress when the spring is made with rectangular wire. Determine and plot the ratio Krw/Kw for square wire with values of spring index, C, ranging from 1.2 to 10. Constant values from Table 14-6 for b/h = 1/1: S 0 1.6844 S 1 2.8219 S 2 2.4577 S 3 1.0591 S 4 0.1721 See Table 14-6 and Mathcad file P1452.

Given:

Solution: 1.

The Wahl factor for round wire is, from equation 14.9a: Kw( C) 4 C 1 4 C 4 0.615 C

2.

The stress concentration factor for rectangular wire is, from equation 14.11: s( C) ln( C) Krw

S S s( C) S s( C) S s( C) S s( C) ( C) e
2 3 4 0 1 2 3 4

3.

Plot the ratio of these two factors over the range:

C 1.2 1.21 10

STRESS CONCENTRATION FACTOR RATIO FOR SQUARE vs. ROUND WIRE


1 Stress Concentration Factor Ratio, Square/Round

0.9

Krw ( C) Kw( C) 0.8

0.7

0.6

4 C Spring Index, C

MACHINE DESIGN - An Integrated Approach, 4th Ed.

14-53-1

PROBLEM 14-53
Statement: Given: Repeat Problem 14-6 with 1-mm square wire instead of round wire. Wire dimension Mean coil dia b 1 mm D 10 mm h 1 mm Total coils Nt 12

Assumptions: The spring wire is steel so that G 80.8 GPa. Solution: 1. 2. See Table 14-7 and Mathcad file P1453. Na Nt 2 Na 10

From Figure 14-9, the number of active coils is Using equation 14.11d and Table 14-7, Shape factor K1 0.180 k K1 b h G D Na
3 3

Spring rate

k 1.454

N mm

3.

Using equation 14.5, Spring Index C D b C 10

MACHINE DESIGN - An Integrated Approach, 4th Ed.

14-54-1

PROBLEM 14-54
Statement: A helical compression spring is required to provide a minimum force of 650 N at installation and have a working deflection of 25 mm. The spring rate is 13 N/mm. The coil must fit in a 53-mm-dia hole with 3-mm clearance. Use 6-mm square, music wire with squared/ground ends. Using a 15% clash allowance, find: (a) The stress at the working deflection. (b) The shut height. (c) The stress at the shut height. (d) The total number of coils. (e) The free length. (f) The natural frequency in Hz. Minimum force Fmin 650 N Spring OD OD 50 mm Working deflection y 25 mm Music wire dims: b 6 mm Constant values from Table 14-6 for b/h = 1/1: S 0 1.6844 S 1 2.8219 S 2 2.4577 See Mathcad file P1454. Spring rate Shear modulus Clash factor h 6 mm S 3 1.0591 k 13 N mm G 80.8 GPa 0.15
1

Given:

S 4 0.1721

Solution: 1.

From the given information, find the maximum force and the spring index. Maximum force Spring index Fmax Fmin k y C OD b b Fmax 975 N C 7.333

2.

Calculate the stress concentration factor from equation 14.11a. s ln( C) Krw e

S S s S s S s S s
2 3 4 0 1 2 3 4

Krw 1.166

3.

Calculate the mean coil diameter D from equation 14.5. Mean coil diameter D C b D 44.0 mm Calculate the shear stress w at the working deflection using the shape factor from Table 14-7 and equation 14.11b. Shape factors Stress at Fmax K1 0.180 K2 2.41

4.

w Krw

K2 Fmax D b h
2

w 558.2 MPa

5.

To get the defined spring rate, the number of active coils must satisfy equation 14.11d, solving for Na yields: Na K1 b h G D k
3 3

Number of active coils

Na 17.021

Na 17

Note that we round it to the nearest 1/4 coil as the manufacturing tolerance cannot achieve better than that accuracy. Having rounded the number of active coils, we must now calculate the spring rate using equation 14.11d: Corrected spring rate k K1 b h G D Na
3 3

k 13.02

N mm

MACHINE DESIGN - An Integrated Approach, 4th Ed. 6. For squared ends the total number of coils, from Figure 14-9:
Total coils 7. The shut height can now be determined. Shut height 8. Lshut h Nt Fmin k Lshut 114 mm Nt Na 2 Nt 19.00

14-54-2

The initial deflection to reach the smaller of the two loads is Initial deflection yinit yinit 49.94 mm

9.

For the given clash allowance factor: Clash allowance

yclash y

yclash 3.75 mm
Lf 193 mm

10. The free length (see Figure 14-8) can now be found from Lf Lshut yclash y yinit 11. The deflection to the shut height is yshut Lf Lshut 12. The force at this shut height is Fshut k yshut 13. The shut-height stress is Stress at Fshut Fshut 1024 N yshut 79 mm

shut Krw

K2 Fshut D b h
2

shut 586.3 MPa

14. The inside and outside coil diameters are Inside coil dia Outside coil dia Di D b Do D b holemin Do 0.05 D pin max Di 0.05 D Di 38 mm Do 50 mm holemin 52.2 mm pin max 35.8 mm

15. The smallest hole and largest pin that should be used with this spring are Smallest hole Largest pin

16. The weight of the spring's active coils is found from equation 14.11b Weight density Weight

0.285 lbf in

Wa b h D Na

Wa 6.545 N

17. The natural frequency of this spring is found from equation 14.11a and is: Natural frequency fn 1 2 k g Wa fn 69.83 Hz

MACHINE DESIGN - An Integrated Approach, 4th Ed.

14-54-1

PROBLEM 14-54
Statement: A helical compression spring is required to provide a minimum force of 650 N at installation and have a working deflection of 25 mm. The spring rate is 13 N/mm. The coil must fit in a 53-mm-dia hole with 3-mm clearance. Use 6-mm square, music wire with squared/ground ends. Using a 15% clash allowance, find: (a) The stress at the working deflection. (b) The shut height. (c) The stress at the shut height. (d) The total number of coils. (e) The free length. (f) The natural frequency in Hz. Minimum force Fmin 650 N Spring OD OD 50 mm Working deflection y 25 mm Music wire dims: b 6 mm Constant values from Table 14-6 for b/h = 1/1: S 0 1.6844 S 1 2.8219 S 2 2.4577 See Mathcad file P1454. Spring rate Shear modulus Clash factor h 6 mm S 3 1.0591 k 13 N mm G 80.8 GPa 0.15
1

Given:

S 4 0.1721

Solution: 1.

From the given information, find the maximum force and the spring index. Maximum force Spring index Fmax Fmin k y C OD b b Fmax 975 N C 7.333

2.

Calculate the stress concentration factor from equation 14.11a. s ln( C) Krw e

S S s S s S s S s
2 3 4 0 1 2 3 4

Krw 1.166

3.

Calculate the mean coil diameter D from equation 14.5. Mean coil diameter D C b D 44.0 mm Calculate the shear stress w at the working deflection using the shape factor from Table 14-7 and equation 14.11b. Shape factors Stress at Fmax K1 0.180 K2 2.41

4.

w Krw

K2 Fmax D b h
2

w 558.2 MPa

5.

To get the defined spring rate, the number of active coils must satisfy equation 14.11d, solving for Na yields: Na K1 b h G D k
3 3

Number of active coils

Na 17.021

Na 17

Note that we round it to the nearest 1/4 coil as the manufacturing tolerance cannot achieve better than that accuracy. Having rounded the number of active coils, we must now calculate the spring rate using equation 14.11d: Corrected spring rate k K1 b h G D Na
3 3

k 13.02

N mm

MACHINE DESIGN - An Integrated Approach, 4th Ed. 6. For squared ends the total number of coils, from Figure 14-9:
Total coils 7. The shut height can now be determined. Shut height 8. Lshut h Nt Fmin k Lshut 114 mm Nt Na 2 Nt 19.00

14-54-2

The initial deflection to reach the smaller of the two loads is Initial deflection yinit yinit 49.94 mm

9.

For the given clash allowance factor: Clash allowance

yclash y

yclash 3.75 mm
Lf 193 mm

10. The free length (see Figure 14-8) can now be found from Lf Lshut yclash y yinit 11. The deflection to the shut height is yshut Lf Lshut 12. The force at this shut height is Fshut k yshut 13. The shut-height stress is Stress at Fshut Fshut 1024 N yshut 79 mm

shut Krw

K2 Fshut D b h
2

shut 586.3 MPa

14. The inside and outside coil diameters are Inside coil dia Outside coil dia Di D b Do D b holemin Do 0.05 D pin max Di 0.05 D Di 38 mm Do 50 mm holemin 52.2 mm pin max 35.8 mm

15. The smallest hole and largest pin that should be used with this spring are Smallest hole Largest pin

16. The weight of the spring's active coils is found from equation 14.11b Weight density Weight

0.285 lbf in

Wa b h D Na

Wa 6.545 N

17. The natural frequency of this spring is found from equation 14.11a and is: Natural frequency fn 1 2 k g Wa fn 69.83 Hz

MACHINE DESIGN - An Integrated Approach, 4th Ed.

15-1-1

PROBLEM 15-1
Statement: Compare the tensile load capacity of a 5/16-18 UNC thread and a 5/16-24 UNF thread made of the same material. Which is stronger? Make the same comparison for M8 x 1.25 and M8 x 1 ISO threads. Compare them all to the strength of a 5/16-14 Acme thread.

Assumptions: The material strength for all threads is S 100 ksi. Solution: 1. See Mathcad file P1501.

Get the tensile stress area for each thread specification from Tables 15-1 (for Unified National Standard sizes), 15-2 (for metric sizes), and 15-3 (for Acme threads). Thread specification 5/16-18 UNC 5/16-24 UNF M8 x 1.25 M8 x 1 5/16-14 Acme Tensile Stress Area a tUNC 0.0524 in
2 2

a tUNF 0.0581 in a tMC 36.61 mm a tMF 39.17 mm a tAcme 0.053 in

2 2

2.

Using the assumed strength, determine the allowable load for each thread specification. Thread specification 5/16-18 UNC 5/16-24 UNF M8 x 1.25 M8 x 1 5/16-14 Acme Equation FUNC a tUNC S FUNF a tUNF S FMC a tMC S FMF a tMF S FAcme a tAcme S Allowable Load FUNC 5240 lbf FUNF 5810 lbf FMC 25.24 kN FMF 27.01 kN FAcme 5300 lbf FAcme 23.58 kN

3.

State the conclusions. 1. The fine thread has a higher capacity than the coarse thread for both the UNS and ISO threads. 2. The Acme thread has a higher capacity than the UNS coarse thread, but has a lower cpacity than the fine UNS thread. 3. The Acme thread has a lower capacity than either of the metric threads.

MACHINE DESIGN - An Integrated Approach, 4th Ed.

15-2-1

PROBLEM 15-2
Statement: A 3/4-6 Acme thraed screw is used to lift a 2-kN load. The mean collar diameter is 40 mm. Find the torque to lift and to lower the load using a ball-bearing thrust washer. What are the efficiencies? Is it sel-locking? Screw diameter Collar diameter Threads per inch d 0.750 in d c 40 mm Nt 6 in
1

Given:

Applied load Radial thread angle

P 2 kN 14.5 deg

Assumptions: 1. The thread coefficient of friction is 0.15. 2. The collar coefficient of friction is c 0.02. Solution: 1. 2. See and Mathcad file P1502. d p 0.667 in 1 p Nt L p 3.

Get the thread pitch diameter from Table 15-3. Determine the thread pitch and lead.

p 0.167 in L 0.167 in

Use equations 15.5 to determine the lifting (up) and lowering (down) torques. Tu P d p 2 P d p 2

dp L cos( ) dp cos( ) L dp L cos( ) dp cos( ) L

c P

dc 2 dc 2

Tu 42.68 in lbf

Td

c P

Td 18.25 in lbf

4.

Use equation 15.7c to determine the lifting (up) and lowering (down) efficiencies. eu P L 2 Tu P L 2 Td eu 27.9 %

ed

ed 65.4 %

5.

Use equation 15.6a to determine if the screw is self-locking. self_locking return "yes" if "no" otherwise self_locking "yes" L cos( )

d p

MACHINE DESIGN - An Integrated Approach, 4th Ed.

15-3-1

PROBLEM 15-3
Statement: A 1 3/8-4 Acme thraed screw is used to lift a 1-ton load. The mean collar diameter is 2 in. Find the torque to lift and to lower the load using a ball-bearing thrust washer. What are the efficiencies? Is it sel-locking? Screw diameter Collar diameter Threads per inch d 1.375 in d c 2.00 in Nt 4 in
1

Given:

Applied load Radial thread angle

P 2000 lbf 14.5 deg

Assumptions: 1. The thread coefficient of friction is 0.15. 2. The collar coefficient of friction is c 0.02. Solution: 1. 2. See and Mathcad file P1503. d p 1.250 in 1 p Nt L p 3.

Get the thread pitch diameter from Table 15-3. Determine the thread pitch and lead.

p 0.250 in L 0.250 in

Use equations 15.5 to determine the lifting (up) and lowering (down) torques. Tu P d p 2 P d p 2

dp L cos( ) dp cos( ) L dp L cos( ) dp cos( ) L

c P

dc 2 dc 2

Tu 316.0 in lbf

Td

c P

Td 153.0 in lbf

4.

Use equation 15.7c to determine the lifting (up) and lowering (down) efficiencies. eu P L 2 Tu P L 2 Td eu 25.2 %

ed

ed 52.0 %

5.

Use equation 15.6a to determine if the screw is self-locking. self_locking return "yes" if "no" otherwise self_locking "yes" L cos( )

d p

MACHINE DESIGN - An Integrated Approach, 4th Ed.

15-4-1

PROBLEM 15-4
Statement: The trailer hitch from Figure 1-1 (p. 12) has loads applied as shown in Figure P3-2. The tongue weight of 100 kg acts downward and the pull force of 4905 N acts horizontally. Using the dimensions of the ball bracket in Figure 1-5 (p. 15), draw a free-body diagram of the ball bracket and find the tensile and shear loads applied to the two bolts that attach the bracket to the channel in Figure 1-1. Size and specify the bolts and their preload for a safety factor of at least 1.7. Hitch dimensions: a 40 mm b 31 mm c 70 mm d 20 mm t 19 mm Tongue weight Pull force Number of bolts Young's modulus Design safety factor Bolt modulus Member modulus Mtongue 100 kg Fpull 4.905 kN Nbolts 2 E 206.8 GPa Nd 1.7 Ebolt E Ememb E

Given:

Assumptions: The shear load will be taken by friction between the hitch and the support. Design Choices: Use M12 x 1.75 , class 8.8 bolts. Material properties for class 8.8: Proof strength Yield strength S p 600 MPa S y 660 MPa

Bolt diameter Clamp length Preload fraction

d b 12 mm l 30 mm fp 0.59
W tongue

70 = c

F pull

40 = a 2 A

19 = t 31 = b Fc2x

C 20 = d D

C D

Fd2 F c2y

FIGURE S15-4
Dimensions and Free Body Diagram for Problem 15-4

Solution: 1.

See Figure S15-4 and Mathcad file P1504.

The weight on the tongue is Wtongue Mtongue g Wtongue 0.981 kN

2.

The FBD of the hitch and bracket assembly is shown in Figure 3-4. The known external forces that act on the ball are Fpull and Wtongue . The reactions on the bracket are at points C and D. The bolts at C provide tensile (Fc2x) and shear (Fc2y) forces, and the bracket resists rotation about point D where the reaction force Fd2 is applied by the channel to which the bracket is bolted.

MACHINE DESIGN - An Integrated Approach, 4th Ed.


3. Solving for the reactions by summing the horizontal and vertical forces and the moments about D:

15-4-2

Fx : Fy : MD :
4.

Fpull Fc2x Fd2 = 0


Fc2y Wtongue = 0 Fc2x d Fpull ( a t b d ) Wtongue c = 0

(1) (2) (3)

Solving equation (3) for Fc2x Fc2x Fpull ( a t b d ) Wtongue c d Fc2x 30.41 kN (4)

5.

Substituting into (1) and solving for Fd2 Fd2 Fc2x Fpull Fd2 25.505 kN (5)

6.

Solving (2) for Fc2y Fc2y Wtongue Fc2y 0.981 kN (6)

7.

The loads applied to the two bolts that attach the bracket to the channel are: Axial force on two bolts Shear force taken by friction P Fc2x 30.4 kN Fc2y 0.98 kN Ptot Nbolts P 15.20 kN At 84.27 mm Fi 29.83 kN
2

Ptot Fc2x

8. 9.

Determine the load per bolt.

Get the tensile stress area from Table 15-2. Fi fp S p At

10. Calculate the preload.

11. Determine the relevant ratios for this joint from equations 15-18a and b. Joint aspect ratio: j r db l Ememb Ebolt j 0.400 r1

Plate to bolt modulus:

12. Calculate Cr = C using equation 15.19 and the coefficients p i from Table 15-8 for j 0.4 . Coefficients from Table 15-8: p 0 0.7351 p 1 1.2612 p 2 1.1111 p 3 0.3779 Joint stiffness constant: C p 3 r p 2 r p 1 r p 0
3 2

C 0.207

MACHINE DESIGN - An Integrated Approach, 4th Ed.

15-4-3

13. The portions of the applied load P felt by the bolt and the material can now be found from equations 15.13. Pb C P Pm ( 1 C) P Fb Fi Pb Fm Fi Pm 15. The maximum tensile stress in the bolt is Pb 3.15 kN Pm 12.1 kN Fb 32.98 kN Fm 17.78 kN

14. Find the resulting loads in bolt and material after the load P is applied.

Fb At

b 391.4 MPa

16. This is a uniaxial stress situation, so the principal stress and von Mises stress are identical to the applied tensile stress. The safety factor against yielding for class 8.8 with S y 660 MPa is then Ny Sy Ny 1.7

17. The load required to separate the joint and the safety factor against joint separation are found from equations 15.14c and 15.14d. P0 Fi 1C P0 P P0 37.6 kN Nsep 2.5

Nsep

MACHINE DESIGN - An Integrated Approach, 4th Ed.

15-5-1

PROBLEM 15-5
Statement: For the hitch of Problem 3-4, determine the horizontal force that will result on the ball from accelerating a 2000-kg trailer to 60 m/sec in 20 sec. Size and specify the bolts and their preload for a safety factor of at least 1.7. Tongue weight Hitch dimensions: a 40 mm Mtongue 100 kg b 31 mm c 70 mm d 20 mm t 19 mm Pull force Number of bolts Young's modulus Design safety factor Bolt modulus Member modulus Fpull 6 kN Nbolts 2 E 206.8 GPa Nd 1.7 Ebolt E Ememb E

Given:

Assumptions: The shear load will be taken by friction between the hitch and the support. Design Choices: Use M12 x 1.75 , class 8.8 bolts. Material properties for class 8.8: Proof strength Yield strength S p 600 MPa S y 660 MPa

Bolt diameter Clamp length Preload fraction

d b 12 mm l 30 mm fp 0.58
W tongue

70 = c

F pull

40 = a 2 A

19 = t 31 = b Fc2x

C 20 = d D

C D

Fd2 F c2y

FIGURE 15-5
Dimensions and Free Body Diagram for Problem 15-5

Solution: 1.

See Figure 15-5 and Mathcad file P1505.

The weight on the tongue is Wtongue Mtongue g Wtongue 0.981 kN

2.

The FBD of the hitch and bracket assembly is shown in Figure 3-4. The known external forces that act on the ball are Fpull and Wtongue . The reactions on the bracket are at points C and D. The bolts at C provide tensile (Fc2x) and shear (Fc2y) forces, and the bracket resists rotation about point D where the reaction force Fd2 is applied by the channel to which the bracket is bolted.

MACHINE DESIGN - An Integrated Approach, 4th Ed.


3. Solving for the reactions by summing the horizontal and vertical forces and the moments about D:

15-5-2

Fx : Fy : MD :
4.

Fpull Fc2x Fd2 = 0


Fc2y Wtongue = 0 Fc2x d Fpull ( a t b d ) Wtongue c = 0

(1) (2) (3)

Solving equation (3) for Fc2x Fc2x Fpull ( a t b d ) Wtongue c d Fc2x 36.43 kN (4)

5.

Substituting into (1) and solving for Fd2 Fd2 Fc2x Fpull Fd2 30.43 kN (5)

6.

Solving (2) for Fc2y Fc2y Wtongue Fc2y 0.981 kN (6)

7.

The loads applied to the two bolts that attach the bracket to the channel are: Axial force on two bolts Shear force taken by friction P Fc2x 36.4 kN Fc2y 0.98 kN Ptot Nbolts P 18.22 kN At 84.27 mm Fi 29.33 kN
2

Ptot Fc2x

8. 9.

Determine the load per bolt.

Get the tensile stress area from Table 15-2. Fi fp S p At

10. Calculate the preload.

11. Determine the relevent ratios for this joint from equations 15-18a and b. Joint aspect ratio: j r db l Ememb Ebolt j 0.400 r1

Plate to bolt modulus:

12. Calculate Cr = C using equation 15.19 and the coefficients p i from Table 15-8 for j 0.4 . Coeficients from Table 15-8: p 0 0.7351 p 1 1.2612 p 2 1.1111 p 3 0.3779 Joint stiffness constant: C p 3 r p 2 r p 1 r p 0
3 2

C 0.207

MACHINE DESIGN - An Integrated Approach, 4th Ed.

15-5-3

13. The portions of the applied load P felt by the bolt and the material can now be found from equations 15.13. Pb C P Pm ( 1 C) P Fb Fi Pb Fm Fi Pm 15. The maximum tensile stress in the bolt is Pb 3.77 kN Pm 14.4 kN Fb 33.10 kN Fm 14.88 kN

14. Find the resulting loads in bolt and material after the load P is applied.

Fb At

b 392.8 MPa

16. This is a uniaxial stress situation, so the principal stress and von Mises stress are identical to the applied tensile stress. The safety factor against yielding for class 8.8 with S y 660 MPa is then Ny Sy Ny 1.7

17. The load required to separate the joint and the safety factor against joint separation are found from equations 15.14c and 15.14d. P0 Fi 1C P0 P P0 37.0 kN Nsep 2.0

Nsep

MACHINE DESIGN - An Integrated Approach, 4th Ed.

15-6-1

PROBLEM 15-6
Statement: For the trailer hitch of Problem 3-4, determine the horizontal force that will result on the ball from an impact between the ball and the tongue of the 2000-kg trailer if the hitch deflects 2.8 mm dynamically on impact. Size and specify the bolts and their preload for a safety factor of at least 1.7. Hitch dimensions: a 40 mm b 31 mm c 70 mm d 20 mm t 19 mm Tongue weight Pull force Number of bolts Young's modulus Design safety factor Bolt modulus Member modulus Mtongue 100 kg Fpull 55.1 kN Nbolts 2 E 206.8 GPa Nd 1.7 Ebolt E Ememb E

Given:

Assumptions: The shear load will be taken by friction between the hitch and the support. Design Choices: Use M24 x 3 , class 12.9 bolts. Material properties for class 12.9: Proof strength Yield strength S p 970 MPa

Bolt diameter Clamp length

d b 24 mm l 30 mm fp 0.55
W tongue

S y 1100 MPa Preload fraction

70 = c

F pull

40 = a 2 A

19 = t 31 = b Fc2x

C 20 = d D

C D

Fd2 F c2y

FIGURE 15-6
Dimensions and Free Body Diagram for Problem 15-6

Solution: 1.

See Figure 15-6 and Mathcad file P1506.

The weight on the tongue is Wtongue Mtongue g Wtongue 0.981 kN

2.

The FBD of the hitch and bracket assembly is shown in Figure 3-4. The known external forces that act on the ball are Fpull and Wtongue . The reactions on the bracket are at points C and D. The bolts at C provide tensile (Fc2x) and shear (Fc2y) forces, and the bracket resists rotation about point D where the reaction force Fd2 is applied by the channel to which the bracket is bolted.

MACHINE DESIGN - An Integrated Approach, 4th Ed.


3. Solving for the reactions by summing the horizontal and vertical forces and the moments about D:

15-6-2

Fx : Fy : MD :
4.

Fpull Fc2x Fd2 = 0


Fc2y Wtongue = 0 Fc2x d Fpull ( a t b d ) Wtongue c = 0

(1) (2) (3)

Solving equation (3) for Fc2x Fc2x Fpull ( a t b d ) Wtongue c d Fc2x 306.48 kN (4)

5.

Substituting into (1) and solving for Fd2 Fd2 Fc2x Fpull Fd2 251.38 kN (5)

6.

Solving (2) for Fc2y Fc2y Wtongue Fc2y 0.981 kN (6)

7.

The loads applied to the two bolts that attach the bracket to the channel are: Axial force on two bolts Shear force taken by friction P Fc2x 306.5 kN Fc2y 0.98 kN Ptot Nbolts P 153.24 kN At 352.5 mm Fi 188.06 kN
2

Ptot Fc2x

8. 9.

Determine the load per bolt.

Get the tensile stress area from Table 15-2. Fi fp S p At

10. Calculate the preload.

11. Determine the relevant ratios for this joint from equations 15-18a and b. Joint aspect ratio: j r db l Ememb Ebolt j 0.800 r1

Plate to bolt modulus:

12. Calculate Cr = C using equation 15.19 and the coefficients p i from Table 15-8 for j 0.8 . Coefficients from Table 15-8: p 0 0.7800 p 1 1.2503 p 2 1.0672 p 3 0.3571 Joint stiffness constant: C p 3 r p 2 r p 1 r p 0
3 2

C 0.240

MACHINE DESIGN - An Integrated Approach, 4th Ed.

15-6-3

13. The portions of the applied load P felt by the bolt and the material can now be found from equations 15.13. Pb C P Pm ( 1 C) P Fb Fi Pb Fm Fi Pm 15. The maximum tensile stress in the bolt is Pb 36.75 kN Pm 116.5 kN Fb 224.81 kN Fm 71.56 kN

14. Find the resulting loads in bolt and material after the load P is applied.

Fb At

b 637.7 MPa

16. This is a uniaxial stress situation, so the principal stress and von Mises stress are identical to the applied tensile stress. The safety factor against yielding for class 8.8 with S y 1.1 10 MPa is then Ny Sy Ny 1.7
3

17. The load required to separate the joint and the safety factor against joint separation are found from equations 15.14c and 15.14d. P0 Fi 1C P0 P P0 247.4 kN Nsep 1.6

Nsep

MACHINE DESIGN - An Integrated Approach, 4th Ed.

15-7-1

PROBLEM 15-7
Statement: A 1/2-in dia UNC, class 7 bolt with rolled threads is preloaded to 80% of its proof strength when clamping a 3-in-thick sandwich of solid steel. Find the safety factors against static yielding and joint separation when a static 1000-lb external load is applied. Use 99% reliability. Bolt diameter d 0.500 in Preload fraction fp 0.80 Clamp length l 3 in Number of bolts Nbolts 1 Applied load Bolt modulus Solution: 1. 2. 3. 4. Ptot 1000 lbf Ebolt E Material properties for class 7: Proof strength S p 105 ksi Yield strength S y 115 ksi
6

Given:

Ultimate strength S ut 133 ksi Young's modulus E 30 10 psi Member modulus Ememb E

See Mathcad file P1507. P Ptot Nbolts P 1000 lbf At 0.1419 in Fi 11920 lbf
2

Determine the load per bolt.

Get the tensile stress area from Table 15-1. Calculate the preload. Fi fp S p At

Determine the relevant ratios for this joint from equations 15-17a and b. Joint aspect ratio: j r d l Ememb Ebolt j 0.167 r1

Plate to bolt modulus:

5.

Calculate Cr = C using equation 15.18 and the coefficients p i from Table 15-8 for j 0.167 . Use linear interpolation between ja 0.1 and jb 0.2 Interpolation equation: p p a p b pb pa jb ja j ja p a p 0 0.5542 p 1 1.0876 p 2 1.0217 p 3 0.3600 C 0.128

Coefficients from Table 15-8:

p 0 p ( 0.4389 0.6118) p 1 p ( 0.9197 1.1715) p 2 p ( 0.8901 1.0875) p 3 p ( 0.3187 0.3806) C p 3 r p 2 r p 1 r p 0


3 2

Joint stiffness constant: 6.

The portions of the applied load P felt by the bolt and the material can now be found from equations 15.13. Pb C P Pm ( 1 C) P Pb 128.3 lbf Pm 871.7 lbf Fb 12048 lbf

Find the resulting loads in bolt and material after the load P is applied. Fb Fi Pb

MACHINE DESIGN - An Integrated Approach, 4th Ed.


Fm Fi Pm 8 The maximum tensile stress in the bolt is Fm 11048 lbf

15-7-2

b
9.

Fb At

b 84.9 ksi

This is a uniaxial stress situation, so the principal stress and von Mises stress are identical to the applied tensile stress. The safety factor against yielding for Grade 7 with S y 115 ksi is then Ny Sy Ny 1.4

10. The load required to separate the joint and the safety factor against joint separation are found from equations 15.14c and 15.14d. P0 Fi 1C P0 P P0 13674 lbf Nsep 13.7

Nsep

MACHINE DESIGN - An Integrated Approach, 4th Ed.

15-8-1

PROBLEM 15-8
Statement: An M14 x 2, class 8.8 bolt with rolled threads is preloaded to 75% of its proof strength when clamping a 30-mm-thick sandwich of solid aluminum. Find the safety factors against static yielding and joint separation when a static 5-kN external load is applied. Material properties for class 8.8: Bolt diameter d 14 mm Proof strength Preload fraction fp 0.75 S p 600 MPa Clamp length Applied load l 30 mm Ptot 5 kN Yield strength S y 660 MPa Number of bolts Nbolts 1 Ultimate strength S ut 830 MPa Young's modulus Ememb 71.7 GPa Young's modulus Ebolt 206.8 GPa Solution: 1. 2. 3. 4. See Table 15-8 and Mathcad file P1508. P Ptot Nbolts P 5 kN At 115.44 mm Fi 52 kN
2

Given:

Determine the load per bolt.

Get the tensile stress area from Table 15-2. Calculate the preload. Fi fp S p At

Determine the relevant ratios for this joint from equations 15-18a and b. Joint aspect ratio: j r d l Ememb Ebolt j 0.467 r 0.347

Plate to bolt modulus:

5.

Calculate Cr = C using equation 15.19 and the coefficients p i from Table 15-8 for j 0.467 . Use linear interpolation between ja 0.4 and jb 0.5 Interpolation equation: p p a p b pb pa jb ja j ja p a p 0 0.7504 p 1 1.2625 p 2 1.1023 p 3 0.3732 C 0.430

Coefficients from Table 15-8:

p 0 p ( 0.7351 0.7580) p 1 p ( 1.2612 1.2632) p 2 p ( 1.1111 1.0979) p 3 p ( 0.3779 0.3708)

Joint stiffness constant: 6.

C p 3 r p 2 r p 1 r p 0

The portions of the applied load P felt by the bolt and the material can now be found from equations 15.13. Pb C P Pm ( 1 C) P Pb 2.1 kN Pm 2.9 kN Fb 54.1 kN Fm 49.1 kN

Find the resulting loads in bolt and material after the load P is applied. Fb Fi Pb Fm Fi Pm

MACHINE DESIGN - An Integrated Approach, 4th Ed.


8 The maximum tensile stress in the bolt is

15-8-2

b
9.

Fb At

b 468.6 MPa

This is a uniaxial stress situation, so the principal stress and von Mises stress are identical to the applied tensile stress. The safety factor against yielding for Grade 8.8 with S y 660 MPa is then Ny Sy Ny 1.4

10. The load required to separate the joint and the safety factor against joint separation are found from equations 15.14c and 15.14d. P0 Fi 1C P0 P P0 91.1 kN Nsep 18.2

Nsep

MACHINE DESIGN - An Integrated Approach, 4th Ed.

15-9-1

PROBLEM 15-9
Statement: A 7/16-in dia UNC, Grade 7 bolt with rolled threads is preloaded to 70% of its proof strength when clamping a 2.75-in-thick sandwich of solid steel. Find the safety factors against fatigue failure, yielding, and joint separation when a 1000-lb (peak) fluctuating external load is applied. Use 99% reliability. Bolt diameter d 0.4375 in Preload fraction fp 0.70 Clamp length l 2.75 in Number of bolts Nbolts 1 Applied load Bolt modulus Solution: 1. Ptot 1000 lbf Ebolt E Material properties for Grade 7: Proof strength S p 105 ksi Yield strength S y 115 ksi
6

Given:

Ultimate strength S ut 133 ksi Young's modulus E 30 10 psi Member modulus Ememb E

See Mathcad file P1509. Pmax Ptot Nbolts Pmax 1000 lbf At 0.1063 in Fi 7813 lbf
2

Determine the load per bolt.

Pmin 0 lbf

2. 3. 4.

Get the tensile stress area from Table 15-1. Calculate the preload. Fi fp S p At

Determine the relevant ratios for this joint from equations 15-18a and b. Joint aspect ratio: j r d l Ememb Ebolt j 0.159 r1

Plate to bolt modulus:

5.

Calculate Cr = C using equation 15.19 and the coefficients p i from Table 15-8 for j 0.159 . Use linear interpolation between ja 0.1 and jb 0.2 Interpolation equation: p p a p b pb pa jb ja j ja p a p 0 0.5411 p 1 1.0685 p 2 1.0067 p 3 0.3553 C 0.124

Coefficients from Table 15-8:

p 0 p ( 0.4389 0.6118) p 1 p ( 0.9197 1.1715) p 2 p ( 0.8901 1.0875) p 3 p ( 0.3187 0.3806)

Joint stiffness constant: 6.

C p 3 r p 2 r p 1 r p 0

The portions of the applied load P felt by the bolt and the material can now be found from equations 15.13. Pb C Pmax Pm ( 1 C) Pmax Pb 124.0 lbf Pm 876.0 lbf Fb 7937 lbf

7.

Find the resulting loads in bolt and material after the load P is applied. Fb Fi Pb

MACHINE DESIGN - An Integrated Approach, 4th Ed.


Fm Fi Pm 8. Fm 6937 lbf

15-9-2

Calculate the alternating and mean components of the fluctuating bolt load. Falt Fb Fi 2 Fb Fi 2 Falt 62.0 lbf Fmean 7875 lbf

Fmean 9.

The nominal mean and alternating stresses in the bolt are: Nominal mean stress

mnom anom

Fmean At Falt At

mnom 74.083 ksi anom 583 psi

Nominal alternating stress

10. The fatigue stress-concentration factor for this thread is found from equation 15.15c and the mean stress-concentration factor factor Kfm is found from equation 6.17. Fatigue factor Nominal maximum stress Nominal minimum stress Kfm Kf 5.7 0.6812 d in Kf 6.00

maxnom mnom anom minnom mnom anom

maxnom 74.667 ksi minnom 73.5 ksi

return Kf if Kf maxnom S y return S y Kf anom if Kf maxnom S y Kfm 1.505

mnom

return 0 if Kf maxnom minnom 2 S y

alt Kf

Falt At Fmean At

alt 3.50 ksi mean 111.50 ksi

mean Kfm
11. The stress at the initial preload is

init Kfm

Fi At

init 110.62 ksi

12. An endurance strength must be found for this material. Using the methods of Section 6.6 we find for S ut 133 ksi S'e 0.5 S ut 13. From the tables and formulas in Section 6.6 we have: Load Size Cload 0.70 Csize 1 S'e 66.5 ksi

MACHINE DESIGN - An Integrated Approach, 4th Ed.


Surface A 2.70 Csurf Temperature Reliability and the endurance limit is S e Cload Csize Csurf Ctemp Creliab S'e S e 28.00 ksi b 0.265
b

15-9-3

S ut A ksi

Csurf 0.739

Ctemp 1 Creliab 0.814

14. The corrected endurance strength and the ultimate tensile strength are used in equation 15.16 to find the safety factor from the Goodman line. Nf S e mean init S ut alt S e S ut init Nf 1.3

15. Calculate the maximum bolt stress and the safety factor against yielding for S y 115 ksi.

b
Ny

Fb At Sy

b 74.667 ksi
Ny 1.5

16. The load required to separate the joint and the safety factor against joint separation are found from equations 15.14c and 15.14d. Nsep Fi Pmax ( 1 C) Nsep 8.9

MACHINE DESIGN - An Integrated Approach, 4th Ed.

15-10-1

PROBLEM 15-10
Statement: An M12 x 1.25, class 9.8 bolt with rolled threads is preloaded to 85% of its proof strength when clamping a 5-cm-thick sandwich of aluminum. Find the safety factors against fatigue failure, yielding, and joint separation when a 2.5-kN (peak) fluctuating external load is applied. Use 99% reliability. Bolt diameter d 12 mm Preload fraction fp 0.85 Clamp length Applied load l 50 mm Ptot 2.5 kN Number of bolts Nbolts 1 Material properties for class 9.8 bolt: Proof strength S p 650 MPa Yield strength S y 720 MPa Ultimate strength S ut 900 MPa Young's modulus Ebolt 206.8 GPa Member modulus Solution: 1. See Mathcad file P1510. Pmax Ptot Nbolts Pmax 3 kN At 92.07 mm Fi 50.87 kN
2

Given:

Ememb 71.7 GPa

Determine the load per bolt.

Pmin 0 kN

2. 3. 4.

Get the tensile stress area from Table 15-1. Calculate the preload. Fi fp S p At

Determine the relevant ratios for this joint from equations 15-18a and b. Joint aspect ratio: j r d l Ememb Ebolt j 0.240 r 0.347

Plate to bolt modulus:

5.

Calculate Cr = C using equation 15.19 and the coefficients p i from Table 15-8 for j 0.24 . Use linear interpolation between ja 0.2 and jb 0.3 Interpolation equation: p p a p b pb pa jb ja j ja p a p 0 0.6444 p 1 1.1999 p 2 1.0996 p 3 0.3822 C 0.345

Coefficients from Table 15-8:

p 0 p ( 0.6118 0.6932) p 1 p ( 1.1715 1.2426) p 2 p ( 1.0875 1.1177) p 3 p ( 0.3806 0.3845)

Joint stiffness constant: 6.

C p 3 r p 2 r p 1 r p 0

The portions of the applied load P felt by the bolt and the material can now be found from equations 15.13. Pb C Pmax Pm ( 1 C) Pmax Pb 0.9 kN Pm 1.6 kN Fb 51.73 kN

7.

Find the resulting loads in bolt and material after the load P is applied. Fb Fi Pb

MACHINE DESIGN - An Integrated Approach, 4th Ed.


Fm Fi Pm 8. Fm 49.23 kN

15-10-2

Calculate the alternating and mean components of the fluctuating bolt load. Falt Fb Fi 2 Fb Fi 2 Falt 0.43 kN Fmean 51.3 kN

Fmean 9.

The nominal mean and alternating stresses in the bolt are: Nominal mean stress

mnom anom

Fmean At Falt At

mnom 557.2 MPa anom 4.68 MPa

Nominal alternating stress

10. The fatigue stress-concentration factor for this thread is found from equation 15.15c and the mean stress-concentration factor factor Kfm is found from equation 6.17. Fatigue factor Nominal maximum stress Nominal minimum stress Kfm Kf 5.7 0.02682 d mm Kf 6

maxnom mnom anom minnom mnom anom

maxnom 561.9 MPa minnom 552.5 MPa

return Kf if Kf maxnom S y return S y Kf anom if Kf maxnom S y Kfm 1.242

mnom

return 0 if Kf maxnom minnom 2 S y 11. The local mean and alternating stresses in the bolt are then: Local mean stress Local alternating stress 12. The stress at the initial preload is

m Kfm mnom a Kf anom

m 691.8 MPa a 28.2 MPa

init Kfm

Fi At

init 686.02 MPa

13. An endurance strength must be found for this material. Using the methods of Section 6.6 we find for S ut 130.534 ksi S'e 0.5 S ut 14. From the tables and formulas in Section 6.6 we have: Load Size Cload 0.70 Csize 1 S'e 450.0 MPa

MACHINE DESIGN - An Integrated Approach, 4th Ed.


Surface A 4.51 Csurf Temperature Reliability and the endurance limit is S e Cload Csize Csurf Ctemp Creliab S'e S e 190.65 MPa b 0.265
b

15-10-3

Sut A MPa

Csurf 0.744

Ctemp 1 Creliab 0.814

15. The corrected endurance strength and the ultimate tensile strength are used in equation 15.16 to find the safety factor from the Goodman line. Nf S e m init S ut a S e S ut init Nf 1.5

16. Calculate the maximum bolt stress and the safety factor against yielding for S y 720 MPa.

b
Ny

Fb At Sy

b 561.9 MPa
Ny 1.3

17. The load required to separate the joint and the safety factor against joint separation are found from equations 15.14c and 15.14d. Nsep Fi Pmax ( 1 C) Nsep 31.0

MACHINE DESIGN - An Integrated Approach, 4th Ed.

15-11-1

PROBLEM 15-11
Statement: Given: Find the tightening torque required for the bolt in Problem 15-7. Bolt diameter d 0.500 in Preload fraction fp 0.80 Applied load Solution: 1. 2. 3. 4. Ptot 1000 lbf Material properties for Grade 7: Proof strength S p 105 ksi Number of bolts Nbolts 1

See Mathcad file P1511. P Ptot Nbolts P 1000 lbf At 0.1419 in Fi 11920 lbf
2

Determine the load per bolt.

Get the tensile stress area from Table 15-1. Calculate the preload. Fi fp S p At

Calculate the tightening torque required for each bolt using equation 15.23. Ti 0.21 Fi d Ti 1252 in lbf

MACHINE DESIGN - An Integrated Approach, 4th Ed.

15-12-1

PROBLEM 15-12
Statement: Given: Find the tightening torque required for the bolt in Problem 15-8. Bolt diameter d 14 mm Preload fraction fp 0.75 Applied load Solution: 1. 2. 3. 4. Ptot 5 kN Material properties for class 8.8: Proof strength S p 600 MPa Number of bolts Nbolts 1

See Mathcad file P1512. P Ptot Nbolts P 5 kN At 115.44 mm Fi 51.9 kN


2

Determine the load per bolt.

Get the tensile stress area from Table 15-1. Calculate the preload. Fi fp S p At

Calculate the tightening torque required for each bolt using equation 15.23. Ti 0.21 Fi d Ti 153 N m

MACHINE DESIGN - An Integrated Approach, 4th Ed.

15-13-1

PROBLEM 15-13
Statement: Given: Find the tightening torque required for the bolt in Problem 15-9. Bolt diameter d 0.4375 in Preload fraction fp 0.70 Applied load Solution: 1. 2. 3. 4. Ptot 5000 lbf Material properties for Grade 7: Proof strength S p 105 ksi Number of bolts Nbolts 1

See Mathcad file P1513. P Ptot Nbolts P 5000 lbf At 0.1063 in Fi 7813 lbf
2

Determine the load per bolt.

Get the tensile stress area from Table 15-1. Calculate the preload. Fi fp S p At

Calculate the tightening torque required for each bolt using equation 15.23. Ti 0.21 Fi d Ti 718 in lbf

MACHINE DESIGN - An Integrated Approach, 4th Ed.

15-14-1

PROBLEM 15-14
Statement: Given: Find the tightening torque required for the bolt in Problem 15-10. Bolt diameter d 12 mm Preload fraction fp 0.85 Applied load Solution: 1. 2. 3. 4. Ptot 20 kN Material properties for class 9.8: Proof strength S p 650 MPa Number of bolts Nbolts 1

See Mathcad file P1514. P Ptot Nbolts P 20 kN At 92.07 mm Fi 50.9 kN


2

Determine the load per bolt.

Get the tensile stress area from Table 15-1. Calculate the preload. Fi fp S p At

Calculate the tightening torque required for each bolt using equation 15.23. Ti 0.21 Fi d Ti 128 N m

MACHINE DESIGN - An Integrated Approach, 4th Ed.

15-15-1

PROBLEM 15-15
Statement: An automobile manufacturer would like a feasibility study of the concept of building-in electric, motor-powered, screw jacks at each end of the car to automatically jack the car wheels off the ground for service. Assuming a 2-ton vehicle with a 60/40 front/rear weight distribution, design a self locking screw jack capable of lifting either end of the car. The jack body will be attached to the car frame and the screw will extend downward to engage the ground. Assume a minimum installed clearance of 8 in under the retracted screw in the up position. It must lift the car frame at least an additional 8 in. Use rolling element thrust bearings. Determine a minimu screw size, safe against column buckling. Determine its required lifting torque and efficiency, and the power required to lift it to full height in 45 sec. What is your recommendation as to the feasibility of this idea? Weight of car Noload extension Time to rertract Wcar 2000 lbf xnoload 8 in tup 45 sec Front weight factor Loaded extension ffront 0.6 xload 8 in

Given:

Design Choices: 1. The thread coefficient of friction is 0.15. 2. The collar coefficient of friction is c 0.02. 3. Use AISI 1050 steel, Q&T @ 400F, with S y 117 ksi, and E 30 10 psi. 4. The column is fixed-free with end condition constant C 2. 5. Use the pitch diameter of the screw to compute the column radius of gyration. 6. Use a buckling design safety factor of Nbd 4. 7. Use a mean collar diameter of d c 2.00 in. 8. The length of engagement of the nut on the screw is Lnut 0.75 in. Solution: 1. See Mathcad file P1515.
6

Using the buckling critera (see Problem 4-53), find the minimum pitch diameter for the screw. Column load Effective column length P ffront Wcar Leff C xload xnoload P 1200 lbf Leff 32 in

2.

Start by calculating the slenderness ratio that divides the unit load vs slenderness ratio graph into Johnson and Euler regions. S rD 2 E Sy S rD 71.143

3.

To start the iterative process, assume that the final design will be an Euler column with the critical load equal to Nbd*P. From equation 4.38b, Pcr =

E A k
L
2

and

k =

I A

Substituting for k2

Pcr =

E I
L
2 2

= Nbd P

Solving for I

Leff Nbd P

E
The required moment of inertia, assuming an Euler column is I 0.0166 in
4

MACHINE DESIGN - An Integrated Approach, 4th Ed.


4.

15-15-2

Using the equation for the moment of inertia of a solid, round rod, solve for the pich diameter of the screw.
1

I= 5.

d
64

d p

64 I

d p 0.763 in

Using this diameter, calculate the slenderness ratio and compare to S rD. If it is greater than S rD the assumption of an Euler column is correct, if not, recalculate using the Johnson equation. Area Ar kr

dp I

Ar 0.457 in kr 0.191 in

Radius of gyration

Ar Leff kr

Slenderness ratio

S r

S r 167.9

Since this is greater than S rD, the assumption of an Euler column is correct. 6. Enter Table 15-3 with this minimum pitch diameter and chose a tentative screw size of 7/8-6. Screw diameter Radial thread angle 7. d 0.875 in Threads per inch Pich daimeter 1 Nt Nt 6 in
1

14.5 deg
p

d p 0.792 in p 0.167 in L 0.167 in

Determine the thread pitch and lead.

L p 8. Use equations 15.5 to determine the extending and retracting torques. Text P d p 2 P d p 2

dp L cos( ) dp cos( ) L dp L cos( ) dp cos( ) L

c P

dc 2 dc 2

Text 130.6 in lbf

Tret

c P

Tret 65.4 in lbf

9.

Use equation 15.7c to determine the lifting (up) and lowering (down) efficiencies. eext eret P L 2 Text P L 2 Tret eext 24.4 % eret 48.7 %

10. Use equation 15.6a to determine if the screw is self-locking. self_locking return "yes" if "no" otherwise self_locking "yes" L cos( )

d p

MACHINE DESIGN - An Integrated Approach, 4th Ed.

15-15-3

11. Check the safety factor against thread shear by stripping using equations 15.8. The area factor for thread stripping is given in Table 15-5 as wi 0.77 for the minor diameter of an Acme thread. Minor diameter Total shear area Shear stress d r 0.708 in As d r L Lnut Nt As 1.668 in
2

P As 0.577 S y

s 719.3 psi

Factor of safety

Ns

Ns 94

12. Determine the power required to lift the car. Rotational speed

xload xnoload tup L

2.133

rad sec

Power required

H Text

H 0.042 hp

MACHINE DESIGN - An Integrated Approach, 4th Ed.

15-16-1

PROBLEM 15-16
Statement: Design a manual screw jack similar to that shown in Figure 15-4 for a 20-ton lift capacity and a 100-mm lift stroke. Assume that the operator can apply a 400-N force at the tip of its bar handle to turn either the screw or the nut, depending on your design. Design the cylindrical bar handle to fail in bending at the design load before the jackscrew fails so that one cannot lift an overload and fail the screw. Use rolling element thrust bearings. Seek a safety factor of 3 for thread or column failure. State all assumptions. Design jack load Lift stroke Young's modulus P 40000 lbf stroke 100 mm E 30 10 psi
6

Given:

Design handle load Design safety factor

Fhandle 400 N Nd 3

Design Choices: 1. The thread coefficient of friction is 0.15. 2. The collar coefficient of friction is c 0.02. 3. For the screw, use AISI 1050 steel, Q&T @ 400F, with S yscrew 117 ksi. 4. For the handle, use AISI 1020 cold rolled steel, with S yhandle 57 ksi. 5. The column is fixed-free with end condition constant C 2. 6. Use the pitch diameter of the screw to compute the column radius of gyration. 7. Use a mean collar diameter of d c 2.00 in. 8. The length of engagement of the nut on the screw is Lnut 2.00 in. 9. The length of the handle from its base to midpoint of grip is Lhandle 10 in Solution: 1. See Mathcad file P1516.

Using the buckling critera (see Problem 4-53), find the minimum pitch diameter for the screw. Effective column length Leff C stroke Leff 7.874 in

2.

Start by calculating the slenderness ratio that divides the unit load vs slenderness ratio graph into Johnson and Euler regions. S rD 2 E S yscrew S rD 71.143

3.

To start the iterative process, assume that the final design will be a Johnson column with the critical load equal to Nbd*P. From equation 4.38b,

Sy Pcr = S y A E 2
A Substituting for k2

k
1

k =
2

I A

I=

d
64

A=

d
4

Sy A 2 L2 Pcr = S y A = Nd P I E 2 1
2 4 Nd P 16 S yscrew Leff d p Syscrew E 2 2

Solving for d p

The minimum required pitch diameter, assuming a Johnson column is 5.

d p 1.185 in

Using this diameter, calculate the slenderness ratio and compare to S rD. If it is less than S rD the assumption of a Johnson column is correct, if not, recalculate using the Euler equation. Area Ar

dp

Ar 1.103 in

MACHINE DESIGN - An Integrated Approach, 4th Ed.


Moment of inertia I

15-16-2
I 0.097 in
4

64

dp

Radius of gyration

kr

I Ar Leff kr

kr 0.296 in

Slenderness ratio

S r

S r 26.6

Since this is less than S rD, the assumption of a Johnson column is correct. 6. Enter Table 15-3 with this minimum pitch diameter and chose a tentative screw size of 1 3/8-5. Screw diameter Radial thread angle 7. d 1.375 in Threads per inch Pich daimeter p 0.250 in L 0.250 in Nt 4 in
1

14.5 deg
p 1 Nt

d p 1.250 in

Determine the thread pitch and lead.

L p 8.

Use equations 15.5 to determine the lifting (up) and lowering (down) torques. Tu P d p 2 P d p 2

dp L cos( ) dp cos( ) L dp L cos( ) dp cos( ) L

c P

dc 2 dc 2

Tu 6319 in lbf

Td

c P

Td 3060 in lbf

9.

Use equation 15.7c to determine the lifting (up) and lowering (down) efficiencies. eu ed P L 2 Tu P L 2 Td eu 25.2 % ed 52.0 %

10. Use equation 15.6a to determine if the screw is self-locking. self_locking return "yes" if "no" otherwise self_locking "yes" 11. Check the safety factor against thread shear by stripping using equations 15.8. The area factor for thread stripping is given in Table 15-5 as wi 0.77 for the minor diameter of an Acme thread. Minor diameter Total shear area Shear stress d r 0.708 in As d r L Lnut Nt As 4.448 in
2

d p

cos( )

P As

s 8.99 ksi

MACHINE DESIGN - An Integrated Approach, 4th Ed.


Ns 0.577 S yscrew Ns 7.5

15-16-3

Factor of safety

12. Find a suitable length and diameter for the handle using a safety factor against yielding of one at the handle design load. Bending moment Handle length M Tu Lhandle M Fhandle M 6319 in lbf Lhandle 70.3 in

Section modulus

M S yhandle
1 3

Z 0.1109 in

Handle diameter

32 Z d

d 1.041 in

Use a handle with diameter

d 1.00 in

However, the length is unrealistic (nearly 6 feet).

MACHINE DESIGN - An Integrated Approach, 4th Ed.

15-17a-1

PROBLEM 15-17a
Statement: Determine the effective spring constant of an aluminum, copper-asbestos, steel sandwich under compressive load. It is uniformly loaded over its 10-cm2 area. The first and third members are each 10 mm thick and the middle member is 1 mm thick, making a total thickness of 21 mm. Which material dominates the calculation? Thicknesses t1 10 mm t2 1 mm t3 10 mm Area Solution: 1. A 10 cm
2

Given:

Modulus

E1 71.7 GPa E2 93 GPa E3 206.8 GPa

See Mathcad file P1517a.

Member stiffnesses Aluminum (1) k1 A E1 t1 A E2 t2 A E3 t3 k1 7.170 10


9 N

Copper-asbestos (2)

k2

k2 9.300 10

10 N

Steel (3)

k3

k3 2.068 10

10 N

2.

Sandwich stiffness: ktot k1 1


1 1 1

ktot 5.04 10

9 N

k2

k3

3.

The aluminum dominates the calculation.

MACHINE DESIGN - An Integrated Approach, 4th Ed.

15-18a-1

PROBLEM 15-18a
Statement: Determine the effective spring constant of an aluminum, copper-asbestos, steel sandwich under compressive load. It is uniformly loaded over its 1.5-in 2 area. The first and third members are each 0.4 in thick and the middle member is 0.04 in thick, making a total thickness of 0.84 in. Which material dominates the calculation? Thicknesses t1 0.40 in t2 0.04 in t3 0.40 in Area Solution: 1. A 1.5 in
2

Given:

Modulus

E1 10.4 10 psi E2 13.5 10 psi E3 30 10 psi


6 6

See Mathcad file P1518a.

Member stiffnesses Aluminum (1) k1 A E1 t1 A E2 t2 A E3 t3 k1 3.900 10


7 lbf

in

Copper-asbestos (2)

k2

k2 5.063 10

8 lbf

in

Steel (3)

k3

k3 1.125 10

8 lbf

in

2.

Sandwich stiffness: ktot k1 1


1 1 1

ktot 2.74 10

7 lbf

k2

k3

in

3.

The aluminum dominates the calculation.

MACHINE DESIGN - An Integrated Approach, 4th Ed.

15-19a-1

PROBLEM 15-19a
Statement:

_____

A preloaded steel bolt similar to that shown in Figure 15-31(a) clamps two flanges of total thickness l. Using the data given in the row(s) assigned in Table P15-1, find the joint stiffness constant. Bolt specification: M8 x 1 Bolt diameter d 8 mm Member material matl "steel" Clamped length l 30 mm Bolt modulus Ebolt 206.8 GPa Ememb return 206.8 GPa if matl = "steel" return 71.0 GPa if matl = "alum"

Given:

Member modulus of elasticity function:

Solution: 1.

See Figure 15-31(a), Table P15-1, Table P15-8, and Mathcad file P1519a.

Determine the relevant ratios for this joint from equations 15-18a and b. Joint aspect ratio: j r d l Ememb Ebolt j 0.267 r 1.000

Plate to bolt modulus:

2.

Calculate Cr = C using equation 15.19 and the coefficients p i from Table 15-8 for j 0.267 . Use linear interpolation between ja 0.2 and jb 0.3 Interpolation equation: p p a p b pb pa jb ja j ja p a p 0 0.6661 p 1 1.2189 p 2 1.1076 p 3 0.3832 C 0.172

Coefficients from Table 15-8:

p 0 p ( 0.6118 0.6932) p 1 p ( 1.1715 1.2426) p 2 p ( 1.0875 1.1177) p 3 p ( 0.3806 0.3845)

Joint stiffness constant:

C p 3 r p 2 r p 1 r p 0

MACHINE DESIGN - An Integrated Approach, 4th Ed.

15-20-1

PROBLEM 15-20
Statement: A single-cylinder air compressor head sees forces that range from 0 to 18.5 kN each cycle. The head is 80-mm-thick aluminum, the unconfined gasket is 1-mm-thick Teflon, and the block is aluminum. The effective clamp length of the cap screw is 120 mm. The piston is 75 mm dia and the cylinder is 140 mm outside dia. Specify a suitable number, class, preload, and bolt circle for the cylinder head cap screws to give a minimum safety factor of 1.2 for any possible failure mode. Total load Clamp length Piston diameter Cylinder diameter Design safety factor Ptot 18.5 kN l 120 mm id 75 mm od 140 mm Nd 1.2 Bolt diameter Number of bolts Preload fraction Bolt circle diameter d 12 mm Nbolts 8 fp 0.90 d bc 107.5 mm Gasket thickness Member mod. Bolt modulus Teflon mod. tg 1 mm Ememb 71.8 GPa Ebolt 206.8 GPa Eg 240 MPa

Given:

Design Choices: Use 10 M12 x 1.75 , class 8.8 cap screws. Material properties for class 8.8: Proof strength S p 600 MPa Yield strength Ultimate strength Solution: 1. See Mathcad file P1520. P Ptot Nbolts
2

S y 660 MPa S ut 830 MPa

Determine the load per bolt.

P 2313 N Pmax P

Pmin 0 N 2. 3. 4. Get the tensile stress area from Table 15-2. Calculate the preload. At 84.27 mm Fi fp S p At

Fi 45.51 kN

Determine the relevant ratios for the joint without the gasket from equations 15-18a and b. Joint aspect ratio: j r d l Ememb Ebolt j 0.1 r 0.347

Plate to bolt modulus: 5.

Calculate Cr = C using equation 15.19 and the coefficients p i from Table 15-8 for j 0.1 . Coefficients from Table 15-8: p 0 0.4389 p 1 0.9197 p 2 0.8901 p 3 0.3187 Joint stiffness constant: C p 3 r p 2 r p 1 r p 0
3 2

C 0.214

6.

We can estimate the stiffness of the bolt kb' from equation 15.17 using its tensile stress area from Table 15-1 and then estimate the material stiffness km for the no gasket case by using the expression for C in equation 15.13c, given kb' and C. Estimated bolt stiffness kb' At Ebolt l 1 d l
1

kb' 132.0

kN mm

MACHINE DESIGN - An Integrated Approach, 4th Ed.

15-20-2

Estimated member stiffness 7.

km kb'

( 1 C) C

km 486

kN mm

Now we will address the unconfined gasket. The bolt stiffness is not affected by the gasket but the material stiffness is. We now have two springs in series, the metal, whose stiffness is calculated above, and the gasket. These combine according to equation 14.2b. The portion of the unconfined gasket subjected to the clamp force can be assumed to be from the outside diameter of the flange shown in Figure 15-33 to the inside diameter of the vessel. The bolt hole should be subtracted from the gasket area. The area of the clamped gasket around one bolt is: Gasket area per bolt Ag

4 Nbolts

od id Nbolts d
2 2

Ag 1259 mm

8.

The stiffness of this piece of gasket material is found from equation 15.11c, Gasket stiffness per bolt kg Ag Eg tg kg 302.1 kN mm

9.

The combined stiffness of the gasketed joint (from equation 14.2b) is Let the aluminum and gasket stiffnesses be km1 km and km2 kg Combined gasket and cover stiffness km 1 1 km1 1 km2 km 186.3 kN mm

10. The joint constant with the unconfined gasket is now Joint constant with unconfined gasket and C kb' km kb' C 0.415

( 1 C) 0.585

11. The portions of the applied load P felt by the bolt and the material can now be found from equations 15.13. Pb C P Pm ( 1 C) P Pb 958.97 N Pm 1353.5 N

12. Find the resulting loads in bolt and material after the load P is applied. Fb Fi Pb Fb 46.46 kN Fm Fi Pm Fm 44.15 kN

13. Calculate the alternating and mean components of the fluctuating bolt load. Falt Fb Fi 2 Fb Fi 2 Falt 0.479 kN Fmean 45.99 kN

Fmean

14. Because these loads are fluctuating, we need to calculate the mean and alternating components of the force felt in the bolt. The mean and alternating forces are Mean force Fmean Fb Fi 2 Fmean 46.0 kN

MACHINE DESIGN - An Integrated Approach, 4th Ed.

15-20-3

Alternating force

Falt

Fb Fi 2

Falt 479 N

15. The nominal mean and alternating stresses in the bolt are: Nominal mean stress

mnom anom

Fmean At Falt At

mnom 545.7 MPa anom 5.69 MPa

Nominal alternating stress

16. The fatigue stress-concentration factor for this thread is found from equation 15.15c and the mean stress-concentration factor factor Kfm is found from equation 6.17. Fatigue factor Nominal maximum stress Nominal minimum stress Kfm Kf 5.7 0.02682 d mm Kf 6

maxnom mnom anom minnom mnom anom

maxnom 551.4 MPa minnom 540.0 MPa

return Kf if Kf maxnom S y return S y Kf anom if Kf maxnom S y Kfm 1.147

mnom

return 0 if Kf maxnom minnom 2 S y 17. The local mean and alternating stresses in the bolt are then: Local mean stress Local alternating stress 18. The stress at the initial preload is

m Kfm mnom a Kf anom

m 625.7 MPa a 34.3 MPa

init Kfm

Fi At

init 619.21 MPa

19. An endurance strength must be found for this material. Using the methods of Section 6.6 we find for S ut 830 MPa S'e 0.5 S ut S'e 415.0 MPa

20. From the tables and formulas in Section 6.6 we have: Load Size Surface Cload 0.70 Csize 1 A 4.51 Csurf Temperature b 0.265
b

Sut A MPa

Csurf 0.76

Ctemp 1

MACHINE DESIGN - An Integrated Approach, 4th Ed.

15-20-4

Reliability and the endurance limit is

Creliab 0.814

S e Cload Csize Csurf Ctemp Creliab S'e

S e 179.64 MPa

21. The corrected endurance strength and the ultimate tensile strength are used in equation 15.16 to find the safety factor from the Goodman line. Nf S e m init S ut a S e S ut init Nf 1.3

22. Calculate the maximum bolt stress and the safety factor against yielding for S y 660 MPa.

b
Ny

Fb At Sy

b 551.38 MPa
Ny 1.2

23. The load required to separate the joint and the safety factor against joint separation are found from equations 15.14c and 15.14d. Nsep Fi Pmax ( 1 C) Nsep 34

24. Check the screw spacing, comparing it to the rules given in the text. Bolt spacing space bolt ratio

d bc
Nbolts

space bolt 42.2 mm ratio 3.5

Space/dia ratio

space bolt d

which is OK

MACHINE DESIGN - An Integrated Approach, 4th Ed.

15-21-1

PROBLEM 15-21
Statement: A single-cylinder engine head sees forces that range from 0 to 4000 lb each cycle. The head is 2.5-in-thick cast iron, the unconfined gasket is 0.125-in-thick copper-asbestos, and the block is cast iron. The effective clamp length of the cap screw is 3.125 in. The piston is 3 in dia and the cylinder is 5.5 in outside dia. Specify a suitable number, class, preload, and bolt circle for the cylinder head cap screws to give a minimum safety factor of 1.2 for any possible failure mode. Total load Clamp length Piston diameter Cylinder dia. Design safety factor Ptot 4000 lbf l 3.125 in id 3.00 in od 5.50 in Nd 1.2 Gasket thickness tg 0.125 in Member mod. Bolt modulus Ememb 15 10 psi Ebolt 30 10 psi
6 6 6

Given:

Gasket modulus Eg 13.5 10 psi

Design Choices: Use 8 5/16-18 UNC , grade 5 cap screws. Material properties for grade 5: Proof strength S p 85 ksi Yield strength Ultimate strength Solution: 1. See Mathcad file P1521. P Ptot Nbolts
2

Bolt diameter d 0.3125 in Number of bolts Nbolts 8 Preload fraction fp 0.90 Bolt circle diameter d bc 4.250 in

S y 92 ksi S ut 120 ksi

Determine the load per bolt.

P 500 lbf Pmax P

Pmin 0 lbf 2. 4. 4. Get the tensile stress area from Table 15-2. Calculate the preload. At 0.0524 in Fi fp S p At

Fi 4009 lbf

Determine the relevant ratios for the joint without the gasket from equations 15-18a and b. Joint aspect ratio: j r d l Ememb Ebolt j 0.1 r 0.5

Plate to bolt modulus: 5.

Calculate Cr = C using equation 15.19 and the coefficients p i from Table 15-8 for j 0.1 . Coefficients from Table 15-8: p 0 0.4389 p 1 0.9197 p 2 0.8901 p 3 0.3187 Joint stiffness constant: C p 3 r p 2 r p 1 r p 0
3 2

C 0.162

6.

We can estimate the stiffness of the bolt kb' from equation 15.17 using its tensile stress area from Table 15-1 and then estimate the material stiffness km for the no gasket case by using the expression for C in equation 15.13c, given kb' and C.

MACHINE DESIGN - An Integrated Approach, 4th Ed.

15-21-2

Estimated bolt stiffness

kb'

At Ebolt l

d 1 l

kb' 457.3 km 2370

kip in

Estimated member stiffness 7.

km kb'

( 1 C) C

kip in

Now we will address the unconfined gasket. The bolt stiffness is not affected by the gasket but the material stiffness is. We now have two springs in series, the metal, whose stiffness is calculated above, and the gasket. These combine according to equation 14.2b. The portion of the unconfined gasket subjected to the clamp force can be assumed to be from the outside diameter of the flange shown in Figure 15-33 to the inside diameter of the vessel. The bolt hole should be subtracted from the gasket area. The area of the clamped gasket around one bolt is: Gasket area per bolt Ag

4 Nbolts Ag Eg tg

od id Nbolts d
2 2

Ag 2.01 in

8.

The stiffness of this piece of gasket material is found from equation 15.11c, Gasket stiffness per bolt kg kg 217028 kip in

9.

The combined stiffness of the gasketed joint (from equation 14.2b) is Let the cast iron and gasket stiffnesses be km1 km and km2 kg Combined gasket and head stiffness km 1 1 km1 1 km2 km 2345 kip in

10. The joint constant with the unconfined gasket is now Joint constant with unconfined gasket C kb' km kb' C 0.163

11. The portions of the applied load P felt by the bolt and the material can now be found from equations 15.13. Pb C P Pm ( 1 C) P Fb Fi Pb Fm Fi Pm Pb 81.61 lbf Pm 418.4 lbf Fb 4090 lbf Fm 3590 lbf

12. Find the resulting loads in bolt and material after the load P is applied.

13. Calculate the alternating and mean components of the fluctuating bolt load. Falt Fb Fi 2 Fb Fi 2 Falt 40.80 lbf Fmean 4.05 kip

Fmean

14. The nominal mean and alternating stresses in the bolt are:

MACHINE DESIGN - An Integrated Approach, 4th Ed.

15-21-3

Nominal mean stress

mnom anom

Fmean At Falt At

mnom 77.3 ksi anom 779 psi

Nominal alternating stress

15. The fatigue stress-concentration factor for this thread is found from equation 15.15c and the mean stress-concentration factor factor Kfm is found from equation 6.17. Fatigue factor Nominal maximum stress Nominal minimum stress Kfm Kf 5.7 0.02682 d mm Kf 5.9

maxnom mnom anom minnom mnom anom

maxnom 538.2 MPa minnom 527.4 MPa

return Kf if Kf maxnom S y return S y Kf anom if Kf maxnom S y Kfm 1.131

mnom

return 0 if Kf maxnom minnom 2 S y 16. The local mean and alternating stresses in the bolt are then: Local mean stress Local alternating stress 17. The stress at the initial preload is

m Kfm mnom a Kf anom

m 602.6 MPa a 31.7 MPa

init Kfm

Fi At

init 86.51 ksi

18. An endurance strength must be found for this material. Using the methods of Section 6.6 we find for S ut 120 ksi S'e 0.5 S ut S'e 60.0 ksi

19. From the tables and formulas in Section 6.6 we have: Load Size Surface Cload 0.70 Csize 1 A 2.7 Csurf Temperature Reliability and the endurance limit is S e Cload Csize Csurf Ctemp Creliab S'e S e 25.957 ksi b 0.265
b

S ut A ksi

Csurf 0.759

Ctemp 1 Creliab 0.814

MACHINE DESIGN - An Integrated Approach, 4th Ed.

15-21-4

20. The corrected endurance strength and the ultimate tensile strength are used in equation 15.16 to find the safety factor from the Goodman line. Nf S e m init S ut a S e S ut init Nf 1.5

21. Calculate the maximum bolt stress and the safety factor against yielding for S y 92 ksi.

b
Ny

Fb At Sy

b 78.06 ksi
Ny 1.2

22. The load required to separate the joint and the safety factor against joint separation are found from equations 15.14c and 15.14d. Nsep Fi Pmax ( 1 C) Nsep 10

23. Check the screw spacing, comparing it to the rules given in the text. Bolt spacing space bolt ratio

d bc
Nbolts

space bolt 1.67 in ratio 5.3

Space/dia ratio

space bolt d

which is OK

MACHINE DESIGN - An Integrated Approach, 4th Ed.

15-22-1

PROBLEM 15-22
Statement: The forged steel connecting rod for the engine of problem 15-21 is split around the 38-mm-dia crankpin and fastened with two bolts and nuts that hold its two halves together. The total load on the two bolts varies from 0 to 8.5 kN each cycle. Design these bolts for infinite life. Specify their size, class, and preload. Total force Number of bolts Ptot 8.5 kN Nbolts 2 Member mod. Bolt modulus Ememb 206.8 GPa Ebolt 206.8 GPa

Given:

Design Choices: Use M12 x 1.25 , class 9.8 bolts. Material properties for class 9.8: Proof strength S p 650 MPa Yield strength Ultimate strength Solution: 1. See Mathcad file P1522. S y 720 MPa S ut 900 MPa

Bolt diameter Preload fraction Design safety factor Clamp length

d 12 mm fp 0.70 Nd 1.5 l 2.5 d

Determine the load per bolt. P Ptot Nbolts P 4.25 kN Pmax P At 92.07 mm Fi 41.9 kN
2

Pmin 0 N 2. 3. 4. Get the tensile stress area from Table 15-2. Calculate the preload. Fi fp S p At

Determine the relevant ratios for this joint from equations 15-18a and b. Joint aspect ratio: j r d l Ememb Ebolt j 0.400 r1

Plate to bolt modulus: 5.

Calculate Cr = C using equation 15.19 and the coefficients p i from Table 15-8 for j 0.4 . Coefficients from Table 15-8: p 0 0.7351 p 1 1.2612 p 2 1.1111 p 3 0.3779 Joint stiffness constant: C p 3 r p 2 r p 1 r p 0
3 2

C 0.207

6.

The portions of the applied load P felt by the bolt and the material can now be found from equations 15.13. Pb C P Pm ( 1 C) P Pb 0.88 kN Pm 3.4 kN Fb 42.77 kN

7.

Find the resulting loads in bolt and material after the load P is applied. Fb Fi Pb

MACHINE DESIGN - An Integrated Approach, 4th Ed.

15-22-2

Fm Fi Pm 8.

Fm 38.52 kN

Calculate the alternating and mean components of the fluctuating bolt load. Falt Fb Fi 2 Fb Fi 2 Falt 0.440 kN Fmean 42.33 kN

Fmean 9.

The nominal mean and alternating stresses in the bolt are: Nominal mean stress

mnom

Fmean At Falt At

mnom 459.8 MPa

Nominal alternating stress

anom

anom 4.78 MPa

10. The fatigue stress-concentration factor for this thread is found from equation 15.15c and the mean stress-concentration factor factor Kfm is found from equation 6.17. Fatigue factor Nominal maximum stress Nominal minimum stress Kfm Kf 5.7 0.02682 d mm Kf 6.0

maxnom mnom anom minnom mnom anom

maxnom 67.379 ksi minnom 65.992 ksi

return Kf if Kf maxnom S y return S y Kf anom if Kf maxnom S y Kfm 1.503

mnom

return 0 if Kf maxnom minnom 2 S y 11. The local mean and alternating stresses in the bolt are then: Local mean stress Local alternating stress 12. The stress at the initial preload is

m Kfm mnom a Kf anom

m 691.2 MPa a 28.78 MPa

init Kfm

Fi At

init 684.0 MPa

13. An endurance strength must be found for this material. Using the methods of Section 6.6 we find for S ut 900 MPa S'e 0.5 S ut 14. From the tables and formulas in Section 6.6 we have: Load Size Surface Cload 0.70 Csize 1 A 4.51 b 0.265 S'e 450.0 MPa

MACHINE DESIGN - An Integrated Approach, 4th Ed.


b

15-22-3

Csurf A Temperature Reliability and the endurance limit is Ctemp 1

Sut MPa

Csurf 0.744

Creliab 0.814

S e Cload Csize Csurf Ctemp Creliab S'e

S e 190.65 MPa

15. The corrected endurance strength and the ultimate tensile strength are used in equation 15.16 to find the safety factor from the Goodman line. Nf S e m init S ut a S e S ut init Nf 1.5

16. Calculate the maximum bolt stress and the safety factor against yielding for S y 720 MPa.

b
Ny

Fb At Sy

b 464.6 MPa
Ny 1.5

17. The load required to separate the joint and the safety factor against joint separation are found from equations 15.14c and 15.14d. Nsep Fi Pmax ( 1 C) Nsep 12.4

MACHINE DESIGN - An Integrated Approach, 4th Ed.

15-23a-1

PROBLEM 15-23a
Statement: (See also Problem 4-33.) The bracket in Figure P15-2 is fastened to the wall by 4 cap screws equispaced on a 10-cm-dia bolt circle and arranged as shown. The wall is the same material as the bracket. The bracket is subjected to a static force F, where F and the beam's other data are given in row a in Table P15-1. Find the forces acting on each of the 4 cap screws due to this loading and choose a suitable cap screw diameter, length, and preload that will give a minimum safety factor of 2 for any possible mode of failure. Tube length Arm length Wall plate thickness Bolt circle diameter Design safety factor Bolt modulus Clamp length L 100 mm a 400 mm t 10 mm d bc 100 mm Nd 2 Ebolt E l 2 t Member modulus Ememb E Applied force Modulus of elasticity Number of bolts (top or bottom) F 50 N E 207 GPa Nbolts 2

Given:

Assumptions: 1. The cap screws and wall provide the couple shown in the FBD of Figure 15-23. This is an idealization, but is conservative. 2. The torsion reaction and the vertical reaction at the wall are provided by frictional forces between the wall and the wall plate on the bracket and the screws are subjected to the horizontal forces only. Design Choices: Use M5 X 0.8 , class 4.6 cap screws. Material properties for class 4.6: Proof strength S p 225 MPa Yield strength S y 240 MPa

Cap screw diameter Preload fraction

d 5 mm fp 0.54

y dbc Ptot d T Ptot T x F

L R

FIGURE 15-23
Free Body Diagram of Tube and Wall Plate for Problem 15-23

Solution: 1.

See Figure 15-23 and Mathcad file P1523a. Ptot d' F L = 0

Calculate the magnitude of the couple in Figure 15-23 by summing moments. Distance d' Total screw force d' d bc sin( 45 deg) Ptot F L d' Ptot Nbolts d' 70.711 mm Ptot 70.71 N P 35.36 N

2.

Determine the load per screw.

MACHINE DESIGN - An Integrated Approach, 4th Ed.

15-23a-2

3. 4. 5.

Get the tensile stress area from Table 15-2. Calculate the preload. Fi fp S p At

At 14.18 mm Fi 1.72 kN

Determine the relevant ratios for this joint from equations 15-18a and b. Joint aspect ratio: j r d l Ememb Ebolt j 0.25 r1

Plate to bolt modulus: 6.

Calculate Cr = C using equation 15.19 and the coefficients p i from Table 15-8 for j 0.25 . Use linear interpolation between ja 0.2 and jb 0.3 Interpolation equation: p p a p b pb pa jb ja j ja p a p 0 0.6525 p 1 1.2070 p 2 1.1026 p 3 0.3825 C 0.165

Coefficients from Table 15-8:

p 0 p ( 0.6118 0.6932) p 1 p ( 1.1715 1.2426) p 2 p ( 1.0875 1.1177) p 3 p ( 0.3806 0.3845)

Joint stiffness constant: 7.

C p 3 r p 2 r p 1 r p 0

The portions of the applied load P felt by the screw and the material can now be found from equations 15.13. Pb C P Pm ( 1 C) P Pb 5.85 N Pm 29.5 N Fb 1.73 kN Fm 1.69 kN

8.

Find the resulting loads in screw and material after the load P is applied. Fb Fi Pb Fm Fi Pm

9.

The maximum tensile stress in the screw is

Fb At

b 121.9 MPa

10. This is a uniaxial stress situation, so the principal stress and von Mises stress are identical to the applied tensile stress. The safety factor against yielding for class 4.6 with S y 240 MPa is then Ny Sy Ny 2.0

11. The load required to separate the joint and the safety factor against joint separation are found from equations 15.14c and 15.14d. P0 Fi 1C P0 P P0 2.1 kN Nsep 58

Nsep

MACHINE DESIGN - An Integrated Approach, 4th Ed.

15-24a-1

PROBLEM 15-24a
Statement: (See also Problem 6-33.) The bracket in Figure P15-2 is fastened to the wall by 4 cap screws equispaced on a 10-cm-dia bolt circle and arranged as shown. The wall is the same material as the bracket. The bracket is subjected to a sinusoidal force time function with Fmax = F, and Fmin = -F, where F and the beam's other data are given in row a in Table P15-1. Find the forces acting on each of the 4 cap screws due to this loading and choose a suitable cap screw diameter, length, and preload that will give a minimum safety factor of 1.5 for any possible mode of failure for N = 5E8 cycles. Tube length Arm length Wall plate thickness Bolt circle diameter Design safety factor L 100 mm a 400 mm t 10 mm d bc 100 mm Nd 1.5 Applied force Modulus (top or bottom) F 50 N Ememb 207 GPa

Given:

Number of bolts Nbolts 2

Assumptions: 1. The cap screws and wall provide the couple shown in the FBD of Figure 15-23. This is an idealization, but is conservative. 2. The torsion reaction and the vertical reaction at the wall are provided by frictional forces between the wall and the wall plate on the bracket and the screws are subjected to the horizontal forces only. Design Choices: Use M4 x 0.7 , class 4.8 cap screws. Material properties for class 4.8: Cap screw diameter d 4 mm Clamp length Proof strength S p 310 MPa l 16 mm Yield strength Ultimate strength S y 340 MPa S ut 420 MPa Preload fraction Modulus fp 0.75 Ebolt 207 GPa

y dbc Ptot d T Ptot T x F

L R

FIGURE 15-24
Free Body Diagram of Tube and Wall Plate for Problem 15-24

Solution: 1.

See Figure 15-24 and Mathcad file P1524a. Ptot d' F L = 0

Calculate the magnitude of the couple in Figure 15-24 by summing moments. Distance d' Total screw force d' d bc sin( 45 deg) Ptot F Ptot Nbolts L d' d' 70.711 mm Ptot 70.71 N P 35.36 N

2.

Determine the load per screw. P

MACHINE DESIGN - An Integrated Approach, 4th Ed.


Pmin P 3. 4. 5. Get the tensile stress area from Table 15-2. Calculate the preload. Fi fp S p At Pmax P At 8.78 mm Fi 2.04 kN
2

15-24a-2

Determine the relevant ratios for this joint from equations 15-18a and b. Joint aspect ratio: j r d l Ememb Ebolt j 0.25 r1

Plate to bolt modulus:

6.

Calculate Cr = C using equation 15.19 and the coefficients p i from Table 15-8 for j 0.25 . Use linear interpolation between ja 0.2 and jb 0.3 Interpolation equation: p p a p b pb pa jb ja j ja p a p 0 0.6525 p 1 1.2070 p 2 1.1026 p 3 0.3825 C 0.165

Coefficients from Table 15-8:

p 0 p ( 0.6118 0.6932) p 1 p ( 1.1715 1.2426) p 2 p ( 1.0875 1.1177) p 3 p ( 0.3806 0.3845)

Joint stiffness constant: 7.

C p 3 r p 2 r p 1 r p 0

The portions of the applied load P felt by the screw and the material can now be found from equations 15.13. Pb C P Pm ( 1 C) P Pb 5.85 N Pm 29.5 N Fb 2.05 kN Fm 2.01 kN

8.

Find the resulting loads in screw and material after the load P is applied. Fb Fi Pb Fm Fi Pm

9.

Calculate the alternating and mean components of the fluctuating screw load. Falt Fb Fi 2 Fb Fi 2 Falt 2.926 N Fmean 2.04 kN

Fmean

10. The nominal mean and alternating stresses in the bolt are: Nominal mean stress

mnom

Fmean At

mnom 232.8 MPa

MACHINE DESIGN - An Integrated Approach, 4th Ed.

15-24a-3

Nominal alternating stress

anom

Falt At

anom 0.333 MPa

11. The fatigue stress-concentration factor for this thread is found from equation 15.15c and the mean stress-concentration factor factor Kfm is found from equation 6.17. Fatigue factor Nominal maximum stress Nominal minimum stress Kfm Kf 5.7 0.6812 d in Kf 5.8

maxnom mnom anom minnom mnom anom

maxnom 233.2 MPa minnom 232.5 MPa

return Kf if Kf maxnom S y return S y Kf anom if Kf maxnom S y Kfm 1.452

mnom

return 0 if Kf maxnom minnom 2 S y 12. The local mean and alternating stresses in the bolt are then: Local mean stress Local alternating stress 13. The stress at the initial preload is

m Kfm mnom a Kf anom

m 338.1 MPa a 1.935 MPa

init Kfm

Fi At

init 337.58 MPa

14. An endurance strength must be found for this material. Using the methods of Section 6.6 we find for S ut 420 MPa S'e 0.5 S ut 15. From the tables and formulas in Section 6.6 we have: Load Size Surface Cload 0.70 Csize 1 A 4.51 Csurf Temperature Reliability and the endurance limit is S e Cload Csize Csurf Ctemp Creliab S'e S e 108.88 MPa b 0.265
b

S'e 210.0 MPa

Sut A MPa

Csurf 0.91

Ctemp 1 Creliab 0.814

16. The corrected endurance strength and the ultimate tensile strength are used in equation 15.16 to find the safety factor from the Goodman line.

MACHINE DESIGN - An Integrated Approach, 4th Ed.


S e S ut init

15-24a-4

Nf

S e m init S ut a

Nf 10

17. Calculate the maximum bolt stress and the safety factor against yielding for S y 340 MPa.

b
Ny

Fb At Sy

b 233.166 MPa
Ny 1.5

18. The load required to separate the joint and the safety factor against joint separation are found from equations 15.14c and 15.14d. Fi Pmax ( 1 C)

Nsep

Nsep 69

MACHINE DESIGN - An Integrated Approach, 4th Ed.

15-25a-1

PROBLEM 15-25a
Statement: (See also Problem 6-34.) The bracket in Figure P15-2 is fastened to the wall by 4 cap screws equispaced on a 10-cm-dia bolt circle and arranged as shown. The wall is the same material as the bracket. The bracket is subjected to a sinusoidal force time function with Fmax = F, and Fmin = 0, where F and the beam's other data are given in row a in Table P15-1. Find the forces acting on each of the 4 cap screws due to this loading and choose a suitable cap screw diameter, length, and preload that will give a minimum safety factor of 1.5 for any possible mode of failure for N = 5E8 cycles. Applied force Tube length L 100 mm F 50 N Modulus Arm length a 400 mm Ememb 207 GPa Wall plate thickness Bolt circle diameter Design safety factor t 10 mm d bc 100 mm Nd 1.5 Number of bolts Nbolts 2 (top or bottom)

Given:

Assumptions: 1. The cap screws and wall provide the couple shown in the FBD of Figure 15-23. This is an idealization, but is conservative. 2. The torsion reaction and the vertical reaction at the wall are provided by frictional forces between the wall and the wall plate on the bracket and the screws are subjected to the horizontal forces only. Design Choices: Use M4 x 0.7 , class 4.8 cap screws. Material properties for class 4.8: Cap screw diameter d 4 mm Clamp length Proof strength S p 310 MPa l 16 mm Yield strength Ultimate strength S y 340 MPa S ut 420 MPa Preload fraction Modulus fp 0.75 Ebolt 207 GPa

y dbc Ptot d T Ptot T x F

L R

FIGURE 15-25
Free Body Diagram of Tube and Wall Plate for Problem 15-25

Solution: 1.

See Figure 15-25 and Mathcad file P1525a. Ptot d F L = 0

Calculate the magnitude of the couple in Figure 15-25 by summing moments. Distance d' Total screw force d' d bc sin( 45 deg) Ptot F L d' Ptot Nbolts d' 70.711 mm Ptot 70.71 N P 35.36 N

2.

Determine the load per screw.

MACHINE DESIGN - An Integrated Approach, 4th Ed.

15-25a-2

Pmin 0 N 3. 4. 5. Get the tensile stress area from Table 15-2. Calculate the preload. Fi fp S p At

Pmax P At 8.78 mm Fi 2.04 kN


2

Determine the relevant ratios for this joint from equations 15-18a and b. Joint aspect ratio: j r d l Ememb Ebolt j 0.25 r1

Plate to bolt modulus:

6.

Calculate Cr = C using equation 15.19 and the coefficients p i from Table 15-8 for j 0.25 . Use linear interpolation between ja 0.2 and jb 0.3 Interpolation equation: p p a p b pb pa jb ja j ja p a p 0 0.6525 p 1 1.2070 p 2 1.1026 p 3 0.3825 C 0.165

Coefficients from Table 15-8:

p 0 p ( 0.6118 0.6932) p 1 p ( 1.1715 1.2426) p 2 p ( 1.0875 1.1177) p 3 p ( 0.3806 0.3845)

Joint stiffness constant: 7.

C p 3 r p 2 r p 1 r p 0

The portions of the applied load P felt by the screw and the material can now be found from equations 15.13. Pb C P Pm ( 1 C) P Pb 5.85 N Pm 29.5 N Fb 2.05 kN Fm 2.01 kN

8.

Find the resulting loads in screw and material after the load P is applied. Fb Fi Pb Fm Fi Pm

9.

Calculate the alternating and mean components of the fluctuating screw load. Falt Fb Fi 2 Fb Fi 2 Falt 2.926 N Fmean 2.04 kN

Fmean

10. The nominal mean and alternating stresses in the bolt are: Nominal mean stress

mnom

Fmean At

mnom 232.8 MPa

MACHINE DESIGN - An Integrated Approach, 4th Ed.

15-25a-3

Nominal alternating stress

anom

Falt At

anom 0.333 MPa

11. The fatigue stress-concentration factor for this thread is found from equation 15.15c and the mean stress-concentration factor factor Kfm is found from equation 6.17. Fatigue factor Nominal maximum stress Nominal minimum stress Kfm Kf 5.7 0.6812 d in Kf 5.8

maxnom mnom anom minnom mnom anom

maxnom 233.2 MPa minnom 232.5 MPa

return Kf if Kf maxnom S y return S y Kf anom if Kf maxnom S y Kfm 1.452

mnom

return 0 if Kf maxnom minnom 2 S y 12. The local mean and alternating stresses in the bolt are then: Local mean stress Local alternating stress 13. The stress at the initial preload is

m Kfm mnom a Kf anom

m 338.1 MPa a 1.935 MPa

init Kfm

Fi At

init 337.58 MPa

14. An endurance strength must be found for this material. Using the methods of Section 6.6 we find for S ut 420 MPa S'e 0.5 S ut 15. From the tables and formulas in Section 6.6 we have: Load Size Surface Cload 0.70 Csize 1 A 4.51 Csurf A Temperature Reliability Ctemp 1 Creliab 0.814 b 0.265
b

S'e 210.0 MPa

Sut MPa

Csurf 0.91

and the endurance limit is S e Cload Csize Csurf Ctemp Creliab S'e S e 108.88 MPa

16. The corrected endurance strength and the ultimate tensile strength are used in equation 15.16 to find the safety factor from the Goodman line.

MACHINE DESIGN - An Integrated Approach, 4th Ed.

15-25a-4

Nf

S e m init S ut a

S e S ut init

Nf 10

17. Calculate the maximum bolt stress and the safety factor against yielding for S y 340 MPa.

b
Ny

Fb At Sy

b 233.2 MPa
Ny 1.5

18. The load required to separate the joint and the safety factor against joint separation are found from equations 15.14c and 15.14d. Fi Pmax ( 1 C)

Nsep

Nsep 69

MACHINE DESIGN - An Integrated Approach, 4th Ed.

15-26-1

PROBLEM 15-26
Statement: (See also Problem 6-42.) A cylindrical tank with hemispherical ends is required to hold 425 kPa of pressurized air at room temperature. The pressure cycles from zero to maximum. The tank diameter is 0.5 m and its length is 1 m. The hemispherical ends are attached by some number of bolts through mating flanges on each part of the tank. A 0.5-mm-thick, compressed asbestos, unconfined gasket is used between the 10-mm-thick steel flanges. Determine a suitable number, class, preload for, and size of bolts to fasten the ends of the tank. Specify the bolt circle and outside diameter of the flange needed to prevent leakage. A minimum safety factor of 2 is desired against leakage and a safety factor of 1.5 against bolt failure for infinite life. Maximum pressure Flange thickness Tank diameter Leakage safety factor Bolt safety factor p max 425 kPa tf 10 mm id 500 mm Nsep 2 Nb 1.5 Gasket thickness Member mod. Bolt modulus Asbestos mod. tg 0.5 mm Ememb 206.8 GPa Ebolt 206.8 GPa Eg 480 MPa

Given:

Design Choices: Use 40 M16 x 1.5 , class 12.9 bolts. Material properties for class 10.9: Proof strength S p 970 MPa Yield strength Ultimate strength Solution: 1. See Mathcad file P1526. Ptot P S y 1100 MPa S ut 1220 MPa

Bolt diameter Number of bolts Preload fraction

d 16 mm Nbolts 40 fp 0.75

Determine the load per bolt.

id
4 Ptot

p max

Ptot 83.4 kN P 2086 N Pmax P At 167.25 mm Fi 121.67 kN


2

Nbolts

Pmin 0 N

2. 3. 4.

Get the tensile stress area from Table 15-2. Calculate the preload. Calculate the clamp length. Clamp length l 2 t f tg Fi fp S p At

l 20.5 mm

5.

Determine the relevant ratios for the joint without the gasket from equations 15-18a and b. Joint aspect ratio: j r d l Ememb Ebolt j 0.78 r1

Plate to bolt modulus: 6.

Calculate Cr = C using equation 15.19 and the coefficients p i from Table 15-8 for j 0.78 . Use linear interpolation between ja 0.7 and jb 0.8 Interpolation equation: Coefficients from Table 15-8: p p a p b pb pa jb ja j ja p a p 0 0.7795

p 0 p ( 0.7773 0.7800)

MACHINE DESIGN - An Integrated Approach, 4th Ed.


p 1 p ( 1.2543 1.2503) p 2 p ( 1.0735 1.0672) p 3 p ( 0.3595 0.3571) Joint stiffness constant: 7. C p 3 r p 2 r p 1 r p 0
3 2

15-26-2
p 1 1.2511 p 2 1.0684 p 3 0.3576 C 0.239

We can estimate the stiffness of the bolt kb' from equation 15.17 using its tensile stress area from Table 15-1 and then estimate the material stiffness km for the no gasket case by using the expression for C in equation 15.13c, given kb' and C. Estimated bolt stiffness kb' At Ebolt l 1 d
1

kb' 947.6 km 3013

kN mm

Estimated member stiffness 8.

km kb'

( 1 C) C

kN mm

Now we will address the unconfined gasket case. The bolt stiffness is not affected by the gasket but the material stiffness is. We now have two springs in series, the metal, whose stiffness is calculated above, and the gasket. These combine according to equation 14.2b. The portion of the unconfined gasket subjected to the clamp force can be assumed to be from the outside diameter of the flange shown in Figure 15-33 to the inside diameter of the vessel. The bolt hole should be subtracted from the gasket area. The area of the clamped gasket around one bolt is: Flange OD Gasket area per bolt od id 6 d Ag od 596 mm od id Nbolts d
2 2 2

4 Nbolts

Ag 1865 mm

9.

The stiffness of this piece of gasket material is found from equation 15.11c, where, from Table 15-10. Gasket stiffness per bolt kg Ag Eg tg kg 1790.3 kN mm

10. The combined stiffness of the gasketed joint (from equation 14.2b) is Let the steel and gasket stiffnesses be km1 km and km2 kg Combined gasket and cover stiffness km 1 1 km1 11. The joint constant with the unconfined gasket is now Joint constant with unconfined gasket and C kb' km kb' C 0.458 1 km2 km 1123.0 kN mm

( 1 C) 0.542

12. The portions of the applied load P felt by the bolt and the material can now be found from equations 15.13. Pb C P Pm ( 1 C) P Pb 0.95 kN Pm 1131.5 N

MACHINE DESIGN - An Integrated Approach, 4th Ed.


13. Find the resulting loads in bolt and material after the load P is applied. Fb Fi Pb Fm Fi Pm Fb 122.63 kN Fm 120.54 kN

15-26-3

14. Because these loads are fluctuating, we need to calculate the mean and alternating components of the force felt in the bolt. The mean and alternating forces are Mean force Fmean Falt Fb Fi 2 Fmean 122.2 kN Falt 477 N

Alternating force

Fb Fi 2

15. The nominal mean and alternating stresses in the bolt are: Nominal mean stress

mnom anom

Fmean At Falt At

mnom 730.4 MPa anom 2.85 MPa

Nominal alternating stress

16. The fatigue stress-concentration factor for this thread is found from equation 15.15c and the mean stress-concentration factor factor Kfm is found from equation 6.17. Fatigue factor Nominal maximum stress Nominal minimum stress Kfm Kf 5.7 0.02682 d mm Kf 6.1

maxnom mnom anom minnom mnom anom

maxnom 733.2 MPa minnom 727.5 MPa

return Kf if Kf maxnom S y return S y Kf anom if Kf maxnom S y Kfm 1.482

mnom

return 0 if Kf maxnom minnom 2 S y 17. The local mean and alternating stresses in the bolt are then: Local mean stress Local alternating stress 18. The stress at the initial preload is

m Kfm mnom a Kf anom

m 1082.5 MPa a 17.5 MPa

init Kfm

Fi At

init 1.08 10 MPa

19. An endurance strength must be found for this material. Using the methods of Section 6.6 we find for S ut 1220 MPa S'e 0.5 S ut S'e 610.0 MPa Cload 0.70 Csize 1

20. From the tables and formulas in Section 6.6 we have: Load Size

MACHINE DESIGN - An Integrated Approach, 4th Ed.


Surface A 4.51 Csurf Temperature Reliability and the endurance limit is S e Cload Csize Csurf Ctemp Creliab S'e S e 238.42 MPa b 0.265
b

15-26-4

Sut A MPa

Csurf 0.686

Ctemp 1 Creliab 0.814

21. The corrected endurance strength and the ultimate tensile strength are used in equation 15.16 to find the safety factor from the Goodman line. Nf S e m init S ut a S e S ut init Nf 1.5

22. Calculate the maximum bolt stress and the safety factor against yielding for S y 1100 MPa.

b
Ny

Fb At Sy

b 733.2 MPa
Ny 1.5

23. The load required to separate the joint and the safety factor against joint separation are found from equations 15.14c and 15.14d. Nsep Fi Pmax ( 1 C) Nsep 108

24. Check the bolt spacing, comparing it to the rule given in the text. Bolt circle dia Bolt spacing d bc 0.5 ( od id) space bolt ratio d bc 548 mm space bolt 43 mm ratio 2.7

d bc
Nbolts

Space/dia ratio

space bolt d

which is OK

MACHINE DESIGN - An Integrated Approach, 4th Ed.

15-27-1

PROBLEM 15-27
Statement: (See also Problem 6-42.) A cylindrical tank with hemispherical ends is required to hold 250 kPa of pressurized air at room temperature. The pressure cycles from zero to maximum. The tank diameter is 0.5 m and its length is 1 m. The hemispherical ends are attached by some number of bolts through mating flanges on each part of the tank. A confined o-ring gasket is used between the 10-mm-thick steel flanges. Determine a suitable number, class, preload for, and size of bolts to fasten the ends of the tank. Specify the bolt circle and outside diameter of the flange needed to prevent leakage. A minimum safety factor of 2 is desired against leakage and a safety factor of 1.5 against bolt failure for infinite life. Maximum pressure Flange thickness Tank diameter Leakage safety factor p max 250 kPa tf 10 mm id 500 mm Nsep 2 Bolt safety factor Member mod. Bolt modulus Nb 1.5 Ememb 206.8 GPa Ebolt 206.8 GPa

Given:

Design Choices: Use 24 M12 x 1.25 , class 10.9 cap screws. Material properties for class 10.9: Proof strength S p 830 MPa Yield strength Ultimate strength Solution: 1. See Mathcad file P1527. Ptot P S y 940 MPa S ut 1040 MPa

Bolt diameter Number of bolts Preload fraction

d 12 mm Nbolts 24 fp 0.75

Determine the load per screw.

id
4 Ptot

p max

Ptot 49.1 kN P 2045.31 N Pmax P

Nbolts
2

Pmin 0 N 2. 3. 4. Get the tensile stress area from Table 15-2. Calculate the preload. Calculate the clamp length. Clamp length 5. l 2 t f At 92.07 mm Fi fp S p At

Fi 57.31 kN l 20 mm

Determine the relevant ratios for the joint without the gasket from equations 15-18a and b. Joint aspect ratio: j r d l Ememb Ebolt j 0.6 r1

Plate to bolt modulus: 6.

Calculate Cr = C using equation 15.19 and the coefficients p i from Table 15-8 for j 0.6 . Coefficients from Table 15-8: p 0 0.7709 p 1 1.2600 p 2 1.0851 p 3 0.3647

MACHINE DESIGN - An Integrated Approach, 4th Ed.


Joint stiffness constant: 7. C p 3 r p 2 r p 1 r p 0
3 2

15-27-2
C 0.231

The portions of the applied load P felt by the bolt and the material can now be found from equations 15.13. Pb C P Pm ( 1 C) P Pb 0.47 kN Pm 1.6 kN Fb 57.79 kN Fm 55.74 kN

8.

Find the resulting loads in bolt and material after the load P is applied. Fb Fi Pb Fm Fi Pm

9.

Calculate the alternating and mean components of the fluctuating bolt load. Falt Fb Fi 2 Fb Fi 2 Falt 0.237 kN Fmean 57.55 kN

Fmean

10. The nominal mean and alternating stresses in the bolt are: Nominal mean stress

mnom anom

Fmean At Falt At

mnom 625.1 MPa anom 2.57 MPa

Nominal alternating stress

11. The fatigue stress-concentration factor for this thread is found from equation 15.15c and the mean stress-concentration factor factor Kfm is found from equation 6.17. Fatigue factor Nominal maximum stress Nominal minimum stress Kfm Kf 5.7 0.02682 d mm Kf 6

maxnom mnom anom minnom mnom anom

maxnom 627.6 MPa minnom 622.5 MPa

return Kf if Kf maxnom S y return S y Kf anom if Kf maxnom S y Kfm 1.479

mnom

return 0 if Kf maxnom minnom 2 S y 12. The local mean and alternating stresses in the bolt are then: Local mean stress Local alternating stress 13. The stress at the initial preload is

m Kfm mnom a Kf anom

m 924.5 MPa a 15.5 MPa

init Kfm

Fi At

init 920.73 MPa

14. An endurance strength must be found for this material. Using the methods of Section 6.6 we find for S ut 1040 MPa

MACHINE DESIGN - An Integrated Approach, 4th Ed.


S'e 0.5 S ut S'e 520.0 MPa

15-27-3

15. From the tables and formulas in Section 6.6 we have: Load Size Surface Cload 0.70 Csize 1 A 4.51 b 0.265
b

Csurf Temperature Reliability and the endurance limit is S e Cload Csize Csurf Ctemp Creliab S'e

Sut A MPa

Csurf 0.716

Ctemp 1 Creliab 0.814 S e 212.03 MPa

16. The corrected endurance strength and the ultimate tensile strength are used in equation 15.16 to find the safety factor from the Goodman line. Nf S e m init S ut a S e S ut init Nf 1.5

17. Calculate the maximum bolt stress and the safety factor against yielding for S y 940 MPa.

b
Ny

Fb At Sy

b 627.6 MPa
Ny 1.5

18. The load required to separate the joint and the safety factor against joint separation are found from equations 15.14c and 15.14d. Nsep Fi Pmax ( 1 C) Nsep 36

19. Check the bolt spacing, comparing it to the rules given in the text. Flange OD Bolt circle dia Bolt spacing od id 6 d d bc 0.5 ( od id) space bolt ratio od 572 mm d bc 536 mm space bolt 70.2 mm ratio 5.8

d bc
Nbolts

Space/dia ratio

space bolt d

which is OK

MACHINE DESIGN - An Integrated Approach, 4th Ed.

15-28-1

PROBLEM 15-28
Statement: Given: Calculate the proof loads (load that causes a tensile stress equal to the proof strength) for 1/2-13 UNC bolts in each SAE grade listed in Table 15-6. Bolt specification: 1/2-13 UNC Bolt diameter Solution: 1. d 0.500 in Tensile stress area At 0.1419 in
2

See Mathcad file P1528.

There are eight grade numbers in Table 15-6 that are applicable to the bolt size specified. The grade numbers an proof strength values are: Grade
i

i 1 2 8

S p
i

"1" "2" "4" "5" "5.2" "7" "8" "8.2" 2. Use equation 15.2 to calculate the proof load.

33 ksi 55 ksi 65 ksi 85 ksi 85 ksi 105 ksi 120 ksi 120 ksi

Fp At S p
i

Fp
i

"2" "4" "5" Grade i "5.2" "7" "8" "8.2"


"1"

lbf

4683 7805 9224

12062 12062 14900 17028 17028

MACHINE DESIGN - An Integrated Approach, 4th Ed.

15-29-1

PROBLEM 15-29
Statement: Given: Calculate the proof loads (load that causes a tensile stress equal to the proof strength) for M20 x 2.50 bolts in each class listed in Table 15-7. Bolt specification: M20 x 2.50 Bolt diameter Solution: 1. d 20 mm Tensile stress area At 244.79 mm
2

See Mathcad file P1529.

There are seven grade numbers in Table 15-7. The grade numbers and proof strength values are: Class
i

i 1 2 7

S p
i

"4.6" "4.8" "5.8" "8.8" "9.8" "10.9" "12.9"

225 MPa 310 MPa 380 MPa 600 MPa 650 MPa 830 MPa 970 MPa

2.

Use equation 15.2 to calculate the proof load.

Fp At S p
i

Fp
i

"4.6" "4.8" "5.8" Class "8.8" i "9.8" "10.9" "12.9"

kN

55 76 93

147 159 203 237

MACHINE DESIGN - An Integrated Approach, 4th Ed.

15-30-1

PROBLEM 15-30
Statement: Given: Bolt specification: 1/2-13 UNC Bolt diameter d 0.500 in Tensile area Bolt modulus Ememb

_____

Determine the joint stiffness constant for the bolt and members in Problem 15-7. At 0.1419 in
6 2

Grip

l 3 in

Member material matl "steel"

Ebolt 30 10 psi return 30 10 psi if matl = "steel" return 10.4 10 psi if matl = "alum"
6 6

Member modulus of elasticity function: Member stiffness parameters: Solution: 1.

See Problem 15-9, Table 15-8, and Mathcad file P1530.

Determine the relevant ratios for this joint from equations 15-18a and b. Joint aspect ratio: j r d l Ememb Ebolt j 0.167 r 1.000

Plate to bolt modulus:

2.

Calculate Cr = C using equation 15.19 and the coefficients p i from Table 15-8 for j 0.167 . Use linear interpolation between ja 0.1 and jb 0.2 Interpolation equation: p p a p b pb pa jb ja j ja p a p 0 0.5542 p 1 1.0876 p 2 1.0217 p 3 0.3600 C 0.128

Coefficients from Table 15-8:

p 0 p ( 0.4389 0.6118) p 1 p ( 0.9197 1.1715) p 2 p ( 0.8901 1.0875) p 3 p ( 0.3187 0.3806)

Joint stiffness constant:

C p 3 r p 2 r p 1 r p 0

MACHINE DESIGN - An Integrated Approach, 4th Ed.

15-31-1

PROBLEM 15-31
Statement: Given:

_____

Determine the joint stiffness constant for the bolt and members in Problem 15-8. Bolt specification: M14 x 2 Bolt diameter d 14 mm Member material matl "alum" Clamped length l 30 mm Bolt modulus Ebolt 206.8 GPa Ememb return 206.8 GPa if matl = "steel" return 71.0 GPa if matl = "alum"

Member modulus of elasticity function:

Solution: 1.

See problem 15-8, Table P15-8, and Mathcad file P1531.

Determine the relevant ratios for this joint from equations 15-18a and b. Joint aspect ratio: j r d l Ememb Ebolt j 0.467 r 0.343

Plate to bolt modulus:

2.

Calculate Cr = C using equation 15.19 and the coefficients p i from Table 15-8 for j 0.467 . Use linear interpolation between ja 0.4 and jb 0.5 Interpolation equation: p p a p b pb pa jb ja j ja p a p 0 0.7504 p 1 1.2625 p 2 1.1023 p 3 0.3732 C 0.432

Coefficients from Table 15-8:

p 0 p ( 0.7351 0.7580) p 1 p ( 1.2612 1.2632) p 2 p ( 1.1111 1.0979) p 3 p ( 0.3779 0.3708)

Joint stiffness constant:

C p 3 r p 2 r p 1 r p 0

MACHINE DESIGN - An Integrated Approach, 4th Ed.

15-32-1

PROBLEM 15-32
Statement: Given: Bolt specification: 7/16-14 UNC Bolt diameter d 0.4375 in Member material matl "steel"

_____

Determine the joint stiffness constant for the bolt and members in Problem 15-9. Clamped length l 2.75 in Bolt modulus Ememb Ebolt 30 10 psi return 30 10 psi if matl = "steel" return 10.4 10 psi if matl = "alum"
6 6 6

Member modulus of elasticity function:

Solution: 1.

See Problem 15-9, Table 15-8, and Mathcad file P1532.

Determine the relevant ratios for this joint from equations 15-18a and b. Joint aspect ratio: j r d l Ememb Ebolt j 0.159 r 1.000

Plate to bolt modulus:

2.

Calculate Cr = C using equation 15.19 and the coefficients p i from Table 15-8 for j 0.159 . Use linear interpolation between ja 0.1 and jb 0.2 Interpolation equation: p p a p b pb pa jb ja j ja p a p 0 0.5411 p 1 1.0685 p 2 1.0067 p 3 0.3553 C 0.124

Coefficients from Table 15-8:

p 0 p ( 0.4389 0.6118) p 1 p ( 0.9197 1.1715) p 2 p ( 0.8901 1.0875) p 3 p ( 0.3187 0.3806)

Joint stiffness constant:

C p 3 r p 2 r p 1 r p 0

MACHINE DESIGN - An Integrated Approach, 4th Ed.

15-33-1

PROBLEM 15-33
Statement: Given:

_____

Determine the joint stiffness constant for the bolt and members in Problem 15-10. Bolt specification: M12 x 1.25 Bolt diameter d 12 mm Member material matl "alum" Clamped length l 50 mm Bolt modulus Ebolt 206.8 GPa Ememb return 206.8 GPa if matl = "steel" return 71.0 GPa if matl = "alum"

Member modulus of elasticity function:

Solution: 1.

See problem 15-10, Table 15-8, and Mathcad file P1533.

Determine the relevant ratios for this joint from equations 15-18a and b. Joint aspect ratio: j r d l Ememb Ebolt j 0.240 r 0.343

Plate to bolt modulus:

2.

Calculate Cr = C using equation 15.19 and the coefficients p i from Table 15-8 for j 0.240 . Use linear interpolation between ja 0.2 and jb 0.3 Interpolation equation: p p a p b pb pa jb ja j ja p a p 0 0.6444 p 1 1.1999 p 2 1.0996 p 3 0.3822 C 0.347

Coefficients from Table 15-8:

p 0 p ( 0.6118 0.6932) p 1 p ( 1.1715 1.2426) p 2 p ( 1.0875 1.1177) p 3 p ( 0.3806 0.3845)

Joint stiffness constant:

C p 3 r p 2 r p 1 r p 0

MACHINE DESIGN - An Integrated Approach, 4th Ed.

15-34a-1

PROBLEM 15-34a
Statement:

_____

Figure P15-3 shows a bolted joint eccentrically loaded in shear. The shear loads are taken by the dowel pins, the number and size of which are given in Table P15-3. Although the figure shows 5 dowel pins, that is not the case for every row in the table. For a = 4 in, b = 4 in, l = 10 in, P = 2500 lb, and the data in the row(s) assigned from Table P15-3, find the magnitude and direction of the total shear force on each dowel. Eccentric load Dimensions: a 4 in b 4 in l 10 in P 2500 lbf Dowel diameters and position coordinates with respect to position C: i 1 2 5 A: d 0.250 in x 0.5 a y 0.5 b x 2.000 in y 2.000 in
1 2 3 4 5 1 2 3 4 5 1 2 3 4 5 1 2 3 4 5 1 2 3 4 5

Given:

B: C: D: E:

d 0.250 in d 0.250 in d 0.250 in d 0.250 in

x 0.5 a x 0 in x 0.5 a x 0.5 a n 5

y 0.5 b y 0 in y 0.5 b y 0.5 b

x 2.000 in x 0.000 in x 2.000 in x 2.000 in

y 2.000 in y 0.000 in y 2.000 in y 2.000 in

Number of dowels in group: Solution: 1. See Mathcad file P1534a.

Calculate the cross-sectional (shear) area of each dowel. A


i

2.

Use equations 15.24 to find the centroid of the dowel group with respect to position C.

A ixi
xbar
i

A iy i
xbar 0.000 in ybar
i

ybar 0.000 in A
i

3.

Determine the radial distance from the group centroid to each dowel. r if d 0 in x xbar
i i

y i ybar
2
1

0 in

4.

Calculate the direct shear at each dowel due to the force P acting through the centroid using equation 15.25a. P F1 if d 0 in 0 lbf i i n F1 500.0 lbf

5.

Calculate the magnitude of the moment acting on the group. M P ( l xbar) M 25000 in lbf

6.

Calculate the magnitude of the indirect shear at each dowel due to the moment acting on the group using equation 15.25b. M r j 1 2 5 F2
i

i 2

rj
j

7.

Calculate the angle that the vector F2i makes with the positive x axis.

MACHINE DESIGN - An Integrated Approach, 4th Ed.


s if ( M 0 in lbf 1 1 )

15-34a-2

if x xbar 0 in y ybar 0 in atan2 x xbar y ybar 0 deg s 90 deg


i i i i i

if d 0 in 0 deg
i i

8.

Calculate the vector components of F2i. F2x F2 cos


i i

i
F2y

F2y F2 sin
i i

9.

Determine the total shear force at each dowel. F


i

F F
2
2x i

1 i

10. Calculate the angle that Fi makes with the positive x axis.

F if d 0 in atan2 F2x F1 F2y 0


i i i i


F2x lbf F2y lbf F

11. Summarize results (magnitude and direction in the last two columns): A in
i 2

2.828 2.828 0.000 2.828 2.828

in

deg

45.0 -45.0 -90.0

lbf

deg

34.2 -52.9 -90.0

0.0491 0.0491 0.0491 0.0491 0.0491

1563 1563 0 -1563 -1563

1563 -1563 0 1563 -1563

1890 2588 500 1890 2588

A B C D E

-225.0 -135.0

145.8 -127.1

MACHINE DESIGN - An Integrated Approach, 4th Ed.

15-35a-1

PROBLEM 15-35a
Statement:

_____

Figure P15-3 shows a bolted and doweled joint eccentrically loaded in shear. The shear loads are taken by dowel pins, the number and size of which are given in Table P15-3. Although the figure shows 5 dowels, that is not the case for every row in the table. For a = 4 in, b = 4 in, l = 10 in, P = 2500 lb, and the data in the row(s) assigned from Table P15-3, find the total shear stress in each dowel. Eccentric load Dimensions: a 4 in b 4 in l 10 in P 2500 lbf Dowel diameters and position coordinates with respect to position C: i 1 2 5 A: d 0.250 in x 0.5 a y 0.5 b x 2.000 in y 2.000 in
1 2 3 4 5 1 2 3 4 5 1 2 3 4 5 1 2 3 4 5 1 2 3 4 5

Given:

B: C: D: E:

d 0.250 in d 0.250 in d 0.250 in d 0.250 in

x 0.5 a x 0 in x 0.5 a x 0.5 a n 5

y 0.5 b y 0 in y 0.5 b y 0.5 b

x 2.000 in x 0.000 in x 2.000 in x 2.000 in

y 2.000 in y 0.000 in y 2.000 in y 2.000 in

Number of dowels in group: Solution: 1. See Mathcad file P1535a.

Calculate the cross-sectional (shear) area of each dowel. A


i

2.

Use equations 15.24 to find the centroid of the dowel group with respect to position C.

A ixi
xbar
i

A iy i
xbar 0.000 in ybar
i

ybar 0.000 in A
i

3.

Determine the radial distance from the group centroid to each dowel. r if d 0 in x xbar
i i

y i ybar
2
1

0 in

4.

Calculate the direct shear at each dowel due to the force P acting through the centroid using equation 15.25a. P F1 if d 0 in 0 lbf i i n F1 500.0 lbf

5.

Calculate the magnitude of the moment acting on the group. M P ( l xbar) M 25000 in lbf

6.

Calculate the magnitude of the indirect shear at each dowel due to the moment acting on the group using equation 15.25b. M r j 1 2 5 F2
i

i 2

rj
j

7.

Calculate the angle that the vector F2i makes with the positive x axis.

MACHINE DESIGN - An Integrated Approach, 4th Ed.

15-35a-2

s if ( M 0 in lbf 1 1 )

if x xbar 0 in y ybar 0 in atan2 x xbar y ybar 0 deg s 90 deg


i i i i i

if d 0 in 0 deg
i i

8.

Calculate the vector components of F2i. F2x F2 cos


i i

i
F2y

F2y F2 sin
i i

9.

Determine the total shear force at each dowel. F


i

F F
2
2x i

1 i

10. Calculate the total shear stress at each dowel.

if d 0 in
i i

F A

i i

0 ksi

11. Summarize results (the shear stress is in the last column): A in


i 2

2.828 2.828 0.000 2.828 2.828

in

deg

45.0 -45.0 -90.0

F2x lbf

F2y lbf

lbf

ksi

A B C D E

0.0491 0.0491 0.0491 0.0491 0.0491

1563 1563 0 -1563 -1563

1563 -1563 0 1563 -1563

1890 2588 500 1890 2588

38.5 52.7 10.2 38.5 52.7

-225.0 -135.0

MACHINE DESIGN - An Integrated Approach, 4th Ed.

15-36a-1

PROBLEM 15-36a
Statement:

_____

Figure P15-3 shows a bolted and doweled joint eccentrically loaded in shear. The shear loads are taken by alloy steel dowel pins, the number and size of which are given in Table P15-3. Although the figure shows 5 dowel pins, that is not the case for every row in the table. For a = 4 in, b = 4 in, l = 10 in, P = 2500 lb, SAE grade 8 bolts, and the data in the row(s) assigned from Table P15-3, find the factor of safety against yielding in shear for each dowel pin. See Table 15-12 for strength data. Eccentric load Dimensions: a 4 in b 4 in l 10 in P 2500 lbf Dowel diameters and position coordinates with respect to position C: i 1 2 5 A: d 0.250 in x 0.5 a y 0.5 b x 2.000 in y 2.000 in
1 2 3 4 5 1 2 3 4 5 1 2 3 4 5 1 2 3 4 5 1 2 3 4 5

Given:

B: C: D: E:

d 0.250 in d 0.250 in d 0.250 in d 0.250 in

x 0.5 a x 0 in x 0.5 a x 0.5 a n 5

y 0.5 b y 0 in y 0.5 b y 0.5 b Shear strength

x 2.000 in x 0.000 in x 2.000 in x 2.000 in S ys 117 ksi

y 2.000 in y 0.000 in y 2.000 in y 2.000 in

Number of dowels in group: Solution: 1. See Mathcad file P1536a.

Calculate the cross-sectional (shear) area of each dowel. A


i

2.

Use equations 15.24 to find the centroid of the dowel group with respect to position C.

A ixi
xbar
i

A iy i
xbar 0.000 in ybar
i

ybar 0.000 in A
i

3.

Determine the radial distance from the group centroid to each dowel. r if d 0 in x xbar
i i

y i ybar
2
1

0 in

4.

Calculate the direct shear at each dowel due to the force P acting through the centroid using equation 15.25a. P F1 if d 0 in 0 lbf i i n F1 500.0 lbf

5.

Calculate the magnitude of the moment acting on the group. M P ( l xbar) M 25000 in lbf

6.

Calculate the magnitude of the indirect shear at each dowel due to the moment acting on the group using equation 15.25b. M r j 1 2 5 F2
i

i 2

rj
j

7.

Calculate the angle that the vector F2i makes with the positive x axis.

MACHINE DESIGN - An Integrated Approach, 4th Ed.


s if ( M 0 in lbf 1 1 )

15-36a-2

if x xbar 0 in y ybar 0 in atan2 x xbar y ybar 0 deg s 90 deg i i i i i if d 0 in 0 deg


i i i

8.

Calculate the vector components of F2i. F2x F2 cos


i i

i
F2y

F2y F2 sin
i i

9.

Determine the total shear force at each dowel. F


i

F F
2
2x i

1 i

10. Calculate the total shear stress at each dowel.

if d 0 in
i i

F A

i i

0 ksi

11. Using equation 5.9a, calculate the factor of safety against yielding in shear. Ny if d 0 in
i

S ys

12. Summarize results (the safety factor is in the last column): A in


i 2

2.828 2.828 0.000 2.828 2.828

in

deg

45.0 -45.0 -90.0

F2x lbf

F2y lbf

lbf

ksi

Ny
i

0.0491 0.0491 0.0491 0.0491 0.0491

1563 1563 0 -1563 -1563

1563 -1563 0 1563 -1563

1890 2588 500 1890 2588

38.5 52.7 10.2 38.5 52.7

3.0 2.2 11.5 3.0 2.2

A B C D E

-225.0 -135.0

MACHINE DESIGN - An Integrated Approach, 4th Ed.

15-37-1

PROBLEM 15-37
Statement: The coefficient of friction for an oil-lubricated, single-start, power screw thread-nut combination is 0.10. Which of the American Standard Acme Threads in Table 15-3 will be self-locking for this thread-nut combination? Which will be the least likely to back down in the presence of dynamic loading? Which will be the most likely to back down in the presence of a dynamic load? Screw-nut coefficient of friction

Given: Solution: 1. 2.

0.10

Thread angle

14.5 deg

See Table 15-3 and Mathcad file P1537. i 1 2 23

There are 23 entries in Table 15-3. Establish an index variable with range 1 .. 23:

Input the values of the major diameter, d; thread pitch, p; and pitch diameter, d p. Note that for a single-start thread L = p. Major Diameter d
i

Thread Pitch L
i

Pitch Diameter d p
i

0.250 0.313 0.375 0.438 0.500 0.625 0.750 0.875 1.000 1.125 1.250 1.375 1.500 1.750 2.000 2.250 2.500 2.750 3.000 3.500 4.000 4.500 5.000

0.063 0.071 0.083 0.083 0.100 0.125 0.167 0.167 0.200 0.200 0.200 0.250 0.250 0.250 0.250 0.333 0.333 0.333 0.500 0.500 0.500 0.500 0.500

0.219 0.277 0.333 0.396 0.450 0.563 0.667 0.792 0.900 1.025 1.150 1.250 1.375 1.625 1.875 2.083 2.333 2.583 2.750 3.250 3.750 4.250 4.750

3.

Calculate the right-hand side of inequality 15.6a. L selflock


i i
i

d p

cos( )

MACHINE DESIGN - An Integrated Approach, 4th Ed.

15-37-2

4.

Display the selflock calculations. If the result is less than the thread will be self locking if not, it will back dow under a static load. d
i 0.250 0.313 0.375 0.438 0.500 0.625 0.750 0.875 1.000 1.125 1.250 1.375 1.500 1.750 2.000 2.250 2.500 2.750 3.000 3.500 4.000 4.500 5.000

selflock
i 0.089 0.079 0.077 0.065 0.068 0.068 0.077 0.065 0.068 0.060 0.054 0.062 0.056 0.047 0.041 0.049 0.044 0.040 0.056 0.047 0.041 0.036 0.032

5.

All of the selflock values are less than so all threads will be selflocking under a static load. The 5.000-inch major diameter thread has the lowest selflock value and will, therefore, be the least likely to back down in the presence of a dynamic load. The 0.250-inch major diameter thread has the highest selflock value and is, therefore, the most likely to back down in the presence of a dynamic load.

MACHINE DESIGN - An Integrated Approach, 4th Ed.

15-38-1

PROBLEM 15-38
Statement: The coefficient of friction for an oil-lubricated, single-start, power screw thread-nut combination is 0.20. Which of the American Standard Acme Threads in Table 15-3 will have the greatest efficiency (neglecting thrust-collar friction)? Screw-nut coefficient of friction

Given: Solution: 1. 2.

0.20

Thread angle

14.5 deg

See Table 15-3 and Mathcad file P1538. i 1 2 23

There are 23 entries in Table 15-3. Establish an index variable with range 1 .. 23:

Input the values of the major diameter, d; thread pitch, p; and pitch diameter, d p. Note that for a single-start thread L = p. Major Diameter d
i

Thread Pitch L
i

Pitch Diameter d p
i

0.250 0.313 0.375 0.438 0.500 0.625 0.750 0.875 1.000 1.125 1.250 1.375 1.500 1.750 2.000 2.250 2.500 2.750 3.000 3.500 4.000 4.500 5.000

0.063 0.071 0.083 0.083 0.100 0.125 0.167 0.167 0.200 0.200 0.200 0.250 0.250 0.250 0.250 0.333 0.333 0.333 0.500 0.500 0.500 0.500 0.500

0.219 0.277 0.333 0.396 0.450 0.563 0.667 0.792 0.900 1.025 1.150 1.250 1.375 1.625 1.875 2.083 2.333 2.583 2.750 3.250 3.750 4.250 4.750

3.

Calculate the efficiency using equation 15.7d.

L e
i

i
i

d p d p L cos( )
i

d p cos( ) L
i

MACHINE DESIGN - An Integrated Approach, 4th Ed.


4. Display the efficiency calculations. d
i 0.250 0.313 0.375 0.438 0.500 0.625 0.750 0.875 1.000 1.125 1.250 1.375 1.500 1.750 2.000 2.250 2.500 2.750 3.000 3.500 4.000 4.500 5.000

15-38-2

e
i 0.301 0.278 0.273 0.241 0.251 0.251 0.274 0.242 0.251 0.228 0.209 0.232 0.216 0.190 0.169 0.196 0.179 0.164 0.216 0.190 0.169 0.152 0.139

5.

Determine the maximum efficiency and its corresponding major diameter. emax max( e) emax 0.301

The major diameter of the Acme thread with the greatest efficiency is 0.250 in. As a general rule, the greater the major diameter the less efficient the screw thread is. However, as can be seen from the table above, this is not always the case.

MACHINE DESIGN - An Integrated Approach, 4th Ed.

15-39-1

PROBLEM 15-39
Statement: Determine the number of engaged screw threads required to make the total stripping-shear area of the engaged threads equal to twice the tensile stress area for each of the fine pitch screw threads in Table 15-2. See Table 15-2, Table 15-5, and Mathcad file P1539.

Solution: 1. 2.

There are 14 entries in the fine-thread columns Table 15-2. Establish an index variable with range 1 .. 14: i 1 2 14 Input the values of the major diameter, d; the pitch, p; the minor diameter, d r; and tensile stress area, At. Major Diameter, mm d
i

Pitch, mm p
i

Minor Diameter, mm d r
i

Tensile Stress Area, mm2 At


i

8 10 12 14 16 18 20 22 24 27 30 33 36 39

1.00 1.25 1.25 1.50 1.50 1.50 1.50 1.50 2.00 2.00 2.00 2.00 3.00 3.00

6.77 8.47 10.47 12.16 14.16 16.16 18.16 20.16 21.55 24.55 27.55 30.55 32.32 35.32

39.17 61.20 92.07 124.55 167.25 216.23 271.50 333.06 384.42 495.74 621.2 760.80 864.94 1028.39

3.

Determine the number of threads required to make the total stripping-shear area equal to twice the tensile stress area. n As = 2 At where n is the number of threads and As is given by equation 15.8a. Solving for n, 2 At

n
i

i i

0.8 d r p

where the factor, 0.8, is taken from Table 15-5.

MACHINE DESIGN - An Integrated Approach, 4th Ed.

15-39-2

4.

Display the values of n:

d
i 8 10 12 14 16 18 20 22 24 27 30 33 36 39

n
i 4.6 4.6 5.6 5.4 6.3 7.1 7.9 8.8 7.1 8.0 9.0 9.9 7.1 7.7

MACHINE DESIGN - An Integrated Approach, 4th Ed.

15-40-1

PROBLEM 15-40
Statement: Given: Calculate the ultimate tensile loads (load that causes a tensile stress equal to the tensile strength) for 1/2-13 UNC bolts in each SAE grade listed in Table 15-6. Bolt specification: 1/2-13 UNC Bolt diameter Solution: 1. d 0.500 in Tensile stress area At 0.1419 in
2

See Table 15-6 and Mathcad file P1540.

There are eight grade numbers in Table 15-6 that are applicable to the bolt size specified. The grade numbers an tensile strength values are: Grade
i

i 1 2 8

S ut
i

"1" "2" "4" "5" "5.2" "7" "8" "8.2" 2. Use equation 15.2 to calculate the proof load.

60 ksi 74 ksi 115 ksi 120 ksi 120 ksi 133 ksi 150 ksi 150 ksi

Fut At S ut
i

Fut
i

"1" "2" "4" "5" Grade i "5.2" "7" "8" "8.2"

lbf

8514

10501 16319 17028 17028 18873 21285 21285

MACHINE DESIGN - An Integrated Approach, 4th Ed.

15-41-1

PROBLEM 15-41
Statement: Given: Calculate the ultimate tensile loads (load that causes a tensile stress equal to the tensile strength) for M20 x 2.50 bolts in each class listed in Table 15-7. Bolt specification: M20 x 2.50 Bolt diameter Solution: 1. d 20 mm Tensile stress area At 244.79 mm
2

See Table 15-7 and Mathcad file P1541.

There are seven grade numbers in Table 15-7. The grade numbers and tensile strength values are: Class
i

i 1 2 7

S ut
i

"4.6" "4.8" "5.8" "8.8" "9.8" "10.9" "12.9"

400 MPa 420 MPa 520 MPa 830 MPa 900 MPa 1040 MPa 1220 MPa

2.

Use equation 15.2 to calculate the ultimate tensile load.

Fut At S ut
i

Fut
i

"4.6" "4.8" "5.8" Class "8.8" i "9.8" "10.9" "12.9"

kN

98

103 127 203 220 255 299

MACHINE DESIGN - An Integrated Approach, 4th Ed.

15-42-1

PROBLEM 15-42
Statement: 3/8-in dia UNC bolts and nuts clamp a 0.75-in-thick aluminum cover plate to a 0.50-in-thick steel flange. Determine the stiffness factor of the joints. Bolt diameter Cover plate Flange Solution: 1. d 0.375 in lc 0.75 in lf 0.50 in Bolt modulus Cover plate Flange Ebolt 30 10 psi Ec 10.4 10 psi Ef 30 10 psi
6 6 6

Given:

See Mathcad file P1542.

Determine the relevant ratios for this joint from equations 15-18. Clamped length: Joint aspect ratio: Plate to bolt modulus: l lc lf j rH rL Low and high modulus plate thicknesses: d l Ef Ebolt Ec Ebolt l 1.25 in j 0.3 rH 1 rL 0.347 TL 0.75 in TH 0.5 in t 0.6 m 0.347

TL lc TH lf

Plate thickness ratio:

t m

TL TL TH Ec Ef

Material modulus ratio:

2.

Calculate the Cr terms CH and CL for rH and rL , respectively, using equation 15.19 and the coefficients p i from Table 15-8 for j 0.3 . Coefficients from Table 15-8: p 0 0.6932 p 3 0.3845 CL p 3 rL p 2 rL p 1 rL p 0 CH p 3 rH p 2 rH p 1 rH p 0
3 2 3 2

p 1 1.2426

p 2 1.1177

CL 0.381 CH 0.184

3.

Use equation 15.20 to calculate Ct. taking the coefficients from Table 15-9 for j 0.3 . Coeficients from Table 15-9: q 0 0.0861 q 3 13.963 Ct q 3 t q 2 t q 1 t q 0
3 2

q 1 8.2344

q 2 22.274

Ct 0.024

MACHINE DESIGN - An Integrated Approach, 4th Ed.


4. Calculate a correction factor to linearize the result from the previous step using equation 15.21. Correction factor: a e

15-42-2

0.0598 ln( j ) 30.1385 ln( j ) 20.4350 ln( j ) 2.3516

a 0.177 5. Using equation 15.22, calculate the joint stiffness factor C. Joint stiffness factor: C CH t a Ct CL CH C 0.303

MACHINE DESIGN - An Integrated Approach, 4th Ed.

15-43-1

PROBLEM 15-43
Statement: M14 x 2 bolts and nuts clamp a 16-mm-thick aluminum cover plate to a 12-mm-thick steel flange. Determine the stiffness factor of the joints. Bolt diameter Cover plate Flange Solution: 1. d 14 mm lc 16 mm lf 12 mm Bolt modulus Cover plate Flange Ebolt 206.8 GPa Ec 71.7 GPa Ef 206.8 GPa

Given:

See Mathcad file P1543.

Determine the relevant ratios for this joint from equations 15-18. Clamped length: Joint aspect ratio: Plate to bolt modulus: l lc lf j rH rL Low and high modulus plate thicknesses: d l Ef Ebolt Ec Ebolt l 28 mm j 0.5 rH 1 rL 0.347 TL 16 mm TH 12 mm t 0.571 m 0.347

TL lc TH lf

Plate thickness ratio:

t m

TL TL TH Ec Ef

Material modulus ratio:

2.

Calculate the Cr terms CH and CL for rH and rL , respectively, using equation 15.19 and the coefficients p i from Table 15-8 for j 0.5 . Coefficients from Table 15-8: p 0 0.7580 p 3 0.3708 CL p 3 rL p 2 rL p 1 rL p 0 CH p 3 rH p 2 rH p 1 rH p 0
3 2 3 2

p 1 1.2632

p 2 1.0979

CL 0.437 CH 0.222

3.

Use equation 15.20 to calculate Ct. taking the coefficients from Table 15-9 for j 0.5 . Coeficients from Table 15-9: q 0 0.0533 q 3 11.457 Ct q 3 t q 2 t q 1 t q 0
3 2

q 1 7.8676

q 2 19.357

Ct 0.366

4.

Calculate a correction factor to linearize the result from the previous step using equation 15.21.

MACHINE DESIGN - An Integrated Approach, 4th Ed.

15-43-2

Correction factor:

a e

0.0598 ln( j ) 30.1385 ln( j ) 20.4350 ln( j ) 2.3516

a 0.135 5. Using equation 15.22, calculate the joint stiffness factor C. Joint stiffness factor: C CH t a Ct CL CH C 0.355

MACHINE DESIGN - An Integrated Approach, 4th Ed.

15-44-1

PROBLEM 15-44
Statement: Given: M16 x 2.0 bolts with nuts clamp a 50-mm-thick aluminum cover plate to a 30-mm-thick steel flange. Determine the stiffness factor of the joints. Bolt diameter Cover plate Flange Solution: 1. d 16 mm lc 50 mm lf 30 mm Bolt modulus Cover plate Flange Ebolt 206.8 GPa Ec 71.7 GPa Ef 206.8 GPa

See Mathcad file P1544.

Determine the relevant ratios for this joint from equations 15-18. Clamped length: Joint aspect ratio: Plate to bolt modulus: l lc lf j rH rL Low and high modulus plate thicknesses: d l Ef Ebolt Ec Ebolt l 80 mm j 0.2 rH 1 rL 0.347 TL 50 mm TH 30 mm t 0.625

TL lc TH lf

Plate thickness ratio: 2.

TL TL TH

Calculate the Cr terms CH and CL for rH and rL , respectively, using equation 15.19 and the coefficients p i from Table 15-8 for j 0.2 . Coefficients from Table 15-8: p 0 0.6118 p 3 0.3806 CL p 3 rL p 2 rL p 1 rL p 0 CH p 3 rH p 2 rH p 1 rH p 0
3 2 3 2

p 1 1.1715

p 2 1.0875

CL 0.320 CH 0.147

3.

Use equation 15.20 to calculate Ct. taking the coefficients from Table 15-9 for j 0.2 . Coeficients from Table 15-9: q 0 0.1010 q 3 15.497 Ct q 3 t q 2 t q 1 t q 0
3 2

q 1 8.5465

q 2 24.166

Ct 0.214

4.

Calculate a correction factor to linearize the result from the previous step using equation 15.21. Correction factor: a e

0.0598 ln( j ) 30.1385 ln( j ) 20.4350 ln( j ) 2.3516

a 0.214 5. Using equation 15.22, calculate the joint stiffness factor C. Joint stiffness factor: C CH t a Ct CL CH C 0.248

MACHINE DESIGN - An Integrated Approach, 4th Ed.

15-45-1

PROBLEM 15-45
Statement: Given: 5/16-18 UNC bolts with nuts clamp a 1.625-in-thick gray cast iron cover plate to a 1.5-in-thick steel flange. Determine the stiffness factor of the joints. Bolt diameter Cover plate Flange Solution: 1. d 0.3125 in lc 1.625 in lf 1.500 in Bolt modulus Cover plate Flange Ebolt 30000 ksi Ec 15000 ksi Ef 30000 ksi

See Mathcad file P1545.

Determine the relevant ratios for this joint from equations 15-18. Clamped length: Joint aspect ratio: Plate to bolt modulus: l lc lf j rH rL Low and high modulus plate thicknesses: d l Ef Ebolt Ec Ebolt l 3.125 in j 0.1 rH 1 rL 0.5 TL 1.625 in TH 1.5 in t 0.52

TL lc TH lf

Plate thickness ratio: 2.

TL TL TH

Calculate the Cr terms CH and CL for rH and rL , respectively, using equation 15.19 and the coefficients p i from Table 15-8 for j 0.1 . Coefficients from Table 15-8: p 0 0.4389 p 3 0.3187 CL p 3 rL p 2 rL p 1 rL p 0 CH p 3 rH p 2 rH p 1 rH p 0
3 2 3 2

p 1 0.9197

p 2 0.8901

CL 0.162 CH 0.091

3.

Use equation 15.20 to calculate Ct. taking the coefficients from Table 15-9 for j 0.1 . Coeficients from Table 15-9: q 0 0.0079 q 3 202.44
5

q 1 17.040 q 4 209.38
4 3 2

q 2 92.832 q 5 82.726 Ct 0.068

Ct q 5 t q 4 t q 3 t q 2 t q 1 t q 0 4. Calculate a correction factor to linearize the result from the previous step using equation 15.21. Correction factor: a e

0.0598 ln( j ) 30.1385 ln( j ) 20.4350 ln( j ) 2.3516

a 0.260 5. Using equation 15.22, calculate the joint stiffness factor C. Joint stiffness factor: C CH t a Ct CL CH C 0.129

MACHINE DESIGN - An Integrated Approach, 4th Ed.

15-46-1

PROBLEM 15-46
Statement: Given: M16 x 1.5 bolts with nuts clamp a 8-mm-thick titanium cover plate to a 8-mm-thick stainless steel flange. Determine the stiffness factor of the joints. Bolt diameter Cover plate Flange Solution: 1. d 16 mm lc 8 mm lf 8 mm Bolt modulus Cover plate Flange Ebolt 206.8 GPa Ec 113.8 GPa Ef 189.6 GPa

See Mathcad file P1546.

Determine the relevant ratios for this joint from equations 15-18. Clamped length: Joint aspect ratio: Plate to bolt modulus: l lc lf j rH rL Low and high modulus plate thicknesses: d l Ef Ebolt Ec Ebolt l 16 mm j 1 rH 0.917 rL 0.55 TL 8 mm TH 8 mm t 0.5

TL lc TH lf

Plate thickness ratio: 2.

TL TL TH

Calculate the Cr terms CH and CL for rH and rL , respectively, using equation 15.19 and the coefficients p i from Table 15-8 for j 1 . Coefficients from Table 15-8: p 0 0.4389 p 3 0.3187 CL p 3 rL p 2 rL p 1 rL p 0 CH p 3 rH p 2 rH p 1 rH p 0
3 2 3 2

p 1 0.9197

p 2 0.8901

CL 0.149 CH 0.098

3.

Use equation 15.20 to calculate Ct. taking the coefficients from Table 15-9 for j 1 . Coeficients from Table 15-9: q 0 0.0079 q 3 202.44
5

q 1 17.040 q 4 209.38
4 3 2

q 2 92.832 q 5 82.726 Ct 0.124

Ct q 5 t q 4 t q 3 t q 2 t q 1 t q 0 4. Calculate a correction factor to linearize the result from the previous step using equation 15.21. Correction factor: a e

0.0598 ln( j ) 30.1385 ln( j ) 20.4350 ln( j ) 2.3516

a 0.095 5. Using equation 15.22, calculate the joint stiffness factor C. Joint stiffness factor: C CH t a Ct CL CH C 0.124

MACHINE DESIGN - An Integrated Approach, 4th Ed.

15-47-1

PROBLEM 15-47
Statement: An M12 X 1.25 soft steel nut is assembled with a hardened steel bolt. The nut is 11 mm thick and has a shear yield-strength of 120 MPa. Determine the axial force that will cause stripping failure of the nut if the nut threads fail before the bolt fails. Pitch Major dia. of bolt d 12 mm Nut thickness h 11 mm Shear strength S ys 120 MPa p 1.25 mm

Given:

Assumptions: The nut major diameter is equal to the bolt major diameter. Solution: 1. See Table 15-2, Table 15-5, and Mathcad file P1547.

Calculate the approximate number of threads in the nut. Number of threads n h p n 8.8

2.

Determine the area factor for nut thread stripping from Table 15-5. Area factor wo 0.88

3.

Calculate the shear area in the nut using equation 15.8b and the number of threads from step 1. Shear area As n d wo p As 364.9 mm
2

4.

Using equation 15.8c, calculate the tensile force that will cause the nut threads to fail in shear. Failure load Ffail As S ys Ffail 43.8 kN

MACHINE DESIGN - An Integrated Approach, 4th Ed.

15-48-1

PROBLEM 15-48
Statement: Compare the yield loads (load that causes a tensile stress equal to the yield strength) to the proof loads (load that causes a tensile stress equal to the proof strength) for M12 x 1.25 bolts in each class listed in Table 15-7. Bolt specification: M12 x 1.25 Bolt diameter Solution: 1. d 12 mm Tensile stress area At 92.07 mm
2

Given:

See Tables 15-2 and 15-7 and Mathcad file P1548.

There are seven grade numbers in Table 15-7. The grade numbers, proof strength, and yield strength values are: Class
i

i 1 2 7

S p
i

S y
i

"4.6" "4.8" "5.8" "8.8" "9.8" "10.9" "12.9"

225 MPa 310 MPa 380 MPa 600 MPa 650 MPa 830 MPa 970 MPa

240 MPa 340 MPa 420 MPa 660 MPa 720 MPa 940 MPa 1100 MPa

2.

Use equation 15.2 to calculate the proof load and the yield load. The percent difference is in the last column. Fp At S p
i i

Fy At S y
i

Fp

"4.6" "4.8" "5.8" Class "8.8" i "9.8" "10.9" "12.9"

kN

Fy

Fy Fp
i

kN

22 31 39 61 66 87 101

Fp

1 %

21 29 35 55 60 76 89

6.7 9.7 10.5 10.0 10.8 13.3 13.4

MACHINE DESIGN - An Integrated Approach, 4th Ed.

15-49-1

PROBLEM 15-49
Statement: An M16 x 1.50, class 4.8 bolt with cut threads is preloaded to 85% of its proof strength when clamping a 20-mm-thick sandwich of solid steel. Find the safety factors against static yielding and joint separation when a static 3-kN external load is applied. Material properties for class 4.8: Bolt diameter d 16 mm Proof strength Preload fraction fp 0.85 S p 310 MPa Clamp length Applied load Bolt modulus Solution: 1. 2. 3. 4. l 20 mm Ptot 3 kN Ebolt E Yield strength S y 340 MPa Number of bolts Nbolts 1 Ultimate strength S ut 420 MPa Young's modulus E 206.8 GPa Member modulus Ememb E

Given:

See Mathcad file P1549. P Ptot Nbolts P 3 kN At 167.25 mm Fi 44 kN


2

Determine the load per bolt.

Get the tensile stress area from Table 15-2. Calculate the preload. Fi fp S p At

Determine the relevant ratios for this joint from equations 15-18a and b. Joint aspect ratio: j r d l Ememb Ebolt j 0.8 r1

Plate to bolt modulus:

5.

Calculate Cr = C using equation 15.19 and the coefficients p i from Table 15-8 for j 0.8 . Coefficients from Table 15-8: p 0 0.7800 p 1 1.2503 p 2 1.0672 p 3 0.3571 Joint stiffness constant: C p 3 r p 2 r p 1 r p 0
3 2

C 0.240

6.

The portions of the applied load P felt by the bolt and the material can now be found from equations 15.13. Pb C P Pm ( 1 C) P Pb 0.7 kN Pm 2.3 kN Fb 44.8 kN Fm 41.8 kN

Find the resulting loads in bolt and material after the load P is applied. Fb Fi Pb Fm Fi Pm

The maximum tensile stress in the bolt is

Fb At

b 38.8 ksi

MACHINE DESIGN - An Integrated Approach, 4th Ed.


9.

15-49-2

This is a uniaxial stress situation, so the principal stress and von Mises stress are identical to the applied tensile stress. The safety factor against yielding for Grade 4.8 with S y 340 MPa is then Ny Sy Ny 1.3

10. The load required to separate the joint and the safety factor against joint separation are found from equations 15.14c and 15.14d. P0 Fi 1C P0 P P0 58.0 kN Nsep 19.3

Nsep

MACHINE DESIGN - An Integrated Approach, 4th Ed.

15-50-1

PROBLEM 15-50
Statement: A 15-mm-thick steel cap is to be fastened to a 15-mm-thick steel flange with 6 bolts and nuts. The external load on the cap is 30 kN. Size and specify the bolts for a safety factor of at least 1.5 and specify the torque required on each bolt to obtain the preload if the threads are lubricated. Number of bolts Nbolts 6 Clamp length l 30 mm Bolt modulus Applied load Ebolt 206.8 GPa Ptot 30 kN Member mod. Ememb 206.8 GPa

Given:

Design Choices: Use M10 x 1.5 , class 5.8 bolts. Material properties for class 5.8: Proof strength S p 380 MPa Yield strength Solution: 1. See Mathcad file P1550. P Ptot Nbolts S y 420 MPa

Bolt diameter Preload fraction

d 10 mm fp 0.70

Determine the load per bolt.

P 5.00 kN

2. 3. 4.

Get the tensile stress area from Table 15-2. Calculate the preload. Fi fp S p At

At 57.99 mm Fi 15.43 kN

Determine the relevant ratios for this joint from equations 15-18a and b. Joint aspect ratio: j r d l Ememb Ebolt j 0.333 r1

Plate to bolt modulus: 5.

Calculate Cr = C using equation 15.19 and the coefficients p i from Table 15-8 for j 0.333 . Use linear interpolation between ja 0.3 and jb 0.4 Interpolation equation: p p a p b pb pa jb ja j ja p a p 0 0.7072 p 1 1.2488 p 2 1.1155 p 3 0.3823 C 0.192

Coefficients from Table 15-8:

p 0 p ( 0.6932 0.7351) p 1 p ( 1.2426 1.2612) p 2 p ( 1.1177 1.1111) p 3 p ( 0.3845 0.3779)

Joint stiffness constant: 6.

C p 3 r p 2 r p 1 r p 0 Pb 1.0 kN Pm 4.0 kN

The portions of the applied load P felt by the bolt and the material can now be found from equations 15.13. Pb C P Pm ( 1 C) P

Find the resulting loads in bolt and material after the load P is applied.

MACHINE DESIGN - An Integrated Approach, 4th Ed.


Fb Fi Pb Fm Fi Pm 8 The maximum tensile stress in the bolt is Fb 16.4 kN Fm 11.4 kN

15-50-2

b
9.

Fb At

b 282.5 MPa

This is a uniaxial stress situation, so the principal stress and von Mises stress are identical to the applied tensile stress. The safety factor against yielding for Grade 5.8 with S y 420 MPa is then Ny Sy Ny 1.5

10. The load required to separate the joint and the safety factor against joint separation are found from equations 15.14c and 15.14d. P0 Fi 1C P0 P P0 19.1 kN Nsep 3.8

Nsep

11. Use equation 15.23d to calculate the tightening torque. Ti 0.21 Fi d Ti 32.4 N m

MACHINE DESIGN - An Integrated Approach, 4th Ed.

15-51-1

PROBLEM 15-51
Statement: Repeat Problem 15-50 with a total external load on the six bolts that varies from 0 to 30 kN per cycle. Design these bolts for infinite life with a factor of safety of at least 1.5. Specify their size, class, preload, and tightening torque. Total force Number of bolts Ptot 30 kN Nbolts 6 Member mod. Bolt modulus Ememb 206.8 GPa Ebolt 206.8 GPa

Given:

Design Choices: Use M10 x 1.5 , class 5.8 bolts with rolled threads. Material properties for class 5.8: Proof strength S p 380 MPa Yield strength Ultimate strength Solution: 1. See Mathcad file P1551. S y 420 MPa S ut 520 MPa

Bolt diameter Preload fraction Design safety factor Clamp length

d 10 mm fp 0.70 Nd 1.5 l 30 mm

Determine the load per bolt. P Ptot Nbolts P 5 kN Pmax P At 57.99 mm Fi 15.4 kN
2

Pmin 0 N 2. 3. 4. Get the tensile stress area from Table 15-2. Calculate the preload. Fi fp S p At

Determine the relevant ratios for this joint from equations 15-18a and b. Joint aspect ratio: j r d l Ememb Ebolt j 0.333 r1

Plate to bolt modulus: 5.

Calculate Cr = C using equation 15.19 and the coefficients p i from Table 15-8 for j 0.333 . Use linear interpolation between ja 0.3 and jb 0.4 Interpolation equation: p p a p b pb pa jb ja j ja p a p 0 0.7072 p 1 1.2488 p 2 1.1155 p 3 0.3823 C 0.192

Coefficients from Table 15-8:

p 0 p ( 0.6932 0.7351) p 1 p ( 1.2426 1.2612) p 2 p ( 1.1177 1.1111) p 3 p ( 0.3845 0.3779)

Joint stiffness constant: 6.

C p 3 r p 2 r p 1 r p 0

The portions of the applied load P felt by the bolt and the material can now be found from equations 15.13. Pb C P Pb 0.96 kN

MACHINE DESIGN - An Integrated Approach, 4th Ed.


Pm ( 1 C) P 7. Fb Fi Pb Fm Fi Pm 8. Pm 4.0 kN Fb 16.38 kN Fm 11.38 kN

15-51-2

Find the resulting loads in bolt and material after the load P is applied.

Calculate the alternating and mean components of the fluctuating bolt load. Falt Fb Fi 2 Fb Fi 2 Falt 0.479 kN Fmean 15.90 kN

Fmean 9.

The fatigue stress-concentration factor Kf 2.2 for rolled threads is taken from Table 15-8. The mean stress-concentration factor is taken as Kfm 1 in this case. The mean and alternating stresses in the bolt are:

alt Kf

Falt At Fmean At

alt 18.17 MPa mean 274.26 MPa

mean Kfm
10. The stress at the initial preload is

init Kfm

Fi At

init 266.00 MPa

11. An endurance strength must be found for this material. Using the methods of Section 6.6 we find for S ut 520 MPa S'e 0.5 S ut 12. From the tables and formulas in Section 6.6 we have: Load Size Surface Cload 0.70 Csize 1 A 4.51 Csurf Temperature Reliability and the endurance limit is S e Cload Csize Csurf Ctemp Creliab S'e S e 127.39 MPa b 0.265
b

S'e 260.0 MPa

Sut A MPa

Csurf 0.86

Ctemp 1 Creliab 0.814

13. The corrected endurance strength and the ultimate tensile strength are used in equation 15.16 to find the safety factor from the Goodman line.

MACHINE DESIGN - An Integrated Approach, 4th Ed.


S e S ut init

15-51-3

Nf

S e mean init S ut alt

Nf 3.1

14. Calculate the maximum bolt stress and the safety factor against yielding for S y 420 MPa.

b
Ny

Fb At Sy

b 282.5 MPa
Ny 1.5

15. The load required to separate the joint and the safety factor against joint separation are found from equations 15.14c and 15.14d. Nsep Fi Pmax ( 1 C) Nsep 3.8

16. Use equation 15.23d to calculate the tightening torque. Ti 0.21 Fi d Ti 32.4 N m

MACHINE DESIGN - An Integrated Approach, 4th Ed.

15-52-1

PROBLEM 15-52
Statement: A 20-mm-thick aluminum cap is to be fastened to a 20-mm-thick aluminum flange with 8 bolts and nuts. The external load on the cap varies from 0 to 40 kN per cycle. Size and specify the bolts for infinite life and a safety factor of at least 1.5 and specify the torque required on each bolt to obtain the preload if the threads are lubricated. Total force Number of bolts Ptot 40 kN Nbolts 8 Member mod. Bolt modulus Ememb 71.7 GPa Ebolt 206.8 GPa

Given:

Design Choices: Use M10 x 1.5 , class 8.8 bolts with rolled threads. Material properties for class 8.8: Proof strength S p 600 MPa Yield strength Ultimate strength Solution: 1. See Mathcad file P1552. S y 660 MPa S ut 830 MPa

Bolt diameter Preload fraction Design safety factor Clamp length

d 10 mm fp 0.70 Nd 1.5 l 40 mm

Determine the load per bolt. P Ptot Nbolts P 5 kN Pmax P At 57.99 mm Fi 24.4 kN
2

Pmin 0 N 2. 3. 4. Get the tensile stress area from Table 15-2. Calculate the preload. Fi fp S p At

Determine the relevant ratios for this joint from equations 15-18a and b. Joint aspect ratio: j r d l Ememb Ebolt j 0.250 r 0.347

Plate to bolt modulus: 5.

Calculate Cr = C using equation 15.19 and the coefficients p i from Table 15-8 for j 0.25 . Use linear interpolation between ja 0.2 and jb 0.3 Interpolation equation: p p a p b pb pa jb ja j ja p a p 0 0.6525 p 1 1.2070 p 2 1.1026 p 3 0.3825 C 0.351

Coefficients from Table 15-8:

p 0 p ( 0.6118 0.6932) p 1 p ( 1.1715 1.2426) p 2 p ( 1.0875 1.1177) p 3 p ( 0.3806 0.3845)

Joint stiffness constant: 6.

C p 3 r p 2 r p 1 r p 0

The portions of the applied load P felt by the bolt and the material can now be found from equations 15.13. Pb C P Pb 1.75 kN

MACHINE DESIGN - An Integrated Approach, 4th Ed.

15-52-2

Pm ( 1 C) P 7. Fb Fi Pb Fm Fi Pm 8.

Pm 3.2 kN Fb 26.11 kN Fm 21.11 kN

Find the resulting loads in bolt and material after the load P is applied.

Calculate the alternating and mean components of the fluctuating bolt load. Falt Fb Fi 2 Fb Fi 2 Falt 0.877 kN Fmean 25.23 kN

Fmean 9.

The fatigue stress-concentration factor Kf 3.0 for rolled threads is taken from Table 15-8. The mean stress-concentration factor is taken as Kfm 1 in this case. The mean and alternating stresses in the bolt are:

alt Kf

Falt At Fmean At

alt 45.34 MPa mean 435.11 MPa

mean Kfm
10. The stress at the initial preload is

init Kfm

Fi At

init 420.00 MPa

11. An endurance strength must be found for this material. Using the methods of Section 6.6 we find for S ut 830 MPa S'e 0.5 S ut 12. From the tables and formulas in Section 6.6 we have: Load Size Surface Cload 0.70 Csize 1 A 4.51 Csurf Temperature Reliability and the endurance limit is S e Cload Csize Csurf Ctemp Creliab S'e S e 179.64 MPa b 0.265
b

S'e 415.0 MPa

Sut A MPa

Csurf 0.76

Ctemp 1 Creliab 0.814

13. The corrected endurance strength and the ultimate tensile strength are used in equation 15.16 to find the safety factor from the Goodman line.

MACHINE DESIGN - An Integrated Approach, 4th Ed.


S e S ut init

15-52-3

Nf

S e mean init S ut alt

Nf 1.8

14. Calculate the maximum bolt stress and the safety factor against yielding for S y 660 MPa.

b
Ny

Fb At Sy

b 450.2 MPa
Ny 1.5

15. The load required to separate the joint and the safety factor against joint separation are found from equations 15.14c and 15.14d. Nsep Fi Pmax ( 1 C) Nsep 7.5

16. Use equation 15.23d to calculate the tightening torque. Ti 0.21 Fi d Ti 51.1 N m

MACHINE DESIGN - An Integrated Approach, 4th Ed.

16-1-1

PROBLEM 16-1
Statement: A submerged-arc complete joint penetration (CJP) butt weld was made between two sections of A36 hot-rolled steel plate. The plate is 10-in wide by 1/2-in-thick. E70 electrode was used. How much tensile load can the assembly withstand across the weld without yielding in either base metal or weld? Plate: Electrode: L 10 in S ut 70 ksi T 0.5 in S y 36 ksi

Given:

Assumptions: There are no stress concentrations or residual stresses. The yield strength of the electrode material is S ey 0.75 S ut S ey 52.5 ksi Solution: 1. See Mathcad file P1601.

Calculate the cross-sectional area of both the plate and the weld. A L T A 5 in
2

2.

Both the plate and weld are in pure tension so the tensile stress is the only nonzero principle stress and is also the von Mises stress. Calculate the load that will cause failure in the weld and in the plate. The lesser of the two is the maximum load that can be applied to the joint. Weld Plate Pweld A S ey Pplate A S y Pweld 262.5 kip Pplate 180 kip

The maximum load that can be applied to the joint without yielding is Pplate 180 kip

MACHINE DESIGN - An Integrated Approach, 4th Ed.

16-2-1

PROBLEM 16-2
Statement: The plate of Problem 16-1 has partial joint penetration (PJP) welds applied from each side. If each weld has a throat dimension of 1/4 in, determine the maximum allowable tensile load across the weld. Plate: Electrode: Weld: L 10 in S ut 70 ksi t 0.25 in two places T 0.5 in S y 36 ksi

Given:

Assumptions: The weld will fail in shear along the weld throat and, from equation 16-1, the maximum allowable shear stress is max 0.3 S ut , max 21 ksi Solution: 1. See Mathcad file P1602.

Calculate the cross-sectional areas of the weld and the plate. Aweld L 2 t Aplate L T Aweld 5 in Aplate 5 in
2 2

2.

Calculate the load that will cause failure in the weld and in the plate. Weld: Plate: Pweld Aweld max Pplate Aplate S y Pweld 105 kip Pplate 180 kip

The limiting load is Pweld 105 kip

MACHINE DESIGN - An Integrated Approach, 4th Ed.

16-3-1

PROBLEM 16-3
Statement: A tee bracket similar to that shown in Figure 16-11 is to have 1/2-in-thick A572 Grade 42 steel welded with 3/16-in fillet welds along both inside corners using an E70 electrode. If it will be subjected to a 20-kip tensile load on the leg of the tee, determine the minimum required length, L, of the bracket based on full-length welds. Plates: T 0.5 in S put 70.3 ksi

Given:

Electrode: Exx 70 ksi Weld: w 0.1875 in two places Load: P 20 kip Assumptions: The weld will fail in shear along the weld throat and, from equation 16-1, the maximum allowable shear stress is max 0.3 Exx , max 21 ksi Solution: 1. See Figure 16-11, Table 16-3, and Mathcad file P1603.

Determine the throat dimension of the welds. t 0.707 w t 0.133 in

2.

Determine the shear area in the throats of the welds as a function of L and then solve for L using equation 4.9. Ashear = L 2 t Solving for L, L= L Ashear 2 t P 2 t max and

xy = max =
L 3.592 in

P Ashear

3.

Check whether this value for L gives an acceptable safety factor against yielding in the fused base metal. The shear area on the leg along the width of the welds is Afusion 2 L w , and, from Table 16-3 the yield strength of the base metal is S y 42 ksi. The shear yield strength of the base metal is S sy 0.577 S y . The factor of safety against yielding in the fused base metal is: S sy Afusion P S sy Nyield Afusion 1.347 in
2

Nyield =

xy

Afusion P

S sy

Nyield 1.63 S sy 24.2 ksi

where

and

MACHINE DESIGN - An Integrated Approach, 4th Ed.

16-4-1

PROBLEM 16-4
Statement: Figure P16-1 shows a bar welded to a base on three sides with 3/16-in fillet welds using an E70 electrode. Material is A572 Grafde 50 hot-rolled steel. What is your recommended maximum static load P that can safely be applied? Bar dims: Electrode: Weld: Material: a 6 in Exx 70 ksi w 0.1875 in S y 50 ksi b 3 in three places S ut 65 ksi d 2 in T 0.25 in

Given:

Assumptions: The weld will fail in shear along the weld throat and, from equation 16-1, the maximum allowable shear stress is max 0.3 Exx , max 21 ksi. Consider the weld as a line. The weld pattern matches that of part 5 of Figure 16-15. The weld will limit the load as Category F. Solution: 1. See Figure P16-1, Figure 16-15, and Mathcad file P1604.

Determine the throat dimension of the welds. t 0.707 w t 0.133 in

2.

Determine the shear area in the throats of the welds as a line function. Aw d 2 b Aw 8 in
2

in

3.

Find the centroid of the weld pattern with the equation given in part 5 of Figure 16-5. xbar b
2

2 b d

xbar 1.125 in

4.

Find the moment arm from the applied load to the centroid. r a b xbar r 7.875 in

5.

Find the unit load due to direct shear and the applied torque as functions of P. fs ( P) P Aw
3 2

T ( P) P r

6.

From part 5 of Figure 16-15 the torsional geometry factor is: Jw ( 2 b d) 12 b ( b d) 2 b d


2

Jw 14.542

in

in

7.

The unit load due to torsional shear is a vector that is perpendicular to a line from the centroid to any point on the weld. It is a maximum at a point furthest from the centroid. In this case, it is the furtherest point to the right on either the top or bottom of the bar. In either case, it will have horizontal and vertical components. The vertical component will add to the direct shear and that total will add vectorially to the horizontal component. The moment arm for the horizontal component is d/2, and for the vertical component it is b xbar. Horizontal unit load: fth ( P) ftv ( P) FR( P) T ( P) d 2 Jw T ( P) ( b xbar) Jw fth ( P) fs ( P) f tv ( P)
2 2

Vertical unit load:

Total unit load:

MACHINE DESIGN - An Integrated Approach, 4th Ed.

16-4-2

8.

Solve for P setting the shear stress equal to the maximum allowable stress, which is (0.3)E70. Guess Given t= P 3 kip FR( P) P Find ( P)

max

Recommended maximum static load: P 2.2 kip Primary shear: Shear due to moment: Horizontal component: fth ( P) 1.194 ftv ( P) 2.239 FR( P) 2.784 kip in kip in kip in fs ( P) 0.276 kip in

Vertical component: Total unit load:

MACHINE DESIGN - An Integrated Approach, 4th Ed.

16-5-1

PROBLEM 16-5
Statement: Figure P16-1 shows a bar welded to a base on three sides with 3/16-in fillet welds using an E70 electrode. Material is A572 Grade 50 hot-rolled steel. What is your recommended maximum dynamic repeated load, zero to Pmax that can be applied for 10E8 cycles with a safety factor of 1.6? Bar dims: Electrode: Weld: a 6 in Exx 70 ksi w 0.1875 in b 3 in three places d 2 in Nfr 1.6 T 0.25 in

Given:

Assumptions: The weld will fail in shear along the weld throat. Consider the weld as a line. The weld pattern matches that of part 5 of Figure 16-15. The weld will limit the load as Category F. Solution: 1. See Figure P16-1, Figure 16-15, and Mathcad file P1605.

Determine the throat dimension of the welds. t 0.707 w t 0.133 in

2.

Determine the shear area in the throats of the welds as a line function. Aw d 2 b Aw 8 in
2

in

3.

Find the centroid of the weld pattern with the equation given in part 5 of Figure 16-5. xbar b
2

2 b d

xbar 1.125 in

4.

Find the moment arm from the applied load to the centroid. r a b xbar r 7.875 in

5.

Find the unit load due to direct shear and the applied torque as functions of P. fs ( P) P Aw
3 2

T ( P) P r

6.

From part 5 of Figure 16-15 the torsional geometry factor is: Jw ( 2 b d) 12 b ( b d) 2 b d


2

Jw 14.542

in

in

7.

The unit load due to torsional shear is a vector that is perpendicular to a line from the centroid to any point on the weld. It is a maximum at a point furthest from the centroid. In this case, it is the furtherest point to the right on either the top or bottom of the bar. In either case, it will have horizontal and vertical components. The vertical component will add to the direct shear and that total will add vectorially to the horizontal component. The moment arm for the horizontal component is d/2, and for the vertical component it is b xbar. Horizontal unit load: fth ( P) ftv ( P) FR( P) T ( P) d 2 Jw T ( P) ( b xbar) Jw fth ( P) fs ( P) f tv ( P)
2 2

Vertical unit load:

Total unit load: 8.

From Table 16-5a, a Category F joint has a shear stress-range endurance strength of S ers 8 ksi. Using the given safety factor, the allowable shear stress is

MACHINE DESIGN - An Integrated Approach, 4th Ed.

16-5-2

Allowable shear stress: 9.

allow

S ers Nfr

allow 5 ksi

Solve for P setting the shear stress equal to the maximum allowable stress. Guess Given t= P 1 kip FR( P) Pmax Find ( P) Pmax 525 lbf lbf in

allow

Recommended maximum dynamic load: fs Pmax 65.6

Primary shear: Shear due to moment: Horizontal component:

fth Pmax 284.3 ftv Pmax 533.1 FR Pmax 662.8

lbf in lbf in lbf in

Vertical component: Total unit load:

MACHINE DESIGN - An Integrated Approach, 4th Ed.

16-6a-1

PROBLEM 16-6a
Statement: Figure P16-2 shows a bracket welded to a wall with a fillet weld using an E70 electrode. For the row a in Table P16-1, determine the fillet weld size needed between the tube and the wall for a static load F. The pipe and wall material is A36 steel.

Bracket dims: OD 3.5 in ID 3.068 in a 2 OD l 2.5 OD Electrode: Exx 70 ksi Load: F 2.5 kip Assumptions: The weld will fail in shear along the weld throat and, from equation 16-1, the maximum allowable shear stress is max 0.3 Exx , max 21 ksi. Consider the weld as a line. The weld pattern Given: matches that of part 9 of Figure 16-5. The weld will limit the load as Category F. Solution: 1. See Figure P16-2, Figure 16-15, and Mathcad file P1606a.

Determine the shear area, bending factor, and torsion factor in the throats of the welds as line functions from Figure 16-15, part 9. Aw OD S w OD 4 OD 4
3 2

Aw 10.996 S w 9.621

in
3

in

in

in in
4

Jw 2.

Jw 33.674

in

The weld has a combined bending and torsion load. The maximum stress will occur at the top, center of the pipe at the wall. There will be a direct shear component, a bending component, and a torsional moment component. Find the unit load due to direct shear. fs F Aw fs 227.364 lbf in kip in

3.

4.

Find the unit load due to bending at the critical point. M F l fb M Sw fb 2.274

5.

Find the unit load due to the torsional moment. T F a ft T OD 2 Jw ft 0.909 kip in

6.

Find the magnitude of the resultant unit load at the critical point ( fs is in the negative y-direction, f b is in the positive x-direction, and ft is in the negative z-direction). Total unit load: FR fs fb ft
2 2 2

FR 2.459

kip in

7.

Using equation 16.1, solve for the minimum required weld size. t FR

max

w 1.414 t w 0.166 in Say w = 3/16 in

t 0.117 in

MACHINE DESIGN - An Integrated Approach, 4th Ed.

16-7a-1

PROBLEM 16-7a
Statement: Figure P16-2 shows a bracket welded to a wall with a fillet weld using an E70 electrode. For the row a in Table P16-1, determine the fillet weld size needed between the tube and the wall for a dynamic load that varies from - 0.1F to +0.2F using a safety factor of 1.5. The pipe and wall material is A36 steel. Bracket dims: Electrode: Load: OD 3.5 in Exx 70 ksi F 2.5 kip ID 3.068 in Nfr 1.5 a 2 OD l 2.5 OD

Given:

Assumptions: The weld will fail in shear along the weld throat. Consider the weld as a line. The weld pattern matches that of part 9 of Figure 16-15. The weld will limit the load as Category F. Solution: 1. See Figure P16-2, Figure 16-15, and Mathcad file P1607a.

Determine the dynamic load range.

F 0.2 F ( 0.1F )
2.

F 750 lbf

Determine the shear area, bending factor, and torsion factor in the throats of the welds as line functions from Figure 16-15, part 9. Aw OD S w OD 4 OD 4
3 2

Aw 10.996 S w 9.621

in
3

in

in

in in
4

Jw 3.

Jw 33.674

in

The weld has a combined bending and torsion load. The maximum stress will occur at the top, center of the pipe at the wall. There will be a direct shear component, a bending component, and a torsional moment component. Find the unit load due to direct shear. fs

4.

F
Aw

fs 68.209

lbf in kip in

5.

Find the unit load due to bending at the critical point. M F l fb M Sw fb 0.682

6.

Find the unit load due to the torsional moment. T F a ft T OD 2 Jw ft 0.273 kip in

7.

Find the magnitude of the resultant unit load at the critical point ( fs is in the negative y-direction, f b is in the positive x-direction, and ft is in the negative z-direction). Total unit load: FR fs fb ft
2 2 2

FR 0.738

kip in

8.

From Table 16-5a, a Category F joint has a shear stress-range endurance strength of S ers 8 ksi. Using the given safety factor, the allowable shear stress is

MACHINE DESIGN - An Integrated Approach, 4th Ed.


Allowable shear stress: 9.

16-7a-2

allow

S ers Nfr

allow 5.333 ksi

Using equation 16.1, solve for the minimum required weld size. FR

allow

w 1.414 t w 0.196 in Say w = 1/4 in

t 0.138 in

MACHINE DESIGN - An Integrated Approach, 4th Ed.

16-8-1

PROBLEM 16-8
Statement: Figure P16-3 shows a bracket machined from 12-mm-thick A572 Grade 50 hot-rolled steel flat stock. It is welded to a support with a fillet weld all around using an E80 electrode. Determine the fillet weld size needed between the bracket and the support for a static load of P = 12 kN. Distance from support to point D Bracket dims: d 75 mm Electrode: Exx 80 ksi Load: P 12 kN a 200 mm b 12 mm Exx 552 MPa

Given:

Assumptions: The weld will fail in shear along the weld throat and, from equation 16-1, the maximum allowable shear stress is max 0.3 Exx , max 165 MPa. Consider the weld as a line. The weld pattern matches that of part 7 of Figure 16-5. The weld will limit the load as Category F. Solution: 1. See Figure P16-2, Figure 16-15, and Mathcad file P1608.

Determine the shear area and bending factor, in the throats of the welds as line functions from Figure 16-15, part 7. Aw 2 b 2 d S w b d d
2

Aw 174

mm

mm mm
3

S w 2775

mm

2. 3.

The weld has a combined bending and shear load. The maximum stress will occur at the top, center of the bracket at the support. There will be a direct shear component and a bending component. Find the unit load due to direct shear. fs P Aw fs 0.069 kN mm kN mm

4.

Find the unit load due to bending at the critical point. M P a fb M Sw fb 0.865

5.

Find the magnitude of the resultant unit load at the critical point. Total unit load: FR fs fb
2 2

FR 0.868

kN mm

6.

Using equation 16.1, solve for the minimum required weld size. t FR w 1.414 t w 7.41 mm Say w = 8 mm

max

t 5.24 mm

MACHINE DESIGN - An Integrated Approach, 4th Ed.

16-9-1

PROBLEM 16-9
Statement: Figure P16-3 shows a bracket machined from 12-mm-thick A572 Grade 50 hot-rolled steel flat stock. It is welded to a support with a fillet weld all around using an E90 electrode. Determine the fillet weld size needed between the bracket and the support for a dynamic load that varies from 0 to +3 kN using a safety factor of 1.8. Distance from support to point D Bracket dims: d 75 mm Electrode: Exx 90 ksi Load: F 3 kN a 200 mm b 12 mm Exx 621 MPa Nfr 1.8

Given:

Assumptions: The weld will fail in shear along the weld throat. Consider the weld as a line. The weld pattern matches that of part 7 of Figure 16-15. The weld will limit the load as Category F. Solution: 1. See Figure P16-2, Figure 16-15, and Mathcad file P1609.

Determine the dynamic load range.

F F 0 kN
2.

F 3 kN

Determine the shear area and bending factor, in the throats of the welds as line functions from Figure 16-15, part 7. Aw 2 b 2 d S w b d d
2

Aw 174

mm

mm mm
3

S w 2775

mm

3. 4.

The weld has a combined bending and shear load. The maximum stress will occur at the top, center of the bracket at the support. There will be a direct shear component and a bending component. Find the unit load due to direct shear. F fs Aw kN mm

fs 0.017

5.

Find the unit load due to bending at the critical point. M F a fb M Sw fb 0.216 kN mm

6.

Find the magnitude of the resultant unit load at the critical point. Total unit load: FR fs fb
2 2

FR 0.217

kN mm

8.

From Table 16-5b, a Category F joint has a shear stress-range endurance strength of S ers 55 MPa. Using the given safety factor, the allowable shear stress is Allowable shear stress: S ers Nfr

allow

allow 30.6 MPa

9.

Using equation 16.1, solve for the minimum required weld size. t FR w 1.414 t w 10.037 mm Say w = 10 mm

allow

t 7.099 mm

MACHINE DESIGN - An Integrated Approach, 4th Ed.

16-10-1

PROBLEM 16-10
Statement: Two 8-mm-thick by 50-mm-wide, A572 Grade 42 steel straps are welded together with fillet welds in a lap joint using an E70 electrode. The tensile load on the straps is 45 kN. What is your recommended weld size for the two full-length welds? Straps: Electrode: Load: T 8 mm Exx 70 ksi P 45 kN W 50 mm Exx 483 MPa S y 345 MPa

Given:

Assumptions: Each weld will carry half the load and will fail in shear along the weld throat and, from equation 16-1, the maximum allowable shear stress is max 0.3 Exx , max 145 MPa Solution: 1. See Figures 16-2 and 16-3, Table 16-1, and Mathcad file P1610.

Determine the transverse load carried by each weld. Transverse load on weld Pt P 2 Pt 22.5 kN

2.

Determine the shear area in the throats of the welds as a function of t and then solve for t using equation 16.1. Ashear = W t Solving for t, t= t Ashear W Pt W max and

xy = max =
t 3.1 mm

Pt Ashear

3.

Convert this throat dimension to a leg width w assuming an equal-leg fillet. Leg width w t cos( 45 deg) w 4.4 mm

4.

Check this against the recommended minimum weld size for this thickness strap. Table 16-2 indicates that a 8-mm-thick part needs at least a 5-mm weld leg width, so increase the weld leg width to 5 mm. Set w 5 mm Check whether this value for w gives an acceptable safety factor against yielding in the fused base metal. The shear area on the leg along the width of the welds is Afusion W w , and, from Table 16-3 the yield strength of the base metal is S y 290 MPa. The shear yield strength of the base metal is S sy 0.577 S y . The factor of safety against yielding in the fused base metal is: S sy Afusion Pt S sy Nyield Afusion 250 mm Afusion Pt
2

5.

Nyield =

xy

S sy

Nyield 1.9 S sy 167 MPa

where

and

This is slightly less than the safety factor against static failure in the weld (see Table 16-1) but is acceptable.

MACHINE DESIGN - An Integrated Approach, 4th Ed.

16-11-1

PROBLEM 16-11
Statement: Two 8-mm-thick by 50-mm-wide, A572 Grade 42 steel straps are welded together with fillet welds in a lap joint using an E70 electrode. Determine the fillet weld size needed for a dynamic load that varies from 0 to +12 kN using a safety factor of 1.5 for an infinite life of the two full-length welds. Straps: Electrode: Load: T 8 mm Exx 70 ksi Fmin 0 kN W 50 mm S ut 485 MPa

Given:

Exx 483 MPa Fmax 12 kN Nfrs 1.5

Assumptions: Each weld will carry half the load and will fail in shear along the weld throat. Since the weld takes the load directly this is a Category F weldment. Solution: 1. See Figure P16-2, Figure 16-15, and Mathcad file P1611. Fmax Fmin 2

Determine the dynamic load range per weld.

F
2.

F 6 kN

From Table 16-5b, a Category F joint has a shear stress-range endurance strength of S ers 55 MPa. Using the given safety factor, the allowable shear stress is Allowable shear stress: S ers Nfrs

allow

allow 36.7 MPa

3.

Using equation 4.9, solve for the allowable shear area and the resulting minimum value of the throat dimension t. Allowable shear area on each throat Minimum throat dimension Ashear

F allow

Ashear 163.6 mm

Ashear W

t 3.273 mm

4.

Convert this throat dimension to a leg width w assuming an equal-leg fillet. Leg width w t cos( 45 deg) w 4.6 mm

5. 6.

Check this against the recommended minimum weld size for this thickness strap. Table 16-2 indicates that a 8-mm-thick part needs at least a 5-mm weld leg width, so increase the weld leg width to 5 mm. Set w 5 mm Check whether this value for w gives an acceptable safety factor against fatigue failure in the fused base metal by assuming that this is a Category C weldment. The most likely point of failure is at the weld toe. The weld leg area is Afusion W w , and, from Table 16-5b the endurance strength is S er 69 MPa Max stress at toe of each weld Factor of safety in base metal

toe

Fmax 2 Afusion S er

toe 24 MPa

Nf

toe

Nf 2.9

Since Nf is larger than Nfrs, the initial assumption that this was a Category F weldment was correct.

MACHINE DESIGN - An Integrated Approach, 4th Ed.

16-12-1

PROBLEM 16-12
Statement: Two 12-mm-thick by 50-mm-wide, weldable aluminum straps are welded together with fillet welds in a lap joint using an aluminum electrode. Determine the fillet weld size needed for a dynamic load that varies from 0 to +5 kN using a safety factor of 2.0 for an infinite life of the two full-length welds. Straps: Load: T 12 mm Fmin 0 kN W 50 mm Fmax 5 kN

Given:

Nfrs 2.0

Assumptions: Each weld will carry half the load and will fail in shear along the weld throat. Since the weld takes the load directly this is a Category F weldment. Solution: 1. See Figure P16-2, Figure 16-15, and Mathcad file P1612. Fmax Fmin 2

Determine the dynamic load range per weld.

F
2.

F 2.5 kN

From Table 16-5b and the notation in the text that the values in Table 16-5 can be reduced by a factor of three for aluminum, a Category F joint has a shear stress-range endurance strength of S ers 0.333 55 MPa S ers 18.3 MPa. Using the given safety factor, the allowable shear stress is Allowable shear stress:

allow

S ers Nfrs

allow 9.2 MPa

3.

Using equation 4.9, solve for the allowable shear area and the resulting minimum value of the throat dimension t. Allowable shear area on each throat Minimum throat dimension Ashear

F allow

Ashear 273 mm

Ashear W

t 5.46 mm

4.

Convert this throat dimension to a leg width w assuming an equal-leg fillet. Leg width w t cos( 45 deg) w 7.7 mm

5.

Check this against the recommended minimum weld size for this thickness strap. Table 16-2 indicates that a 12-mm-thick part needs at least a 5-mm weld leg width. The calculated leg width is adequate but increase it to the next higher mm integer. Set w 8 mm Check whether this value for w gives an acceptable safety factor against fatigue failure in the fused base metal by assuming that this is a Category C weldment. The most likely point of failure is at the weld toe. The weld leg area is Afusion W w , and, from Table 16-5b the endurance strength is S er 0.333 69 MPa S er 23.0 MPa Max stress at toe of each weld Factor of safety in base metal

6.

toe

Fmax 2 Afusion S er

toe 6.25 MPa

Nf

toe

Nf 3.7

Since Nf is larger than Nfrs, the initial assumption that this was a Category F weldment was correct.

MACHINE DESIGN - An Integrated Approach, 4th Ed.

17-1-1

PROBLEM 17-1
Statement: Find the torque that a 2-surface, dry disk clutch can transmit if the outside and inside lining diameters are 120 mm and 70 mm, respectively, and the applied axial force is 10 kn. Assume uniform wear and m = 0.4. Is the pressure on the lining acceptable? What lining materials would be suitable? Number of surfaces Axial force Friction coefficient Assumptions: Uniform wear model. Solution: 1. See Mathcad file P1701. ro 60 mm ri 35 mm T Ns F ro ri 2 T 380 N m Ns 2 F 10 kN 0.4 Clutch dimensions: Outside diameter Inside diameter od 120 mm id 70 mm

Given:

Calculate the outside and inside radii of the disk. Outside radius ro 0.5 od Inside radius ri 0.5 id

2.

Using equation 17.6, calculate the torque capacity.

3.

Calculate the maximum lining pressure using equation 17.5a. Maximum pressure p max 2 ri ro ri F

p max 1819 kPa 4. From Table 17-1, we see that either a molded or sintered metal lining are suitable.

MACHINE DESIGN - An Integrated Approach, 4th Ed.

17-2-1

PROBLEM 17-2
Statement: Find the torque that a 2-surface, dry disk clutch can transmit if the outside and inside lining diameters are 120 mm and 70 mm, respectively, and the applied axial force is 10 kn. Assume uniform pressure and = 0.4. Is the pressure on the lining acceptable? What lining materials would be suitable? Number of surfaces Axial force Friction coefficient Assumptions: Uniform pressure model. Solution: 1. See Mathcad file P1702. Ns 2 F 10 kN 0.4 Clutch dimensions: Outside diameter Inside diameter od 120 mm id 70 mm

Given:

Calculate the outside and inside radii of the disk. Outside radius Inside radius ro 0.5 od ri 0.5 id ro 60 mm ri 35 mm

2.

Using equation 17.3, calculate the torque capacity.

ro3 ri3 T Ns F 3 2 2 ro ri
2 T
3 3 Ns ro ri

T 389 N m

3.

Calculate the lining pressure using equation 17.2c. Maximum pressure p 3 2 p 1340 kPa

4.

From Table 17-1, we see that either a molded or sintered metal lining are suitable.

MACHINE DESIGN - An Integrated Approach, 4th Ed.

17-3-1

PROBLEM 17-3
Statement: Design a single-surface disk clutch to transmit 100 N-m of torque at 750 rpm using a molded lining with a maximum pressure of 1000 kPa and = 0.25. Assume uniform wear. Find the outside and inside diameters required if ri = 0.577 ro. What is the power transmitted? Number of surfaces Friction coefficient Disk radius factor Assumptions: Uniform wear model. Solution: 1. See Mathcad file P1703. Ns 1 Torque capacity Maximum pressure Rotational speed T 100 N m p max 1000 kPa

Given:

0.25
fr 0.577

750 rpm

Use equation 17.5b and the disk radius factor to solve for the required outside and inside radii. T = ri p max ro ri
2 2

1 3

ri = fr ro

ro

T 2 fr p max 1 fr

ro 69.16 mm

Round this to

ro 70 mm ri fr ro ri 40.39 mm

Then, the inside radius is Round this to 2. Convert the radii to diameters. Outside diameter Inside diameter 3. Calculate the maximum transmitted power. Transmitted power ri 40 mm

od 2 ro id 2 ri

od 140 mm id 80 mm

H T

H 7.85 kW

MACHINE DESIGN - An Integrated Approach, 4th Ed.

17-4-1

PROBLEM 17-4
Statement: Design a single-surface disk clutch to transmit 100 N-m of torque at 750 rpm using a molded lining with a maximum pressure of 1000 kPa and = 0.25. Assume uniform pressure. Find the outside and inside diameters required if ri = 0.577 ro. What is the power transmitted? Number of surfaces Friction coefficient Disk radius factor Assumptions: Uniform pressure model. Solution: 1. See Mathcad file P1704. Ns 1 Torque capacity Maximum pressure Rotational speed T 100 N m p 1000 kPa 750 rpm

Given:

0.25
fr 0.577

Use equation 17.2c and the disk radius factor to solve for the required outside and inside radii. T= 2 3 p ro ri
3 3

Ns
1 3

ri = fr ro

3 T ro 1 fr 3 2 p N s Round this to

ro 61.832 mm

ro 62 mm ri fr ro ri 35.774 mm

Then, the inside radius is Round this to 2. Convert the radii to diameters. Outside diameter Inside diameter 3. Calculate the maximum transmitted power. Transmitted power ri 36 mm

od 2 ro id 2 ri

od 124 mm id 72 mm

H T

H 7.85 kW

MACHINE DESIGN - An Integrated Approach, 4th Ed.

17-5-1

PROBLEM 17-5
Statement: How many surfaces are needed in a wet disk clutch to transmit 120 N-m of torque at 1000 rpm using a sintered lining with a maximum pressure of 1800 kPa and = 0.06. Assume uniform wear. Find the outside and inside diameters required if ri = 0.577 ro. How many disks are needed? What is the power transmitted? Friction coefficient Disk radius factor Rotational speed Assumptions: Uniform wear model. Design Choices: The number of surfaces depends on the disk dimensions. Let ri 30 mm Solution: 1. 2. See Mathcad file P1705. ro ri fr ro 51.993 mm ro 52 mm

Given:

0.06
fr 0.577

Torque capacity Maximum pressure

T 120 N m p max 1800 kPa

1000 rpm

Calculate the outside radius. Convert the radii to diameters. Outside diameter Inside diameter

od 2 ro id 2 ri

od 104 mm id 60 mm

3.

Calculate the required axial force using equation 17.5a. Axial force F 2 ri p max ro ri F 7.464 kN

4.

Using equation 17.6, calculate the number of friction surfaces required. Number surfaces Ns ceil

F ro ri

2 T

Ns 7

5.

Calculate the maximum transmitted power. Transmitted power H T H 12.6 kW

MACHINE DESIGN - An Integrated Approach, 4th Ed.

17-6-1

PROBLEM 17-6
Statement: How many surfaces are needed in a wet disk clutch to transmit 120 N-m of torque at 1000 rpm using a sintered lining with a maximum pressure of 1800 kPa and = 0.06. Assume uniform pressure. Find the outside and inside diameters required if ri = 0.577 ro. How many disks are needed? What is the power transmitted? Friction coefficient Disk radius factor Rotational speed Assumptions: Uniform pressure model. Design Choices: The number of surfaces depends on the disk dimensions. Let ri 30 mm Solution: 1. 2. See Mathcad file P1706. ro ri fr ro 51.993 mm ro 52 mm

Given:

0.06
fr 0.577

Torque capacity Pressure

T 120 N m p 1800 kPa

1000 rpm

Calculate the outside radius. Convert the radii to diameters. Outside diameter Inside diameter

od 2 ro id 2 ri F p ro ri
2 2

od 104 mm id 60 mm

3.

Calculate the required axial force using equation 17.1b. Axial force

F 10.201 kN

4.

Using equation 17.3, calculate the number of friction surfaces required.

Number surfaces

3 T ro2 ri2 Ns ceil F r 3 r 3 i 2 o


H T

Ns 5

5.

Calculate the maximum transmitted power. Transmitted power H 12.6 kW

MACHINE DESIGN - An Integrated Approach, 4th Ed.

17-7-1

PROBLEM 17-7
Statement: Given: Figure P17-1 shows a single short-shoe drum brake. Find its torque capacity and required actuating force for the dimensions given below. What value of c will make it self-locking? Pivot to load Pivot to y-axis Pivot to x-axis Maximum pressure a 100mm b 70 mm e 20 mm p max 1300 kPa Drum radius Drum width Shoe angle Friction coeff. r 30 mm w 50 mm 35 deg 0.3

Assumptions: Short-shoe theory is appropriate. The drum rotates CCW. Solution: 1. See Figure P17-1 and Mathcad file P1707.

Determine the normal force on the drum from equation 17.8. Normal force Fn p max r w Fn 1.191 kN

2.

Use equation 17.10 to calculate the torque capacity. Torque capacity T Fn r T 10.7 N m

3.

Determine the required actuating force from equation 17.11b and the brake geometry. Distance c Actuation force c r e Fa Fn b c a c 10 mm Fa 798 N

4.

Check to see if the brake is self-locking using the relationship given in the text. self_locking return "yes" if c b "no" otherwise self_locking "no"

5.

Calculate the value of c that would make the brake self-locking use the above relationship. Value of c to self-lock clock b clock 233.3 mm

MACHINE DESIGN - An Integrated Approach, 4th Ed.

17-8-1

PROBLEM 17-8
Statement: Given: Figure P17-1 shows a single short-shoe drum brake. Find its torque capacity and required actuating force for the dimensions given below. What value of c will make it self-locking? Pivot to load Pivot to y-axis Pivot to x-axis Maximum pressure a 100mm b 70 mm e 20 mm p max 1300 kPa Drum radius Drum width Shoe angle Friction coefficient r 30 mm w 50 mm 35 deg 0.3

Assumptions: Short-shoe theory is appropriate. The drum rotates CW. Solution: 1. See Figure P17-1 and Mathcad file P1708.

Determine the normal force on the drum from equation 17.8. Normal force Fn p max r w Fn 1.191 kN

2.

Use equation 17.10 to calculate the torque capacity. Torque capacity T Fn r T 10.7 N m

3.

Determine the required actuating force from equation 17.11b and the brake geometry modified for CW rotation. Distance c Actuation force c r e Fa Fn b c a c 10 mm Fa 870 N

4.

Since the brake is not self-energizing with CW rotation, it cannot be self-locking for any value of c.

MACHINE DESIGN - An Integrated Approach, 4th Ed.

17-9-1

PROBLEM 17-9
Statement: Figure P17-1 shows a single short-shoe drum brake. Find its torque capacity and required actuating force for the dimensions given below. What value of c will make it self-locking? Pivot to load Pivot to y-axis Pivot to x-axis Maximum pressure a 8.00 in b 6.00 in e 4.00 in p max 250 psi Drum radius Drum width Shoe angle Friction coeff. r 5.00 in w 1.50 in 30 deg 0.35

Given:

Assumptions: Short-shoe theory is appropriate. The drum rotates CCW. Solution: 1. See Figure P17-1 and Mathcad file P1709.

Determine the normal force on the drum from equation 17.8. Normal force Fn p max r w Fn 981.7 lbf

2.

Use equation 17.10 to calculate the torque capacity. Torque capacity T Fn r T 1718 in lbf

3.

Determine the required actuating force from equation 17.11b and the brake geometry. Distance c Actuation force c r e Fa Fn b c a c 1.000 in Fa 693 lbf

4.

Check to see if the brake is self-locking using the relationship given in the text. self_locking return "yes" if c b "no" otherwise self_locking "no"

5.

Calculate the value of c that would make the brake self-locking use the above relationship. Value of c to self-lock clock b clock 17.1 in

MACHINE DESIGN - An Integrated Approach, 4th Ed.

17-10-1

PROBLEM 17-10
Statement: Figure P17-1 shows a single short-shoe drum brake. Find its torque capacity and required actuating force for the dimensions given below. What value of c will make it self-locking? Pivot to load Pivot to y-axis Pivot to x-axis Maximum pressure a 8.00 in b 6.00 in e 4.00 in p max 250 psi Drum radius Drum width Shoe angle Friction coeff. r 5.00 in w 1.50 in 30 deg 0.35

Given:

Assumptions: Short-shoe theory is appropriate. The drum rotates CW. Solution: 1. See Figure P17-1 and Mathcad file P1710.

Determine the normal force on the drum from equation 17.8. Normal force Fn p max r w Fn 981.7 lbf

2.

Use equation 17.10 to calculate the torque capacity. Torque capacity T Fn r T 1718 in lbf

3.

Determine the required actuating force from equation 17.11b and the brake geometry modified for CW rotation. Distance c Actuation force c r e Fa Fn b c a c 1.000 in Fa 779 lbf

4.

Since the brake is not self-energizing with CW rotation, it cannot be self-locking for any value of c.

MACHINE DESIGN - An Integrated Approach, 4th Ed.

17-11-1

PROBLEM 17-11
Statement: Figure P17-2 shows a double short-shoe drum brake. Find its torque capacity and required actuating force for the dimensions given below. What value of c will make it self-locking? Hint: Calculate the effects of each shoe separately and superpose them. Pivot to load Pivot to y-axis Pivot to x-axis Maximum pressure a 90mm b 80 mm e 30 mm p max 1500 kPa Drum radius Drum width Shoe angle Friction coefficient r 40 mm w 60 mm 25 deg 0.25

Given:

Assumptions: Short-shoe theory is appropriate. The drum rotates CCW. Solution: See Figure P17-2 and Mathcad file P1711.

Top shoe - self energizing 1. Determine the normal force on the drum from equation 17.8. Normal force 2. Fn1 p max r w Fn1 1.571 kN

Use equation 17.10 to calculate the torque capacity. Torque capacity T1 Fn1 r T1 15.71 N m

3.

Determine the required actuating force from equation 17.11b and the brake geometry. Distance c Actuation force c r e Fa Fn1 b c a c 10 mm Fa 1353 N

4.

Check to see if the brake is self-locking using the relationship given in the text. self_locking return "yes" if c b "no" otherwise self_locking "no"

5.

Calculate the value of c that would make the brake self-locking use the above relationship. Value of c to self-lock clock b clock 320 mm

Bottom shoe - non self energizing 6. The actuation force is the same on both shoes and is equal to the actuation force on the top shoe. Use equation 17.11b to solve for the normal force on the bottom shoe. Normal force 7. Fn2 Fa a b c Fn2 1476 N

Use equation 17.10 to calculate the torque capacity. Torque capacity T2 Fn2 r T2 14.76 N m

Both shoes 8. The total torque capacity is Ttot T1 T2 Ttot 30.5 N m

MACHINE DESIGN - An Integrated Approach, 4th Ed.

17-12-1

PROBLEM 17-12
Statement: Figure P17-2 shows a double short-shoe drum brake. Find its torque capacity and required actuating force for the dimensions given below. What value of c will make it self-locking? Hint: Calculate the effects of each shoe separately and superpose them. Pivot to load Pivot to y-axis Pivot to x-axis Maximum pressure a 12.00 in b 8.00 in e 3.00 in p max 200 psi Drum radius Drum width Shoe angle Friction coeff. r 6.00 in w 2.00 in 25 deg 0.28

Given:

Assumptions: Short-shoe theory is appropriate. The drum rotates CCW. Solution: See Figure P17-2 and Mathcad file P1712.

Top shoe - self energizing 1. Determine the normal force on the drum from equation 17.8. Normal force 2. Fn1 p max r w Fn1 1047 lbf

Use equation 17.10 to calculate the torque capacity. Torque capacity T1 Fn1 r T1 1759 in lbf

3.

Determine the required actuating force from equation 17.11b and the brake geometry. Distance c Actuation force c r e Fa Fn1 b c a c 3.000 in Fa 625 lbf

4.

Check to see if the brake is self-locking using the relationship given in the text. self_locking return "yes" if c b "no" otherwise self_locking "no"

5.

Calculate the value of c that would make the brake self-locking use the above relationship. Value of c to self-lock clock b clock 28.6 in

Bottom shoe - non self energizing 6. The actuation force is the same on both shoes and is equal to the actuation force on the top shoe. Use equation 17.11b to solve for the normal force on the bottom shoe. Normal force 7. Fn2 Fa a b c Fn2 848 lbf

Use equation 17.10 to calculate the torque capacity. Torque capacity T2 Fn2 r T2 1425 in lbf

Both shoes 8. The total torque capacity is Ttot T1 T2 Ttot 3184 in lbf

MACHINE DESIGN - An Integrated Approach, 4th Ed.

17-13-1

PROBLEM 17-13
Statement: Given: Figure P17-3 shows a single long-shoe drum brake. Find its torque capacity and required actuating force for the dimensions given below. Pivot to load Pivot to Y-axis Pivot to X-axis Maximum pressure Friction coeff. a x 100mm b x 70 mm b y 20 mm p max 1300 kPa Drum radius Drum width Shoe start angle Shoe end angle r 30 mm w 50 mm

1 25 deg 2 125 deg

0.3

Assumptions: Long-shoe theory is appropriate. The drum rotates CCW and the brake is self-energizing. Solution: 1. See Figure P17-3 and Mathcad file P1713.

Determine the maximum shoe angle.

max

return 90 deg if 2 90 deg

2 otherwise max 90 deg


2. Use equation 17.15 to calculate the torque capacity. Torque capacity T w r
2

sin max

p max

cos 1 cos 2

T 26.0 N m 3. Determine the required actuating force from equations 17.14 and the brake geometry. Dimension b Normal moment MFn w r b 1 1 2 1 sin 2 2 sin 2 1 2 4 sin max p max MFn 184.423 N m Friction moment MFf w r sin max p max r cos 2 cos 1 b 2 sin 2 sin 1
2 2

bx by

b 72.801 mm

MFf 15.487 N m Fa MFn MFf ax Fa 1689 N

Actuating force

MACHINE DESIGN - An Integrated Approach, 4th Ed.

17-14-1

PROBLEM 17-14
Statement: Figure P17-3 shows a single long-shoe drum brake. Find its torque capacity and required actuating force for the dimensions given below. Pivot to load Pivot to Y-axis Pivot to X-axis Maximum pressure Friction coefficient a x 8.00 in b x 6.00 in b y 4.00 in p max 250 psi Drum radius Drum width Shoe start angle Shoe end angle r 5.00 in w 1.50 in

Given:

1 35 deg 2 155 deg

0.35

Assumptions: Long-shoe theory is appropriate. The drum rotates CCW and the brake is self-energizing. Solution: 1. See Figure P17-3 and Mathcad file P1714.

Determine the maximum shoe angle.

max

return 90 deg if 2 90 deg

2 otherwise max 90 deg


2. Use equation 17.15 to calculate the torque capacity. Torque capacity T w r
2

sin max

p max

cos 1 cos 2

T 5662 in lbf 3. Determine the required actuating force from equations 17.14 and the brake geometry. Dimension b Normal moment MFn w r b 1 1 2 1 sin 2 2 sin 2 1 2 4 sin max p max MFn 19925 in lbf Friction moment MFf w r sin max p max r cos 2 cos 1 b 2 sin 2 sin 1
2 2

bx by

b 7.211 in

MFf 6017 in lbf Fa MFn MFf ax Fa 1738 lbf

Actuating force

MACHINE DESIGN - An Integrated Approach, 4th Ed.

17-15-1

PROBLEM 17-15
Statement: Given: Figure P17-4 shows a double long-shoe drum brake. Find its torque capacity and required actuating force for the dimensions given below. Pivot to load Pivot to Y-axis Pivot to X-axis Maximum pressure Friction coeff. a x 90mm b x 80 mm b y 30 mm p max 1500 kPa Drum radius Drum width Shoe start angle Shoe end angle r 40 mm w 30 mm

1 30 deg 2 160 deg

0.25

Assumptions: Long-shoe theory is appropriate. The drum rotates CCW. Solution: See Figure P17-4 and Mathcad file P1715.

Top shoe - self-energizing 1. Determine the maximum shoe angle.

max

return 90 deg if 2 90 deg

2 otherwise max 90 deg


2. Use equation 17.15 to calculate the torque capacity. Torque capacity T1 w r
2

sin max

p max

cos 1 cos 2

T1 32.50 N m 3. Determine the required actuating force from equations 17.14 and the brake geometry. Dimension b Normal moment MFn w r b 1 1 2 1 sin 2 2 sin 2 1 2 4 sin max p max MFn 232.482 N m Friction moment MFf w r sin max p max r cos 2 cos 1 b 2 sin 2 sin 1
2 2

bx by

b 85.44 mm

MFf 35.06 N m Fa MFn MFf ax Fa 2194 N

Actuating force Bottom shoe - nonself-energizing 4.

The magnitude of the actuation force is the same on both shoes and is equal to the actuation force on the top shoe. This will result in a lower maximum pressure on the lower, nonself-energizing, shoe. Determine the maximum pressure on the lower shoe by solving equation 17.14c (using the plus sign) for p max2.

MACHINE DESIGN - An Integrated Approach, 4th Ed.


a x Fa w r sin max
2 2 2 b sin( ) d sin( ) ( r b cos( ) ) d 1 1

17-15-2

p max2

p max2 1107 kPa 5. Use equation 17.15 to calculate the torque capacity. Torque capacity T2 w r
2

sin max

p max2

cos 1 cos 2

T2 23.98 N m 6. The torque capacity of the double brake is the sum of the torque capacity of the two. Total torque capacity Ttot T1 T2 Ttot 56.5 N m

MACHINE DESIGN - An Integrated Approach, 4th Ed.

17-16-1

PROBLEM 17-16
Statement: Given: Figure P17-4 shows a double long-shoe drum brake. Find its torque capacity and required actuating force for the dimensions given below. Pivot to load Pivot to Y-axis Pivot to X-axis Maximum pressure Friction coeff. a x 12.00 in b x 8.00 in b y 3.00 in p max 200 psi Drum radius Drum width Shoe start angle Shoe end angle r 6.00 in w 2.00 in

1 25 deg 2 145 deg

0.28

Assumptions: Long-shoe theory is appropriate. The drum rotates CCW. Solution: See Figure P17-4 and Mathcad file P1716.

Top shoe - self-energizing 1. Determine the maximum shoe angle.

max

return 90 deg if 2 90 deg

2 otherwise max 90 deg


2. Use equation 17.15 to calculate the torque capacity. Torque capacity T1 w r
2

sin max

p max

cos 1 cos 2

T1 6957 in lbf 3. Determine the required actuating force from equations 17.14 and the brake geometry. Dimension b Normal moment MFn w r b 1 1 2 1 sin 2 2 sin 2 1 2 4 sin max p max MFn 30218 in lbf Friction moment MFf w r sin max p max r cos 2 cos 1 b 2 sin 2 sin 1
2 2

bx by

b 8.544 in

MFf 6525 in lbf Fa MFn MFf ax Fa 1974 lbf

Actuating force

Bottom shoe - nonself-energizing 4. The magnitude of the actuation force is the same on both shoes and is equal to the actuation force on the top shoe. This will result in a lower maximum pressure on the lower, nonself-energizing, shoe. Determine the maximum pressure on the lower shoe by solving equation 17.14c (using the plus sign) for p max2.

MACHINE DESIGN - An Integrated Approach, 4th Ed.


p max2 a x Fa w r sin max
2 2 2 b sin( ) d sin( ) ( r b cos( ) ) d 1 1

17-16-2

p max2 129.0 psi 5. Use equation 17.15 to calculate the torque capacity. Torque capacity T2 w r
2

sin max

p max2

cos 1 cos 2

T2 4486 in lbf 6. The torque capacity of the double brake is the sum of the torque capacity of the two. Total torque capacity Ttot T1 T2 Ttot 11443 in lbf

MACHINE DESIGN - An Integrated Approach, 4th Ed.

17-17a-1

PROBLEM 17-17a
Statement: The short-shoe approximation is considered to be valid for brake shoes with an included angle of up to about 45 deg. For the brake shown in Figure P17-3, calculate its torque capacity and required actuating force by both the short-shoe method and the long-shoe method and compare the results for the dimensions given below. Pivot to load Pivot to Y-axis Pivot to X-axis Maximum pressure Friction coefficient a x 90mm b x 80 mm b y 30 mm p max 1500 kPa Drum radius Drum width Shoe start angle Shoe end angle r 40 mm w 30 mm

Given:

1 75 deg 2 105 deg

0.3

Assumptions: The drum rotates CCW and the brake is self-energizing. Solution: See Figure P17-3 and Mathcad file P1717a.

Short-shoe method 1. 2. Calculate the total shoe angle.

2 1

30 deg

Determine the normal force on the drum from equation 17.8. Normal force Fn p max r w Fn 0.942 kN

3.

Use equation 17.10 to calculate the torque capacity. Torque capacity Tshort Fn r Tshort 11.3 N m

4.

Determine the required actuating force from equation 17.11b and the brake geometry. Distance c Actuation force c r b y Fashort Fn bx c ax c 10 mm Fashort 806 N

Long-shoe method 5. Determine the maximum shoe angle.

max

return 90 deg if 2 90 deg

2 otherwise max 90 deg


6. Use equation 17.15 to calculate the torque capacity. Torque capacity Tlong w r
2

sin max

p max

cos 1 cos 2

Tlong 11.2 N m 7. Determine the required actuating force from equations 17.14 and the brake geometry. Dimension b Normal moment MFn w r b 1 1 2 1 sin 2 2 sin 2 1 2 4 sin max p max MFn 78.711 N m b bx by
2 2

b 85.44 mm

MACHINE DESIGN - An Integrated Approach, 4th Ed.


Friction moment MFf w r sin max p max r cos 2 cos 1 b 2 sin 2 sin 1
2 2

17-17a-2

MFf 11.181 N m Falong MFn MFf ax Falong 750 N

Actuating force

8.

Compare the results.

Method Short shoe Long shoe

Torque Capacity Tshort 11.3 N m Tlong 11.2 N m

Actuation Force Fashort 806 N Falong 750 N

MACHINE DESIGN - An Integrated Approach, 4th Ed.

17-18a-1

PROBLEM 17-18a
Statement: The short-shoe approximation is considered to be valid for brake shoes with an included angle of up to about 45 deg. For the brake shown in Figure P17-4, calculate its torque capacity and required actuating force by both the short-shoe method and the long-shoe method and compare the results for the dimensions given below. Pivot to load Pivot to Y-axis Pivot to X-axis Maximum pressure Friction coeff. Assumptions: The drum rotates CCW. Solution: See Figure P17-4 and Mathcad file P1718a. a x 90mm b x 80 mm b y 30 mm p max 1500 kPa Drum radius Drum width Shoe start angle Shoe end angle r 40 mm w 30 mm

Given:

1 75 deg 2 105 deg

0.3

Short-shoe method Top shoe - self energizing 1. 2. Calculate the total shoe angle.

2 1
Fn p max r w Tshort1 Fn r c r b y Fashort Fn bx c ax

30 deg
Fn 0.942 kN Tshort1 11.3 N m c 10 mm Fashort 806 N

Determine the normal force on the drum from equation 17.8. Normal force

3.

Use equation 17.10 to calculate the torque capacity. Torque capacity

4.

Determine the required actuating force from equation 17.11b and the brake geometry. Distance c Actuation force Bottom shoe - nonself energizing

5.

The actuation force is the same on both shoes and is equal to the actuation force on the top shoe. Use equation 17.11b to solve for the normal force on the bottom shoe. Normal force Fn2 Fashort ax bx c Fn2 874 N

6.

Use equation 17.10 to calculate the torque capacity. Torque capacity Both shoes Tshort2 Fn2 r Tshort Tshort1 Tshort2 Tshort2 10.49 N m Tshort 21.8 N m

7.

The total torque capacity is

Long-shoe method Top shoe - self energizing 8. Determine the maximum shoe angle.

max

return 90 deg if 2 90 deg

2 otherwise max 90 deg


9. Use equation 17.15 to calculate the torque capacity. Torque capacity Tlong1 w r
2

sin max

p max

cos 1 cos 2

MACHINE DESIGN - An Integrated Approach, 4th Ed.


Tlong1 11.2 N m 10. Determine the required actuating force from equations 17.14 and the brake geometry. Dimension b Normal moment MFn w r b sin max p max 1 1 2 1 sin 2 2 sin 2 1 2 4 b bx by
2 2

17-18a-2

b 85.44 mm

MFn 78.711 N m Friction moment MFf w r sin max p max r cos 2 cos 1 b 2 sin 2 sin 1
2 2

MFf 11.181 N m Falong MFn MFf ax Falong 750 N

Actuating force Bottom shoe - nonself-energizing

11. The magnitude of the actuation force is the same on both shoes and is equal to the actuation force on the top shoe. This will result in a lower maximum pressure on the lower, nonself-energizing, shoe. Determine the maximum pressure on the lower shoe by solving equation 17.14c (using the plus sign) for p max2. p max2 a x Falong w r sin max
2 2 2 b sin( ) d sin( ) ( r b cos( ) ) d 1 1

p max2 1127 kPa 12. Use equation 17.15 to calculate the torque capacity. Torque capacity Tlong2 w r
2

sin max

p max2

cos 1 cos 2

Tlong2 8.40 N m 13. The torque capacity of the double brake is the sum of the torque capacity of the two. Total torque capacity 14. Compare the results. Method Short shoe Long shoe Torque Capacity Tshort 21.8 N m Tlong 19.6 N m Actuation Force Fashort 806 N Falong 750 N Tlong Tlong1 Tlong2 Tlong 19.6 N m

MACHINE DESIGN - An Integrated Approach, 4th Ed.

17-19a-1

PROBLEM 17-19a
Statement: The short-shoe approximation is considered to be valid for brake shoes with an included angle of up to about 45 deg. For the brake shown in Figure P17-3, calculate its torque capacity and required actuating force by both the short-shoe method and the long-shoe method and compare the results for the dimensions given below. Pivot to load Pivot to Y-axis Pivot to X-axis Maximum pressure Friction coeff. a x 8.00 in b x 6.00 in b y 4.00 in p max 250 psi Drum radius Drum width Shoe start angle Shoe end angle r 5.00 in w 1.50 in

Given:

1 75 deg 2 105 deg

0.35

Assumptions: The drum rotates CCW and the brake is self-energizing. Solution: See Figure P17-3 and Mathcad file P1719a.

Short-shoe method 1. 2. Calculate the total shoe angle.

2 1

30 deg

Determine the normal force on the drum from equation 17.8. Normal force Fn p max r w Fn 982 lbf

3.

Use equation 17.10 to calculate the torque capacity. Torque capacity Tshort Fn r Tshort 1718 in lbf

4.

Determine the required actuating force from equation 17.11b and the brake geometry. Distance c Actuation force c r b y Fashort Fn bx c ax c 1.000 in Fashort 693 lbf

Long-shoe method 5. Determine the maximum shoe angle.

max

return 90 deg if 2 90 deg

2 otherwise max 90 deg


6. Use equation 17.15 to calculate the torque capacity. Torque capacity Tlong w r
2

sin max

p max

cos 1 cos 2

Tlong 1698 in lbf 7. Determine the required actuating force from equations 17.14 and the brake geometry. Dimension b Normal moment MFn w r b 1 1 2 1 sin 2 2 sin 2 1 2 4 sin max p max MFn 6920 in lbf b bx by
2 2

b 183.162 mm

MACHINE DESIGN - An Integrated Approach, 4th Ed.


Friction moment MFf w r sin max p max r cos 2 cos 1 b 2 sin 2 sin 1
2 2

17-19a-2

MFf 1698 in lbf Falong MFn MFf ax Falong 653 lbf

Actuating force

8.

Compare the results.

Method Short shoe Long shoe

Torque Capacity Tshort 1718 in lbf Tlong 1698 in lbf

Actuation Force Fashort 693 lbf Falong 653 lbf

MACHINE DESIGN - An Integrated Approach, 4th Ed.

17-20a-1

PROBLEM 17-20a
Statement: The short-shoe approximation is considered to be valid for brake shoes with an included angle of up to about 45 deg. For the brake shown in Figure P17-4, calculate its torque capacity and required actuating force by both the short-shoe method and the long-shoe method and compare the results for the dimensions given below. Pivot to load Pivot to Y-axis Pivot to X-axis Maximum pressure Friction coeff. Assumptions: The drum rotates CCW. Solution: See Figure P17-4 and Mathcad file P1720a. a x 8.00 in b x 6.00 in b y 4.00 in p max 250 psi Drum radius Drum width Shoe start angle Shoe end angle r 5.00 in w 2.00 in

Given:

1 75 deg 2 105 deg

0.35

Short-shoe method Top shoe - self energizing 1. 2. Calculate the total shoe angle.

2 1
Fn p max r w Tshort1 Fn r c r b y Fashort Fn bx c ax

30 deg
Fn 1309 lbf Tshort1 2291 in lbf c 1.000 in Fashort 924 lbf

Determine the normal force on the drum from equation 17.8. Normal force

3.

Use equation 17.10 to calculate the torque capacity. Torque capacity

4.

Determine the required actuating force from equation 17.11b and the brake geometry. Distance c Actuation force Bottom shoe - nonself energizing

5.

The actuation force is the same on both shoes and is equal to the actuation force on the top shoe. Use equation 17.11b to solve for the normal force on the bottom shoe. Normal force Fn2 Fashort ax bx c Fn2 1165 lbf

6.

Use equation 17.10 to calculate the torque capacity. Torque capacity Both shoes Tshort2 Fn2 r Tshort Tshort1 Tshort2 Tshort2 2038 in lbf Tshort 4329 in lbf

7.

The total torque capacity is

Long-shoe method Top shoe - self energizing 8. Determine the maximum shoe angle.

max

return 90 deg if 2 90 deg

2 otherwise max 90 deg


9. Use equation 17.15 to calculate the torque capacity.

MACHINE DESIGN - An Integrated Approach, 4th Ed.


Torque capacity Tlong1 w r
2

17-20a-2
sin max p max cos 1 cos 2

Tlong1 2265 in lbf 10. Determine the required actuating force from equations 17.14 and the brake geometry. Dimension b Normal moment MFn w r b sin max p max 1 1 2 1 sin 2 2 sin 2 1 2 4 b bx by
2 2

b 7.211 in

MFn 9227 in lbf Friction moment MFf w r sin max p max r cos 2 cos 1 b 2 sin 2 sin 1
2 2

MFf 2265 in lbf Falong MFn MFf ax Falong 870 lbf

Actuating force Bottom shoe - nonself-energizing

11. The magnitude of the actuation force is the same on both shoes and is equal to the actuation force on the top shoe. This will result in a lower maximum pressure on the lower, nonself-energizing, shoe. Determine the maximum pressure on the lower shoe by solving equation 17.14c (using the plus sign) for p max2. a x Falong w r sin max
2 2 2 b sin( ) d sin( ) ( r b cos( ) ) d 1 1

p max2

p max2 151.5 psi 12. Use equation 17.15 to calculate the torque capacity. Torque capacity Tlong2 w r
2

sin max

p max2

cos 1 cos 2

Tlong2 1372 in lbf 13. The torque capacity of the double brake is the sum of the torque capacity of the two. Total torque capacity 14. Compare the results. Method Short shoe Long shoe Torque Capacity Tshort 4329 in lbf Tlong 3637 in lbf Actuation Force Fashort 924 lbf Falong 870 lbf Tlong Tlong1 Tlong2 Tlong 3637 in lbf

MACHINE DESIGN - An Integrated Approach, 4th Ed.

17-21-1

PROBLEM 17-21
Statement: Given: Figure P17-2 shows a double short-shoe drum brake. Find the reaction forces in the global XY system for the dimensions given below. Pivot to load Pivot to y-axis Pivot to x-axis a 90 mm b 80 mm e 30 mm Drum radius Drum width Shoe angle r 40 mm w 60 mm

25 deg

Maximum pressure p max 1500 kPa

Friction coefficient 0.25

Assumptions: Short-shoe theory is appropriate. The drum rotates CCW. Solution: See Figure P17-2 and Mathcad file P1721.

Top shoe - self energizing 1. Determine the normal force on the drum from equation 17.8. Normal force 2. Calculate the friction force. Fn1 p max r w Ff1 Fn1 Fn1 1571 N Ff1 392.7 N

3. Determine the required actuating force from equation 17.11b and the brake geometry modified for CW rotation. Distance c Actuation force c r e Fa1 Fn1 b c a c 10.000 mm Fa1 1353 N

4. The reaction forces on the top shoe pivot are: Rx1 Ff1 Ry1 Fa1 Fn1 Bottom shoe - nonself energizing 5. The magnitude of the actuation force is the same on both shoes and is equal to the actuation force on the top shoe. Use equation 17.11b to solve for the normal force on the bottom shoe. Actuation force Fa2 Fa1 Fn2 Fa2 Ff2 Fn2 a b c Fa2 1353 N Fn2 1476 N Ff2 368.9 N Rx1 392.7 N Ry1 218.2 N

Normal force

6. Calculate the friction force.

7. The reaction forces on the top shoe pivot are: Rx2 Ff2 Ry2 Fa2 Fn2 Rx2 368.9 N Ry2 123.0 N

MACHINE DESIGN - An Integrated Approach, 4th Ed.

17-22-1

PROBLEM 17-22
Statement: Given: Figure P17-2 shows a double short-shoe drum brake. Find the reaction forces in the global XY system for the dimensions given below. Pivot to load Pivot to y-axis Pivot to x-axis Maximum pressure a 12.00 in b 8.00 in e 3.00 in p max 200 psi Drum radius Drum width Shoe angle Friction coefficient r 6.00 in w 2.00 in 25 deg 0.28

Assumptions: Short-shoe theory is appropriate. The drum rotates CCW. Solution: See Figure P17-2 and Mathcad file P1722.

Top shoe - self energizing 1. Determine the normal force on the drum from equation 17.8. Normal force Fn1 p max r w Calculate the friction force. Ff1 Fn1 Fn1 1047.2 lbf Ff1 293.2 lbf

2.

3. Determine the required actuating force from equation 17.11b and the brake geometry modified for CW rotation. Distance c c r e c 3.000 in b c Actuation force Fa1 Fn1 Fa1 624.828 lbf a 4. The reaction forces on the top shoe pivot are: Rx1 Ff1 Ry1 Fa1 Fn1 Bottom shoe - nonself energizing 5. The magnitude of the actuation force is the same on both shoes and is equal to the actuation force on the top shoe. Use equation 17.11b to solve for the normal force on the bottom shoe. Actuation force Normal force 6. 7. Calculate the friction force. Fa2 Fa1 Fn2 Fa2 a b c Fa2 625 lbf Fn2 848.183 lbf Ff2 237.5 lbf Rx2 237.5 lbf Ry2 223.4 lbf Rx1 293.2 lbf Ry1 422.4 lbf

Ff2 Fn2

The reaction forces on the top shoe pivot are: Rx2 Ff2 Ry2 Fa2 Fn2

MACHINE DESIGN - An Integrated Approach, 4th Ed.

17-23-1

PROBLEM 17-23
Statement: Given: Find the reaction forces at the arm pivot in the global XY system for the brake in Figure P17-3 for the dimensions given below. Pivot to load Pivot to Y-axis Pivot to X-axis Maximum pressure Friction coefficient a x 100mm b x 70 mm b y 20 mm p max 1300 kPa Drum radius Drum width Shoe start angle Shoe end angle r 30 mm w 50 mm

1 25 deg 2 125 deg

0.3

Assumptions: Long-shoe theory is appropriate. The drum rotates CCW and the brake is self-energizing. Solution: 1. See Figure P17-3 and Mathcad file P1723.

Determine the maximum shoe angle.

max

return 90 deg if 2 90 deg

2 otherwise max 90 deg


2. Determine the required actuating force from equations 17.14 and the brake geometry. Dimension b Normal moment MFn w r b 1 1 2 1 sin 2 2 sin 2 1 2 4 sin max p max MFn 184.423 N m Friction moment MFf w r sin max p max r cos 2 cos 1 b 2 sin 2 sin 1
2 2

bx by

b 72.801 mm

MFf 15.487 N m Fa MFn MFf ax Fa 1689 N

Actuating force 3.

Let the angle between the global X axis and the x1 axis be . Rotation angle

atan

by bx

15.945 deg

4.

First, find the reaction forces in the x1-y1 system by summing forces in the x1 and y1 directions

Fx1 Fy1

Fa sin( ) cos( ) dFn sin( ) dFf Rx1 = 0 Fa cos( ) sin( ) dFn cos( ) dFf Ry1 = 0

where Rx1 and Ry1 are both assumed to act in their respective positive directions. 5. Expand the integrals and solve these two equations for Rx1 and Ry1, respectively.

MACHINE DESIGN - An Integrated Approach, 4th Ed.


2 2 sin( ) cos( ) d sin( ) 2 d Rx1 Fa sin( ) w r sin max 1 1

17-23-2

p max

Rx1 744.0 N
2 2 sin( ) 2 d sin( ) cos( ) d Ry1 Fa cos( ) w r sin max 1 1

p max

Ry1 1052.9 N 6. Transform these forces to the global XY system. RX Rx1 cos( ) Ry1 sin( ) RY Rx1 sin( ) Ry1 cos( ) RX 1005 N RY 808 N

MACHINE DESIGN - An Integrated Approach, 4th Ed.

17-24-1

PROBLEM 17-24
Statement: Given: Find the reaction forces at the arm pivot in the global XY system for the brake in Figure P17-3 for the dimensions given below. Pivot to load Pivot to Y-axis Pivot to X-axis Maximum pressure Friction coefficient a x 8.00 in b x 6.00 in b y 4.00 in p max 250 psi Drum radius Drum width Shoe start angle Shoe end angle r 5.00 in w 1.50 in

1 35 deg 2 155 deg

0.35

Assumptions: Long-shoe theory is appropriate. The drum rotates CCW and the brake is self-energizing. Solution: 1. See Figure P17-3 and Mathcad file P1724.

Determine the maximum shoe angle.

max

return 90 deg if 2 90 deg

2 otherwise max 90 deg


2. Determine the required actuating force from equations 17.14 and the brake geometry. Dimension b Normal moment MFn w r b 1 1 2 1 sin 2 2 sin 2 1 2 4 sin max p max MFn 19925 in lbf Friction moment MFf w r sin max p max r cos 2 cos 1 b 2 sin 2 sin 1
2 2

bx by

b 7.211 in

MFf 6017 in lbf Fa MFn MFf ax Fa 1738 lbf

Actuating force 3.

Let the angle between the global X axis and the x1 axis be . Rotation angle

atan

by bx

33.69 deg

4.

First, find the reaction forces in the x1-y1 system by summing forces in the x1 and y1 directions

Fx1 Fy1

Fa sin( ) cos( ) dFn sin( ) dFf Rx1 = 0 Fa cos( ) sin( ) dFn cos( ) dFf Ry1 = 0

where Rx1 and Ry1 are both assumed to act in their respective positive directions. 5. Expand the integrals and solve these two equations for Rx1 and Ry1, respectively.

MACHINE DESIGN - An Integrated Approach, 4th Ed.


2 2 sin( ) cos( ) d sin( ) 2 d Rx1 Fa sin( ) w r sin max 1 1

17-24-2

p max

Rx1 2072.3 lbf


2 2 sin( ) 2 d sin( ) cos( ) d Ry1 Fa cos( ) w r sin max 1 1

p max

Ry1 1267.3 lbf 6. Transform these forces to the global XY system. RX Rx1 cos( ) Ry1 sin( ) RY Rx1 sin( ) Ry1 cos( ) RX 2427 lbf RY 95.1 lbf

MACHINE DESIGN - An Integrated Approach, 4th Ed.

17-25-1

PROBLEM 17-25
Statement: Given: Find the reaction forces at the arm pivot in the global XY system for the brake in Figure P17-4 for the dimensions given below. Pivot to load Pivot to Y-axis Pivot to X-axis Maximum pressure Friction coefficient a x 90mm b x 80 mm b y 30 mm p max 1500 kPa Drum radius Drum width Shoe start angle Shoe end angle r 40 mm w 30 mm

1 30 deg 2 160 deg

0.25

Assumptions: Long-shoe theory is appropriate. The drum rotates CCW. Solution: 1. See Figure P17-4 and Mathcad file P1725.

Determine the maximum shoe angle.

max

return 90 deg if 2 90 deg

2 otherwise max 90 deg


2. Determine the required actuating force from equations 17.14 and the brake geometry. Dimension b Normal moment MFn w r b 1 1 2 1 sin 2 2 sin 2 1 2 4 sin max p max MFn 232.482 N m Friction moment MFf w r sin max p max r cos 2 cos 1 b 2 sin 2 sin 1
2 2

bx by

b 85.44 mm

MFf 35.06 N m Fa MFn MFf ax Fa 2194 N

Actuating force 3.

Let the angle between the global X axis and the x1 axis be . Rotation angle

atan

by bx

20.556 deg

Top shoe - self-energizing 4. First, find the reaction forces in the x1-y1 system by summing forces in the x1 and y1 directions

Fx1 Fy1

Fa sin( ) cos( ) dFn sin( ) dFf Rx1 = 0 Fa cos( ) sin( ) dFn cos( ) dFf Ry1 = 0

where Rx1 and Ry1 are both assumed to act in their respective positive directions.

MACHINE DESIGN - An Integrated Approach, 4th Ed.


5. Expand the integrals and solve these two equations for Rx1 and Ry1, respectively.
2 2 sin( ) cos( ) d sin( ) 2 d Rx1 Fa sin( ) w r sin max 1 1

17-25-2

p max

Rx1 1570.2 N
2 2 sin( ) 2 d sin( ) cos( ) d Ry1 Fa cos( ) w r sin max 1 1

p max

Ry1 637.2 N 6. Transform these forces to the global XY system. RXtop Rx1 cos( ) Ry1 sin( ) RYtop Rx1 sin( ) Ry1 cos( ) Bottom shoe - nonself-energizing 7. The magnitude of the actuation force is the same on both shoes and is equal to the actuation force on the top shoe. This will result in a lower maximum pressure on the lower, nonself-energizing, shoe. Determine the maximum pressure on the lower shoe by solving equation 17.14c (using the plus sign) for p max2. a x Fa w r sin max
2 2 2 b sin( ) d sin( ) ( r b cos( ) ) d 1 1

RXtop 1694 N RYtop 45.3 N

p max2

p max2 1107 kPa 8. First, find the reaction forces in the x2-y2 system by summing forces in the x2 and y2 directions. However, for a nonself-energizing shoe, the signs of the Ff terms are opposite to those for the self-energizing case. Note also that the y2 axis points toward the shoe (opposite from what is shown in Figure P17-4).

Fx2 Fy2

Fa sin( ) cos( ) dFn sin( ) dFf Rx2 = 0 Fa cos( ) sin( ) dFn cos( ) dFf Ry2 = 0

where Rx2 and Ry2 are both assumed to act in their respective positive directions. 9. Expand the integrals and solve these two equations for Rx2 and Ry2, respectively.
2 2 sin( ) cos( ) d sin( ) 2 d Rx2 Fa sin( ) w r sin max 1 1

p max2

MACHINE DESIGN - An Integrated Approach, 4th Ed. Rx2 356.6 N


2 2 sin( ) 2 d sin( ) cos( ) d Ry2 Fa cos( ) w r sin max 1 1

17-25-3

p max2

Ry2 24.0 N 10. Transform these forces to the global XY system. RXbot Rx2 cos( ) Ry2 sin( ) RYbot Rx2 sin( ) Ry2 cos( ) RXbot 325 N RYbot 147.7 N

MACHINE DESIGN - An Integrated Approach, 4th Ed.

17-26-1

PROBLEM 17-26
Statement: Given: Find the reaction forces at the arm pivot in the global XY system for the brake in Figure P17-4 for the dimensions given below. Pivot to load Pivot to Y-axis Pivot to X-axis Maximum pressure Friction coefficient a x 12.00 in b x 8.00 in b y 3.00 in p max 200 psi Drum radius Drum width Shoe start angle Shoe end angle r 6.00 in w 2.00 in

1 25 deg 2 145 deg

0.28

Assumptions: Long-shoe theory is appropriate. The drum rotates CCW. Solution: 1. See Figure P17-4 and Mathcad file P1726.

Determine the maximum shoe angle.

max

return 90 deg if 2 90 deg

2 otherwise max 90 deg


2. Determine the required actuating force from equations 17.14 and the brake geometry. Dimension b Normal moment MFn w r b 1 1 2 1 sin 2 2 sin 2 1 2 4 sin max p max MFn 30218 in lbf Friction moment MFf w r sin max p max r cos 2 cos 1 b 2 sin 2 sin 1
2 2

bx by

b 8.544 in

MFf 6525 in lbf Fa MFn MFf ax Fa 1974 lbf

Actuating force 3.

Let the angle between the global X axis and the x1 axis be . Rotation angle

atan

by bx

20.556 deg

Top shoe - self-energizing 4. First, find the reaction forces in the x1-y1 system by summing forces in the x1 and y1 directions

Fx1 Fy1

Fa sin( ) cos( ) dFn sin( ) dFf Rx1 = 0 Fa cos( ) sin( ) dFn cos( ) dFf Ry1 = 0

where Rx1 and Ry1 are both assumed to act in their respective positive directions.

MACHINE DESIGN - An Integrated Approach, 4th Ed.


5. Expand the integrals and solve these two equations for Rx1 and Ry1, respectively.
2 2 sin( ) cos( ) d sin( ) 2 d Rx1 Fa sin( ) w r sin max 1 1

17-26-2

p max

Rx1 1503.1 lbf


2 2 sin( ) 2 d sin( ) cos( ) d Ry1 Fa cos( ) w r sin max 1 1

p max

Ry1 1738.6 lbf 6. Transform these forces to the global XY system. RXtop Rx1 cos( ) Ry1 sin( ) RYtop Rx1 sin( ) Ry1 cos( ) Bottom shoe - nonself-energizing 7. The magnitude of the actuation force is the same on both shoes and is equal to the actuation force on the top shoe. This will result in a lower maximum pressure on the lower, nonself-energizing, shoe. Determine the maximum pressure on the lower shoe by solving equation 17.14c (using the plus sign) for p max2. a x Fa w r sin max
2 2 2 b sin( ) d sin( ) ( r b cos( ) ) d 1 1

RXtop 2018 lbf RYtop 1100 lbf

p max2

p max2 129.0 psi 8. First, find the reaction forces in the x2-y2 system by summing forces in the x2 and y2 directions. However, for a nonself-energizing shoe, the signs of the Ff terms are opposite to those for the self-energizing case. Note also that the y2 axis points toward the shoe (opposite from what is shown in Figure P17-4).

Fx2 Fy2

Fa sin( ) cos( ) dFn sin( ) dFf Rx2 = 0 Fa cos( ) sin( ) dFn cos( ) dFf Ry2 = 0

where Rx2 and Ry2 are both assumed to act in their respective positive directions. 9. Expand the integrals and solve these two equations for Rx2 and Ry2, respectively.
2 2 sin( ) cos( ) d sin( ) 2 d Rx2 Fa sin( ) w r sin max 1 1

p max2

Rx2 61.7 lbf

MACHINE DESIGN - An Integrated Approach, 4th Ed.

17-26-3

2 2 sin( ) 2 d sin( ) cos( ) d Ry2 Fa cos( ) w r sin max 1 1

p max2

Ry2 399.3 lbf 10. Transform these forces to the global XY system. RXbot Rx2 cos( ) Ry2 sin( ) RYbot Rx2 sin( ) Ry2 cos( ) RXbot 82.5 lbf RYbot 396 lbf

MACHINE DESIGN - An Integrated Approach, 4th Ed.

17-27-1

PROBLEM 17-27
Statement: A clutch is needed for an electric motor that transmits 20 kW at 1100 rpm. The clutch will attach directly to the motor housing face plate and is to have the same housing diameter as the motor, which is 125 mm. The minimum radial clearance between the housing OD and the clutch disk OD is 5 mm. The clutch output shaft will have the same diameter as the motor shaft, which is 15 mm. Design a multiple disk clutch for this application. State all assumptions and design choices. Specify the clutch material, outside disk radius, inside disk radius, and the required actuation force. Shaft diameter Transmitted power Housing dia. Assumptions: Uniform pressure model. Design choices: Use a molded clutch material running against steel with: Maximum pressure Friction coefficient 0.25 Disk outside radius ro 0.5 d h cr ro 57.5 mm Solution: 1. See Mathcad file P1727. d s 15 mm H 20 kW d h 125 mm Rotational speed Disk radial clearance

Given:

1100 rpm
cr 5 mm

p 1000 kPa

Calculate the transmitted torque. T H

T 173.6 N m

2.

Tentatively choose an inside radius for the clutch discs and use equation 17.2c to solve for the required number of friction faces. Let Ns ri 0.5 d s 5 mm 3 T
3 3 2 p ro ri

T=

2 3

p ro ri
3

Ns

Ns 1.762

3.

Increase this to the next higher integer and solve for the required inside radius.
1

Let

Ns 2

ri ro
3

2 p N s

3 T

ri 28.969 mm

Round this to 4. Convert the radii to diameters. Outside diameter Inside diameter 5.

ri 29 mm

od 2 ro id 2 ri

od 115 mm id 58 mm

Calculate the actuation force required using equation 17.3. 3 T ro ri


2 3 2

Actuation force

3 2 Ns ro ri

F 7748 N

MACHINE DESIGN - An Integrated Approach, 4th Ed.

17-28-1

PROBLEM 17-28
Statement: A clutch is needed for an electric motor that transmits 25 hp at 800 rpm. The clutch will attach directly to the motor housing face plate and is to have the same housing diameter as the motor, which is 5.5 in. The minimum radial clearance between the housing OD and the clutch disk OD is 0.25 in. The clutch output shaft will have the same diameter as the motor shaft, which is 0.625 in. Design a multiple disk clutch for this application. State all assumptions and design choices. Specify the clutch material, outside disk radius, inside disk radius, and the required actuation force. Shaft diameter Transmitted power Housing dia. Assumptions: Uniform pressure model. Design choices: Use a molded clutch material running against steel with: Maximum pressure Friction coefficient 0.25 Disk outside radius ro 0.5 d h cr ro 2.50 in Solution: 1. See Mathcad file P1728. d s 0.625 in H 25 hp d h 5.5 in Rotational speed Disk radial clearance

Given:

800 rpm
cr 0.25 in

p 150 psi

Calculate the transmitted torque. T H T 1970 in lbf

2.

Tentatively choose an inside radius for the clutch discs and use equation 17.2c to solve for the required number of friction faces. 2 3 3 Let ri 0.5 d s 0.25 in T = p ro ri Ns 3 Ns 3 T
3 3 2 p ro ri

Ns 1.623

3.

Increase this to the next higher integer and solve for the required inside radius.
1

Let

Ns 2

ri ro
3

2 p N s

3 T

ri 1.456 in

Round this to 4. Convert the radii to diameters. Outside diameter Inside diameter 5.

ri 1.50 in

od 2 ro id 2 ri

od 5.00 in id 3.00 in

Calculate the actuation force required using equation 17.3. 3 T ro ri


2 3 2

Actuation force

2 Ns ro ri

F 1929 lbf

MACHINE DESIGN - An Integrated Approach, 4th Ed.

17-29-1

PROBLEM 17-29
Statement: Find the torque that a dual-pad, caliper disc brake with pad angle of 60 deg can transmit if the outside and inside lining diameters are 160 mm and 90 mm, respectively, and the applied axial force is 3 kN. Assume uniform wear and = 0.35. Is the pressure on the lining acceptable? What lining materials would be suitable? Number of pads Axial force Friction coefficient Assumptions: Uniform wear model. Solution: 1. See Mathcad file P1729. ro 80 mm ri 45 mm T Np F ro ri 2 T 131.25 N m Np 2 F 3 kN 0.35 Clutch dimensions: Outside diameter Inside diameter Pad angle od 160 mm id 90 mm 0.333

Given:

Calculate the outside and inside radii of the disk. Outside radius ro 0.5 od Inside radius ri 0.5 id

2.

Using equation 17.6, calculate the torque capacity.

3.

Calculate the maximum lining pressure using equation 17.5a. Maximum pressure p max

ri ro ri

p max 1821 kPa 4. From Table 17-1, we see that either a molded or sintered metal lining is suitable.

MACHINE DESIGN - An Integrated Approach, 4th Ed.

17-30-1

PROBLEM 17-30
Statement: Find the torque that a 2-pad, caliper disc brake with pad angle of 60 deg can transmit if the outside and inside lining diameters are 160 mm and 90 mm, respectively, and the applied axial force is 3 kN. Assume uniform pressure and = 0.35. Is the pressure on the lining acceptable? What lining materials would be suitable? Number of pads Axial force Friction coefficient Assumptions: Uniform pressure model. Solution: 1. See Mathcad file P1730. ro 80 mm ri 45 mm 2 ro ri T Np F 3 2 2 ro ri
3 3

Given:

Np 2 F 3 kN 0.35

Clutch dimensions: Outside diameter Inside diameter Pad angle od 160 mm id 90 mm 0.333

Calculate the outside and inside radii of the disk. Outside radius ro 0.5 od Inside radius ri 0.5 id

2.

Using equation 17.3, calculate the torque capacity.

T 134.7 N m

3.

Calculate the lining pressure using equation 17.2c. Pressure p 3 T

Np ro ri
3 3

p 1311 kPa

4.

From Table 17-1, we see that either a molded or sintered metal lining is suitable.

MACHINE DESIGN - An Integrated Approach, 4th Ed.

17-31-1

PROBLEM 17-31
Statement: Design a dual-pad caliper disc brake to provide a braking force of 240 N at the periphery of a 750-mm-dia wheel that is rotating at 670 rpm. Use an inside radius to outside radius ratio of 0.577. Assume uniform wear. State all assumptions and design choices. Specify the brake material, outside pad radius, inside pad radius, pad angle, and the required actuation force. Number of pads Peripheral force Disk radius factor Assumptions: Uniform wear model. Design Choices: Friction coefficient Pad angle (90 deg) Solution: 1. 2. See Mathcad file P1731. T 0.5 d w Fw T 90 N m Np 2 Fw 240 N fr 0.577 Wheel speed

Given:

670 rpm

Wheel diameter d w 750 mm

0.30 0.50

Pad material Max. pressure

Sintered metal p max 1500 kPa

Calculate the torque required

Using equation 17.5b and the disk radius factor, solve for the required outside and inside radii. T=

ri p max
2

ro ri
2

Np
1 3

ri = fr ro

ro

2 T 2 pmax fr 1 fr Np

ro 69.16 mm Then the inside radius is Round this to 3.

round this to ri fr ro ri 40 mm

ro 70 mm ri 40.39 mm

Solve equation 17.5a for the actuating force, F. Actuating force F ri p max ro ri F 2.827 kN T Np F

Rated torque

ro ri
2

T 93.3 N m 4. DESIGN SUMMARY Brake material Coefficient of friction Pad inside radius Pad outside radius Pad angle Actuating force Maximum pressure Rated torque Sintered metal 0.30 ri 40 mm ro 70 mm

90 deg
F 2.83 kN p max 1500 kPa T 93.3 N m

MACHINE DESIGN - An Integrated Approach, 4th Ed.

17-32-1

PROBLEM 17-32
Statement: Design a dual-pad caliper disc brake to provide a braking force of 240 N at the periphery of a 750-mm-dia wheel that is rotating at 670 rpm. Use an inside radius to outside radius ratio of 0.577. Assume uniform pressure. State all assumptions and design choices. Specify the brake material, outside pad radius, inside pad radius, pad angle, and the required actuation force. Number of pads Peripheral force Disk radius factor Assumptions: Uniform wear model. Design Choices: Friction coefficient Pad angle (90 deg) Solution: 1. 2. See Mathcad file P1732. T 0.5 d w Fw T 90 N m Np 2 Fw 240 N fr 0.577 Wheel speed

Given:

670 rpm

Wheel diameter d w 750 mm

0.30 0.50

Pad material Sintered metal Uniform pressure p 1500 kPa

Calculate the torque required

Using equation 17.2c and the disk radius factor, solve for the required outside and inside radii. T=

p 3 3 ro ri Np
3
1 3

ri = fr ro

ro

3 T 3 p 1 fr Np

ro 61.832 mm Then the inside radius is Round this to 3.

round this to ri fr ro ri 36 mm

ro 62 mm ri 35.774 mm

Solve equation 17.1b for the actuating force, F. Actuating force F

2 F 3.002 kN

p 2 2 ro ri

Rated torque

2 ro ri T Np F 3 2 2 ro ri T 90.3 N m

4.

DESIGN SUMMARY Brake material Coefficient of friction Pad inside radius Pad outside radius Pad angle Actuating force Uniform pressure Rated torque

Sintered metal 0.30 ri 36 mm ro 62 mm

90 deg
F 3.00 kN p 1500 kPa T 90.3 N m

MACHINE DESIGN - An Integrated Approach, 4th Ed.

17-33-1

PROBLEM 17-33
Statement: An ultra-light solar racecar weighs 500 lb with driver. It has two 20-in-dia bicycle wheels in front that are to have dual-pad caliper disk brakes on each wheel. The brakes must be capable of bringing the car to a stop in a distance of 150 feet from a speed of 45 mph. Neglecting aerodynamic and rolling resistance forces, design dual-pad caliper disc brakes for the car. Use an inside radius to outside radius ratio of 0.577. Assume uniform wear. State all assumptions and design choices. Specify the brake material, outside pad radius, inside pad radius, pad angle, and the required actuation force. Number of pads Stopping distance Disk radius factor Assumptions: Uniform wear model Design Choices: Friction coefficient Pad angle Solution: 1. See Mathcad file P1733. Np 4 Ds 150 ft fr 0.577 Constant deceleration Car weight Initial speed Level surface Pad material Max. pressure Molded p max 225 psi W 500 lbf Vi 45 mph

Given:

Wheel diameter d w 20 in

0.30 60 deg

Calculate the torque required. For uniform deceleration, the stopping distance, initial velocity, and deceleration are related by Vi
2

Ds =

2 d

solving for the deceleration,

Vi

2 Ds W

d 14.52 ft s

The force required between the tires and ground is Thus, the torque required by the brakes is 2.

Fw

d g T 0.5 d w Fw

Fw 225.6 lbf T 188.04 lbf ft

Using equation 17.5b and the disk radius factor, solve for the required outside and inside radii. T=

ri p max
2

ro ri
2

Np
1 3

ri = fr ro

ro

2 T 2 pmax fr 1 fr Np

ro 3.461 in Then the inside radius is Round this to 3.

round this to ri fr ro ri 2.00 in

ro 3.50 in ri 2.019 in

Solve equation 17.5a for the actuating force, F. Actuating force F ri p max ro ri F 706.9 lbf T Np F

Rated torque

ro ri
2

T 194.4 lbf ft

MACHINE DESIGN - An Integrated Approach, 4th Ed.

17-33-2

4.

DESIGN SUMMARY Brake material Coefficient of friction Pad inside radius Pad outside radius Pad angle Actuating force Maximum pressure Rated torque

Molded 0.30 ri 2.00 in ro 3.50 in

60 deg
F 706.9 lbf p max 225 psi T 194 lbf ft

MACHINE DESIGN - An Integrated Approach, 4th Ed.

17-34-1

PROBLEM 17-34
Statement: An ultra-light solar racecar weighs 500 lb with driver. It has two 20-in-dia bicycle wheels in front that are to have dual-pad caliper disk brakes on each wheel. The brakes must be capable of bringing the car to a stop in a distance of 150 feet from a speed of 45 mph. Neglecting aerodynamic and rolling resistance forces, design dual-pad caliper disc brakes for the car. Use an inside radius to outside radius ratio of 0.577. Assume uniform pressure. State all assumptions and design choices. Specify the brake material, outside pad radius, inside pad radius, pad angle, and the required actuation force. Number of pads Stopping distance Disk radius factor Assumptions: Uniform pressure model Design Choices: Friction coefficient Pad angle Solution: 1. See Mathcad file P1734. Np 4 Ds 150 ft fr 0.577 Constant deceleration Car weight Initial speed Level surface Pad material Pressure Molded p 225 psi W 500 lbf Vi 45 mph

Given:

Wheel diameter d w 20 in

0.30 60 deg

Calculate the torque required. For uniform deceleration, the stopping distance, initial velocity, and deceleration are related by Vi
2

Ds =

2 d

solving for the deceleration,

Vi

2 Ds W

d 14.52 ft s

The force required between the tires and ground is Thus, the torque required by the brakes is 2.

Fw

d g T 0.5 d w Fw

Fw 225.6 lbf T 188.04 lbf ft

Using equation 17.2c and the disk radius factor, solve for the required outside and inside radii. T=

p 3 3 ro ri Np
3
1 3

ri = fr ro

ro

3 T 3 p 1 fr Np

ro 3.095 in Then the inside radius is Round this to 3.

round this to ri fr ro ri 1.750 in

ro 3.125 in ri 1.803 in

Solve equation 17.1b for the actuating force, F. Actuating force F

2 F 789.7 lbf

p 2 2 ro ri
3 3

Rated torque

2 ro ri T Np F 3 2 2 ro ri T 197.6 lbf ft

MACHINE DESIGN - An Integrated Approach, 4th Ed.

17-34-2

4.

DESIGN SUMMARY Brake material Coefficient of friction Pad inside radius Pad outside radius Pad angle Actuating force Uniform pressure Rated torque

Molded 0.30 ri 1.750 in ro 3.125 in

60 deg
F 789.7 lbf p 225 psi T 198 lbf ft

You might also like